0% found this document useful (0 votes)
2 views

Spring Practice Merger

The document presents a series of clinical cases involving various patients with joint pain, swelling, and other related symptoms, leading to different diagnoses such as chronic gouty arthritis, osteoarthritis, and glomerulonephritis. It also includes questions regarding the most likely diagnoses and appropriate treatments based on the symptoms and examination findings. The answers to the questions are provided, indicating the correct diagnosis or treatment for each case.

Uploaded by

sinhakrisin
Copyright
© © All Rights Reserved
We take content rights seriously. If you suspect this is your content, claim it here.
Available Formats
Download as PDF, TXT or read online on Scribd
0% found this document useful (0 votes)
2 views

Spring Practice Merger

The document presents a series of clinical cases involving various patients with joint pain, swelling, and other related symptoms, leading to different diagnoses such as chronic gouty arthritis, osteoarthritis, and glomerulonephritis. It also includes questions regarding the most likely diagnoses and appropriate treatments based on the symptoms and examination findings. The answers to the questions are provided, indicating the correct diagnosis or treatment for each case.

Uploaded by

sinhakrisin
Copyright
© © All Rights Reserved
We take content rights seriously. If you suspect this is your content, claim it here.
Available Formats
Download as PDF, TXT or read online on Scribd
You are on page 1/ 695

52 year old male with chronic joint pain.

There is asymmetric soft tissue swelling,


predominantly in the DIP of the right first digit, with calcifications. Further
questioning revealed a history of repeated attacks of acute joint pain. There is bony
erosion at multiple DIP's of the right hand that is periarticular, well corticated and
has a thin overhanging edge. There is no associated osteopenia. Choose the most likely
diagnosis from the below list of options.
A) Ankylosing spondylitis
B) Erythema nodosum
C) Osteoarthritis
D) Chronic gouty arthritis
E) Psoriatic arthritis
ANSWER: D
53-year-old male with foot pain and swelling for many years. There are "punched out"
articular erosions with overhanging cortex and associated soft tissue masses
consistent with tophaceous gout and gouty arthritis. Choose the most likely changes
from the below list of options.
A) Urate crystals on joint aspirate
B) High ESR
C) Erosions on X-ray
D) Positive blood culture
E) Positive rheumatoid factor
ANSWER: A
55 y.o. male with swelling and pain of his fingers. Patient also with similar
complaints regarding his great toe of the left foot. Soft tissue swelling with subtle
soft tissue nodules primarily of the 2nd and 3rd digits. Erosions with overhanging
edges at the distal aspects of the proximal phalanges of the 2nd and 3rd digits and
metatarsophalangeal joints. Choose the most likely diagnosis from the below list of
options.
A) Ankylosing spondylitis
B) Chronic Gouty arthritis
C) Rheumatoid arthritis
D) Osteoarthritis
E) Psoriatic arthritis
ANSWER: B
55 years old, came to a musculoskeletal specialist seeking advice for a 3-year history
of progressively worsening pain in both knees. Her knees were stiff for about 20
minutes when she arose in the morning and for a few minutes after getting up from a
chair during the day. She had difficulty walking > 30 minutes because of pain, and her
symptoms were exacerbated by kneeling, squatting, or descending stairs. Although
sitting, resting, and reclining relieved her pain, she became stiff if she stayed in
one position for too long. Her symptoms were worse on humid or cold days, and she
occasionally felt as if one of her knees would give out. Which of the following
changes will reveal X-ray examination?
A) Erosions
B) Subluxations
C) Osteophytes
D) Osteoporosis
E) Ankylosis
ANSWER: C
55 years old, came to a musculoskeletal specialist seeking advice for a 3-year history
of progressively worsening pain in both knees. Her knees were stiff for about 20
minutes when she arose in the morning and for a few minutes after getting up from a
chair during the day. She had difficulty walking > 30 minutes because of pain, and her
symptoms were exacerbated by kneeling, squatting, or descending stairs. Although
sitting, resting, and reclining relieved her pain, she became stiff if she stayed in
one position for too long. Her symptoms were worse on humid or cold days, and she
occasionally felt as if one of her knees would give out. Choose the most likely
diagnosis from the below list of options.
A) Ankylosing spondylitis
B) Erythema nodosum
C) Osteoarthritis
D) Gout
E) Psoriatic arthritis
ANSWER: C
57 yo male with swelling and pain of his fingers. Patient also with similar complaints
regarding his great toe of the left foot. Soft tissue swelling with subtle soft tissue
nodules primarily of the 2nd and 3rd digits. Erosions with overhanging edges at the
distal aspects of the proximal phalanges of the 2nd and 3rd digits and
metatarsophalangeal joints. Which of the following medications is indicated for the
treatment of chronic arthritis?
A) Сolchicine
B) Сorticosteroids
C) Indomethacin
D) Allopurinol
E) Ibuprofen
ANSWER: D
59 yo male with swelling and pain of his fingers. Patient also with similar complaints
regarding his great toe of the right foot. Soft tissue swelling with subtle soft
tissue nodules primarily of the 2nd and 3rd digits. Erosions with overhanging edges at
the distal aspects of the proximal phalanges of the 2nd and 3rd digits and
metatarsophalangeal joints. Which of the following medications is indicated for the
treatment of exacerbation of arthritis?
A) Allopurinol
B) Corticosteroids
C) Colchicine
D) Indomethacin
E) Ibuprofen
ANSWER: C
65-year-old, women. She has been under treatment for hypertension and is finally
stable on medication. Initial Chief complaint: painful left knee, arthritis in left
hip. She had had arthritic pain in her right hip confirmed by x-ray about 10 years ago.
The knee pain is a newer problem and she suspects it is also arthritis. There is
slight crepitus with motion of the right knee. Which of the following changes will
reveal X-ray examination?
A) Erosions
B) Subluxations
C) Osteophytes
D) Osteoporosis
E) Ankylosis
ANSWER: C
70-year-old, married, teacher. She has been under treatment for hypertension and is
finally stable on medication. Initial Chief complaint: painful right knee, arthritis
in right hip. She had had arthritic pain in her right hip confirmed by x-ray about 10
years ago. The knee pain is a newer problem and she suspects it is also arthritis.
There is slight crepitus with motion of the right knee. Which of the following
treatment do use?
A) Methotrexat
B) Colchicines
C) Chondroitin sulfate
D) Tetracycline
E) Allopurinol
ANSWER: C
79 y/o male with foot pain and swelling for many years. There are "punched out"
articular erosions with overhanging cortex and associated soft tissue masses. Serum
uric acid was markedly elevated. Choose the most likely diagnosis from the below list
of options.
A) Ankylosing spondylitis
B) Rheumatoid arthritis
C) Osteoarthritis
D) Chronic Gouty arthritis
E) Psoriatic arthritis
ANSWER: D
A 14-year-old boy presents with a faint leg rash, bloody diarrhoea and oliguria.
Further investigations reveal heavy proteinuria and an elevated serum IgA choose the
single most likely diagnosis from the below list of options.
A) Membranous glomerulonephritis
B) Idiopathic crescenteric GN
C) Henoch-Schonlein purpura
D) Goodpasture's syndrome
E) Postinfectious glomerulonephritis
ANSWER: C
A 18 year old male, who has had a recent cold, presents with abdominal pain, joint
pain and a purpuric rash of his buttocks and legs. He is found to have haematuria and
proteinuria. Bloods reveal an eosinophilia, raised IgA and raised ESR. What is the
most likely diagnosis?
A) Henoch schonlein purpura
B) Glomerulonephritis
C) Haemolytic uraemic syndrome
D) Idiopathic thrombocytopenic purpura
E) Horseshoe kidney
ANSWER: A
A 19-year-old male is referred for evaluation of a cardiac murmur before clearance to
play collegiate sports. He has no symptoms, has never had syncope, and has no known
family history of cardiac disease. A 3/6 stystolic murmur is heard at the apex with no
radiation. On squatting, the murmur decreases in intensity, and on sustained handgrip,
the murmur also is decreased. The murmur is probably to which of the following?
A) Hypertrophic cardiomyopathy
B) Mitral valve prolapsed
C) Dilated cardiomyopathy
D) Pulmonary stenosis
E) Aortic regurgitation
ANSWER: B
A 19-year-old woman has painless hematuria. Five days ago, she developed a sore throat,
fever (temperature, 37.8 °C [100 °F]), and dry cough. Four days ago, she noted
painless gross hematuria on two occasions. Six months ago, at the time of a college
physical examination, she was told of microscopic hematuria and advised to have this
rechecked. She does not use recreational drugs. She has taken acetaminophen for the
past several days. At presentation, the patients sore throat is improved and she no
longer has fever. On examination, she is alert and in no distress. Blood pressure is
135/85 mm Hg, pulse rate is 78/min, and temperature is 37 °C (98.6 °F). Skin
examination is normal, and the fundi are normal. No exudate is present in the pharynx.
Neck examination shows no significant lymphadenopathy. The lungs are clear. The heart
is in regular sinus rhythm, with no murmur or rub. Abdominal examination is normal.
The joints are normal. Laboratory studies: Leukocyte count -9400/?L. Hemoglobin -135
g/L. Hematocrit -39%. Blood urea nitrogen -12 mg/dL. Serum creatinine -1.0 mg/dL.
Serum antinuclear antibodies Negative. Serum C3 90 mg/dL. Serum C4 -20 mg/dL. Urine
protein :creatinine ratio -0.5. Urinalysis pH 5.5; specific gravity - 1.012; trace
proteinuria, 3+ hematuria. Urine microscopy shows red blood cells, occasional red
blood cell casts, and 1 to 3 leukocytes/hpf. Which test is the best option to assist
in diagnosis?
A) Renal computed tomography
B) Renal ultrasonography
C) Cystoscopy
D) Magnetic resonance angiography of renal arteries
E) Daily proteinuria
ANSWER: E
A 20- year-old patient with a history of preceding streptococcal infection complains
of malaise, headache, anorexia, subfebrile fever. On exam, mild generalized edema, BP
of 150/90 mm Hg, HR of 100/min, RR of 20/min, the urinalysis showed increased protein,
red cell casts and hyaline casts. What is your diagnosis
A) Acute poststreptococcal glomerulonephritis
B) Acute pyelonephritis
C) Rheumatic fever
D) Essential hypertension
E) Bacterial endocarditis
ANSWER: A
A 20- year-old patient with a history of preceding streptococcal infection complains
of malaise, headache, anorexia, subfebrile fever. On exam, mild generalized edema, BP
of 150/90 mm Hg, HR of 100/min, RR of 20/min, the urinalysis showed increased protein,
red cell casts and hyaline casts. What investigations can help you to confirm
diagnosis of glomerulonephritis?
A) Biochemical blood test, daily proteinuria, renal ultrasonography
B) Biochemical blood test
C) Nechiporenko test, daily proteinuria, ECG
D) CT scanning, renal ultrasonography, abdominal X-ray
E) Renal ultrasonography, ECG, repeat urinalysis
ANSWER: A
A 20- year-old patient with a history of preceding streptococcal infection complains
of malaise, headache, anorexia, subfebrile fever. On exam, mild generalized edema, BP
of 150/90 mm Hg, HR of 100/min, RR of 20/min, the urinalysis showed increased protein,
red cell casts and hyaline casts. What investigations help you to confirm diagnosis of
acute glomerulonephritis?
A) Daily proteinuria, biochemical blood test, renal ultrasound
B) ECG, biochemical blood test, renal ultrasound
C) Daily proteinuria, ECG, renal ultrasound
D) Daily proteinuria, chest X-ray, renal ultrasound
E) Daily proteinuria, biochemical blood test, chest X-ray
ANSWER: A
A 20- year-old patient with a history of preceding streptococcal infection complains
of malaise, headache, anorexia, subfebrile fever. On exam, generalized edema, BP of
170/90 mm Hg, HR of 100/min, RR of 20/min, the urinalysis showed increased protein,
red cell casts and hyaline casts. Rapidly progressive glomerulonephritis was diagnosed.
On the third day of treatment pulmonary bleeding has appeared. What is the most
credible diagnosis:
A) Systemic lupus erythematosus
B) Gudpascher`s syndrome
C) Pulmonary edema
D) Pulmonary infarction due to tromboembolism
E) Pneumonia
ANSWER: B
A 20-year-old female is seen in the ED with symptoms of periodic headaches, sweating,
and nausea with vomiting. She also complains of feeling light-headed with standing.
Her BP on presentation is 230/135, with a HR of 92. On standing, the patient has BP of
205/100, with a HR of 136. On ophthalmologic examination the patient has mild blurring
of the optic discs without hemorrhage. The examination is otherwise normal. What is
the best medication for the management of this patient’s hypertension?
A) Phentolamine
B) Fenoldopam
C) Esmolol
D) Nicardipine
E) Diazoxide
ANSWER: A
A 23 year-old man presented with low back pain, pain in the right knee and sore eyes.
His past history included an episode of diarrhea 3 weeks earlier and he had a positive
history of back pain. Pelvic radiography shows no changes in joints. Choose the most
likely diagnosis from the below list of options.
A) Reiter's syndrome
B) Ankylosing spondylitis
C) Psoriatic arthritis
D) Joint sepsis
E) Polyarteritis nodosa
ANSWER: A
A 23-year old male seeks medical attention for the recent onset of headaches. The
headaches are described as “pounding” and occur during the day and night. He has
minimal relief with acetaminophen. Physical examination is notable for a BP of 185/115
mmHg in the right arm, a HR of 70/min, arteriovenous nicking on funduscopic
examination, normal jugular veins and carotid arteries, reduced pulses in both lower
extremities. Review of symptoms is positive only for leg fatigue with exertion. The
most likely diagnosis is:
A) Aortic coarctation
B) Conn syndrome
C) Liddle syndrome
D) Cushing syndrome
E) Essential hypertension
ANSWER: A
A 23-year old male seeks medical attention for the recent onset of headaches. The
headaches are described as “pounding” and occur during the day and night. He has
minimal relief with acetaminophen. Physical examination is notable for a BP of 185/115
mmHg in the right arm, a HR of 70/min, arteriovenous nicking on funduscopic
examination, normal jugular veins and carotid arteries, reduced pulses in both lower
extremities. Review of symptoms is positive only for leg fatigue with exertion. The
patient described in above is most likely to have which of the following associated
cardiac abnormalities?
A) Bicuspid aortic valve
B) Mitral stenosis
C) Preexcitation syndrome
D) RBBB
E) Tricuspid atresia
ANSWER: A
A 23-year-old woman has come to the clinic for a pre-employment examination. She
recently moved to the area from out of state and got a job at a local small business.
Her past medical history is significant only for diabetes mellitus type I, which she
has had since age 13. Her only medication is insulin, which is infused via an insulin
pump. She denies smoking or using illicit drugs. She admits to social alcohol
consumption less than once a week and says she runs 2 miles daily. She is not sexually
active. You’ve performed a full physical examination. Her temperature is 37.1 (98.8 F),
blood pressure is 136/89 mm Hg, pulse is 54/min, and respirations are 18/min. Her skin
is warm and dry. Cardiovascular examination reveals a normal S1, S2 with no murmurs
appreciated. Respirations are equal bilaterally without any abnormal breath sounds.
Extremities show no clubbing, cyanosis, or edema. Strength is equal bilaterally and
sensation is full throughout. The patient exhibits normal reflexes. She returns to the
clinic several more times and her blood pressure remains elevated. What findings in
the urinalysis confirm diagnosis of diabetic nepropathy?
A) WBC casts
B) RBC casts
C) microalbuminuria
D) Increasing specific gravity
E) Epithelial cell casts
ANSWER: C
A 23-year-old woman was diagnosed with systemic lupus erythematosus (SLE) at age 13
years when she presented with polyarticular joint swelling. On a regimen of
methotrexate and hydroxychloroquine, her joint pain and swelling have been relatively
well controlled. The patient’s fingers exhibit both boutonniere and swan-neck
deformities, but she is able to make a normal closed fist. What will radiographs of
the patient’s hands likely reveal?
A) Bone fragmentation
B) Carpal fusion
C) Erosions in the proximal joints
D) Marked subchondral cysts
E) Normal joints
ANSWER: E
A 24- year-old patient with a history of preceding streptococcal infection complains
of malaise, headache, anorexia, subfebrile fever. On exam, moderate generalized edema,
BP of 150/90 mm Hg, HR of 100/min, RR of 20/min, the urinalysis proteinuria - 6,0 g/l,
red cell casts and hyaline casts. Nephrotic syndrome was diagnosed. What is the main
link in pathogenesis of this syndrome:
A) Damage of tubules by endogenous toxins
B) Inflammation of renal interstitium, which is caused by infectious agents
C) Damage of glomerulus by immune complexes and antibodies to basal membrane
D) All answers are true
E) None of resulted
ANSWER: C
A 24-year-old graduate student has a several-week history of aching pains in knees,
ankles, and elbows and intermittent abdominal pain. Three days ago, he had two
episodes of painless hematuria. He has no fever, chills, or weight loss. There is no
history of hair loss, but he has had an intermittent, nonpruritic rash located below
the knees bilaterally. There has been no recent upper respiratory infection. He takes
acetaminophen occasionally for the aching joints. He has no history of kidney disease
or intravenous drug use. On examination, the patient appears well. Weight is 84.4 kg
(186 Ib). Blood pressure is 120/84 mm Hg, pulse rate is 70/min, and temperature is
37.2 °C (99 °F). No jugulovenous distention is present. The lungs are clear, and
cardiac examination shows regular sinus rhythm with no murmur. On abdominal
examination, the liver and spleen are not palpable, and no mass or tenderness is
present. Pulses in the extremities are normal and there is no edema, but the patient
has a petechial macular papular rash over the lower extremities. Laboratory studies:
Leukocyte count 8,4x109/L. Hemoglobin - 141 g/L. Hematocrit - 41 %. Blood urea
nitrogen - 12 mg/dL. Serum creatinine 1.1 mg/dL. Serum sodium - 138 mmol/L. Serum
chloride - 104 mmol/L. Serum potassium - 4.0 mmol/L. Serum bicarbonate - 26 mmol/L.
Serum antinuclear antibody Negative. Serum C3 - 85 mg/dL. Serum C4 - 21 mg/dL. Urine
protein: creatinine ratio 0.2. Urinalysis Specific gravity 1.030, pH 5.0, 1 +
proteinuria, 4+ hematuria. Urine microscopy reveals many dysmorphic erythrocytes and
erythrocyte casts. The lung fields are clear on chest radiography. Biopsy of skin
lesion shows IgA deposition and leukocytoclastic change. What is the most appropriate
treatment of this patient?
A) Antibiotics
B) Antihypertensive medications
C) Corticosteroids
D) Diuretics
E) Antipyretic medications
ANSWER: C
A 24-year-old previously healthy woman presents to the emergency department because of
sudden shortness of breath and right-sided pleuritic chest pain. She has been using
oral contraceptives for the last several years. The physical examination demonstrates
a normotensive, tachycardic, and tachypneic woman. Which of the following would be the
most useful in EXCLUDING the diagnosis of pulmonary embolism (PE)?
A) Normal PaO2 by arterial blood gas determination
B) Absence of right heart strain on ECG
C) Intermediate probability V/Q (ventilation/perfusion) lung scan
D) Normal plasma level of D-dimer by ELISA (enzyme-linked immunosorbent assay)
E) Normal cardiopulmonary examination
ANSWER: D
A 24-year-old previously healthy woman presents to the emergency department because of
sudden shortness of breath and right-sided pleuritic chest pain. She has been using
oral contraceptives for the last several years. The physical examination demonstrates
a normotensive, tachycardic, and tachypneic woman. Which of the following would be the
most useful in EXCLUDING the diagnosis of pulmonary embolism (PE)?
A) Normal PaO2 by arterial blood gas determination
B) Absence of right heart strain on ECG
C) Intermediate probability V/Q (ventilation/perfusion) lung scan
D) Normal plasma level of D-dimer by ELISA (enzyme-linked immunosorbent assay)
E) Normal cardiopulmonary examination
ANSWER: D
A 25 -year-old woman complained of edema on the face and legs, elevation in blood
pressure up to 160/100 mm Hg, and weakness. She fell ill 3 weeks after sore throat. On
urinalysis, protein of 0.5 g/L, erythrocytes of 17 – 20/field, leukocytes of 2 –
3/field, erythrocyte casts. What treatment should be initiated after establishing of
the exact diagnosis?
A) Heparin
B) Penicillin OS
C) Tetracycline
D) Dipyridamole
E) Ciprofloxacine
ANSWER: B
A 25 year old woman who is 20 weeks pregnant is diagnosed with pyelonephritis. She had
suffered recurrent urinary infections since childhood and her family history reveals
that her mother had a history of hypertension and had been told she had a kidney
problem. Examination was normal and Urea and Creatinine were both normal. What is the
most likely diagnosis?
A) autosomal dominant polycystic kidney disease
B) bladder outlet obstruction
C) normal physiological urinary stasis of pregnancy
D) reflux nephropathy
E) renal stone disease
ANSWER: D
A 25 year-old man presents because of a painful, swollen knee and ankle of 2 weeks'
duration. He has never had joint disease prior to this time. The patient also
complains of low back pain and a recent history UTI. On examination he has vesicles
(some of which have crusted over) on the palms, soles, and glans penis; injected
conjunctivae; a swollen right index finger; and arthritis of the right knee and left
ankle. Choose the most likely diagnosis from the below list of options.
A) Reiter's syndrome
B) Psoriatic arthropathy
C) Osteoarthritis
D) Haemochromatosis
E) Gonococcal arthritis
ANSWER: A
A 25-year- woman develops acute abdominal pain and bloody diarrhea. Several weeks
later, she complains of diffuse musculoskeletal pain involving the neck, left wrist,
left knee, and low back. On examination she has marked swelling, warmth, and loss of
motion of the left wrist and left knee, as well as pain and restricted motion of the
neck and lower lumbar spine. Choose the most likely diagnosis from the below list of
options.
A) Reactive arthritis
B) Rheumatoid arthritis
C) Haemochromatosis
D) Osteoarthritis
E) Gonococcal arthritis
ANSWER: A
A 25-year-old woman with end-stage chronic renal failure complained of fatigue, hair
loss, and brittle nails. On exam, pallor of skin, pulse rate of 94/min. On blood cell
count, Hb of 90 g/L, RBC 3,0x1012/L, color index of 0.7, ESR of 20 mm/h. Serum iron
level was 8.7 mcmol/L. What is the main in pathogenesis of anemia in this patient?
A) Reduce production of erytropoetin
B) Iron intake deficiency
C) High level of creatinin in blood
D) Рeavy menstruation
E) Hypoproteinemia
ANSWER: A
A 26-year-old woman was brought to the office because of a "red rash" that she noticed
today. She says that 3 days ago he had a cough, runny nose, and fever that responded
to ibuprofen. In the office, his temperature is 37 C (98.6 F) and he has a normal
physical examination with the exception of an erythematous, blanching macular rash on
his legs. You’ve diagnosed her with a viral exanthem and advise to drink liquids and
to use ibuprofen as needed for fever. One week later, she came back to the office and
reports that the rash has "changed", she has developed colicky abdominal pain several
times per day, and she is complaining of left knee pain. In the office, her
temperature is 37.2 C (99 F), blood pressure is 100/65 mm Hg, pulse is 100/min, and
respiratory rate is 20/min. A physical examination reveals a well-appearing patient
with palpable purpura of both lower extremities, normal neck examination, clear lungs,
and a soft, non-tender abdomen. Her left knee is painful on flexion, but it is not
erythematous or warm, and there does not seem to be an effusion. His gait is normal.
The most appropriate study at this time is
A) arthrocentesis
B) colonoscopy
C) cultures of blood, urine, and cerebrospinal fluid
D) echocardiography
E) urinalysis
ANSWER: E
A 27-year-old female has hypertension that is difficult to control. She was diagnosed
at age 25. Since that time she has been on increasing amounts of medication. Her
current regimen consists of labetalol 1000 mg bid, lisinopril 40 mg qd, clonidine 0.1
mg bid, and amlodipine 5 mg qd. On physical examination she appears to be without
distress. Her BP is 170/100, and HR is 84 bpm. Cardiac exam is unremarkable, without
rubs, gallops, or murmurs. She has good peripheral pulses and has no edema. Laboratory
studies reveal a potassium of 2.8 mEq/dL and a serum bicarbonate of 32 mEq/dL. Fasting
blood glucose 114 mg/dL. What is the likely diagnosis?
A) Congenital adrenal hyperplasia
B) Fibromuscular dysplasia
C) Cushing’s syndrome
D) Conn’s syndrome
E) Pheochromocytoma
ANSWER: D
A 27-year-old man presents because of a painful, swollen knee and ankle of 2 weeks'
duration. He has never had joint disease prior to this time. The patient also
complains of low back pain and a recent history of clear penile discharge. On
examination he has vesicles (some of which have crusted over) on the palms, soles, and
glans penis; injected conjunctivae; a swollen right index finger; and arthritis of the
right knee and left ankle. Choose the most likely treatment from the below list of
options.
A) Cyclophosphanum
B) Penicillin
C) Tetracycline
D) Prednisone
E) Ceftriaxone
ANSWER: C
A 28-y.o. previously healthy woman presents to the emergency department because of
sudden shortness of breath and right-sided pleuritic chest pain. She has been using
oral contraceptives for the last several years. The physical examination demonstrates
a normotensive, tachycardic, and tachypneic woman. Which of the following would be the
most useful in EXCLUDING the diagnosis of pulmonary embolism (PE)?
A) Normal PaO2 by arterial blood gas determination
B) Absence of right heart strain on ECG
C) Intermediate probability V/Q (ventilation/perfusion) lung scan
D) Normal plasma level of D-dimer by ELISA (enzyme-linked immunosorbent assay)
E) Normal cardiopulmonary examination
ANSWER: D
A 28-year old woman has experienced episodes of myalgias, erythema on the cheeks,
pleural effusion, pericarditis and arthralgias without joint deformity over course of
several years. Which of the following is the probable diagnosis:
A) Dermatomyositis
B) Scleroderma
C) Rheumatoid arthritis
D) Polyarteritis nodosa
E) SLE
ANSWER: E
A 28-year-old woman comes to the clinic complaining of loss of control of urination
and "dribbling" of urine. She has had recurrent urinary tract infections over the past
11 months, which were treated with antibiotics. She has been following safe sex
practices and denies any history of sexually transmitted diseases, but complains of
moderate pain during intercourse. Physical examination and pelvic examination are
normal. The most appropriate treatment in this patient is
A) antibiotics
B) spasmolitics
C) antipiretics
D) diuretics
E) none of them
ANSWER: A
A 28-year-old woman presents with a 12-month history of morning stiffness, pain, and
swelling in her hands and wrists. She has noticed general fatigue, and the hand
stiffness lasts about an hour. Joints of all her extremities are involved
symmetrically. The patient does not smoke or drink. On physical examination, her
temperature is 37.8 °C. Examination of the joints reveals thickening and tenderness of
the metacarpophalangeal joints and proximal interphalangeal joints bilaterally. Both
wrists lack complete flexion and extension and are warm to the touch. Choose the most
likely diagnostic antibody for the disease from the below list of options.
A) Antimicrosomal antibody
B) Antireticulin antibody
C) Anti-dsDNA
D) P-ANCA antibody
E) Rheumatoid factor
ANSWER: E
A 28-year-old woman presents with a 18-month history of morning stiffness, pain, and
swelling in her hands and wrists. She has noticed general fatigue, and the hand
stiffness lasts about an hour. Joints of all her extremities are involved
symmetrically. The patient does not smoke or drink. On physical examination, her
temperature is 38 °C (100.4 °F). Examination of the joints reveals thickening and
tenderness of the metacarpophalangeal joints and proximal interphalangeal joints
bilaterally. Both wrists lack complete flexion and extension and are warm to the
touch. . Which of the following conditions is LEAST likely to occur in late
extraarticular seropositive rheumatoid arthritis?
A) Neutropenia
B) Dry eyes
C) Leg ulcers
D) Sensorimotor polyneuropathy
E) Hepatitis
ANSWER: E
A 28-yr-old man presented with fever, myalgia and abdominal pain and progressive
numbness in his feet. On examination his temperature was 38.8°C, BP was 190/110 and
pulse was 120. His abdomen was tender with guarding and absent bowel sounds. Choose
the single most likely treatment from the list of options given below.
A) Tetracycline, prednisone
B) Ciprofloxacin, NSAIDs
C) Beta 2 agonist, heparin
D) Cyclophosphamide, prednisone
E) IV aminophylline
ANSWER: D
A 28-yr-old man presented with fever, myalgia and abdominal pain. On examination his
temperature was 38.8°C, BP was 190/110 and pulse was 120. His abdomen was tender with
guarding and absent bowel sounds. Choose the most likely diagnosis from the below list
of options.
A) Kawaski's disease
B) Giant cell arteritis
C) Microscopic polyangitis
D) Antiphospholipid syndrome
E) Polyarteritis nodosa
ANSWER: E
A 29-year-old woman is critically ill. The illness was manifested by oliguria, edema,
severe headache. Pasternatsky’s sigh is negative in both sides. On lab exam, WBC of
10.000/mcL; on urinalysis protein of 1,6g/L, erythrocyturia. What investigation can
help in confirmation diagnosis of pyelonephritis?
A) Renal ultrasonography
B) Repeat urinalysis
C) Biochemical blood test
D) CT scanning
E) None of them
ANSWER: A
A 29-year-old woman is critically ill. The illness was manifested by oliguria, edema,
severe headache. Pasternatsky’s sigh is negative in both sides. On lab exam, WBC of
10.000/mcL; on urinalysis protein of 0,9 g/L, erythrocyturia. What syndrome is leading?
A) Nephrotic syndrome
B) Nephritic syndrome
C) Urinary syndrome
D) Uremic syndrome
E) Edematous syndrome
ANSWER: C
A 29-year-old woman is critically ill. The illness was manifested by high fever,
chills, sweating, aching pain in lumbar area, a discomfort in urination, and frequent
urination. Pasternatsky’s sigh is positive in both sides. On lab exam, WBC of
20.000/mcL; on urinalysis protein of 0.6g/L, leukocyturia, bacteriuria. Your
preliminary diagnosis.
A) Acute pyelonephritis
B) Exacerbation pf chronic pyelonephritis
C) Acute glomerulonephritis
D) Acute cystitis
E) Nephrolithiasis
ANSWER: A
A 29-year-old woman is critically ill. The illness was manifested by high fever,
chills, sweating, aching pain in lumbar area, a discomfort in urination, and frequent
voiding. Pasternatsky’s sigh is positive in both sides. On lab exam, WBC of 20.000/mcL;
on urinalysis protein of 0.6g/L, leukocyturia, bacteriuria. What syndromes are leading
in this patient?.
A) Urinary syndrome, pain syndrome, intoxication syndrome
B) Nephritic syndrome, pain syndrome, intoxication syndrome
C) Urinary syndrome, hypertensive syndrome, intoxication syndrome
D) Urinary syndrome, disuric syndrome, intoxication syndrome
E) Nephrotic syndrome, pain syndrome, intoxication syndrome
ANSWER: A
A 29-year-old woman is critically ill. The illness was manifested by high fever,
chills, sweating, aching pain in lumbar area, a discomfort in urination, and frequent
voiding. Pasternatsky’s sigh is positive in both sides. On lab exam, WBC of 20.000/mcL;
on urinalysis protein of 0.6g/L, leukocyturia, bacteriuria. Prescribe antibacterial
treatment to this patient.
A) Ceftriaxone 1.0 i/m 2 times a day
B) Claritromycin 500 mg 2 times a day orally
C) Penicillin B 1000000 i/m once a day
D) Meropenem 1 g 3 times a day orally
E) Patient doesn’t need antibacterial treatment
ANSWER: A
A 29-year-old woman, who has been undergoing treatment for hypertension for the past 2
years, comes to the office because of chills and right-sided flank pain. She has had
chronic "low back pain" that you have been treating unsuccessfully with nonsteroidal
anti-inflammatory drugs. She has never had any diagnostic studies performed to
evaluate her hypertension or back pain. She is estranged from her family, but she
knows that her mother and brother have been treated for hypertension starting at age
25. She has not spoken to them in 10 years and so she is unaware of any other medical
conditions. Her temperature is 38.1 C (100.6 F), blood pressure is 130/90 mm Hg, and
pulse is 65/min. On physical examination today there is marked right-sided flank
tenderness. Cardiac examination reveals a mid-systolic click. Urinalysis shows pyuria
and white blood cell casts. You prescribe a 14-day course of trimethoprim-
sulfamethoxazole, schedule a renal ultrasound and a follow-up visit. You tell her to
call your office immediately if the symptoms worsen. On the return visit, she says
that she feels much better, but still has dull flank pain. Her temperature is 37.0 C
(98.6 F). The ultrasound report is in the chart and states that there are 7 cysts in
her right kidney and 5 cysts in her left kidney. What investigation would you
prescribe next?:
A) Daily proteinuria, urine culture
B) Urine culture, ECG
C) ECG, CBC
D) Evaluating of creatinine, urea, electrolytes, daily proteinuria
E) CBC, urine culture
ANSWER: D
A 29-yr-old homosexual male presented to A&E with a markedly swollen and tender left
knee. He says it's been present for about a month but has only recently become painful.
He admitted losing weight and on examination a temperature of 38В°C was found. He had
no other complaints apart from a 2-month history of cough, which he attributed to his
heavy smoking. Choose the most likely diagnosis from the below list of options.
A) Rheumatoid arthritis
B) Osteoarthritis
C) Reiter's syndrome
D) Infective arthritis
E) Felty's syndrome
ANSWER: C
A 30-year-old woman presents to your office complaining of flank pain and fever for
the past 4 days. She has a history of multiple urinary tract infections. She has
suffered 2 in the previous 18 months that were treated successfully with trimethoprim-
sulfamethoxazole. She is sexually active and her last menstrual period was 6 days ago.
She takes no routine medications. Her temperature is 39.0 C (103.1 F), blood pressure
is 130/65 mm Hg, and pulse is 110/min. She has prominent right costovertebral angle
(CVA) tenderness. What is the probable diagnosis?
A) Acute pielonephritis
B) Chronic pielonephritis
C) Amyloidosis of kidney
D) Acute tubulointerstitial nephritis
E) Chronic tubulointerstitial nephritis
ANSWER: B
A 30-yr-old computer engineer with a long history of asthma and rhinitis presents with
wheezing cough and fever. A CXR shows patchy consolidation. Physical examination shows
multiple tender subcutaneous nodules and purpura. Choose the most likely diagnosis
from the below list of options.
A) Polyarteritis nodosa
B) Mitral stenosis
C) Pneumonia
D) SLE
E) Churg-Strauss syndrome
ANSWER: A
A 30-yr-old woman has pain and stiffness in her hands, wrists, elbows, knees and
ankles. There is swelling of her MCP joints bilaterally. On physical examination, her
temperature is 38 °C (100.4 °F). Examination of the joints reveals thickening and
tenderness of the metacarpophalangeal joints and proximal interphalangeal joints
bilaterally. Both wrists lack complete flexion and extension and are warm to the touch.
Choose the most likely diagnostic antibody for Rheumatoid arthritis from the below
list of options.
A) Antimicrosomal antibody
B) Antireticulin antibody
C) Anti-dsDNA
D) P-ANCA antibody
E) Rheumatoid factor
ANSWER: E
A 31-year-old man with a history of preceding tonsillitis complained of headache in
occipital area, edema. On physical examination, the skin was pale; there was face and
extremitas edema, blood pressure of 140/90 mm Hg. On EchoCG, there was concentric
hypertrophy of the left ventricle. Ultrasound examination of the kidneys revealed
thinned cortical layer. Urinalysis showed proteinuria of 3.5 g/day, hematurea. The
level of protein in blood was 58 g/l. What changes in lab tests can confirm diagnosis
of nephritic syndrome?
A) Hypoalbumiemia, hypocalcemia, dislipidemia, hypercoagulability
B) Hypoalbumiemia, hypercholesterolemia, hyperkalemia, hypercoagulability
C) Hyperglycemia, hypercholesterolemia, dislipidemia, hypercoagulability
D) Hypoalbumiemia, hypercholesterolemia, dislipidemia, hypercoagulability
E) Hypoalbumiemia, edema, dislipidemia, hypercoagulability
ANSWER: D
A 31-year-old white woman with diabetes presents for management of her hypertension.
She developed type 1 diabetes mellitus at 9 years of age and claims that the condition
is under reasonable control. However, she knows that she has diabetic retinopathy and
proteinuria. Her blood pressure had been elevated in the range of 140 to 150/90 mm Hg
at the last three visits to her family physician. The patient does not smoke, drink
alcohol, or use recreational drugs. She adheres to a no-added-salt diet. The only
medication that she takes is insulin. On examination, blood pressure is 152/90 mm Hg
seated and 146/88 mm Hg standing. Body weight is 82 kg (181 Ib). The examination is
normal other than nonproliferative diabetic retinopathy. What part of kidney is
affected?
A) Renal cortex
B) Ureter
C) Pelvis
D) Glomerulus
E) Nephron
ANSWER: D
A 31-year-old woman is critically ill. The illness was manifested by oliguria, edema,
severe headache. Pasternatsky’s sigh is negative in both sides. On lab exam, WBC of
10.000/mcL; on urinalysis protein of 1,6g/L, erythrocyturia. What questions you want
to ask the patient?.
A) Whether there was a fever?
B) Did you have respiratory infection previously?
C) Does your mother have such disease?
D) Do you have children?
E) There is enough information for diagnosis
ANSWER: B
A 31-year-old woman presents to your office complaining of flank pain and fever for
the past 2 days. She has a history of multiple urinary tract infections. She has
suffered 2 in the previous 18 months that were treated successfully with trimethoprim-
sulfamethoxazole. She is sexually active and her last menstrual period was 6 days ago.
She takes no routine medications. Her temperature is 39.5 C (103.1 F), blood pressure
is 130/65 mm Hg, and pulse is 110/min. She has prominent right costovertebral angle
(CVA) tenderness. The most appropriate management of this patient is to
A) admit her to the intensive care unit
B) initiate therapy with ampicillin and gentamicin
C) obtain blood cultures and send her home pending results
D) obtain urine cultures and initiate therapy with ampicillin and gentamicin
E) obtain urine cultures and send her home pending results
ANSWER: D
A 31-year-old woman presents to your office complaining of flank pain and fever for
the past 2 days. She has a history of multiple urinary tract infections. She has
suffered 2 in the previous 18 months that were treated successfully with trimethoprim-
sulfamethoxazole. She is sexually active and her last menstrual period was 6 days ago.
She takes no routine medications. Her temperature is 39.5 C (103.1 F), blood pressure
is 130/65 mm Hg, and pulse is 110/min. She has prominent right costovertebral angle
(CVA) tenderness. What investigations will be the next?
A) blood cultures, urynalisis, CBC
B) ECG, chest X-ray, CBC
C) urine cultures, ECG
D) urine culture, biochemical blood test, ultrasound
E) CBC, ultrasound, ECG
ANSWER: D
A 31-yr-old male presented to A&E with a markedly swollen and tender left knee. He
says it's been present for about a month but has only recently become painful. He
admitted losing weight and on examination a temperature of 39,5 °C was found. He had
no other complaints apart from a 2-month history of cough, which he attributed to his
heavy smoking. Choose the most likely diagnosis from the below list of options.
A) Reiter's syndrome
B) Felty's syndrome
C) Osteoarthritis
D) Infective arthritis
E) Rheumatoid arthritis
ANSWER: A
A 33-year-old man is evaluated because of 4-year history of skin ulcers on his lower
extremities. The ulcers usually begin with tender, erythematous nodules that then
ulcerate and are resistant to treatment. A previous punch biopsy of the skin allegedly
showed thrombotic lesions in small blood vessels of the dermis, polymorphonuclear
neutrophils infiltrate all layers of the vessel wall and perivascular areas. The
physical examination is remarkable for the ulcers, and also for a diffuse livedo
reticularis pattern of skin discoloration and wasting of the web space between the
thumb and index finger on the left hand. His ability to grasp with that hand is
minimal. He has a right foot drop. Choose the single most likely treatment from the
list of options given below.
A) Tetracycline, prednisone
B) Prednisone
C) Ciprofloxacin, NSAIDs
D) Beta 2 agonist, heparin
E) IV aminophylline
ANSWER: B
A 33-yr-old woman has pain and stiffness in her hands, wrists, elbows, knees and
ankles. There is swelling of her MCP joints bilaterally. The patient does not smoke or
drink. On physical examination, her temperature is 37.8 °C. Examination of the joints
reveals thickening and tenderness of the metacarpophalangeal joints and proximal
interphalangeal joints bilaterally. Both wrists lack complete flexion and extension
and are warm to the touch. Choose the most likely diagnosis from the below list of
options
A) High ESR
B) High serum urate
C) Positive blood culture
D) Positive rheumatoid factor
E) High WBC count
ANSWER: D
A 34-year-old female with hypertension is considering becoming pregnant. Which of the
following medications would be absolutely CONTRAINDICATED to control her BP during
pregnancy?
A) Methyldopa
B) Metoprolol
C) Labetalol
D) Captopril
E) Nifidipine
ANSWER: D
A 34-year-old male with isolated essential hypertension presents to clinic and is
found to have a BP of 180/100 mmHg after failure of behavioral modifications. What is
the most appropriate next step?
A) Start hydrochlorothiazide
B) Start hydrochlorothiazide and lisinopril
C) Repeat BP in 4 weeks
D) Start amlodipine
E) Start doxazozin
ANSWER: B
A 34-year-old male with isolated essential hypertension presents to clinic and is
found to have a BP of 180/100 mmHg after failure of behavioral modifications. What is
the most appropriate next step?
A) Start bisoprolol
B) Start hydrochlorothiazide and lisinopril
C) Repeat BP in 4 weeks
D) Start amlodipine
E) Start doxazozin
ANSWER: B
A 34-year-old white male computer analyst is referred for evaluation of hypertension
(185/95 mm Hg) discovered during a blood pressure screening at his workplace. The
patient states that he is well and has not seen a physician in many years. He
describes himself as “a fitness freak, as he is an active jogger, abstains from
alcohol, and limits his salt and fat intake. He denies any knowledge of hypertension,
cardiovascular disease, renal disease, or diabetes mellitus. He takes no medications
regularly. Family history is significant in that his father was known to be
hypertensive and died of a stroke at 64 years of age. His older brother is being
treated for hypertension. On examination, the patient appears well, with a blood
pressure of 174/98 mm Hg while seated and standing. Body weight is 71 kg (157 Ib), and
height is 178 cm (70). Optic fundus examination is significant for grade II
hypertensive retinopathy. Electrocardiography demonstrates normal sinus rhythm with
left ventricular hypertrophy. What investigation will you prescribe to exclude renal
arterial hypertension first of all?
A) CBC, biochemical blood test, renal ultrasound
B) Urinalysis, biochemical blood test, renal ultrasound
C) Urinalysis, biochemical blood test, ECG monitoring
D) Urinalysis, arterial blood pressure monitoring, renal ultrasound
E) Urinalysis, CBC, renal ultrasound
ANSWER: B
A 34-year-old woman presents with a 23-month history of morning stiffness, pain, and
swelling in her hands and wrists. She has noticed general fatigue, and the hand
stiffness lasts about an hour. Joints of all her extremities are involved
symmetrically. The patient does not smoke or drink. On physical examination, her
temperature is 38 °C (100.4 °F). Examination of the joints reveals thickening and
tenderness of the metacarpophalangeal joints and proximal interphalangeal joints
bilaterally. Both wrists lack complete flexion and extension and are warm to the touch.
Which one of the following statements about rheumatoid arthritis is true?
A) Serum C-reactive protein measurement is a useful measure of joint inflammation
B) Most patients progress to complete disability
C) Oral corticosteroids form the mainstay of drug therapy
D) Epstein-Barr virus is known to be the causative agent.
E) Always responds to plasma exchange therapy
ANSWER: A
A 35-year-old construction worker reports having had fevers and chills for several
days. Examination reveals a temperature of 102°F, HR of 110 bpm, and BP of 120/85 mm
Hg. His teeth are in poor condition. His lungs are clear, and cardiac examination is
unremarkable. Blood cultures are drawn and grow Streptococcus viridans. He is
diagnosed with SBE. Despite antibiotics, the patient continues to have persistent
fever and develops acute dyspnea on the fifth hospital day. Physical examination is
likely to reveal:
A) An HSM at the apex
B) An early-peaking, crescendo-decrescendo murmur at the upper sternal border
C) Weak and delayed carotid upstrokes
D) An apical middiastolic murmur with presystolic accentuation and an OS
E) A three-component pericardial friction rub
ANSWER: A
A 35-year-old woman is in the hospital for a flare of nephritis related to systemic
lupus erythematosus (SLE). On rounds in the morning, she complains of right hip pain.
She states that for the last several weeks, she has had a deep aching in the hip and
now it is getting much worse since she was in the hospital. It hurts her both at rest
and with motion. She denies any history of trauma, and has not started any new
activities. There have been no fevers. Her only outpatient medication is prednisone 10
mg daily, and she takes ibuprofen for pain relief, which has helped minimally. However,
now she is on a higher dose of intravenous steroid. Her temperature is 37.2 C (99 F),
blood pressure is 132/82 mm Hg, and pulse is 72/min. There is no pain on palpation
over the hip but pain is present with range of motion. Laboratory studies show a
leukocyte count 8,100mm3 and a hematocrit 34%. What investigations can confirm
diagnosis of lupus-nephritis?
A) Renal ultrasound, CBC, abdominal X-ray
B) Biochemical blood test, ECG, CBC
C) Urynalysis, biochemical blood test, renal ultrasound
D) Urynalysis, biochemical blood test, ECG
E) Urynalysis, EchoCG, renal ultrasound
ANSWER: C
A 36-year-old female with diabetes mellitus and hyperlipidemia and no history of
hypertension is noted at her yearly clinic visit to have new-onset hypertension with a
BP of 180/110 mmHg. She undergoes screening for secondary causes of hypertension and
is found to have a pheochromocytoma. What of the following medications is
contraindicated as monotherapy?
A) Metoprolol
B) Lisinopril
C) Phentolamine
D) Hydrochlorothiazide
E) Captopril
ANSWER: A
A 36-year-old female with diabetes mellitus and hyperlipidemia and no history of
hypertension is noted at her yearly clinic visit to have new-onset hypertension with a
BP of 180/110 mmHg. She undergoes screening for secondary causes of hypertension and
is found to have a pheochromocytoma. What of the following medications is
contraindicated as monotherapy?
A) Propranolol
B) Lisinopril
C) Am;odipine
D) Hydrochlorothiazide
E) Captopril
ANSWER: A
A 37-year-old comes to the clinic for a required pre-employment physical examination.
She has no past medical history and has no complaints. Her temperature is 37.0 C (98.6
F), blood pressure is 110/60 mm Hg, pulse is 63/min, and respirations are 14/min. She
is currently menstruating. A urine culture, which is required by her new job, reveals
greater than 100,000 E. Coli colony-forming units. The most appropriate next step in
management of this patient's urine culture findings is
A) no further management is indicated
B) prescribe ciprofloxacin, orally, for 14 days
C) prescribe trimethoprim-sulfamethoxazole, orally, for 3 days
D) repeat urine culture in 2 weeks
E) send her for urological evaluation
ANSWER: A
A 37-year-old comes to the clinic for a required pre-employment physical examination.
She has no past medical history and has no complaints. Her temperature is 37.0 C (98.6
F), blood pressure is 110/60 mm Hg, pulse is 63/min, and respirations are 14/min. She
is currently menstruating. A urine culture, which is required by her new job, reveals
greater than 100,000 E. Coli colony-forming units. Your recommendations for this
patient?
A) repeat urine culture after menstruation
B) Prescribe renal ultrasound
C) prescribe biochemical blood test
D) prescribe excretory urography
E) send her for urological evaluation
ANSWER: A
A 37-year-old man develops acute abdominal pain and bloody diarrhea. Several weeks
later, he complains of diffuse musculoskeletal pain involving the neck, left wrist,
left knee, and low back. On examination he has marked swelling, warmth, and loss of
motion of the left wrist and left knee, as well as pain and restricted motion of the
neck and lower lumbar spine. Choose the most likely treatment from the below list of
options.
A) Cyclophosphanum
B) Penicillin
C) Tetracycline
D) Prednisone
E) Ceftriaxone
ANSWER: C
A 37-year-old woman develops acute abdominal pain and bloody diarrhea. Several weeks
later, she complains of diffuse musculoskeletal pain involving the neck, left wrist,
left knee, and low back. On examination she has marked swelling, warmth, and loss of
motion of the left wrist and left knee, as well as pain and restricted motion of the
neck and lower lumbar spine. Choose the most likely diagnosis from the below list of
options.
A) Rheumatoid arthritis
B) Reactive arthritis
C) Osteoarthritis
D) Haemochromatosis
E) Gonococcal arthritis
ANSWER: B
A 37-year-old woman presents with a 6-month history of morning stiffness, pain, and
swelling in her hands and wrists. She has noticed general fatigue, and the hand
stiffness lasts about an hour. Joints of all her extremities are involved
symmetrically. The patient does not smoke or drink. On physical examination, her
temperature is 38 °C (100.4 °F). Examination of the joints reveals thickening and
tenderness of the metacarpophalangeal joints and proximal interphalangeal joints
bilaterally. Both wrists lack complete flexion and extension and are warm to the touch.
X-ray examination of which joints you need for confirming diagnosis?
A) Hands
B) Ankles
C) Knees
D) Elbows
E) Spine
ANSWER: A
A 38-year-old man complained of headache. On physical examination, the skin was pale;
there was edema of face and hands, blood pressure was 160/120 mm Hg. On EchoCG, there
was concentric hypertrophy of the left ventricle. Ultrasound examination of the
kidneys revealed thinned cortical layer. Urinalysis showed proteinuria of 3.7 g/day.
What syndrome is leading?
A) Nephrotic syndrome
B) Nephritic syndrome
C) Urinary syndrome
D) Uremic syndrome
E) Edematous syndrome
ANSWER: A
A 38-year-old man is admitted to the hospital for acute deterioration in renal
function. He was seen in your office 2 days prior for some mild upper respiratory
complaints, including a sore throat, cough, and fever. He was prescribed cephalexin
and sent home. Today, his laboratory data returned and shows a blood urea nitrogen
level of 67 mg/dL and a creatinine level of 2.1 mg/dL. You called him and told him to
meet you at the hospital for further evaluation. On admission his BUN is now 109 mg/dL
and his creatinine is 4.2 mg/dL. The appropriate tests are ordered and an
electrocardiogram shows QRS complex widening and tall, peaked T waves. His temperature
is 38.3 C (101.0 F). He has an erythematous oropharynx with some mild tonsillar
exudate. His lungs are clear. It is observed that he has urinated only 5-10 ml in the
past 2 hours since his hospitalization. A urinalysis shows red cell casts and
dysmorphic red blood cells. The most appropriate next step is to
A) administer high-dose methyl-prednisolone, intravenously
B) administer low-dose methyl-prednisolone, intravenously
C) administer penicillin, intravenously
D) prescribe high-dose cyclophosphamide, orally
E) prescribe penicillin, orally
ANSWER: A
A 38-year-old man is admitted to the hospital for acute deterioration in renal
function. He was seen in your office 2 days prior for some mild upper respiratory
complaints, including a sore throat, cough, and fever. He was prescribed cephalexin
and sent home. Today, his laboratory data returned and shows a blood urea nitrogen
level of 67 mg/dL and a creatinine level of 2.1 mg/dL. You called him and told him to
meet you at the hospital for further evaluation. On admission his BUN is now 109 mg/dL
and his creatinine is 4.2 mg/dL. The appropriate tests are ordered and an
electrocardiogram shows QRS complex widening and tall, peaked T waves. His temperature
is 38.3 C (101.0 F). He has an erythematous oropharynx with some mild tonsillar
exudate. His lungs are clear. It is observed that he has urinated only 5-10 ml in the
past 2 hours since his hospitalization. A urinalysis shows red cell casts and
dysmorphic red blood cells. Wht disorders can explain changes on ECG?
A) Hyperkalemia
B) Hypokalemia
C) Hypercalcemia
D) Hypocalcemia
E) Hyperphosphatemia
ANSWER: A
A 38-year-old man presents for hypertension discovered during a pre-employment
examination. He is healthy but has a family history of hypertension in both parents
and two siblings. He has no history of cardiovascular disease and does not use tobacco,
alcohol, or recreational drugs. He is taking no medications. The patient appears well.
Height is 173 cm (68), body weight is 78 kg (172 Ib), and blood pressure is 158/102 mm
Hg seated and standing. The physical examination is otherwise normal. A complete blood
count and electrolyte panel are normal. The serum creatinine concentration is 1.8
mg/dL, and urinalysis reveals 2+ proteinuria. Which is the most appropriate
antihypertensive therapy for this patient?
A) Intensive lifestyle modification
B) Diuretic
C) Nondihydropyridine calcium channel blocker
D) Angiotensin-converting enzyme inhibitor
E) none of them
ANSWER: D
A 38-year-old man with a 13-yr history of back pain, presented with acute pain and
swelling of one knee. On examination, the joint was tender and restricted in movement.
X-ray of the knee showed periarticular osteoporosis. On investigation, he had a raised
erythrocyte sedimentation rate of 32 mm/h, a mild anemia (Hb 106g/l) but no detectable
serum rheumatoid factor. The knee effusion was aspirated; the fluid contained a
polymorphonuclear leucocytosis but no organisms or rheumatoid factor. Fifteen months
later he developed an iritis in his left eye, low back pain and stiffness. His
peripheral joints were normal but pain could be elicited in both sacroiliac joints.
Choose the medications for patient with shown changes from the below list of options.
A) Methotrexate
B) Actovegin
C) Mydocalm
D) Warfarin
E) Pentoxifylline
ANSWER: A
A 38-year-old man with a 13-yr history of back pain, presented with acute pain and
swelling of one knee. On examination, the joint was tender and restricted in movement.
X-ray of the knee showed periarticular osteoporosis. On investigation, he had a raised
erythrocyte sedimentation rate of 102mm/h, a mild anaemia (Hb 106g/l) but no
detectable serum rheumatoid factor. The knee effusion was aspirated; the fluid
contained a polymorphonuclear leucocytosis but no organisms or rheumatoid factor.
Fifteen months later he developed an iritis in his left eye, low back pain and
stiffness. His peripheral joints were normal but pain could be elicited in both
sacroiliac joints. Choose the most likely diagnosis from the below list of options for
patient with shown changes
A) Ankylosing spondylitis
B) Reactive arthritis
C) Osteoarthritis
D) Gout
E) Psoriatic arthritis
ANSWER: A
A 38-year-old man with a 13-yr history of back pain,presented with acute pain and
swelling of one knee. On examination, the joint was tender and restricted in movement.
X-ray of the knee showed periarticular osteoporosis. On investigation, he had a raised
erythrocyte sedimentation rate of 32 mm/h, a mild anemia (Hb 106g/l) but no detectable
serum rheumatoid factor. The knee effusion was aspirated; the fluid contained a
polymorphonuclear leucocytosis but no organisms or rheumatoid factor. Fifteen months
later he developed an iritis in his left eye, low back pain and stiffness. His
peripheral joints were normal but pain could be elicited in both sacroiliac joints.
Which of the following medications is indicated for the treatment of the disease?
A) Sulfasalazine
B) Cyclosporine
C) Diclofenac
D) Pentoxifylline
E) Corticosteroids
ANSWER: A
A 39-year-old woman with a 3-year history of rheumatoid arthritis is evaluated because
of slight shortness of breath of new onset when she climbs the stairs in her home. She
also has a dry hacking cough of 3 weeks duration. She has been active, working part-
time as a secretary as well as caring for two young children at home. She has been
taking methotrexate, 20 mg by mouth each week, and folic acid, 1 mg/d, for 1 year, and
she says her joints have been great. On physical examination, her temperature is
37.7 °C (99.8 °F). Fine crackles are audible at the bases of both lungs on deep
inspiration. There is some tenderness with full palmar flexion and dorsi flexion of
the right wrist; the rest of the joints are brought through full range of painless
motion. Complete blood count and serum albumin levels are normal. Chest radiograph is
normal. Which of the following is the best next step in this patient management?
A) Order liver biopsy.
B) Obtain radiographs of wrists.
C) Stop the methotrexate.
D) Chest X-ray
E) Initiate prednisone therapy.
ANSWER: C
A 40-year-old diabetic patient presents with a blood pressure (BP) of 145/90 mmHg and
proteinuria. What is the best medication for the initial management of this patient’s
hypertension?
A) Calcium channel blockers
B) Beta blockers
C) ACE-inhibitors / angiotensin receptor blockers
D) Alpha blockers
E) Thiazide diuretics
ANSWER: C
A 40-year-old diabetic patient presents with a blood pressure (BP) of 145/90 mmHg and
proteinuria. Which BP profile represents the best therapeutic goal for this patient?
A) <160/90
B) <140/90
C) <130/85
D) <125/75
E) <140/85
ANSWER: C
A 40-year-old diabetic patient presents with a blood pressure (BP) of 145/90 mmHg and
proteinuria. What is the best medication for the initial management of this patient’s
hypertension?
A) Calcium channel blockers
B) Beta blockers
C) ACE-inhibitors / angiotensin receptor blockers
D) Alpha blockers
E) Diuretics
ANSWER: C
A 40-year-old diabetic patient presents with a blood pressure (BP) of 145/90 mmHg and
proteinuria. What is the best medication for the initial management of this patient’s
hypertension?
A) Calcium channel blockers
B) Beta blockers
C) ACE-inhibitors / angiotensin receptor blockers
D) Alpha blockers
E) Diuretics
ANSWER: C
A 40-year-old man complained of headache in occipital area. On physical examination,
the skin was pale; there was face and hand edema, blood pressure of 170/130 mm Hg. On
EchoCG, there was concentric hypertrophy of the left ventricle. Ultrasound examination
of the kidneys revealed thinned cortical layer. Urinalysis showed proteinuria of 3.5
g/day. What is the probable diagnosis
A) Chronic glomerulonephritis
B) Essential arterial hypertension
C) Chronic pyelonephritis
D) Polycystic disease of the kidneys.
E) Cushing’s disease
ANSWER: A
A 40-year-old man complained of headache in occipital area. On physical examination,
the skin was pale; there was face and hand edema, blood pressure of 170/130 mm Hg. On
EchoCG, there was concentric hypertrophy of the left ventricle. Ultrasound examination
of the kidneys revealed thinned cortical layer. Urinalysis showed proteinuria of 1.5
g/day. What criteria of nephritic syndrome in this patient?
A) Proteinuria 1-3 g/day, severe hypertension, edema
B) Proteinuria more than 3 g/day, severe hypertension, edema
C) Proteinuria more than 3 g/day, concentric hypertrophy of the left ventricle, edema
D) Proteinuria 1 g/day, concentric hypertrophy of the left ventricle, high fever
E) Proteinuria more 1-2 g/day, high fever, edema
ANSWER: A
A 40-year-old man complained of headache in occipital area. On physical examination,
the skin was pale; there was face and hand edema, blood pressure of 170/130 mm Hg. On
EchoCG, there was concentric hypertrophy of the left ventricle. Ultrasound examination
of the kidneys revealed thinned cortical layer. Urinalysis showed proteinuria of 5.5
g/day. What investigation is the most helpful for diagnosis confirmation and
prescribing of treatment?
A) Renal biopsy.
B) Arterial blood pressure monitoring.
C) Daily proteinuria.
D) Nechiporenko test.
E) Biochemical blood test
ANSWER: A
A 40-year-old man has recurrent nephrolithiasis due to idiopathic hypercalciuria. He
has had more than 40 calcium oxalate stones in the past 5 years. He is started on
hydrochlorothiazide therapy and a low-sodium diet. During treatment, his 24-hour
urinary calcium concentration decreases from 385 mg/d to 180 mg/d. No new stones have
formed in the past 6 months; however, hypokalemia has developed (serum potassium level,
2.9 meq/L). Taking the hypokalemia into account, what therapy should the patient
receive for hypercalciuric stone disease?
A) High-potassium diet plus hydrochlorothiazide
B) Acetazolamide plus hydrochlorothiazide
C) Magnesium oxide plus hydrochlorothiazide
D) Amiloride plus hydrochlorothiazide
E) none of them
ANSWER: D
A 40-yr-old male presents with fever, malaise, weight loss and joint pain, chest pain.
Choose the single most likely treatment from the list of options given below.
A) Tetracycline
B) Ciprofloxacin
C) Beta 2 agonist
D) Cyclophosphamide
E) IV aminophylline
ANSWER: D
A 41-year-old female with chronic obstructive pulmonary disease is found on multiple
office visits to have elevated BP measurements. Which of the following medications is
contraindicated?
A) Hydrochlorothiazide
B) Atenolol
C) Lisinopril
D) None of the above
E) All of the above
ANSWER: B
A 41-yr-old woman has pain and stiffness in her hands, wrists, elbows, knees and
ankles. There is swelling of her MCP joints bilaterally. On physical examination, her
temperature is 38 °C (100.4 °F). Examination of the joints reveals thickening and
tenderness of the metacarpophalangeal joints and proximal interphalangeal joints
bilaterally. Both wrists lack complete flexion and extension and are warm to the touch.
Choose the most likely diagnosis from the below list of options. Drug of choice for
patient with rheumatoid artritis
A) Methotrexate
B) NSAIDs
C) Hydroxychloroquine sulphate
D) Sulphasalazine
E) D-penicillamine
ANSWER: A
A 42-year-old female with chronic obstructive pulmonary disease is found on multiple
office visits to have elevated BP measurements. Which of the following medications is
contraindicated?
A) Hydrochlorothiazide
B) Metoprolol
C) Lisinopril
D) Amlodipine
E) All of the above
ANSWER: B
A 42-year-old female with chronic obstructive pulmonary disease is found on multiple
office visits to have elevated BP measurements. Which of the following medications is
contraindicated?
A) Hydrochlorothiazide
B) Metoprolol
C) Perindopril
D) None of the above
E) All of the above
ANSWER: B
A 42-year-old male comes in for a routine physical examination. He is noted to have a
BMI of 30, impaired fasting glucose, and a BP of 135/85 mmHg. What is the best
treatment plan for this individual?
A) Aggressive lifestyle modification
B) Institute thiazide diuretic regimen
C) No treatment at this time
D) Initiate an ACE-inhibitor
E) Initiate a beta-blocker
ANSWER: A
A 42-year-old male comes in for a routine physical examination. He is noted to have a
BMI of 30, impaired fasting glucose, and a BP of 135/85 mmHg. What is the best
treatment plan for this individual?
A) Aggressive lifestyle modification
B) Thiazide diuretic
C) No treatment at this time
D) ACE-inhibitor
E) Beta-blocker
ANSWER: A
A 42-year-old man presents with Loffler endocarditis. Expected findings include all of
the following EXCEPT:
A) Eosinophilia
B) Signs and symptoms of heart failure
C) Asthma and nasal polyposis
D) Right ventricular pressure tracing showing "dip and plateau" pattern
E) Normal left ventricular ejection fraction
ANSWER: C
A 42-year-old woman presents with a 2-month history of morning stiffness, pain, and
swelling in her hands and wrists. She has noticed general fatigue, and the hand
stiffness lasts about an hour. Joints of all her extremities are involved
symmetrically. The patient does not smoke or drink. On physical examination, her
temperature is 38 °C (100.4 °F). Examination of the joints reveals thickening and
tenderness of the metacarpophalangeal joints and proximal interphalangeal joints
bilaterally. Both wrists lack complete flexion and extension and are warm to the touch.
The patient has hemoglobin is 10 g/dL, erythrocyte sedimentation rate is 50 mm/h, and
serum rheumatoid factor positive. Choose the most likely diagnosis from the below list
of options. Which of the following tests is most likely to yield the diagnosis?
A) Pelvic examination and cervical culture
B) Joint fluid aspiration
C) Antinuclear antibody (ANA) testing
D) Rheumatoid factor testing
E) Streptococcal enzyme testing
ANSWER: D
A 44-year-old man with a history of a bicuspid AV is diagnosed with endocarditis.
Despite 2 weeks of appropriate antibiotic therapy, he has persistent fevers and
bacteremia. A diagnostic study is performed and a paravalvular abscess discovered.
What is the next step in this patient's management?
A) Add gentamicin
B) Refer for surgery
C) Continue current antibiotic therapy
D) Change nafcillin to ceftriaxone
E) Change nafcillin to vancomycin
ANSWER: B
A 44-yr-old man presented with a 2-month history of fatigue, abdominal pain and
progressive numbness in his feet. He recently developed moderate polyarthritis. On
examination there was evidence of left median nerve mononeuritis, livedo reticularis,
BP 160/110 mmHg (he has always been normotensive), and laboratory studies reveal
anemia of chronic disease, erythrocyte sedimentation rate – 47 mm/h. Choose the single
most likely treatment from the list of options given below.
A) Prednisone 20 mg/d
B) Prednisone 15 mg/d
C) Ciprofloxacin, NSAIDs
D) Prednisone 1 mg/kg/d
E) Tetracycline, prednisone
ANSWER: D
A 45 year old female is admitted with cellulitus requiring IV antibiotics. She is
commenced on IV benzylpenicillin and IV flucloxacillin. She is commenced on Diclofenac
for pain. After 3 days she develops pyrexia, arthralgia, maculopapular rash,
haematuria and oliguria. Bloods reveal acute renal failure and eosinophilia. A renal
biopsy reveals interstitial cellular infiltrate with eosinophils present and variable
tubular necrosis. What is the most likely diagnosis?
A) Penicillin induced acute tubulointerstitial nephritis
B) NSAID induced acute tubulointerstitial nephritis
C) Chronic tubulointerstitial nephritis
D) Glomerulonephritis
E) Anaphylaxis
ANSWER: A
A 45-year-old man is hospitalized after 5 days of severe diarrhea and anorexia. He has
a history of renal failure secondary to chronic glomerulonephritis and received a
living related renal transplant 2 years earlier. He takes cyclosporine, mycophenolate
mofetil, and prednisone, as well as amlodipine to treat hypertension. He has had no
rejection episodes or opportunistic infections. The remainder of his history is
unremarkable. On admission, blood pressure is 100/70 mm Hg and pulse rate is 100/min
while supine; blood pressure is 80/50mm Hg and pulse rate is 120/min while standing.
Body temperature is 37.5°C (99.5 °F). Physical examination reveals a thin man in no
distress. Mucus membranes are dry, and neck veins are flat at 30-degree elevation.
Cardiopulmonary examination is unremarkable except for resting tachycardia. Abdominal
palpation reveals diffuse tenderness without peritoneal signs. No peripheral edema is
present. Laboratory studies: Blood urea nitrogen 40 mg/dL Serum creatinine - 1.0 mg/dL.
Serum sodium - 134 mmol/L. Serum potassium - 3.8 meq/L. Serum chloride - 108 meq/L.
Serum bicarbonate - 16 meq/L. What contributes to the rapid progression of the disease
in this patient?
A) Renal failure
B) Dehydration
C) Immunostimulation
D) Immunosupression
E) Hypotension
ANSWER: D
A 45-year-old woman presents with a 2-month history of morning stiffness, pain, and
swelling in her hands and wrists. She has noticed general fatigue, and the hand
stiffness lasts about an hour. Joints of all her extremities are involved
symmetrically. The patient does not smoke or drink. On physical examination, her
temperature is 38 °C (100.4 °F). Examination of the joints reveals thickening and
tenderness of the metacarpophalangeal joints and proximal interphalangeal joints
bilaterally. Both wrists lack complete flexion and extension and are warm to the touch.
The patient has hemoglobin is 10 g/dL, erythrocyte sedimentation rate is 50 mm/h, and
serum rheumatoid factor positive. Choose the most likely diagnosis from the below list
of options. Which of the following systemic manifestations is LEAST characteristic of
early adult rheumatoid arthritis?
A) Vague musculoskeletal symptoms
B) Fatigue
C) High fever
D) Weight loss
E) Muscle wasting
ANSWER: C
A 45-year-old woman who has had slowly progressive renal failure begins to complain of
increasing numbness and prickling sensations in her legs. Examination reveals loss of
pinprick and vibration sensation below the knees, absent ankle jerks, and impaired
pinprick sensation in the hands. Serum creatinine concentration, checked during her
most recent clinic visit, is 790 umol/L (8.9 mg/dL). The woman's physician should now
recommend
A) a therapeutic trial of phenytoin
B) a therapeutic trial of pyridoxine (vitamin B6)
C) a therapeutic trial of cyanocobalamin (vitamin B12)
D) initiation of renal replacement therapy
E) neurologic referral for nerve conduction studies
ANSWER: D
A 46-year-old woman with rheumatoid arthritis is evaluated because the subcutaneous
nodules that were present on her elbows have increased in size, and she has developed
some nodules on the Achilles tendons bilaterally. She has been treated for 8 months
with methotrexate in an ascending dose schedule. She is currently taking 17.5 mg/week,
along with enteric-coated aspirin, 3 g/d. This regimen has alleviated her joint pain;
her morning stiffness is limited to 35 minutes, as opposed to the previous duration of
2 hours. On examination the nodules are not warm, and her joints appear much improved.
No other physical abnormalities are found. Which of the following is the most
appropriate next step in this patients management?
A) Biopsy a newly developed nodule.
B) Reassure the patient that these nodules occur occasionally with methotrexate
therapy.
C) Initiate antituberculosis therapy.
D) Measure fasting serum lipids, including triglycerides.
E) Measure serum uric acid.
ANSWER: B
A 47-yr-old man presented with a 7-month history of fatigue, abdominal pain and
progressive numbness in his feet. He recently developed moderate polyarthritis. On
examination there was evidence of left median nerve mononeuritis, livedo reticularis,
BP 160/110 mmHg (he has always been normotensive), and laboratory studies reveal
anemia of chronic disease, high erythrocyte sedimentation rate. Urine analysis reveals
proteinuria, erythrocyturia. Chest radiography showed cardiomegaly. Choose the single
most likely treatment from the list of options given below.
A) Tetracycline, prednisone
B) Prednisone
C) Ciprofloxacin, NSAIDs
D) Beta 2 agonist, heparin
E) IV aminophylline
ANSWER: B
A 48-year old man with an anterior STEMI receives fibrinolytic therapy with reteplase.
Thirty min. after the second bolus he continues to have chest pain and his ECG
demonstrates mild (estimated 30% resolution) of the ST segment elevations. What is the
next correct step in his management?
A) Readministration of a different fibrinolytic agent
B) Administration of GP IIb/IIIa inhibitor
C) Immediate coronary angiography and rescue PCI
D) Symptomatic relief of angina and heart failure
E) Medical therapy and PCI at a later date
ANSWER: C
A 48-year old man with an anterior STEMI receives fibrinolytic therapy with reteplase.
Thirty min after the second bolus he continues to have chest pain and his ECG
demonstrates mild (estimated 30% resolution) of the ST segment elevations. What is the
next correct step in his management?
A) Readministration of a different fibrinolytic agent
B) Administration of GP IIb/IIIa inhibitor
C) Immediate coronary angiography and rescue PCI
D) Symptomatic relief of angina and heart failure
E) Medical therapy and PCI at a later date
ANSWER: C
A 48-year old man with an anterior STEMI receives fibrinolytic therapy with reteplase.
Thirty min. after the second bolus he continues to have chest pain and his ECG
demonstrates mild (estimated 20% resolution) of the ST segment elevations. What is the
next correct step in his management?
A) Readministration of a different fibrinolytic agent
B) Administration of GP IIb/IIIa inhibitor
C) Immediate coronary angiography and rescue PCI
D) Symptomatic relief of angina and heart failure
E) Medical therapy and PCI at a later date
ANSWER: C
A 48-year-old man is admitted to the coronary care unit with an acute inferior
myocardial infarction. Two hours after admission, his blood pressure is 86/52 mmHg;
his heart rate is 40 beats per minute with sinus rhythm. Which of the following would
be the most appropriate initial therapy?
A) Immediate insertion of a temporary transvenous pacemaker
B) Intravenous administration of atropine sulfate, 0.6 mg
C) Administration of normal saline, 500 mL over 15 min
D) Intravenous administration of dobutamine, 0.35 mg/min
E) Intravenous administration of isoproterenol, 5.0 g/min
ANSWER: B
A 48-year-old man is admitted to the coronary care unit with an acute inferior
myocardial infarction. Two hours after admission, his blood pressure is 86/52 mmHg;
his heart rate is 40 beats per minute with sinus rhythm. Which of the following would
be the most appropriate initial therapy?
A) Immediate insertion of a temporary transvenous pacemaker
B) Intravenous administration of atropine sulfate, 0.6 mg
C) Administration of normal saline, 300 mL over 15 min
D) Intravenous administration of dobutamine, 0.35 mg/min
E) Intravenous administration of isoproterenol, 5.0 g/min
ANSWER: B
A 48-year-old man is admitted to the coronary care unit with an acute inferior
myocardial infarction. Two hours after admission, his blood pressure is 86/52 mmHg;
his heart rate is 40 beats per minute with sinus rhythm. Which of the following would
be the most appropriate initial therapy?
A) Immediate insertion of a temporary transvenous pacemaker
B) Intravenous administration of atropine sulfate, 0.6 mg
C) Administration of normal saline, 300 mL over 15 min
D) Intravenous administration of dobutamine, 0.35 mg/min
E) Intravenous administration of isoproterenol, 5.0 g/min
ANSWER: B
A 48-year-old obese male with hypertension, dyslipidemia, and diabetes mellitus
presents to the outpatient clinic for his yearly physical. He has refused medications
in the past, but now is willing to consider treatment. His BP is 145/95 mmHg with a HR
of 80 bpm. His laboratory data are significant for the presence of microalbuminuria.
Which of the following medications would be the most appropriate?
A) Carvedilol
B) Methyldopa
C) Lisinopril
D) Torasemide
E) Terazozin
ANSWER: C
A 48-year-old obese male with hypertension, dyslipidemia, and diabetes mellitus
presents to the outpatient clinic for his yearly physical. He has refused medications
in the past, but now is willing to consider treatment. His BP is 145/95 mmHg with a HR
of 80 bpm. His laboratory data are significant for the presence of microalbuminuria.
Which of the following medications would be the most appropriate?
A) Carvedilol
B) Methyldopa
C) Lisinopril
D) Chlorthalidone
E) Terazozin
ANSWER: C
A 48-year-old obese male with hypertension, dyslipidemia, and diabetes mellitus
presents to the outpatient clinic for his yearly physical. He has refused medications
in the past, but now is willing to consider treatment. His BP is 145/95 mmHg with a HR
of 80 bpm. His laboratory data are significant for the presence of microalbuminuria.
Which of the following medications would be the most appropriate?
A) Carvedilol
B) Methyldopa
C) Lisinopril
D) Chlorthalidone
E) Terazozin
ANSWER: C
A 48-year-old woman presented with shortness of breath and oedema. She had felt
generally unwell for the last few weeks with increasing tiredness, loss of appetite
and abdominal discomfort. Her serum creatinine was 105 .mol/L (60–110). She was
treated with intravenous furosemide at doses up to 120 mg twice a day, but there was
no improvement in her symptoms and her renal function deteriorated. On examination,
her blood pressure was 125/65 mmHg and her jugular venous pressure was not elevated.
She had edema to the thigh. Auscultation of the chest revealed dullness to percussion
on the left. Investigations: serum sodium 128 mmol/L (137–144), serum potassium 3.2
mmol/L (3.5–4.9), serum urea 31.5 mmol/L (2.5–7.0), serum creatinine 351 umol/L (60–
110), serum albumin 15 g/L (37–49), 24-h urinary total protein 15.8 g (<0.2) What is
the most appropriate next step in management?
A) increase dose of furosemide
B) intravenous sodium chloride 0.9%
C) start enalapril
D) start metolazone
E) start steroidtherapy
ANSWER: E
A 50-year old man presents to the emergency room about 1.5 hrs after the onset of
chest pain. An ECG showed 3 mm ST elevation in the anterior and lateral leads. Which
of the following statements regarding his management is correct?
A) Primary PCI is recommended over thrombolytics because PCI has short-term mortality
benefit, reduced reinfarction risk, and reduced risk of stroke
B) Thrombolysis is recommended because door-to-baloon time is &amp;amp;gt;90 min.
C) Half-dose thrombolysis should be recommended; then the patient should be
transferred for PCI.
D) The patient should be referred for emergency CABG for better survival advantage.
E) Scheduled CABG
ANSWER: B
A 50-year old man presents to the emergency room about 1.5 hrs after the onset of
chest pain. An ECG showed 3 mm ST elevation in the anterior and lateral leads. Which
of the following statements regarding his management is correct?
A) Primary PCI is recommended over thrombolytics because PCI has short-term mortality
benefit, reduced reinfarction risk, and reduced risk of stroke
B) Thrombolysis is recommended because door-to-baloon time is &amp;amp;gt;90 min.
C) Half-dose thrombolysis should be recommended; then the patient should be
transferred for PCI.
D) The patient should be referred for emergency CABG for better survival advantage.
E) Scheduled CABG
ANSWER: A
A 50-year-old male is evaluated because of pain in the right inguinal area, lower back,
and hands. The inguinal pain worsens as he walks, and all pain increases as the day
progresses. On physical examination, internal rotation of the right hip elicits the
groin pain. The second, third, and fourth metacarpophalangeal joints are swollen, warm,
and tender to pressure. Radiographs show severe osteoarthritis in the right hip. Which
of the following is the best test to confirm the diagnosis?
A) Rheumatoid factor
B) HLA-B27
C) Serum iron and iron-binding capacity
D) Fasting plasma glucose
E) Serum uric acid
ANSWER: C
A 50-year-old male with chronic kidney disease and hypertension has a BP of 165/110
mmHg. What is this patient's target BP according to the JNC 7 guidelines?
A) 140/90 mmHg
B) 130/80 mmHg
C) 120/80 mmHg
D) 115/70 mmHg
E) 110/70 mmHg
ANSWER: B
A 50-year-old male with chronic kidney disease and hypertension has a BP of 165/110
mmHg. What is this patient's target BP according to the JNC 7 guidelines?
A) 140/90 mmHg
B) &amp;amp;lt;130/80 mmHg
C) 120/80 mmHg
D) 115/70 mmHg
E) 110/70 mmHg
ANSWER: B
A 50-year-old woman presents with a 2-month history of morning stiffness, pain, and
swelling in her hands and wrists. She has noticed general fatigue, and the hand
stiffness lasts about an hour. Joints of all her extremities are involved
symmetrically. The patient does not smoke or drink. On physical examination, her
temperature is 38 °C (100.4 °F). Examination of the joints reveals thickening and
tenderness of the metacarpophalangeal joints and proximal interphalangeal joints
bilaterally. Both wrists lack complete flexion and extension and are warm to the touch.
The patient hemoglobin is 10 g/dL, erythrocyte sedimentation rate is 50 mm/h, and
serum rheumatoid factor positive. Choose the most likely diagnosis from the below list
of options.
A) Ankylosing spondylitis
B) Rheumatoid arthritis
C) Osteoarthritis
D) Gout
E) Psoriatic arthritis
ANSWER: B
A 50-yr-old woman complains of stiffness in her fingers worse at the end of the day.
The DIP joints and the first metacarpophalangeal joints are affected. Choose the most
likely diagnosis from the below list of options.
A) Osteoarthritis
B) Rheumatoid arthritis
C) Psoriatic arthropathy
D) Haemochromatosis
E) Gonococcal arthritis
ANSWER: A
A 51-year-old man transfers to your practice. His medical history is positive for
primary hypertension without TOD. He has no history of renal or prostatic disease.
Laboratory values obtained from his former primary care physician show normal results
for BUN, serum creatinine, electrolytes, urinalysis, PSA, and ECG. He takes the alpha-
blocker doxazosin, 2 mg at bedtime.On examination, BP is 150/90 mmHg seated and
standing. The remainder of the examination is normal. What is the appropriate course
of action regarding the patient’s antihypertensive therapy?
A) Advise a low-sodium diet
B) Discontinue doxazosin therapy and consider an alternative agent
C) Advise high dietary intake of calcium and potassium
D) Increase the doxazosin to 4 mg a day
E) Advise magnesium supplements
ANSWER: B
A 51-year-old man transfers to your practice. His medical history is positive for
primary hypertension without TOD. He has no history of renal or prostatic disease.
Laboratory values obtained from his former primary care physician show normal results
for BUN, serum creatinine, electrolytes, urinalysis, PSA, and ECG. He takes the alpha-
blocker doxazosin, 2 mg at bedtime.On examination, BP is 150/90 mmHg seated and
standing. The remainder of the examination is normal.
A) What is the appropriate course of action regarding the patient’s antihypertensive
therapy?
B) Advise a low-sodium diet
C) Discontinue doxazosin therapy and consider an alternative agent
D) Advise high dietary intake of calcium and potassium
E) Increase the doxazosin to 4 mg a day
F) Advise magnesium supplements
ANSWER: C
A 51-year-old man transfers to your practice. His medical history is positive for
primary hypertension without TOD. He has no history of renal or prostatic disease.
Laboratory values obtained from his former primary care physician show normal results
for BUN, serum creatinine, electrolytes, urinalysis, PSA, and ECG. He takes the alpha-
blocker doxazosin, 2 mg at bedtime.On examination, BP is 150/90 mmHg seated and
standing. The remainder of the examination is normal. What is the appropriate course
of action regarding the patient’s antihypertensive therapy?
A) Advise a low-sodium diet
B) Discontinue doxazosin therapy and consider an alternative agent
C) Advise high dietary intake of calcium and potassium
D) Increase the doxazosin to 4 mg a day
E) Advise magnesium supplements
ANSWER: B
A 51-yr-old woman complains of stiffness in her fingers worse at the end of the day.
The DIP joints and the first metacarpophalangeal joints are affected. Choose the most
likely diagnosis from the below list of options.
A) Rheumatoid arthritis
B) Psoriatic arthropathy
C) Osteoarthritis
D) Haemochromatosis
E) Gonococcal arthritis
ANSWER: C
A 52-year-old male with diabetes mellitus, hypertension, and hyperlipidemia presents
to the ER with hypertensive emergency. His mean arterial pressure is 160 mmHg. Which
medications would be most appropriate therapy for this patient?
A) Nitroprusside
B) Enteral metoprolol
C) Fenoldopam
D) Intravenous nitroglycerine
E) Any of the above
ANSWER: A
A 53-yr-old woman complains of redness, swelling and stiffness in the distal
interphalangeal joints of her hands, but has no other joint complaints. Choose the
most likely diagnosis from the below list of options.
A) Gout
B) Still's disease
C) Pseudogout
D) Rheumatoid arthritis
E) Osteoarthritis
ANSWER: E
A 54-year old man with a history of hypertension presents with chest pain that started
2 hours ago and is diagnosed in the Emergency Department with anterior STEMI. His BP
is 200/100 mmHg. He receives i.v. metoprolol and the BP falls to 170/85 mmHg. What is
the most appropriate reperfusion strategy?
A) Fibrinolytic therapy
B) Angiography and PCI
C) Medical therapy alone and PCI at a later date
D) CABG
E) None of the above
ANSWER: B
A 55-year-old male presents with severe substernal chest pain for the last hour. It
began at rest and is associated with dyspnea and nausea. The electrocardiogram shows
bradycardia with a Mobitz type II second-degree block. Chest plain film is normal.
Which of the following is likely to be found in addition on the electrocardiogram?
A) ST elevation in V1-V3
B) Wellen’s T waves
C) ST elevation in II, III, aVF
D) ST depression in I and aVL
E) No other abnormality
ANSWER: C
A 55-year-old male presents with severe substernal chest pain for the last hour. It
began at rest and is associated with dyspnea and nausea. The electrocardiogram shows
bradycardia with a Mobitz type II second-degree block. Chest plain film is normal.
Which of the following is likely to be found in addition on the electrocardiogram?
A) ST elevation in V1-V3
B) Wellen’s T waves
C) ST elevation in II, III, aVF
D) ST depression in I and aVL
E) No other abnormality
ANSWER: C
A 55-year-old male presents with severe substernal chest pain for the last hour. It
began at rest and is associated with dyspnea and nausea. The electrocardiogram shows
bradycardia with a Mobitz type II second-degree block. Chest plain film is normal.
Which of the following is likely to be found in addition on the electrocardiogram?
A) ST elevation in V1-V3
B) Wellen’s T waves
C) ST elevation in II, III, aVF
D) ST depression in I and aVL
E) No other abnormality
ANSWER: C
A 55-year-old male with diabetes mellitus presents to his cardiologist with a BP of
165/95 mmHg. According to the JNC 7 guidelines, he would be classified as what stage
of hypertension and what is his target BP measurement?
A) Prehypertension, 140/90 mmHg
B) Stage 2, 130/80 mmHg
C) Stage 1, 140/90 mmHg
D) Stage 2, 140/90 mmHg
E) Stage 3, 120/70 mmHg
ANSWER: B
A 55-year-old male with diabetes mellitus presents to his cardiologist with a BP of
165/95 mmHg. According to the JNC 7 guidelines, he would be classified as… What stage
of hypertension and what is his target BP measurement?
A) Prehypertension, 140/90 mmHg
B) Stage 2, 130/80 mmHg
C) Stage 1, 140/90 mmHg
D) Stage 2, 140/90 mmHg
E) Stage 3, 120/70 mmHg
ANSWER: B
A 55-year-old man comes to the emergency department with pain on urination, fever and
chills. He also complains of perineal and suprapubic tenderness as well as dysuria and
hesitancy. His allergies include codeine, sulfonamides, and quinidine. Temperature is
38.5 C (101.3 F), blood pressure is 132/90 mm Hg, pulse is 88/min, and respirations
are 18/min. Abdominal examination is remarkable for suprapubic tenderness. Digital
rectal examination demonstrates a swollen, boggy, and exquisitely painful prostate
gland. Laboratory studies show a leukocyte count of 11,500/mm3, creatinine of 0.9
mg/dL, and blood urea nitrogen of 16 mg/dL. A urinalysis shows too numerous to count
white blood cells and Gram-negative rods. The most appropriate treatment for this
patient is
A) Clarithromycin 500 mg by mouth twice daily for 14 days
B) meropenem 1 gram intravenously daily for 5 days
C) ciprofloxacin 500 mg by mouth twice daily for 14 days
D) clindamycin 300 mg 4 times daily for 10 days
E) trimethoprim-sulfamethoxazole 1 double strength tablet twice daily for 14 days
ANSWER: C
A 55-year-old man presented to the clinic with a nodule on his elbow. He reported that
the nodule had been bothering him for 2 years and had steadily increased in size
recently. Further questioning revealed a history of repeated attacks of acute joint
pain. The physical examination revealed a rounded, subcutaneous nodule over the elbow,
which was tender and rubbery to the touch. The examination was also notable for a
subcutaneous nodule at the left metatarsal-phalangeal joint and left metacarpal-
phalangeal joint, as well as evidence of arthritis involving both hands. Which of the
following is characteristic of the arthritis?
A) Depositions of monosodium urate monohydrate crystals within the periarticular soft
tissues
B) Overgrowth of bones within joints
C) Autoimmune synovitis
D) Osteoporosis
E) Rheumatoid nodules over joints
ANSWER: A
A 55-year-old man presented to the clinic with a nodule on his elbow. He reported that
the nodule had been bothering him for 3 years and had steadily increased in size
recently. Further questioning revealed a history of repeated attacks of acute joint
pain. The physical examination revealed a rounded, subcutaneous nodule over the elbow,
which was tender and rubbery to the touch. The examination was also notable for a
subcutaneous nodule at the left metatarsal-phalangeal joint and left metacarpal-
phalangeal joint, as well as evidence of arthritis involving both hands. Which of the
following changes will reveal X-ray examination?
A) Erosions
B) Subluxations
C) Osteophytes
D) Osteoporosis
E) "punched-out" lytic lesion
ANSWER: E
A 55-year-old man undergoes intravenous pyelography (IVP) as part of a workup for
hypertension. A 3-cm solitary radiolucent mass is noted in the left kidney; the study
otherwise is normal. The man complains of no symptoms referable to the urinary tract,
and examination of urinary sediment is within normal limits. Which of the following
studies should be performed next?
A) Repeat intravenous pyelography in 6 months
B) Early-morning urine collections for cytology (three samples)
C) Selective renal arteriography
D) Renal ultrasonography
E) CT scanning (with contrast enhancement) of the left kidney
ANSWER: D
A 55-year-old patient presents to you with a history of having recently had a
myocardial infarction with a 5-day hospital stay while away on a business trip. He
reports being told he had mild congestive heart failure then, but is asymptomatic now
with normal physical exam. You recommend which of the following medications?
A) An ACE inhibitor
B) Digoxin
C) Diltiazem
D) Furosemide (Lasix)
E) Hydralazine plus nitrates
ANSWER: A
A 55-year-old patient presents to you with a history of having recently had a
myocardial infarction with a 5-day hospital stay while away on a business trip. He
reports being told he had mild congestive heart failure then, but is asymptomatic now
with normal physical exam. You recommend which of the following medications?
A) An ACE inhibitor
B) Digoxin
C) Diltiazem
D) Furosemide (Lasix)
E) Hydralazine plus nitrates
ANSWER: A
A 55-year-old patient presents to you with a history of having recently had a
myocardial infarction with a 5-day hospital stay while away on a business trip. He
reports being told he had mild congestive heart failure then, but is asymptomatic now
with normal physical exam. You recommend which of the following medications?
A) An ACE inhibitor
B) Digoxin
C) Diltiazem
D) Furosemide (Lasix)
E) Hydralazine plus nitrates
ANSWER: A
A 56-year-old male on hydralazine, hydrochlorothiazide, lisinopril, and metoprolol
begins to develop a malar rash and arthralgias. Which of the above antihypertensive
agents is known to cause drug-induced lupus?
A) Hydrochlorothiazide
B) Lisinopril
C) Hydralazine
D) Metoprolol
E) None of the above
ANSWER: C
A 56-year-old male on hydralazine, hydrochlorothiazide, lisinopril, and metoprolol
begins to develop a malar rash and arthralgias. Which of the above antihypertensive
agents is known to cause drug-induced lupus?
A) Hydrochlorothiazide
B) Lisinopril
C) Hydralazine
D) Metoprolol
E) None of the above
ANSWER: C
A 56-year-old male on hydralazine, hydrochlorothiazide, lisinopril, and metoprolol
begins to develop a malar rash and arthralgias. Which of the above antihypertensive
agents is known to cause drug-induced lupus?
A) Hydrochlorothiazide
B) Lisinopril
C) Hydralazine
D) Metoprolol
E) None of the above
ANSWER: C
A 57-year-old man is admitted to the CCU with an acute inferior myocardial infarction.
Two hours after admission, his blood pressure is 86/52 mmHg; his heart rate is 40
beats per minute with sinus rhythm. Which of the following would be the most
appropriate initial therapy?
A) Immediate insertion of a temporary transvenous pacemaker
B) Intravenous administration of atropine sulfate, 0.6 mg
C) Administration of normal saline, 300 mL over 15 min
D) Intravenous administration of dobutamine, 0.35 mg/min
E) Intravenous administration of isoproterenol, 5.0 mcg/min
ANSWER: B
A 57-year-old man is admitted to the CCU with an acute inferior myocardial infarction.
Two hours after admission, his blood pressure is 86/52 mmHg; his heart rate is 40
beats per minute with sinus rhythm. Which of the following would be the most
appropriate initial therapy?
A) Immediate insertion of a temporary transvenous pacemaker
B) Intravenous administration of atropine sulfate, 0.6 mg
C) Administration of normal saline, 300 mL over 15 min
D) Intravenous administration of dobutamine, 0.35 mg/min
E) Intravenous administration of isoproterenol, 5.0 mcg/min
ANSWER: B
A 57-year-old man presented to the clinic with a nodule on his elbow. He reported that
the nodule had been bothering him for 2 years and had steadily increased in size
recently. Further questioning revealed a history of repeated attacks of acute joint
pain. The physical examination revealed a rounded, subcutaneous nodule over the elbow,
which was tender and rubbery to the touch. The examination was also notable for a
subcutaneous nodule at the left metatarsal-phalangeal joint and left metacarpal-
phalangeal joint, as well as evidence of arthritis involving both hands. Which of the
following changes will reveal X-ray examination?
A) Erosions
B) Subluxations
C) Osteophytes
D) Osteoporosis
E) "punched-out" lytic lesion
ANSWER: E
A 57-year-old woman with diabetes and hypertension comes to the office because of a 5-
day history of weakness and lethargy. The symptoms developed slowly but have worsened
over the last 2 days. She denies any chest pain, dyspnea, fever, cough, abdominal pain,
or dysuria, but has noticed a decrease in her urine output. One week prior to
admission she underwent a CT scan of the abdomen with intravenous contrast for routine
follow up of an abdominal aortic aneurysm, which is stable at 4 cm. She regularly
takes hydrochlorothiazide, glyburide, captopril, and aspirin. Her temperature is 37.0
C (98.6 F), blood pressure is 165/94 mm Hg, pulse is 92/min, and respirations are
14/min. Physical examination shows 2+ pitting edema in the lower extremities and 1+
peripheral pulses. Urinalysis shows trace protein, 1-3 white blood cells, tubular
casts and no red casts. What complication has developed?
A) contrast-induced nephropathy
B) Acute glomerulonephritis
C) Acute pyelonephritis
D) Acute renal failure
E) Progressing of chronic renal failure
ANSWER: A
A 58 year old male presents with severe lower back pain, lethargy, anorexia and
peripheral oedema. He is found to be hypercalcaemic and have renal impairment and
nephrotic syndrome. A renal biopsy reveals deposits which after staining with congo
red appear as apple green birefringence under polarised light. There is is evidence of
a paraprotein band on serum electrophoresis. What is the most likely diagnosis?
A) AL amyloidosis secondary to multiple myeloma
B) Primary amyloidosis
C) MGUS
D) Nephrotic syndrome
E) None of them
ANSWER: A
A 58-year-old black woman presents for routine follow-up of diabetes mellitus and
hypertension. She feels well but states that she stopped taking verapamil because of
constipation. Current medications include glipizide, pravastatin, and aspirin;
evidence of drug intolerance includes angiotensin-converting enzyme inhibitor cough.
On examination, blood pressure is 156/92 mm Hg seated and standing. Except for the
patient’s findings for background diabetic retinopathy, the remainder of the
examination is normal. Recent laboratory values are a serum creatinine concentration
of 1.6 mg/dL, 24-hour urinary protein excretion of 1.5 g/d, and creatinine clearance
of 45 mL/min. On the basis of recent evidence, what is the most efficacious therapy to
slow the progression of the patient’s type 2 diabetic nephropathy?
A) Angiotensin-converting enzyme inhibitor
B) Angiotensin receptor blocker
C) Dihydropyridine calcium antagonist
D) ?-Blocker
E) none of them
ANSWER: B
A 58-year-old black woman presents for routine follow-up of diabetes mellitus and
hypertension. She feels well but states that she stopped taking verapamil because of
constipation. Current medications include glipizide, pravastatin, and aspirin;
evidence of drug intolerance includes angiotensin-converting enzyme inhibitor cough.
On examination, blood pressure is 156/92 mm Hg seated and standing. Except for the
patient’s findings for background diabetic retinopathy, the remainder of the
examination is normal. Recent laboratory values are a serum creatinine concentration
of 1.6 mg/dL, 24-hour urinary protein excretion of 1.5 g/d, and creatinine clearance
of 45 mL/min. What group of medication can prevent futher development of diabetic
nephropathy in this patient?
A) Perindopril
B) Telmisartan
C) Diltiazem
D) Doxasosin
E) Furosemide
ANSWER: B
A 58-year-old female smoker admitted to the ICU with respiratory distress due to
pneumonia. This was complicated by an anterior MI, with management including cautious
use of beta-blockers. She now develops 10 to 12 PVC’s per hour, occasional couplets,
and a few short runs of ventricular tachycardia, although BP and oxygen saturation
remain stable. Choose the best next step in antiarrhythmic management.
A) Amiodarone
B) Atropine
C) Digoxin
D) Observation
E) Quinidine
ANSWER: A
A 59-year-old man presents to the emergency department with a 3-day history of
worsening weakness, decreased mental acuity and responsiveness, and slurred speech.
The patient had been experiencing worsening weakness over the past 6 months. The
patients medical history includes bipolar disorder (diagnosed 10 years ago) and
hypothyroidism (diagnosed 5 years ago). His medications are lithium, 300 mg/d, and
levothyrroxine, 150 ?g/d. The patient is disoriented and lethargic, with slurred
speech and periods of agitation. A fine tremor and hyperreflexia are present. On
physical examination, supine blood pressure is 148/79 mm Hg, pulse rate 101/min,
respiratory rate 16/min, temperature 37.7 °C (99.9 °F). While he is standing, his
blood pressure is 142/80 mm Hg and heart rate is 108/min. The mucous membranes are dry,
and the neck veins are flat. Cardiac, pulmonary, and abdominal examinations are normal.
No lower extremity edema is noted. Laboratory studies: Blood urea nitrogen 8,2 mmol/L.
Serum creatinine 9.2 mmol/L. Serum sodium 162 mmol/L. Serum potassium 6.7 mmol/L.
Serum chloride 131 mmol/L. Serum bicarbonate 17.2 mmol/L. Serum calcium 10.7 mg/dL.
Serum albumin 45 g/L. Serum lithium 4.5 meq/L. Serum thyroid-stimulating hormone
<0.08 ?U/mL. Urinalysis pH 7.5; specific gravity 1 .007; 1 + proteinuria, no hematuria.
Urine microscopy occasional cellular casts; 2 leukocytes/hpf. Urine output 4775 mL/24
h. Electrocardiography shows normal sinus rhythm and peaked T waves. Renal
ultrasonography reveals normal echogenicity bilaterally, with a left kidney size of
9.9 cm and a right kidney size of 8.7 cm; no cyst, mass, or hydronephrosis is
visualized. What is the next step most appropriate in the management of this patient?
A) Begin intravenous infusion of normal saline for volume repletion
B) Administer 1 ampule of dextrose and 10 U of insulin intravenously for hyperkalemia
C) Transfer to the intensive care unit and perform emergent peritoneal dialysis
D) Begin intravenous infusion of half-normal saline followed by 80 mg of furosemide
intravenously for hypercalcemia
E) Transfer to the intensive care unit and perform emergent hemodialysis
ANSWER: E
A 59-year-old white male computer analyst is referred for evaluation of hypertension
(185/95 mm Hg) discovered during a blood pressure screening at his workplace. The
patient states that he is well and has not seen a physician in many years. He
describes himself as “a fitness freak, as he is an active jogger, abstains from
alcohol, and limits his salt and fat intake. He denies any knowledge of hypertension,
cardiovascular disease, renal disease, or diabetes mellitus. He takes no medications
regularly. Family history is significant in that his father was known to be
hypertensive and died of a stroke at 64 years of age. His older brother is being
treated for hypertension. On examination, the patient appears well, with a blood
pressure of 174/98 mm Hg while seated and standing. Body weight is 71 kg (157 Ib), and
height is 178 cm (70). Optic fundus examination is significant for grade II
hypertensive retinopathy. The remainder of the examination is normal. Complete blood
count, electrolyte panel, blood urea nitrogen level, creatinine concentration,
thyroid-stimulating hormone level, and results of urinalysis are normal.
Electrocardiography demonstrates normal sinus rhythm with left ventricular hypertrophy.
To reduce the patient’s cardiovascular morbidity and mortality, which therapy would
you prescribe?
A) Doxazosin
B) Losartan
C) Atenolol
D) Hydralazine
E) Niphedipin
ANSWER: B
A 59-year-old woman with CHF and an EF of 30% comes to your office for follow-up. She
is on carvedilol, enalapril, aspirin, atorvastatin calcium, digoxin, and furosemide.
She has been doing well without any rehospitalization. However, she wants to improve
her exercise tolerance. What should you recommend?
A) Cardiac transplantation
B) I.V. dobutamine
C) Higher doses of ACE inhibitor
D) Adding spironolactone
E) Enrolling her in an exercise training program
ANSWER: E
A 59-year-old woman with CHF and an EF of 30% comes to your office for follow-up. She
is on carvedilol, enalapril, aspirin, atorvastatin calcium, digoxin, and furosemide.
She has been doing well without any rehospitalization. However, she wants to improve
her exercise tolerance. What should you recommend?
A) Cardiac transplantation
B) I.V. dobutamine
C) Higher doses of ACE inhibitor
D) Adding spironolactone
E) Enrolling her in an exercise training program
ANSWER: E
A 60-year old white man consults you with a headache. Examination is unremarkable,
except that the blood pressure is raised and subsequent measurements confirm readings
of 170/106 mmHg. He is overweight, smokes 15 cigarettes per day and drinks 2 pints of
beer per day. His investigations reveal a fasting cholesterol of 6 mmol/l with a
normal blood sugar and electrolyte profile and normal renal function. There is no
evidence of target organ damage.
A) This man needs immediate treatment with antihypertensive drugs, aspirin and a
statin
B) Initiate antihypertensive therapy with two-drug combination
C) At this age it is far more important to control the hyperlipidaemia than the BP
D) This man’s blood pressure could be wholly ascribed to his alcohol intake
E) This patient can be observed and counseled on maintaining a reasonable BMI
ANSWER: B
A 60-year old white man consults you with a headache. Examination is unremarkable,
except that the blood pressure is raised and subsequent measurements confirm readings
of 170/106 mmHg. He is overweight, smokes 15 cigarettes per day and drinks 2 pints of
beer per day. His investigations reveal a fasting cholesterol of 6 mmol/l with a
normal blood sugar and electrolyte profile and normal renal function. There is no
evidence of target organ damage.
A) This man needs immediate treatment with antihypertensive drugs, aspirin and a
statin
B) Initiate antihypertensive therapy with two-drug combination
C) At this age it is far more important to control the hyperlipidaemia than the BP
D) This man’s blood pressure could be wholly ascribed to his alcohol intake
E) This patient can be observed and counseled on maintaining a reasonable BMI
ANSWER: B
A 60-year old white man consults you with a headache. Examination is unremarkable,
except that the blood pressure is raised and subsequent measurements confirm readings
of 170/106 mmHg. He is overweight, smokes 15 cigarettes per day and drinks 2 pints of
beer per day. His investigations reveal a fasting cholesterol of 6 mmol/l with a
normal blood sugar and electrolyte profile and normal renal function. There is no
evidence of target organ damage.
A) This man needs immediate treatment with antihypertensive drugs, aspirin and a
statin
B) Initiate antihypertensive therapy with two-drug combination
C) At this age it is far more important to control the hyperlipidaemia than the BP
D) This man’s blood pressure could be wholly ascribed to his alcohol intake
E) This patient can be observed and counseled on maintaining a reasonable BMI
ANSWER: B
A 60-year old white man consults you with a headache. Examination is unremarkable,
except that the blood pressure is raised and subsequent measurements confirm readings
of 170/106 mmHg. He is overweight, smokes 15 cigarettes per day and drinks 2 pints of
beer per day. His investigations reveal a fasting cholesterol of 6 mmol/l with a
normal blood sugar and electrolyte profile and normal renal function. There is no
evidence of target organ damage.
A) This man needs immediate treatment with antihypertensive drugs, aspirin and a
statin
B) Initiate antihypertensive therapy with two-drug combination
C) At this age it is far more important to control the hyperlipidaemia than the BP
D) This man’s blood pressure could be wholly ascribed to his alcohol intake
E) This patient can be observed and counseled on maintaining a reasonable BMI
ANSWER: B
A 60-year old white man consults you with a headache. Examination is unremarkable,
except that the blood pressure is raised and subsequent measurements confirm readings
of 170/106 mmHg. He is overweight, smokes 15 cigarettes per day and drinks 2 pints of
beer per day. His investigations reveal a fasting cholesterol of 6 mmol/l with a
normal blood sugar and electrolyte profile and normal renal function. There is no
evidence of target organ damage.
A) This man needs immediate treatment with antihypertensive drugs, aspirin and a
statin
B) Initiate antihypertensive therapy with two-drug combination
C) At this age it is far more important to control the hyperlipidaemia than the BP
D) This man’s blood pressure could be wholly ascribed to his alcohol intake
E) This patient can be observed and counseled on maintaining a reasonable BMI
ANSWER: B
A 60-year old white man consults you with a headache. Examination is unremarkable,
except that the blood pressure is raised and subsequent measurements confirm readings
of 170/106 mmHg. He is overweight, smokes 15 cigarettes per day and drinks 2 pints of
beer per day. His investigations reveal a fasting cholesterol of 6 mmol/l with a
normal blood sugar and electrolyte profile and normal renal function. There is no
evidence of target organ damage.
A) This man needs immediate treatment with antihypertensive drugs, aspirin and a
statin
B) Initiate antihypertensive therapy with two-drug combination
C) At this age it is far more important to control the hyperlipidaemia than the BP
D) This man’s blood pressure could be wholly ascribed to his alcohol intake
E) This patient can be observed and counseled on maintaining a reasonable BMI
ANSWER: B
A 60-year-old male patient on aspirin, nitrates, and a beta blocker, being followed
for chronic stable angina, presents to the ER with a history of two to three episodes
of more severe and long-lasting anginal chest pain each day over the past 3 days. His
ECG and cardiac enzymes are normal. The best course of action of the following is to
A) Admit the patient and begin intravenous digoxin.
B) Admit the patient and begin intravenous heparin
C) Admit the patient and give prophylactic thrombolytic therapy
D) Admit the patient for observation with no change in medication
E) Discharge the patient from the ER with increases in nitrates and beta blockers
ANSWER: B
A 60-year-old male patient on aspirin, nitrates, and a beta blocker, being followed
for chronic stable angina, presents to the ER with a history of two to three episodes
of more severe and long-lasting anginal chest pain each day over the past 3 days. His
ECG and cardiac enzymes are normal. The best course of action of the following is to
A) Admit the patient and begin intravenous digoxin.
B) Admit the patient and begin intravenous heparin
C) Admit the patient and give prophylactic thrombolytic therapy
D) Admit the patient for observation with no change in medication
E) Discharge the patient from the ER with increases in nitrates and beta blockers
ANSWER: B
A 60-year-old male patient on aspirin, nitrates, and a beta blocker, being followed
for chronic stable angina, presents to the ER with a history of two to three episodes
of more severe and long-lasting anginal chest pain each day over the past 3 days. His
ECG and cardiac enzymes are normal. The best course of action of the following is to
A) Admit the patient and begin intravenous digoxin.
B) Admit the patient and begin intravenous heparin
C) Admit the patient and give prophylactic thrombolytic therapy
D) Admit the patient for observation with no change in medication
E) Discharge the patient from the ER with increases in nitrates and beta blockers
ANSWER: B
A 60-year-old white female presents with epigastric pain, nausea and vomiting, heart
rate of 50, and pronounced first-degree AV block on ER cardiac monitor. Blood pressure
is 130/80. The coronary artery most likely to be involved in this process. It is the…
A) Right coronary
B) Left anterior descending
C) Circumflex
D) Left coronary
E) Left main
ANSWER: A
A 60-year-old white female presents with epigastric pain, nausea and vomiting, heart
rate of 50, and pronounced first-degree AV block on ER cardiac monitor. Blood pressure
is 130/80. The coronary artery most likely to be involved in this process is the
A) Right coronary
B) Left main
C) Left anterior descending
D) Circumflex
E) Left coronary
ANSWER: A
A 60-year-old woman with adult polycystic kidney disease is seen urgently in the
office for high fever. The illness started abruptly and involves chills and dysuria.
She has had hypertension for the past 5 years, treated with quinapril and
hydrochlorothiazide. She has lost approximately 8 kg (17 Ib) of weight over the last 3
months. On physical examination, the patient appears thin and frail. Body weight is 48
kg (106 Ib). Blood pressure is 90/70 mm Hg, pulse rate is 110/min, respiratory rate is
24/min, and body temperature is 39 °C (102 °F). The kidneys are palpable bilaterally,
and she has right costovertebral angle tenderness. Serum creatinine concentration is
1.1 mg/dL, and urinalysis shows pyuria and bacteriuria. The patient is admitted and
prescribed intravenous ampicillin and gentamicin to treat pyelonephritis. Why does the
dosage of antibiotic need to be adjusted in this patient?
A) The infection is in a cyst
B) The glomerular filtration rate is reduced
C) The patient is septic
D) The patient has hypertension
E) none of them
ANSWER: B
A 60-yr-old alcoholic man presents with a hot swollen first metatarsophalangeal joint
and a lesion on the rim of the left pinna. Choose the investigation for diagnosis from
the below list of options.
A) HLA-B27
B) Synovial fluid analysis
C) X-ray
D) Anti-dsDNA antibody
E) Rheumatoid factor
ANSWER: B
A 62-year old woman, current smoker, diabetic, hypertensive and hyperlipidemic,
presented to the ER with a 2 day history of intermittent chest pain. The patient does
not have any signs of heart failure. An ECG showed 2-mm depression in V2-V6. The
coronary angiogram showed 90% stenosis in the proximal third of the LAD, 80% stenosis
in the proximal RCA, with 30% disease in the middle circumflex artery. The best plan
of treatment for this patient is
A) PCI to both the LAD and RCA now
B) Referral for two-vessel CABG
C) PCI to the LAD with staged intervention to the RCA
D) Medical management with no PCI or CABG
E) Thrombolysis is recommended
ANSWER: B
A 62-year old woman, current smoker, diabetic, hypertensive and hyperlipidemic,
presented to the ER with a 2 day history of intermittent chest pain. The patient does
not have any signs of heart failure. An ECG showed 2-mm depression in V2-V6. The
coronary angiogram showed 90% stenosis in the proximal third of the LAD, 80% stenosis
in the proximal RCA, with 30% disease in the middle circumflex artery. The best plan
of treatment for this patient is
A) PCI to both the LAD and RCA now
B) Referral for two-vessel CABG
C) PCI to the LAD with staged intervention to the RCA
D) Medical management with no PCI or CABG
E) Thrombolysis is recommended
ANSWER: B
A 64-year-old woman presents with a 1-year history of pain in her thumbs. A focused
examination reveals squaring at the base of both first digits, worse on the right, and
pain on pressure over the first carpal metacarpal joints. She also has non tender bony
overgrowth at the distal interphalangeal joints. The patient says that her mother had
the same fingers and she worries that she will become crippled. Choose the most likely
diagnosis from the below list of options.
A) Ankylosing spondylitis
B) Rheumatoid arthritis
C) Osteoarthritis
D) Gout
E) Psoriatic arthritis
ANSWER: C
A 64-year-old woman presents with a 1-year history of pain in her thumbs. A focused
examination reveals squaring at the base of both first digits, worse on the right, and
pain on pressure over the first carpal metacarpal joints. She also has nontender bony
overgrowth at the distal interphalangeal joints. On physical examination, there is
crepitus on passive motion of the left knee, which has a slight varus deformity. Her
feet turn out slightly, and the heels are in valgus. Which of the following changes
will reveal X-ray examination?
A) Erosions
B) Subluxations
C) Osteophytes
D) Osteoporosis
E) Ankylosis
ANSWER: C
A 64-year-old woman presents with a 1-year history of pain in her thumbs. A focused
examination reveals squaring at the base of both first digits, worse on the right, and
pain on pressure over the first carpal metacarpal joints. She also has nontender bony
overgrowth at the distal interphalangeal joints. The patient says that her mother had
“the same fingers” and she worries that she will become crippled. Choose the most
likely diagnosis from the below list of options.
A) Ankylosing spondylitis
B) Rheumatoid arthritis
C) Osteoarthritis
D) Gout
E) Psoriatic arthritis
ANSWER: C
A 65-year old woman developed cardiogenic shock 10 hrs after presenting with anterior
wall MI. The most appropriate management strategy is....
A) Administration of thrombolytics
B) Watchful waiting after initiation of inotropic support and insertion of an intra-
aortic balloon pump
C) Coronary angiography with revascularization within 18 hrs of shock onset
D) Half-dose fibrinolysis and GP IIb/IIIa
E) Half-dose thrombolysis should be recommended; then the patient should be
transferred for PCI.
ANSWER: C
A 65-year old woman developed cardiogenic shock 10 hrs after presenting with anterior
wall MI. The most appropriate management strategy is....
A) Administration of thrombolytics
B) Watchful waiting after initiation of inotropic support and insertion of an intra-
aortic balloon pump
C) Coronary angiography with revascularization within 18 hrs of shock onset
D) Half-dose fibrinolysis and GP IIb/IIIa
E) Half-dose thrombolysis should be recommended; then the patient should be
transferred for PCI.
ANSWER: C
A 65-year-old woman presents with a 1-year history of pain in her thumbs. A focused
examination reveals squaring at the base of both first digits, worse on the right, and
pain on pressure over the first carpal metacarpal joints. She also has nontender bony
overgrowth at the distal interphalangeal joints. On physical examination, there is
crepitus on passive motion of the left knee, which has a slight varus (“bow leg”)
deformity. Her feet turn out slightly, and the heels are in valgus. Which of the
following changes will reveal X-ray examination?
A) Erosions
B) Subluxations
C) Osteophytes
D) Osteoporosis
E) Ankylosis
ANSWER: C
A 65-year-old woman with a 30-year history of rheumatoid arthritis presented with a 6-
week history of progressive ankle oedema. She had been treated with regular gold
injections for 5 years, but these had been stopped 9 months previously. She had also
been taking diclofenac for the past 2 years. On examination, she had pitting oedema to
her knees and a sacral pad. Investigations: haemoglobin 106 g/L (115–165), platelet
count 164x109/L (150–400), serum sodium 143 mmol/L (137–144), serum potassium 4.4
mmol/L (3.5–4.9), serum creatinine 223 umol/L (60–110), serum albumin 19 g/L (37–49),
24-h urinary total protein 7.8 g (<0.2) What is the most likely diagnosis?
A) crescentic glomerulonephritis
B) gold-induced membranous nephropathy
C) interstitial nephritis
D) minimal change nephropathy
E) renal amyloidosis
ANSWER: E
A 68 year old male diagnosed with nephrotic syndrome receives steroid therapy without
benefit. His investigations show an albumin of 20 g/L (37 - 49), Total cholesterol of
12 mmol/l, dipstick urinanalysis reveals +++ protein and a renal biopsy shows focal
segmental glomerulosclerosis. Which one of the following medications is most likely to
preserve renal function?
A) dietary salt restriction
B) low dietary protein intake
C) ramipril
D) simvastatin
E) warfarin
ANSWER: C
A 68-year-old man is evaluated because of a 3-year history of bilateral knee pain and
low back pain. He has some stiffness for approximately 15 minutes when she awakens in
the morning, and during the afternoon her pain is worse. On physical examination, he
has slight swelling and tenderness to pressure of the distal interphalangeal joints 2-
5 on both hands. There is slight crepitus with motion of the right knee. Which of the
following treatment do use?
A) Methotrexat
B) Glucosamine
C) Colchicines
D) Tetracycline
E) Allopurinol
ANSWER: B
A 68-year-old woman has had four or five episodes of joint pain and swelling, lasting
3 to 8 days, involving the right knee and left elbow. She is asymptomatic between
attacks, and sulindac, 200 mg twice daily, has usually relieved the symptoms. Her most
recent episode was 4 months ago. On physical examination, none of her joints is
swollen or tender, but there is marked crepitus on extension of the knee. She also has
a positive bulge sign over the left knee and pain on full extension of the left elbow.
Which one of the following tests would confirm the diagnosis?
A) Arthrocentesis of the knee and laboratory analysis of the synovial fluid
B) Measurement of serum uric acid
C) Measurement of serum rheumatoid factor
D) Radiograph of the knee
E) MRI of the knee with gadolinium contrast
ANSWER: A
A 70-year-old retired banker with no past medical history presents to the emergency
department 4 h after the onset of severe substernal crushing chest pain with radiation
to the left arm and neck. Electrocardiography reveals significant ST-segment elevation
in leads I, L, V5, and V6. The patient has no clear-cut medical contraindications to
anticoagulation. Which of the following would be the optimal management strategy at
this time?
A) Intravenous tissue plasminogen activator alone
B) Intravenous tissue plasminogen activator and aspirin
C) Intravenous tissue plasminogen activator and heparin
D) Intravenous tissue plasminogen activator, heparin, and aspirin
E) Thrombolytic therapy is contraindicated because of the patient's age
ANSWER: D
A 70-year-old retired banker with no past medical history presents to the emergency
department 4 h after the onset of severe substernal crushing chest pain with radiation
to the left arm and neck. Electrocardiography reveals significant ST-segment elevation
in leads I, L, V5, and V6. The patient has no clear-cut medical contraindications to
anticoagulation. Which of the following would be the optimal management strategy at
this time?
A) Intravenous tissue plasminogen activator alone
B) Intravenous tissue plasminogen activator and aspirin
C) Intravenous tissue plasminogen activator and heparin
D) Intravenous tissue plasminogen activator, heparin, and aspirin
E) Thrombolytic therapy is contraindicated because of the patient's age
ANSWER: D
A 70-yr-old woman complains of arthritis in the fingers and big toe. On examination
she has bony swellings of the first carpometacarpal joint and the distal
interphalangeal joints and has an affected metatarsophalangeal joint. Choose the most
likely diagnosis from the below list of options.
A) Rheumatoid arthritis
B) Hyperparathyroidism
C) Psoriatic arthropathy
D) Osteoarthritis
E) Haemochromatosis
ANSWER: D
A 71-year-old female is admitted with acute myocardial infarction and congestive heart
failure, then has an episode of ventricular tachycardia. She is prescribed multiple
medications and soon develops confusion and slurred speech. The most likely cause of
this confusion is
A) Captopril
B) Digoxin
C) Furosemide
D) Lidocaine
E) Nitroglycerin
ANSWER: D
A 71-year-old woman is receiving parenteral methicillin for leg cellulitis. Over 2
days she develps macroscopic hematuria, oliguria, and marked deterioration in renal
functioning. Features suggestive of methicillin-induced acute interstitial nephritis
include all of the following EXCEPT
A) fever and arthralgias
B) eosinophils in the urine
C) RBC casts in the urine
D) return of renal functioning on discontinuing the drug
E) return of renal functioning after prednisone therapy
ANSWER: C
A 71-year-old woman is receiving parenteral methicillin for leg cellulitis. Over 2
days she develps macroscopic hematuria, oliguria, and marked deterioration in renal
functioning. Features suggestive of methicillin-induced acute interstitial nephritis
include all of the following EXCEPT
A) fever and arthralgias
B) eosinophils in the urine
C) nephritic range of proteinuria
D) return of renal functioning on discontinuing the drug
E) return of renal functioning after prednisone therapy
ANSWER: C
A 72-year-old male with a 2-h history of chest pain, ST segment elevation in leads II,
III, aVF, and sinus bradycardia at a rate of 40 with hypotension. Choose the best next
step in antiarrhythmic management.
A) Amiodarone
B) Atropine
C) Diltiazem
D) Metoprolol
E) Observation
ANSWER: B
A 72-year-old male with a 2-h history of chest pain, ST segment elevation in leads II,
III, aVF, and sinus bradycardia at a rate of 40 with hypotension. Choose the best next
step in antiarrhythmic management.
A) Amiodarone
B) Atropine
C) Diltiazem
D) Metoprolol
E) Observation
ANSWER: B
A 72-year-old retired lawyer is evaluated because of swelling of his right knee. He
has mild osteoarthritis and stays active swimming and playing tennis and golf. On
physical examination, he has an effusion in the right knee. At a point in the mid-
thigh (measured from the superior border of the patella), the right thigh is 4 cm
smaller in diameter than the left. There is crepitus with right knee flexion. The knee
is not warm. Which of the following is the best next step in the management of this
patient?
A) Order radiographs of both knees and treat the patient with ibuprofen.
B) Order radiographs of the right hip and right knee.
C) Aspirate the joint fluid in the right knee and refer the patient for physical
therapy.
D) Aspirate the joint fluid in the right knee and obtain blood cultures.
E) Treat the patient with celecoxib, and advise him to stay off his feet for 48 hours.
ANSWER: C
A 73-yr-old fit farmer presents with pain on weight bearing and restricted movements
of the right hip. Choose the most likely X-ray changes from the below List of options.
A) Cystic changes and lytic lesions
B) Joint spaces are narrowed with reactive subchondral sclerosis, osteophytes
C) Periarticular osteopenia, marginal erosions
D) acroosteolysis
E) Fluffy bone periostitis and erosions at the interphalangeal joints
ANSWER: B
A 75-year old woman presents to a community hospital (without invasive capabilities)
with 6 hrs of chest pain and is diagnosed with inferior STEMI. The anticipated
transfer to a medical center with primary PCI capability will take more than 2 hours.
What is the preferred reperfusion therapy?
A) Fibrinolytic therapy at presentation to the hospital
B) Transfer for angiography and PCI
C) Half-dose fibrinolysis and GP IIb/IIIa inhibition at the presentation hospital and
transfer for rescue PCI if it fails to produce reperfusion
D) PCI at a later date
E) CABG
ANSWER: A
A 75-year old woman presents to a community hospital (without invasive capabilities)
with 6 hrs of chest pain and is diagnosed with inferior STEMI. The anticipated
transfer to a medical center with primary PCI capability will take 2 hours. What is
the preferred reperfusion therapy?
A) Fibrinolytic therapy at presentation to the hospital
B) Transfer for angiography and PCI
C) Half-dose fibrinolysis and GP IIb/IIIa inhibition at the presentation hospital and
transfer for rescue PCI if it fails to produce reperfusion
D) PCI at a later date
E) CABG
ANSWER: A
A 75-year old woman presents to a community hospital (without invasive capabilities)
with 6 hrs of chest pain and is diagnosed with inferior STEMI. The anticipated
transfer to a medical center with primary PCI capability will take 2 hours. What is
the preferred reperfusion therapy?
A) Fibrinolytic therapy at presentation to the hospital
B) Transfer for angiography and PCI
C) Half-dose fibrinolysis and GP IIb/IIIa inhibition at the presentation hospital and
transfer for rescue PCI if it fails to produce reperfusion
D) PCI at a later date
E) CABG
ANSWER: A
A 75-year-old female is admitted with acute myocardial infarction and congestive heart
failure, then has an episode of ventricular tachycardia. She is prescribed multiple
medications and soon develops confusion and slurred speech. The most likely cause of
this confusion is
A) Captopril
B) Digoxin Furosemide
C) Lidocaine
D) Nitroglycerin
ANSWER: C
A 77-year old woman with a history of GERD presents in the first hour of an acute
inferior STEMI. The BP is normal. Invasive facilities are not readily available. What
is the preferred strategy for reperfusion therapy?
A) Fibrinolytic therapy
B) Transfer for immediate coronary angiography and PCI
C) Half-dose fibrinolysis and GP IIb/IIIa
D) Symptomatic relief of angina only
E) CABG
ANSWER: A
A 77-yr-old woman with long-standing rheumatoid arthritis presented with fever and
dysuria. Her past history included recurrent chest and urinary infections. On
examination she was hyperpigmented and emaciated. Her hands and feet were severely
deformed. Abdominal examination revealed splenomegaly but no hepatomegaly or
lymphadenopathy. Choose the most likely diagnosis from the below list of options.
A) Gout
B) Still's disease
C) Felty's syndrome
D) Rheumatoid arthritis
E) Osteoarthritis
ANSWER: C
A 78-year-old woman with CHF (EF, 25%), chronic AFib, GERD, HTN, hyperlipidemia,
diabetes, and osteoporosis takes 12 different pills. Recently her neighbour, a nurse
told her that she does not need to take digoxin because she is on amiodarone. She
wants to eliminate digoxin from her medication regimen, and she wants to know why you
put her on it in the first place. What is your answer?
A) Digoxin improves survival
B) Digoxin reduces hospitalizations
C) Digoxin improves contractility
D) Digoxin decreases the volume of distribution of amiodarone
E) Digoxin reduces sympathetic nervous system activity
ANSWER: C
A middle aged man presents with fever and variable heart murmur. He also complains of
fever, malaise and night sweats. On examination you find clubbing and splenomegaly.
Choose the single most likely diagnosis from the list of options below.
A) Infective endocarditis
B) Pericardial effusion
C) SLE
D) Bacteraemia
E) Acute myocarditis
ANSWER: A
A patient is found to be in acute renal failure with a potassium of 6.8 mmol/L. Her
ECG shows tall tented T waves. She is commenced on cardiac monitoring and is given
insulin dextrose infusion and salbutamol nebulisers. Which other treatment should be
given?
A) Calcium Gluconate
B) Further insulin dextrose
C) Repeat Potassium
D) Calcium resonium
E) Sodium bicarbonate
ANSWER: A
A previously healthy 58-year-old man is admitted to the hospital because of an acute
inferior myocardial infarction. Within several hours, he becomes oliguric and
hypotensive (blood pressure is 90/60 mmHg). Insertion of a pulmonary artery (Swan-Ganz)
catheter reveals the following pressures: pulmonary capillary wedge, 4 mmHg; pulmonary
artery, 22/4 mmHg; and mean right atrial, 11 mmHg. This man would best be treated with
A) fluids
B) digoxin
C) norepinephrine
D) dopamine
E) intraaortic balloon counterpulsation
ANSWER: A
A previously healthy 58-year-old man is admitted to the hospital because of an acute
inferior myocardial infarction. Within several hours, he becomes oliguric and
hypotensive (blood pressure is 90/60 mmHg). Insertion of a pulmonary artery (Swan-Ganz)
catheter reveals the following pressures: pulmonary capillary wedge, 4 mmHg; pulmonary
artery, 22/4 mmHg; and mean right atrial, 11 mmHg. This man would best be treated with
A) fluids
B) Digoxin
C) Dopamine
D) intraaortic balloon counterpulsation
E) Norepinephrine
ANSWER: A
A seventeen-year-old girl had a throat infection three weeks ago. Since then she has
developed a fever and an acute migratory polyarthritis involving the knees, ankles and
wrists. Her parents have noticed her legs twitching. She has a transient erythematous
rash, with ring and crescent shaped patches. Which is the SINGLE MOST likely diagnosis?
A) Rheumatic fever
B) Allergic polyarthritis
C) Juvenile chronic arthritis
D) Lyme disease
E) Parvovirus
ANSWER: A
An 18-year-old white man is seen in your office because of blood in urine dipstick
analysis during a sports physical. He has no history of gross hematuria, recent fever
or chills, or upper respiratory illness. He has been in good health. He takes
cetirizine for seasonal allergies. He has no family history of renal disease. Review
of systems is negative. On examination, blood pressure is 140/98 mm Hg, weight is 80
kg (176 Ib), and height is 170 cm (67’). He has a muscular build. The remainder of the
physical examination is normal. Repeated urinalysis at2 p.m. reveals a pH of 5.0,
specific gravity of 1.015,2+ blood, 2+ protein. Which test is the best option to
assist in diagnosis?
A) Renal computed tomography
B) Renal ultrasonography
C) Cystoscopy
D) Magnetic resonance angiography of renal arteries
E) Daily proteinuria
ANSWER: E
An 80-year old woman with chest discomfort for the last 15 hours and progressively
worsening dyspnea over the last few hours presents to the Emergency Department and is
diagnosed with anterior STEMI. What, if any, reperfusion strategy should be undertaken?
A) Fibrinolysis
B) Immediate coronary angiography and PCI
C) Symptomatic relief of angina and heart failure
D) CABG
E) None of the above
ANSWER: C
An elderly man presents with a red, warm swollen metatarsophalangeal joint following a
right total hip replacement operation. Choose the most likely diagnosis from the below
list of options.
A) Ankylosing spondylitis
B) Reactive arthritis
C) Osteoarthritis
D) Acute Gouty arthritis
E) Psoriatic arthritis
ANSWER: D
An elderly man started frusemide 2-weeks ago and now presents with a red, hot, swollen
first metatarsal phalangeal joint. Choose the most likely diagnosis from the below
list of options.
A) Ankylosing spondylitis
B) Joint sepsis
C) Osteoarthritis
D) Gout
E) Psoriatic arthritis
ANSWER: D
An obese 54-yer-old man denies exertional chest discomfort or dyspnea, but lives a
fairly sedentary lifestyle. Physical exam reveals an obese man with normal BP, an
unremarkable cardiac exam. His fasting lipids are as follows: TC 240 mg/dL, TG 250
mg/dL, HDL-C 35 mg/dL, LDL-C (calculated)155. Initial therapy should include:
A) Therapeutic lifestyle correction (TLC) only
B) TLC plus a statin with a goal of reducing LDL-C to &amp;amp;lt;130 mg/dL
C) TLC plus niacine or fibrate, with the goals of reducing LDL-C to &amp;amp;lt;100
mg/dL and TG &amp;amp;lt;150 mg/dL
D) TLC plus a statin with a goal of reducing LDL-C to &amp;amp;lt;100 mg/dL
E) TLC plus a statin, as well as niacin or a fibrate, with the goals of reducing LDL-C
to &amp;amp;lt;100 mg/dL and TG &amp;amp;lt;150 mg/dL
ANSWER: A
An obese nonsmoking 44-year-old man with treated hypertension and no family history of
CHD. He has had elevated LDL-C in the past and is taking atorvastatin 20 mg daily. His
latest lipid panel is as follows: TC 220, HDL-C 40, LDL-C 120, TG 600. After
recommending lifestyle modifications, what would be your next step in lipid management?
A) Increase the HDL-H
B) Lower the LDL-C
C) Lower the TG
D) Lower the TG and increase the HDL-C
E) Lower the non-HDL-C
ANSWER: C
As a cardiologist you see a 55-year-old male with diabetes mellitus and a BP of 165/95
mmHg. According to the JNC 7 guidelines, he would be classified as what stage of
hypertension and what is his target BP measurement?
A) Prehypertension, 140/90 mmHg
B) Stage 2, 130/80 mmHg
C) Stage 1, 140/90 mmHg
D) Stage 2, 140/90 mmHg
E) Stage 3, 120/70 mmHg
ANSWER: B
As a cardiologist you see a 55-year-old male with diabetes mellitus and a BP of 165/95
mmHg. According to the JNC 7 guidelines, he would be classified as what stage of
hypertension and what is his target BP measurement?
A) Prehypertension, 140/90 mmHg
B) Stage 2, 130/80 mmHg
C) Stage 1, 140/90 mmHg
D) Stage 2, 140/90 mmHg
E) Stage 3, 120/70 mmHg
ANSWER: B
At a routine company physical examination, an asymptomatic 46-year-old man is found to
have a BP of 150/110 mm Hg, but no other abnormalities are present. What
investigations can confirm renal arterial hypertension?
A) urinalisis, renal ultrasound, ECG
B) Nechiporenko test, renal ultrasound, biochemical blood test
C) urinalisis, ECG, biochemical blood test
D) urinalisis, renal ultrasound, biochemical blood test
E) urinalisis, CBC, biochemical blood test
ANSWER: D
At a routine examination, an asymptomatic 46-year-old man is found to have a BP of
150/110 mmHg, but no other abnormalities are present. What do you do next?
A) Reassure the patient and repeat the physical examination in 12 months
B) Initiate antihypertensive therapy
C) Obtain repeated BP recordings in your office and/or the patient’s home or work site
D) Hospitalize patient for renal arteriography
E) Order a 24-h ambulatory BP monitoring
ANSWER: C
At a routine examination, an asymptomatic 46-year-old man is found to have a BP of
150/110 mmHg, but no other abnormalities are present. What do you do next?
A) Reassure the patient and repeat the physical examination in 12 months
B) Initiate antihypertensive therapy
C) Obtain repeated BP recordings in your office and/or the patient’s home or work site
D) Hospitalize patient for renal arteriography
E) Order a 24-h ambulatory BP monitoring
ANSWER: C
At a routine examination, an asymptomatic 46-year-old man is found to have a BP of
150/110 mmHg, but no other abnormalities are present. What do you do next?
A) Reassure the patient and repeat the physical examination in 12 months
B) Initiate antihypertensive therapy
C) Obtain repeated BP recordings in your office and/or the patient’s home or work site
D) Hospitalize patient for renal arteriography
E) Order a 24-h ambulatory BP monitoring
ANSWER: C
Dyspnoea on exertion, abdominal pain and progressive numbness in his feet. He recently
developed mild polyarthritis in his hands. On examination there was evidence of left
median nerve mononeuritis. Chest radiography showed cardiomegaly. Choose the most
likely diagnosis from the below list of options.
A) Rheumatoid arthritis
B) Polymyalgia rheumatica
C) Polymyositis
D) Polyarteritis nodosa
E) Scleroderma
ANSWER: D
For the case below, select the most significant adverse effect of the antihypertensive
and/or cardiac agent in question: a 45-year old female has been on diuretic, but BP
remains elevated at 145/95, leading to the proposed addition of lisinopril. Which key
potential adverse effect should be discussed?
A) Increased triglyceride levels
B) Peripheral edema
C) Lupus-like syndrome
D) Cough
E) Gynecomastia
ANSWER: D
For the case below, select the most significant adverse effect of the antihypertensive
and/or cardiac agent in question: a 45-year old female has been on diuretic, but BP
remains elevated at 145/95, leading to the proposed addition of lisinopril. Which key
potential adverse effect should be discussed?
A) Increased triglyceride levels
B) Peripheral edema
C) Lupus-like syndrome
D) Cough
E) Gynecomastia
ANSWER: D
For the case below, select the most significant adverse effect of the antihypertensive
and/or cardiac agent in question: a 58-year old male truck driver has significant
hypertension, still not controlled on a diuretic plus calcium channel blocker.
Clonidine is being considered as the next medication, but in this patient the concerns
are sedation, sexual dysfunction, and this side effect.
A) Increased triglyceride levels
B) Cough
C) Gynecomastia
D) Rebound hypertension
E) Urinary retention
ANSWER: D
For the case below, select the most significant adverse effect of the antihypertensive
and/or cardiac agent in question: a 59-year old male truck driver has significant
hypertension, still not controlled on a diuretic plus calcium channel blocker.
Clonidine is being considered as the next medication, but in this patients the
concerns are sedation, sexual dysfunction, and ..... this side effect.
A) Increased triglyceride levels
B) Cough
C) Gynecomastia
D) Rebound hypertension
E) Urinary retention
ANSWER: D
For the case below, select the most significant adverse effect of the antihypertensive
and/or cardiac agent in question: a 68-year old male with advanced chronic heart
failure and BP 145/90 will have spironolactone added to his regimen, but should be
informed about .... this possible side effect.
A) Increased triglyceride levels
B) Peripheral edema
C) Lupus-like syndrome
D) Cough
E) Gynecomastia
ANSWER: E
For the case below, select the most significant adverse effect of the antihypertensive
and/or cardiac agent in question: a 45-year old female has been on diuretic, but BP
remains elevated at 145/95, leading to the proposed addition of lisinopril. Which key
potential adverse effect should be discussed?
A) Increased triglyceride levels
B) Peripheral edema
C) Lupus-like syndrome
D) Cough
E) Gynecomastia
ANSWER: D
For the case below, select the most significant adverse effect of the antihypertensive
and/or cardiac agent in question: a 68-year old male with advanced chronic heart
failure and BP 145/90 will have spironolactone added to his regimen, but should be
informed about .... this possible side effect.
A) Increased triglyceride levels
B) Peripheral edema
C) Lupus-like syndrome
D) Cough
E) Gynecomastia
ANSWER: E
G . is an obese nonsmoking 43-year-old man with treated hypertension and no family
history of CHD. He has had elevated LDL-C in the past and is taking atorvastatin 20 mg
daily. His latest lipid panel is as follows: TC -220, HDL-C 40, LDL-C 120, TG 450.
After recommending lifestyle modifications, which of the following would allow you to
achieve the goal of lowering the non-HDL-C?
A) Increase the dose of atorvastatin
B) Add a fibrate
C) Add niacin
D) Add ezetimibe
E) Any of the above
ANSWER: E
G . is an obese nonsmoking 44-year-old man with treated hypertension and no family
history of CHD. He has had elevated LDL-C in the past and is taking atorvastatin 20 mg
daily. His latest lipid panel is as follows: TC 220, HDL-C 40, LDL-C 120, TG 450.
After recommending lifestyle modifications, which of the following would allow you to
achieve the goal of lowering the non-HDL-C?
A) Increase the dose of atorvastatin
B) Add a fibrate
C) Add niacin
D) Add ezetimibe
E) Any of the above
ANSWER: E
H . is an obese nonsmoking 44-year-old man with treated hypertension and no family
history of CHD. He has had elevated LDL-C in the past and is taking atorvastatin 20 mg
daily. His latest lipid panel is as follows: TC 220, HDL-C 40, LDL-C 120, TG 450.
After recommending lifestyle modifications, what is your first goal of drug treatment?
A) Increase the HDL-H
B) Lower the LDL-C
C) Lower the TG
D) Lower the TG and increase the HDL-C
E) Lower the non-HDL-C
ANSWER: E
J . is an obese 54-yer-old man with peripheral arterial disease revealed by arterial
duplex study. He denies exertional chest discomfort of dyspnea, but lives a fairly
sedentary lifestyle. Physical exam reveals an obese man with normal BP, an
unremarkable cardiac exam. His fasting lipids are as follows: TC 240 mg/dL, TG 250
mg/dL, HDL-C 35 mg/dL, LDL-C (calculated)155. Initial therapy should include:
A) TLC (therapeutic lifestyle correction) only
B) TLC plus a statin with a goal of reducing LDL-C to &amp;amp;lt;100 mg/dL
C) TLC plus niacine or fibrate, with the goals of reducing LDL-C to &amp;amp;lt;100
mg/dL and TG &amp;amp;lt;150 mg/dL
D) TLC plus a statin with a goal of reducing LDL-C to &amp;amp;lt;130 mg/dL
E) TLC plus a statin, as well as niacin or a fibrate, with the goals of reducing LDL-C
to &amp;amp;lt;100 mg/dL and TG &amp;amp;lt;150 mg/dL
ANSWER: B
L . is a 71-year-old retired female whose father died at age 52 of a “massive MI”.
She is very worried about her own risk of a heart attack. She watches her weight
(BMI=23), does not smoke, and keeps physically fit. She denies angina or dyspnea on
exertion, claudication, or history of TIA symptoms. Her fasting lipids are as follows:
TC 250 mg/dL, TG 120 mg/dL, HDL-C 42 mg/dL, LDL-C (calculated)151. Initial therapy
should include:
A) Nothing beyond her current lifestyle measures
B) Weight loss to bring BMI&amp;amp;lt;20
C) TLC plus a statin to reduce LDL-C to &amp;amp;lt;130
D) TLC to reduce LDL-C to &amp;amp;lt;130
E) Statin plus a niacin to reduce LDL-C&amp;amp;lt;130 and increase HDL-
C&amp;amp;gt;50 mg/dL
ANSWER: D
On the third day of hospitalization, he developed acute, severe pain and swelling of
the left elbow. Within next few hours, pain worsened and he was unable to move the
elbow joint, which was tender, erythematous, and swollen on examination. Never
investigated in the past, we also noted a firm 4-6 cm mass on each elbow, and another
one surrounding the proximal inter-phalangeal joint of right middle finger. When asked
particularly, he noticed similar episodes of arthritis of great toe in the past.
Choose the most likely diagnosis from the below list of options.
A) Ankylosing spondylitis
B) Erythema nodosum
C) Osteoarthritis + synovitis
D) Exacerbation of chronic gouty arthritis
E) Psoriatic arthritis
ANSWER: D
On the third day of hospitalization, he developed acute, severe pain and swelling of
the left elbow. Within next few hours, pain worsened and he was unable to move the
elbow joint, which was tender, erythematous, and swollen on examination. Never
investigated in the past, we also noted a firm 4- 6 cm mass on each elbow, and another
one surrounding the proximal inter-phalangeal joint of right middle finger. When asked
particularly, he noticed similar episodes of arthritis of great toe in the past. Which
of the following medications is indicated for the treatment of the arthritis?
A) Allopurinol
B) Corticosteroids
C) Colchicine
D) Tetracycline
E) Mydocalmi
ANSWER: C
Patient 18 years old has respiratory viral infection with high fever of 38-39 degrees.
In labs: proteinuria - 0,066 g/l. In a week urinalysis has not discovered any
pathological changes. What is the probable reason of transient proteinuria:
A) Acute glomerulonephritis
B) Feverish proteinuria
C) Acute pyelonephritis
D) Acute tubulointerstitial nephritis
E) Diabetic nephropathy
ANSWER: B
Patient K., 55 years old, complains of absence of urination during 2 days, dyspnea at
rest, swollen abdomen, insomnia. Five days ago after the using of bully beef of home
preparation heavy diarrhea, vomiting, increase of body temperature to 380С have
appeared. During 3 days he did not go to the doctor, treated oneself with herbs, on
the fourth day he stopped to urinate. At the examination: respiration in lower lobes
is weakened, heart rate – 100 beats/min, BP - 160/100 mm Hg, there is free fluid in
abdominal cavity. Serum creatinine - 0,73 mmol/l, urea of blood – 39 mmol/l, serum
potassium - 6,4 mmol/l. Specify the condition of the patient:
A) Prerenal acute renal failure
B) Renal acute renal failure
C) Postrenal acute renal failure
D) Acute tubulointerstitial nephritis
E) Not enough information
ANSWER: A
Patient N., 69 years old, complains of osalgia, dicreasing of appetite, subfebrile
body temperature. In labs: normochromic anemia, ESR - 65 mm/h, proteinuria - 3,3 g/l,
total protein - 105 g/l. Reason of proteinuria of “overflow” is:
A) Increase of permeability of glomerular basal membrane
B) Inability of renal tubules to reabsorb protein
C) Increase of maintenance of low molecular weight protein in blood plasma
D) Increase of “pore” sizes of glomerular capillaries endothelium
E) Loss of negative charge of basal membrane
ANSWER: C
Patient T., 25 years old, complains of polyuria (2,5 l/day), malaise, mild flank pain.
In labs: increasing of serum creatinine (300 umol/l), proteinuria 0,99 g/l. Doctor has
suspected development of acute tubulointerstitial nephritis due to panadol that patien
took for respiratory viral infection. What should doctor do next?
A) Removal of panadol
B) Prescription of loratidin
C) Prescription of dimedrol
D) Prescription of plasmapheresis
E) Hemodialysis
ANSWER: A
Patient, 32 years old, complains on aching pain in loin on the right, increase of body
temperature to subfebrile level. In the anamnesis: there was pain in the right half of
loin with chill, acute increase of body temperature 10 years ago during pregnancy and
after births; there is an increase of arterial blood pressure during last 5 years. In
the urinalysis: proteinuria - 0,99 g/l, leucocytes - 30-40 in the visual field,
eruthrosytes - 2-4 in the visual field, serum creatinine - 102 umol/l. What is the
most reliable diagnosis:
A) Renal tuberculosis
B) Chronic glomerulonephritis
C) Chronic pyelonephritis
D) Renal amyloidosis
E) Hypertonic disease
ANSWER: C
Patient, 38 years old, complains on thirst, nausea, polyuria. For many years takes
analgetics because of migraine. In labs: moderate normochromic anemia, proteinuria -
0,33 g/l, serum creatinine – 300 umol/l. What is the most probable diagnosis?
A) Chronic glomerulonephritis
B) Chronic tubulointerstitial nephritis
C) Chronic pyelonephritis
D) Renal tuberculosis
E) Myelomic nephropathy
ANSWER: B
Patient, 39 years old, complains on headache, decreasing of appetite, nausea, vomiting
in the morning, periodic nasal bleeding. He has had chronic tonsillitis, from 15 years
- pathology in the analyses of urine. At the examination: pallor of skin, traces of
scratching on skin, BP - 200/140 mm Hg. In the analysis of urine: proteinuria - 0,99
g/l, leucocytes - 3-5 in the visual field, erythrocytes - 8-10 in the visual field,
RBC casts - 3-4 in the visual field. What from the biochemical tests is the most
informative for estimation of the patient condition:
A) Serum creatinine
B) Serum sodium
C) Serum uric acid
D) Fibrinogen of blood
E) Bilirubin of blood
ANSWER: A
Patient, 45 years old, during 8 years has been treating with concerning fibrocavernous
tuberculosis. On examination - edema of lower extremities, diarrhea appeared 2 months
ago. In the urinalysis: proteinuria - 3,5 g/l, leukocyturia - 10-12 in the visual
field, microhematuria, hyaline cylinders - 3-4 in the visual field. What is the most
reliable diagnosis?
A) Renal amyloidosis
B) Chronic glomerulonephritis
C) Acute glomerulonephritis
D) Renal tuberculosis
E) Acute pyelonephritis
ANSWER: A
Postoperatively, the patient develops a fever, and pyuria is noted. Laboratory studies
show a leukocyte count of 12,000/?L, a blood urea nitrogen level of 24 mmol/L, and a
serum creatinine concentration of 140 mmol/L. The urine culture grows Escherichia coli,
and treatment with trimethoprim-sulfamethoxazole is started. Three days later, the
pyuria and fever have resolved. The leukocyte count is 10,000/?L, blood urea nitrogen
level is 24, serum creatinine is 180 mmol/L. Urinalysis shows no leukocytes in high-
power fields. What is the most likely explanation for the elevated serum creatinine
concentration?
A) Acute interstitial nephritis
B) Acute pyelonephritis
C) Obstructive uropathy
D) Reduced creatinine excretion
E) Acute tubular necrosis
ANSWER: D
Q . A 54-year-old man with a long-standing history of hypertension presents to the
emergency department with the sudden onset of tearing pain in his midback. He appears
dyspneic. Vital signs show a blood pressure of 90/65 mm Hg, respirations of 24
breaths/min, and heart rate of 110 beats/min. He has crackles in one third of the lung
fields bilaterally. Cardiac exam reveals an early decrescendo diastolic murmur at the
left sternal border. Computed tomography reveals a false lumen and thrombus in the
ascending aorta. Which of the following statements is true?
A) Emergent surgery will most likely be required
B) One could expect to hear "pistol shot" sounds when auscultating over the femoral
artery
C) Aortic regurgitation is unlikely as his pulse pressure is not widened
D) Gentle compression of the nail bed should result in capillary pulsation
E) An intra-aortic balloon pump should be inserted urgently to treat hypotension and
stabilize until surgery
ANSWER: A
Q . A 75-year-old man is diagnosed with aortic stenosis. What is the most likely
etiology of his valvular disorder?
A) Degenerative calcific valve
B) Bicuspid aortic valve
C) Infective endocarditis
D) Rheumatic heart disease
E) Subvalvular aortic ridge
ANSWER: A
The condition of a 50-year-old obese woman with a 5-year history of mild hypertension
controlled by a thiazide diuretic is being evaluated because proteinuria was noted
during her routine yearly medical visit. Physical examination disclosed a height of
167.6 cm (66 in.), weight 91 kg (202 lb), blood pressure 130/80 mmHg, and trace pedal
edema. Laboratory values are as follows: Serum creatinine: 106 umol/L (1.2 mg/dL)
Creatinine clearance: 87 mL/min. Urinalysis: pH 5.0; specific gravity 1.018; protein 3
+; no glucose; occasional coarse granular cast. Urine protein excretion: 5.9 g/d The
results of a renal biopsy are :sixty percent of the glomeruli appeared sclerotic as
“halfmoons”; the remainder were unremarkable. The most likely diagnosis is
A) hypertensive nephrosclerosis
B) focal and segmental sclerosis
C) minimal-change (nil) disease
D) membranous glomerulopathy
E) crescentic glomerulonephritis
ANSWER: E
The man, 23 years old, complains on edema of face, headache, dizzinesses, oliguria,
brown color of urine. These complaints appeared after pharyngitis.Objectively: edema
on the face, cardiac tones are muffled, accent of II tone above aorta, heart rate –
86/min, BP - 170/110 mm Hg. In the general analysis of urine will be:
A) Erythrocyturia, leukocyturia, leukocyte cylinders
B) Leukocyturia, proteinuria, hyaline cylinders
C) Erythrocyturia, oxaluria, waxy cylinders
D) Erythrocyturia, proteinuria, erythrocyte cylinders
E) Hemoglobinuria, bilirubinuria, granular cylinders
ANSWER: D
The patient P., 40 years old, complains on edema of face, shins. The diagnosis of
rheumatoid arthritis was confirmed four years ago. Objectively: anasarca, BP 120/70 mm
Hg. In blood test: Нb - 115 g/l, ESR - 30 mm/h; in the urinalysis: proteinuria - 4,2
g/l, leucocytes - 6-8 in the visual field, hyaline and granular cylinders - 5-6 in the
visual field. What complication has developed?
A) Secondary amyloidosis
B) Chronic glomerulonephritis
C) Tuberculosis of kidneys
D) Myelomic nephropathy
E) Chronic pyelonephritis
ANSWER: A
The patient, 48 years old, who arrived from the places of imprisonment, complains ofn
headache, decline of vision, nausea, thirst. Arterial hypertension has been diagnosed
8 years ago. At the examination: skin is pale, pulse – 90, BP - 220/140 mm Hg, liver -
on 3 cm below the costal arc. CBC: erythrocytes - 2,1x1012/l, leucocytes - 7,8х109/l,
ESR - 48 mm/h, in the analysis of urine: specific gravity - 1007; proteinuria - 1,65
g/l, erythrocytes - 10-12 in the visual field, leucocytes - 3-4 in the visual field,
hyaline cylinders - 4-5 in the visual field, serum creatinine - 243 umol/l. Your
previous diagnosis:
A) Chronic pyelonephritis
B) Chronic glomerulonephritis
C) Hypertonic disease
D) Amyloidosis of kidneys
E) Renal tuberculosis
ANSWER: B
Two weeks after hospital discharge for documented myocardial infarction, a 65-year-old
returns to your office very concerned about low-grade fever and pleuritic chest pain.
There is no associated shortness of breath. Lungs are clear to auscultation and heart
exam is free of significant murmurs, gallops, or rubs. ECG is unchanged from the last
one in the hospital. The most effective therapy is likely
A) Antibiotics
B) Anticoagulation with warfarin (Coumadin)
C) An anti-inflammatory agent
D) An increase in antianginal medication
E) An antianxiety agent
ANSWER: C
Two weeks after hospital discharge for documented myocardial infarction, a 65-year-old
returns to your office very concerned about low-grade fever and pleuritic chest pain.
There is no associated shortness of breath. Lungs are clear to auscultation and heart
exam is free of significant murmurs, gallops, or rubs. ECG is unchanged from the last
one in the hospital. The most effective therapy is likely
A) Antibiotics
B) Anticoagulation with warfarin (Coumadin)
C) An anti-inflammatory agent
D) An increase in antianginal medication
E) An antianxiety agent
ANSWER: C
Two weeks after therapy is initiated in the 31-year-old white woman with diabetes
presents for management of her hypertension. Type 1 diabetes mellitus was diagnosed at
9 years of age. Patient claims that the condition is under reasonable control, her
blood pressure decreases from 150/90 mm Hg to 128/80mm Hg, and she feels well.
Repeated laboratory testing reveals an increase in serum creatinine concentration from
1.9 mg/dL to 2.1 mg/dL. The potassium concentration is 4.2 mmol/L. Which of the
following is the most appropriate course of action?
A) Discontinue antihypertensive therapy
B) Perform noninvasive screening for possible renal artery stenosis
C) Perform renal angiography
D) Continue antihypertensive therapy and monitor kidney function
E) none of them
ANSWER: D
Two years after undergoing mitral valve replacement, a 48-year-old man has a
cerebrovascular accident. Except for fever, general physical examination is
noncontributory. A transesophageal echocardiogram shows an oscillating mass on the
mitral valve but no evidence of perivalvular extension or abscess. Six sets of blood
cultures grow Enterococcus faecalis, which is resistant to penicillin and ampicillin
but sensitive to vancomycin. A decision is made to treat medically, and the patient is
begun on vancomycin, 1 g intravenously every 12 hours, and gentamicin, 80mg
intravenously every 8 hours. The technician notes that the vancomycin peak and trough
levels and the gentamicin trough level are in the desirable range but that the
laboratory therapeutic peak range for gentamicin is 4 to 8 Oјg/mL. Repeat blood
cultures show no growth, and complete blood count and serum creatinine values are
normal. Which of the following is most appropriate at this time?
A) Keep both the gentamicin and the vancomycin doses unchanged
B) Increase the gentamicin dose; keep the vancomycin unchanged
C) Decrease the interval between the gentamicin doses; keep the vancomycin unchanged
D) Increase the vancomycin dose; keep the gentamicin unchanged
E) Increase the gentamicin dose; keep the vancomycin unchanged and decrease the
interval between the gentamicin doses.
ANSWER: A
Two years after undergoing mitral valve replacement, a 48-year-old man has a
cerebrovascular accident. Except for fever, general physical examination is
noncontributory. A transesophageal echocardiogram shows an oscillating mass on the
mitral valve but no evidence of perivalvular extension or abscess. Six sets of blood
cultures grow Enterococcus faecalis, which is resistant to penicillin and ampicillin
but sensitive to vancomycin. A decision is made to treat medically, and the patient is
begun on vancomycin, 1 g intravenously every 12 hours, and gentamicin, 80mg
intravenously every 8 hours. The technician notes that the vancomycin peak and trough
levels and the gentamicin trough level are in the desirable range but that the
laboratory’s therapeutic peak range for gentamicin is 4 to 8 μg/mL. Repeat blood
cultures show no growth, and complete blood count and serum creatinine values are
normal.
A) Keep both the gentamicin and the vancomycin doses unchanged
B) Increase the gentamicin dose; keep the vancomycin unchanged
C) Decrease the interval between the gentamicin doses; keep the vancomycin unchanged
D) Increase the vancomycin dose; keep the gentamicin unchanged
E) Increase the gentamicin dose; keep the vancomycin unchanged and decrease the
interval between the gentamicin doses.
ANSWER: A
V . is a 71-year-old retired female whose father died at age 52 of a “massive MI”.
She is very worried about her own risk of a heart attack. She watches her weight
(BMI=23), does not smoke, and keeps physically fit. She denies angina or dyspnea on
exertion, claudication, or history of TIA symptoms. Her fasting lipids are as follows:
TC 250 mg/dL, TG 120 mg/dL, HDL-C 42 mg/dL, LDL-C (calculated)151. If V. were found to
have untreated hypertension (SBP of 160), with all other data the same, what would be
the preferred initial treatment in addition to BP control?
A) Nothing beyond her current lifestyle measures
B) Weight loss to bring BMI&amp;amp;lt;20
C) TLC plus a statin to reduce LDL-C to &amp;amp;lt;100
D) TLC to reduce LDL-C to &amp;amp;lt;130
E) Statin plus a niacin to reduce LDL-C&amp;amp;lt;130 and increase HDL-
C&amp;amp;gt;50 mg/dL
ANSWER: C
V . is an obese nonsmoking 44-year-old man with treated hypertension and no family
history of CHD. He has had elevated LDL-C in the past and is taking atorvastatin 20 mg
daily. His latest lipid panel is as follows: TC 220, HDL-C 40, LDL-C 120, TG 450.
After recommending lifestyle modifications, what is your first goal of drug treatment?
A) Increase the HDL-H
B) Lower the LDL-C
C) Lower the TG
D) Lower the TG and increase the HDL-C
E) Lower the non-HDL-C
ANSWER: E
You are seeing in your office a patient with the chief complaint of relatively sudden
onset of shortness of breath and weakness but no chest pain. ECG shows nonspecific ST-
T changes. You would be particularly attuned to the possibility of painless, or silent,
myocardial infarction in the
A) Advanced coronary artery disease patient with unstable angina on multiple
medications
B) Elderly diabetic
C) Premenopausal female
D) Inferior MI patient
E) MI patient with PVCs
ANSWER: B
You see a 62-year-old man with isolated essential hypertension, currently taking
hydrochlorothiazide 25 mg PO daily, comes to you for his first clinic visit. He notes
that his BP at home is always less than 135/80 mmHg, but in clinic it is always at
least 150/90 mmHg. What is the next step?
A) Increase dose of thiazide
B) Addition of second antihypertensive medication
C) Do nothing as he has white coat hypertension
D) Search for secondary causes of hypertension
E) Start metoprolol
ANSWER: C
You see a diabetic patient presents with BP readings that are 155/95 or higher. All of
the following statements about the treatment of this patient's hypertension are
correct EXCEPT:
A) Pharmacologic blocade of the renin-angiotensin system reduces the risk of both
microvascular and macrovascular events
B) Aggressive BP control reduces cardiovascular events more in diabetics than in
nondiabetics
C) Calcium channel blockers show no benefit in reducing cardiovascular events
D) The goal BP for this patient is &amp;amp;lt;130/80 mmHg
E) All the above statements are correct
ANSWER: C
You see a diabetic patient presents with BP readings that are 155/95 or higher. All of
the following statements about the treatment of this patient's hypertension are
correct EXCEPT:
A) Pharmacologic blocade of the renin-angiotensin system reduces the risk of both
microvascular and macrovascular events
B) Aggressive BP control reduces cardiovascular events more in diabetics than in
nondiabetics
C) Calcium channel blockers show no benefit in reducing cardiovascular events
D) The goal BP for this patient is &amp;amp;lt;130/80 mmHg
E) All the above statements are correct
ANSWER: C
Z . is a 48-year-old obese male with hypertension, dyslipidemia, and diabetes mellitus.
He presents to the outpatient clinic for his yearly physical. He has refused
medications in the past, but now is willing to consider treatment. His BP is 145/95
mmHg with a HR of 80 bpm. His laboratory data are significant for the presence of
microalbuminuria. Which of the following medications would be the most appropriate?
A) Carvedilol
B) Methyldopa
C) Lisinopril
D) Chlorthalidone
E) Terazozin
ANSWER: C
20-years-old man suffers from bronchial asthma, the attacks of dyspnea arise 3-4 times
a day. Night attacks are present 2 times a week. FEV1 - 70 %, its variability during
day is 30 %. What is your diagnosis?
A) Moderate persistent BA
B) Status asthmaticus
C) Intermittent BA
D) Severe persistent BA
E) Mild persistent BA
ANSWER: A
25-year-old-girl, presents with complaints of fever and weakness. On examination there
is splenomegaly of 3 cm below the costal margin. Hb is 6 g/dL, WBC is 2.0 x 109/L,
platelets count is 50 x 109/L. Which of the following is the least likely diagnosis?
A) Acute lymphocytic leukemia
B) Anemia of chronic disease
C) Aplastic anemia
D) Megaloblastic anemia
E) Sideroblastic
ANSWER: C
28 year-old patient complaints of weakness, dizziness, noise in ears, deranged
appetite. Physical examination: paleness of skin. Hair is thin, brittle, alopecia,
nails are flat and brittle. Unpleasant smell from a mouth is felt. In blood analysis
hypochromia and microcytosis. What is the most likely diagnosis?
A) Iron deficiency anaemia
B) Pernicious anaemia
C) Hemolitic anaemia
D) Acute leukaemia
E) Erythremia
ANSWER: A
35 y.o. woman is suspected of aplastic anemia. The bone marrow punction has been
administered with the diagnostic purpose. What possible changes in the bone marrow?
A) Replacement of marrow elements with adipose tissue
B) Replacement of marrow elements with fibrous tissue
C) Prevalence of megaloblasts
D) Presence of blast cells
E) Absolute lymphocytosis
ANSWER: A
35 year - old female has chronic menorrhagia. On physical examination signs of
hypochromic anaemia are found. What data is specific for diagnostics?
A) Blood protein
B) Reticulocytes count
C) Serum iron
D) Assessment of red blood cells shape
E) ESR
ANSWER: C
35 year-old patient iron-deficiency anaemia was diagnosed. Which of clinical findings
is typical for the disease?
A) Gray hair
B) Fissures of mouth angles
C) Koilonychias
D) Loss of smell sensation
E) Loss of skin sensitiveness
F) Folic acid deficiency anaemia
ANSWER: C
40 y.o. man is suspected of aplastic anemia. What possible changes in the bone marrow?
A) specimens are hypocellular
B) specimens are with fibrous tissue
C) specimens with megaloblasts
D) Presence of blast cells
E) Absolute lymphocytosis
ANSWER: A
42 year-old mother of 6 children. She complains of fatigue, palpitation, brittleness
of nails, falling hair. RBC - 2,4 x 1012/L, haemoglobin –68 g/l, colour index – 0,7,
reticulocytes – 1 %, thrombocytes – 200 x 109/L, leukocytes – 6,6 x 109/L,
anisocytosis, poikylocytosis, ESR – 5 mm/hour. What is the mostlikely diagnosis?
A) Vitamin b12 and folic acid deficiency anaemia
B) Iron deficincy anaemia
C) Syderoachrestic anaemia
D) Haemolytic anaemia
E) Aplastic anaemia
ANSWER: B
44-year-old woman complained of attacks of right subcostal pain after fatty meal for 1
year. Last week the attacks have repeated every day and became more painful. What
diagnostic study would you recommend?
A) Ultrasound examination of the gallbladder
B) Blood cell count
C) Ultrasound study of the pancreas
D) X-ray examination of the gastrointestinal tract
E) None of above
ANSWER: A
45 year-old patient with Crohn’s disease. The patient develops general weakness and
dyspnea last time. Iron deficiency anaemia is diagnosed. Which way of administration
iron supplementation is the most acceptable in this case?
A) Per oss
B) Parenteral
C) Per rectum
D) Inhalation
E) Subcutaneous
F) Microcytic
ANSWER: B
47-year-old man is evaluated because of cough that has persisted for 6 months. He has
no postnasal drip, wheeze, or heartburn. His physical examination, chest radiograph,
and spirometry are normal. He receives no benefit from a 3-month trial of twice-daily
proton-pump inhibitors, intranasal corticosteroids, and antihistamines. He has a
family history of allergies. Which of the following would suggest the diagnosis of
this patient?
A) Methacholine challenge testing
B) 24-hour esophageal pH monitoring
C) CT scan of the sinuses
D) Bronchoscopy
E) CT scan of the chest
ANSWER: A
55-yr-old woman underwent upper GI endoscopy for assessment of dysphagia. Three hours
later she complained of severe chest pain. On examination there was crepitus in the
neck. Choose the single most likely investigation from the list of options.
A) Barium swallow
B) Oesophageal manometry
C) Upper GI endoscopy and biopsy
D) CT scan chest
E) Chest radiography
ANSWER: E
60-yr-old male non-smoker demonstrates such results of his pulmonary function tests:
FEV1—1.4 l, FVC—3.5 l, FEV1/FVC—40%. After bronchodilator trial—FEV1/FVC—59%. After 2
weeks of prednisolone 30 mg daily FEV1/FVC = 72%. What is the diagnosis?
A) Bronchial asthma
B) COPD
C) Emphysema
D) Tracheal compression
E) Pulmonary fibrosis
ANSWER: A
65 year-old male was passing large voluminous foul smelling stools for 4-6 days; over
last 3 months he has developed anemic. Most likely etiology of anemia is:
A) Vitamin B12 deficiency
B) Iron deficiency
C) Folic acid deficiency
D) Pyridoxin deficiency
E) Sideroblastic
ANSWER: B
65 year-old patient presents with bone pain. On examination there is elevated ESR of
100 mm/h, serum proteins-120 g/L, lytic lesions in the skull, serum creatinine of 3.5
mg/dL and serum calcium of 14 mg/dL. Bone marrow revealed plasma cells of 21%. What is
the most likely diagnosis?
A) Waldenstrom's macroglobulinemia
B) Hyperparathyroidism
C) Osteomalacia
D) Multiple myeloma
E) ALL
ANSWER: D
A 15-yr-old Jamaican boy complains of abdominal and bone pains of sudden onset. He is
found to be pyrexial and has recently had a chest infection. What is the diagnosis?
A) Sickle cell disease
B) Anaemia of chronic disease
C) Pernicious anaemia
D) Autoimmune hemolytic anaemia
E) Aplastic anaemia
ANSWER: A
A 16 y.o. boy admitted to the hospital after dental extraction bleeding. Family
history is unremarkable. CBC: RBC 2.0 х 1012/L, Hemoglobin 60 g/L, Hematocrit 29 %,
MCV 64 fl, Reticulocyte Count 3.0 %, Pl 241 х 109/L, WBC 7.9 х 109/L, Segmented
Neutrophils 60 %, Band Neutrophils 4 %, Monocytes 5 %, Eosinophils 3 %, Basophils 2 %,
Lymphocytes 27 %, ESR 7 mm/h, Anisocytosis, hypochromia +, Bleeding time 5 min,
Prothtombin time 12 seconds, Partial thromboplstin time 45 seconds, Factor VIII 3 %.
What is the most likely diagnosis?
A) Hemophilia A
B) Idiopathic thrombocytopenic purpura
C) Henoch–Schonlein purpura
D) Haemolytic uremic syndrom
E) Macrocytic hyperchromic anemia
ANSWER: A
A 17-year-old previously healthy man presents with a history of shortness of breath on
exertion, particularly during basketball season, when he sometimes needs to sit down
during practice to catch his breath. He does not notice any shortness of breath with
routine activity. There is no family history of asthma. On physical examination, he is
in no respiratory distress. His lungs are clear, with no wheezing during either tidal
breathing or forced expiration. His heart is normal. Baseline spirometry is normal.
What is the next diagnostic step?
A) Exercise testing with postexercise spirometry
B) Allergy testing
C) Methacholine challenge testing
D) Overnight oximetry
E) Repeat lung volumes and diffusing capacity
ANSWER: A
A 18-yr-old Asian girl presents with anaemia. She remembers her brother died at the
age of 5 after an illness since birth, which required repeated transfusions. What is
the nesessary test?
A) Coombs' test
B) Serum B12
C) Hb electrophoresis
D) Urine for Bence-Jones proteins
E) Thrombin time, fibrin degradation products
ANSWER: C
A 19-yr-old patient presents with wheezing attacks and episodic shortness of breath.
His PEFR is 400 l/min. What is the best method of treatment?
A) ?2 agonist
B) Erythromycin
C) Plasmaphoresis
D) Cyclophosphamide
E) Co-trimoxazole
ANSWER: A
A 19-yr-old student presents with a neck swelling. On examination the swelling moves
up with swallowing and protrusion of the tongue. Choose the most likely diagnosis
A) Thyroglossal cyst
B) De Quervain's cyst
C) Hypothyroidism
D) Graves' disease
E) Simple goiter
ANSWER: A
A 20-year-old female exchange student from Paris has had bouts of jaundice, fever,
malaise, arthralgias, and marked elevation of hepatic transaminases over the past 6
months. The patient was not exposed to hepatotoxic drugs. Hypergammaglobulinemia has
been noted. Serologic evaluation for infection with hepatitis A, B, and C has been
negative, as have tests for systemic lupus. Liver biopsy now reveals bridging necrosis.
Which of the following tests will be most helpful in confirming the diagnosis?
A) Antibodies to liver and kidney microsomal antigens
B) Hemoglobin electrophoresis
C) Rheumatoid factor
D) Antibodies to hepatitis D virus
E) Antibodies to hepatitis E virus
ANSWER: A
A 20-yr-old men presents with wheezing and shortness of breath. His PEFR is 400 l/min.
Spirography: FEV1, FVC and Typhno index are low. What is the diagnosis?
A) Asthma
B) COPD
C) Emphysema
D) Pulmonary embolism
E) Pneumonia
ANSWER: A
A 20-yr-old patient complains of worsening vision over several months. On examination
fundoscopy is difficult even through dilated pupils. There appears to be opacity of
the lens. Choose the most likely diagnosis
A) Coronary artery disease
B) Cerebrovascular disease
C) Nephropathy
D) Cataracts
E) Polyneuropathy
ANSWER: D
A 20-yr-old woman is too breathless to speak. Her pulse is 120/min, respiratory rate
is 30/min and PEFR is 100 l/min. Examination reveals a very quiet chest and CXR is
normal. Choose the single most likely management:
A) Nebulised salbutamol
B) Pleural aspiration
C) Heparin
D) Intramuscular adrenaline
E) Rapid infusion of saline
ANSWER: A
A 20-yr-old woman presents with a 4 cm solid mass in the left thyroid lob. FNAC
reveals papillary carcinoma. Choose the most likely management.
A) Total thyroid lobectomy
B) Ablative dose of radioactive iodine
C) External beam radiation
D) Propranolol
E) Total thyroidectomy and removal of central group of lymph nodes
ANSWER: E
A 21-yr-old man: Bilirubin—45 mcmol/l—conjugated—7, unconjugated—38, alkaline
phosphatase-norm, ALT—Norm, Blood film reported as normal. Choose the single most
likely diagnosis from the list of options above.
A) Gilbert's syndrome
B) Alcoholic cirrhosis
C) Hepatitis
D) Carcinoma of head of pancreas
E) Primary biliary cirrhosis
ANSWER: A
A 22-year-old vegetarian patient with signs of malnutrition consulted a doctor about
smell and taste distortion, angular stomatitis, spoonshaped nails, alopecia.
Objectively: blue sclerae. The patient was diagnosed with iron deficiency anemia. What
is the specific clinical syndrome?
A) Sideropenic
B) Leukocytosis
C) Haemorrhagic
D) Haemolytic
E) Myelodysplastic
ANSWER: A
A 22-year-old woman complains of gum bleeding, menorrhagia. Physical examination
reveals petechiae, bruises on her legs. CBC: RBC 2.5 х 1012/L, Hemoglobin 67 g/L, MCV
64 fl, CI – 0,7, Pl 38 х 109/L, Giant platelets +, WBС 7.9 х 109/L. All of the
followings are true about the disease except:
A) Treatment with steroids
B) Autoimmune nature
C) Prolonged clotting time
D) Splenectomy indicated
E) Prolonged bleeding time
ANSWER: E
A 22-year-old woman who previously was healthy presents with dizziness and nausea.
Routine blood tests show a blood glucose level of 52 mg/dL. Results of kidney and
liver function tests are normal, but the patient’s potassium level is slightly
elevated. Which of the following tests should be performed next?
A) Insulin and C-peptide levels
B) Adrenocorticotrophic hormone (ACTH) stimulation test
C) Random cortisol levels
D) Dexamethasone suppression test
E) 24-Hour urine collection for free cortisol
ANSWER: B
A 22-yr-old man who was driving was involved with a high speed collision. He was
wearing his seat belt and now complains of upper abdominal pain. His CXR is normal.
Choose the single most likely investigation from the list of options above.
A) Serum amylase
B) Arterial blood gases
C) Barium enema and colonic biopsy
D) ESR
E) Serum electrolytes
ANSWER: A
A 25-year-old man is evaluated because of a 3-month history of episodic dyspnea at
rest. The episodes occur approximately three times per week and are accompanied by
cough. His symptoms awaken him at night approximately three times per month. He had
asthma as a child, which resolved. His temperature is 36.5 C, pulse rate 85/min and
regular, respiration rate 14/min, and blood pressure 125/75 mm Hg. The only
abnormality noted on physical examination is bilateral wheezing without crackles.
Chest radiograph is normal. Spirometry shows a forced expiratory volume in 1 sec (FEV1)
78% of predicted, and a forced vital capacity 93% of predicted. He has a 17% (430 ml)
improvement in FEV1 after using bronchodilators. Which of the following is the most
appropriate treatment regimen for this patient?
A) Albuterol and a low-dose inhaled corticosteroid
B) Albuterol
C) Albuterol and a long-acting ?2-agonist
D) A long-acting ?2-agonist
ANSWER: A
A 25-year-old patient complains of general weakness, fatigue, fever. RBC 2.9 х 1012/L,
Hb 77 g/L, Ht 34 %, MCV 88 fl, Reticulocyte Count 0,1 %, Pl 87 х 109/L, WBC 48,5 х
109/L, Segmented Neutrophils 15 %, Band Neutrophils 1 %, Metamyelocytes -0%,
Myelocytes 0-%, Lymphoblasts 50 %, Monocytes 4 %, Eosinophils 0 %, Basophils 0 %,
Lymphocytes 30 %, ESR 50 mm/h. What is the most likely diagnosis?
A) Acute leukemia
B) Erythremia
C) Chronic lymphocytic leukemia
D) Chronic myeloid leukemia
E) Multiple myeloma
ANSWER: A
A 27-year-old Hispanic woman who is 13 weeks pregnant has had recurrent episodes of
biliary colic for the past 6 months. The episodes are now increasing in frequency, and
one episode was associated with mild pancreatitis. Abdominal ultrasonography shows
multiple gallstones. Results of routine laboratory studies are normal. When is the
most appropriate time for this patient to undergo laparoscopic cholecystectomy?
A) During the second trimester
B) Immediately
C) During the third trimester
D) Post partum
E) Conservative therapy is recommended
ANSWER: A
A 27-year-old male presents to the health center for right upper quadrant abdominal
pain, generalized pruritus, and jaundice for 3 days. He states that the pain came on
gradually and awoke him early on the morning of presentation. His past medical history
is pertinent for ulcerative colitis, although he has not taken any medication in 4
years. His temperature is (39.2°C) and physical exam shows pain in the right subcostal
region with deep inspiration, and generalized jaundice. What is the most serious
complication of the most likely diagnosis?
A) Perforation of rectum.
B) Perforation of stomach wall.
C) Infarction of small bowel.
D) Cholangiocarcinoma.
E) Pancreatic pseudocyst formation.
ANSWER: D
A 27-year-old man complains to a physician of chronic gastric pain. The pain is
sometimes relieved by food. EGDS shows antral erosive gastritis, biopsy of antral
mucous presents Hеlicobacter Pylori. What can be diagnosed in this case?
A) Gastritis of type B
B) Menetrier’s disease
C) Rigid antral gastritis
D) Gastritis of A type
E) Reflux – gastritis
ANSWER: A
A 27-year-old man is evaluated because of two episodes of painless melena, the last of
which occurred 6 hours ago. He has been taking naproxen for the past 6 weeks for a
sports-related shoulder injury. Medical history is otherwise noncontributory. On
physical examination, temperature is 36.0 C (96.8 F), pulse rate is 98/min and regular,
respiration rate is 18/min, and blood pressure is 104/89 mm Hg without orthostatic
changes. Digital rectal examination discloses very dark stool that is positive for
occult blood. Hemoglobin is 9.3 g/dL, leukocyte count is 10,800/?L, platelet count is
250,000/?L, and other routine laboratory studies are normal. Nasogastric aspirate is
negative. Which of the following should be done next?
A) Order red blood cell transfusion to achieve a hemoglobin value above 10 g/dL
B) Establish access with two intravenous lines and infuse isotonic saline, 500 mL/h
for 6 to 8 hours; then reassess the hemoglobin value
C) Transfuse 6 units of platelets to correct the platelet dysfunction caused by
naproxen
D) Perform upper endoscopy
E) Order an upper gastrointestinal series
ANSWER: D
A 27-yr-old pregnant woman is found to have thyrotoxicosis due to Grave's disease
during second trimester of her pregnancy. Choose the most likely management.
A) Radioactive iodine
B) Subtotal thyroidectomy
C) Propanolol
D) Carbimazole
E) Thyroxine
ANSWER: D
A 27-yr-old woman presents with fever, sore throat and dysphagia. On examination she
has a fine tremor and a diffusely tender thyroid. Radioisotope scan shows no uptake.
Choose the most likely diagnosis
A) Thyroglossal cyst
B) De Quervain's cyst
C) Hypothyroidism
D) Lymphoma
E) Simple goiter
ANSWER: B
A 27-yr-old woman was found to have glycosuria at a routine antenatal clinic visit. A
glucose tolerance test confirmed diagnosis of gestational diabetes. Choose the most
likely management.
A) Metformin
B) No change in treatment required
C) IV insulin sliding scale
D) Gliclazide
E) Dietary adjustment
ANSWER: E
A 28 years old patient, complaints on cough with small amount of colourless sputum,
pain in the right half of thorax during breathing, shortness of breath, increase of
temperature to 39 °С. Felt ill rapidly. Used aspirin. Objectively: herpes on lips. In
lower lobe of right lung there is dull percussion sound, bronchial breathing. X-ray:
there is homogeneous infiltration of right lower lobe. What is the most possible
etiology of pneumonia?
A) Pneumococcus
B) Staphylococcus
C) Mycoplasma
D) Legionella
E) Klebsiella
ANSWER: A
A 29 year-old man has Crohns ileocolitis complicated by perianal fistulae. Ileocecal
resection was performed 5 years ago. Approximately 4 months ago, he developed
abdominal pain, cramping, and a new enterocutaneous fistula. An upper gastrointestinal
series with small bowel follow-through showed 4 cm of inflammatory changes at the
neoterminal ileum plus an enterocutaneous fistula originating from this area. After
treatment with metronidazole and mesalamine, his abdominal pain improved and the
fistula appeared to resolve. Which of the following is most appropriate for
maintaining remission in this patient?
A) Start 6-mercaptopurine
B) Continue metronidazole
C) Start budesonide
D) Start prednisone
E) tart oral cyclosporine
ANSWER: A
A 29 year-old woman has hemoglobin level 7.8 g/dl, with a reticulocyte count of 1 %.
The peripheral blood smear showed microcytic hypochromic anemia. The serum iron was
low and the total iron binding capacity was high. The most likely cause of anemia is
A) Iron deficiency anemia.
B) Beta-thalassemia minor.
C) Sideroblastic anemia.
D) Anemia due to chronic infection.
E) Porphyria
ANSWER: A
A 29-year-old patient complains of weight loss, night sweating, general weakness,
fatigue, fever. CBC reveals: RBC 1.1 х 1012/L, Hb 48 g/L, Pl 40 х 109/L, WBC 138,6 х
109/L. In bone marrow - Blasts with Auer rods 70 %. What is the most likely diagnosis?
A) Acute nonlymphoblastic (myeloid) leukemia
B) Chronic lymphocytic leukemia III st
C) Chronic myeloid leukemia, accelerated phase
D) Chronic myeloid leukemia, chronic phase
E) Chronic lymphocytic leukemia IV st
ANSWER: A
A 30-yr-old woman presents with a 2 cm thyroid nodule. FNAC suggests a colloid nodule.
Choose the most likely management.
A) Total thyroid lobectomy
B) Ablative dose of radioactive iodine
C) External beam radiation
D) Chemotherapy
E) Reassure and repeat FNAC in 1 yr
ANSWER: E
A 30-yr-old woman presents with weight gain, constipation, lethargy and flaky rash.
Choose the most likely diagnosis
A) Thyroglossal cyst
B) De Quervain's cyst
C) Hypothyroidism
D) Lymphoma
E) Simple goiter
ANSWER: C
A 31-year-old man has a 2-month history of abdominal pain and bloating. An upper
gastrointestinal series with small bowel follow-through shows ulcerations and
inflammatory changes in the distal 12 cm of the terminal ileum. Colonoscopy is normal
except for erythema and small linear ulcerations seen on cannulation of the terminal
ileum. Which of the following is the most appropriate therapy for this patient at this
time?
A) pH-release mesalamine
B) Mesalamine enemas
C) Balsalazide
D) Olsalazine
E) Sulfasalazine
ANSWER: A
A 31-year-old woman with ulcerative colitis has been taking prednisone for the past
year. Each time the prednisone is tapered below 20 mg/d, her symptoms return. She is
subsequently started on 6-mercaptopurine, 50 mg/d. Three days after beginning the new
drug, she develops worsening abdominal pain with radiation to her back. She does not
have a rash. Her leukocyte count is 3200/?L. Which of the following is the most likely
cause of this patients new symptoms?
A) Pancreatitis due to initiation of 6-mercaptopurine
B) Pancreatitis due to continuation of prednisone
C) Flare of ulcerative colitis
D) An abdominal and psoas abscess secondary to 6-mercaptopurine-induced neutropenia
E) None of above
ANSWER: A
A 32 -year-old woman complained on epigastric pain just after meal, heartburn, and
nausea. Stomach endoscopy revealed a large amount of mucus, hyperemia, erosions of
mucous membrane of stomach. Helicobacter infection test is positive.
A) Chronic type A gastritis
B) Peptic ulcer of the stomach
C) Chronic type C gastritis
D) Menetrier’s disease
E) Chronic type B gastritis
ANSWER: E
A 32-year-old father of two is involved in a head-on motor vehicle accident and is
admitted to your hospital. He has sustained a blunt force trauma to the upper thorax
but injuries appear only to be several cracked ribs and a minor concussion. On
hospital day 3, he reports the uncontrollable urge to drink copious amounts of water,
states that he has been urinating much more frequently and that his urine is very thin
in appearance. The blood and a urin analysis are normal except for high plasma
osmolarity and low urine osmolarity. You decide to do a water deprivation test. Thus
for 24 hours, the patient is restricted to a sip of water every hour, and on day 2 of
the test is clinically judged to be dehydrated. Vasopressin (ADH) is then given by IV
and his urine osmolarity is seen to rise above a baseline dehydrated state reading.
The most likely diagnosis is central diabetes insipidus. What is the next step in the
management of this patient?
A) Hydrochlorothiazide (HCTZ).
B) Furosemide (Lasix).
C) Hydrocortisone orally.
D) Aggressive fluid replacement.
E) Desmopressin (DDAVP).
ANSWER: E
A 32-year-old woman has a 4-month history of hoarseness and throat clearing.
Evaluation by an otolaryngologist disclosed laryngeal inflammation suggestive of
gastroesophageal reflux disease, and the patient is referred to you. She is otherwise
asymptomatic and denies heartburn, regurgitation, dysphagia, and weight loss. The
patient maintains an active lifestyle and currently takes no medications. Physical
examination and routine laboratory studies are normal. Which of the following should
be done next?
A) Upper endoscopy
B) Esophageal manometry
C) Ambulatory 24-hour esophageal pH monitoring
D) Barium swallow
E) Trial of acid suppressive therapy
ANSWER: E
A 32-yr-old male presents with nausea and acute abdominal pain boring through to the
back. The epigastrium is very tender. He has had three similar bouts in the past 18
months. A barium meal is normal. Choose the single most likely diagnosis
A) Gastric atrophy
B) Gastric ulcer
C) Basal pneumonia
D) Chronic cholecystitis
E) Non-ulcer dyspepsia
ANSWER: E
A 32-yr-old man presents with spontaneous bruising and recurrent infections with
marked lethargy. There is no recent treatment history. His Hb is 56 g/l, RBC 2,8 x
1012/L, WBC 2.8 platelets 48,5. What is the diagnosis?
A) Aplastic anaemia
B) Autoimmune hemolytic anaemia
C) Anaemia of chronic disease
D) Iron deficiency
E) Pernicious anaemia
ANSWER: A
A 32-yr-old woman has had type 1 diabetes for 15 yr. She injects Isophane insulin
twice a day and rarely tests her blood glucose at home. She attends the diabetic
clinic for the first time in over a year and informs you that she is 12 weeks pregnant.
Choose the most likely management.
A) Metformin
B) No change in treatment required
C) IV insulin sliding scale
D) One long-acting and 3 short- acting insulin
E) Dietary adjustment
ANSWER: D
A 33-year-old woman presents to the clinic with a positive home pregnancy test. The
patient has a history of Hashimoto’s thyroiditis, which has been successfully managed
with levothyroxine 125 µg daily for the past 4 years; serum thyroid-stimulating
hormone (TSH) levels have been between 0.5 and 1.5 µIU/mL (normal, 0.3-5.5 µIU/mL).
She has a family history of thyroid disease, and her mother also takes levothyroxine.
In the office, the pregnancy test is confirmed; this is her first pregnancy. To remain
biochemically and clinically euthyroid, how should this patient be managed?
A) Levothyroxine should be increased
B) Levothyroxine should be decreased
C) Triiodothyronine (T3) should be taken in addition to levothyroxine
D) The pregnancy should be terminated
E) None of above
ANSWER: A
A 34-year-old male presents with substernal discomfort. The symptoms are worse after
meals, particularly a heavy evening meal, and are sometimes associated with hot/sour
fluid in the back of the throat and nocturnal awakening. The patient denies difficulty
swallowing, pain on swallowing, or weight loss. The symptoms have been present for 6
weeks; the patient has gained 20 lb in the past 2 years. Your initial approach is
A) therapeutic trial of ranitidine
B) Exercise test with thallium imaging
C) Esophagogastroduodenoscopy
D) CT scan of the chest
E) Barium contrast study is indicated
ANSWER: A
A 34-year-old woman is evaluated because of a 1-year history of increased dyspnea on
exertion. She has no symptoms at rest but has to stop about 15 minutes into her
aerobics class because of dyspnea and occasional cough. She usually recovers fully in
about an hour. One year ago she was able to do aerobics for 45 minutes without
difficulty. Her vital signs are normal, and her physical examination is normal,
including clear breath sounds. Baseline spirometry is also normal. Which of the
following would be best next step in the management of this patient?
A) Inhaled albuterol prior to exercise
B) Oral leukotriene inhibitors
C) Long-acting theophylline
D) Inhaled ipratropium bromide prior to exercise
E) Inhaled corticosteroids
ANSWER: A
A 34-year-old woman is hospitalized after a minor episode of hematemesis. She denies
current use of nonsteroidal anti-inflammatory drugs. Two years ago, a duodenal ulcer
and Helicobacter pylori infection were diagnosed by upper endoscopy, and bismuth
subsalicylate, metronidazole, tetracycline, and ranitidine were prescribed for 14 days.
However, the patient took the medications for only 8 days because of nausea. Upper
endoscopy performed during the current admission shows a clean-based duodenal ulcer,
and rapid urease testing of an endoscopic mucosal biopsy specimen is positive for H.
pylori. Which of the following is the most appropriate management at this time?
A) Bismuth subsalicylate, metronidazole, tetracycline, and ranitidine for 4 days
B) Bismuth subsalicylate, metronidazole, amoxicillin, and ranitidine for 14 days
C) A proton pump inhibitor for 14 days
D) A proton pump inhibitor, clarithromycin, and amoxicillin for 14 days
E) Ranitidini (H2-histaminoblokers)
ANSWER: D
A 34-year-old woman is scheduled to undergo flexible sigmoidoscopy because of bloody
bowel movements of 4 days duration. She has had diarrhea for about 10 days, but 4 days
ago she developed frequent, low-volume bloody stools that are associated with urgency.
The patient has had episodic diarrhea for about 3 months, during which time she noted
and ignored bloody bowel movements on four occasions. Stool specimens obtained 3 days
ago are negative for pathogens. Sigmoidoscopic findings include inflammation from the
rectum to 20 cm with no areas of normal intervening mucosa. Which of the following is
the most likely diagnosis?
A) Ulcerative proctocolitis
B) Crohn’s disease
C) Irritable bowel syndrome
D) lschemic colitis
E) None of above
ANSWER: A
A 34-yr-old man presents with insidious onset weakness and weight loss. On examination,
he has hyperpigmentation of the palmar creases and postural hypotension. Choose the
most likely investigation
A) Dexamethasone suppression test
B) ACTH stimulation test
C) Fasting blood glucose
D) T3, T4 and TSH levels
E) Water deprivation test
ANSWER: B
A 34-yr-old woman points to an area of acute epigastric pain with his right index
finger. The pain is worse at night and taking food relieves it. Taking antacids also
relieves it. Choose the single most likely diagnosis
A) Gastric atrophy
B) Acute pancreatitis
C) Basal pneumonia
D) Chronic cholecystitis
E) Esophagitis
ANSWER: E
A 35 -year-old man complained of squeezed epigastric pain 1 hour after meal and
heartburn. She has been ill for 2 years. On palpation, there was moderate tenderness
in pyloroduodenal area. Antral gastritis was revealed on gastroscopy. Which study can
establish genesis of the disease?
A) USD of abdomen
B) Gastrin level in blood
C) Examination of stomach secretion
D) Examination of stomach motor function
E) Revealing of Helicobacter infection in gastric mucosa
ANSWER: E
A 35 -year-old patient complains on severe weakness and fever. Disease started with
pain in the throat and fever. Physical examination: skin and mucous are pale,
ecchymosis, purpura of skin and subcutaneous tissue. Peripheral blood smear: RBC - 1,5
х 1012 /L, Нв - 50 g/L. Blast – 90 %,. What is the most likely diagnosis?
A) Acute leukemia
B) Thrombocytopenic Purpura
C) Hypoplastic Anemia
D) AIDS
E) Leukemoid reaction
ANSWER: A
A 35-year-old man complains to a physician of chronic vague gastric pain of several
years' duration. The pain is sometimes relieved by food. Which of the following is the
most appropriate test to noninvasively determine the H. pylori?
A) Culture of gastric biopsy
B) Rapid urease test
C) Repeat qualitative IgA and IgG antibodies against H. pylori
D) Repeat quantitative IgA and IgG antibodies against H. pylori
E) Urea breath test
ANSWER: E
A 35-year-old man consults a gastroenterologist because of chronic heartburn for
several years. The heartburn tends to be worse at night, and he frequently tastes
refluxed gastric contents when he goes to bed. He found that his symptoms were a
little better when he avoided his customary late evening alcoholic drink; however,
this modest improvement has subsequently deteriorated. The gastroenterologist performs
esophageal manometry with pH monitoring, which demonstrates decreased pressure of the
lower esophageal sphincter and the presence of acid in the esophagus. Biopsy of the
proximal end of one of these fingers shows surface epithelium with regular columnar
cells with small, ovoid nuclei admixed with goblet cells. Which of the following is
the most likely diagnosis?
A) Achalasia
B) Barrett esophagus
C) Corrosive esophagitis
D) Esophageal adenocarcinoma
E) Zenker diverticulum
ANSWER: B
A 35-year-old patient with alcohol abuse and abnormal diet complains of pain in
epigastrium that occurs in 1-1,5 hours after having meal. Esophagogastroduodeno-scopy:
marked hyperemia, small defects, easy appearance of sores on the mucous membrane in
the antral section of the stomach. What is the most probable reason for this pathology?
A) Helicobacter pуlori infection
B) Presence of antibodies to parietal cells
C) Alimentary factor
D) Nervous overstrain
E) Toxic action of alcohol
ANSWER: A
A 35-year-old woman who underwent successful therapy for acute leukemia 10 years ago
is seropositive for hepatitis C. Polymerase chain reaction testing confirms the
presence of virus in a blood sample. She has mildly elevated serum transaminases. A
liver biopsy discloses moderately severe chronic hepatitis without fibrosis. The most
appropriate therapy is
A) interferon plus ribavirin
B) acyclovir
C) ribavirin
D) propranolol
E) no therapy is indicated
ANSWER: A
A 35-year-old woman with amenorrhea is found to have an enlarged pituitary glan. Her
prolactin level is 80 ng/L (normal less than 20 ng/L), and her thyrotropin level is
100 mlU/ml (normal, 0.5 to 4.5 mlU/ml). Which of the following is the best treatment
option for this patient?
A) Administration of bromocriptine
B) Administration of L-thyroxine
C) Irradiation of the pituitary gland
D) Resection of the pituitary gland
E) Use of oral contraceptives
ANSWER: B
A 35-yr-old woman with bone pain, drowsiness and thirst: Calcium—3.3, phosphate—0.75,
ALP—190, PTH—low-normal, PTH—activity high, glucose—6 mmol/l. Choose the most likely
diagnosis
A) Primary hyperparathyroidism
B) Tertiary hyperparathyroidism
C) Hypoparathyroidism
D) Hyperthyroidism
E) Hyperparathyroidism with ectopic PTH
ANSWER: E
A 36-yr-old woman presented with generalised pruritus for 5 months. On examination she
was tanned and there were spider naevi on her chest. The liver was palpable one finger
breadth below the costal margin as well as the tip of the spleen. Choose the single
most likely diagnosis from the list of options above.
A) Wilson's disease
B) Primary biliary cirrhosis
C) Cholecystitis
D) Haemochromatosis
E) Alcoholic liver cirrhosis
ANSWER: B
A 37-year-old man complains of pains in epigastrium which are relieved by food intake.
EGDS shows antral erosive gastritis, biopsy of antral mucous presents Hеlicobacter
Pylori. Diagnosis is:
A) Gastritis of type A
B) Reflux - gastritis
C) Menetrier's gastritis
D) Rigid antral gastritis
E) Gastritis of type B
ANSWER: E
A 37-year-old man with asthma is evaluated because he continues to have frequent
attacks and now feels his short-acting ?2-agonist is not providing relief. He states
he is using his medications, including a long-acting ?2-agonist inhaler, inhaled high-
dose corticosteroids, and a short-acting ?2-agonist inhaler as rescue medication. He
has symptoms daily and nocturnal symptoms about twice per week. On physical
examination, he is in mild respiratory distress. He is afebrile. Pulse rate is 90/min
and regular, respiration rate is 18/min, and blood pressure is 140/85 mm Hg. He has
bilateral wheezing. Spirometry shows a forced expiratory volume in 1 sec (FEV1) 65% of
predicted; it improves with bronchodilators to 85% of predicted. He has no history of
recent viral upper respiratory infections or rhinitis or symptoms of gastroesophageal
reflux disease. Which of the following is the best next step in this patient’s
management?
A) Observe the patient using the metered-dose inhaler
B) Add a leukotriene inhibitor
C) Switch to an oral ?2-agonist and have the patient return for a pill count
D) Initiate oral prednisone therapy and have the patient return for a pill count
E) Have the patient return with a symptom and treatment log.
ANSWER: A
A 37-year-old patient complains of epigastric pain that occurs in 1-1,5 hours after
having meal. Esophagogastroduodeno-scopy: marked hyperemia, small defects, easy
appearance of sores on the mucous membrane in the antral section of the stomach. What
is the most probable reason for this pathology?
A) Helicobacter pуlori infection
B) Presence of antibodies to parietal cells
C) Alimentary factor
D) Nervous overstrain
E) Toxic action of alcohol
ANSWER: A
A 37-year-old woman has a 4-month history of epigastric discomfort and heartburn.
Symptoms are usually exacerbated postprandially, especially if she eats spicy foods.
The patient denies dysphagia, weight loss, and decreased appetite. Treatment with a
proton pump inhibitor, once daily for 4 weeks, resulted in only minimal improvement.
Increasing the medication to twice daily for an additional 4 weeks did not improve her
symptoms, and the patient wants to know what other management options are available.
Referral for which of the following procedures is most appropriate at this time?
A) Ambulatory 24-hour esophageal pH monitoring
B) Upper endoscopy with esophageal dilation
C) Barium swallow
D) Surgical fundoplication
E) USD
ANSWER: A
A 37-year-old woman has a 4-month history of epigastric discomfort and heartburn.
Symptoms are usually exacerbated postprandially, especially if she eats spicy foods.
The patient denies dysphagia, weight loss, and decreased appetite. Treatment with a
proton pump inhibitor, once daily for 4 weeks, resulted in only minimal improvement.
Increasing the medication to twice daily for an additional 4 weeks did not improve her
symptoms, and the patient wants to know what other management options are available.
Referral for which of the following procedures is most appropriate at this time?
A) Upper endoscopy with esophageal dilation
B) Barium swallow
C) Surgical fundoplication
D) USD
E) Ambulatory 24-hour esophageal pH monitoring
ANSWER: E
A 37-year-old woman sees her physician because of gum bleeding, menorrhagia. Physical
examination reveals petechiae, bruises on her legs. CBC: RBC 2.5 х 1012/L, Hemoglobin
67 g/L, MCV 64 fl, CI – 0,7, Pl 41 х 109/L, Giant platelets +, WBС 7.9 х 109/L, ESR 7
mm/h, Bleeding time 17 min. What is the most probably diagnosis?
A) Henoch–Schonlein purpura
B) Thrombotic thrombocytopenic purpura
C) Chronic lymphocytic leukemia
D) Macrocytic hyperchromic anemia
E) Idiopathic thrombocytic purpura
ANSWER: E
A 37-yr-old woman presents with weight loss, muscular weakness, oligomenorrhea,
diarrhea and blurring of vision. On examination, there is exophthalmos and proximal
myopathy. Choose the most likely diagnosis
A) Thyroglossal cyst
B) Graves' disease
C) Hypothyroidism
D) Lymphoma
E) Simple goiter
ANSWER: B
A 38-year-old former hemodialysis nurse is seen because of a 6-month history of
fatigue and amenorrhea. On examination she has scleral icterus, a mildly tender liver,
and a tibial rash consistent with erythema nodosum. ALT and AST levels are both in the
range increased, while alkaline phosphatase and serum albumin levels are normal.
Hepatitis serologic testing detects HBsAg and IgG anti-HBcAg. Liver biopsy discloses a
mononuclear cell portal infiltrate and hepatocyte destruction at the periphery of
lobules. Which of the following therapeutic strategies is best?
A) Administration of interferon, 10 million units three times per week for 4 months
B) Administration of prednisone, 20 to 40 mg/d for 2 months and then a taper based on
the response
C) Administration of prednisone, 10 mg every other day for 3 months
D) Administration of acyclovir, 400 mg every 6 h for 2 weeks
E) Administration of low-dose cyclophosphamide, 50 mg/d for 2 months
ANSWER: A
A 38-year-old male insurance agent with a benign past medical history presents to his
primary care physician complaining of indigestion. He notes that “heartburn” has
occurred weekly for about 1 year, especially after eating a heavy meal. He has no risk
factors for coronary artery disease and does not complain of weight loss, vomiting,
dysphagia, or bleeding. Physical and routine laboratory examinations are unrevealing.
Which of the following is the most appropriate next step?
A) Upper gastrointestinal barium radiography
B) Upper gastrointestinal endoscopy
C) Ambulatory esophageal pH testing
D) Serology for H. Pylori
E) Prescribe omeprazole
ANSWER: D
A 38-year-old male insurance agent with a benign past medical history presents to his
primary care physician complaining of indigestion. He notes that “heartburn” has
occurred weekly for about 1 year, especially after eating a heavy meal. He has no risk
factors for coronary artery disease and does not complain of weight loss, vomiting,
dysphagia, or bleeding. Physical and routine laboratory examinations are unrevealing.
Which of the following is the most appropriate next step?
A) Upper gastrointestinal endoscopy
B) Ambulatory esophageal pH testing
C) Upper gastrointestinal barium radiography
D) Prescribe omeprazole
E) Serology for H. Pylori
ANSWER: E
A 38-year-old man complains of chronic heartburn for several years. The heartburn
tends to be worse at night, and he frequently tastes refluxed gastric contents when he
goes to bed. He found that his symptoms were a little better when he avoided his
customary late evening alcoholic drink; however, this modest improvement has
subsequently deteriorated. The esophageal manometry with pH monitoring demonstrates
decreased pressure of the lower esophageal sphincter and the presence of acid in the
esophagus. Biopsy of the proximal end of esophagus shows surface epithelium with
regular columnar cells with small, ovoid nuclei admixed with goblet cells. Which of
the following is the most likely diagnosis?
A) Achalasia
B) Barrett esophagus
C) Corrosive esophagitis
D) Esophageal adenocarcinoma
E) Zenker diverticulum
ANSWER: B
A 38-year-old man who works as a reporter for a travel magazine comes to his physician
because of the acute onset of jaundice, malaise, and temperatures to 38.5 С .He had
returned from Burma 2 weeks ago, where he spent 4 weeks. He says that he abstains from
alcohol beverages and does not take any medications. Laboratory studies show elevated
serum aminotransferases, high bilirubin (both total and direct), and negative serology
for hepatitis A virus (HAV) and С virus (HCV) infection. He was vaccinated for
hepatitis В virus (HBV) 3 years ago and is now positive for anti-HBsAg antibodies.
Which of the following serologic markers should be tested as the most appropriate next
step in diagnosis?
A) Anti-HCV IgG antibodies by RIBA
B) Anti-HDV IgG antibodies
C) Anti-HEV IgM antibodies
D) Anti-HGV IgG antibodies
E) HBsAg
ANSWER: C
A 39-year-old patient complains of weight loss, night sweating, general weakness,
fatigue, fever, gum bleeding. Bone marrow examination reveals blasts with Auer rods
87 %. What is the most likely diagnosis?
A) Acute nonlymphoblastic (myeloid) leukemia
B) Erythremia
C) Chronic lymphocytic leukemia
D) Chronic myeloid leukemia
E) Multiple myeloma
ANSWER: A
A 39-yr-old woman presented with jaundice and painless depigmented patches on her
hands, neck and face. On exami_nation multiple spider naevi were found. Choose the
single most likely diagnosis from the list of options above.
A) Haemochromatosis
B) Primary biliary cirrhosis
C) Wilson's disease
D) Chronic active hepatitis
E) Pancreatic carcinoma
ANSWER: D
A 40-year-old male with long-standing alcohol abuse complains of abdominal swelling,
which has been progressive over several months. He has a history of gastrointestinal
bleeding. On physical exam, there are spider angiomas and palmar erythema. Abdominal
collateral vessels are seen around the umbilicus. There is shifting dullness, and
bulging flanks are noted. An important first step in the patient’s evaluation is
A) Diagnostic paracentesis
B) UGI series
C) Ethanol level
D) CT scan
E) Esophagogastroduodenoscopy
ANSWER: A
A 40-year-old white female complains of pruritus. She has an elevated alkaline
phosphatase and positive antimitochondrial antibody test. What is the most likely
disease?
A) Primary biliary cirrhosis
B) Sclerosing cholangitis
C) Anaerobic liver abscess
D) Hepatoma
E) Hepatitis C
ANSWER: A
A 40-year-old woman, a nurse, is evaluated because of worsening asthma symptoms. She
has had mild, intermittent asthma since college, for which she has been using an
albuterol inhaler as needed, usually less than once a month. During the past 3 months,
she has experienced cough, tightness of the chest, and wheezing, which improve after
the use of inhaled albuterol. She uses the inhaler twice a day on average and has
awakened at least twice a week with nocturnal cough. She works three consecutive 12-
hour day shifts, and the cough is regularly worse at the end of each shift. During her
days off, she has fewer asthma symptoms and feels significantly better by the time she
returns to work. She has a history of allergic rhinitis that has also recently become
more symptomatic. Approximately 6 months ago, she acquired a kitten that sleeps in the
bedroom. She has lived in her home for 6 years, and it is carpeted and has heavy
draperies. Chest examination is notable for good air entry. There are scattered end-
expiratory wheezes. In addition to treatment with inhaled corticosteroids, which of
the following interventions is most likely to benefit this patient?
A) Avoiding exposure to latex products
B) Treatment with an oral antihistamine
C) Getting rid of the kitten
D) Removing the carpets and draperies from her home
E) Treatment with a leukotriene-modifying drug
ANSWER: A
A 40-yr-old diabetic actor is started on Propanolol for stage fright. He collapses
after a day shooting. He has not changed his insulin regimen. Choose the most likely
management.
A) Sugary drink
B) Insulin sliding scale, Heparin and 0.45% saline
C) Insulin sliding scale, 0.9% NS and potassium replacement
D) Chest X-ray
E) 50 ml of 50% dextrose IV
ANSWER: E
A 40-yr-old man presents to his GP complaining of change in appearance and headaches.
His brow is more prominent and his nose had broadened. He states that his shoes are
too small and he has tingling in certain fingers worse at night. Choose the most
likely management.
A) Propanolol
B) Calciferol
C) Carbimazole
D) Thyroxine
E) Octreotide (somatostatin)
ANSWER: E
A 40-yr-old woman has a long history of pruritus, arthralgia and mild jaundice. She
presents with haematemesis and is found to have splenomegaly. Endoscopy shows
oesophageal varices. Choose the single most likely diagnosis from the list of options
above. Choose the single most likely diagnosis from the list of options above.
A) Portal hypertension
B) Myelofibrosis
C) Infective endocarditis
D) None of above
E) Gilbert's syndrome
ANSWER: A
A 40-yr-old woman presents with a hard, nodular midline neck mass. Blood tests reveal
the presence of antibodies to thyroglobulin. Choose the most likely diagnosis.
A) Thyroglossal cyst
B) Hashimoto's thyroiditis
C) Toxic multinodular goiter
D) Thyroid storm
E) Graves' disease
ANSWER: B
A 40-yr-old woman presents with a solitary nodule in the right thyroid lob. FNAC
suggests follicular adenoma. Choose the most likely management.
A) Total thyroid lobectomy
B) Ablative dose of radioactive iodine
C) External beam radiation
D) Chemotherapy
E) Reassure and repeat FNAC in 1 yr
ANSWER: A
A 40-yr-old woman presents with fatigue, dyspnoea, paraesthesiae and a sore red tongue.
Her blood film shows hypersegmented polymorphs, an MCV of > 110 fl and a low Hb. What
is your diagnosis?
A) Pernicious anaemia
B) Iron-deficiency anemia.
C) Sickle cell anaemia
D) Hemolytic anemia.
E) Aplastic anemia.
ANSWER: A
A 41-year-old morbidly obese female comes to the emergency department with colicky
abdominal pain in her right upper abdomen. She complains that this is similar to, yet
more severe than, the pain that often occurs after meals for the past 4 months. Her
past medical history is positive for diabetes mellitus type 2, hypertension,
hyperlipidemia, and smoking. On physical exam, her temperature is 100.5°F (38.1°C)
and her sclera appear mildly icteric. What imaging modality may be limited in this
patient?
A) MRCP.
B) CT scan.
C) Ultrasonography.
D) Esophogastroduodenoscopy (EGD).
E) IDA scan.
ANSWER: C
A 42-year-old patient complains of weight loss, night sweating, general weakness,
fatigue. Physical examination reveals lymphadenopathy, splenomegaly. CBC reveals: RBC
2.34 х 1012/L, Hemoglobin 78 g/L, Hematocrit 40 %, MCV 90 fl, Reticulocyte Count 1.0 %,
Pl 245 х 109/L, WBC 71.9 х 109/L, Segmented Neutrophils 2 %, Band Neutrophils 0 %,
Monocytes -0% Eosinophils -0 %, Basophils -0%, Lymphocytes 98%, ESR 24mm/h,Smudge
cells +. What is the most likely diagnosis?
A) Acute leukemia
B) Chronic lymphocytic leukemia III st
C) Chronic lymphocytic leukemia I st
D) Chronic myeloid leukemia
E) Chronic lymphocytic leukemia IV st
ANSWER: B
A 42-year-old patient complains of weight loss, night sweating. CBC reveals: RBC 3.75
х 1012/L, Hb 125 g/L, Ht 42 %, MCV 91 fl, -Reticulocyte Count 1.0 %, Pl 280 х 109/L,
WBC 47.9 х 109/L, Segmented Neutrophils 3 %, Band Neutrophils 1 %, Monocytes 2 %,
Eosinophils 1 %, Basophils 0 %, Lymphocytes 93 %, ESR 7 mm/h, Smudge cells +. What is
the most likely diagnosis?
A) Acute leukemia
B) Erythremia
C) Chronic lymphocytic leukemia
D) Chronic myeloid leukemia
E) Multiple myeloma
ANSWER: C
A 42-year-old patient complains of weight loss, night sweating. Physical examination
reveals lymphadenopathy, splenomegaly. CBC reveals: RBC 3.75 х 1012/L, Hb 125 g/L, Ht
42 %, MCV 91 fl, -Reticulocyte Count 1.0 %, Pl 280 х 109/L, WBC 47.9 х 109/L,
Segmented Neutrophils 3 %, Band Neutrophils 1 %, Monocytes 2 %, Eosinophils 1 %,
Basophils 0 %, Lymphocytes 93 %, ESR 7 mm/h, Smudge cells +. What is the most likely
diagnosis?
A) Acute leukemia
B) Chronic lymphocytic leukemia III st
C) Chronic lymphocytic leukemia II st
D) Chronic myeloid leukemia
E) Chronic lymphocytic leukemia IV st
ANSWER: C
A 42-year-old patient suffering from alco_holism has advanced liver disease with
ascites. Examination reveals asterixis of the hands, ankle clonus, and spider angiomas
on the face and chest. Precipitat_ing factors to look for include all of the
fol_lowing EXCEPT:
A) heart insufficiency
B) bleeding esophageal varices
C) excessive diuretic therapy
D) non-compliance with lactulose therapy
E) spontaneous bacterial peritonitis
ANSWER: A
A 42-yr-old man has hypertension, hyperglycaemia, myopathy, thinning of the skin,
buffalo hump and truncal obesity. Choose the most likely investigation
A) Dexamethasone suppression test
B) ACTH stimulation test
C) Fasting blood glucose
D) T3, T4 and TSH levels
E) Water deprivation test
ANSWER: A
A 44-yr-old mother of five children complained of yellow skin and abdominal pain,
especially after meals. She was overweight and she said that she did not like going
out to restaurants because of embarrassing flatulence. Choose the single most likely
diagnosis from the list of options above.
A) Gallstones
B) Hepatitis
C) Carcinoma of bile duct
D) Alcoholism
E) Carcinoma pancreas
ANSWER: A
A 44-yr-old woman complains of intense pruritus and yellowing of her skin. On physical
examination you notice xanthomata and skin pigmentation. Choose the single most likely
diagnosis from the list of options above.
A) Primary biliary cirrhosis
B) Hepatitis
C) Carcinoma of bile duct
D) Alcoholism
E) Haemolytic anaemia
ANSWER: A
A 44-yr-old woman presents with tachycardia, atrial fibrillation, double vision and
swelling above her ankles. She has lid lag on examination. Choose the most likely
investigation
A) Dexamethasone suppression test
B) ACTH stimulation test
C) Fasting blood glucose
D) T3, T4 and TSH levels
E) Water deprivation test
ANSWER: D
A 45 year-old man undergoes a routine physical examination with screening blood
studies. Physical examination is notable for an increased liver diameter; the liver
edge is palpable and without irregularities. Blood studies show elevated liver enzymes.
The clinician suspects alcoholic hepatitis. Which of the following findings would tend
to support this diagnosis?
A) Alanine aminotransferase = 2000 U/L
B) Aspartate aminotransferase (AST)/alanine aminotransferase (ALT) ratio = 2.5
C) Gamma-glutamyl transferase (GGT) = 20 U/L (norm
D) Mean corpuscular volume (MCV) = 65 urn3
E) Platelet count = 600,000/mm3
ANSWER: B
A 45-year-old female with long-standing alcohol abuse complains of abdominal swelling,
which has been progressive over several months. On physical exam, there are spider
angiomas and palmar erythema. Abdominal collateral vessels are seen around the
umbilicus. A paracentesis is performed. The serum albumin minus ascitic fluid albumin
equals 1.4 g/dL. The most likely diagnosis is
A) Portal hypertension
B) Pancreatitis
C) Tuberculous peritonitis
D) Hepatoma
E) No ascitis
ANSWER: A
A 45-year-old man for 1 month has complained of epigastric and right subcostal aching
pain, pruritus, indigestion, dark color of the urine and acholic stool, fever, and
significant weight loss. On exam: jaundice, presence of Curvuasier’s sign. US scan did
not reveal stones in the gallbladder and choledochus. What is the most likely
diagnosis?
A) Cancer of the pancreas head
B) Gallbladder stones
C) Chronic pancreatitis
D) Chronic cholangitis
E) Chronic hepatitis
ANSWER: A
A 45-year-old man with history of high blood pressure that has been difficult to
control with a variety of antihypertensive medications presents with persistent
headaches, excessive sweating, and palpitations. Routine blood tests are normal.
Electrocardiogram shows supraventricular tachycardia. What is this patient’s most
likely diagnosis?
A) Pheochromocytoma
B) Primary hyperaldosteronism
C) Cushing’s disease
D) Hyperthyroidism
E) Malignant hypertension
ANSWER: A
A 45-year-old man with history of high blood pressure that has been difficult to
control with a variety of antihypertensive medications presents with persistent
headaches, excessive sweating, and palpitations. Routine blood tests are normal.
Electrocardiogram shows supraventricular tachycardia. It was suspected
pheochromocytoma. What is this patient’s most likely test?
A) cortisol level
B) aldosteron level
C) 17-hydroxycorticosteroids
D) urine metanephrines
E) TSH
ANSWER: D
A 45-year-old man with history of high blood pressure that has been difficult to
control with a variety of antihypertensive medications presents with persistent
headaches, excessive sweating, and palpitations. Routine blood tests are normal.
Electrocardiogram shows supraventricular tachycardia. Which of the following is the
best initial pharmacologic therapy choice for this patient?
A) ?-Blockers
B) Diuretics
C) Steroids
D) Methimazole
E) ?-Blockers
ANSWER: E
A 45-year-old woman complains of right upper quadrant pain, which occurs after she
eats a large meal. Occasionally the episodes are accompanied by nausea and vomiting. A
plain x-ray of the abdomen discloses gallstones. Ultrasonography reveals gallstones
and a normalsized common bile duct. The patient’s blood chemistry and CBC are normal.
The most therapeutic maneuver at this time would be
A) laparoscopic cholecystectomy
B) observation
C) ursodeoxycholic acid
D) shock wave lithotripsy
E) ursodeoxycholic acid and shock wave lithotripsy
ANSWER: A
A 45-yr-old patient complains of giddiness and falls. She also suffers with
intermittent vomiting and sweating and occasional faecal incontinence at night. Choose
the most likely diagnosis
A) Coronary artery disease
B) Cerebrovascular disease
C) Nephropathy
D) Retinopathy
E) Autonomic neuropathy
ANSWER: E
A 45-yr-old woman presents with a diffuse swelling of the thyroid gland. On
examination she has a stare, lid lag, and lid retraction. On the dorsum of her legs
she has areas of raised peau d'orange-like thickened skin. Blood tests reveal thyroid-
stimulating immunoglobulins against the TSH receptor site. Choose the most likely
diagnosis.
A) Thyroglossal cyst
B) Hashimoto's thyroiditis
C) Toxic multinodular goiter
D) Thyroid storm
E) Graves' disease
ANSWER: E
A 46-year-old woman who works as a nurse is evaluated because of a 2-year history of
episodic wheezing and a squeaky voice. This past spring, her symptoms worsened,
requiring her to seek medical attention; she was placed on a short-acting ?2-agonist
that did not provide much relief. She has no history of wheezing and says that these
changes began after a severe influenza infection 3 years ago. Currently she feels well
and has had no symptoms for several months; she is not taking any medications.
Physical examination shows no abnormalities, and baseline spirometry is normal. What
is the best test to evaluate this patient’s condition?
A) Methacholine challenge testing
B) Bronchoscopy to evaluate her trachea
C) Exercise echocardiogram
D) CT scan of the sinuses
E) Non of above
ANSWER: A
A 47-year-old female presents to the health clinic for symptoms of weight she can't
seem to lose despite strict dieting and concerns of menstrual irregularity. She states
she also has developed multiple abdominal stretch marks and has noticed a deepening of
her voice and appearance of facial hair. As part of her initial work up, you obtain a
24-hour urine cortisol level, which is returned as 120 ng/ml (N 20 100 ng/ml). What is
the most likely diagnosis in this patient?
A) Metabolic syndrome.
B) Diabetes mellitus type II.
C) Cushing's syndrom
D) Addison's diseas
E) Conn's syndrom
ANSWER: C
A 47-year-old female presents to the health clinic for symptoms of weight she can't
seem to lose despite strict dieting and concerns of menstrual irregularity. She states
she also has developed multiple abdominal stretch marks and has noticed a deepening of
her voice and appearance of facial hair. As part of her initial work up, you obtain a
24-hour urine cortisol level, which is returned as 120 ng/ml (N 20 100 ng/ml). The
most likely diagnosis in this patient is Cushing's syndrome.What is the most
appropriate next step in diagnosis of this patient?
A) Cosyntropin (ACTH) stimulation test.
B) Dexamethasone suppression test.
C) Cosyntropin (ACTH) suppression test.
D) Dexamethasone stimulation test.
E) Urine catecholamine collection over a 24-hour perio
ANSWER: B
A 47-yr-old agricultural worker complains of a chronic cough, purulent sputum and
abdominal distention. He has just arrived in England from Spain where he was picking
grapes. Choose the single most likely diagnosis from the list of options above.
A) Tuberculosis
B) Cirrhosis
C) Malabsorption
D) Pancreatitis
E) Peptic ulcer
ANSWER: A
A 48 years old patient, complaints on weakness, dyspnea, pain in the left half of
thorax, permanent cough with viscid sputum, in which particles of blood are sometimes
determined. For the last 3 months lost 5 kg of body mass. On the X-ray of lungs there
is total homogeneous shade determined from the left side. Organs of mediastinum are
displaced to the left. What diagnosis is possible?
A) Lung athelectasis
B) Lung gangrene
C) Total exudative pleurisy
D) Pneumonia
E) Empyema of pleura
ANSWER: A
A 48-year-old patient complains of fatigue and easy bruising of 3 weeks’ duration.
Physical findings included pale, scattered ecchymoses and petechiae. RBC – 1.95 x
1012/L; Hb – 73 g/L; HCT 20%; PLT – 23 x 109/L; and WBC – 182 x 109/L with 84% blasts,
that contained Auric rods; peroxidase stain was positive; What is the most probable
diagnosis?
A) Megaloblastic anemia
B) Hemolytic anemia
C) Thrombocytopenia
D) Acute myeloid leukemia
E) Chronic leukemia
ANSWER: D
A 48-year-old woman develops fevers, chills, and icteric sclera. In addition to a
fever of 39.2_C, the physical examination is remarkable for an ill-appearing jaundiced
female with right upper quadrant pain. Ultrasonography reveals a dilated common bile
duct with stones in the gallbladder and in the duct itself. The patient is placed on
broad-spectrum antibiotics to cover organisms known to infect the biliary tract. The
procedure most appropriate now is
A) endoscopic retrograde cholangiopancreatography
B) laparoscopic cholecystectomy
C) placement of an external stent for bilary drainage
D) laparotomy to canulate the common bile duct, remove the stone, and perform a
cholecystectomy
E) antibiotics for several days
ANSWER: A
A 49 years old woman complains of weakness, malaise, anorexia, fever, dental bleeding.
Petechiae on a skin. Laboratory findings: Hb of 70 g/L, RBC of 2.2 x 1012/L, the
platelet count is 30 x 109/L, the WBC 100,5 x 109/L, blasts in peripheral blood smear.
What is the most probable diagnosis?
A) Megaloblastic anemia
B) Acute leukemia
C) Hemolytic anemia
D) Thrombocytopenia
E) Chronic leukemia
ANSWER: B
A 49-yr-old man who enjoys drinking presents with pallor, epistaxis and bleeding. On
physical examination you find the spleen to be enlarged and the liver to be slightly
enlarged. Choose the single most likely investigation from the list of options above.
A) Liver function test
B) Abdominal USG
C) Bone marrow biopsy
D) Blood culture
E) White cell count
ANSWER: A
A 49-yr-old woman presents with goiter. On examination, the thyroid is firm and
rubbery. Thyroid microsomal antibodies are positive in high titer. Choose the most
likely diagnosis
A) Thyroglossal cyst
B) Hashimoto's thyroiditis
C) Hypothyroidism
D) Graves' disease
E) Simple goiter
ANSWER: B
A 50 years old woman complains of weakness, anorexia, fever, gums bleeding. Spleen,
liver, and lymphatic nodes are enlarged, petechiae on a skin. Laboratory findings: the
platelet count is 90 x 109/L, WBC 100 x 109/L, blasts 40%. ? What is the most probable
diagnosis?
A) Acute leukemia
B) Megaloblastic anemia
C) Hemolytic anemia
D) Thrombocytopenia
E) Chronic leukemia
ANSWER: A
A 50-year-old man without significant past medical history or recent exposure to
alcohol presents with midepigastric abdominal pain, nausea, and vomiting. The physical
examination is remarkable for the absence of jaundice and any other specific physical
findings. Which of the following is the best strategy for screening for acute
pancreatitis? Measurement of both serum amylase and serum
A) Lipase
B) Measurement of serum lipase
C) Measurement of serum amylase
D) Isoamylase level analysis
E) None of above
ANSWER: A
A 50-yr-old patient complains of burning pain in the feet, worse at night or on
walking. He describes the sensation as like walking on hot coals. Choose the most
likely diagnosis
A) Coronary artery disease
B) Cerebrovascular disease
C) Nephropathy
D) Retinopathy
E) Polyneuropathy
ANSWER: E
A 50-yr-old woman presents to her GP for fatigue, depression and weight gain. She also
complains of constipation and poor memory. On examination, she has puffy face and
coarse facial features, thin eyebrows and a large tongue. Choose the most likely
management
A) Propanolol
B) Calciferol
C) Carbimazole
D) Thyroxine
E) Octreotide (somatostatin)
ANSWER: D
A 50-yr-old woman presents with a thyroid goiter. A core biopsy reveals evidence of
lymphoma. Choose the most likely management.
A) Total thyroid lobectomy
B) Thyroxine
C) External beam radiation
D) Propranolol
E) Reassure and repeat FNAC in 1 yr
ANSWER: C
A 50-yr-old woman presents with fever, tachycardia restlessness, hypertension and
vomiting. On examination she has diffuse swelling of the thyroid gland and strabismus
with diplopia. Choose the most likely diagnosis.
A) Thyroglossal cyst
B) Hashimoto's thyroiditis
C) Toxic multinodular goiter
D) Thyroid storm
E) Graves' disease
ANSWER: D
A 51-year-old female comes to the emergency department complaining of left lower
quadrant abdominal pain. She describes an acute illness accompanying the pain with
subjective fever and diarrhea over the last 8 hours. Abdominal exam shows tenderness
in the LLQ of the abdomen, no rebound tenderness at McBurney's point, and negative
Murphy's sign. What imaging modality is most appropriate for this patient?
A) Ultrasound of the abdomen.
B) CT scan with and without contras
C) Colonoscopy.
D) Barium enema.
E) Plain upright abdominal x-ray.
ANSWER: B
A 51-year-old woman undergoes colorectal cancer screening. She feels well, has no
significant medical history, takes no medications, and has no family history of
colorectal cancer. Physical examination and complete blood count are normal. Which of
the following is the most appropriate screening program for colorectal cancer in this
patient?
A) Colonoscopy every 10 years
B) Flexible sigmoidoscopy every 2 to 3 years
C) Barium enema examination every 3 years
D) Fecal occult blood testing every 2 to 3 years
E) CT colonography (virtual colonoscopy) every 10 years
ANSWER: A
A 52-yr-old man has been gaining weight. He complains of a chronic cough, acne and
bruising. On physical examination you find his legs and arms to be abnormally thin.
Choose the most likely investigation
A) Chest X-ray
B) Serum cortisol
C) Dexamethasone suppression test
D) Water deprivation test
E) T3, T4 and TSH
ANSWER: C
A 52-yr-old man presents with painless lump in the neck and a chronic cough. Physical
examination finds tachycardia and pallor. He feels that he has lost weight, but he is
not certain. He does not smoke or drink. Choose the most likely diagnosis.
A) Toxic adenoma
B) Adrenal hyperplasia
C) Hyperthyroidism
D) Hypothyroidism
E) Follicular carcinoma
ANSWER: E
A 53-year-old woman with a history of mild persistent asthma is evaluated because of a
recent increase in her symptoms, with dyspnea and cough occurring daily and a cough
that awakens her once a week. She is currently using low-dose inhaled corticosteroids.
She has no symptoms of rhinitis or gastroesophageal reflux. On physical examination,
she has intermittent wheezing bilaterally. Which of the following is the most
appropriate change in her therapy?
A) Add a long-acting ?2-agonist
B) Initiate azithromycin therapy
C) Add a nebulized short-acting ?2-agonist
D) Add inhaled ipratropium bromide
E) Add a leukotriene inhibitor
ANSWER: A
A 55-year-old male alcoholic has recurrent attacks of severe mid-epigastric pain after
eating. Serum amylase determinations after such attacks have been in the normal range.
The examination reveals mild cachexia but is otherwise unremarkable. On further
questioning, the patient states that he has been sober for the past 10 years but prior
to that time had multiple episodes of alcohol-induced pancreatitis. He is currently
taking pancreatic replacement enzymes by mouth. An ERCP reveals a stricture of the
pancreatic duct but is otherwise unremarkable. Computed tomography of the abdomen
reveals calcifications in the pancreas but does not show any evidence of malignancy.
The patient is taking 30 mg of continuous-release morphine sulfate twice a day. The
best strategy at this point would be to
A) resect the head of the pancreas
B) double the dose of pancreatic replacement enzymes
C) double the dose of morphine
D) institute a low-fat diet
E) begin a continuous search for other causes ofabdominal pain
ANSWER: A
A 55-year-old man is evaluated in the emergency department because of an acute, severe
asthma attack; he is hospitalized in the intensive care unit for aggressive medical
therapy and monitoring. He is expectorating thick greenish sputum. His medical history
includes hypertension, cholecystectomy and glaucoma. Chest radiograph reveals
hyperinflation only. Medical therapy in the emergency department included repeated
doses of aerosolized albuterol and ipratropium, as well as methylprednisolone, 125mg
administered intravenously. Peak expiratory flow rate is unimproved at 80 l/min. Which
of the following is the most appropriate next step in this patient’s management?
A) Intravenous magnesium sulfate
B) Nebulized ipratropium bromide administered by face mask
C) Broad-spectrum antibiotics targeting community-acquired respiratory pathogens
D) Inhaled corticosteroids
ANSWER: A
A 55-year-old man with no prior medical history presents with right flank pain. During
an evaluation for kidney stones, he has a computed tomography scan of the abdomen that
reveals a 2 ? 2-cm mass in the left adrenal gland. The patient’s vital signs, kidney
function, and electrolyte levels are within normal limits. Which of the following
tests should be ordered next to evaluate the adrenal mass?
A) Aldosterone and renin levels
B) Random cortisol levels
C) Testosterone levels
D) 24-Hour urine collection for metanephrines
E) No further testing
ANSWER: D
A 55-yr-old man presents with fever, sweats and weight loss. He also suffers from gout
and enlargement of lymphatic nodes, which are nontender, symmetrical. On examination
he has an enlarged spleen and liver. Blood tests reveal a lymphocytosis and anaemia.
The Philadelphia chromosome is negative. What is the most probable diagnosis?
A) ALL
B) CLL
C) Multiple myeloma
D) Polycythemia vera
E) CML
ANSWER: B
A 55-yr-old presented with following reports on a routine screen: Calcium-2.85 mmol/l,
phosphate—0.8, ALP—110, PTH—raised, 25-OH vitamin D—low-normal. Choose the most
likely diagnosis
A) Primary hyperparathyroidism
B) Tertiary hyperparathyroidism
C) Hypoparathyroidism
D) Hyperthyroidism
E) Paget's disease of bone
ANSWER: A
A 56-year-old patient with cirrhosis of the liver presents with massive hemetemesis.
Two large-bore intravenous lines are placed; somatostatin, fluids, and blood products
are administered; and the patient is intubated. Emergency endoscopy reveals bleeding
esophageal varices. The patient becomes stable hemodynamically but is still bleeding.
The most appropriate next step is
A) endoscopic variceal band ligation
B) intravenous vasopressin
C) balloon tamponade
D) endoscopic injection sclerotherapy
E) intravenous propranolol
ANSWER: A
A 57-year-old man with severe persistent asthma is evaluated on routine follow-up. He
states that his asthma has been under good control for the last 3 months on high-dose
inhaled corticosteroids and a long-acting ?2-agonist. He uses a short-acting ?2-
agonist only three times per week as a rescue medication, and he has nocturnal
symptoms very rarely. Peak expiratory flows have been stable. His physical examination
is normal, including clear breath sounds, and spirometry is normal. It is decided that
he will keep using the short-acting ?2-agonist as a rescue medication. Which of the
following is the best next step in this patient’s management?
A) Continue the long-acting ?2-agonist and decrease the dose of inhaled corticosteroid
B) Stop the long-acting ?2-agonist and decrease the dose of inhaled corticosteroid
C) Continue current therapy and have the patient return in 6 months
D) Stop the long-acting ?2-agonist and maintain the dose of inhaled corticosteroid
ANSWER: A
A 58-year-old patient complains of weight loss, night sweating, sensation of fullness
in left hypochondrium. CBC reveals: RBC 3.9х 1012/L, Hb 118g/L, Ht 41%, MCV 88 fl, Pl
645 х 109/L, WBC 89,4х 109/L, Segmented Neutrophils 62%, Band Neutrophils 10%,
Metamyelocytes 3%, Myelocytes 5%, Monocytes 4%, Eosinophils 8%, Basophils 6%,
Lymphocytes 2%, ESR 27mm/h. What is the most likely diagnosis?
A) Acute leukemia
B) Chronic myeloid leukemia, accelerated phase
C) Chronic lymphocytic leukemia
D) Chronic myeloid leukemia, chronic phase
E) Multiple myeloma
ANSWER: D
A 58-year-old woman complains of increasing fatigue and easy bruising of 3 weeks
duration. Physical findings included pale, scattered ecchymoses and petechiae and mild
hepatosplenomegaly. CBC: RBC – 2.55 x 1012/L; Hb – 73 g/L; HCT 20%; PLT – 23 x 109/L;
and WBC – 162 x 109/L with 82% blasts; peroxidase stain is positive. What is the most
probable diagnosis?
A) Megaloblastic anemia
B) Hemolytic anemia
C) Thrombocytopenia
D) Acute leukemia
E) Chronic leukemia
F) Chronic hemolytic anemia
ANSWER: D
A 58-yr-old man was diagnosed with diabetes at a routine medical examination 3 months
ago. His BMI is 32 despite losing 5 kg by following the dietician's advice. His home
blood glucose readings range from 7 to 11 and his HbAlc is 10%. Choose the most likely
management.
A) Metformin
B) No change in treatment required
C) IV insulin sliding scale
D) Gliclazide
E) Dietary adjustment
ANSWER: A
A 58-yr-old woman presents with fever, sweats and weight loss. On examination she has
an enlarged spleen. Blood tests reveal a lymphocytosis and anaemia. The Philadelphia
chromosome is negative. What is the diagnosis?
A) ALL
B) Multiple myeloma
C) CLL
D) Polycythemia rubra vera
E) CML
ANSWER: C
A 59-yr-old man presents with obstructive jaundice. USG shows no gallstones. The liver
appears normal and the common bile duct measures 12 mm in diameter. His past medical
history includes partial gastrectomy 15 yr ago for peptic ulcer. Choose the single
most likely investigation from the list of options above.
A) Percutaneous transhepatic
B) Cholangiography
C) MRI scan
D) Barium follow through
E) CTscan
ANSWER: B
A 60-year-old alcoholic man is admitted to the emergency department with hematemesis.
His pulse is 110/min, blood pressure is 100/60 mm Hg, and respirations are 19/min. He
has multiple spider angiomata on his back and chest, with bilateral gynecomastia.
Abdominal examination is significant for hepatosplenomegaly, and his abdomen is
distended and tympanic on percussion; a fluid level is easily detectable. His
testicles are small, and a rectal examination produces guaiac-negative stool. His
hematocritis 23%. After placement of a nasogastric tube, 400 mL of bright red blood is
evacuated. After initial fluid resuscitation, which of the following is the most
appropriate next step in management?
A) Barium swallow
B) Esophageal balloon tamponade
C) Esophagogastroscopy
D) Exploratory celiotomy
E) Selective angiography
ANSWER: C
A 60-year-old man with biopsy-proven hepatic cirrhosis is hospitalized because of
massive ascites and pedal edema. There is no evidence of respiratory compromise or
hepatic encephalopathy. Bed rest, sodium and water restriction, and the administration
of spironolactone produce no significant weight change after 5 days. Which of the
following therapeutic measures would be most appropriate at this time?
A) Therapeutic paracentesis
B) None of them
C) Oral acetazolamide, 250 mg/d
D) Placement of a peritoneovenous shunt
E) Intravenous furosemide, 80 mg now
ANSWER: A
A 60-yr-old man is brought to A&E in an unconscious state. His glucose is 35 mmol/l.
His arterial blood gas shows a pH of 7.2 and a PaCO2 of 28 mm Hg. Serum Na is 140, K
is 3.0, Cl is 100 and HCO3" is – 18 mEq/L. Choose the most likely management.
A) Sugary drink
B) Insulin sliding scale, Heparin and 0.45% saline
C) Insulin sliding scale, 0.9% NS and potassium replacement
D) Chest X-ray
E) 50 ml of 50% dextrose IV
ANSWER: C
A 61-year-old patient complains of weight loss, night sweating, general weakness,
fatigue, sensation of fullness in left hypochondrium. CBC reveals: RBC 4.0х 1012/L, Hb
88g/L, Hematocrit 34%, MCV 88 fl, Pl 98 х 109/L, WBC 125,4х 109/L, Segmented
Neutrophils 35%, Band Neutrophils 8%, Metamyelocytes 6%, Myelocytes 5%, Myeloblasts
10%, Monocytes 4%,Eosinophils 8% Basophils 22%, Lymphocytes 2%, ESR mm/h 27mm/h. What
is the most likely diagnosis?
A) Acute leukemia
B) Chronic myeloid leukemia, accelerated phase
C) Chronic lymphocytic leukemia
D) Chronic myeloid leukemia, chronic phase
E) Multiple myeloma
ANSWER: B
A 61-year-old white man has a 2-month history of dysphagia for solid foods. He has
lost 6.7 kg during this time. The patient has chronic heartburn that is relieved by
antacids. He also has hypertension for which he takes atenolol and diltiazem. Which of
the following is the most likely diagnosis
A) Esophageal adenocarcinoma
B) Pill-induced esophagitis
C) Esophageal web
D) diffuse esophageal spasm
E) None of above
ANSWER: A
A 64-year-old patient complains of weight loss, night sweating, general weakness,
fatigue, nose bleeding. Physical examination reveals lymphadenopathy, ecchymosis. CBC
reveals: RBC 3.00 х 1012/L, Hb 98 g/L, Ht 40 %, MCV 98 fl, Reticulocyte Count 1.0 %,
Pl 45 х 109/L, WBC 61.9 х 109/L, Segmented Neutrophils 2 %, Band Neutrophils 1 %,
Monocytes -0% Eosinophils -0 %, Basophils -0%, Lymphocytes 97%, ESR 24mm/h, Smudge
cells ++. What is the most likely diagnosis?
A) Acute leukemia
B) Chronic lymphocytic leukemia III st
C) Chronic lymphocytic leukemia I st
D) Chronic myeloid leukemia
E) Chronic lymphocytic leukemia IV st
ANSWER: E
A 65-yr-old man has had type 2 diabetes for 4 years, for which he was taking
Chlorpropamide. He presents with an acute MI and his laboratory blood glucose is 11
mmol/l. Choose the most likely management.
A) Metformin
B) No change in treatment required
C) IV insulin sliding scale
D) Gliclazide
E) Dietary adjustment
ANSWER: C
A 65-yr-old patient is becoming increasingly confused. She has periods where her
confusion seems to be stable and then seems to rapidly deteriorate in a stepwise
progression. On examination there are extensor plantars but leg reflexes are
diminished. Choose the most likely diagnosis
A) Coronary artery disease
B) Cerebrovascular disease
C) Nephropathy
D) Retinopathy
E) Polyneuropathy
ANSWER: B
A 66-year-old man complains of fever, significant weight loss, bone and joint pain,
and bleeding gums. On examination - paleness, lymphadenopathy, hepato- and
splenomegaly. WBC – 170 x 109/L with 13% lymphocytes, 1% monocytes, 21% basophiles,
29% neutrophils, 10% blasts, 12% promyelocytes, 12% myelocytes, 2% metamyelocytes. ESR
– 22 mm/h. The Philadelphia chromosome positive. Which agent is used for induction or
consolidation therapy?
A) Prednisolone
B) Сytosar
C) Gatifloxacin
D) Vinblastine
E) Alendronic acid
ANSWER: B
A 67-year- male presents with a complaint of fatigue. There is no history of alcohol
abuse or liver disease. Scleral icterus is noted on physical exam. The liver and
spleen are nonpalpable. The patient is noted to have a normocytic, normochromic anemia.
The first step in evaluation of this patient is
A) CT scan of the abdomen
B) Hepatitis profile
C) Liver function tests, including direct versus indirect bilirubin and urine
bilirubin
D) Abdominal ultrasound
E) Percutaneous transhepatic Cholangiography
ANSWER: C
A 67-year-old alcoholic man is admitted to the emergency department with hematemesis.
His pulse is 100/min, blood pressure is 100/60 mm Hg, and respirations are 19/min.
Abdominal examination is significant for hepatosplenomegaly, and his abdomen is
distended and tympanic on percussion; a fluid level is easily detectable. His
testicles are small, and a rectal examination produces guaiac-negative stool. His
hematocritis 24%. After placement of a nasogastric tube, 420 mL of bright red blood is
evacuated. After initial fluid resuscitation, which of the following is the most
appropriate next step in management?
A) Barium swallow
B) Esophageal balloon tamponade
C) Esophagogastroscopy
D) Exploratory celiotomy
E) Transjugular intrahepatic portosystemic shunt
ANSWER: C
A 67-year-old male presents with conjugated hyperbilirubinema, with bilirubin detected
in the urine. Serum bilirubin is 12 mg/dL, AST and ALT are in normal range, and
alkaline phosphatase is 300 U/L (3 times normal). The next step in evaluation is
A) Ultrasound or CT scan
B) Hepatitis profile
C) Reticulocyte count
D) Family history for hemochromatosis
E) Colonoscopy
ANSWER: A
A 67-year-old woman had her first colonoscopy 1 month ago for routine colorectal
cancer screening. A 6-mm tubular adenoma of the sigmoid colon was removed. She has no
family history of colorectal cancer. She asks what can be done to decrease her risk of
developing colorectal cancer. Which of the following is the most appropriate
surveillance for this patient
A) Repeat colonoscopy in 5 year
B) Repeat colonoscopy in 1 years
C) Aspirin, 81 mg daily
D) A high-fiber, low-fat diet
E) None of above
ANSWER: A
A 67-yr-old man is noted to have a glucose level of 37 mmol/l and a Na+ of 163 mmol/l.
He has no prior history of diabetes and has been on IV fluids for a week. His other
medications include IV Cefuroxime, Metronidazole, and Dexamethasone. Choose the most
likely management.
A) Insulin sliding scale, Heparin and 0.9% saline
B) Insulin sliding scale, Heparin and 0.45% saline
C) Sugary drink
D) Chest X-ray
E) 50 ml of 50% dextrose IV
ANSWER: B
A 68-year-old man is evaluated because of nausea, vomiting, and upper abdominal pain
and distention of 2 days duration. He has no fever, chills, or jaundice. On physical
examination, he appears uncomfortable and has orthostatic hypotension. Abdominal
examination discloses distention, tympany on percussion, and rushes on auscultation.
Serum aspartate aminotransferase is 0,34 U/L, serum alanine aminotransferase is 0,61
U/L, and serum total bilirubin is 19,6 mcmol/L. Plain radiographs of the abdomen show
pneumobilia with multiple air-fluid levels in the jejunum. No free air is seen.
Abdominal ultrasonography shows four gallstones measuring 2 to 4 cm. Because of
pneumobilia, the biliary tree cannot be further visualized. Which of the following is
the most appropriate next step in this patient’s management?
A) Exploratory laparotomy for bowel obstruction
B) Cholecystectomy
C) Endoscopic retrograde cholangiopancreatography
D) CT scan of the abdomen
E) Magnetic resonance cholangiopancreatography
ANSWER: A
A 68-year-old patient complains of weight loss, night sweating, sensation of fullness
in left hypochondrium. CBC reveals: RBC 3.9х 1012/L, Hb 128g/L, Ht 41%, MCV 78fl,
Reticulocyte Count 1.0 %, Pl 945 х 109/L, WBC 125,4х 109/L, Segmented Neutrophils 62%,
Band Neutrophils 10%, Metamyelocytes 3%, Myelocytes 5%, Monocytes 4%, Eosinophils 8%,
Basophils 6%, Lymphocytes 2%, ESR 27mm/h. What is the most likely diagnosis?
A) Acute leukemia
B) Erythremia
C) Chronic lymphocytic leukemia
D) Chronic myeloid leukemia
E) Multiple myeloma
ANSWER: D
A 68-yr-old patient presents with bone pain, anaemia and renal failure. Her bone
marrow reveals 32 % of plasma cells. What is the most probable diagnosis?
A) Multiple myeloma
B) Myeloid metaplasia
C) AML
D) CLL
E) ALL
ANSWER: A
A 70-year-old male presents with a complaint of fatigue. There is no history of
alcohol abuse or liver disease; the patient is on no medication. Scleral icterus is
noted on physical exam. There is no evidence for chronic liver disease on physical
exam, and the liver and spleen are nonpalpable. The patient is noted to have a
normocytic, normochromic anemia. The first step in evaluation of this patient is
A) CT scan of the abdomen
B) Hepatitis profile
C) Liver function tests, including direct versus indirect bilirubin and urine
bilirubin
D) Abdominal ultrasound
E) Esophagogastroduodenoscopy
ANSWER: C
A 70-yr-old man presents with bone pain, anaemia. His bone marrow reveals plasma cells
30 %. What is the most probable diagnosis?
A) Myeloid metaplasia
B) AML
C) Multiple myeloma
D) CLL
E) Megaloblastic anaemia
ANSWER: C
A 70-yr-old woman develops severe hip pain while gardening. Hip joint X-ray shows
fracture neck of femur. She gives a history of lower back pain and malaise. She is
tender over the lumbar spine. ESR is 110 mm/hr. What is your diagnosis?
A) Multiple myeloma
B) Waldenstorm's macroglobulinaemia
C) CML
D) CLL
E) Acute leukemia
ANSWER: A
A 70-yr-old woman presents to her GP with weight loss and depression. On examination
she is noted to have buccal pigmentation and pigmented scars. She appears dehydrated.
Her BP is 100/60 mm Hg. Choose the most likely management.
A) Long-term replacement with glucocorticoids and mineralocorticoids
B) Calciferol
C) Carbimazole
D) Thyroxine
E) Octreotide (somatostatin)
ANSWER: A
A 71-year-old male presents with a complaint of fatigue. There is no history of
alcohol abuse or liver disease; the patient is on no medication. Scleral icterus is
noted on physical exam. The patient is noted to have conjugated hyperbilirubinema,
with bilirubin detected in the urine. Serum bilirubin is 12 mg/dL, AST and ALT are in
normal range, and alkaline phosphatase is 300 U/L (3 times normal). The next step in
evaluation is
A) Ultrasound or CT scan
B) Hepatitis profile
C) Reticulocyte count
D) Family history for hemochromatosis
E) Esophagogastroduodenoscopy
ANSWER: A
A 75-year-old man has been deferring colon cancer screening because she is afraid to
undergo colonoscopy. She learned of a new technique called virtual colonoscopy that
she thinks may be more tolerable and asks you about the relative merits of this
procedure. Which of the following statements is true regarding virtual colonoscopy?
A) It detects colorectal cancers and large adenomas quite well, but may miss small
polyps
B) It is more acceptable to patients because it does not require any bowel preparation
C) It is a noninvasive procedure that images the colon using ultrasound
D) Its sensitivity and specificity for detecting colon cancers and polyps is similar
to that of conventional colonoscopy
E) It does not require any instrumentation of the bowel
ANSWER: A
A 76-year-old man, who is in a rehabilitation facility after fracturing a leg,
develops acute diarrhea. He has no history of intestinal disorders. The patient has
regular colonoscopic screenings. His last colonoscopy, done 2 years ago, was normal.
He has not received antibiotics in the past year. His roommate is taking oral
metronidazole for a diarrheal syndrome that he developed while in the rehabilitation
facility. Use of which of the following would most likely have prevented development
of the patients diarrheal syndrome?
A) Good hand-washing technique
B) Prophylactic antibiotics
C) Prophylactic probiotic agents
D) Prophylactic loperamid
E) None of above
ANSWER: A
A man of 59 years old complaints of abdominal discomfort, gum bleeding, large
ecchymosis, weakness, sternal tenderness, fever, skin nodules. Laboratory findings:
the white cell count 540 x 109/L, levels of basophils, eosinophils and platelets are
increased; and a few normoblasts are seen; Er – 3,1 x 1012/L, blast 48%. What is the
most probable diagnosis?
A) Megaloblastic anemia
B) Hemolytic anemia
C) Acute leukemia
D) Thrombocytopenia
E) Chronic leukemia
ANSWER: C
A man, 35years old, complains of weakness, palpitation, tinnitus dizziness. Data of
anamnesis: peptic gastric ulcer, repeated bleeding. Physical examination: skin is pale.
Systolic murmur is heard at the apex, HR 100 per min, BP 100/70 mm of Hg. CBC: ESR –
2.8 x 1012/L, haemoglobin content - 69 gr/l, colour index is 0,7. What is your
treatment?
A) Iron parenteral
B) Folic acid
C) iron supplementation oral
D) Vit B12
E) Antibiotics
ANSWER: C
A middle-aged woman complains of irritability and weight loss. She says she has
palpitations. On physical examination you find mild tachycardia and goiter. There are
no eye changes. A thyroid scan determines a single hot nodule. Choose the most likely
diagnosis.
A) Toxic adenoma
B) Adrenal hyperplasia
C) Hyperthyroidism
D) Hypothyroidism
E) Follicular carcinoma
ANSWER: A
A nursing student has just completed her hepatitis B vaccine series. On reviewing her
laboratory studies (assuming she has no prior exposure to hepatitis B), you expect
A) Positive test for hepatitis B surface antigen
B) Antibody against hepatitis B surface antigen (anti-HBS) alone
C) Antibody against hepatitis core antigen (anti-HBC)
D) Antibody against both surface and core antigen
E) Antibody against hepatitis E antigen
ANSWER: B
A paracentesis is performed on 43-year-old patient with long-standing alcohol abuse.
On physical exam, there are spider angiomas and palmar erythema. Abdominal collateral
vessels are seen around the umbilicus. The serum albumin minus ascitic fluid albumin
equals 1.4 g/dL. The most likely diagnosis is
A) Portal hypertension
B) Pancreatitis
C) Tuberculous peritonitis
D) Hepatoma
E) No ascitis
ANSWER: A
A patient presents to a physician with severe jaundice. Physical examination reveals a
nodular, enlarged liver. In addition to the generalized nodularity of the liver, the
physician can feel one nodule that is much larger than the others. CT of the abdomen
confirms multinodular cirrhosis and demonstrates a 7-cm mass near the lower border of
the liver. CT-guided biopsy of this mass shows a malignant tumor derived from hepatic
parenchymal cells. Which of the following risk factors is most strongly associated
with the development of this tumor?
A) Anatoxin exposure
B) Hemochromatosis
C) Hepatitis В virus infection
D) Opistharchis infection
E) Thorotrast exposure
ANSWER: C
A patient who has long-standing diabetes mellitus and severe, burning pain in the feet
and hands as a result of peripheral neuropathy asks the physician why an
antidepressant has been prescribed. What is the physician’s best response?
A) “Many people experiencing chronic pain become depressed.”
B) “The antidepressants may counteract the chemicals causing your pain.”
C) “You are less likely to become addicted from using antidepressants than you are
from using other types of pain killers.”
D) “The antidepressants also have strong anti-inflammatory properties and can reduce
the pain you have from inflammation.”
E) None of above
ANSWER: B
A patient who is 2 days postoperative from a bowel resection tells her physician that
she is having a hard time “catching her breath,” feels nauseated, and has chest pains
when she inhales. The physician suspects that she is having a pulmonary embolism. What
intervention should the physician perform before notifying the physician?
A) Increase the IV flow rate
B) Apply oxygen by mask or nasal cannula at 5 l/min
C) Assess the chest and axillary area for the presence of petechiae
D) Place the patient in shock position, with her head and neck flat and her legs
elevated
E) Non of above
ANSWER: B
A woman 22 year-old, complains of general weakness, shortness of breath, brittle nails,
hair loss. She has menorrhagia. Physical examination reveals paleness of skin,
systolic murmur on auscultation. What changes do you expect in her blood analysis?
A) Increased level of serum iron
B) High colour index
C) Decreased level of serum iron
D) Increased level of free bilirubin
E) Decreased amount of thrombocytes
ANSWER: C
A young man presents with a neck lump. It is painless and had been bothering him for
the past 4 months. He has no other symptoms or signs. On palpation you find the lump
to be single discrete not particularly hard and confined to the thyroid gland itself.
His cervical lymph nodes are enlarged. Choose the most likely diagnosis.
A) Toxic adenoma
B) Adrenal hyperplasia
C) Hyperthyroidism
D) Hypothyroidism
E) Papillary carcinoma
ANSWER: E
A young woman complains of wheeze, dyspnoea and cough. She cannot sleep at night
because of a chronic cough. She and her mother love animals and together they have 14
cats. Her PEFR is normal but her CXR suggests hyperinflation. What is the previous
diagnosis?
A) Bronchial asthma
B) Bronchogenic carcinoma
C) Emphysema
D) Respiratory failure
E) Bronchitis
ANSWER: A
After emotional exertion patient of 24 y.o. developed dyspnea with prolonged
expiration, distant wheezes, frequent night symptoms. Such changes limit his physical
activity. PEV and FEV1 < 60 %, daily variability -30 %. What diagnosis is possible?
A) Severe persistent bronchial asthma
B) Mild persistent bronchial asthma
C) Persistent bronchial asthma of moderate severity
D) Intermittent bronchial asthma
E) Bronchospastic syndrome of allergic origin
ANSWER: A
An 14-yr-old boy presents with painful bones, jaundice and anaemia. He is noted to
have splenomegaly. His blood film reveals target cells. What is your diagnosis?
A) Hemolytic anemia.
B) Sickle cell anaemia
C) Iron-deficiency anemia.
D) B12-deficiency anemia
E) Aplastic anemia.
ANSWER: B
An 18-yr-old girl complains of her appearance. She is much too fat, she says. She also
complains of missed periods and hairiness. On physical examination you find her to be
10 kg overweight. Choose the most likely investigation
A) Abdominal ultrasonography
B) Serum cortisol
C) Dexamethasone suppression test
D) Water deprivation test
E) T3, T4 and TSH
ANSWER: C
An 69 year old asymptomatic woman was detected to have a monoclonal spike on serum
electrophoresis (IgG levels 1.5 g/dl). Bone marrow revealed plasma cells of 30%. What
is the possible diagnosis?
A) Multiple myeloma
B) Indolent myeloma
C) Monoclonal gammopathy of unknown significance
D) Waldenstorm's macroglobulinemia
E) Amyloidosis
ANSWER: A
Drug abuser, a 41-yr-old man, presents with fever, cough and breathlessness. This was
preceded by viral influenza. Chest radiograph shows multiple abscesses. What is the
most possible etiology of disease?
A) Staphylococcus aureus
B) Cryptococcus
C) Streptococcus pneumoniae
D) Legionella pneumonia
E) Mycobacterium avium
ANSWER: A
Female B., 44 years old, complains on cough with mucous sputum, increase of
temperature to 39 °С, weakness, dyspnea, sweating. Breathing rate - 26/min, skin is
moist. Below left scapula there is shortening of percussion sound. Breathing during
auscultation is weakened, moist rales. Blood test: L - 11х109/l, ESR - 29 mm/h. Your
previous diagnosis?
A) Left-side lower lobe pneumonia
B) Gangrene of lungs
C) Left-side exudative pleurisy
D) Cancer of left side lower lobe
E) Pulmonary abscess
ANSWER: A
Female patient K., 46 years old, has flue with fever. After a while she
noticesthoracic pain, cough with yellow-green sputum (amount-150 ml a day), sometimes
with some blood. Objectively: breathing rate - 36/min. In lungs from the right side
lower scapula there is dull sound during percussion, hard breathing, and moist rales.
Blood test: L - 18,6х109/l, ESR -64 mm/h. Analysis of sputum: L -80-100 , Er - 40-50,
elastic fibers, cocci. X-ray: lung roots are enlarged, from the right side lower lobe
is heterogeneously infiltrated with two lighter areas. What is the most possible
previous diagnosis?
A) Right-side pneumonia with abscesses
B) Peripheral cancer
C) Infiltrative tuberculosis in the phase of disintegration
D) Exudative pleurisy
E) Infarction-pneumonia
ANSWER: A
Female, 34 years old, has an increase of body temperature to 38 °С, cough with
purulent sputum, weakness, dyspnea, pain in a thorax during breathing. During
percussion there is shortening of sound in the lower part of left lung, during
auscultation – moist rales. What method of investigation is the decisive one to
confirm diagnosis?
A) X-ray examination
B) Bacteriological analysis of sputum
C) Spirometry
D) Pneumotachometry
E) Bronchography
ANSWER: A
Girl, 18 y.o., pets seller, complaints mainly during working time on the attacks of
dry cough, running nose. She often suffers from acute viral respiratory infections.
Her mother is ill with bronchial asthma. Objectively: breathing rate - 18/min. Heart
rate - 80/min, BP - 110/70. In lungs vesicular breathing, dry wheezes are heard in
distance. Tones of heart are weaker than normally. Test with berotec showed
reversibility of bronchial obstruction. What tactic will be the best for the patient?
A) To change job
B) To use intal
C) To use monteleucast
D) To use berotec constantly
E) To use antihystaminic preparations
ANSWER: A
Girl, 23 y.o., for 2 years is ill with bronchial asthma. Recently attacks of dyspnea
became more frequent and started to arise 4-5 times a week, night attacks - 2-3 times
a month. She used salbutamol to remove the symptoms. Test with the antigen of home
dust is positive. Objectively: condition is satisfactory. Breathing rate - 20/min.
Heart rate - 76/min, BP -120/80. In lungs breathing is vesicular. Tones of heart are a
little weak, rhythm is normal. What mechanism is desicive in development of bronchial
obstruction in this case?
A) Hyperreactivity of bronchi
B) Тrachео-bronchial dyskinesia
C) Violation of metabolism of arachidonic acid
D) Adrenergic disorders
E) Activity of the parasympathetic nervous system is increased
ANSWER: A
In the biochemical profile of patient B., 29 yr-old, it is elevated serum T4 and low
radioactive iodine uptake. Choose the most likely diagnosis.
A) Non-toxic goitre
B) Hashimoto's thyroiditis
C) Subacute thyroiditis
D) Hypothyroidism
E) Graves' disease
ANSWER: C
In the biochemical profile of patient J., 38 yr-old, who has a neck mass, it is normal
T3 and T4. Choose the most likely diagnosis.
A) Non-toxic goitre
B) Hashimoto's thyroiditis
C) Subacute thyroiditis
D) Hypothyroidism
E) Graves' disease
ANSWER: A
In the biochemical profile of patient L., 56 yr-old., it is normal TSH, free T4 and T3.
Decreased serum total T4. Choose the most likely diagnosis.
A) Non-toxic goitre
B) Hashimoto's thyroiditis
C) Thyroid binding globulin deficiency
D) Hypothyroidism
E) Graves' disease
ANSWER: C
In the biochemical profile of patient S., 43 yr-old, it is elevated serum T4 and
increased radioactive iodine uptake. Choose the most likely diagnosis.
A) Non-toxic goitre
B) Hashimoto's thyroiditis
C) Subacute thyroiditis
D) Hypothyroidism
E) Graves' disease
ANSWER: E
Male patient F., 48 years old, during a week stayed at home with diagnosis of
respiratory viral infection. Doctor noticed complaints on cough with small amount of
mucus-purulent sputum, weakness. Objectively: condition is relatively satisfactory. T
- 37,2 °С. Breathing rate - 18/min., pulse - 80/min., BP - 110/70. In lungs there is
vesicular breathing, with a hard tint, single dry wheezes. Tones of heart are a little
dull, rhythm is correct. What is the treatment tactic?
A) To prescribe antibacterial therapy
B) To stay at home for some more days
C) To go to work
D) To send patient to pulmonologist
E) To hospitalize patient to the pulmonological department
ANSWER: A
Male patient G., 56 years old, complaints on permanent pain in a thorax which disturbs
for last 2 months. Pain is not connected with breathing. There is also cough with
particles of blood in sputum. Weakness, fatigue are present. On the chest X-ray in the
lower lobe of right lung there is spherical shadow, size 4x6 cm, related to the lung
root. What is the most possible diagnosis?
A) Perypheral lung cancer
B) Tuberculoma
C) Metastasis
D) Pulmonary abscess
E) Pneumonia
ANSWER: A
Man, 32 y.o., complaints on attack of expiratory dyspnea, which lasts for 48 hours,
cough with small amount of sputum. He is ill with bronchial asthma for 5 years, was
treated with glucocorticosteroids, used inhalers. Objectively: condition is severe,
patient sits. Diffuse cyanosis, pulse -110/min, BP - 110/70. Tones of heart are weak,
II tone is louder above the pulmonary artery. During percussion in lungs there is
“bang-box” sound, large amount of dry wheezes. In blood there is eosinophylia - 18 %.
What medicines are drugs of choice for this patient?
A) Corticosteroids
B) ?2-agonists
C) Theophyllin
D) Cholynolytics
E) Antihystamines
ANSWER: A
Man, 39 y.o., 8 last years is ill with bronchial asthma. Rapidly during physical work
he felt worsening of breathing, cough, distance wheezes appeared and dyspnea began to
increase. Medicine of what pharmacological group is it better to recommend for the
patient to remove such attacks of dyspnea?
A) Agonists of ?2-adrenoreceptors
B) Methylxantines
C) ?2-adrenoblockers
D) Inhalated glucocorticoids
E) Oral glucocorticoids
ANSWER: A
Man, 43 y.o., complaints on dyspnea during physical activity. Objectively: temperature
36,4 °С, breathing rate - 20/min, pulse - 78/min, BP-125/80. Emphysematous form of
thorax. In lungs – weak vesicular breathing. What test should be passed by patient at
home to decide question about efficiency of prescribed broncholytics?
A) Peakflowmetry
B) Spirography
C) ECG-control of overload of right chambers of heart
D) Bronchoscopy
E) Analysis of sputum (amount and microscopy)
ANSWER: A
Man, 46 y.o., suffers for the last 10 years from bronchial asthma. Rapidly during
physical work he felt worsening of breathing, cough, distance wheezes appeared and
dyspnea began to increase. Medicine of what pharmacological group is it better to
recommend for the patient to remove such attacks of dyspnea?
A) Agonists of ?2-adrenoreceptors
B) Atropine
C) Intal
D) Epinephrine
E) Monteleucast
ANSWER: A
On a routine blood examination a 43-yr-old woman is found to have very high serum
calcium level. She has complained recently of bouts of abdominal pain and recurrent
UTI. On physical examination you find an enlarged thyroid gland. Choose the most
likely diagnosis.
A) Toxic adenoma
B) Adrenal hyperplasia
C) Hyperthyroidism
D) Hypothyroidism
E) Parathyroid carcinoma
ANSWER: E
Over a 2-month period, a 50-year-old woman with a history of polycythemia vera
develops abdominal pain and gross ascites. Physical examination demonstrates smooth
hepatomegaly and mild jaundice. Pressure applied over the liver fails to distend the
jugular veins. The abdominal wall is grossly edematous and shows a tortuous venous
pattern. Edema of the legs is prominent. Which of the following is the most likely
diagnosis?
A) Hepatocellular carcinoma
B) Primary sclerosing cholangitis
C) Steatosis
D) Budd-Chiari syndrome
E) Hepatic cirrhosis
ANSWER: D
Patient A., 35 y.o., noticed infrequent (rarer than 1 time a week) attacks of dyspnea,
which are easily removed with inhalations of ?2-agonists of short action. During
attack in lungs are heard dry wheezes, between attacks FEV1 is more than 80 % from
normal. What is the diagnosis?
A) Intermittent bronchial asthma
B) Persistent bronchial asthma of moderate severity
C) Mild persistent bronchial asthma
D) Severe persistent bronchial asthma
E) Given information is not enough for determination of severity of bronchial asthma
ANSWER: A
Patient A., 45 years old, suffers from chronic cholecystits during last 5 years. She
was admitted to the emergency department with acute pain in right hypochondriac area
and high temperature. Laboratory findings reveal leucocytosis, high ESR. Put
preliminary diagnosis.
A) Chronic cholecystitis, acute phase.
B) Chronic cholecystitis, subacute phase.
C) Chronic cholecystitis, phase of remission.
D) Dyskinezia of bile ducts.
E) Rotor’s syndrome.
ANSWER: A
Patient B., 25 years old engineer, appeared during a fire in the area of high
concentration of CO (an industrial accident). In hospital delivered in the
irresponsible state. What laboratory tests are the early criteria of estimation of
severity of the state?
A) Estimation of blood viscosity
B) Anemia
C) Leucocytosis
D) Carboxihemoglobinemia
E) Methemoglobinemia
ANSWER: D
Patient complaints on attacks of dyspnea, which arise 1-2 times a week, night symptoms
- 2 times a month and even more frequently. For a patient night sleep is violated as a
result of attacks of dyspnea. FEV1 > 80 % from normal. What diagnosis would you
suspect?
A) Mild persistent BA
B) Severe persistent BA
C) Intermittent BA
D) Moderate persistent BA
E) Status asthmaticus
ANSWER: A
Patient D., 56 yr-old, complains on muscle weakness, bradycardia and hypotension. ECG
shows tall peaked T waves. Choose the most likely diagnosis.
A) Hypokalaemia
B) Hyponatraemia
C) Hypervitaminosis A
D) Hyperkalaemia
E) Hypoglycaemia
ANSWER: D
Patient F., 46 years old, was hospitalized urgently with acute attack of dyspnea. Last
5 years he has been working on poultry farm. During examination bronchial asthma was
diagnosed. What additional diagnostic methods are necessary to confirm the
professional genesis of asthma?
A) sanitary-hygienic characteristics of the work conditions
B) echocardioscopy
C) allergic and immunological tests
D) investigation of the function of external breath
E) roentgenography of pulmonary system
ANSWER: C
Patient G., 36 years old, works on a poultry factory. She was emergently hospitalized
with acute attack of dyspnea. During observation bronchial asthma was diagnosed. What
additional methods of research must be conducted above all things to confirm the
professional genesis of bronchial asthma?
A) roentgenologic research of breathing organs
B) professional route of patient
C) sanitary-hygienic characteristic of work conditions
D) research of external breathing function
E) allergic and immunological tests
ANSWER: E
Patient G., 38 yr-old, complains on muscle weakness and ectopic beats. ECG shows
flattened or inverted T waves. Choose the most likely diagnosis.
A) Hypokalaemia
B) Hyponatraemia
C) Hypervitaminosis A
D) Hyperkalaemia
E) Hypoglycaemia
ANSWER: A
Patient G., 47 y.o., with long history of bronchial asthma, has developed more
frequent attacks of dyspnea. Inhalations of astmopent and berotec are not effective.
From prescription of what medicine is it better to begin the intensive treatment?
A) Glucocorticoids
B) Oxygen therapy
C) Bronchodylators
D) Infusion therapy
E) Heart glycosides
ANSWER: A
Patient has severe attack of bronchial asthma which lasts more than 1 hour. Usage of
beta-agonists in inhalation, euphylline intravenously and cholynolytics was not
effective. What medicines are necessary for emergency therapy?
A) Glucocorticosteroids intravenously
B) Beta-agonists intravenously
C) Inhaled glucocorticosteroids
D) Antihystaminic
E) Nonsteroid anti-inflammatory medicines
ANSWER: A
Patient J., 36 y.o., complains for fever (39 C), pain in the left part of the chest.
Pleuropneumonia was diagnosed in the patient. What onset is typical for
pleuropneumonia?
A) Acute
B) Latent
C) Fulminant
D) Gradual
E) Non of the above
ANSWER: A
Patient J., 38 yr-old, complains on sweating, palpitations, tremors, drowsiness and
fatigue. Choose the most likely diagnosis.
A) Hypokalaemia
B) Hyponatraemia
C) Hypervitaminosis A
D) Hyperkalaemia
E) Hypoglycaemia
ANSWER: E
Patient J., 45 y.o., complaints on dyspnea during insignificant physical exertion,
cough with minimal amount of “glass-like” sputum, attacks of dyspnea up to 3 times a
day, more often at night, sweating. She is ill for more than 5 years. Has an allergy
on dust, cockroaches. For treatment uses bekotid for near the year. Diagnosis?
A) Bronchial asthma
B) Eosinophylic pulmonary infiltrate
C) COPD
D) Bronchiectasis with bronchial spasm
E) Pulmonary vasculitis (syndrome of Charg - Stross)
ANSWER: A
Patient L, 41 years is suffering from diabetes type 1 for 16 years, receiving insulin
therapy. He complaints of swelling and shortness of breath. On examination: glycemia -
7.1 mmol / l, cholesterol - 6.2 mmol / l, Creatinine - 0.21 mmol / l. What will you
recommend the patient first of all?
A) Increase the dose of insulin
B) ACE inhibitors
C) Statins
D) Lipoic acid
E) Enterosorbents
ANSWER: E
Patient M., 30 years old, during last 3 years works as a nurse in manipulations
cabinet. Last year during the contact with penicilline she started to complain on
discomfort in throat, sneezing, attack of cough and dyspnea which disappear after
inhalation of salbutamol. During last months attacks of dyspnea became more severe and
occurred only at contact with penicilline. During the life she had not any diseases
including allergic. She hadn’t received antibiotics. Can we consider the bronchial
asthma is professional in this patient?
A) no, we can’t
B) yes, we can if we have conclusion about attacks of bronchial asthma
C) yes, we can if we have conclusion about appearance of bronchial asthma attacks
after contact with penicilline
D) yes, we can
E) yes, we can, if allergic and immunological tests are positive
ANSWER: E
Patient S., 43 yr-old, has perioral paraesthesia, carpopedal spasm and generalised
seizures. Choose the most likely diagnosis.
A) Hypokalaemia
B) Hyponatraemia
C) Hypocalcaemia
D) Hyperkalaemia
E) Hypoglycaemia
ANSWER: C
Patient U., 44 yr-old, complains on severe abdominal pain, nausea, vomiting,
constipation, polyuria and polydipsia. Choose the most likely diagnosis.
A) Hypercalcaemia
B) Hyponatraemia
C) Hypervitaminosis A
D) Hyperkalaemia
E) Hypoglycaemia
ANSWER: A
Patient Y.,49 y.o., complaints on dyspnea, cough. Sputum is absent. Used many puffs of
salbutamol, intal, but without any efficacy. Objectively: sits, leaning on a table.
Total cyanosis of the body. Peripheral edema is absent. Breathing is superficial,
dyspnea, during auscultation breathing cannot be heard in some areas of lungs; wheezes
are diffuse, expiration is considerably prolonged. Tones of heart are weak,
tachycardia. Pulse - 112/min, BP - 110/70. Liver is near the edge of costal arch. What
is the previous diagnosis?
A) Status asthmaticus
B) Bronchial asthma of moderate severity
C) COPD
D) Aspiration of foreign body
E) Heart asthma
ANSWER: A
Patient, 28 y.o., has running nose, attacks of dyspnea at night once a week. Felt ill
after viral respiratory infection which was treated with acetilsalicylic acid.
Eosynophylia was found in blood and sputum. What disease may be suspected?
A) Aspirin bronchial asthma
B) Eosinophylic infiltrate of lungs
C) Bronchial asthma of physical exertion
D) Allergic rhinitis
E) Bronchial asthma, exogenous form
ANSWER: A
Patient, 30 y.o., after a viral infection has daily symptoms of dyspnea, which causes
lowering of activity and bad sleep; night symptoms are more frequent than once a week.
PEV and FEV1 - 60-80 %, daily variability > 30 %. There is a necessity of daily usage
of ?2-agonists of short action. What is the diagnosis?
A) Persistent bronchial asthma of moderate severity
B) Mild persistent bronchial asthma
C) Intermittent bronchial asthma
D) Severe persistent bronchial asthma
E) Status asthmaticus
ANSWER: A
Patient, 42 years old, complaints on attacks of dyspnea, every time uses 1-2 doses of
salbutamol. These attacks are accompanied with cough and minimal amount of viscid
glassy sputum. He is ill for 8 years. Objectively: temperature - 36,7 C; breathing
rate – 21/min.; pulse-90/min.; BP - 130/80; FEV1 - 77 %. In lungs – solitary dry
wheezes. Blood test: eosinophyles - 6 %. What medicines are “basic” in the treatment
of this patient?
A) Antiinflammatory
B) Cholynolytics
C) Mucolytics
D) Antihystaminic
E) ?2-agonists
ANSWER: A
Patient, 44 y.o., complaints on attack of dyspnea, which arises suddenly at night.
Connects this attack with overcooling. He is ill for more than 10 years. Thorax of
emphysematous form. During percussion in lungs – “bang-box” sound. During
auscultation there is plenty of dry wheezes. In blood: moderate leucocytosis,
eosinophylia - 10 %. On the chest X-ray film – increased pneumatization of pulmonary
tissue. What diagnosis is the most possible?
A) Bronchial asthma, exacerbation phase
B) Bronchiectasis, exacerbation phase
C) COPD, exacerbation phase
D) Chronic bronchitis
E) Eosinophylic pulmonary vasculitis
ANSWER: A
PatientI., a 50 years old man, is evaluated in the emergency department because of
fever, nonproductive cough and 2-day history of myalgia and headache. He has also had
nausea and diarrhea. He is a heavy smoker. On physical examination, he is slightly
disoriented. Temperature is 38.9 C, pulse rate is 110/min, respiration rate is 32/min.
Chest radiograph shows fluffy infiltrates to the right upper and lower lobes. Results
of laboratory testing show serum sodium of 128 meq/L, blood urea nitrogen of 42 mg/dL,
serum creatinine of 2.2 mg/dL, and serum creatine kinase of 250 U/L. Which one of the
following is best next step in the management of this patient’s pneumonia?
A) Initiate empiric antibiotic therapy for Legionella
B) Order direct fluorescent antibody testing of the sputum for Legionella
C) Order serologic testing for Legionella
D) Send a urine specimen for measurement of Legionella antigen
E) All of the above
ANSWER: A
Physical examination of patient revealed lymphadenopathy, splenomegaly. CBC: RBCs -
3,6 x 1012/l, Hb- 87 g/l, Pl – 45 x 109/l, WBCs – 13 x 109/l, blasts - 87%, band
neutrophils - 1%, segmented neutrophils - 7%, lymphocytes - 5%, ESR - 55 mm/h. What is
the most likely diagnosis?
A) Acute leukemia
B) Erythremia
C) Chronic lymphocytic leukemia
D) Chronic myeloid leukemia
E) Multiple myeloma
ANSWER: A
The 30-year-old woman whose father has type 1 diabetes mellitus asks the physician
what her chances are of developing diabetes because of her father's disease. The risk
for becoming diabetic is:
A) 20-50%
B) 0%
C) 50%
D) 50-80%
E) None of above
ANSWER: A
The 45-year-old diabetic patient has proliferative retinopathy, nephropathy, and
peripheral neuropathy. What should the physician teach this patient about exercise?
A) “The type of exercise that would most efficiently help you to lose weight, decrease
insulin requirements, and maintain cardiovascular health would be jogging for 20
minutes 4 to 7 days each week.”
B) “Considering the complications you already have, vigorous exercise for an hour each
day is needed to prevent progression of disease.”
C) “Considering the complications you already have, you should avoid engaging in any
form of exercise.”
D) “Swimming or water aerobics 30 minutes each day would be the safest exercise
routine for you.”
E) None of above
ANSWER: D
The patient diabetic patient asks the physician why it is necessary to maintain blood
glucose levels no lower than about 74 mg/dL. What is the physician’s best response?
A) “Glucose is the only fuel form used by body cells to produce energy needed for
physiologic activity.”
B) “The central nervous system, which cannot store glucose, requires a continuous
supply of glucose for fuel.”
C) “Without a minimum level of glucose circulating in the blood, erythrocytes cannot
produce ATP.”
D) “The presence of glucose in the blood counteracts the formation of lactic acid and
prevents acidosis.”
E) None of above
ANSWER: B
The patient getting ready to engage in a 30-minute, moderate-intensity exercise
program performs a self-assessment. Which data indicate that exercise should be
avoided at this time?
A) Ketone bodies in the urine
B) Weight 1 pound higher than the week before
C) Blood sugar level of 155 mg/dL
D) Pulse rate of 66 beats/min
E) None of above
ANSWER: A
The patient has clinical signs which may suggest acromegaly. Which of the following is
the most specific test to diagnose acromegaly?
A) Magnetic resonance imaging of the pituitary gland
B) Measurement of growth hormone (GH) levels during an oral glucose tolerance test
C) Measurement of insulin-like growth factor I serum levels
D) Measurement of random GH blood levels
E) An octreotide scan
ANSWER: C
The patient newly diagnosed with type 2 diabetes tells the physician that since he has
increased his intake of fiber, he is having loose stools, flatulence, and abdominal
cramping. What is the physician’s best response?
A) “Decrease your intake of water and other fluids.”
B) “Decrease your intake of fiber now and gradually add high-fiber foods back into
your diet.”
C) “You must have allergies to high-fiber foods and will need to avoid them in the
future.”
D) “Taking an antacid 1 hour before meals or 2 hours after meals should reduce the
intensity of your bowel problems.”
E) None of above
ANSWER: B
The patient on an intensified insulin regimen consistently has a fasting blood glucose
between 270 and 280 mg/dL, a postprandial blood glucose level below 200 mg/dL, ketones
in urine ++and a hemoglobin A1c level of 8.5%. What is the physician’s interpretation
of these findings?
A) The patient has developing night hypoglycemia.
B) The patient is at increased risk for developing hyperglycemia.
C) The patient is demonstrating signs of insulin resistance.
D) The patient is demonstrating good control of blood glucose.
E) None of above
ANSWER: A
The patient tells the physician that he enjoys having a glass of wine on Saturdays
when dining out with friends. He asks if having type 1 diabetes will prohibit him from
this activity. What is the physician’s best response?
A) “Insulin activity is dramatically reduced under the influence of alcohol and
drinking even one glass of wine will increase your insulin requirements.”
B) “Diabetics have decreased kidney function and should avoid ingesting alcohol in all
forms at all times.”
C) “You shouldn't drink any alcohol because it is likely to increase your sense of
hunger and make you overeat.”
D) “One glass of wine can be ingested with a meal and is counted as two fat exchanges.”
E) None of above
ANSWER: D
The patient with diabetes is visually impaired and wants to know if syringes can be
prefilled and stored for use later. What is the physician’s best response?
A) “Yes, prefilled syringes can be stored for up to 3 weeks in the refrigerator in a
vertical position with the needle pointing up.”
B) “Yes, prefilled syringes can be stored for up to 3 weeks in the refrigerator,
placed in a horizontal position.”
C) “Insulin reacts with plastic, so prefilled syringes must be made of glass.”
D) “No, insulin cannot be stored for any length of time outside of the container.”
E) None of above
ANSWER: A
The patient with hyperthyroid symptoms is having hormone studies done to confirm the
diagnosis. Which set of values indicates Graves’ disease hyperthyroidism?
A) Elevated T3, elevated T4, high TSH levels
B) Elevated T3, normal T4, low TSH levels
C) Elevated T3, low T4, high TSH levels
D) Low T3, normal T4, high TSH levels
E) None of above
ANSWER: B
The patient with hypothyroidism as a result of Hashimoto’s thyroiditis asks the
physician how long she will have to take thyroid medication. What is the physician’s
best response?
A) “You will need to take the thyroid medication until the goiter is completely gone.”
B) None of above
C) “The thyroiditis will be cured with antibiotics, and then you will no longer need
the thyroid medication.”
D) “You will need thyroid replacement hormone therapy for the rest of your life
because the thyroid gland function will not return.”
E) “When your thyroid function studies indicate a normal blood level of thyroid
hormones, you will be able to discontinue the medication.”
ANSWER: D
The patient, 51 years old, suffers from diabetes 7 years, gets glibenclamide 15 mg a
day. Complains of shortness of breath and pitting edema. On examination of blood
pressure - 180/110 mm Hg. Laboratory: glycemia - 6.2 mmol / L, creatinine - 0.15 mmol
/ L, glomerular filtration - 62 ml / min. What medication will you recommend to the
patient?
A) Gliclazide
B) Gliquidone
C) Glimepiride
D) Pioglitazone
E) Repaglinide
ANSWER: B
When taking the blood pressure of a patient after a parathyroidectomy, the physician
notes that the patient's hand has gone into flexion contractions. What is the
physician’s interpretation of this observation?
A) Hypokalemia
B) Hyperkalemia
C) Hyponatremia
D) Hypocalcemia
E) None of above
ANSWER: D
Which the following type of anemia can be normocytic normochromic anemia with
reticulocytosis?
A) Haemolytic anaemia
B) Vitamin B12 deficiency anaemia
C) Acute posthemorrhagic anaemia
D) Chronic iron deficiency anaemia
E) Aplastic anaemia
ANSWER: A
Which the following type of anemia can be with Cabot rings?
A) Vitamin B12 deficiency anaemia
B) Acute posthemorrhagic anaemiaChronic iron deficiency anaemia
C) Aplastic anaemia
D) Haemolytic anaemia
ANSWER: D
A 27-year-old male presents to the health center for right upper quadrant abdominal
pain, generalized pruritus, and jaundice for 3 days. He states that the pain came on
gradually and awoke him early on the morning of presentation. His past medical history
is pertinent for ulcerative colitis, although he has not taken any medication in 4
years. His temperature is (39.2°C) and physical exam shows pain in the right subcostal
region with deep inspiration, and generalized jaundice. What is the most serious
complication of the most likely diagnosis?
A) Perforation of rectum.
B) Perforation of stomach wall.
C) Infarction of small bowel.
D) Cholangiocarcinoma.
E) Pancreatic pseudocyst formation.
ANSWER: D
A 27-year-old man complains to a physician of chronic gastric pain. The pain is
sometimes relieved by food. EGDS shows antral erosive gastritis, biopsy of antral
mucous presents Hеlicobacter Pylori. What can be diagnosed in this case?
A) Gastritis of type B
B) Menetrier’s disease
C) Rigid antral gastritis
D) Gastritis of A type
E) Reflux – gastritis
ANSWER: A
A 27-year-old man is evaluated because of two episodes of painless melena, the last of
which occurred 6 hours ago. He has been taking naproxen for the past 6 weeks for a
sports-related shoulder injury. Medical history is otherwise noncontributory. On
physical examination, temperature is 36.0 C (96.8 F), pulse rate is 98/min and regular,
respiration rate is 18/min, and blood pressure is 104/89 mm Hg without orthostatic
changes. Digital rectal examination discloses very dark stool that is positive for
occult blood. Hemoglobin is 9.3 g/dL, leukocyte count is 10,800/?L, platelet count is
250,000/?L, and other routine laboratory studies are normal. Nasogastric aspirate is
negative. Which of the following should be done next?
A) Order red blood cell transfusion to achieve a hemoglobin value above 10 g/dL
B) Establish access with two intravenous lines and infuse isotonic saline, 500 mL/h
for 6 to 8 hours; then reassess the hemoglobin value
C) Transfuse 6 units of platelets to correct the platelet dysfunction caused by
naproxen
D) Perform upper endoscopy
E) Order an upper gastrointestinal series
ANSWER: D
A 27-yr-old pregnant woman is found to have thyrotoxicosis due to Grave's disease
during second trimester of her pregnancy. Choose the most likely management.
A) Radioactive iodine
B) Subtotal thyroidectomy
C) Propanolol
D) Carbimazole
E) Thyroxine
ANSWER: D
A 27-yr-old woman presents with fever, sore throat and dysphagia. On examination she
has a fine tremor and a diffusely tender thyroid. Radioisotope scan shows no uptake.
Choose the most likely diagnosis
A) Thyroglossal cyst
B) De Quervain's cyst
C) Hypothyroidism
D) Lymphoma
E) Simple goiter
ANSWER: B
A 27-yr-old woman was found to have glycosuria at a routine antenatal clinic visit. A
glucose tolerance test confirmed diagnosis of gestational diabetes. Choose the most
likely management.
A) Metformin
B) No change in treatment required
C) IV insulin sliding scale
D) Gliclazide
E) Dietary adjustment
ANSWER: E
A 28 years old patient, complaints on cough with small amount of colourless sputum,
pain in the right half of thorax during breathing, shortness of breath, increase of
temperature to 39 °С. Felt ill rapidly. Used aspirin. Objectively: herpes on lips. In
lower lobe of right lung there is dull percussion sound, bronchial breathing. X-ray:
there is homogeneous infiltration of right lower lobe. What is the most possible
etiology of pneumonia?
A) Pneumococcus
B) Staphylococcus
C) Mycoplasma
D) Legionella
E) Klebsiella
ANSWER: A
A 29 year-old man has Crohns ileocolitis complicated by perianal fistulae. Ileocecal
resection was performed 5 years ago. Approximately 4 months ago, he developed
abdominal pain, cramping, and a new enterocutaneous fistula. An upper gastrointestinal
series with small bowel follow-through showed 4 cm of inflammatory changes at the
neoterminal ileum plus an enterocutaneous fistula originating from this area. After
treatment with metronidazole and mesalamine, his abdominal pain improved and the
fistula appeared to resolve. Which of the following is most appropriate for
maintaining remission in this patient?
A) Start 6-mercaptopurine
B) Continue metronidazole
C) Start budesonide
D) Start prednisone
E) tart oral cyclosporine
ANSWER: A
A 29 year-old woman has hemoglobin level 7.8 g/dl, with a reticulocyte count of 1 %.
The peripheral blood smear showed microcytic hypochromic anemia. The serum iron was
low and the total iron binding capacity was high. The most likely cause of anemia is
A) Iron deficiency anemia.
B) Beta-thalassemia minor.
C) Sideroblastic anemia.
D) Anemia due to chronic infection.
E) Porphyria
ANSWER: A
A 29-year-old patient complains of weight loss, night sweating, general weakness,
fatigue, fever. CBC reveals: RBC 1.1 х 1012/L, Hb 48 g/L, Pl 40 х 109/L, WBC 138,6 х
109/L. In bone marrow - Blasts with Auer rods 70 %. What is the most likely diagnosis?
A) Acute nonlymphoblastic (myeloid) leukemia
B) Chronic lymphocytic leukemia III st
C) Chronic myeloid leukemia, accelerated phase
D) Chronic myeloid leukemia, chronic phase
E) Chronic lymphocytic leukemia IV st
ANSWER: A
A 30-yr-old woman presents with a 2 cm thyroid nodule. FNAC suggests a colloid nodule.
Choose the most likely management.
A) Total thyroid lobectomy
B) Ablative dose of radioactive iodine
C) External beam radiation
D) Chemotherapy
E) Reassure and repeat FNAC in 1 yr
ANSWER: E
A 30-yr-old woman presents with weight gain, constipation, lethargy and flaky rash.
Choose the most likely diagnosis
A) Thyroglossal cyst
B) De Quervain's cyst
C) Hypothyroidism
D) Lymphoma
E) Simple goiter
ANSWER: C
A 31-year-old man has a 2-month history of abdominal pain and bloating. An upper
gastrointestinal series with small bowel follow-through shows ulcerations and
inflammatory changes in the distal 12 cm of the terminal ileum. Colonoscopy is normal
except for erythema and small linear ulcerations seen on cannulation of the terminal
ileum. Which of the following is the most appropriate therapy for this patient at this
time?
A) pH-release mesalamine
B) Mesalamine enemas
C) Balsalazide
D) Olsalazine
E) Sulfasalazine
ANSWER: A
A 31-year-old woman with ulcerative colitis has been taking prednisone for the past
year. Each time the prednisone is tapered below 20 mg/d, her symptoms return. She is
subsequently started on 6-mercaptopurine, 50 mg/d. Three days after beginning the new
drug, she develops worsening abdominal pain with radiation to her back. She does not
have a rash. Her leukocyte count is 3200/?L. Which of the following is the most likely
cause of this patients new symptoms?
A) Pancreatitis due to initiation of 6-mercaptopurine
B) Pancreatitis due to continuation of prednisone
C) Flare of ulcerative colitis
D) An abdominal and psoas abscess secondary to 6-mercaptopurine-induced neutropenia
E) None of above
ANSWER: A
A 32 -year-old woman complained on epigastric pain just after meal, heartburn, and
nausea. Stomach endoscopy revealed a large amount of mucus, hyperemia, erosions of
mucous membrane of stomach. Helicobacter infection test is positive.
A) Chronic type A gastritis
B) Peptic ulcer of the stomach
C) Chronic type C gastritis
D) Menetrier’s disease
E) Chronic type B gastritis
ANSWER: E
A 32-year-old father of two is involved in a head-on motor vehicle accident and is
admitted to your hospital. He has sustained a blunt force trauma to the upper thorax
but injuries appear only to be several cracked ribs and a minor concussion. On
hospital day 3, he reports the uncontrollable urge to drink copious amounts of water,
states that he has been urinating much more frequently and that his urine is very thin
in appearance. The blood and a urin analysis are normal except for high plasma
osmolarity and low urine osmolarity. You decide to do a water deprivation test. Thus
for 24 hours, the patient is restricted to a sip of water every hour, and on day 2 of
the test is clinically judged to be dehydrated. Vasopressin (ADH) is then given by IV
and his urine osmolarity is seen to rise above a baseline dehydrated state reading.
The most likely diagnosis is central diabetes insipidus. What is the next step in the
management of this patient?
A) Hydrochlorothiazide (HCTZ).
B) Furosemide (Lasix).
C) Hydrocortisone orally.
D) Aggressive fluid replacement.
E) Desmopressin (DDAVP).
ANSWER: E
A 32-year-old woman has a 4-month history of hoarseness and throat clearing.
Evaluation by an otolaryngologist disclosed laryngeal inflammation suggestive of
gastroesophageal reflux disease, and the patient is referred to you. She is otherwise
asymptomatic and denies heartburn, regurgitation, dysphagia, and weight loss. The
patient maintains an active lifestyle and currently takes no medications. Physical
examination and routine laboratory studies are normal. Which of the following should
be done next?
A) Upper endoscopy
B) Esophageal manometry
C) Ambulatory 24-hour esophageal pH monitoring
D) Barium swallow
E) Trial of acid suppressive therapy
ANSWER: E
A 32-yr-old male presents with nausea and acute abdominal pain boring through to the
back. The epigastrium is very tender. He has had three similar bouts in the past 18
months. A barium meal is normal. Choose the single most likely diagnosis
A) Gastric atrophy
B) Gastric ulcer
C) Basal pneumonia
D) Chronic cholecystitis
E) Non-ulcer dyspepsia
ANSWER: E
A 32-yr-old man presents with spontaneous bruising and recurrent infections with
marked lethargy. There is no recent treatment history. His Hb is 56 g/l, RBC 2,8 x
1012/L, WBC 2.8 platelets 48,5. What is the diagnosis?
A) Aplastic anaemia
B) Autoimmune hemolytic anaemia
C) Anaemia of chronic disease
D) Iron deficiency
E) Pernicious anaemia
ANSWER: A
A 32-yr-old woman has had type 1 diabetes for 15 yr. She injects Isophane insulin
twice a day and rarely tests her blood glucose at home. She attends the diabetic
clinic for the first time in over a year and informs you that she is 12 weeks pregnant.
Choose the most likely management.
A) Metformin
B) No change in treatment required
C) IV insulin sliding scale
D) One long-acting and 3 short- acting insulin
E) Dietary adjustment
ANSWER: D
A 33-year-old woman presents to the clinic with a positive home pregnancy test. The
patient has a history of Hashimoto’s thyroiditis, which has been successfully managed
with levothyroxine 125 µg daily for the past 4 years; serum thyroid-stimulating
hormone (TSH) levels have been between 0.5 and 1.5 µIU/mL (normal, 0.3-5.5 µIU/mL).
She has a family history of thyroid disease, and her mother also takes levothyroxine.
In the office, the pregnancy test is confirmed; this is her first pregnancy. To remain
biochemically and clinically euthyroid, how should this patient be managed?
A) Levothyroxine should be increased
B) Levothyroxine should be decreased
C) Triiodothyronine (T3) should be taken in addition to levothyroxine
D) The pregnancy should be terminated
E) None of above
ANSWER: A
A 34-year-old male presents with substernal discomfort. The symptoms are worse after
meals, particularly a heavy evening meal, and are sometimes associated with hot/sour
fluid in the back of the throat and nocturnal awakening. The patient denies difficulty
swallowing, pain on swallowing, or weight loss. The symptoms have been present for 6
weeks; the patient has gained 20 lb in the past 2 years. Your initial approach is
A) therapeutic trial of ranitidine
B) Exercise test with thallium imaging
C) Esophagogastroduodenoscopy
D) CT scan of the chest
E) Barium contrast study is indicated
ANSWER: A
A 34-year-old woman is evaluated because of a 1-year history of increased dyspnea on
exertion. She has no symptoms at rest but has to stop about 15 minutes into her
aerobics class because of dyspnea and occasional cough. She usually recovers fully in
about an hour. One year ago she was able to do aerobics for 45 minutes without
difficulty. Her vital signs are normal, and her physical examination is normal,
including clear breath sounds. Baseline spirometry is also normal. Which of the
following would be best next step in the management of this patient?
A) Inhaled albuterol prior to exercise
B) Oral leukotriene inhibitors
C) Long-acting theophylline
D) Inhaled ipratropium bromide prior to exercise
E) Inhaled corticosteroids
ANSWER: A
A 34-year-old woman is hospitalized after a minor episode of hematemesis. She denies
current use of nonsteroidal anti-inflammatory drugs. Two years ago, a duodenal ulcer
and Helicobacter pylori infection were diagnosed by upper endoscopy, and bismuth
subsalicylate, metronidazole, tetracycline, and ranitidine were prescribed for 14 days.
However, the patient took the medications for only 8 days because of nausea. Upper
endoscopy performed during the current admission shows a clean-based duodenal ulcer,
and rapid urease testing of an endoscopic mucosal biopsy specimen is positive for H.
pylori. Which of the following is the most appropriate management at this time?
A) Bismuth subsalicylate, metronidazole, tetracycline, and ranitidine for 4 days
B) Bismuth subsalicylate, metronidazole, amoxicillin, and ranitidine for 14 days
C) A proton pump inhibitor for 14 days
D) A proton pump inhibitor, clarithromycin, and amoxicillin for 14 days
E) Ranitidini (H2-histaminoblokers)
ANSWER: D
A 34-year-old woman is scheduled to undergo flexible sigmoidoscopy because of bloody
bowel movements of 4 days duration. She has had diarrhea for about 10 days, but 4 days
ago she developed frequent, low-volume bloody stools that are associated with urgency.
The patient has had episodic diarrhea for about 3 months, during which time she noted
and ignored bloody bowel movements on four occasions. Stool specimens obtained 3 days
ago are negative for pathogens. Sigmoidoscopic findings include inflammation from the
rectum to 20 cm with no areas of normal intervening mucosa. Which of the following is
the most likely diagnosis?
A) Ulcerative proctocolitis
B) Crohn’s disease
C) Irritable bowel syndrome
D) lschemic colitis
E) None of above
ANSWER: A
A 34-yr-old man presents with insidious onset weakness and weight loss. On examination,
he has hyperpigmentation of the palmar creases and postural hypotension. Choose the
most likely investigation
A) Dexamethasone suppression test
B) ACTH stimulation test
C) Fasting blood glucose
D) T3, T4 and TSH levels
E) Water deprivation test
ANSWER: B
A 34-yr-old woman points to an area of acute epigastric pain with his right index
finger. The pain is worse at night and taking food relieves it. Taking antacids also
relieves it. Choose the single most likely diagnosis
A) Gastric atrophy
B) Acute pancreatitis
C) Basal pneumonia
D) Chronic cholecystitis
E) Esophagitis
ANSWER: E
A 35 -year-old man complained of squeezed epigastric pain 1 hour after meal and
heartburn. She has been ill for 2 years. On palpation, there was moderate tenderness
in pyloroduodenal area. Antral gastritis was revealed on gastroscopy. Which study can
establish genesis of the disease?
A) USD of abdomen
B) Gastrin level in blood
C) Examination of stomach secretion
D) Examination of stomach motor function
E) Revealing of Helicobacter infection in gastric mucosa
ANSWER: E
A 35 -year-old patient complains on severe weakness and fever. Disease started with
pain in the throat and fever. Physical examination: skin and mucous are pale,
ecchymosis, purpura of skin and subcutaneous tissue. Peripheral blood smear: RBC - 1,5
х 1012 /L, Нв - 50 g/L. Blast – 90 %,. What is the most likely diagnosis?
A) Acute leukemia
B) Thrombocytopenic Purpura
C) Hypoplastic Anemia
D) AIDS
E) Leukemoid reaction
ANSWER: A
A 35-year-old man complains to a physician of chronic vague gastric pain of several
years' duration. The pain is sometimes relieved by food. Which of the following is the
most appropriate test to noninvasively determine the H. pylori?
A) Culture of gastric biopsy
B) Rapid urease test
C) Repeat qualitative IgA and IgG antibodies against H. pylori
D) Repeat quantitative IgA and IgG antibodies against H. pylori
E) Urea breath test
ANSWER: E
A 35-year-old man consults a gastroenterologist because of chronic heartburn for
several years. The heartburn tends to be worse at night, and he frequently tastes
refluxed gastric contents when he goes to bed. He found that his symptoms were a
little better when he avoided his customary late evening alcoholic drink; however,
this modest improvement has subsequently deteriorated. The gastroenterologist performs
esophageal manometry with pH monitoring, which demonstrates decreased pressure of the
lower esophageal sphincter and the presence of acid in the esophagus. Biopsy of the
proximal end of one of these fingers shows surface epithelium with regular columnar
cells with small, ovoid nuclei admixed with goblet cells. Which of the following is
the most likely diagnosis?
A) Achalasia
B) Barrett esophagus
C) Corrosive esophagitis
D) Esophageal adenocarcinoma
E) Zenker diverticulum
ANSWER: B
A 35-year-old patient with alcohol abuse and abnormal diet complains of pain in
epigastrium that occurs in 1-1,5 hours after having meal. Esophagogastroduodeno-scopy:
marked hyperemia, small defects, easy appearance of sores on the mucous membrane in
the antral section of the stomach. What is the most probable reason for this pathology?
A) Helicobacter pуlori infection
B) Presence of antibodies to parietal cells
C) Alimentary factor
D) Nervous overstrain
E) Toxic action of alcohol
ANSWER: A
A 35-year-old woman who underwent successful therapy for acute leukemia 10 years ago
is seropositive for hepatitis C. Polymerase chain reaction testing confirms the
presence of virus in a blood sample. She has mildly elevated serum transaminases. A
liver biopsy discloses moderately severe chronic hepatitis without fibrosis. The most
appropriate therapy is
A) interferon plus ribavirin
B) acyclovir
C) ribavirin
D) propranolol
E) no therapy is indicated
ANSWER: A
A 35-year-old woman with amenorrhea is found to have an enlarged pituitary glan. Her
prolactin level is 80 ng/L (normal less than 20 ng/L), and her thyrotropin level is
100 mlU/ml (normal, 0.5 to 4.5 mlU/ml). Which of the following is the best treatment
option for this patient?
A) Administration of bromocriptine
B) Administration of L-thyroxine
C) Irradiation of the pituitary gland
D) Resection of the pituitary gland
E) Use of oral contraceptives
ANSWER: B
A 35-yr-old woman with bone pain, drowsiness and thirst: Calcium—3.3, phosphate—0.75,
ALP—190, PTH—low-normal, PTH—activity high, glucose—6 mmol/l. Choose the most likely
diagnosis
A) Primary hyperparathyroidism
B) Tertiary hyperparathyroidism
C) Hypoparathyroidism
D) Hyperthyroidism
E) Hyperparathyroidism with ectopic PTH
ANSWER: E
A 36-yr-old woman presented with generalised pruritus for 5 months. On examination she
was tanned and there were spider naevi on her chest. The liver was palpable one finger
breadth below the costal margin as well as the tip of the spleen. Choose the single
most likely diagnosis from the list of options above.
A) Wilson's disease
B) Primary biliary cirrhosis
C) Cholecystitis
D) Haemochromatosis
E) Alcoholic liver cirrhosis
ANSWER: B
A 37-year-old man complains of pains in epigastrium which are relieved by food intake.
EGDS shows antral erosive gastritis, biopsy of antral mucous presents Hеlicobacter
Pylori. Diagnosis is:
A) Gastritis of type A
B) Reflux - gastritis
C) Menetrier's gastritis
D) Rigid antral gastritis
E) Gastritis of type B
ANSWER: E
A 37-year-old man with asthma is evaluated because he continues to have frequent
attacks and now feels his short-acting ?2-agonist is not providing relief. He states
he is using his medications, including a long-acting ?2-agonist inhaler, inhaled high-
dose corticosteroids, and a short-acting ?2-agonist inhaler as rescue medication. He
has symptoms daily and nocturnal symptoms about twice per week. On physical
examination, he is in mild respiratory distress. He is afebrile. Pulse rate is 90/min
and regular, respiration rate is 18/min, and blood pressure is 140/85 mm Hg. He has
bilateral wheezing. Spirometry shows a forced expiratory volume in 1 sec (FEV1) 65% of
predicted; it improves with bronchodilators to 85% of predicted. He has no history of
recent viral upper respiratory infections or rhinitis or symptoms of gastroesophageal
reflux disease. Which of the following is the best next step in this patient’s
management?
A) Observe the patient using the metered-dose inhaler
B) Add a leukotriene inhibitor
C) Switch to an oral ?2-agonist and have the patient return for a pill count
D) Initiate oral prednisone therapy and have the patient return for a pill count
E) Have the patient return with a symptom and treatment log.
ANSWER: A
A 37-year-old patient complains of epigastric pain that occurs in 1-1,5 hours after
having meal. Esophagogastroduodeno-scopy: marked hyperemia, small defects, easy
appearance of sores on the mucous membrane in the antral section of the stomach. What
is the most probable reason for this pathology?
A) Helicobacter pуlori infection
B) Presence of antibodies to parietal cells
C) Alimentary factor
D) Nervous overstrain
E) Toxic action of alcohol
ANSWER: A
A 37-year-old woman has a 4-month history of epigastric discomfort and heartburn.
Symptoms are usually exacerbated postprandially, especially if she eats spicy foods.
The patient denies dysphagia, weight loss, and decreased appetite. Treatment with a
proton pump inhibitor, once daily for 4 weeks, resulted in only minimal improvement.
Increasing the medication to twice daily for an additional 4 weeks did not improve her
symptoms, and the patient wants to know what other management options are available.
Referral for which of the following procedures is most appropriate at this time?
A) Ambulatory 24-hour esophageal pH monitoring
B) Upper endoscopy with esophageal dilation
C) Barium swallow
D) Surgical fundoplication
E) USD
ANSWER: A
A 37-year-old woman has a 4-month history of epigastric discomfort and heartburn.
Symptoms are usually exacerbated postprandially, especially if she eats spicy foods.
The patient denies dysphagia, weight loss, and decreased appetite. Treatment with a
proton pump inhibitor, once daily for 4 weeks, resulted in only minimal improvement.
Increasing the medication to twice daily for an additional 4 weeks did not improve her
symptoms, and the patient wants to know what other management options are available.
Referral for which of the following procedures is most appropriate at this time?
A) Upper endoscopy with esophageal dilation
B) Barium swallow
C) Surgical fundoplication
D) USD
E) Ambulatory 24-hour esophageal pH monitoring
ANSWER: E
A 37-year-old woman sees her physician because of gum bleeding, menorrhagia. Physical
examination reveals petechiae, bruises on her legs. CBC: RBC 2.5 х 1012/L, Hemoglobin
67 g/L, MCV 64 fl, CI – 0,7, Pl 41 х 109/L, Giant platelets +, WBС 7.9 х 109/L, ESR 7
mm/h, Bleeding time 17 min. What is the most probably diagnosis?
A) Henoch–Schonlein purpura
B) Thrombotic thrombocytopenic purpura
C) Chronic lymphocytic leukemia
D) Macrocytic hyperchromic anemia
E) Idiopathic thrombocytic purpura
ANSWER: E
A 37-yr-old woman presents with weight loss, muscular weakness, oligomenorrhea,
diarrhea and blurring of vision. On examination, there is exophthalmos and proximal
myopathy. Choose the most likely diagnosis
A) Thyroglossal cyst
B) Graves' disease
C) Hypothyroidism
D) Lymphoma
E) Simple goiter
ANSWER: B
A 38-year-old former hemodialysis nurse is seen because of a 6-month history of
fatigue and amenorrhea. On examination she has scleral icterus, a mildly tender liver,
and a tibial rash consistent with erythema nodosum. ALT and AST levels are both in the
range increased, while alkaline phosphatase and serum albumin levels are normal.
Hepatitis serologic testing detects HBsAg and IgG anti-HBcAg. Liver biopsy discloses a
mononuclear cell portal infiltrate and hepatocyte destruction at the periphery of
lobules. Which of the following therapeutic strategies is best?
A) Administration of interferon, 10 million units three times per week for 4 months
B) Administration of prednisone, 20 to 40 mg/d for 2 months and then a taper based on
the response
C) Administration of prednisone, 10 mg every other day for 3 months
D) Administration of acyclovir, 400 mg every 6 h for 2 weeks
E) Administration of low-dose cyclophosphamide, 50 mg/d for 2 months
ANSWER: A
A 38-year-old male insurance agent with a benign past medical history presents to his
primary care physician complaining of indigestion. He notes that “heartburn” has
occurred weekly for about 1 year, especially after eating a heavy meal. He has no risk
factors for coronary artery disease and does not complain of weight loss, vomiting,
dysphagia, or bleeding. Physical and routine laboratory examinations are unrevealing.
Which of the following is the most appropriate next step?
A) Upper gastrointestinal barium radiography
B) Upper gastrointestinal endoscopy
C) Ambulatory esophageal pH testing
D) Serology for H. Pylori
E) Prescribe omeprazole
ANSWER: D
A 38-year-old male insurance agent with a benign past medical history presents to his
primary care physician complaining of indigestion. He notes that “heartburn” has
occurred weekly for about 1 year, especially after eating a heavy meal. He has no risk
factors for coronary artery disease and does not complain of weight loss, vomiting,
dysphagia, or bleeding. Physical and routine laboratory examinations are unrevealing.
Which of the following is the most appropriate next step?
A) Upper gastrointestinal endoscopy
B) Ambulatory esophageal pH testing
C) Upper gastrointestinal barium radiography
D) Prescribe omeprazole
E) Serology for H. Pylori
ANSWER: E
A 38-year-old man complains of chronic heartburn for several years. The heartburn
tends to be worse at night, and he frequently tastes refluxed gastric contents when he
goes to bed. He found that his symptoms were a little better when he avoided his
customary late evening alcoholic drink; however, this modest improvement has
subsequently deteriorated. The esophageal manometry with pH monitoring demonstrates
decreased pressure of the lower esophageal sphincter and the presence of acid in the
esophagus. Biopsy of the proximal end of esophagus shows surface epithelium with
regular columnar cells with small, ovoid nuclei admixed with goblet cells. Which of
the following is the most likely diagnosis?
A) Achalasia
B) Barrett esophagus
C) Corrosive esophagitis
D) Esophageal adenocarcinoma
E) Zenker diverticulum
ANSWER: B
A 38-year-old man who works as a reporter for a travel magazine comes to his physician
because of the acute onset of jaundice, malaise, and temperatures to 38.5 С .He had
returned from Burma 2 weeks ago, where he spent 4 weeks. He says that he abstains from
alcohol beverages and does not take any medications. Laboratory studies show elevated
serum aminotransferases, high bilirubin (both total and direct), and negative serology
for hepatitis A virus (HAV) and С virus (HCV) infection. He was vaccinated for
hepatitis В virus (HBV) 3 years ago and is now positive for anti-HBsAg antibodies.
Which of the following serologic markers should be tested as the most appropriate next
step in diagnosis?
A) Anti-HCV IgG antibodies by RIBA
B) Anti-HDV IgG antibodies
C) Anti-HEV IgM antibodies
D) Anti-HGV IgG antibodies
E) HBsAg
ANSWER: C
A 39-year-old patient complains of weight loss, night sweating, general weakness,
fatigue, fever, gum bleeding. Bone marrow examination reveals blasts with Auer rods
87 %. What is the most likely diagnosis?
A) Acute nonlymphoblastic (myeloid) leukemia
B) Erythremia
C) Chronic lymphocytic leukemia
D) Chronic myeloid leukemia
E) Multiple myeloma
ANSWER: A
A 39-yr-old woman presented with jaundice and painless depigmented patches on her
hands, neck and face. On exami_nation multiple spider naevi were found. Choose the
single most likely diagnosis from the list of options above.
A) Haemochromatosis
B) Primary biliary cirrhosis
C) Wilson's disease
D) Chronic active hepatitis
E) Pancreatic carcinoma
ANSWER: D
A 40-year-old male with long-standing alcohol abuse complains of abdominal swelling,
which has been progressive over several months. He has a history of gastrointestinal
bleeding. On physical exam, there are spider angiomas and palmar erythema. Abdominal
collateral vessels are seen around the umbilicus. There is shifting dullness, and
bulging flanks are noted. An important first step in the patient’s evaluation is
A) Diagnostic paracentesis
B) UGI series
C) Ethanol level
D) CT scan
E) Esophagogastroduodenoscopy
ANSWER: A
A 40-year-old white female complains of pruritus. She has an elevated alkaline
phosphatase and positive antimitochondrial antibody test. What is the most likely
disease?
A) Primary biliary cirrhosis
B) Sclerosing cholangitis
C) Anaerobic liver abscess
D) Hepatoma
E) Hepatitis C
ANSWER: A
A 40-year-old woman, a nurse, is evaluated because of worsening asthma symptoms. She
has had mild, intermittent asthma since college, for which she has been using an
albuterol inhaler as needed, usually less than once a month. During the past 3 months,
she has experienced cough, tightness of the chest, and wheezing, which improve after
the use of inhaled albuterol. She uses the inhaler twice a day on average and has
awakened at least twice a week with nocturnal cough. She works three consecutive 12-
hour day shifts, and the cough is regularly worse at the end of each shift. During her
days off, she has fewer asthma symptoms and feels significantly better by the time she
returns to work. She has a history of allergic rhinitis that has also recently become
more symptomatic. Approximately 6 months ago, she acquired a kitten that sleeps in the
bedroom. She has lived in her home for 6 years, and it is carpeted and has heavy
draperies. Chest examination is notable for good air entry. There are scattered end-
expiratory wheezes. In addition to treatment with inhaled corticosteroids, which of
the following interventions is most likely to benefit this patient?
A) Avoiding exposure to latex products
B) Treatment with an oral antihistamine
C) Getting rid of the kitten
D) Removing the carpets and draperies from her home
E) Treatment with a leukotriene-modifying drug
ANSWER: A
A 40-yr-old diabetic actor is started on Propanolol for stage fright. He collapses
after a day shooting. He has not changed his insulin regimen. Choose the most likely
management.
A) Sugary drink
B) Insulin sliding scale, Heparin and 0.45% saline
C) Insulin sliding scale, 0.9% NS and potassium replacement
D) Chest X-ray
E) 50 ml of 50% dextrose IV
ANSWER: E
A 40-yr-old man presents to his GP complaining of change in appearance and headaches.
His brow is more prominent and his nose had broadened. He states that his shoes are
too small and he has tingling in certain fingers worse at night. Choose the most
likely management.
A) Propanolol
B) Calciferol
C) Carbimazole
D) Thyroxine
E) Octreotide (somatostatin)
ANSWER: E
A 40-yr-old woman has a long history of pruritus, arthralgia and mild jaundice. She
presents with haematemesis and is found to have splenomegaly. Endoscopy shows
oesophageal varices. Choose the single most likely diagnosis from the list of options
above. Choose the single most likely diagnosis from the list of options above.
A) Portal hypertension
B) Myelofibrosis
C) Infective endocarditis
D) None of above
E) Gilbert's syndrome
ANSWER: A
A 40-yr-old woman presents with a hard, nodular midline neck mass. Blood tests reveal
the presence of antibodies to thyroglobulin. Choose the most likely diagnosis.
A) Thyroglossal cyst
B) Hashimoto's thyroiditis
C) Toxic multinodular goiter
D) Thyroid storm
E) Graves' disease
ANSWER: B
A 40-yr-old woman presents with a solitary nodule in the right thyroid lob. FNAC
suggests follicular adenoma. Choose the most likely management.
A) Total thyroid lobectomy
B) Ablative dose of radioactive iodine
C) External beam radiation
D) Chemotherapy
E) Reassure and repeat FNAC in 1 yr
ANSWER: A
A 40-yr-old woman presents with fatigue, dyspnoea, paraesthesiae and a sore red tongue.
Her blood film shows hypersegmented polymorphs, an MCV of > 110 fl and a low Hb. What
is your diagnosis?
A) Pernicious anaemia
B) Iron-deficiency anemia.
C) Sickle cell anaemia
D) Hemolytic anemia.
E) Aplastic anemia.
ANSWER: A
A 41-year-old morbidly obese female comes to the emergency department with colicky
abdominal pain in her right upper abdomen. She complains that this is similar to, yet
more severe than, the pain that often occurs after meals for the past 4 months. Her
past medical history is positive for diabetes mellitus type 2, hypertension,
hyperlipidemia, and smoking. On physical exam, her temperature is 100.5°F (38.1°C)
and her sclera appear mildly icteric. What imaging modality may be limited in this
patient?
A) MRCP.
B) CT scan.
C) Ultrasonography.
D) Esophogastroduodenoscopy (EGD).
E) IDA scan.
ANSWER: C
A 42-year-old patient complains of weight loss, night sweating, general weakness,
fatigue. Physical examination reveals lymphadenopathy, splenomegaly. CBC reveals: RBC
2.34 х 1012/L, Hemoglobin 78 g/L, Hematocrit 40 %, MCV 90 fl, Reticulocyte Count 1.0 %,
Pl 245 х 109/L, WBC 71.9 х 109/L, Segmented Neutrophils 2 %, Band Neutrophils 0 %,
Monocytes -0% Eosinophils -0 %, Basophils -0%, Lymphocytes 98%, ESR 24mm/h,Smudge
cells +. What is the most likely diagnosis?
A) Acute leukemia
B) Chronic lymphocytic leukemia III st
C) Chronic lymphocytic leukemia I st
D) Chronic myeloid leukemia
E) Chronic lymphocytic leukemia IV st
ANSWER: B
A 42-year-old patient complains of weight loss, night sweating. CBC reveals: RBC 3.75
х 1012/L, Hb 125 g/L, Ht 42 %, MCV 91 fl, -Reticulocyte Count 1.0 %, Pl 280 х 109/L,
WBC 47.9 х 109/L, Segmented Neutrophils 3 %, Band Neutrophils 1 %, Monocytes 2 %,
Eosinophils 1 %, Basophils 0 %, Lymphocytes 93 %, ESR 7 mm/h, Smudge cells +. What is
the most likely diagnosis?
A) Acute leukemia
B) Erythremia
C) Chronic lymphocytic leukemia
D) Chronic myeloid leukemia
E) Multiple myeloma
ANSWER: C
A 42-year-old patient complains of weight loss, night sweating. Physical examination
reveals lymphadenopathy, splenomegaly. CBC reveals: RBC 3.75 х 1012/L, Hb 125 g/L, Ht
42 %, MCV 91 fl, -Reticulocyte Count 1.0 %, Pl 280 х 109/L, WBC 47.9 х 109/L,
Segmented Neutrophils 3 %, Band Neutrophils 1 %, Monocytes 2 %, Eosinophils 1 %,
Basophils 0 %, Lymphocytes 93 %, ESR 7 mm/h, Smudge cells +. What is the most likely
diagnosis?
A) Acute leukemia
B) Chronic lymphocytic leukemia III st
C) Chronic lymphocytic leukemia II st
D) Chronic myeloid leukemia
E) Chronic lymphocytic leukemia IV st
ANSWER: C
A 42-year-old patient suffering from alco_holism has advanced liver disease with
ascites. Examination reveals asterixis of the hands, ankle clonus, and spider angiomas
on the face and chest. Precipitat_ing factors to look for include all of the
fol_lowing EXCEPT:
A) heart insufficiency
B) bleeding esophageal varices
C) excessive diuretic therapy
D) non-compliance with lactulose therapy
E) spontaneous bacterial peritonitis
ANSWER: A
A 42-yr-old man has hypertension, hyperglycaemia, myopathy, thinning of the skin,
buffalo hump and truncal obesity. Choose the most likely investigation
A) Dexamethasone suppression test
B) ACTH stimulation test
C) Fasting blood glucose
D) T3, T4 and TSH levels
E) Water deprivation test
ANSWER: A
A 44-yr-old mother of five children complained of yellow skin and abdominal pain,
especially after meals. She was overweight and she said that she did not like going
out to restaurants because of embarrassing flatulence. Choose the single most likely
diagnosis from the list of options above.
A) Gallstones
B) Hepatitis
C) Carcinoma of bile duct
D) Alcoholism
E) Carcinoma pancreas
ANSWER: A
A 44-yr-old woman complains of intense pruritus and yellowing of her skin. On physical
examination you notice xanthomata and skin pigmentation. Choose the single most likely
diagnosis from the list of options above.
A) Primary biliary cirrhosis
B) Hepatitis
C) Carcinoma of bile duct
D) Alcoholism
E) Haemolytic anaemia
ANSWER: A
A 44-yr-old woman presents with tachycardia, atrial fibrillation, double vision and
swelling above her ankles. She has lid lag on examination. Choose the most likely
investigation
A) Dexamethasone suppression test
B) ACTH stimulation test
C) Fasting blood glucose
D) T3, T4 and TSH levels
E) Water deprivation test
ANSWER: D
A 45 year-old man undergoes a routine physical examination with screening blood
studies. Physical examination is notable for an increased liver diameter; the liver
edge is palpable and without irregularities. Blood studies show elevated liver enzymes.
The clinician suspects alcoholic hepatitis. Which of the following findings would tend
to support this diagnosis?
A) Alanine aminotransferase = 2000 U/L
B) Aspartate aminotransferase (AST)/alanine aminotransferase (ALT) ratio = 2.5
C) Gamma-glutamyl transferase (GGT) = 20 U/L (norm
D) Mean corpuscular volume (MCV) = 65 urn3
E) Platelet count = 600,000/mm3
ANSWER: B
A 45-year-old female with long-standing alcohol abuse complains of abdominal swelling,
which has been progressive over several months. On physical exam, there are spider
angiomas and palmar erythema. Abdominal collateral vessels are seen around the
umbilicus. A paracentesis is performed. The serum albumin minus ascitic fluid albumin
equals 1.4 g/dL. The most likely diagnosis is
A) Portal hypertension
B) Pancreatitis
C) Tuberculous peritonitis
D) Hepatoma
E) No ascitis
ANSWER: A
A 45-year-old man for 1 month has complained of epigastric and right subcostal aching
pain, pruritus, indigestion, dark color of the urine and acholic stool, fever, and
significant weight loss. On exam: jaundice, presence of Curvuasier’s sign. US scan did
not reveal stones in the gallbladder and choledochus. What is the most likely
diagnosis?
A) Cancer of the pancreas head
B) Gallbladder stones
C) Chronic pancreatitis
D) Chronic cholangitis
E) Chronic hepatitis
ANSWER: A
A 45-year-old man with history of high blood pressure that has been difficult to
control with a variety of antihypertensive medications presents with persistent
headaches, excessive sweating, and palpitations. Routine blood tests are normal.
Electrocardiogram shows supraventricular tachycardia. What is this patient’s most
likely diagnosis?
A) Pheochromocytoma
B) Primary hyperaldosteronism
C) Cushing’s disease
D) Hyperthyroidism
E) Malignant hypertension
ANSWER: A
A 45-year-old man with history of high blood pressure that has been difficult to
control with a variety of antihypertensive medications presents with persistent
headaches, excessive sweating, and palpitations. Routine blood tests are normal.
Electrocardiogram shows supraventricular tachycardia. It was suspected
pheochromocytoma. What is this patient’s most likely test?
A) cortisol level
B) aldosteron level
C) 17-hydroxycorticosteroids
D) urine metanephrines
E) TSH
ANSWER: D
A 45-year-old man with history of high blood pressure that has been difficult to
control with a variety of antihypertensive medications presents with persistent
headaches, excessive sweating, and palpitations. Routine blood tests are normal.
Electrocardiogram shows supraventricular tachycardia. Which of the following is the
best initial pharmacologic therapy choice for this patient?
A) ?-Blockers
B) Diuretics
C) Steroids
D) Methimazole
E) ?-Blockers
ANSWER: E
A 45-year-old woman complains of right upper quadrant pain, which occurs after she
eats a large meal. Occasionally the episodes are accompanied by nausea and vomiting. A
plain x-ray of the abdomen discloses gallstones. Ultrasonography reveals gallstones
and a normalsized common bile duct. The patient’s blood chemistry and CBC are normal.
The most therapeutic maneuver at this time would be
A) laparoscopic cholecystectomy
B) observation
C) ursodeoxycholic acid
D) shock wave lithotripsy
E) ursodeoxycholic acid and shock wave lithotripsy
ANSWER: A
A 45-yr-old patient complains of giddiness and falls. She also suffers with
intermittent vomiting and sweating and occasional faecal incontinence at night. Choose
the most likely diagnosis
A) Coronary artery disease
B) Cerebrovascular disease
C) Nephropathy
D) Retinopathy
E) Autonomic neuropathy
ANSWER: E
A 45-yr-old woman presents with a diffuse swelling of the thyroid gland. On
examination she has a stare, lid lag, and lid retraction. On the dorsum of her legs
she has areas of raised peau d'orange-like thickened skin. Blood tests reveal thyroid-
stimulating immunoglobulins against the TSH receptor site. Choose the most likely
diagnosis.
A) Thyroglossal cyst
B) Hashimoto's thyroiditis
C) Toxic multinodular goiter
D) Thyroid storm
E) Graves' disease
ANSWER: E
A 46-year-old woman who works as a nurse is evaluated because of a 2-year history of
episodic wheezing and a squeaky voice. This past spring, her symptoms worsened,
requiring her to seek medical attention; she was placed on a short-acting ?2-agonist
that did not provide much relief. She has no history of wheezing and says that these
changes began after a severe influenza infection 3 years ago. Currently she feels well
and has had no symptoms for several months; she is not taking any medications.
Physical examination shows no abnormalities, and baseline spirometry is normal. What
is the best test to evaluate this patient’s condition?
A) Methacholine challenge testing
B) Bronchoscopy to evaluate her trachea
C) Exercise echocardiogram
D) CT scan of the sinuses
E) Non of above
ANSWER: A
A 47-year-old female presents to the health clinic for symptoms of weight she can't
seem to lose despite strict dieting and concerns of menstrual irregularity. She states
she also has developed multiple abdominal stretch marks and has noticed a deepening of
her voice and appearance of facial hair. As part of her initial work up, you obtain a
24-hour urine cortisol level, which is returned as 120 ng/ml (N 20 100 ng/ml). What is
the most likely diagnosis in this patient?
A) Metabolic syndrome.
B) Diabetes mellitus type II.
C) Cushing's syndrom
D) Addison's diseas
E) Conn's syndrom
ANSWER: C
A 47-year-old female presents to the health clinic for symptoms of weight she can't
seem to lose despite strict dieting and concerns of menstrual irregularity. She states
she also has developed multiple abdominal stretch marks and has noticed a deepening of
her voice and appearance of facial hair. As part of her initial work up, you obtain a
24-hour urine cortisol level, which is returned as 120 ng/ml (N 20 100 ng/ml). The
most likely diagnosis in this patient is Cushing's syndrome.What is the most
appropriate next step in diagnosis of this patient?
A) Cosyntropin (ACTH) stimulation test.
B) Dexamethasone suppression test.
C) Cosyntropin (ACTH) suppression test.
D) Dexamethasone stimulation test.
E) Urine catecholamine collection over a 24-hour perio
ANSWER: B
A 47-yr-old agricultural worker complains of a chronic cough, purulent sputum and
abdominal distention. He has just arrived in England from Spain where he was picking
grapes. Choose the single most likely diagnosis from the list of options above.
A) Tuberculosis
B) Cirrhosis
C) Malabsorption
D) Pancreatitis
E) Peptic ulcer
ANSWER: A
A 48 years old patient, complaints on weakness, dyspnea, pain in the left half of
thorax, permanent cough with viscid sputum, in which particles of blood are sometimes
determined. For the last 3 months lost 5 kg of body mass. On the X-ray of lungs there
is total homogeneous shade determined from the left side. Organs of mediastinum are
displaced to the left. What diagnosis is possible?
A) Lung athelectasis
B) Lung gangrene
C) Total exudative pleurisy
D) Pneumonia
E) Empyema of pleura
ANSWER: A
A 48-year-old patient complains of fatigue and easy bruising of 3 weeks’ duration.
Physical findings included pale, scattered ecchymoses and petechiae. RBC – 1.95 x
1012/L; Hb – 73 g/L; HCT 20%; PLT – 23 x 109/L; and WBC – 182 x 109/L with 84% blasts,
that contained Auric rods; peroxidase stain was positive; What is the most probable
diagnosis?
A) Megaloblastic anemia
B) Hemolytic anemia
C) Thrombocytopenia
D) Acute myeloid leukemia
E) Chronic leukemia
ANSWER: D
A 48-year-old woman develops fevers, chills, and icteric sclera. In addition to a
fever of 39.2_C, the physical examination is remarkable for an ill-appearing jaundiced
female with right upper quadrant pain. Ultrasonography reveals a dilated common bile
duct with stones in the gallbladder and in the duct itself. The patient is placed on
broad-spectrum antibiotics to cover organisms known to infect the biliary tract. The
procedure most appropriate now is
A) endoscopic retrograde cholangiopancreatography
B) laparoscopic cholecystectomy
C) placement of an external stent for bilary drainage
D) laparotomy to canulate the common bile duct, remove the stone, and perform a
cholecystectomy
E) antibiotics for several days
ANSWER: A
A 49 years old woman complains of weakness, malaise, anorexia, fever, dental bleeding.
Petechiae on a skin. Laboratory findings: Hb of 70 g/L, RBC of 2.2 x 1012/L, the
platelet count is 30 x 109/L, the WBC 100,5 x 109/L, blasts in peripheral blood smear.
What is the most probable diagnosis?
A) Megaloblastic anemia
B) Acute leukemia
C) Hemolytic anemia
D) Thrombocytopenia
E) Chronic leukemia
ANSWER: B
A 49-yr-old man who enjoys drinking presents with pallor, epistaxis and bleeding. On
physical examination you find the spleen to be enlarged and the liver to be slightly
enlarged. Choose the single most likely investigation from the list of options above.
A) Liver function test
B) Abdominal USG
C) Bone marrow biopsy
D) Blood culture
E) White cell count
ANSWER: A
A 49-yr-old woman presents with goiter. On examination, the thyroid is firm and
rubbery. Thyroid microsomal antibodies are positive in high titer. Choose the most
likely diagnosis
A) Thyroglossal cyst
B) Hashimoto's thyroiditis
C) Hypothyroidism
D) Graves' disease
E) Simple goiter
ANSWER: B
A 50 years old woman complains of weakness, anorexia, fever, gums bleeding. Spleen,
liver, and lymphatic nodes are enlarged, petechiae on a skin. Laboratory findings: the
platelet count is 90 x 109/L, WBC 100 x 109/L, blasts 40%. ? What is the most probable
diagnosis?
A) Acute leukemia
B) Megaloblastic anemia
C) Hemolytic anemia
D) Thrombocytopenia
E) Chronic leukemia
ANSWER: A
A 50-year-old man without significant past medical history or recent exposure to
alcohol presents with midepigastric abdominal pain, nausea, and vomiting. The physical
examination is remarkable for the absence of jaundice and any other specific physical
findings. Which of the following is the best strategy for screening for acute
pancreatitis? Measurement of both serum amylase and serum
A) Lipase
B) Measurement of serum lipase
C) Measurement of serum amylase
D) Isoamylase level analysis
E) None of above
ANSWER: A
A 50-yr-old patient complains of burning pain in the feet, worse at night or on
walking. He describes the sensation as like walking on hot coals. Choose the most
likely diagnosis
A) Coronary artery disease
B) Cerebrovascular disease
C) Nephropathy
D) Retinopathy
E) Polyneuropathy
ANSWER: E
A 50-yr-old woman presents to her GP for fatigue, depression and weight gain. She also
complains of constipation and poor memory. On examination, she has puffy face and
coarse facial features, thin eyebrows and a large tongue. Choose the most likely
management
A) Propanolol
B) Calciferol
C) Carbimazole
D) Thyroxine
E) Octreotide (somatostatin)
ANSWER: D
A 50-yr-old woman presents with a thyroid goiter. A core biopsy reveals evidence of
lymphoma. Choose the most likely management.
A) Total thyroid lobectomy
B) Thyroxine
C) External beam radiation
D) Propranolol
E) Reassure and repeat FNAC in 1 yr
ANSWER: C
A 50-yr-old woman presents with fever, tachycardia restlessness, hypertension and
vomiting. On examination she has diffuse swelling of the thyroid gland and strabismus
with diplopia. Choose the most likely diagnosis.
A) Thyroglossal cyst
B) Hashimoto's thyroiditis
C) Toxic multinodular goiter
D) Thyroid storm
E) Graves' disease
ANSWER: D
A 51-year-old female comes to the emergency department complaining of left lower
quadrant abdominal pain. She describes an acute illness accompanying the pain with
subjective fever and diarrhea over the last 8 hours. Abdominal exam shows tenderness
in the LLQ of the abdomen, no rebound tenderness at McBurney's point, and negative
Murphy's sign. What imaging modality is most appropriate for this patient?
A) Ultrasound of the abdomen.
B) CT scan with and without contras
C) Colonoscopy.
D) Barium enema.
E) Plain upright abdominal x-ray.
ANSWER: B
A 51-year-old woman undergoes colorectal cancer screening. She feels well, has no
significant medical history, takes no medications, and has no family history of
colorectal cancer. Physical examination and complete blood count are normal. Which of
the following is the most appropriate screening program for colorectal cancer in this
patient?
A) Colonoscopy every 10 years
B) Flexible sigmoidoscopy every 2 to 3 years
C) Barium enema examination every 3 years
D) Fecal occult blood testing every 2 to 3 years
E) CT colonography (virtual colonoscopy) every 10 years
ANSWER: A
A 52-yr-old man has been gaining weight. He complains of a chronic cough, acne and
bruising. On physical examination you find his legs and arms to be abnormally thin.
Choose the most likely investigation
A) Chest X-ray
B) Serum cortisol
C) Dexamethasone suppression test
D) Water deprivation test
E) T3, T4 and TSH
ANSWER: C
A 52-yr-old man presents with painless lump in the neck and a chronic cough. Physical
examination finds tachycardia and pallor. He feels that he has lost weight, but he is
not certain. He does not smoke or drink. Choose the most likely diagnosis.
A) Toxic adenoma
B) Adrenal hyperplasia
C) Hyperthyroidism
D) Hypothyroidism
E) Follicular carcinoma
ANSWER: E
A 53-year-old woman with a history of mild persistent asthma is evaluated because of a
recent increase in her symptoms, with dyspnea and cough occurring daily and a cough
that awakens her once a week. She is currently using low-dose inhaled corticosteroids.
She has no symptoms of rhinitis or gastroesophageal reflux. On physical examination,
she has intermittent wheezing bilaterally. Which of the following is the most
appropriate change in her therapy?
A) Add a long-acting ?2-agonist
B) Initiate azithromycin therapy
C) Add a nebulized short-acting ?2-agonist
D) Add inhaled ipratropium bromide
E) Add a leukotriene inhibitor
ANSWER: A
A 55-year-old male alcoholic has recurrent attacks of severe mid-epigastric pain after
eating. Serum amylase determinations after such attacks have been in the normal range.
The examination reveals mild cachexia but is otherwise unremarkable. On further
questioning, the patient states that he has been sober for the past 10 years but prior
to that time had multiple episodes of alcohol-induced pancreatitis. He is currently
taking pancreatic replacement enzymes by mouth. An ERCP reveals a stricture of the
pancreatic duct but is otherwise unremarkable. Computed tomography of the abdomen
reveals calcifications in the pancreas but does not show any evidence of malignancy.
The patient is taking 30 mg of continuous-release morphine sulfate twice a day. The
best strategy at this point would be to
A) resect the head of the pancreas
B) double the dose of pancreatic replacement enzymes
C) double the dose of morphine
D) institute a low-fat diet
E) begin a continuous search for other causes ofabdominal pain
ANSWER: A
A 55-year-old man is evaluated in the emergency department because of an acute, severe
asthma attack; he is hospitalized in the intensive care unit for aggressive medical
therapy and monitoring. He is expectorating thick greenish sputum. His medical history
includes hypertension, cholecystectomy and glaucoma. Chest radiograph reveals
hyperinflation only. Medical therapy in the emergency department included repeated
doses of aerosolized albuterol and ipratropium, as well as methylprednisolone, 125mg
administered intravenously. Peak expiratory flow rate is unimproved at 80 l/min. Which
of the following is the most appropriate next step in this patient’s management?
A) Intravenous magnesium sulfate
B) Nebulized ipratropium bromide administered by face mask
C) Broad-spectrum antibiotics targeting community-acquired respiratory pathogens
D) Inhaled corticosteroids
ANSWER: A
A 55-year-old man with no prior medical history presents with right flank pain. During
an evaluation for kidney stones, he has a computed tomography scan of the abdomen that
reveals a 2 ? 2-cm mass in the left adrenal gland. The patient’s vital signs, kidney
function, and electrolyte levels are within normal limits. Which of the following
tests should be ordered next to evaluate the adrenal mass?
A) Aldosterone and renin levels
B) Random cortisol levels
C) Testosterone levels
D) 24-Hour urine collection for metanephrines
E) No further testing
ANSWER: D
A 55-yr-old man presents with fever, sweats and weight loss. He also suffers from gout
and enlargement of lymphatic nodes, which are nontender, symmetrical. On examination
he has an enlarged spleen and liver. Blood tests reveal a lymphocytosis and anaemia.
The Philadelphia chromosome is negative. What is the most probable diagnosis?
A) ALL
B) CLL
C) Multiple myeloma
D) Polycythemia vera
E) CML
ANSWER: B
A 55-yr-old presented with following reports on a routine screen: Calcium-2.85 mmol/l,
phosphate—0.8, ALP—110, PTH—raised, 25-OH vitamin D—low-normal. Choose the most
likely diagnosis
A) Primary hyperparathyroidism
B) Tertiary hyperparathyroidism
C) Hypoparathyroidism
D) Hyperthyroidism
E) Paget's disease of bone
ANSWER: A
A 56-year-old patient with cirrhosis of the liver presents with massive hemetemesis.
Two large-bore intravenous lines are placed; somatostatin, fluids, and blood products
are administered; and the patient is intubated. Emergency endoscopy reveals bleeding
esophageal varices. The patient becomes stable hemodynamically but is still bleeding.
The most appropriate next step is
A) endoscopic variceal band ligation
B) intravenous vasopressin
C) balloon tamponade
D) endoscopic injection sclerotherapy
E) intravenous propranolol
ANSWER: A
A 57-year-old man with severe persistent asthma is evaluated on routine follow-up. He
states that his asthma has been under good control for the last 3 months on high-dose
inhaled corticosteroids and a long-acting ?2-agonist. He uses a short-acting ?2-
agonist only three times per week as a rescue medication, and he has nocturnal
symptoms very rarely. Peak expiratory flows have been stable. His physical examination
is normal, including clear breath sounds, and spirometry is normal. It is decided that
he will keep using the short-acting ?2-agonist as a rescue medication. Which of the
following is the best next step in this patient’s management?
A) Continue the long-acting ?2-agonist and decrease the dose of inhaled corticosteroid
B) Stop the long-acting ?2-agonist and decrease the dose of inhaled corticosteroid
C) Continue current therapy and have the patient return in 6 months
D) Stop the long-acting ?2-agonist and maintain the dose of inhaled corticosteroid
ANSWER: A
A 58-year-old patient complains of weight loss, night sweating, sensation of fullness
in left hypochondrium. CBC reveals: RBC 3.9х 1012/L, Hb 118g/L, Ht 41%, MCV 88 fl, Pl
645 х 109/L, WBC 89,4х 109/L, Segmented Neutrophils 62%, Band Neutrophils 10%,
Metamyelocytes 3%, Myelocytes 5%, Monocytes 4%, Eosinophils 8%, Basophils 6%,
Lymphocytes 2%, ESR 27mm/h. What is the most likely diagnosis?
A) Acute leukemia
B) Chronic myeloid leukemia, accelerated phase
C) Chronic lymphocytic leukemia
D) Chronic myeloid leukemia, chronic phase
E) Multiple myeloma
ANSWER: D
A 58-year-old woman complains of increasing fatigue and easy bruising of 3 weeks
duration. Physical findings included pale, scattered ecchymoses and petechiae and mild
hepatosplenomegaly. CBC: RBC – 2.55 x 1012/L; Hb – 73 g/L; HCT 20%; PLT – 23 x 109/L;
and WBC – 162 x 109/L with 82% blasts; peroxidase stain is positive. What is the most
probable diagnosis?
A) Megaloblastic anemia
B) Hemolytic anemia
C) Thrombocytopenia
D) Acute leukemia
E) Chronic leukemia
F) Chronic hemolytic anemia
ANSWER: D
A 58-yr-old man was diagnosed with diabetes at a routine medical examination 3 months
ago. His BMI is 32 despite losing 5 kg by following the dietician's advice. His home
blood glucose readings range from 7 to 11 and his HbAlc is 10%. Choose the most likely
management.
A) Metformin
B) No change in treatment required
C) IV insulin sliding scale
D) Gliclazide
E) Dietary adjustment
ANSWER: A
A 58-yr-old woman presents with fever, sweats and weight loss. On examination she has
an enlarged spleen. Blood tests reveal a lymphocytosis and anaemia. The Philadelphia
chromosome is negative. What is the diagnosis?
A) ALL
B) Multiple myeloma
C) CLL
D) Polycythemia rubra vera
E) CML
ANSWER: C
A 59-yr-old man presents with obstructive jaundice. USG shows no gallstones. The liver
appears normal and the common bile duct measures 12 mm in diameter. His past medical
history includes partial gastrectomy 15 yr ago for peptic ulcer. Choose the single
most likely investigation from the list of options above.
A) Percutaneous transhepatic
B) Cholangiography
C) MRI scan
D) Barium follow through
E) CTscan
ANSWER: B
A 60-year-old alcoholic man is admitted to the emergency department with hematemesis.
His pulse is 110/min, blood pressure is 100/60 mm Hg, and respirations are 19/min. He
has multiple spider angiomata on his back and chest, with bilateral gynecomastia.
Abdominal examination is significant for hepatosplenomegaly, and his abdomen is
distended and tympanic on percussion; a fluid level is easily detectable. His
testicles are small, and a rectal examination produces guaiac-negative stool. His
hematocritis 23%. After placement of a nasogastric tube, 400 mL of bright red blood is
evacuated. After initial fluid resuscitation, which of the following is the most
appropriate next step in management?
A) Barium swallow
B) Esophageal balloon tamponade
C) Esophagogastroscopy
D) Exploratory celiotomy
E) Selective angiography
ANSWER: C
A 60-year-old man with biopsy-proven hepatic cirrhosis is hospitalized because of
massive ascites and pedal edema. There is no evidence of respiratory compromise or
hepatic encephalopathy. Bed rest, sodium and water restriction, and the administration
of spironolactone produce no significant weight change after 5 days. Which of the
following therapeutic measures would be most appropriate at this time?
A) Therapeutic paracentesis
B) None of them
C) Oral acetazolamide, 250 mg/d
D) Placement of a peritoneovenous shunt
E) Intravenous furosemide, 80 mg now
ANSWER: A
A 60-yr-old man is brought to A&E in an unconscious state. His glucose is 35 mmol/l.
His arterial blood gas shows a pH of 7.2 and a PaCO2 of 28 mm Hg. Serum Na is 140, K
is 3.0, Cl is 100 and HCO3" is – 18 mEq/L. Choose the most likely management.
A) Sugary drink
B) Insulin sliding scale, Heparin and 0.45% saline
C) Insulin sliding scale, 0.9% NS and potassium replacement
D) Chest X-ray
E) 50 ml of 50% dextrose IV
ANSWER: C
A 61-year-old patient complains of weight loss, night sweating, general weakness,
fatigue, sensation of fullness in left hypochondrium. CBC reveals: RBC 4.0х 1012/L, Hb
88g/L, Hematocrit 34%, MCV 88 fl, Pl 98 х 109/L, WBC 125,4х 109/L, Segmented
Neutrophils 35%, Band Neutrophils 8%, Metamyelocytes 6%, Myelocytes 5%, Myeloblasts
10%, Monocytes 4%,Eosinophils 8% Basophils 22%, Lymphocytes 2%, ESR mm/h 27mm/h. What
is the most likely diagnosis?
A) Acute leukemia
B) Chronic myeloid leukemia, accelerated phase
C) Chronic lymphocytic leukemia
D) Chronic myeloid leukemia, chronic phase
E) Multiple myeloma
ANSWER: B
A 61-year-old white man has a 2-month history of dysphagia for solid foods. He has
lost 6.7 kg during this time. The patient has chronic heartburn that is relieved by
antacids. He also has hypertension for which he takes atenolol and diltiazem. Which of
the following is the most likely diagnosis
A) Esophageal adenocarcinoma
B) Pill-induced esophagitis
C) Esophageal web
D) diffuse esophageal spasm
E) None of above
ANSWER: A
A 64-year-old patient complains of weight loss, night sweating, general weakness,
fatigue, nose bleeding. Physical examination reveals lymphadenopathy, ecchymosis. CBC
reveals: RBC 3.00 х 1012/L, Hb 98 g/L, Ht 40 %, MCV 98 fl, Reticulocyte Count 1.0 %,
Pl 45 х 109/L, WBC 61.9 х 109/L, Segmented Neutrophils 2 %, Band Neutrophils 1 %,
Monocytes -0% Eosinophils -0 %, Basophils -0%, Lymphocytes 97%, ESR 24mm/h, Smudge
cells ++. What is the most likely diagnosis?
A) Acute leukemia
B) Chronic lymphocytic leukemia III st
C) Chronic lymphocytic leukemia I st
D) Chronic myeloid leukemia
E) Chronic lymphocytic leukemia IV st
ANSWER: E
A 65-yr-old man has had type 2 diabetes for 4 years, for which he was taking
Chlorpropamide. He presents with an acute MI and his laboratory blood glucose is 11
mmol/l. Choose the most likely management.
A) Metformin
B) No change in treatment required
C) IV insulin sliding scale
D) Gliclazide
E) Dietary adjustment
ANSWER: C
A 65-yr-old patient is becoming increasingly confused. She has periods where her
confusion seems to be stable and then seems to rapidly deteriorate in a stepwise
progression. On examination there are extensor plantars but leg reflexes are
diminished. Choose the most likely diagnosis
A) Coronary artery disease
B) Cerebrovascular disease
C) Nephropathy
D) Retinopathy
E) Polyneuropathy
ANSWER: B
A 66-year-old man complains of fever, significant weight loss, bone and joint pain,
and bleeding gums. On examination - paleness, lymphadenopathy, hepato- and
splenomegaly. WBC – 170 x 109/L with 13% lymphocytes, 1% monocytes, 21% basophiles,
29% neutrophils, 10% blasts, 12% promyelocytes, 12% myelocytes, 2% metamyelocytes. ESR
– 22 mm/h. The Philadelphia chromosome positive. Which agent is used for induction or
consolidation therapy?
A) Prednisolone
B) Сytosar
C) Gatifloxacin
D) Vinblastine
E) Alendronic acid
ANSWER: B
A 67-year- male presents with a complaint of fatigue. There is no history of alcohol
abuse or liver disease. Scleral icterus is noted on physical exam. The liver and
spleen are nonpalpable. The patient is noted to have a normocytic, normochromic anemia.
The first step in evaluation of this patient is
A) CT scan of the abdomen
B) Hepatitis profile
C) Liver function tests, including direct versus indirect bilirubin and urine
bilirubin
D) Abdominal ultrasound
E) Percutaneous transhepatic Cholangiography
ANSWER: C
A 67-year-old alcoholic man is admitted to the emergency department with hematemesis.
His pulse is 100/min, blood pressure is 100/60 mm Hg, and respirations are 19/min.
Abdominal examination is significant for hepatosplenomegaly, and his abdomen is
distended and tympanic on percussion; a fluid level is easily detectable. His
testicles are small, and a rectal examination produces guaiac-negative stool. His
hematocritis 24%. After placement of a nasogastric tube, 420 mL of bright red blood is
evacuated. After initial fluid resuscitation, which of the following is the most
appropriate next step in management?
A) Barium swallow
B) Esophageal balloon tamponade
C) Esophagogastroscopy
D) Exploratory celiotomy
E) Transjugular intrahepatic portosystemic shunt
ANSWER: C
A 67-year-old male presents with conjugated hyperbilirubinema, with bilirubin detected
in the urine. Serum bilirubin is 12 mg/dL, AST and ALT are in normal range, and
alkaline phosphatase is 300 U/L (3 times normal). The next step in evaluation is
A) Ultrasound or CT scan
B) Hepatitis profile
C) Reticulocyte count
D) Family history for hemochromatosis
E) Colonoscopy
ANSWER: A
A 67-year-old woman had her first colonoscopy 1 month ago for routine colorectal
cancer screening. A 6-mm tubular adenoma of the sigmoid colon was removed. She has no
family history of colorectal cancer. She asks what can be done to decrease her risk of
developing colorectal cancer. Which of the following is the most appropriate
surveillance for this patient
A) Repeat colonoscopy in 5 year
B) Repeat colonoscopy in 1 years
C) Aspirin, 81 mg daily
D) A high-fiber, low-fat diet
E) None of above
ANSWER: A
A 67-yr-old man is noted to have a glucose level of 37 mmol/l and a Na+ of 163 mmol/l.
He has no prior history of diabetes and has been on IV fluids for a week. His other
medications include IV Cefuroxime, Metronidazole, and Dexamethasone. Choose the most
likely management.
A) Insulin sliding scale, Heparin and 0.9% saline
B) Insulin sliding scale, Heparin and 0.45% saline
C) Sugary drink
D) Chest X-ray
E) 50 ml of 50% dextrose IV
ANSWER: B
A 68-year-old man is evaluated because of nausea, vomiting, and upper abdominal pain
and distention of 2 days duration. He has no fever, chills, or jaundice. On physical
examination, he appears uncomfortable and has orthostatic hypotension. Abdominal
examination discloses distention, tympany on percussion, and rushes on auscultation.
Serum aspartate aminotransferase is 0,34 U/L, serum alanine aminotransferase is 0,61
U/L, and serum total bilirubin is 19,6 mcmol/L. Plain radiographs of the abdomen show
pneumobilia with multiple air-fluid levels in the jejunum. No free air is seen.
Abdominal ultrasonography shows four gallstones measuring 2 to 4 cm. Because of
pneumobilia, the biliary tree cannot be further visualized. Which of the following is
the most appropriate next step in this patient’s management?
A) Exploratory laparotomy for bowel obstruction
B) Cholecystectomy
C) Endoscopic retrograde cholangiopancreatography
D) CT scan of the abdomen
E) Magnetic resonance cholangiopancreatography
ANSWER: A
A 68-year-old patient complains of weight loss, night sweating, sensation of fullness
in left hypochondrium. CBC reveals: RBC 3.9х 1012/L, Hb 128g/L, Ht 41%, MCV 78fl,
Reticulocyte Count 1.0 %, Pl 945 х 109/L, WBC 125,4х 109/L, Segmented Neutrophils 62%,
Band Neutrophils 10%, Metamyelocytes 3%, Myelocytes 5%, Monocytes 4%, Eosinophils 8%,
Basophils 6%, Lymphocytes 2%, ESR 27mm/h. What is the most likely diagnosis?
A) Acute leukemia
B) Erythremia
C) Chronic lymphocytic leukemia
D) Chronic myeloid leukemia
E) Multiple myeloma
ANSWER: D
A 68-yr-old patient presents with bone pain, anaemia and renal failure. Her bone
marrow reveals 32 % of plasma cells. What is the most probable diagnosis?
A) Multiple myeloma
B) Myeloid metaplasia
C) AML
D) CLL
E) ALL
ANSWER: A
A 70-year-old male presents with a complaint of fatigue. There is no history of
alcohol abuse or liver disease; the patient is on no medication. Scleral icterus is
noted on physical exam. There is no evidence for chronic liver disease on physical
exam, and the liver and spleen are nonpalpable. The patient is noted to have a
normocytic, normochromic anemia. The first step in evaluation of this patient is
A) CT scan of the abdomen
B) Hepatitis profile
C) Liver function tests, including direct versus indirect bilirubin and urine
bilirubin
D) Abdominal ultrasound
E) Esophagogastroduodenoscopy
ANSWER: C
A 70-yr-old man presents with bone pain, anaemia. His bone marrow reveals plasma cells
30 %. What is the most probable diagnosis?
A) Myeloid metaplasia
B) AML
C) Multiple myeloma
D) CLL
E) Megaloblastic anaemia
ANSWER: C
A 70-yr-old woman develops severe hip pain while gardening. Hip joint X-ray shows
fracture neck of femur. She gives a history of lower back pain and malaise. She is
tender over the lumbar spine. ESR is 110 mm/hr. What is your diagnosis?
A) Multiple myeloma
B) Waldenstorm's macroglobulinaemia
C) CML
D) CLL
E) Acute leukemia
ANSWER: A
A 70-yr-old woman presents to her GP with weight loss and depression. On examination
she is noted to have buccal pigmentation and pigmented scars. She appears dehydrated.
Her BP is 100/60 mm Hg. Choose the most likely management.
A) Long-term replacement with glucocorticoids and mineralocorticoids
B) Calciferol
C) Carbimazole
D) Thyroxine
E) Octreotide (somatostatin)
ANSWER: A
A 71-year-old male presents with a complaint of fatigue. There is no history of
alcohol abuse or liver disease; the patient is on no medication. Scleral icterus is
noted on physical exam. The patient is noted to have conjugated hyperbilirubinema,
with bilirubin detected in the urine. Serum bilirubin is 12 mg/dL, AST and ALT are in
normal range, and alkaline phosphatase is 300 U/L (3 times normal). The next step in
evaluation is
A) Ultrasound or CT scan
B) Hepatitis profile
C) Reticulocyte count
D) Family history for hemochromatosis
E) Esophagogastroduodenoscopy
ANSWER: A
A 75-year-old man has been deferring colon cancer screening because she is afraid to
undergo colonoscopy. She learned of a new technique called virtual colonoscopy that
she thinks may be more tolerable and asks you about the relative merits of this
procedure. Which of the following statements is true regarding virtual colonoscopy?
A) It detects colorectal cancers and large adenomas quite well, but may miss small
polyps
B) It is more acceptable to patients because it does not require any bowel preparation
C) It is a noninvasive procedure that images the colon using ultrasound
D) Its sensitivity and specificity for detecting colon cancers and polyps is similar
to that of conventional colonoscopy
E) It does not require any instrumentation of the bowel
ANSWER: A
A 76-year-old man, who is in a rehabilitation facility after fracturing a leg,
develops acute diarrhea. He has no history of intestinal disorders. The patient has
regular colonoscopic screenings. His last colonoscopy, done 2 years ago, was normal.
He has not received antibiotics in the past year. His roommate is taking oral
metronidazole for a diarrheal syndrome that he developed while in the rehabilitation
facility. Use of which of the following would most likely have prevented development
of the patients diarrheal syndrome?
A) Good hand-washing technique
B) Prophylactic antibiotics
C) Prophylactic probiotic agents
D) Prophylactic loperamid
E) None of above
ANSWER: A
A man of 59 years old complaints of abdominal discomfort, gum bleeding, large
ecchymosis, weakness, sternal tenderness, fever, skin nodules. Laboratory findings:
the white cell count 540 x 109/L, levels of basophils, eosinophils and platelets are
increased; and a few normoblasts are seen; Er – 3,1 x 1012/L, blast 48%. What is the
most probable diagnosis?
A) Megaloblastic anemia
B) Hemolytic anemia
C) Acute leukemia
D) Thrombocytopenia
E) Chronic leukemia
ANSWER: C
A man, 35years old, complains of weakness, palpitation, tinnitus dizziness. Data of
anamnesis: peptic gastric ulcer, repeated bleeding. Physical examination: skin is pale.
Systolic murmur is heard at the apex, HR 100 per min, BP 100/70 mm of Hg. CBC: ESR –
2.8 x 1012/L, haemoglobin content - 69 gr/l, colour index is 0,7. What is your
treatment?
A) Iron parenteral
B) Folic acid
C) iron supplementation oral
D) Vit B12
E) Antibiotics
ANSWER: C
A middle-aged woman complains of irritability and weight loss. She says she has
palpitations. On physical examination you find mild tachycardia and goiter. There are
no eye changes. A thyroid scan determines a single hot nodule. Choose the most likely
diagnosis.
A) Toxic adenoma
B) Adrenal hyperplasia
C) Hyperthyroidism
D) Hypothyroidism
E) Follicular carcinoma
ANSWER: A
A nursing student has just completed her hepatitis B vaccine series. On reviewing her
laboratory studies (assuming she has no prior exposure to hepatitis B), you expect
A) Positive test for hepatitis B surface antigen
B) Antibody against hepatitis B surface antigen (anti-HBS) alone
C) Antibody against hepatitis core antigen (anti-HBC)
D) Antibody against both surface and core antigen
E) Antibody against hepatitis E antigen
ANSWER: B
A paracentesis is performed on 43-year-old patient with long-standing alcohol abuse.
On physical exam, there are spider angiomas and palmar erythema. Abdominal collateral
vessels are seen around the umbilicus. The serum albumin minus ascitic fluid albumin
equals 1.4 g/dL. The most likely diagnosis is
A) Portal hypertension
B) Pancreatitis
C) Tuberculous peritonitis
D) Hepatoma
E) No ascitis
ANSWER: A
A patient presents to a physician with severe jaundice. Physical examination reveals a
nodular, enlarged liver. In addition to the generalized nodularity of the liver, the
physician can feel one nodule that is much larger than the others. CT of the abdomen
confirms multinodular cirrhosis and demonstrates a 7-cm mass near the lower border of
the liver. CT-guided biopsy of this mass shows a malignant tumor derived from hepatic
parenchymal cells. Which of the following risk factors is most strongly associated
with the development of this tumor?
A) Anatoxin exposure
B) Hemochromatosis
C) Hepatitis В virus infection
D) Opistharchis infection
E) Thorotrast exposure
ANSWER: C
A patient who has long-standing diabetes mellitus and severe, burning pain in the feet
and hands as a result of peripheral neuropathy asks the physician why an
antidepressant has been prescribed. What is the physician’s best response?
A) “Many people experiencing chronic pain become depressed.”
B) “The antidepressants may counteract the chemicals causing your pain.”
C) “You are less likely to become addicted from using antidepressants than you are
from using other types of pain killers.”
D) “The antidepressants also have strong anti-inflammatory properties and can reduce
the pain you have from inflammation.”
E) None of above
ANSWER: B
A patient who is 2 days postoperative from a bowel resection tells her physician that
she is having a hard time “catching her breath,” feels nauseated, and has chest pains
when she inhales. The physician suspects that she is having a pulmonary embolism. What
intervention should the physician perform before notifying the physician?
A) Increase the IV flow rate
B) Apply oxygen by mask or nasal cannula at 5 l/min
C) Assess the chest and axillary area for the presence of petechiae
D) Place the patient in shock position, with her head and neck flat and her legs
elevated
E) Non of above
ANSWER: B
A woman 22 year-old, complains of general weakness, shortness of breath, brittle nails,
hair loss. She has menorrhagia. Physical examination reveals paleness of skin,
systolic murmur on auscultation. What changes do you expect in her blood analysis?
A) Increased level of serum iron
B) High colour index
C) Decreased level of serum iron
D) Increased level of free bilirubin
E) Decreased amount of thrombocytes
ANSWER: C
A young man presents with a neck lump. It is painless and had been bothering him for
the past 4 months. He has no other symptoms or signs. On palpation you find the lump
to be single discrete not particularly hard and confined to the thyroid gland itself.
His cervical lymph nodes are enlarged. Choose the most likely diagnosis.
A) Toxic adenoma
B) Adrenal hyperplasia
C) Hyperthyroidism
D) Hypothyroidism
E) Papillary carcinoma
ANSWER: E
A young woman complains of wheeze, dyspnoea and cough. She cannot sleep at night
because of a chronic cough. She and her mother love animals and together they have 14
cats. Her PEFR is normal but her CXR suggests hyperinflation. What is the previous
diagnosis?
A) Bronchial asthma
B) Bronchogenic carcinoma
C) Emphysema
D) Respiratory failure
E) Bronchitis
ANSWER: A
After emotional exertion patient of 24 y.o. developed dyspnea with prolonged
expiration, distant wheezes, frequent night symptoms. Such changes limit his physical
activity. PEV and FEV1 < 60 %, daily variability -30 %. What diagnosis is possible?
A) Severe persistent bronchial asthma
B) Mild persistent bronchial asthma
C) Persistent bronchial asthma of moderate severity
D) Intermittent bronchial asthma
E) Bronchospastic syndrome of allergic origin
ANSWER: A
An 14-yr-old boy presents with painful bones, jaundice and anaemia. He is noted to
have splenomegaly. His blood film reveals target cells. What is your diagnosis?
A) Hemolytic anemia.
B) Sickle cell anaemia
C) Iron-deficiency anemia.
D) B12-deficiency anemia
E) Aplastic anemia.
ANSWER: B
An 18-yr-old girl complains of her appearance. She is much too fat, she says. She also
complains of missed periods and hairiness. On physical examination you find her to be
10 kg overweight. Choose the most likely investigation
A) Abdominal ultrasonography
B) Serum cortisol
C) Dexamethasone suppression test
D) Water deprivation test
E) T3, T4 and TSH
ANSWER: C
An 69 year old asymptomatic woman was detected to have a monoclonal spike on serum
electrophoresis (IgG levels 1.5 g/dl). Bone marrow revealed plasma cells of 30%. What
is the possible diagnosis?
A) Multiple myeloma
B) Indolent myeloma
C) Monoclonal gammopathy of unknown significance
D) Waldenstorm's macroglobulinemia
E) Amyloidosis
ANSWER: A
Drug abuser, a 41-yr-old man, presents with fever, cough and breathlessness. This was
preceded by viral influenza. Chest radiograph shows multiple abscesses. What is the
most possible etiology of disease?
A) Staphylococcus aureus
B) Cryptococcus
C) Streptococcus pneumoniae
D) Legionella pneumonia
E) Mycobacterium avium
ANSWER: A
Female B., 44 years old, complains on cough with mucous sputum, increase of
temperature to 39 °С, weakness, dyspnea, sweating. Breathing rate - 26/min, skin is
moist. Below left scapula there is shortening of percussion sound. Breathing during
auscultation is weakened, moist rales. Blood test: L - 11х109/l, ESR - 29 mm/h. Your
previous diagnosis?
A) Left-side lower lobe pneumonia
B) Gangrene of lungs
C) Left-side exudative pleurisy
D) Cancer of left side lower lobe
E) Pulmonary abscess
ANSWER: A
Female patient K., 46 years old, has flue with fever. After a while she
noticesthoracic pain, cough with yellow-green sputum (amount-150 ml a day), sometimes
with some blood. Objectively: breathing rate - 36/min. In lungs from the right side
lower scapula there is dull sound during percussion, hard breathing, and moist rales.
Blood test: L - 18,6х109/l, ESR -64 mm/h. Analysis of sputum: L -80-100 , Er - 40-50,
elastic fibers, cocci. X-ray: lung roots are enlarged, from the right side lower lobe
is heterogeneously infiltrated with two lighter areas. What is the most possible
previous diagnosis?
A) Right-side pneumonia with abscesses
B) Peripheral cancer
C) Infiltrative tuberculosis in the phase of disintegration
D) Exudative pleurisy
E) Infarction-pneumonia
ANSWER: A
Female, 34 years old, has an increase of body temperature to 38 °С, cough with
purulent sputum, weakness, dyspnea, pain in a thorax during breathing. During
percussion there is shortening of sound in the lower part of left lung, during
auscultation – moist rales. What method of investigation is the decisive one to
confirm diagnosis?
A) X-ray examination
B) Bacteriological analysis of sputum
C) Spirometry
D) Pneumotachometry
E) Bronchography
ANSWER: A
Girl, 18 y.o., pets seller, complaints mainly during working time on the attacks of
dry cough, running nose. She often suffers from acute viral respiratory infections.
Her mother is ill with bronchial asthma. Objectively: breathing rate - 18/min. Heart
rate - 80/min, BP - 110/70. In lungs vesicular breathing, dry wheezes are heard in
distance. Tones of heart are weaker than normally. Test with berotec showed
reversibility of bronchial obstruction. What tactic will be the best for the patient?
A) To change job
B) To use intal
C) To use monteleucast
D) To use berotec constantly
E) To use antihystaminic preparations
ANSWER: A
Girl, 23 y.o., for 2 years is ill with bronchial asthma. Recently attacks of dyspnea
became more frequent and started to arise 4-5 times a week, night attacks - 2-3 times
a month. She used salbutamol to remove the symptoms. Test with the antigen of home
dust is positive. Objectively: condition is satisfactory. Breathing rate - 20/min.
Heart rate - 76/min, BP -120/80. In lungs breathing is vesicular. Tones of heart are a
little weak, rhythm is normal. What mechanism is desicive in development of bronchial
obstruction in this case?
A) Hyperreactivity of bronchi
B) Тrachео-bronchial dyskinesia
C) Violation of metabolism of arachidonic acid
D) Adrenergic disorders
E) Activity of the parasympathetic nervous system is increased
ANSWER: A
In the biochemical profile of patient B., 29 yr-old, it is elevated serum T4 and low
radioactive iodine uptake. Choose the most likely diagnosis.
A) Non-toxic goitre
B) Hashimoto's thyroiditis
C) Subacute thyroiditis
D) Hypothyroidism
E) Graves' disease
ANSWER: C
In the biochemical profile of patient J., 38 yr-old, who has a neck mass, it is normal
T3 and T4. Choose the most likely diagnosis.
A) Non-toxic goitre
B) Hashimoto's thyroiditis
C) Subacute thyroiditis
D) Hypothyroidism
E) Graves' disease
ANSWER: A
In the biochemical profile of patient L., 56 yr-old., it is normal TSH, free T4 and T3.
Decreased serum total T4. Choose the most likely diagnosis.
A) Non-toxic goitre
B) Hashimoto's thyroiditis
C) Thyroid binding globulin deficiency
D) Hypothyroidism
E) Graves' disease
ANSWER: C
In the biochemical profile of patient S., 43 yr-old, it is elevated serum T4 and
increased radioactive iodine uptake. Choose the most likely diagnosis.
A) Non-toxic goitre
B) Hashimoto's thyroiditis
C) Subacute thyroiditis
D) Hypothyroidism
E) Graves' disease
ANSWER: E
Male patient F., 48 years old, during a week stayed at home with diagnosis of
respiratory viral infection. Doctor noticed complaints on cough with small amount of
mucus-purulent sputum, weakness. Objectively: condition is relatively satisfactory. T
- 37,2 °С. Breathing rate - 18/min., pulse - 80/min., BP - 110/70. In lungs there is
vesicular breathing, with a hard tint, single dry wheezes. Tones of heart are a little
dull, rhythm is correct. What is the treatment tactic?
A) To prescribe antibacterial therapy
B) To stay at home for some more days
C) To go to work
D) To send patient to pulmonologist
E) To hospitalize patient to the pulmonological department
ANSWER: A
Male patient G., 56 years old, complaints on permanent pain in a thorax which disturbs
for last 2 months. Pain is not connected with breathing. There is also cough with
particles of blood in sputum. Weakness, fatigue are present. On the chest X-ray in the
lower lobe of right lung there is spherical shadow, size 4x6 cm, related to the lung
root. What is the most possible diagnosis?
A) Perypheral lung cancer
B) Tuberculoma
C) Metastasis
D) Pulmonary abscess
E) Pneumonia
ANSWER: A
Man, 32 y.o., complaints on attack of expiratory dyspnea, which lasts for 48 hours,
cough with small amount of sputum. He is ill with bronchial asthma for 5 years, was
treated with glucocorticosteroids, used inhalers. Objectively: condition is severe,
patient sits. Diffuse cyanosis, pulse -110/min, BP - 110/70. Tones of heart are weak,
II tone is louder above the pulmonary artery. During percussion in lungs there is
“bang-box” sound, large amount of dry wheezes. In blood there is eosinophylia - 18 %.
What medicines are drugs of choice for this patient?
A) Corticosteroids
B) ?2-agonists
C) Theophyllin
D) Cholynolytics
E) Antihystamines
ANSWER: A
Man, 39 y.o., 8 last years is ill with bronchial asthma. Rapidly during physical work
he felt worsening of breathing, cough, distance wheezes appeared and dyspnea began to
increase. Medicine of what pharmacological group is it better to recommend for the
patient to remove such attacks of dyspnea?
A) Agonists of ?2-adrenoreceptors
B) Methylxantines
C) ?2-adrenoblockers
D) Inhalated glucocorticoids
E) Oral glucocorticoids
ANSWER: A
Man, 43 y.o., complaints on dyspnea during physical activity. Objectively: temperature
36,4 °С, breathing rate - 20/min, pulse - 78/min, BP-125/80. Emphysematous form of
thorax. In lungs – weak vesicular breathing. What test should be passed by patient at
home to decide question about efficiency of prescribed broncholytics?
A) Peakflowmetry
B) Spirography
C) ECG-control of overload of right chambers of heart
D) Bronchoscopy
E) Analysis of sputum (amount and microscopy)
ANSWER: A
Man, 46 y.o., suffers for the last 10 years from bronchial asthma. Rapidly during
physical work he felt worsening of breathing, cough, distance wheezes appeared and
dyspnea began to increase. Medicine of what pharmacological group is it better to
recommend for the patient to remove such attacks of dyspnea?
A) Agonists of ?2-adrenoreceptors
B) Atropine
C) Intal
D) Epinephrine
E) Monteleucast
ANSWER: A
On a routine blood examination a 43-yr-old woman is found to have very high serum
calcium level. She has complained recently of bouts of abdominal pain and recurrent
UTI. On physical examination you find an enlarged thyroid gland. Choose the most
likely diagnosis.
A) Toxic adenoma
B) Adrenal hyperplasia
C) Hyperthyroidism
D) Hypothyroidism
E) Parathyroid carcinoma
ANSWER: E
Over a 2-month period, a 50-year-old woman with a history of polycythemia vera
develops abdominal pain and gross ascites. Physical examination demonstrates smooth
hepatomegaly and mild jaundice. Pressure applied over the liver fails to distend the
jugular veins. The abdominal wall is grossly edematous and shows a tortuous venous
pattern. Edema of the legs is prominent. Which of the following is the most likely
diagnosis?
A) Hepatocellular carcinoma
B) Primary sclerosing cholangitis
C) Steatosis
D) Budd-Chiari syndrome
E) Hepatic cirrhosis
ANSWER: D
Patient A., 35 y.o., noticed infrequent (rarer than 1 time a week) attacks of dyspnea,
which are easily removed with inhalations of ?2-agonists of short action. During
attack in lungs are heard dry wheezes, between attacks FEV1 is more than 80 % from
normal. What is the diagnosis?
A) Intermittent bronchial asthma
B) Persistent bronchial asthma of moderate severity
C) Mild persistent bronchial asthma
D) Severe persistent bronchial asthma
E) Given information is not enough for determination of severity of bronchial asthma
ANSWER: A
Patient A., 45 years old, suffers from chronic cholecystits during last 5 years. She
was admitted to the emergency department with acute pain in right hypochondriac area
and high temperature. Laboratory findings reveal leucocytosis, high ESR. Put
preliminary diagnosis.
A) Chronic cholecystitis, acute phase.
B) Chronic cholecystitis, subacute phase.
C) Chronic cholecystitis, phase of remission.
D) Dyskinezia of bile ducts.
E) Rotor’s syndrome.
ANSWER: A
Patient B., 25 years old engineer, appeared during a fire in the area of high
concentration of CO (an industrial accident). In hospital delivered in the
irresponsible state. What laboratory tests are the early criteria of estimation of
severity of the state?
A) Estimation of blood viscosity
B) Anemia
C) Leucocytosis
D) Carboxihemoglobinemia
E) Methemoglobinemia
ANSWER: D
Patient complaints on attacks of dyspnea, which arise 1-2 times a week, night symptoms
- 2 times a month and even more frequently. For a patient night sleep is violated as a
result of attacks of dyspnea. FEV1 > 80 % from normal. What diagnosis would you
suspect?
A) Mild persistent BA
B) Severe persistent BA
C) Intermittent BA
D) Moderate persistent BA
E) Status asthmaticus
ANSWER: A
Patient D., 56 yr-old, complains on muscle weakness, bradycardia and hypotension. ECG
shows tall peaked T waves. Choose the most likely diagnosis.
A) Hypokalaemia
B) Hyponatraemia
C) Hypervitaminosis A
D) Hyperkalaemia
E) Hypoglycaemia
ANSWER: D
Patient F., 46 years old, was hospitalized urgently with acute attack of dyspnea. Last
5 years he has been working on poultry farm. During examination bronchial asthma was
diagnosed. What additional diagnostic methods are necessary to confirm the
professional genesis of asthma?
A) sanitary-hygienic characteristics of the work conditions
B) echocardioscopy
C) allergic and immunological tests
D) investigation of the function of external breath
E) roentgenography of pulmonary system
ANSWER: C
Patient G., 36 years old, works on a poultry factory. She was emergently hospitalized
with acute attack of dyspnea. During observation bronchial asthma was diagnosed. What
additional methods of research must be conducted above all things to confirm the
professional genesis of bronchial asthma?
A) roentgenologic research of breathing organs
B) professional route of patient
C) sanitary-hygienic characteristic of work conditions
D) research of external breathing function
E) allergic and immunological tests
ANSWER: E
Patient G., 38 yr-old, complains on muscle weakness and ectopic beats. ECG shows
flattened or inverted T waves. Choose the most likely diagnosis.
A) Hypokalaemia
B) Hyponatraemia
C) Hypervitaminosis A
D) Hyperkalaemia
E) Hypoglycaemia
ANSWER: A
Patient G., 47 y.o., with long history of bronchial asthma, has developed more
frequent attacks of dyspnea. Inhalations of astmopent and berotec are not effective.
From prescription of what medicine is it better to begin the intensive treatment?
A) Glucocorticoids
B) Oxygen therapy
C) Bronchodylators
D) Infusion therapy
E) Heart glycosides
ANSWER: A
Patient has severe attack of bronchial asthma which lasts more than 1 hour. Usage of
beta-agonists in inhalation, euphylline intravenously and cholynolytics was not
effective. What medicines are necessary for emergency therapy?
A) Glucocorticosteroids intravenously
B) Beta-agonists intravenously
C) Inhaled glucocorticosteroids
D) Antihystaminic
E) Nonsteroid anti-inflammatory medicines
ANSWER: A
Patient J., 36 y.o., complains for fever (39 C), pain in the left part of the chest.
Pleuropneumonia was diagnosed in the patient. What onset is typical for
pleuropneumonia?
A) Acute
B) Latent
C) Fulminant
D) Gradual
E) Non of the above
ANSWER: A
Patient J., 38 yr-old, complains on sweating, palpitations, tremors, drowsiness and
fatigue. Choose the most likely diagnosis.
A) Hypokalaemia
B) Hyponatraemia
C) Hypervitaminosis A
D) Hyperkalaemia
E) Hypoglycaemia
ANSWER: E
Patient J., 45 y.o., complaints on dyspnea during insignificant physical exertion,
cough with minimal amount of “glass-like” sputum, attacks of dyspnea up to 3 times a
day, more often at night, sweating. She is ill for more than 5 years. Has an allergy
on dust, cockroaches. For treatment uses bekotid for near the year. Diagnosis?
A) Bronchial asthma
B) Eosinophylic pulmonary infiltrate
C) COPD
D) Bronchiectasis with bronchial spasm
E) Pulmonary vasculitis (syndrome of Charg - Stross)
ANSWER: A
Patient L, 41 years is suffering from diabetes type 1 for 16 years, receiving insulin
therapy. He complaints of swelling and shortness of breath. On examination: glycemia -
7.1 mmol / l, cholesterol - 6.2 mmol / l, Creatinine - 0.21 mmol / l. What will you
recommend the patient first of all?
A) Increase the dose of insulin
B) ACE inhibitors
C) Statins
D) Lipoic acid
E) Enterosorbents
ANSWER: E
Patient M., 30 years old, during last 3 years works as a nurse in manipulations
cabinet. Last year during the contact with penicilline she started to complain on
discomfort in throat, sneezing, attack of cough and dyspnea which disappear after
inhalation of salbutamol. During last months attacks of dyspnea became more severe and
occurred only at contact with penicilline. During the life she had not any diseases
including allergic. She hadn’t received antibiotics. Can we consider the bronchial
asthma is professional in this patient?
A) no, we can’t
B) yes, we can if we have conclusion about attacks of bronchial asthma
C) yes, we can if we have conclusion about appearance of bronchial asthma attacks
after contact with penicilline
D) yes, we can
E) yes, we can, if allergic and immunological tests are positive
ANSWER: E
Patient S., 43 yr-old, has perioral paraesthesia, carpopedal spasm and generalised
seizures. Choose the most likely diagnosis.
A) Hypokalaemia
B) Hyponatraemia
C) Hypocalcaemia
D) Hyperkalaemia
E) Hypoglycaemia
ANSWER: C
Patient U., 44 yr-old, complains on severe abdominal pain, nausea, vomiting,
constipation, polyuria and polydipsia. Choose the most likely diagnosis.
A) Hypercalcaemia
B) Hyponatraemia
C) Hypervitaminosis A
D) Hyperkalaemia
E) Hypoglycaemia
ANSWER: A
Patient Y.,49 y.o., complaints on dyspnea, cough. Sputum is absent. Used many puffs of
salbutamol, intal, but without any efficacy. Objectively: sits, leaning on a table.
Total cyanosis of the body. Peripheral edema is absent. Breathing is superficial,
dyspnea, during auscultation breathing cannot be heard in some areas of lungs; wheezes
are diffuse, expiration is considerably prolonged. Tones of heart are weak,
tachycardia. Pulse - 112/min, BP - 110/70. Liver is near the edge of costal arch. What
is the previous diagnosis?
A) Status asthmaticus
B) Bronchial asthma of moderate severity
C) COPD
D) Aspiration of foreign body
E) Heart asthma
ANSWER: A
Patient, 28 y.o., has running nose, attacks of dyspnea at night once a week. Felt ill
after viral respiratory infection which was treated with acetilsalicylic acid.
Eosynophylia was found in blood and sputum. What disease may be suspected?
A) Aspirin bronchial asthma
B) Eosinophylic infiltrate of lungs
C) Bronchial asthma of physical exertion
D) Allergic rhinitis
E) Bronchial asthma, exogenous form
ANSWER: A
Patient, 30 y.o., after a viral infection has daily symptoms of dyspnea, which causes
lowering of activity and bad sleep; night symptoms are more frequent than once a week.
PEV and FEV1 - 60-80 %, daily variability > 30 %. There is a necessity of daily usage
of ?2-agonists of short action. What is the diagnosis?
A) Persistent bronchial asthma of moderate severity
B) Mild persistent bronchial asthma
C) Intermittent bronchial asthma
D) Severe persistent bronchial asthma
E) Status asthmaticus
ANSWER: A
Patient, 42 years old, complaints on attacks of dyspnea, every time uses 1-2 doses of
salbutamol. These attacks are accompanied with cough and minimal amount of viscid
glassy sputum. He is ill for 8 years. Objectively: temperature - 36,7 C; breathing
rate – 21/min.; pulse-90/min.; BP - 130/80; FEV1 - 77 %. In lungs – solitary dry
wheezes. Blood test: eosinophyles - 6 %. What medicines are “basic” in the treatment
of this patient?
A) Antiinflammatory
B) Cholynolytics
C) Mucolytics
D) Antihystaminic
E) ?2-agonists
ANSWER: A
Patient, 44 y.o., complaints on attack of dyspnea, which arises suddenly at night.
Connects this attack with overcooling. He is ill for more than 10 years. Thorax of
emphysematous form. During percussion in lungs – “bang-box” sound. During
auscultation there is plenty of dry wheezes. In blood: moderate leucocytosis,
eosinophylia - 10 %. On the chest X-ray film – increased pneumatization of pulmonary
tissue. What diagnosis is the most possible?
A) Bronchial asthma, exacerbation phase
B) Bronchiectasis, exacerbation phase
C) COPD, exacerbation phase
D) Chronic bronchitis
E) Eosinophylic pulmonary vasculitis
ANSWER: A
PatientI., a 50 years old man, is evaluated in the emergency department because of
fever, nonproductive cough and 2-day history of myalgia and headache. He has also had
nausea and diarrhea. He is a heavy smoker. On physical examination, he is slightly
disoriented. Temperature is 38.9 C, pulse rate is 110/min, respiration rate is 32/min.
Chest radiograph shows fluffy infiltrates to the right upper and lower lobes. Results
of laboratory testing show serum sodium of 128 meq/L, blood urea nitrogen of 42 mg/dL,
serum creatinine of 2.2 mg/dL, and serum creatine kinase of 250 U/L. Which one of the
following is best next step in the management of this patient’s pneumonia?
A) Initiate empiric antibiotic therapy for Legionella
B) Order direct fluorescent antibody testing of the sputum for Legionella
C) Order serologic testing for Legionella
D) Send a urine specimen for measurement of Legionella antigen
E) All of the above
ANSWER: A
Physical examination of patient revealed lymphadenopathy, splenomegaly. CBC: RBCs -
3,6 x 1012/l, Hb- 87 g/l, Pl – 45 x 109/l, WBCs – 13 x 109/l, blasts - 87%, band
neutrophils - 1%, segmented neutrophils - 7%, lymphocytes - 5%, ESR - 55 mm/h. What is
the most likely diagnosis?
A) Acute leukemia
B) Erythremia
C) Chronic lymphocytic leukemia
D) Chronic myeloid leukemia
E) Multiple myeloma
ANSWER: A
The 30-year-old woman whose father has type 1 diabetes mellitus asks the physician
what her chances are of developing diabetes because of her father's disease. The risk
for becoming diabetic is:
A) 20-50%
B) 0%
C) 50%
D) 50-80%
E) None of above
ANSWER: A
The 45-year-old diabetic patient has proliferative retinopathy, nephropathy, and
peripheral neuropathy. What should the physician teach this patient about exercise?
A) “The type of exercise that would most efficiently help you to lose weight, decrease
insulin requirements, and maintain cardiovascular health would be jogging for 20
minutes 4 to 7 days each week.”
B) “Considering the complications you already have, vigorous exercise for an hour each
day is needed to prevent progression of disease.”
C) “Considering the complications you already have, you should avoid engaging in any
form of exercise.”
D) “Swimming or water aerobics 30 minutes each day would be the safest exercise
routine for you.”
E) None of above
ANSWER: D
The patient diabetic patient asks the physician why it is necessary to maintain blood
glucose levels no lower than about 74 mg/dL. What is the physician’s best response?
A) “Glucose is the only fuel form used by body cells to produce energy needed for
physiologic activity.”
B) “The central nervous system, which cannot store glucose, requires a continuous
supply of glucose for fuel.”
C) “Without a minimum level of glucose circulating in the blood, erythrocytes cannot
produce ATP.”
D) “The presence of glucose in the blood counteracts the formation of lactic acid and
prevents acidosis.”
E) None of above
ANSWER: B
The patient getting ready to engage in a 30-minute, moderate-intensity exercise
program performs a self-assessment. Which data indicate that exercise should be
avoided at this time?
A) Ketone bodies in the urine
B) Weight 1 pound higher than the week before
C) Blood sugar level of 155 mg/dL
D) Pulse rate of 66 beats/min
E) None of above
ANSWER: A
The patient has clinical signs which may suggest acromegaly. Which of the following is
the most specific test to diagnose acromegaly?
A) Magnetic resonance imaging of the pituitary gland
B) Measurement of growth hormone (GH) levels during an oral glucose tolerance test
C) Measurement of insulin-like growth factor I serum levels
D) Measurement of random GH blood levels
E) An octreotide scan
ANSWER: C
The patient newly diagnosed with type 2 diabetes tells the physician that since he has
increased his intake of fiber, he is having loose stools, flatulence, and abdominal
cramping. What is the physician’s best response?
A) “Decrease your intake of water and other fluids.”
B) “Decrease your intake of fiber now and gradually add high-fiber foods back into
your diet.”
C) “You must have allergies to high-fiber foods and will need to avoid them in the
future.”
D) “Taking an antacid 1 hour before meals or 2 hours after meals should reduce the
intensity of your bowel problems.”
E) None of above
ANSWER: B
The patient on an intensified insulin regimen consistently has a fasting blood glucose
between 270 and 280 mg/dL, a postprandial blood glucose level below 200 mg/dL, ketones
in urine ++and a hemoglobin A1c level of 8.5%. What is the physician’s interpretation
of these findings?
A) The patient has developing night hypoglycemia.
B) The patient is at increased risk for developing hyperglycemia.
C) The patient is demonstrating signs of insulin resistance.
D) The patient is demonstrating good control of blood glucose.
E) None of above
ANSWER: A
The patient tells the physician that he enjoys having a glass of wine on Saturdays
when dining out with friends. He asks if having type 1 diabetes will prohibit him from
this activity. What is the physician’s best response?
A) “Insulin activity is dramatically reduced under the influence of alcohol and
drinking even one glass of wine will increase your insulin requirements.”
B) “Diabetics have decreased kidney function and should avoid ingesting alcohol in all
forms at all times.”
C) “You shouldn't drink any alcohol because it is likely to increase your sense of
hunger and make you overeat.”
D) “One glass of wine can be ingested with a meal and is counted as two fat exchanges.”
E) None of above
ANSWER: D
The patient with diabetes is visually impaired and wants to know if syringes can be
prefilled and stored for use later. What is the physician’s best response?
A) “Yes, prefilled syringes can be stored for up to 3 weeks in the refrigerator in a
vertical position with the needle pointing up.”
B) “Yes, prefilled syringes can be stored for up to 3 weeks in the refrigerator,
placed in a horizontal position.”
C) “Insulin reacts with plastic, so prefilled syringes must be made of glass.”
D) “No, insulin cannot be stored for any length of time outside of the container.”
E) None of above
ANSWER: A
The patient with hyperthyroid symptoms is having hormone studies done to confirm the
diagnosis. Which set of values indicates Graves’ disease hyperthyroidism?
A) Elevated T3, elevated T4, high TSH levels
B) Elevated T3, normal T4, low TSH levels
C) Elevated T3, low T4, high TSH levels
D) Low T3, normal T4, high TSH levels
E) None of above
ANSWER: B
The patient with hypothyroidism as a result of Hashimoto’s thyroiditis asks the
physician how long she will have to take thyroid medication. What is the physician’s
best response?
A) “You will need to take the thyroid medication until the goiter is completely gone.”
B) None of above
C) “The thyroiditis will be cured with antibiotics, and then you will no longer need
the thyroid medication.”
D) “You will need thyroid replacement hormone therapy for the rest of your life
because the thyroid gland function will not return.”
E) “When your thyroid function studies indicate a normal blood level of thyroid
hormones, you will be able to discontinue the medication.”
ANSWER: D
The patient, 51 years old, suffers from diabetes 7 years, gets glibenclamide 15 mg a
day. Complains of shortness of breath and pitting edema. On examination of blood
pressure - 180/110 mm Hg. Laboratory: glycemia - 6.2 mmol / L, creatinine - 0.15 mmol
/ L, glomerular filtration - 62 ml / min. What medication will you recommend to the
patient?
A) Gliclazide
B) Gliquidone
C) Glimepiride
D) Pioglitazone
E) Repaglinide
ANSWER: B
When taking the blood pressure of a patient after a parathyroidectomy, the physician
notes that the patient's hand has gone into flexion contractions. What is the
physician’s interpretation of this observation?
A) Hypokalemia
B) Hyperkalemia
C) Hyponatremia
D) Hypocalcemia
E) None of above
ANSWER: D
Which the following type of anemia can be normocytic normochromic anemia with
reticulocytosis?
A) Haemolytic anaemia
B) Vitamin B12 deficiency anaemia
C) Acute posthemorrhagic anaemia
D) Chronic iron deficiency anaemia
E) Aplastic anaemia
ANSWER: A
Which the following type of anemia can be with Cabot rings?
A) Vitamin B12 deficiency anaemia
B) Acute posthemorrhagic anaemiaChronic iron deficiency anaemia
C) Aplastic anaemia
D) Haemolytic anaemia
ANSWER: D
33-year-old patient, presents with complaints of tightness of fingers. There is also
history of dysphagia. Choose the most likely results of the investigations from the
below list of options.
A) Anticentromere antibody
B) Anti dsDNA antibodies
C) ANCA
D) Antimitochondrial antibody
E) Smooth muscle antibody
ANSWER: A
37-year-old patient, presents with induration and atrophy of the fingertips, dysphagia,
Raynaud's phenomenon. Choose the most likely results of the investigations from the
below list of options.
A) Anticentromere antibody
B) Anti dsDNA antibodies
C) ANCA
D) Antimitochondrial antibody
E) Smooth muscle antibody
ANSWER: A
52 year old male with chronic joint pain. There is asymmetric soft tissue swelling,
predominantly in the DIP of the right first digit, with calcifications. Further
questioning revealed a history of repeated attacks of acute joint pain. There is bony
erosion at multiple DIP's of the right hand that is periarticular, well corticated and
has a thin overhanging edge. There is no associated osteopenia. Choose the most likely
diagnosis from the below list of options.
A) Ankylosing spondylitis
B) Erythema nodosum
C) Osteoarthritis
D) Chronic gouty arthritis
E) Psoriatic arthritis
ANSWER: D
55 y.o. male with swelling and pain of his fingers. Patient also with similar
complaints regarding his great toe of the left foot. Soft tissue swelling with subtle
soft tissue nodules primarily of the 2nd and 3rd digits. Erosions with overhanging
edges at the distal aspects of the proximal phalanges of the 2nd and 3rd digits and
metatarsophalangeal joints. Choose the most likely diagnosis from the below list of
options.
A) Ankylosing spondylitis
B) Chronic Gouty arthritis
C) Rheumatoid arthritis
D) Osteoarthritis
E) Psoriatic arthritis
ANSWER: B
55 years old, came to a musculoskeletal specialist seeking advice for a 3-year history
of progressively worsening pain in both knees. Her knees were stiff for about 20
minutes when she arose in the morning and for a few minutes after getting up from a
chair during the day. She had difficulty walking > 30 minutes because of pain, and her
symptoms were exacerbated by kneeling, squatting, or descending stairs. Although
sitting, resting, and reclining relieved her pain, she became stiff if she stayed in
one position for too long. Her symptoms were worse on humid or cold days, and she
occasionally felt as if one of her knees would give out. Choose the most likely
diagnosis from the below list of options.
A) Ankylosing spondylitis
B) Erythema nodosum
C) Osteoarthritis
D) Gout
E) Psoriatic arthritis
ANSWER: C
55 years old, came to a musculoskeletal specialist seeking advice for a 3-year history
of progressively worsening pain in both knees. Her knees were stiff for about 20
minutes when she arose in the morning and for a few minutes after getting up from a
chair during the day. She had difficulty walking > 30 minutes because of pain, and her
symptoms were exacerbated by kneeling, squatting, or descending stairs. Although
sitting, resting, and reclining relieved her pain, she became stiff if she stayed in
one position for too long. Her symptoms were worse on humid or cold days, and she
occasionally felt as if one of her knees would give out. Which of the following
changes will reveal X-ray examination?
A) Erosions
B) Subluxations
C) Osteophytes
D) Osteoporosis
E) Ankylosis
ANSWER: C
57 yo male with swelling and pain of his fingers. Patient also with similar complaints
regarding his great toe of the left foot. Soft tissue swelling with subtle soft tissue
nodules primarily of the 2nd and 3rd digits. Erosions with overhanging edges at the
distal aspects of the proximal phalanges of the 2nd and 3rd digits and
metatarsophalangeal joints. Which of the following medications is indicated for the
treatment of chronic arthritis?
A) Сolchicine
B) Сorticosteroids
C) Indomethacin
D) Allopurinol
E) Ibuprofen
ANSWER: D
59 yo male with swelling and pain of his fingers. Patient also with similar complaints
regarding his great toe of the right foot. Soft tissue swelling with subtle soft
tissue nodules primarily of the 2nd and 3rd digits. Erosions with overhanging edges at
the distal aspects of the proximal phalanges of the 2nd and 3rd digits and
metatarsophalangeal joints. Which of the following medications is indicated for the
treatment of exacerbation of arthritis?
A) Allopurinol
B) Corticosteroids
C) Colchicine
D) Indomethacin
E) Ibuprofen
ANSWER: C
67-year-old man with chronic obstructive pulmonary disease is evaluated because of
chronic dyspnea, minimally productive cough, and limited exercise tolerance. He thinks
his dyspnea on exertion has worsened. He stopped smoking cigarettes 8 years ago and is
currently using an ipratropium inhaler four times per day and salmeterol discus twice
per day. His body mass index, which 6 months ago was 21, is now 19. On physical
examination, he is afebrile, his pulse rate is 94/min and regular, and respiration
rate is 20/min. His breathing is unlabored at rest. He has signs of chest
hyperinflation and decreased breath sounds without wheezing. He has no peripheral
edema. The remainder of his examination is normal; results of a fecal occult blood
test are negative. Baseline spirometry is unchanged. Forced expiratory volume in 1 sec
(FEV1) 35% of predicted Forced vital capacity (FVC) 85% of predicted FEV1/FVC ratio50%
PO2 62 mm Hg PCO2 45 mm Hg pH 7.38 (with the patient breathing room air) Chest
radiograph reveals only hyperinflation. What is the best way to manage this patient’s
weight loss?
A) Refer him for pulmonary rehabilitation with exercise and nutritional counseling.
B) Provide dietary instructions to increase his caloric intake.
C) Treat him with anabolic steroids
D) Prescribe oxygen supplementation to improve his oxygen consumption.
E) Add inhaled corticosteroids to his medical regimen.
ANSWER: A
73-yr-old fit farmer presents with pain on weight bearing and restricted movements of
the right hip. Choose the most likely diagnosis from the below list of options.
A) Ankylosing spondylitis
B) Erythema nodosum
C) Osteoarthritis
D) Gout
E) Psoriatic arthritis
ANSWER: C
79 y/o female with foot pain and swelling for many years. There are "punched out"
articular erosions with overhanging cortex and associated soft tissue masses. Serum
uric acid was markedly elevated. Choose the most likely diagnosis from the below list
of options.
A) Ankylosing spondylitis
B) Rheumatoid arthritis
C) Osteoarthritis
D) Chronic Gouty arthritis
E) Psoriatic arthritis
ANSWER: D
A 16-yr-old boy has had a pyrexia, rash, raised ESR, 1 week after faringitis.
Subcutaneous nodules and a generalised rash is noted on the trunk. Choose the single
most likely diagnosis from the list of options below.
A) Erythema marginatum
B) Maculopapular rash
C) Erythema multiforme
D) Lichen planus
E) Erythema nodosum
ANSWER: A
A 20-yr-old man presents with urethitis and a painful swollen knee. Choose the most
likely diagnosis from the below list of options.
A) Rheumatoid arthritis
B) Reactive arthritis
C) Osteoarthritis
D) Haemochromatosis
E) Gonococcal arthritis
ANSWER: B
A 21-year-old woman is referred to you for evaluation of new-onset Raynaud’s
phenomenon. She has no other symptoms. Her physical examination is unremarkable except
for dilated nail-fold capillaries and a few scattered telangiectasias. Tests for
specific ANA reveal the presence of antibodies to topoisomerase I. The most likely
eventual diagnosis for this patient is which of the following?
A) Isolated Raynaud’s disease
B) Diffuse systemic sclerosis
C) Limited systemic sclerosis
D) Mixed connective tissue disease (MCTD)
E) SLE
ANSWER: B
A 21-yr-old student with cystic fibrosis rapidly deteriorated and developed acute
respiratory failure while in hospital. Which infection is the most possible cause of
deterioration of his state?
A) Pseudomonas aeroginosa
B) Pneumocystis carinii
C) Chlamydia psittaci
D) Ecoli
E) Mycobacterium tuberculosis
ANSWER: A
A 22-yr-old male soldier presents with a 2-week history of a swollen right knee,
conjunctivitis and arthritis. Choose the most likely diagnosis from the below list of
options.
A) Ankylosing spondylitis
B) Reactive arthritis
C) Osteoarthritis
D) Gout
E) Psoriatic arthritis
ANSWER: B
A 25-year-old woman is referred to you for evaluation of new-onset Raynaud’s
phenomenon. She has no other symptoms. Her physical examination is unremarkable except
for dilated nail-fold capillaries and a few scattered telangiectasias. Tests for
specific ANA reveal the presence of antibodies to topoisomerase I.Choose the most
likely treatment from the below list of options.
A) D-penicillamine
B) Methotrexate
C) Indomethacin
D) Aspirin
E) Nifidipine
ANSWER: A
A 25-yr-old man presents with a 10-yr history of back pain, peripheral arthritis,
worse in the morning and one episode of iritis. Choose the medications from the below
list of options.
A) Methotrexate
B) Actovegin
C) Mydocalm
D) Warfarin
E) Pentoxifylline
ANSWER: A
A 25-yr-old woman presents with deep venous thrombosis in the right leg. Her past
history includes three miscarriages. Her blood tests show mild thrombocytopenia and a
positive serology test for syphilis. Choose the most likely diagnosis from the below
list of options.
A) Takayasu's arteritis
B) Polymyalgia rheumatica
C) Mixed connective tissue disorder
D) Antiphospholipid syndrome
E) Polyarteritis nodosa
ANSWER: D
A 27-yr-old woman presents with a 3-week history of fever, pleuritic chest pain,
stiffness and swelling in the wrists, MCP joints and PIP joints. On examination, there
is bilateral pretibial oedema. Choose the most likely treatment from the below list of
options.
A) Asprin
B) Indomethacin
C) Prostoglandines
D) Procainamide
E) Steroids
ANSWER: E
A 29-yr-old woman presented with DVT of the left calf. She had a history of recurrent
abortions and arthritis. Choose the most likely diagnosis from the below list of
options.
A) Protein C deficiency
B) Thrombotic thrombocytopenia
C) Antiphospholipid syndrome
D) SLE
E) Polycythemia rubra vera
ANSWER: C
A 31-year-old woman presents to the emergency department complaining of episodes of
dizziness, lightheadedness, palpitations, sweats, anxiety, and confusion. On the
morning of admission, she reports that she almost passed out. Her husband, who is a
diabetic patient who requires insulin, checked her blood sugar level and noted it to
be low. Her symptoms resolved after drinking some orange juice. She is admitted to the
hospital for a prolonged fast. After 18 hours, she becomes symptomatic, and her blood
is drawn. The serum glucose concentration is 48 mg/dl, the serum insulin level is high,
and test results are negative for insulin antibodies. The C-peptide level is low, and
tests for sulfonylurea and meglitinides are negative. Which of the following is the
most likely diagnosis for this patient?
A) Insulinoma
B) Factitial hypoglycemia
C) Noninsulinomapancreatogenous hypoglycemia syndrome (NIPHS)
D) Insulin autoimmune hypoglycemia
E) None of above
ANSWER: B
A 32-year-old man presents to your clinic as a new patient to establish primary care.
He has a 2-year history of hypertension, which is managed with a calcium channel
blocker. He has no knowledge of the cause of his hypertension. He is currently without
complaints and only wants a refill on his medication. His physical examination is
unremarkable, but laboratory results show hypokalemia. Which of the following
statements regarding hyperaldosteronism is true?
A) The most common causes of secondary hyperaldosteronism are congestive heart failure
and cirrhosis with ascites
B) Treatment of primary adrenal hyperaldosteronism is spironolactone
C) Patients with primary adrenal hyperaldosteronism usually present with hypertension,
hypokalemia, and metabolic acidosis
D) Diagnosis of primary adrenal hyperaldosteronism is confirmed by elevations in the
levels of both renin and aldosterone
E) None of them
ANSWER: A
A 32-year-old man presents to your clinic for evaluation of headaches. He has had
episodic pounding headaches for 6 months. He never had headaches like this before in
his life. He denies using illicit drugs. He has no family history of hypertension. On
physical examination, the patient is hypertensive, with a blood pressure of 180/105 mm
Hg. No further abnormalities are noted. You begin a workup for secondary causes of
hypertension. Which of the following statements regarding pheochromocytomas is true?
A) 90% of pheochromocytomas are malignant
B) Almost all patients with pheochromocytomas have episodic hypertension, making
diagnosis difficult in a single clinic visit
C) The incidence of pheochromocytoma is increased in patients with multiple endocrine
neoplasia type 1
D) Treatment of pheochromocytoma is surgical; alpha blockade should be induced with
phenoxybenzamine, beginning 7 days before surgery
E) None of them
ANSWER: D
A 32-year-old woman is evaluated because of a 5-year history of Raynaud’s phenomenon
and thickened skin over the fingers, small patches of calcinosis on the distal fingers
and dorsum of one hand. She reports mild dyspnea when she climbs stairs. What changes
reveals chest X-ray?
A) Bronchiolitis with organizing pneumonia
B) Bibasal pneumofibrosis
C) Lung cancer
D) Emphysema
E) Pulmonary tromboembolism
ANSWER: B
A 35 year old lady complains dysphagia, Raynaud's phenomenon, sclerodactyly.
Investigations show anticentromere antinuclear antibody. The likely diagnosis is:
A) Systemic lupus erythematosis
B) Systemic sclerosis
C) Mixed connective tissue disorder
D) Rheumatoid arthritis
E) Dermatopolymyositis
ANSWER: B
A 35-yr-old woman presented with arthralgia, myalgia, facial rash and blood pressure
of 190/110 and impaired renal function. Urine microscopy showed scanty red cells and
granular casts. Renal biopsy showed linear IgG on glomerular basement membrane. Choose
the most likely diagnosis from the below list of options.
A) Tuberculosis
B) Renal tubular acidosis
C) Lupus nephritis
D) Uraemia
E) Renal artery stenosis
ANSWER: C
A 36-year-old woman presents with delirium and congestive heart failure. Her husband
indicates that she has been losing weight and becoming more anxious and irritableover
the past 3 months. Over the past several weeks she has developed dyspnea and
peripheral edema. She has previously been healthy and takes no medications. Her
husband says that she drinks alcohol moderately and has never used illicit drugs. On
physical examination, she is awake, anxious, and confused. Her temperature is 38°C and
her heart rate is 142 and regular. She has jugular venous distension to 16 cm above
the sternal angle as well as bibasilar rales. In addition, she has a diffuse goiter
with a soft bruit over each lobe, as well as a stare expression and exophthalmos. CXR
shows pulmonary edema and cardiomegaly. Her EKG reveals sinus tachycardia but is
otherwise unremarkable. What is the best approach to management of this patient?
A) Admit to the general medicine ward, obtain serum-free T 4 and T SH, order a
radioiodine uptake and scan, and begin furosemide 40 mg IV daily.
B) Order free T 4 and T SH, start the patient on propranolol 20 mg potid and lasix 40
mg po bid, obtain a radioiodine uptake and scan, and follow closely as an outpatient .
C) Obtain free T 4, T SH, and thyroid-stimulating immunoglobulin levels, begin
methimazole 10 mg potid, and follow closely as an outpatient .
D) Admit to the general medicine ward, obtain blood and urine cultures and an
echocardiogram, and begin treatment with broad-spectrum antibiotics and furosemide.
E) Admit the patient to the intensive care unit, order free T 4 and T SH, and begin
high-dose propranolol, propylthiouracil, potassium iodide, corticosteroids, furosemide,
and acetaminophen.
ANSWER: E
A 38-year-old woman who is a longtime patient of yours comes to the clinic for her
annual appointment. Despite multiple attempts at dietary and behavior modification,
she has been gaining weight regularly since she was 23 years of age. In addition to
obesity, she has moderately controlled hypertension and glucose intolerance and
worsening osteoarthritis in her knees bilaterally. She requests to be placed on a more
intensive weight-loss therapy. On the basis of this patient's current BMI of 36 kg/m2,
which of the following would likely be the most appropriate therapy?
A) 3 to 6 months of intensive dietary restriction
B) 3 to 6 months of intensive dietary restriction and exercise
C) Initiation of pharmacologic therapy
D) Referral for bariatric surgery
E) A and B
ANSWER: D
A 39-year-old male developed acute, severe pain and swelling of the left elbow. Within
next few hours, pain worsened and he was unable to move the elbow joint, which was
tender, erythematous, and swollen on examination. Choose the most likely diagnosis
from the below list of options.
A) Ankylosing spondylitis
B) Erythema nodosum
C) Osteoarthritis + synoviitis
D) Gouty arthritis
E) Psoriatic arthritis
ANSWER: D
A 40-year-old diabetic patient presents with a blood pressure (BP) of 145/90 mmHg and
proteinuria. What is the best medication for the initial management of this patient’s
hypertension?
A) Calcium channel blockers
B) Beta blockers
C) ACE-inhibitors / angiotensin receptor blockers
D) Alpha blockers
E) Diuretics
ANSWER: C
A 40-year-old diabetic patient presents with a blood pressure (BP) of 145/90 mmHg and
proteinuria. Which BP profile represents the best therapeutic goal for this patient?
A) &amp;amp;lt;160/90
B) &amp;amp;lt;140/90
C) &amp;amp;lt;130/85
D) &amp;amp;lt;125/75
E) &amp;amp;lt;140/85
ANSWER: C
A 40-year-old diabetic patient presents with a blood pressure (BP) of 145/90 mmHg and
proteinuria. What is the best medication for the initial management of this patient’s
hypertension?
A) Calcium channel blockers
B) Beta blockers
C) ACE-inhibitors / angiotensin receptor blockers
D) Alpha blockers
E) Diuretics
ANSWER: C
A 42-yr-old woman complains of a 4-month history of Raynaud's phenomenon, progressive
skin tightness, thickening of fingers and hands, dyspnoea on exertion and dysphagia.
Choose the most likely treatment from the below list of options.
A) D-penicillamine
B) Methotrexate
C) Indomethacin
D) Aspirin
E) Nifidipine
ANSWER: A
A 45-year-old man is evaluated in the emergency department because of an acute, severe
asthma attack; he is hospitalized in the intensive care unit for aggressive medical
therapy and monitoring. He is expectorating thick greenish sputum. His medical history
includes hypertension, cholecystectomy, and glaucoma. Chest radiograph reveals
hyperinflation only. Medical therapy in the emergency department included repeated
doses of aerosolized albuterol and ipratropium, as well as methylprednisolone, 125mg
administered intravenously. Peak expiratory flow rate is unimproved at 80 L/min. Which
of the following is the most appropriate next step in this patient’s management?
A) Intravenous magnesium sulfate
B) Nebulized ipratropium bromide administered by face mask
C) Broad-spectrum antibiotics targeting community-acquired respiratory pathogens
D) Inhaled corticosteroids
E) Non of above
ANSWER: A
A 45-year-old man with a history of primary hyperparathyroidism comes to your clinic
for a follow-up visit. He was diagnosed 3 years ago after routine blood tests revealed
an elevation in calcium level. He has no complaints. Review of systems is negative,
and his physical examination is unremarkable. His family history is negative for
similar problems. His calcium level is 11.5 mg/dl. A DEXA scan shows a T score of –2
at the hip. What is the most appropriate treatment regimen for this patient?
A) Observation, with routine follow-up visits that include assessment of calcium
levels and DEXA scans
B) Start a bisphosphonate
C) Refer to an experienced surgeon for parathyroid surgery
D) Administer calcium, 1,000 to 1,500 mg/day, and vitamin D, 400 to 800 IU/day
E) All of them
ANSWER: C
A 46-year-old African-American man presents to your office for a routine visit. On his
last visit, which was 18 months ago, he was noted to be mildly hypertensive and obese.
He underwent blood test screening for high lipid levels, thyroid disease, and glucose
intolerance; all results were normal. His chief complaint today is excess fatigue. He
reports that he frequently falls asleep during the day, occasionally when driving a
car. He also reports occasional morning headaches. Which of the following approaches
are more likely to disclose the cause of this patient's fatigue?
A) Repeat thyroid function tests
B) Transthoracic echocardiogram
C) Pulmonary function tests
D) The patient should be asked about his sleeping habits and referred for a sleep
study
E) Repeat US of thyroid gland
ANSWER: D
A 50-year-old obese woman has long-standing type 2 diabetes mellitus inadequately
controlled on metformin and pioglitazone. Insulin glargine (15 units subcutaneouslyat
bedtime) has recently been started because of a hemoglobin A1C level of 8.4. Over the
weekend, she develops nausea, vomiting, and diarrhea after exposure to family members
with a similar illness. Afraid of hypoglycemia, the patient omits the insulin for 3
nights. Over the next 24 hours, she develops lethargy and is brought to the emergency
room. On examination, she is afebrile and unresponsive to verbal command. Blood
pressure is 84/52. Skin turgor is poor and mucous membranes dry. Neurological
examination is nonfocal; she does not have neck rigidity. Laboratory results are as
follows: Na: 126 mEq/L, K 4.0 mEq/L Cl: 95 mEq/L HCO3: 22 mEq/L, Glucose: 1100 mg/dL,
BUN: 84 mg/dL, Creatinine: 3.0 mg/dL. Which of the following is the most likely cause
of this patient’s coma?
A) Diabetic ketoacidosis
B) Hyperosmolar coma
C) Syndrome of inappropriate ADH secretion
D) Drug-induced hyponatremia
E) Bacterial meningitis
ANSWER: B
A 50-year-old woman is 5 ft 7 in tall and weighs 185 lb. T here is a family history of
diabetes mellitus. Fasting blood glucose (FBG) is 160 mg/dL and 155 mg/dL on
twooccasions. HgA1c is 7.8%. You educate the patient on medical nutrition therapy. She
returns for reevaluation in 8 weeks. She states she has followed diet and exercise
recommendations, but her FBG remains between 130 and 140 and HgA1C is 7.3%. She is
asymptomatic, and physical examination shows no abnormalities. Which of the following
is the treatment of choice?
A) Thiazolidinedionessuchaspioglitazone
B) Encourage compliance with medical nutrition therapy
C) Insulinglargineatbedtime
D) Metformin
E) Glipizide
ANSWER: D
A 50-year-old woman is evaluated for hypertension. Her blood pressure is 130/98. She
complains of polyuria and mild muscle weakness. She is on no blood pressuremedication.
On physical examination, the PMI is displaced to the sixth intercostal space. T here
is no sign of congestive heart failure and no edema. Laboratory values are as follows:
Na+: 147 mEq/dL, K+: 2.6 mEq/dL, Cl?: 112 mEq/dL, HCO3: 27 mEq/dL. T he patient denies
the use of diuretics or over-the-counter agents to decrease fluid retention or promote
weight loss. She does not eat licorice. Which of the following is the most useful
initial diagnostic test?
A) 24-hour urine for cortisol
B) Urinary metanephrine
C) Plasma renin activity
D) Renal angiogram
E) Ratio of serum aldosterone to plasma renin activity
ANSWER: E
A 52-yr-old woman complains of a 8-month history of Raynaud's phenomenon, progressive
skin tightness, thickening of fingers and hands, dyspnoea on exertion and dysphagia.
Choose the most likely diagnosis from the below list of options.
A) Rheumatoid arthritis
B) Polymyalgia rheumatica
C) SLE
D) Polyarteritis nodosa
E) Scleroderma
ANSWER: E
A 53-year-old woman presents to your clinic complaining of transient episodes of
diaphoresis, asthenia, near syncope, and clouding of thought process; she has had
these symptoms for several months. These episodes most commonly occur several hours
after she eats. She has no other significant medical history and takes no medications.
A prolonged fast is begun, during which the patient becomes symptomatic. Her serum
glucose concentration at the time is 43 mg/dl. The insulin level is elevated, and no
insulin antibodies are present. The C-peptide level is high, and tests for the use of
sulfonylureas and meglitinides are negative. What is the most effective therapy for
this patient's condition?
A) Observe the patient and schedule a follow-up fast 2 to 3 months from now
B) Adjust the patient's diet to include smaller, more frequent meals
C) Obtain a transabdominal ultrasound and refer the patient to surgery for resection
D) Begin diazoxide, 400 mg t.i.d., and verapamil, 180 mg q.d., and have the patient
appear for a follow-up visit in 3 months
E) Begin phenytoin and octreotide and have the patient appear for a follow-up visit in
3 months
ANSWER: C
A 54-year-old woman comes to your clinic for a routine visit. She has no active
complaints. Her medical history is positive only for mild asthma and arterial
hypertension. Her only medications are albuterol, which she administers with a
measured-dose inhaler as needed, and an angiotensin-converting enzyme inhibitor. She
smokes one pack of cigarettes a day. She has a strong family history of osteoporosis.
Her physical examination is unremarkable. You have a discussion with her regarding her
risk of osteoporosis, and you decide to obtain a dual-energy x-ray absorptiometry
(DEXA) scan for screening. The results show a T score of –2.6. Her creatinine and
albumin levels are normal, her liver function tests are normal except for a slightly
elevated alkaline phosphatase level, and her calcium level is 11 mg/dl. What is the
most appropriate step to take next in the treatment of this patient?
A) Start bisphosphonate, calcium, and vitamin D, and reassess in 6 months
B) Measure the parathyroid hormone (PTH) level with a two-site immunoradiometric assay
(IRMA, or so-called intact PTH) and assess 24-hour urinary calcium output
C) Order CT scans of the chest and abdomen to look for an occult malignancy
D) Start hormone replacement therapy with estrogens and progestins
E) A and B
ANSWER: B
A 55-year-old man presents to the office with erectile dysfunction. He has mild
diabetes and is on an ACE inhibitor for hypertension. He and his wife enjoy a
goodrelationship, and there is little external stress. He has, however, noted a
lessening of sexual desire; they have not had intercourse in the past 6 months. T he
general physical examination is normal. In particular, his peripheral sensation to
monofilament is intact, and vascular examination of the lower extremities is normal.
Testicular size is mildly decreased bilaterally. Which of the following is the most
appropriate first step in evaluation?
A) Serum-free testosterone and gonadotrophin levels
B) Hemoglobin A1C and ankle-brachial index
C) Psychologicalevaluation
D) T herapeutic trial of sildenafil
E) Morning total testosterone and prolactin level
ANSWER: E
A 55-year-old woman comes to your office with the results of a screening DEXA scan.
Her T score is –2.7. She has not had any symptoms and denies having any previous
fractures. She underwent menopause 3 years ago. Six months ago, she underwent
mammography and had a Pap smear, both of which were negative. She has no family
history of breast cancer. Her physical examination, including examination of the
breasts, is normal. Her laboratory workup shows no evidence of conditions that are
secondary causes of osteoporosis. What is the most appropriate recommendation
regarding the management of this patient's osteoporosis?
A) Start bisphosphonate therapy, start calcium and vitamin D therapy, and recommend
exercise
B) Start hormone replacement therapy, start calcium and vitamin D therapy, and
recommend exercise
C) Start calcitonin, calcium, and vitamin D therapy, and recommend exercise
D) Do not start therapy until the osteoporosis becomes symptomatic
E) None of above
ANSWER: A
A 57-year-old man with advanced chronic obstructive pulmonary disease (COPD) and
systemic hypertension is evaluated because of a 6-day history of productive cough and
shortness of breath. He uses inhaled albuterol and ipratropium bromide, a long-acting
theophylline preparation, and lisinopril. He uses supplemental oxygen at night and
during ambulation. Ciprofloxacin is prescribed for an exacerbation of COPD. Three days
later, having had nausea for a day, the man is brought to the emergency department
after he is found nearly unconscious. Arterial oxygen saturation is 89%, with the
patient breathing room air. Electrocardiogram shows normal sinus rhythm with
nonspecific ST-T changes in the lateral chest leads. Which of the following is likely
to have interacted with ciprofloxacin and caused the symptoms that brought the man to
the emergency department?
A) Theophylline
B) Albuterol
C) Ipratropium bromide
D) Lisinopril
E) Oxygen
ANSWER: A
A 58-year-old man is referred to your office after evaluation in the emergency room
for abdominal pain. The patient was diagnosed with gastritis, but a CT scan
withcontrast performed during the workup of his pain revealed a 2-cm adrenal mass. T
he patient has no history of malignancy and denies erectile dysfunction. Physical
examination reveals a BP of 122/78 with no gynecomastia or evidence of Cushing
syndrome. His serum potassium is normal. What is the next step in determining whether
this patient’s adrenal mass should be resected?
A) Plasma aldosterone/renin ratio
B) Estradiol level
C) Plasma metanephrines and dexamethasone-suppressed cortisol level
D) Testosterone level
E) Repeat CT scan in 6 months
ANSWER: C
A 59-year-old man with advanced chronic obstructive pulmonary disease is evaluated
because of a daily cough productive of white or yellow sputum, dyspnea after climbing
one flight of stairs, and a recent 4.5-kg weight loss with no associated change in
appetite or food intake. The patient stopped smoking 4 years ago. On physical
examination, he has diminished breath sounds throughout all lung fields. Arterial
oxygen saturation measured by pulse oximetry with the patient at rest, breathing room
air, is 87%. Chest radiograph suggests hyperinflation of the lungs but shows no
pulmonary infiltrates or abnormalities of the cardiac silhouette. Pulmonary function
studies show a forced expiratory volume in 1 sec 39% of predicted and forced vital
capacity 78% of predicted. Which of the following may prolong life in this patient?
A) Supplemental oxygen
B) Albuterol
C) Ipratropium bromide
D) Theophylline
E) Lisinopril
ANSWER: A
A 66-year-old woman presents to a walk-in clinic with muscle spasms. She complains
that for the past 2 days she has had muscle spasms in her hands, arms, and legs. She
has a medical history of cervical Hodgkin lymphoma, which was treated with radiation.
She does not take any medications or vitamins. On physical examination, the Trousseau
sign is positive. Her calcium level is 6.8 mg/dl; the albumin level is normal. On the
basis of this patient's history, what is the most likely diagnosis, and what should be
the treatment?
A) Hypoparathyroidism secondary to radiation therapy; start PTH injections
B) Vitamin D deficiency secondary to poor intake and lack of sunlight; start
calcitriol
C) Vitamin D deficiency secondary to poor intake and lack of sunlight; start
cholecalciferol
D) Hypoparathyroidism secondary to radiation therapy; start calcium and calcitriol
E) A and B.
ANSWER: D
A 67-year-old man is evaluated because of a 3-week history of cough productive of
blood-streaked sputum. A chest radiograph shows an infiltrate in the right upper lobe.
He is treated with antibiotics for 2 weeks, but the blood-streaked sputum persists. A
CT scan of the chest shows a mass obstructing the right upper lobe and evidence of
postobstructive pneumonitis. Examination of the mediastinum shows enlarged lymph nodes
in the right paratracheal space. A positron emission tomography (PET) scan shows
uptake in the mass itself and in the lymph nodes in the right paratracheal space.
Fiberoptic bronchoscopy is performed; an endobronchial lesion is identified and a
sample is taken for biopsy. The biopsy shows squamous cell carcinoma. Which of the
following is the best next step in this patients management?
A) Perform immediate mediastinoscopy
B) Refer him for radiation therapy
C) Refer him for surgery
D) Perform a repeat positron emission tomography scan
E) Non of above
ANSWER: A
A 73-yr-old fit farmer presents with pain on weight bearing and restricted movements
of the right hip. Choose the most likely X-ray changes from the below List of options.
A) Cystic changes and lytic lesions
B) Joint spaces are narrowed with reactive subchondral sclerosis, osteophytes
C) Periarticular osteopenia, marginal erosions
D) acroosteolysis
E) Fluffy bone periostitis and erosions at the interphalangeal joints
ANSWER: B
A blood test for determination of serum iron is indicated for the patient with iron
deficiency anaemia. Indicate normal concentration of serum iron in norm (µg/dL)
A) 20-30
B) 10-25
C) 170-240
D) 30-170
E) 15-55
ANSWER: D
A newly diagnosed diabetic patient presents with BP readings that are 155/95 or higher.
All of the following statements about the treatment of this patient's hypertension are
correct EXCEPT:
A) Pharmacologic blockade of the renin-angiotensin system reduces the risk of both
microvascular and macrovascular events
B) Aggressive BP control reduces cardiovascular events more in diabetics than in
nondiabetics
C) Calcium channel blockers show no benefit in reducing cardiovascular events
D) The goal BP for this patient is &amp;amp;lt;130/80 mmHg
E) All the above statements are correct
ANSWER: C
A young hypertensive patient has serum potassium 2.8 mEq/l and increased aldosterone
level with decreased plasma renin activity. The likely diagnosis is....
A) Renal artery stenosis
B) Ectopic ACTH syndrome
C) Conn syndrome
D) Liddle syndrome
E) Cushing syndrome
ANSWER: C
Absolute contraindication for recombinant erythropoietin prescription:
A) Noncontrolled blood hypertension
B) uremic neuropathy
C) hypoparathyroidism
D) aseptic bone necrosis
E) carbohydrate disturbunces
ANSWER: A
Accurate statement about rheumatoid factors include all of the following EXCEPT?
A) They are antibodies to the Fc fragment of immunoglobulin G
B) They are associated with several conditions in which there is chronic antigenic
stimulation
C) Their presence in the serum of persons with rheumatoid arthritis correlates with a
worse prognosis than that for persons with seronegative disease
D) Rheumatoid factor may be seen in association with many other diseases
E) They are associated with Still's syndrome
ANSWER: E
Acute nephritic syndrome is characterized by sudden onset of all of following except:
A) acute renal failure ;
B) oliguria (&amp;amp;lt;400 mL of urine per day);
C) hematuria;
D) hypertension;
E) Proteinuria more3.5 g per 24 h.
ANSWER: E
All of the following are characteristic findings of ostium primum atrial septal defect
(ASD) EXCEPT:
A) Precordial heave
B) Fixed split S2
C) Right axis deviation
D) Systolic ejection murmur
E) Prominent pulmonary vascular markings on CXR
ANSWER: C
All of the following are characteristic findings of ostiumprimum atrial septal defect
(ASD) EXCEPT:
A) Precordial heave
B) Fixed split S2
C) Right axis deviation
D) Systolic ejection murmur
E) Prominent pulmonary vascular markings on CXR
ANSWER: C
All of the following are ECG manifestations of Wolff-Parkinson-White syndrome EXCEPT
A) Narrow QRS
B) Initial slurred QRS upstroke
C) Short PQ interval
D) Normal QT interval
E) All of the above
ANSWER: A
All of the following are features of renovascular hypertension due to fibromuscular
hyperplasia, as opposed to atherosclerosis, EXCEPT:
A) Age &amp;amp;lt;50 years
B) Female gender
C) More likely to progress to complete renal artery occlusion
D) No family history of hypertension
E) Absence of carotid bruits
ANSWER: C
All of the following are indications for surgical closure of an ASD EXCEPT:
A) Significant symptoms in a 65-year-old
B) RV dysfunction
C) Pulmonary vascular resistance &amp;amp;gt;15 Wood units that does not diminish with
vasodilators
D) An asymptomatic 20-year-old with a Qp/Qs of 1,7 with no pulmonary hypertension
E) RV enlargement
ANSWER: C
All of the following are risk factors for hypertension, EXCEPT
A) Increased body weight
B) Family history of hypertension
C) Excessive intake of sodium
D) Regular use of one glass of wine per day
E) Cigarette smoking
ANSWER: D
All of the following are X-ray signs of Osteoarthritis except
A) Joint space narrowing
B) Marginal osteophytes
C) Subchondral cysts
D) Bony sclerosis
E) Marginal erosions
ANSWER: E
All of the following characteristics are typical of hypertensive crisis EXCEPT:
A) Diastolic BP &amp;amp;gt;120 mmHg
B) Retinal hemorrhages
C) Constriction of cerebral arterioles with decreased vascular permeability
D) Proteinuria
E) Microangiopathic hemolytic anemia
ANSWER: C
All of the following characteristics are typical of hypertensive crisis EXCEPT:
A) Diastolic BP &amp;amp;gt;120 mmHg
B) Retinal hemorrhages
C) Constriction of cerebral arterioles with decreased vascular permeability
D) Proteinuria
E) Microangiopathic hemolytic anemia
ANSWER: C
All of the following characteristics are typical of hypertensive crisis EXCEPT:
A) Diastolic BP &amp;amp;gt;120 mmHg
B) Retinal hemorrhages
C) Constriction of cerebral arterioles with decreased vascular permeability
D) Proteinuria
E) Microangiopathic hemolytic anemia
ANSWER: C
All of the following findings are suggestive of LVH except:
A) (S in V1 + R in V5 or V6) &amp;amp;gt; 35 mm
B) R in aVL &amp;amp;gt; 11 mm
C) R in aVF &amp;amp;gt; 20 mm
D) (R in I + S in III) &amp;amp;gt; 25 mm
E) R in aVR &amp;amp;gt; 8 mm
ANSWER: E
All of the following findings are suggestive of RVH except:
A) R in V1=7 mm (15 mm with RBBB)
B) (R in V1 + S in V5 or V6) &amp;amp;gt; 10 mm
C) R &amp;amp;lt; S in V6
D) R or R’ in V1
E) S in V3 + R in aVL &amp;amp;gt; 28 mm (20 mm for women)
ANSWER: E
All of the following interventions have a BP-lowering effect EXCEPT:
A) A diet that reduces caloric intake by 1000 calories per day
B) Reduction of dietary sodium
C) Daily magnesium supplements
D) Tobacco cessation
E) Reduction of ethanol consumption to less than 30 ml/day
ANSWER: C
All of the following physical examination findings are usually associated with
ostiumsecundum ASD EXCEPT:
A) Precordial heave
B) Fixed split S2
C) Lateral and inferior displacement of the apex beat
D) Soft systolic ejection murmur in the second left intercostal space
E) Normal S1
ANSWER: C
All of the following physical examination findings are usually associated with ostium
secundum ASD EXCEPT:
A) Precordial heave
B) Fixed split S2
C) Lateral and inferior displacement of the apex beat
D) Soft systolic ejection murmur in the second left intercostal space
E) Normal S1
ANSWER: C
All of the following physical findings may be seen in osteoarthritis except
A) Heberden nodes
B) Bouchar nodes
C) Bony crepitus on joint movement
D) Bounniere deformity
E) Acute arthritis of metatarsophalangeal joints.
ANSWER: E
All of the following statements concerning hypertension are true, EXCEPT
A) In a western adult population the prevalence of hypertension exceeds 20%
B) Hypertension is a major risk factor for cardiovascular and cerebrovascular disease
C) The pathophysiology of hypertension differs in black adults compared to South
Asians and whites
D) People of African descent commonly have a high renin type hypertension
E) An increase in potassium intake may significantly reduce blood pressure in
hypertensive patients
ANSWER: D
All of the following statements concerning target organ damage are true EXCEPT
A) Microalbuminuria is a sensitive marker of hypertension-induced renal damage
B) An increase in serum creatinine when antihypertensive therapy is intensified is a
sign of progressive renal deterioration
C) Electrocardiography should be part of all routine assessment of subjects with high
BP
D) Echocardiography is much more sensitive than electrocardiography in diagnosing left
ventricular hypertrophy
E) Grades 1 and 2 arteriolar retinal changes do not have a significant prognostic
value
ANSWER: B
All of the following statements concerning target organ damage are true EXCEPT
A) Microalbuminuria is a sensitive marker of hypertension-induced renal damage
B) An increase in serum creatinine when antihypertensive therapy is intensified is a
sign of progressive renal deterioration
C) Electrocardiography should be part of all routine assessment of subjects with high
BP
D) Echocardiography is much more sensitive than electrocardiography in diagnosing left
ventricular hypertrophy
E) Grades 1 and 2 arteriolar retinal changes do not have a significant prognostic
value
ANSWER: B
All of the following statements concerning target organ damage are true EXCEPT
A) Microalbuminuria is a sensitive marker of hypertension-induced renal damage
B) An increase in serum creatinine when antihypertensive therapy is intensified is a
sign of progressive renal deterioration
C) Electrocardiography should be part of all routine assessment of subjects with high
BP
D) Echocardiography is much more sensitive than electrocardiography in diagnosing left
ventricular hypertrophy
E) Grades 1 and 2 arteriolar retinal changes do not have a significant prognostic
value
ANSWER: B
All of the following statements regarding myocardial stunning are true EXCEPT:
A) Stunning is a state of depressed myocardial function due to chronic hypoperfusion
B) Stunning can be global or regional
C) Stunning can follow cardiac surgery with cardiopulmonary bypass
D) Oxygen-free radicals and excess intracellular calcium likely contribute to stunning
E) Stunning affects both systolic and diastolic function
ANSWER: A
All of the following statements regarding patent ductus arteriosus (PDA) are true
EXCEPT:
A) The majority of cases close spontaneously after infancy
B) There is a higher incidence in mothers who acquired rubella during pregnancy
C) A decrease in the duration and intensity of the murmur has a poor prognostic
implication
D) Left ventricular hypertrophy precedes RV hypertrophy
E) If it is uncorrected, approximately one third of patients die by the age of 40
years
ANSWER: A
All of the following statements regarding patent ductusarteriosus (PDA) are true
EXCEPT:
A) The majority of cases close spontaneously after infancy
B) There is a higher incidence in mothers who acquired rubella during pregnancy
C) A decrease in the duration and intensity of the murmur has a poor prognostic
implication
D) Left ventricular hypertrophy precedes RV hypertrophy
E) If it is uncorrected, approximately one third of patients die by the age of 40
years
ANSWER: A
All of the following statements regarding the association of oral contraceptive pills
and hypertension are correct EXCEPT:
A) The likelihood of developing hypertension is increased by significant alcohol
consumption
B) The incidence of hypertension is about twice as great in pill users as in non-users
C) The likelihood of developing hypertension is dependent on the age of the user
D) Hypertension resolves in almost all cases after discontinuation of oral
contraceptives
E) The mechanism for contraceptive-induced hypertension likely involves renin-
aldosterone-mediated volume expansion
ANSWER: D
All seen in multiple myeloma, except:
A) Lytic bone lesions
B) Hypercalcemia
C) Plasmacytosis of bone marrow &amp;amp;gt;10%
D) Plasma cells in bone marrow &amp;amp;lt; 2%
E) Bone fractures
ANSWER: D
All the following features favor ventricular tachycardia as the cause of broad-complex
tachycardia, EXCEPT:
A) QRS duration with right bundle branch block morphology (RBBB) is greater than 140
ms, and greater than 160 ms with left LBBB morphology
B) LAD with RBBB morphology, extreme LAD (northwest axis) with LBBB morphology
C) AV dissociation
D) Response to carotid sinus massage
E) Concordance of the QRS pattern in all precordial leads (i.e., all positive or all
negative deflections)
ANSWER: D
All the following patients should be evaluated for secondary causes of hypertension
EXCEPT
A) A 47-year-old male with strong family history hypertension who presents to your
office with a BP of 150/100
B) A 26-year-old female with hematuria and a family history of early renal failure who
has a BP of 160/90
C) A 73-year-old male with no past history with a BP of 165/90
D) A 58-year-old male with a history of hypertension since age 45 whose BP has become
increasingly difficult to control on four antihypertensive agents
E) A 31-year-old female with complaints of severe headaches, weight gain, and new-
onset DM with a BP of 145/90
ANSWER: A
All the following patients should be evaluated for secondary causes of hypertension
EXCEPT
A) A 47-year-old male with strong family history hypertension who presents to your
office with a BP of 150/100
B) A 26-year-old female with hematuria and a family history of early renal failure who
has a BP of 160/90
C) A 73-year-old male with no past history with a BP of 165/90
D) A 58-year-old male with a history of hypertension since age 45 whose BP has become
increasingly difficult to control on four antihypertensive agents
E) A 31-year-old female with complaints of severe headaches, weight gain, and new-
onset DM with a BP of 145/90
ANSWER: A
All the following patients should be evaluated for secondary causes of hypertension
EXCEPT
A) A 47-year-old male with strong family history hypertension who presents to your
office with a BP of 150/100
B) A 26-year-old female with hematuria and a family history of early renal failure who
has a BP of 160/90
C) A 73-year-old male with no past history with a BP of 165/90
D) A 58-year-old male with a history of hypertension since age 45 whose BP has become
increasingly difficult to control on four antihypertensive agents
E) A 31-year-old female with complaints of severe headaches, weight gain, and new-
onset DM with a BP of 145/90
ANSWER: A
An elderly man presents with a red, warm swollen metatarsophalangeal joint following a
right total hip replacement operation. Choose the most likely diagnosis from the below
list of options.
A) Ankylosing spondylitis
B) Reactive arthritis
C) Osteoarthritis
D) Acute Gouty arthritis
E) Psoriatic arthritis
ANSWER: D
An elderly man started frusemide 2-weeks ago and now presents with a red, hot, swollen
first metatarsal phalangeal joint. Choose the most likely diagnosis from the below
list of options.
A) Ankylosing spondylitis
B) Joint sepsis
C) Osteoarthritis
D) Gout
E) Psoriatic arthritis
ANSWER: D
An idiopathic left ventricular tachycardia (VT) shows the following...
A) Typical LBBB morphology
B) LAD and RBBB morphology
C) Associated with a high risk of sudden cardiac death and almost always requires ICD
therapy
D) Never responds to verapamil
E) All of the above
ANSWER: B
Anti-double stranded DNA is highly specific for
A) Systemic sclerosis
B) SLE.
C) Polymyositis
D) Dermatopolymyositis
E) Henoch schonlein purpura
ANSWER: B
ARF in association with fever, arthralgias, and a pruritic erythematous rash following
exposure to a new drug suggest:
A) vasculitis;
B) allergic interstitial nephritis;
C) hemolytic-uremic syndrome;
D) thrombotic thrombocytopenic purpura;
E) glomerulonephritis.
ANSWER: B
At a routine examination, an asymptomatic 46-year-old man is found to have a BP of
150/110 mmHg, but no other abnormalities are present. What do you do next?
A) Reassure the patient and repeat the physical examination in 12 months
B) Initiate antihypertensive therapy
C) Obtain repeated BP recordings in your office and/or the patient’s home or work site
D) Hospitalize patient for renal arteriography
E) Order a 24-h ambulatory BP monitoring
ANSWER: C
Bacterial endocarditis prophylaxis is indicated in all adults who have the following
congenital heart disease EXCEPT:
A) VSD
B) Coarctation of the aorta
C) Secundum ASD
D) Hypertrophic obstructive cardiomyopathy
E) PDA
ANSWER: C
Bacterial endocarditis prophylaxis is indicated in all adults who have the following
congenital heart disease EXCEPT:
A) VSD
B) Coarctation of the aorta
C) Secundum ASD
D) Hypertrophic obstructive cardiomyopathy
E) PDA
ANSWER: C
Besides pulmonary valve stenosis, which of the following is the most common associated
cardiac defect present in patients with PA stenosis?
A) VSD
B) ASD
C) Coarctation of the aorta
D) PDA
E) Bicuspid aortic valve
ANSWER: A
Besides pulmonary valve stenosis, which of the following is the most common associated
cardiac defect present in patients with PA stenosis?
A) VSD
B) ASD
C) Coarctation of the aorta
D) PDA
E) Bicuspid aortic valve
ANSWER: A
Beta blockers are accepted treatment to prevent VT in:
A) Brugada syndrome, RVOT-VT, short QT syndrome and long QT syndrome
B) Brugada syndrome, RVOT-VT and long QT syndrome
C) RVOT-VT and long QT syndrome
D) Short QT syndrome and long QT syndrome
E) Brugada syndrome and short QT syndrome
ANSWER: C
Bundle branch re-entry ventricular tachycardia is most commonly associated with:
A) Enhanced automaticity in the right bundle
B) Enhanced automaticity in the left bundle
C) Supranormal conduction in the His bundle
D) Abnormally slow conduction in the His bundle
E) All of the above
ANSWER: D
Bupropion hydrochloride (Zyban) is contraindicated in patients with a history of which
of the following?
A) Seizures
B) Insulin-dependent diabetes mellitus
C) Severe chronic obstructive pulmonary disease (with reversible component)
D) Longer than 40-year history of tobacco use
E) Recent MI
ANSWER: A
Cardiac involvement in SSC are all of the following EXCEPT
A) Pericardial effusion
B) Cor pulmonale
C) Conduction abnormalities
D) Vegetations on undersurface of A.V. valves
E) arrhythmias
ANSWER: D
Causes of Membranoproliferative (Mesangiocapillary) Glomerulonephritis (MPGN)
associated with chronic infections are all the following except:
A) Hepatitis B and C,
B) HIV,
C) bacterial endocarditis,
D) Leukemias,
E) visceral abscess
ANSWER: E
Characterise ECG feature of hyperkalemia:
A) U waves
B) Narrow QRS complex
C) Tall T waves
D) Sort PR interval
E) Pathologic Q wave
ANSWER: C
Choose the adult congenital disorder corresponding to the following physical
examination fingings: cyanosis, digital clubbing, loud P2, and a variable Graham-Steel
murmur
A) Eisenmenger's syndrome
B) Coarctation of the aorta
C) PDA
D) Ebstein's anomaly
E) Tetralogy of Fallot
ANSWER: A
Choose the adult congenital disorder corresponding to the following physical
examination fingings: wide pulse pressure, prominent LV impulse, and a continuous
machinery murmur enveloping S2
A) Eisenmenger's syndrome
B) Coarctation of the aorta
C) Patent ductusarteriosus
D) Ebstein's anomaly
E) Tetralogy of Fallot
ANSWER: C
Choose the adult congenital disorder corresponding to the following physical
examination fingings: RV lift with a loud systolic ejection murmur along the left
sternal border, with a single S2
A) Eisenmenger's syndrome
B) Coarctation of the aorta
C) PDA
D) Ebstein's anomaly
E) Tetralogy of Fallot
ANSWER: E
Choose the adult congenital disorder corresponding to the following physical
examination fingings: loud S1, holosystolic murmur in left sternal border, systolic
ejection click, and hepatomegaly
A) Eisenmenger's syndrome
B) Coarctation of the aorta
C) PDA
D) Ebstein's anomaly
E) Tetralogy of Fallot
ANSWER: D
Choose the adult congenital disorder corresponding to the following physical
examination fingings: weak or delayed femoral pulses, harsh systolic murmur in the
back, and a systolic ejection click in the aortic area
A) Eisenmenger's syndrome
B) Coarctation of the aorta
C) PDA
D) Ebstein's anomaly
E) Tetralogy of Fallot
ANSWER: B
Choose the adult congenital disorder corresponding to the following physical
examination fingings: cyanosis, digital clubbing, loud P2, and a variable Graham-Steel
murmur
A) Eisenmenger's syndrome
B) Coarctation of the aorta
C) PDA
D) Ebstein's anomaly
E) Tetralogy of Fallot
ANSWER: A
Choose the adult congenital disorder corresponding to the following physical
examination fingings: wide pulse pressure, prominent LV impulse, and a continuous
machinery murmur enveloping S2
A) Eisenmenger's syndrome
B) Coarctation of the aorta
C) Patent ductus arteriosus
D) Ebstein's anomaly
E) Tetralogy of Fallot
ANSWER: C
Choose the drug which is given in usual dosages, may be effective in treating urinary
tract infections in patients with acute renal failure?
A) Gentamicin;
B) Nitrofuratoin;
C) Cephalexin;
D) Carbenicillin;
E) Chloramphenicol
ANSWER: C
Classical picture of acute gouty arthritis includes all of the following except
A) Excruciating and sudden pain
B) Stiffness in the joint
C) Low-grade fever may also be present
D) Warmness
E) Overgrowth at the distal interphalangeal joints
ANSWER: E
Classification of Systemic Sclerosis includes all of the following EXCEPT
A) Limited cutaneous scleroderma
B) Diffuse cutaneous scleroderma
C) Scleroderma sine scleroderma
D) Eczema
E) Overlap syndromes
ANSWER: D
Common findings in autosomal dominant polycystic kidney disease are all of thr
following except?
A) Flank or back pain
B) gross hematuria
C) abdominal mass
D) frequent UTI's
E) nephrotic synrome
ANSWER: E
Commonest site of lytic lesion, in multiple myeloma is :
A) Vertebral column
B) Femur
C) Clavicle
D) Pelvis
E) Fingers phalangers
ANSWER: A
Complications of long-term dialysis include all the following EXCEPT
A) Increased risk of gastrointestinal malignancy
B) myocardial infarction
C) carpal tunnel syndrome
D) protein-calorie malnutrition
E) high-output congestive heart failure
ANSWER: A
Congenital long QT syndrome can lead to
A) Complete heart block
B) Polymorphic ventricular tachycardia
C) Acute myocardial infarction
D) Recurrent supraventricular tachycardia
E) All of the above
ANSWER: B
Congenital MR is commonly encountered in all of the following conditions EXCEPT:
A) Cor triatriatum
B) Ostium primum ASD
C) Coarctation of the aorta
D) Congenitally corrected TGA
E) Subaortic stenosis
ANSWER: A
Coronary sinus ASD is commonly associated with...
A) Supravalvular aortic stenosis
B) Supravalvular pulmonic stenosis
C) Cleft mitral valve
D) Anomalous pulmonary venous drainage
E) Persistent left superior vena cava
ANSWER: E
Coronary sinus ASD is commonly associated with...
A) Supravalvular aortic stenosis
B) Supravalvular pulmonic stenosis
C) Cleft mitral valve
D) Anomalous pulmonary venous drainage
E) Persistent left superior vena cava
ANSWER: E
Drug-induced systemic lupus erythematosus (SLE) can be characterized by which of the
following statements?
A) Twenty percent of patients receiving procainamide develop drug-induced lupus
B) Nephritis is a frequent consequence of hydralazine-induced lupus
C) If patients with drug-induced lupus fail to respond within several weeks of
discontinuing the offending agent, a trial of corticosteroids is indicated
D) If a patient with drug-induced lupus has persistent symptoms for longer than 6
months, an anti-ds antibody and CH50 levels should be drawn
E) None of the above
ANSWER: B
Each of the following statements regarding hypertension is true EXCEPT:
A) Increased LV muscle mass in hypertension is an independent risk factor of cardiac
mortality
B) The risk of ventricular arrhythmias in hypertension is increased 2-fold in the
presence of LV hypertrophy
C) Low birth weight is associated with an increased incidence of hypertension later in
life
D) Chronic caffeine consumption is associated with an increased risk of hypertension
E) In hypertensive adolescents, essential hypertension is the most likely etiology
ANSWER: D
Each of the following statements regarding hypertension is true EXCEPT:
A) Pure "white coat hypertension is found in 20 to 30% of patients
B) When measuring BP, an inappropriately small cuff size results in a spuriously low
systolic measurement
C) Pseudohypertension may occur in patients with sclerotic brachial arteries
D) Chronic renal disease is the second most common cause of hypertension after
essential hypertension
E) Coarctation of the aorta, Cushing disease, primary aldosteronism account for
approximately 1% of all hypertensive patients
ANSWER: B
Each of the following statements regarding hypertension is true EXCEPT:
A) The prevalence of hypertension rises progressively with age in both men and women
B) Systolic and diastolic hypertension are each associated with an increased crisk of
coronary heart disease
C) The target blood pressure for hypertensive patients with cardiovascular disease or
diabetes is 140/90 mmHg
D) Patients with prehypertension (SBP 120-139 mmHg or DBP 80-89 mmHg) benefit from
lifestyle modifications incl. weight reduction, regular exercise, and smoking
cessation
E) Drug therapy of hypertension benefits patients greater than 80 years of age
ANSWER: C
Each of the following statements regarding hypertension is true EXCEPT:
A) Pure "white coat hypertension is found in 20 to 30% of patients
B) When measuring BP, an inappropriately small cuff size results in a spuriously low
systolic measurement
C) Pseudohypertension may occur in patients with sclerotic brachial arteries
D) Chronic renal disease is the second most common cause of hypertension after
essential hypertension
E) Coarctation of the aorta, Cushing disease, primary aldosteronism account for
approximately 1% of all hypertensive patients
ANSWER: B
Each of the following statements regarding hypertension is true EXCEPT:
A) Increased LV muscle mass in hypertension is an independent risk factor of cardiac
mortality
B) The risk of ventricular arrhythmias in hypertension is increased 2-fold in the
presence of LV hypertrophy
C) Low birth weight is associated with an increased incidence of hypertension later in
life
D) Chronic caffeine consumption is associated with an increased risk of hypertension
E) In hypertensive adolescents, essential hypertension is the most likely etiology
ANSWER: D
Each of the following statements regarding pheochromocytoma is true EXCEPT:
A) Approximately 15% of pheochromocytomas are extra-adrenal
B) Ten percent of pheochromocytomas are malignant
C) Ten percent of adrenal pheochromocytomas are bilateral
D) Hypertension related to pheochromocytoma is nearly always episodic and only rarely
persistent
E) Pheochromocytoma can be inherited
ANSWER: D
Each of the following statements regarding splitting of the second heart sound is true
EXCEPT:
A) Severe pulmonic valvular stenosis is associated with a softened P2
B) Delayed closure of the pulmonic valve with inspiration contributes to physiologic
splitting of S2
C) Fixed splitting of S2 is the auscultatory hallmark of an ostium secundum atrial
septal defect
D) Paradoxical splitting of S2 is expected in patients with a right ventricular
electronically paced rhythm
E) Right bundle branch block is associated with paradoxical splitting of S2
ANSWER: E
Echocardiographic findings for infective endocarditis are all of the following EXCEPT:
A) New valvar stenosis
B) Mobile echo dense masses attached to valve leaflets or endocardium
C) Periannular abscesses
D) New dehiscence of a prosthetic valve
E) New valvar regurgitation
ANSWER: A
Endocrine function of kidneys include synthesis all of the following except
A) Renin
B) Erythropoetin
C) Prostaglandins
D) Aldosterone
E) Calcitriol
ANSWER: D
Eosinophiluria (>5% of urine leukocytes) is a common finding (~90%) in:
A) allergic interstitial nephritis induced by NSAIDs;
B) antibiotic-induced allergic interstitial nephritis;
C) thrombotic thrombocytopenic purpura;
D) amyloidosis;
E) glomerulonephritis.
ANSWER: A
Feature that distinguish rheumatic fever from myocarditis:
A) migratory arthritis
B) ST-T-wave abnormalities
C) Arrhythmias
D) heart failure
E) tachycardia.
ANSWER: A
Feature that distinguishes rheumatic heart diseases from infective endocarditis is:
A) Vegetations
B) Doppler echocardiographic restriction of mitral valve leaflets, their calcification
C) Pulmonary hypertension
D) Cardiomegaly
E) Syncope.
ANSWER: A
For a hypertensive 48-yr-old lady who has been on hydralazine therapy choose the
single most likely diagnosis from the below list of options.
A) SLE
B) Nephroblastoma
C) Haemolytic uraemic syndrome
D) Juvenile chronic arthritis
E) Henoch-Schonlein purpura
ANSWER: A
For Positive fluorescent antinuclear antibody choose the single most likely diagnosis
from the below list of options.
A) Wegener's granulomatosis
B) Goodpasture's syndrome
C) Multiple myeloma
D) Lupus nephritis
E) Diabetic neprtropathy
ANSWER: D
For the Systemic lupus erythematosus choose the most likely diagnostic antibody from
the below list of options:
A) Antimicrosomal antibody
B) C-ANCA
C) Anti-dsDNA
D) Antiparietal cell antibody
E) Antiacetylcholine receptor antibody
ANSWER: C
For the Systemic lupus erythematosus choose the most likely diagnostic antibody from
the below list of options.
A) Antimicrosomal antibody
B) C-ANCA
C) Antinuclea antibody
D) Antiparietal cell antibody
E) Antiacetylcholine receptor antibody
ANSWER: C
Fragmented RBC's are seen in
A) ITP
B) Spherocytosis
C) DIC
D) Patient on cardiac valves
E) CLL
ANSWER: C
Fundus picture of stage 2 of Keith-Wegener classification includes
A) Copper wire AV nipping and focal spasm
B) Silver wire increased reflex
C) Exudate and hemorrhagic spots
D) Papilloedema
E) Retinal hemorrhages
ANSWER: A
Fundus picture of stage 2 of Keith-Wegener classification includes
A) Copper wire AV nipping and focal spasm
B) Silver wire increased reflex
C) Exudate and hemorrhagic spots
D) Papilloedema
E) Retinal hemorrhages
ANSWER: A
Giant a waves in JVP (“cannon” waves) are seen in
A) Sinus bradycardia
B) Atrial fibrillation
C) First-degree AV-block
D) Complete heart block
E) Atrial ectopic
ANSWER: D
Heavy proteinuria is a frequent finding in patients with combined allergic
interstitial nephritis and minimal change glomerulopathy that treated with:
A) NSAIDs;
B) echinaceae;
C) heparinotherapy;
D) platelet aggregation inhibitor;
E) angiotensin-converting enzyme (ACE) inhibitors.
ANSWER: A
How many nephrons are there in each kidney?
A) 100 – 200 thousands
B) 200 – 300 thousands
C) 500 – 700 thousands
D) 1,2 – 1,5 millions
E) 2 – 3 millions
ANSWER: D
Hyperuricemia is defined as a serum uric acid concentration above
A) 7 mg per dL (420 Вµmol per L).
B) 4 mg per dL (240 Вµmol per L).
C) 5 mg per dL (300 Вµmol per L).
D) 8 mg per dL (480 Вµmol per L).
E) 9 mg per dL (540 Вµmol per L).
ANSWER: A
If there is suspicion of phaeochromocytoma, the first line drug is:
A) Sodium-nitroprusside
B) Nifedipine
C) Phentolamine
D) Verapamil
E) Enalapril
ANSWER: C
If you are in your office and a patient sees you with the following symptoms:long
history of hypertension, home BP usually 160/100 mmHg, suddenly developed severe
headache, blurred vision, weakness in the right arm, blood pressure: 200/110 mmHg,
What will you do:
A) Give the patient 20 mg nifedipine orally, and check the BP in 15 min. If it is
still &amp;amp;gt;200/100 mmHg, give another dose of nifedipine
B) Give the patient 20 mg nifedipine, and check the BP in 30 min. If it is 160/80 mmHg,
send the patient home and give another appointment in two weeks
C) Give the patient 10 mg amlodipine, send the patient home and give another
appointment in two days
D) Call the ambulance and send the patient to the nearest hospital
E) Give the patient some pain-killer drug (e.g. acetaminophen), call the ambulance and
send the patient to the nearest hospital with stroke-unit
ANSWER: E
Immunological phenomena in infective endocarditis are all of the following EXCEPT:
A) Erythema marginatum
B) Glomerular nephritis
C) Rheumatoid factor
D) Osler's nodes
E) Roth spots
ANSWER: A
In case of acute poststreptococcal glomerulonephritisthe most appropriate treatment is
A) Corticosteroids
B) Cytotoxics
C) Antibiotics, symptomatic treatment
D) Memranostabilisation therapy
E) Plasmapheresis
ANSWER: C
In case of IgA nephropathy the most appropriate treatment is
A) Corticosteroids
B) Cytotoxics
C) Antibiotics
D) Memranostabilisative therapy
E) Plasmapheresis
ANSWER: A
In case of Membranous nephropathy the most appropriate treatment is
A) Corticosteroids
B) Cytotoxics
C) Antibiotics
D) Memranostabilisation therapy
E) Plasmapheresis
ANSWER: B
In case of minimal change disease the most appropriate treatment is
A) Corticosteroids
B) Cytotoxics
C) Antibiotics
D) Membrane stabilization therapy
E) Plasmapheresis
ANSWER: A
In case of nephrotic syndrome in diabetic nephropathy with ESRD the most appropriate
treatment is
A) Corticosteroids
B) Cytotoxics
C) symptomatic treatment
D) Memranostabilisotion therapy
E) Renal replacement therapy
ANSWER: E
In case of resistant nephrotic syndrome in mesangiocapillary glomerulonephritis the
most appropriate treatment is
A) Corticosteroids
B) Cytotoxics
C) symptomatic treatment
D) Memranostabilisotion therapy
E) Plasmapheresis folowed corticosteroids
ANSWER: E
In patients of African descent the drug of choice for initial treatment of
hypertension is:
A) Verapamil
B) Atenolol
C) Perindopril
D) Hydrochlorothiazide
E) Amlodipine
ANSWER: D
In patients with a history of stroke or TIA the preferred drug combination is:
A) ACE-inhibitor and diuretic
B) Calcium-channel blocker and beta-blocker
C) Beta-blocker and diuretic
D) Angiotensin receptor blocker and beta blocker
E) Beta- and alpha blocker
ANSWER: A
In patients with a history of stroke or TIA the preferred drug combination is:
A) ACE-inhibitor and diuretic
B) Calcium-channel blocker and beta-blocker
C) Beta-blocker and diuretic
D) Angiotensin receptor blocker and beta blocker
E) Beta- and alpha blocker
ANSWER: A
In patients with a history of stroke or TIA the preferred drug combination is:
A) ACE-inhibitor and diuretic
B) Calcium-channel blocker and beta-blocker
C) Beta-blocker and diuretic
D) Angiotensin receptor blocker and beta blocker
E) Beta- and alpha blocker
ANSWER: A
In patients with nephritic syndrome the energy intake of diet is provided by increased
A) Increased content of Salt
B) Increased content of fats
C) Increased content of proteins
D) Inceased content of carbohydrates
E) All of then
ANSWER: D
In patients with ventricular ectopic beats:
A) Caffeine restriction always reduces symptoms
B) High caffeine intake should always be discouraged in symptomatic patients
C) There is good evidence that reducing caffeine intake reduces ectopics
D) Hypertension has no association with frequency of ectopic beats
E) LVH is not associated with increased frequency of ectopics
ANSWER: B
In the elderly with systolic hypertension antihypertensive therapy should be initiated
if SBP
A) ?120 mmHg
B) ?130 mmHg
C) ?140 mmHg
D) ?160 mmHg
E) ?170 mmHg
ANSWER: C
In the elderly with systolic hypertension antihypertensive therapy should be initiated
if SBP
A) ?120 mmHg
B) ?130 mmHg
C) ?140 mmHg
D) ?160 mmHg
E) ?170 mmHg
ANSWER: C
In the elderly with systolic hypertension antihypertensive therapy should be initiated
if SBP
A) ?120 mmHg
B) ?130 mmHg
C) ?140 mmHg
D) ?160 mmHg
E) ?170 mmHg
ANSWER: D
In what disease selective proteinuria is often observed?
A) Acute glomerulonephritis
B) Chronic glomerulonephritis
C) Renal amyloidosis
D) Diabetic nephrosclerosis
E) Tuberculosis of kidneys
ANSWER: A
In which disease we expect to see vegetations?
A) Infective endocarditis
B) Viral myocarditis
C) Acute pericarditis
D) Rheumatic heart disease
E) Hypertrophied cardiomyopathy
ANSWER: A
In which of the following cases is surgical correction recommended?
A) Asymptomatic small VSD to decrease risk of endocarditis
B) PDA with severe pulmonary hypertension
C) Asymptomatic subaortic stenosis with severe aortic valve insufficiency
D) Coarctation of the aorta with a transcoarctation gradient of 20 mmHg
E) Small ASD to prevent paradoxical embolization
ANSWER: C
In which of the following diseases test for anticentromere antibody is positive?
A) Scleroderma
B) Mixed connective tissue disorder
C) SLE
D) Rheumatoid arthritis
E) Polymyalgia rheumatica
ANSWER: A
In which of the following has the high-sensitivity C-reactive protein (CRP) been shown
to be predictive of risk?
A) Acute MI
B) Acute coronary syndromes
C) Chronic stable angina
D) Peripheral vascular disease
E) All of the above
ANSWER: E
In which of the following has the high-sensitivity C-reactive protein (CRP) been shown
to be predictive of risk?
A) Acute MI
B) Acute coronary syndromes
C) Chronic stable angina
D) Peripheral vascular disease
E) All of the above
ANSWER: E
Indication to renal biopsy are:
A) Nephrotic syndrome
B) Suspicion for renal tumor
C) Suspicion for renal tuberculosis
D) Edema
E) Suspicion for renal abscess
ANSWER: A
Is there a difference in the effects of furosemide and thiazide diuretics on the
calcium excretion by the kidney?
A) Yes, furosemide increase calcium excretion by the kidney and thiazide diuretics
decrease calcium excretion by the kidney
B) Yes, furosemide decrease calcium excretion by the kidney and thiazide diuretics
increase calcium excretion by the kidney
C) Yes, furosemide increase calcium excretion by the kidney and thiazide diuretics are
neutral
D) Yes, thiazide diuretics decrease calcium excretion by the kidney
E) No, the difference is absent
ANSWER: A
Laboratory finding in anemia of chronic disease is
A) TIBC decreased
B) Serum iron decreased
C) Ferritin decreased
D) Hypoalbulinemia
E) Reticulocytosis
ANSWER: A
Low serum iron and increased total iron binding capacity are seen in:
A) Iron deficiency anaemia
B) Anemia of chronic infection
C) Megaloblastic anemia
D) Sideroblastic anaemia
E) Folic acid deficiency anemia
ANSWER: A
Major causes of prerenal acute renal failure are all of the following except:
A) Hypovolemia;
B) Low cardiac output;
C) Renovascular obstruction;
D) Hyperviscosity syndrome;
E) Altered renal systemic vascular resistance ratio.
ANSWER: D
Major criteria of rheumatic fever are the following EXCEPT:
A) Recent group A streptococcal infection
B) Carditis
C) Migratory polyarthritis
D) Sydenham's chorea
E) Erythema marginatum
ANSWER: A
Match the chest radiography findings (marked cardiomegaly, severe right atrial
enlargement, and normal lung fields) with the corresponding congenital disorder
A) Eisenmenger's syndrome
B) Patent ductus arteriosus (PDA)
C) Ebstein's anomaly
D) Tetralogy of Fallot
E) Coarctation of the aorta
ANSWER: C
Match the chest radiography findings (prominent central PAs and peripheral PA prunning)
with the corresponding congenital disorder
A) Eisenmenger's syndrome
B) Patent ductus arteriosus (PDA)
C) Ebstein's anomaly
D) Tetralogy of Fallot
E) Coarctation of the aorta
ANSWER: A
Match the chest radiography findings (pulmonary plethora, prominent ascending aorta,
proximal PA dilatation) with the corresponding congenital disorder
A) Eisenmenger's syndrome
B) Patent ductus arteriosus (PDA)
C) Ebstein's anomaly
D) Tetralogy of Fallot
E) Coarctation of the aorta
ANSWER: B
Match the chest radiography findings (right aortic arch, RV enlargement, and a "boot-
shaped" heart) with the corresponding congenital disorder
A) Eisenmenger's syndrome
B) Patent ductus arteriosus (PDA)
C) Ebstein's anomaly
D) Tetralogy of Fallot
E) Coarctation of the aorta
ANSWER: D
Microscopic examination of urine in pyelonephritis reveals all of the following except:
A) numerous pus cells;
B) microorganisms;
C) fatty casts;
D) some red cells;
E) some epithelial cells.
ANSWER: C
Microscopic examination should check for all of following except:
A) crystals;
B) cells;
C) casts;
D) infecting organisms;
E) ketones.
ANSWER: E
Minor criteria for infective endocarditis are all of the following EXCEPT:
A) Migratory arthritis
B) Fever &amp;amp;gt;38В°C
C) Embolic/vascular phenomena
D) Immunological phenomena
E) Blood cultures compatible but not typical for endocarditis
ANSWER: A
Most common type of Hodgkin's lymphoma is:
A) Lymphocyte predominance
B) Mixed cellularity
C) Nodular sclerosis
D) Plasmacells
E) Lymphocyte depletion
ANSWER: C
Most patients with type I MPGN present with all of the following except:
A) heavy proteinuria;
B) nephrotic syndrome;
C) light proteinuria;
D) active urinary sediment;
E) normal or mildly impaired GFR.
ANSWER: C
Most sensitive and specific test for diagnosis of iron deficiency is:
A) Serum iron levels
B) Serum ferritin levels
C) Serum transferrin receptor population
D) Transferrin saturation
E) MCHC&amp;amp;lt;50%
ANSWER: B
Nephrotic syndrome may occur in all of the following morphological cases except
A) IgA nephropathy
B) minimal change disease
C) Mesangiocapillary glomerulonephritis
D) Membranous nephropathe
E) Mesangioproliferative glomerulonephritis
ANSWER: A
Nephrotic syndrome may occur in all of the following morphological cases except
A) Proliferative glomerulonephritis
B) Pyelonephritis
C) Dibetic nephropathy
D) Amyloidosis
E) minimal change disease
ANSWER: B
Noonan's syndrome is commonly associated with...
A) Supravalvular aortic stenosis
B) Supravalvular pulmonic stenosis
C) Cleft mitral valve
D) Anomalous pulmonary venous drainage
E) Persistent left superior vena cava
ANSWER: B
Noonan's syndrome is commonly associated with...
A) Supravalvular aortic stenosis
B) Supravalvular pulmonic stenosis
C) Cleft mitral valve
D) Anomalous pulmonary venous drainage
E) Persistent left superior vena cava
ANSWER: B
Normal levels of Glomerular filtration rate (GFR) are
A) 140-190 ml/min
B) 90-120 ml/min
C) 60-90 ml/min
D) 30-60 ml/min
E) 90-190 ml/min
ANSWER: B
Normal sizes of kidneys in adults are
A) 10-12 сm х 5-6 сm х 3-4 сm
B) 15-16 сm х 7-8 сm х 5-6 сm
C) 7-8 сm х 3-4 сm х 2-3 сm
D) 5-7 сm х 2-4 сm х 1-3 сm
E) 18-22 сm х 8-10 сm х 6-7 сm
ANSWER: A
On routine physical examination, a 28-year-old woman is found to have a thyroid nodule.
She denies pain, hoarseness, hemoptysis, or local symptoms. Serum T SH isnormal. Which
of the following is the best next step in evaluation?
A) Thyroidultrasonography
B) Thyroidscan
C) Surgicalresection
D) Fine needle aspiration of thyroid
E) Nofurtherevaluation
ANSWER: D
Optimal blood pressure (BP) is defined as a BP level of
A) &amp;amp;lt;120/80 mmHg
B) &amp;amp;lt;130/80 mmHg
C) &amp;amp;lt;140/90 mmHg
D) &amp;amp;lt;160/100 mmHg
E) &amp;amp;lt;125/80 mmHg
ANSWER: A
Ostium primum ASD is commonly associated with...
A) Supravalvular aortic stenosis
B) Supravalvular pulmonic stenosis
C) Cleft mitral valve
D) Anomalous pulmonary venous drainage
E) Persistent left superior vena cava
ANSWER: C
Ostiumprimum ASD is commonly associated with...
A) Supravalvular aortic stenosis
B) Supravalvular pulmonic stenosis
C) Cleft mitral valve
D) Anomalous pulmonary venous drainage
E) Persistent left superior vena cava
ANSWER: C
Overproduction of urate may be in all of the following except
A) Primary idiopathic hyperuricemia
B) Hypertension
C) Hemolytic processes
D) Lymphoproliferative disease
E) Myeloproliferative disease
ANSWER: B
Overproduction of urate may be in all of the following except
A) Polycythemia vera
B) Psoriasis (severe)
C) Peptic ulcer
D) Exercise
E) Alcohol
ANSWER: C
Patient H., a 52 years old man, is evaluated because of a 2-month history of
nonproductive cough, myalgias, and low-grade fever. When his illness began, a chest
radiograph showed bilateral alveolar infiltrates, and a presumptive diagnosis of
community-acquired pneumonia was made. He was treated with oral azithromycin without
effect, followed by a 10-day course of levofloxacin, also without effect. During the
course of his illness he has lost 4.5 kg without significant anorexia. He is a
lifetime nonsmoker and works as an office manager. He has no pets and no unusual
hobbies. On physical examination, his vital signs are normal, except of respiration
rate of 22/min. He is in mild respiratory distress on exertion. On examination of the
chest, bilateral crackles are audible, without wheezing. Chest radiograph shows
bilateral alveolar infiltrates, which are changed in location from those seen on his
original radiographs. Pulmonary function tests show forced expiratory volume in 1 sec
(FEV1) 75% of predicted, forced vital capacity (FVC) 72% of predicted, total lung
capacity 80% of predicted, and diffusing lung capacity for carbon monoxide 65% of
predicted. Arterial blood gas values, with the patient breathing room air, are PO2 62
mm Hg, PCO2 42 mm Hg, and pH 7.39.Which of the following is the most likely diagnosis?
A) Cryptogenic organizing pneumonitis
B) Hypersensitivity pneumonitis
C) Resistant pneumococcal pneumonia
D) Chlamydia pneumonia
E) Bronchoalveolar cell carcinoma
ANSWER: A
Patient undergoes genetic testing, which reveals a t(9;22) translocation. Which of the
following would a complete blood count most likely show?
A) Increased lymphocyte count
B) Increased plasmacyte count
C) Numerous lymphoblasts
D) Increased granulocytes
E) Pancytopenia
ANSWER: D
PatientI., a 50 years old man, is evaluated in the emergency department because of
fever, nonproductive cough and 2-day history of myalgia and headache. He has also had
nausea and diarrhea. He is a heavy smoker. On physical examination, he is slightly
disoriented. Temperature is 38.9 C, pulse rate is 110/min, respiration rate is 32/min.
Chest radiograph shows fluffy infiltrates to the right upper and lower lobes. Results
of laboratory testing show serum sodium of 128 meq/L, blood urea nitrogen of 42 mg/dL,
serum creatinine of 2.2 mg/dL, and serum creatine kinase of 250 U/L. Which one of the
following is best next step in the management of this patient’s pneumonia?
A) Initiate empiric antibiotic therapy for Legionella
B) Order direct fluorescent antibody testing of the sputum for Legionella
C) Order serologic testing for Legionella
D) Send a urine specimen for measurement of Legionella antigen
E) All of the above
ANSWER: A
Physicians should diagnose isolated office hypertension (so-called “white-coat
hypertension”) whenever office BP is ?140/90 mmHg at several visits, while 24-h
ambulatory BP is
A) &amp;amp;lt;160/100 mmHg
B) &amp;amp;lt;140/90 mmHg
C) &amp;amp;lt;130/85 mmHg
D) &amp;amp;lt;125/80 mmHg
E) &amp;amp;lt;120/70 mmHg
ANSWER: C
Processes of ultrafiltration in kidneys are stopped when systemic arterial blood
pressure is;
A) &amp;amp;lt;60 mm Hg
B) 140 -160 mm Hg
C) 120 -140 mm Hg
D) 90 -100 mm Hg
E) 70 -80 mm Hg
ANSWER: A
Regarding antithrombotic treatment in AF:
A) Patients with asymptomatic AF have less risk of embolism than symptomatic patients
B) Patients with atrial flutter do not need antithrombotic treatment due to the low
risk of embolism of this arrhythmia
C) Warfarin is the best option in patients with high risk of thromboembolism
D) Aspirin has a similar effect in stroke reduction to placebo
E) Paroxysmal AF has the lowest risk of embolism
ANSWER: C
Regarding ventricular ectopics and exercise:
A) Ectopics are usually benign if they are supressed on exercise
B) Exercise-induced VEBs are not associated with adverse prognosis
C) RVOT ectopics are usually supressed on exercise
D) Ectopics induced on recovery after exercise are benign
E) All of the above
ANSWER: A
Renal biopsy is particularly useful when clinical assessment and laboratory
investigations suggest diagnoses of all of following except:
A) glomerulonephritis,
B) hemolytic-uremic syndrome,
C) thrombotic thrombocytopenic purpura,
D) allergic interstitial nephritis.
E) ischemic renal injury
ANSWER: E
Renin-dependent hypertension includes:
A) Primary hyperaldosteronism
B) Essential hypertension
C) Renovascular hypertension
D) Pheochromocytoma
E) Cushing syndrome
ANSWER: C
Renin-dependent hypertension includes:
A) Primary hyperaldosteronism
B) Essential hypertension
C) Renovascular hypertension
D) Pheochromocytoma
E) Cushing syndrome
ANSWER: C
Renin-dependent hypertension includes:
A) Primary hyperaldosteronism
B) Essential hypertension
C) Renovascular hypertension
D) Pheochromocytoma
E) Cushing syndrome
ANSWER: C
Renin-dependent hypertension includes:
A) Primary hyperaldosteronism
B) Essential hypertension
C) Renovascular hypertension
D) Pheochromocytoma
E) Cushing syndrome
ANSWER: C
Rheumatic fever caused by:
A) beta-hemolytic streptococci.
B) Staphylococcus lugdunensis
C) Enterococci
D) The HACEK group
E) H. influenza
ANSWER: A
Risk factors and possible causes of osteoarthritis. Risk factors and possible causes
of osteoarthritis are all of the following except
A) Age
B) Obesity
C) Stress
D) Prior injury
E) Acromegaly
ANSWER: C
Secondary osteoarthritis may be caused by all of except
A) Hemaphilia
B) Hemochromatosis
C) Gouty arthritis
D) Diabetes mellitus
E) After trauma
ANSWER: D
Serological findings in patients with systemic lupus erythematosus (SLE) will NOT
include which one of the following:
A) A raised serum IgG level
B) Antibodies to double-stranded DNA
C) Decreased C3 and C4 levels
D) A positive ANA
E) Antibodies to cyclic citrullinated peptide (anti-CCP antibodies)
ANSWER: E
Sinus venosus ASD is commonly associated with...
A) Supravalvular aortic stenosis
B) Supravalvular pulmonic stenosis
C) Cleft mitral valve
D) Anomalous pulmonary venous drainage
E) Persistent left superior vena cava
ANSWER: D
Sinus venosus ASD is commonly associated with...
A) Supravalvular aortic stenosis
B) Supravalvular pulmonic stenosis
C) Cleft mitral valve
D) Anomalous pulmonary venous drainage
E) Persistent left superior vena cava
ANSWER: D
Specific indications to renal biopsy are all of following except:
A) adult nephritic syndrome
B) uncontrolled hypertension
C) persistent proteinuria &amp;amp;gt; 1 g/24 hours
D) adult acute nephritic syndrome
E) systemic diseases with renal involvement
ANSWER: B
Specific indications to renal biopsy are all of following except:
A) adult nephritic syndrome
B) acute pyelonephritis
C) persistent proteinuria &amp;amp;gt; 1 g/24 hours
D) adult acute nephritic syndrome
E) systemic diseases with renal involvement
ANSWER: B
Sudden cardiac death in patients >35 years old is most commonly associated with:
A) Hypertrophic cardiomyopathy
B) Coronary artery disease
C) Congenital long QT syndrome
D) Long QT syndrome secondary to various medications
E) arrhythymogenic RV cardiomyopathy
ANSWER: B
Target organs in hypertension include all of the following EXCEPT
A) Brain and eyes
B) Heart
C) Kidneys
D) Peripheral arteries
E) Liver
ANSWER: E
The amount of protein in urine in case of the urinary syndrome:
A) up to 1 g / day
B) up to 3.5 g / day
C) up to 4.5 g / day
D) up to 5.5 g / day
E) up to 9.5 g / day
ANSWER: A
The bad prognostic indicator for scleroderma is:
A) Renal involvement
B) Goose skin appearance
C) Oedema legs
D) Widening of pulse pressure
E) Calcification
ANSWER: A
The best medication for the treatment of isolated systolic hypertension in the elderly:
A) ACE-inhibitor and diuretic
B) Dihydropiridine calcium-channel blocker and diuretic
C) Beta-blocker and diuretic
D) Non-dihydropiridine calcium-channel blocker and diuretic
E) Beta- and alpha blocker
ANSWER: B
The following cardiovascular malformations are all associated with congenital rubella
EXCEPT:
A) PDA
B) PA stenosis
C) Ebstein's anomaly
D) Tetralogy of Fallot
E) Coarctation of the aorta
ANSWER: C
The following medications can be used in pregnancy EXCEPT
A) Methyldopa
B) Labetalol
C) ACE-inhibitor / ARB
D) Hydralazine
E) Nifidipine
ANSWER: C
The following statements regarding the association of oral contraceptive pills and
hypertension are correct EXCEPT:
A) The likelihood of developing hypertension is increased by significant alcohol
consumption
B) The incidence of hypertension is about twice as great in pill users as in non-users
C) The likelihood of developing hypertension is dependent on the age of the user
D) Hypertension resolves in almost all cases after discontinuation of oral
contraceptives
E) The mechanism for contraceptive-induced hypertension likely involves renin-
aldosterone-mediated volume expansion
ANSWER: D
The indications for dialysis in renal failure are all of the following except?
A) Refractory fluid overload
B) Refractory hyperkalemia
C) BUN &amp;amp;gt; 130
D) Pericardial friction rub
E) Hemorragiс stroke
ANSWER: E
The initial antihypertensive medication recommended for patients who have no
compelling indications or contraindications is
A) ACE-inhibitor
B) Calcium-channel blocker
C) Diuretics
D) Beta blocker
E) Any of the above
ANSWER: E
The initial antihypertensive medication recommended for patients who have no
compelling indications or contraindications is
A) ACE-inhibitor
B) Calcium-channel blocker
C) Diuretics
D) Beta blocker
E) Any of the above
ANSWER: E
The initial antihypertensive medication recommended for patients who have no
compelling indications or contraindications is
A) ACE-inhibitor
B) Calcium-channel blocker
C) Diuretics
D) Beta blocker
E) Any of the above
ANSWER: E
The initial manifestations of WPW syndrome may include which of the following?
A) Atrial fibrillation
B) AV re-entry tachycardia
C) Ventricular fibrillation
D) Wide-QRS tachycardia
E) All of the above
ANSWER: E
The lesions that constitute tetralogy of Fallot include all of the following EXCEPT
A) A ventricular septal defect
B) An overriding aorta
C) An atrial septal defect
D) Right ventricular outflow obstruction
E) Right ventricular hypertrophy
ANSWER: C
The main clinical signs of tubular damage are all of the following, except:
A) Acidosis
B) Glycosuria
C) Hypo-, isosthenuria
D) arterial hypertension;
E) Cylindruria
ANSWER: D
The main diagnostic criterion of multiple myeloma is
A) Lytic bone marrow lesions
B) Decreased B2 microglobulin
C) Bence Jones proteinuria
D) Plasmacytosis &amp;amp;gt; 10%
E) Rouleaux formation on peripheral smear
ANSWER: D
The mechanisms of proteinuria are all of the following except?
A) Overflow of elevated normal or abnormal serum proteins
B) Decreased reabsorption of normal filtered proteins
C) Increased glomerular permeability
D) Changes in renal hemodyndamics
E) Hypercoagubility
ANSWER: E
The most common findings at presentation of RPGN are all of the following except?
A) Oliguria
B) Hypertension
C) Edema
D) Active sediment with erythrocytes and casts
E) Isolated mild proteinurea
ANSWER: E
The most common organism in acute bacterial pyelonephritis is
A) Klebsiella
B) Chlamydia
C) E. Coli
D) Pseudomonas
E) Candida
ANSWER: C
The most common type of arrhythmia in WPW syndrome is....
A) Ventricular tachycardia
B) Ventricular fibrillation
C) Ventricular premature complexes
D) Atrial ectopics
E) AV re-entry tachycardia
ANSWER: E
The nephrotic syndrome is characterized by all of following except:
A) А heavy proteinuria;
B) hypoproteinaemia;
C) light proteinuria;
D) hyperproteinaemia;
E) generalized oedema.
ANSWER: D
The proportion of essential (primary) hypertension among all hypertension causes is as
high as
A) 25-30%
B) 40-45%
C) 60-65%
D) 70-75%
E) 90-95%
ANSWER: E
The proportion of essential hypertension among all hypertension causes is as high as
A) 25-30%
B) 40-45%
C) 60-65%
D) 70-75%
E) 90-95%
ANSWER: E
The timing of an "innocent" murmur is usually:
A) Early systolic
B) Presystolic
C) Midsystolic
D) Holosystolic
E) Early diastolic
ANSWER: C
The total daily loss of iron amounts to about:
A) 2 mg
B) 0.5 mg
C) 1 mg
D) 10 mg
E) 0.05 mg
ANSWER: C
Thiazide diuretics can contribute to each of the following metabolic effects EXCEPT:
A) Hypomagnesemia
B) Hypouricemia
C) Hypercalcemia
D) Hypercholesterolemia
E) Hyponatremia
ANSWER: B
Thiazide diuretics can contribute to each of the following metabolic effects EXCEPT:
A) Hypomagnesemia
B) Hypouricemia
C) Hypercalcemia
D) Hypercholesterolemia
E) Hyponatremia
ANSWER: B
This 43 year-old female had a history- old female had a history of shortness of breath
and palpitations for 7 years with a progressive increase one month before death. She
was known to have both systolic and diastolic murmurs, dilated chambers. Is the
disease pericardial, myocardial or endocardial
A) Endocardial
B) Pericardial
C) Myocardial
D) Pericardial, myocardial and endocardial
E) Pericardial and myocardial
ANSWER: C
This 43 year-old female had a history: old female had a history of shortness of breath
and palpitations for 7 years with a progressive increase one month before death. She
was known to have both systolic and diastolic murmus, dilated chambers. Choose the
most likely diagnosis from the below list of options.
A) Dilated cardiomyopathy
B) Infective endocarditis
C) Degenerative valvular disease
D) Connective tissue disease
E) Congenital valvular disease
ANSWER: A
Urinary examination according to Zimnitsky is needed for evaluation of all of
following except:
A) the common diuresis;
B) daily urine;
C) renal clearance;
D) night urine;
E) fluctuation of the specific gravity of urine in every portion.
ANSWER: C
Urinary tract infection has all of the following clues to diagnosis except:
A) Bacteriuria more 105 colonies per millilitre;
B) Pyuria;
C) Bladder tenderness;
D) signs of renal osteodystrophy
E) urinary frequency.
ANSWER: D
Use of echocardiogram in the assessment of AF:
A) It is not necessary to perform a TTE in patients with AF
B) TEE is always needed to allow better assessment of the MV and LA
C) TTE should be performed at baseline to diagnose associated structural/functional
heart disease
D) TTE is necessary to determine stroke risk stratification for thromboprophylaxis in
all patients
E) When TTE is technically difficult, MRI is needed to exclude cardiac abnormalities
ANSWER: C
Vegetations on undersurface of A.V. valves are found in:
A) Acute Rheumatic corditis
B) Limban Sack's endocarditis
C) Non thrombotic bacterial endocarditis
D) Chronic rheumatic carditis
E) MVP
ANSWER: B
Vegetations on undersurface of A.V. valves are found in:
A) Limban Sack's endocarditis
B) Acute Rheumatic corditis
C) Non thrombotic bacterial endocarditis
D) Chronic rheumatic carditis
E) MVP
ANSWER: A
Ventricular ectopic beats (VEBs) occurring in apparently normal hearts are:
A) Associated with a bad prognosis
B) Never frequent or complex
C) Less frequent with increasing age
D) May be associated with symptoms of “missed beats”
E) Always symptomatic
ANSWER: D
Wenckebach phenomenon is defined as...
A) Progressive shortening of PR interval till a beat is dropped
B) Progressive lengthening of PR interval till a beat is dropped
C) Irregular heart rate and PVC’s
D) Shortened QT intervals
E) Slurred QRS complex
ANSWER: B
What amount of proteinuria per day is indicative of nephrotic syndrome?
A) Proteinuria of more than 3.5 grams/day
B) Proteinuria of more than 2.5 grams/day
C) Proteinuria of more than 1.5 grams/day
D) Proteinuria of more than 3.0 grams/day
E) Proteinuria of more than 2.0 grams/day
ANSWER: A
What are 6 common findings in autosomal dominant polycystic kidney disease?
A) Flank or back pain, gross hematuria, abdominal mass, frequent UTI's, hypertension,
and nephrolithiasis
B) Flank or back pain, gross hematuria, small kidneys, frequent UTI's, hypertension,
and nephrolithiasis
C) Flank or back pain, gross hematuria, abdominal mass, nephritic syndrome,
hypertension, and nephrolithiasis
D) Flank or back pain, gross hematuria, abdominal mass, frequent UTI's, hypotension,
and nephrolithiasis
E) None of them
ANSWER: A
What are the chief laboratory findings in patients with chronic lymphatic leukemia?
A) Thrombocytosis
B) Thrombocytosis and lymphocytosis
C) Lymphocytosis
D) No specific findings
E) Philadelphia chromosome in abnormal cells
ANSWER: C
What are the common findings of IgA nephropathy?
A) Heavy proteinurea
B) Asymptomatic microscopic hematuria/proteinuria
C) White cells;
D) Isolated proteinurea
E) pyuria;
ANSWER: B
What are the findings of nephritic syndrome on urinanalysis?
A) RBC casts and hematuria
B) WBS casts and pyuria
C) Isolated proteinuria
D) Hematuria and pyuria
E) No pathological findings
ANSWER: A
What changes in the urine are typical for chronic pyelonephritis:
A) Hematuria, cylindruria
B) Pyuria, cylindruria, proteinuria
C) Cylindruria
D) Hematuria, bakteriyuriya
E) Eritrotsituriya, cylindruria, proteinuria
ANSWER: B
What changes of blood electrolytes are typical for nephrotic syndrome?
A) Increasing of potassium
B) Reduction of potassium
C) Increasing of sodium
D) Reduced sodium
E) Increased phosphorus
ANSWER: C
What does “kidney hyperplasia” mean?
A) Congenital decrease of kidney sizes
B) Congenital increase of kidney sizes
C) Congenital cystic transformation of kidneys
D) Congenital tubular disorders of kidneys
E) Congenital glomerular disorders of kidneys
ANSWER: A
What genetic sign indicates on chronic myelogenous leukemia
A) No sings
B) XXX or XXY combination
C) Philadelphia chromosome in abnormal cells
D) Chromosomes in the 19-st pare
E) Lack of chromosome 13 pare
ANSWER: C
What investigation can detect vegetations in approximately 90% of patients with proven
endocarditis:
A) Transoesophageal echocardiogram
B) Echocardiogram
C) ECG
D) Chest X-ray
E) CT
ANSWER: A
What is specific for rheumatic fever?
A) Subcutaneous nodule
B) Erythema marginatum
C) Osler's nodes
D) Erythema nodosum
E) Janeways lesions
ANSWER: A
What is the anuria?
A) anuria is less than 500 ml urine per 24 hours
B) anuria is less than 400 ml urine per 24 hours
C) anuria is less than 300 ml urine per 24 hours
D) anuria is less than 200 ml urine per 24 hours
E) anuria is less than 100 ml urine per 24 hours
ANSWER: E
What is the best test to evaluate the patients risk for diabetic nephropathy and
cardiovascular disease?
A) Microalbumin-to-creatinine ratio
B) Serum protein electrophoresis
C) Urine amino acid levels
D) 24-hour urine total protein
E) Annual urine dipstick analysis
ANSWER: A
What is the cause of hypoproteinemia in chronic glomerulonephritis?
A) Hyperlipidemia
B) Arterial hypertension
C) Proteinuria
D) Oedema
E) Abnormal liver function
ANSWER: C
What is the correct feature of the hypertensive urgency?
A) It is always a life-threatening situation
B) Patient must be hospitalized
C) Patients can be managed as outpatients
D) Therapy should not be given orally
E) None of the above
ANSWER: C
What is the hematuria?
A) Greater than 3-5 red cells per high power field
B) Greater than 0-1 red cells per high power field
C) Greater than 1-2 red cells per high power field
D) Greater than 2-3 red cells per high power field
E) Greater than 1-4 red cells per high power field
ANSWER: A
What is the most common coexisting congenital anomaly in patients with coarctation of
the aorta?
A) Cleft mitral valve
B) Bicuspid aortic valve
C) Ebstein's anomaly
D) VSD
E) PDA
ANSWER: B
What is the most common coexisting congenital anomaly in patients with coarctation of
the aorta?
A) Cleft mitral valve
B) Bicuspid aortic valve
C) Ebstein's anomaly
D) VSD
E) PDA
ANSWER: B
What is the most common etiology of nephrotic syndrome in African Americans?
A) Post streptococcal
B) IgA nephropathy
C) Rapidly progressive glomerulonephritis
D) Crescentric glomerulonephritis
E) Focal segmental glomerulosclerosis
ANSWER: E
What is the most common form of idiopathic glomerulonephritis?
A) IgA nephropathy
B) minimal change disease
C) Proliferative glomerulonephritis
D) Membranous nephropathe
E) Mesangioproliferative glomerulonephritis
ANSWER: A
What is the most likely diagnosis in a patient with positive Chvostek's and/or
Trousseau's sign and leg cramps?
A) Hypocalcemia
B) Hypopotassemia
C) Hypophosphatemia
D) Hypermagnesiemia
E) Hypercalcemia
ANSWER: A
What is the oliguria?
A) Oliguria is &amp;amp;lt;600 ml of urine per 24
B) Oliguria is &amp;amp;lt;400 ml of urine per 24
C) Oliguria is &amp;amp;lt;800 ml of urine per 24 hours
D) Oliguria is is less than 100 ml urine per 24 hours
E) Oliguria is &amp;amp;lt;500 ml of urine per 24
ANSWER: B
What is the primary cause of volume overload in end stage renal disease?
A) because of inablility of kidney to excrete sodium.
B) because of inablility of kidney to excrete potassium
C) because of inablility of kidney to excrete Magnesium
D) because of inablility of secondary hyperaldosteronism
E) none of them
ANSWER: A
What microorganism is responsible for development of acute glomerulonephritis?
A) Colon bacillus
B) Klebsiella
C) Beta hemolytic streptococcus
D) Aurococcus
E) Proteus vulgaris
ANSWER: C
What proteins are mainly lost in patients sufferring from nephrotic syndrome?
A) Albumins
B) Gama-globulin
C) Alpha-1-globulins
D) Alpha-2 globulins
E) Beta-globulin
ANSWER: A
What substance indicates the disturbed excretory function of kidneys best of all:
A) Creatinine
B) Urea
C) Uric acid
D) Potassium
E) Sodium
ANSWER: A
What substances are reabsorbed in renal tubules completely?
A) Aminoacids
B) Creatinine
C) Sodium
D) Potassium
E) Chlorum
ANSWER: A
What substances are reabsorbed in renal tubules completely?
A) Glucose
B) Creatinine
C) Sodium
D) Potassium
E) Chlorum
ANSWER: A
What underlying diagnosis is suggested in a patient with acute renal failure and white
blood cells or white blood cell casts?
A) Interstitial nephritis
B) Glomerulonephritis
C) Pyelonephritis
D) Renal stones
E) Amyloidosis
ANSWER: A
What will you do if you are on duty in a hospital and a hypertensive patient, who has
been treated with long-acting ?-blocker, long-acting dihydropyridine calcium
antagonist and long-acting a-1 adrenoceptor blocker, is admitted with the following
symptoms: headache, dyspnea, ankle-swelling, BP 180/110 mmHg, pulmonary rales, LVH on
ECG, echo: LVH, left ventricular diastolic dysfunction, ejection fraction 46%.
A) Amlodipine orally
B) Nifedipine orally
C) Enalaprilate iv
D) Enalaprilate iv + furosemide iv
E) Esmolol iv
ANSWER: D
When present, each of the following heart sounds occurs shortly after S2 EXCEPT:
A) Opening snap
B) Third heart sound
C) Ejection clic
D) Tumor plop
E) Pericardial knock
ANSWER: C
Which analgesics are associated with analgesic nephropathy?
A) Phenacitin and acetaminophen
B) Aspirin
C) Ibuprofen
D) Diclofenac
E) All of them
ANSWER: A
Which BP profile below identifies the patient with the highest risk for development of
cardiovascular complications?
A) 160/90 mmHg
B) 160/65 mmHg
C) 140/100 mmHg
D) 130/90 mmHg
E) 120/70 mmHg
ANSWER: B
Which cardiac valve is most frequently affected as a result of rheumatic fever?
A) Mitral
B) Aortic
C) Tricuspid
D) Pulmonic
E) Mitral and Pulmonic
ANSWER: A
Which clinical sign is not typical for acute pyelonephritis?
A) Raising the temperature to febrile digits with a fever
B) Backache
C) dysuric symptom
D) The presence of hypertension
E) There is no right answer
ANSWER: D
Which drugs should not be used in pregnancy?
A) Fluoroquinolones;
B) Phospomycine;
C) Aminoglucosides;
D) macrocrystalline nitrofurantoin;
E) amoxicillin
ANSWER: A
Which is the preferred initial treatment strategy for rate control in AF patients?
A) Digoxin
B) Beta-blockers or rate limiting CCB
C) Combination therapy of beta-blockers and rate-limiting CCB’s
D) Amiodarone
E) Combination therapy of digoxin and beta-blockers
ANSWER: B
Which lifestyle change has been shown to produce the biggest reduction in systolic
blood pressure?
A) 10-kg weight loss
B) Dietary sodium reduction
C) Moderation of alcohol consumption
D) Change to a vegetarian diet
E) Magnesium supplement
ANSWER: A
Which macroscopic changes in urine can take place during acute glomerulonephritis?
A) Yellow urine
B) Urine colour "meat slops"
C) Clear urine
D) Bright red urine
E) Pale yellow, opacity
ANSWER: B
Which of the following ACE-inhibitors are not prodrugs?
A) Captopril, lisinopril, ramipril
B) Lisinopril, enalapril, banazepril
C) Captopril, lisinopril, enalaprilat
D) Moexipril, captopril, lisinopril
E) Perindopril, ramipril, enalapril
ANSWER: C
Which of the following antihypertensive agents is a known cause of autoimmune
hemolytic anemia?
A) Metoprolol
B) Methyldopa
C) Captopril
D) Losartan
E) Monoxidine
ANSWER: B
Which of the following antihypertensive agents is a known cause of autoimmune
hemolytic anemia?
A) Metoprolol
B) Methyldopa
C) Captopril
D) Losartan
E) Monoxidine
ANSWER: B
Which of the following antihypertensive agents is INCORRECTLY matched with the
indication for therapy?
A) ACE inhibitor – diabetic nephropathy
B) Beta-blocker – coronary artery disease
C) Calcium channel blocker – angina pectoris
D) Hydrochlorthiazide – diabetes mellitus
E) Loop diuretic – heart failure
ANSWER: D
Which of the following antihypertensive agents is INCORRECTLY matched with the
indication for therapy?
A) ACE inhibitor – diabetic nephropathy
B) Beta-blocker – coronary artery disease
C) Calcium channel blocker – angina pectoris
D) Diuretics – heart failure
E) Loop diuretic – gout
ANSWER: E
Which of the following antimicrobial therapies would be inappropriate and potentially
harmful in a patient receiving cyclosporine?
A) Amoxicillin-clavulanate
B) Erythromycin
C) Cefuroxime axetil
D) Ciprofloxacin
E) Penicillin
ANSWER: B
Which of the following APhS symptoms predictably respond(s) to treatment with heparin?
A) Livedo reticularis
B) Cardiac valve disease
C) Recurrent fetal loss
D) Cognitive dysfunction
E) Renal microangiopathy
ANSWER: C
Which of the following autoantibodies is more likely to be found in patients with
systemic sclerosis?
A) anti-Smith (anti-Sm)
B) Anti-ribonucleoprotein (RNP)
C) Anti-DNA topoisomerase 1 (anti-Scl70)
D) Anti-citrullinated peptide
E) Anticentromere
ANSWER: C
Which of the following blood pressure levels in pregnancy should be considered an
emergency requiring hospitalization?
A) ?140/90 mmHg
B) ?150/95 mmHg
C) ?160/100 mmHg
D) ?170/110 mmHg
E) A rise in SBP ?25 mmHg or a rise in DPB ?15 mmHg compared to pre-pregnancy values,
or those in the first trimester
ANSWER: D
Which of the following clinical features is NOT commonly found in limited systemic
sclerosis?
A) Raynaud's phenomenon
B) Oesophageal dysfunction
C) Eczema
D) Sclerodactyly
E) Telangiectasia
ANSWER: C
Which of the following diagnostic studies is most likely to demonstrate the cause of
the headaches?
A) MRI of the head
B) MRI of the kidney
C) MRI of the thorax
D) 24-h urinary 5-HIAA
E) 24-h urinary free cortisol
ANSWER: C
Which of the following doesn’t occur in Diffuse cutaneous scleroderma
A) Vegetations on undersurface of A.V. valves
B) Raynaud phenomenon
C) Truncal and acral skin involvement
D) Nailfold capillary dilation
E) interstitial lung
ANSWER: A
Which of the following has been shown to decrease the level of the high-sensitivity
CRP?
A) Unopposed estrogen
B) Amlodipine besylate (Norvasc)
C) Simvastatin
D) All of the above
E) None of the above
ANSWER: C
Which of the following is a typical immune complex disease?
A) Osler-Weber-Rendu syndrome
B) Urticaria pigmentosa
C) Lupus erythematosus
D) Gastric ulcer
E) Pneumonia
ANSWER: C
Which of the following is an absolute contraindication to pregnancy?
A) Surgically corrected transposition of great arteries (TGA)
B) Congenitally corrected TGA
C) Ebstein's anomaly
D) Eisenmenger's syndrome
E) Status post Fontan operation
ANSWER: D
Which of the following is an absolute contraindication to pregnancy?
A) Surgically corrected transposition of great arteries (TGA)
B) Congenitally corrected TGA
C) Ebstein's anomaly
D) Eisenmenger's syndrome
E) Status post Fontan operation
ANSWER: D
Which of the following is an effective adjunct to increase the rate of smoking
cessation?
A) Aldosterone
B) Bupropion
C) Sildenafil
D) Cimetidine
E) Buspirone
ANSWER: B
Which of the following is an initiation for institution of therapy with
hydroxychloroquine in a patient with systemic lupus erythematosus (SLE)?
A) Thrombocytopenia
B) Subacute cutaneous lupus
C) Central nervous system vasculitis
D) Diffuse proliferative glomerulonephritis (DPGN)
E) Gangrene of digits
ANSWER: B
Which of the following is not one of the major Jones criteria for the diagnosis of
rheumatic fever?
A) Urethritis
B) Carditis
C) Arthritis
D) Chorea
E) Erythema marginatum
ANSWER: A
Which of the following is one of the earliest predictors of whether a patient with
Raynaud’s phenomenon will go on to develop true scleroderma?
A) Constipation
B) Decreased vital capacity
C) Thickening of the forearm skin
D) Dilated and dropout capillaries at the nail base of fingers
E) None of the above
ANSWER: D
Which of the following is the most common late manifestation of acute rheumatic fever?
A) Chorea
B) Pharyngitis
C) Erythema marginatum
D) Iritis
E) Enthesopathy
ANSWER: A
Which of the following occurs rarely in Limited cutaneous scleroderma
A) Skin involvement
B) Raynaud phenomenon
C) Dilated nailfold capillary loops
D) Renal disease
E) Anticentromere antibody
ANSWER: D
Which of the following pairs of medical conditions and antihypertensive medications
would be incorrect to use in a patient with essential hypertension?
A) Beta-blocker and a history of myocardial infarction
B) Alpha-blocker and prostatic hypertrophy
C) Thiazide diuretic and gout
D) Amlodipine and heart failure
E) ACE-inhibitor and diabetes mellitus
ANSWER: C
Which of the following statements about microalbuminuria is TRUE?
A) To be of clinixal value, microalbuminuria must be measured in a timed 12- to 24-
hour sample
B) Microalbuminuria is a cardiovascular risk factor that is independent of traditional
Framingham risk factors
C) Microalbuminuria is a predictor of risk only in patients with diabetes
D) Microalbuminuria is present when the “spot” urine albumin-to-creatinine ration is
&amp;amp;gt;500 mg/g
E) All of the above statements are correct
ANSWER: B
Which of the following statements about microalbuminuria is TRUE?
A) To be of clinixal value, microalbuminuria must be measured in a timed 12- to 24-
hour sample
B) Microalbuminuria is a cardiovascular risk factor that is independent of traditional
Framingham risk factors
C) Microalbuminuria is a predictor of risk only in patients with diabetes
D) Microalbuminuria is present when the “spot” urine albumin-to-creatinine ration is
&amp;amp;gt;500 mg/g
E) All of the above statements are correct
ANSWER: B
Which of the following statements best characterized tricuspid regurgitation?
A) Infective endocarditis is a common cause
B) The murmur heard in tricuspid regurgitation decreases with inspiration
C) Large a waves in jugular venous pulse are characteristic
D) Correction of coexistent left ventricular failure rarely improves the condition
E) Cardiac catheterization is required for the diagnosis
ANSWER: A
Which of the following statements regarding antihypertensive agents and atrial
fibrillation (AF) is TRUE?
A) Losartan has been shown to decrease new-onset AF more effectively than atenolol
B) Valsartan has been shown to decrease new-onset AF more effectively than amlodipine
C) Atenolol has been shown to decrease new-onset AF more effectively than captopril
D) Nifedipine has been shown to decrease new-onset AF more effectively than diltiazem
E) All of the above are false
ANSWER: A
Which of the following statements regarding prevention of stroke is correct?
A) Treatment of systolic hypertension does not decrease the risk of stroke in patients
older than age 60
B) Hypertension should not be a target of secondary prevention after an ischemic
stroke because elevated BP is desirable to maintain adequate cerebral perfusion
C) Treatment with HMG-CoA reductase inhibitors reduces the risk of recurrent stroke
D) The combination of aspirin and clopidogrel is superior to aspirin alone for
prevention of recurrent stroke
E) None of the above
ANSWER: C
Which of the following statements regarding prevention of stroke is correct?
A) Treatment of systolic hypertension does not decrease the risk of stroke in patients
older than age 60
B) Hypertension should not be a target of secondary prevention after an ischemic
stroke because elevated BP is desirable to maintain adequate cerebral perfusion
C) Treatment with HMG-CoA reductase inhibitors reduces the risk of recurrent stroke
D) The combination of aspirin and clopidogrel is superior to aspirin alone for
prevention of recurrent stroke
E) None of the above
ANSWER: C
Which of the following statements regarding the use of cyclophosphamide in the
treatment of rheumatic disorders such as lupus nephritis and vasculitis is correct?
A) The dose of cyclophosphamide should be increased until substantial leukopenia is
observed.
B) The absorption of oral cyclophosphamide is poor, so the intravenous route is
preferred.
C) Oral cyclophosphamide should be administered at night and fluids restricted in
order to decrease the formation of urine.
D) Regular monitoring of the patient’s complete blood cell count and urinalysis is
required.
E) None of the above
ANSWER: D
Which of the following symptoms are not typical for nephrotic syndrome?
A) massive proteinuria
B) Edema
C) Gross hematuria
D) Hypercholesterolemia
E) Bacteriuria
ANSWER: E
Which of the following syndromes is associated with pumonary arterio-venous fistula?
A) Williams syndrome
B) Weber-Osler-Rendu syndrome
C) Bland-Garland-White syndrome
D) Kartagener's syndrome
E) Crouzon's syndrome
ANSWER: B
Which of the following will most likely respond to verapamil?
A) RVOT tachycardia
B) Idiopathic LV tachycardia
C) Bundle branch re-entry tachycardia
D) ARVD ventricular tachycardia
E) All of the above
ANSWER: B
Which of these statements is true regarding renin-angiotensin system-blocking agents?
A) Less effective when combined with a diuretic than when used alone
B) More effective in patients of African descent than in white patients
C) Preserve kidney function in addition to lowering blood pressure
D) Are the first-line antihypertensive medications in pregnancy
E) All of the above
ANSWER: C
Which of these statements is true regarding renin-angiotensin system-blocking agents?
A) Less effective when combined with a diuretic than when used alone
B) More effective in patients of African descent than in white patients
C) Preserve kidney function in addition to lowering blood pressure
D) Are the first-line antihypertensive medications in pregnancy
E) All of the above
ANSWER: C
Which of these statements is true regarding renin-angiotensin system-blocking agents?
A) Less effective when combined with a diuretic than when used alone
B) More effective in patients of African descent than in white patients
C) Preserve kidney function in addition to lowering blood pressure
D) Are the first-line antihypertensive medications in pregnancy
E) All of the above
ANSWER: C
Which one of the following statements about autoimmune diseases is true
A) Affect about 1 person in every 1000
B) Are more common in women
C) Tend to begin in early childhood
D) Are an inevitable consequence of autoimmune responses
E) Are usually mediated by type I hypersensitivity
ANSWER: B
Which one of the following statements about systemic connective tissue diseases is not
true?
A) Can cluster within families
B) Are often associated with particular HLA alleles
C) Are usually mediated by type I hypersensitivity
D) May occur in humans with inherited defects in apoptosis
E) Are more common in women
ANSWER: C
Which option is better in regard to cardioversion?
A) Synchronised DC shock to restore sinus rhythm
B) Amiodarone in all patients to restore sinus rhythm
C) Flecainide in patients with IHD to maintain sinus rhythm
D) Digoxin to restore sinus rhythm
E) Propaphenone to restore sinus rhythm in HCMP patients
ANSWER: A
Which parameter of arterial blood pressure (BP) yields the best prognostic information
in patients at risk for cardiovascular disease?
A) Systolic BP
B) Diastolic BP
C) Mean arterial BP
D) Pulse pressure
E) None
ANSWER: D
White blood cell casts and nonpigmented granular casts suggest;
A) pyelonephritis;
B) vasculitis;
C) thrombotic thrombocytopenic purpura;
D) glomerulonephritis;
E) amyloidosis.
ANSWER: A
Williams syndrome is commonly associated with...
A) Supravalvular aortic stenosis
B) Supravalvular pulmonic stenosis
C) Cleft mitral valve
D) Anomalous pulmonary venous drainage
E) Persistent left superior vena cava
ANSWER: B
Williams syndrome is commonly associated with...
A) Supravalvular aortic stenosis
B) Supravalvular pulmonic stenosis
C) Cleft mitral valve
D) Anomalous pulmonary venous drainage
E) Persistent left superior vena cava
ANSWER: B
With respect to renovascular disease, all of the statements are true EXCEPT:
A) Worsening renal function with ACE-inhibitor therapy suggests bilateral renovascular
disease
B) Atherosclerotic disease most commonly involves the proximal third of the main renal
artery
C) The most common form of fibroplastic renovascular disease in adults involves the
media
D) The incidence of renovascular hypertension falls with increased age
E) Patients with severe, accelerated hypertension have the highest prevalence of
renovascular disease
ANSWER: D
With which of the following is sildenafil acetate (Viagra) most likely to interact
adversely?
A) Fibrinolytic therapy
B) Primary PCI
C) Nitrates
D) Aspirin
E) Beta-blockers
ANSWER: C
Woman of 35 years with Raynauds phenomenon, polyarthritis, dysphagia of 5 years and
mild sclerodactyly, blood showing Anticentromere antibody positive, the likely cause
is:
A) Scleroderma
B) Mixed connective tissue disorder
C) SLE
D) Rheumatoid arthritis
E) Polymyalgia rheumatica
ANSWER: A
You see a diabetic patient presents with BP readings that are 155/95 or higher. All of
the following statements about the treatment of this patient's hypertension are
correct EXCEPT:
A) Pharmacologic blocade of the renin-angiotensin system reduces the risk of both
microvascular and macrovascular events
B) Aggressive BP control reduces cardiovascular events more in diabetics than in
nondiabetics
C) Calcium channel blockers show no benefit in reducing cardiovascular events
D) The goal BP for this patient is &amp;amp;lt;130/80 mmHg
E) All the above statements are correct
ANSWER: C
35 years-old patient is treated in the intensive care department after trauma with
development of protracted compression syndrome. Objectively: respiration rate – 30/min,
heart rate 100/min, central venous pressure - 159. On ECG - ventricular extrasystole,
high waves T. In labs: serum potassium – 7,2 mmol/l, serum creatinine 900 umol/l. What
complication has developed?
A) Paroxismal tachicardia
B) Acute respiratoru distress syndrome
C) Acute renal failure
D) Acute heart failure
E) Acute myocardial infarction
ANSWER: C
61 year old man presents to his internist with jaundice. The patient is receiving no
medication and his only symptomatic complaints is mild fatigue over the past 2 months.
Physical examination is remarkable only for the presence of scleral icterus. The
patient has no significant past medical history. Analysis of serum chemistry reveals
the following: AST 0.58, ALT- 0.58. Total bilirubin : 81.7 mcmol/L Direct Bilirubin :
46 mcmol/L Alkaline phosphatase: 2,2. Which of the following is the next appropriate
diagnostic step?
A) Ultrasound
B) Liver biopsy
C) Total blood count
D) Coprogram
E) Diastase of urine
ANSWER: A
65-year-old, women. She has been under treatment for hypertension and is finally
stable on medication. Initial Chief complaint: painful left knee, arthritis in left
hip. She had had arthritic pain in her right hip confirmed by x-ray about 10 years ago.
The knee pain is a newer problem and she suspects it is also arthritis. There is
slight crepitus with motion of the right knee. Which of the following changes will
reveal X-ray examination?
A) Erosions
B) Subluxations
C) Osteophytes
D) Osteoporosis
E) Ankylosis
ANSWER: C
70-year-old, married, teacher. She has been under treatment for hypertension and is
finally stable on medication. Initial Chief complaint: painful right knee, arthritis
in right hip. She had had arthritic pain in her right hip confirmed by x-ray about 10
years ago. The knee pain is a newer problem and she suspects it is also arthritis.
There is slight crepitus with motion of the right knee. Which of the following
treatment do use?
A) Methotrexat
B) Colchicines
C) Chondroitin sulfate
D) Tetracycline
E) Allopurinol
ANSWER: C
A 12 year old girl has history of recurrent bulky stoolls and abdominal pain since 3
year of age. She has moderate pallor and lose of weight. Which of the following is the
most appropriate investigation to make a specific diagnosis?
A) Small intestinal biopsy
B) Barium studies
C) 24-hr- fecal fat estimation
D) Urinary d-xylose test
E) None of above
ANSWER: A
A 17-year-old student presents with a 2-week history of periorbital and lower
extremity swelling that developed over 3 to 4 days. The patient had been well until
this point. He takes no medications and has no history of recreational drug use. On
examination, blood pressure is 110/70 mm Hg, pulse rate is 92/min, and temperature is
37 °C (98.6 °F). Skin examination is normal. He has marked periorbital edema. No
jugulovenous distention is present. The lungs are clear, and the heart is in regular
sinus rhythm, with no murmur or gallop. The abdomen is nontender, and the liver and
spleen are not palpable. Scrotal edema is present, as is 4+ edema of the legs and
thighs. Laboratory studies: Complete blood count is Normal. Blood urea nitrogen 25
mg/dL. Serum creatinine 1.2 mg/dL. Serum total protein 65 g/L. Serum albumin 33 g/L
Urine protein: creatinine ratio 18. Urinalysis pH 6.0; specific gravity 1.020; 4+
proteinuria, trace hematuria; many hyaline, granular, and fatty casts. Renal biopsy
shows 15 glomeruli. Light microscopy is normal. Immunofluorescence shows nonspecific
staining for C3. Electron microscopy shows fusion of podocyte foot processes. What is
the morphological renal diagnosis in this patient?
A) Membranous nephropathy
B) Minimal change disease
C) Membranoproliferative glomerulonephritis
D) Focal and segmental glomerulonephritis
E) Alports disease
ANSWER: B
A 20 yrs adult presents with severe aplastic anemia What is most effective treatment:
A) a-interferon
B) IL-2
C) ATG therapy
D) Bone marrow transplantation
E) Chloramphenicol
ANSWER: D
A 21-yr-old female presents with elevated serum T4 and low radioactive iodine uptake.
Put diagnosis, please.
A) Non-toxic goiter
B) Hashimoto's thyroiditis
C) Subacute thyroiditis
D) Hypothyroidism
E) Graves' disease
ANSWER: C
A 22-year-old pregnant is diagnosed with Graves' disease shortly. Appropriate therapy
includes:
A) Propylthiouracil therapy with the goal of maintaining her thyroid function tests in
the highnormal
B) Radioactive iodine to ablate her thyroid gland or slightly high range
C) Methimazole therapy
D) A beta blocker
E) Propylthiouracil therapy with care taken to maintain her thyroid function tests in
the midnormal range
ANSWER: A
A 24-year-old graduate student has a several-week history of aching pains in knees,
ankles, and elbows and intermittent abdominal pain. Three days ago, he had two
episodes of painless hematuria. He has no fever, chills, or weight loss. There is no
history of hair loss, but he has had an intermittent, nonpruritic rash located below
the knees bilaterally. There has been no recent upper respiratory infection. He takes
acetaminophen occasionally for the aching joints. He has no history of kidney disease
or intravenous drug use. On examination, the patient appears well. Weight is 84.4 kg
(186 Ib). Blood pressure is 120/84 mm Hg, pulse rate is 70/min, and temperature is
37.2 °C (99 °F). No jugulovenous distention is present. The lungs are clear, and
cardiac examination shows regular sinus rhythm with no murmur. On abdominal
examination, the liver and spleen are not palpable, and no mass or tenderness is
present. Pulses in the extremities are normal and there is no edema, but the patient
has a petechial macular papular rash over the lower extremities. Laboratory studies:
Leukocyte count 8,4x109/L. Hemoglobin - 141 g/L. Hematocrit - 41 %. Blood urea
nitrogen - 12 mg/dL. Serum creatinine 1.1 mg/dL. Serum sodium - 138 mmol/L. Serum
chloride - 104 mmol/L. Serum potassium - 4.0 mmol/L. Serum bicarbonate - 26 mmol/L.
Serum antinuclear antibody Negative. Serum C3 - 85 mg/dL. Serum C4 - 21 mg/dL. Urine
protein :creatinine ratio0.2. Urinalysis Specific gravity 1.030, pH 5.0, 1 +
proteinuria, 4+ hematuria Urine microscopy reveals many dysmorphic erythrocytes and
erythrocyte casts. The lung fields are clear on chest radiography. Biopsy of skin
lesion shows IgA deposition and leukocytoclastic change. What is the most likely cause
of renal disease in this patient?
A) Lupus nephritis
B) Acute post—streptrococcal glomerulonephritis
C) Henoch-Schonlein purpura
D) Allergic interstitial nephritis
E) Goodpastures syndrome
ANSWER: C
A 25 yrs women presents with bloody diarrhea & is diagnosed as a case of Ulcerative
colitis. Which of the following condition is not associated:
A) Pancreatitis
B) Iritis
C) Ankylosing spondylitis
D) Sclerosing cholengitis
E) None of above
ANSWER: A
A 26-year-old woman was brought to the office because of a "red rash" that she noticed
today. She says that 3 days ago he had a cough, runny nose, and fever that responded
to ibuprofen. In the office, his temperature is 37 C (98.6 F) and he has a normal
physical examination with the exception of an erythematous, blanching macular rash on
his legs. You’ve diagnosed her with a viral exanthem and advise to drink liquids and
to use ibuprofen as needed for fever. One week later, she came back to the office and
reports that the rash has "changed", she has developed colicky abdominal pain several
times per day, and she is complaining of left knee pain. In the office, her
temperature is 37.2 C (99 F), blood pressure is 100/65 mm Hg, pulse is 100/min, and
respiratory rate is 20/min. A physical examination reveals a well-appearing patient
with palpable purpura of both lower extremities, normal neck examination, clear lungs,
and a soft, non-tender abdomen. Her left knee is painful on flexion, but it is not
erythematous or warm, and there does not seem to be an effusion. His gait is normal.
What affection of kidney could develop in this patient?
A) Acute interstitial nephritis
B) Chronic interstitial nephritis
C) Glomerulonephritis
D) Pyelonephritis
E) She is not in risk of kidney affection
ANSWER: A
A 31-year man presented with nausea, fever and jaundice of 5 days duration. The
biochemical tests revealed a bilirubin of 35 mcmol/L (conjugated 15,9 mcmol/L), AST
0.99, ALT- 1.07. The serological tests showed presence of HBs Ag, IgM anti-HBc and Hbe
Ag. The most likely diagnosis is:
A) Acute hepatitis B infection with high infectivity.
B) Chronic hepatitis B infection with high infectivity.
C) Chronic hepatitis infection with low infectivity.
D) Acute hepatitis B infection with low infectivity
E) None of above
ANSWER: A
A 31-year-old multiparous woman at39weeksof pregnancy is admitted in active labor with
a breech presentation. She has had several episodes of nausea and vomiting in the past
3 days. She has no history of jaundice, alcohol abuse, or renal disease. Her prenatal
care has been sporadic, and she had not seen a physician for 1 month before delivery.
Preoperative laboratory studies are as follows: Hematocrit 36.9% Platelet count
168x109/L Blood urea nitrogen 9 mmol/L. Serum creatinine 170 mmol/L Serum electrolytes
Normal. Serum glucose 3,0 mmol/L. Serum ALT 446 U/L. Serum AST 463 U/L. Serum total
bilirubin 3.5 mg/dL Serum direct bilirubin 2.5 mg/dL. Serum alkaline phosphatase 553
U/L. Serum lipase 83 U/L. Serum amylase 43 U/L. The patient undergoes cesarean section
under general anesthesia. Estimated blood loss is 500 mL. She is given epinephrine,
phenylephrine, meperidine, morphine, and fentanyl preoperatively. No perioperative
hypotension is noted. Postoperative urine output is 1.2 L. Five hours after surgery,
she has vaginal hemorrhage. She undergoes transfusion and is given normal saline.
You’ve called to see the patient because of continued renal insufficiency. On physical
examination, she is in no acute distress. Blood pressure is 112/72 mm Hg, with no
orthostatic changes; pulse rate 140/min; respiratory rate 14/min; temperature 37.0°C
(98.6 °F). She has scleral icterus. There is no neck vein distention or hepatojugular
reflux. The cardiac examination is normal except for tachycardia. The pulmonary
examination is normal. The abdomen is protuberant and soft, and bowel sounds are heard.
Trace bilateral lower extremity edema is present. Postoperative laboratory studies are
as follows: Hematocrit - 27.9%. Leukocyte count – 7,4x109/L. Platelet count – 8,8
x109/L .Peripheral smear No schistocytes. Blood urea nitrogen - 9 mmol/L. Serum
creatinine - 150 mmol/d.L Serum electrolytes – Normal. Serum glucose – 3,8 mmol/L.
Serum ALT - 240 U/L. Serum AST - 210 U/L. Serum total bilirubin - 2.5 mg/dL. Serum
direct bilirubin 1.0 mg/dL. Serum alkaline phosphatase 250 U/L. Serum lipase - 60 U/L.
Serum amylase - 35 U/L. Prothrombin time - 15.2s. International normalized ratio - 1.7.
Partial thromboplastin time - 44 s. d-Dimer is Positive. Plasma fibrinogen - 78 ?g/dL.
Urine creatinine - 45 mg/dL. Urine sodium - 86 meq/L. Urinalysis pH 5; specific
gravity 1.009; 2+ hematuria; no proteinuria, ketonuria or glucosuria. Urine
microscopy :Fine granular casts, rare erythrocytes What is the most likely diagnosis?
A) Preeclampsia
B) Fatty liver of pregnancy
C) Pregnancy-associated microangiopathic hemolytic anemia
D) Acute cortical necrosis
E) Prerenal azotemia
ANSWER: B
A 32-yr-old woman presents with weight gain, constipation, lethargy and flaky rash.
Put diagnosis, please.
A) De Quervain's thyroiditis
B) Hypothyroidism
C) Hashimoto's thyroiditis
D) Graves' disease
E) Follicular carcinoma
ANSWER: B
A 35-year-old man had HIV infection diagnosed 2 months ago. His serum creatinine
concentration was 60 umol/L. Treatment with highly active antiretroviral therapy with
zidovudine, lamivudine, and abacavir was recommended, but he wished to wait before
starting treatment. He is brought to clinic by a friend who states that the patient
has had fever, confusion, and disorientation for 1 day. Physical examination reveals
blood pressure 110/70 mm Hg and pulse rate 100/min that is regular supine and standing.
The chest is clear, without cardiac murmur or gallop, and the abdomen is normal.
Moderate bilateral lower extremity edema is present. Laboratory studies: Hemoglobin -
78 g/L. Leukocyte count - 10,2000/?L. Platelet count - 19,000/?L. Blood urea nitrogen
37 mmol/L. Serum creatinine - 270 mmol/L,. Serum sodium - 136 meq/L Serum potassium
5.2 meq/L. Serum chloride - 99 meq/L. Serum bicarbonate - 22 meq/L. Urinalysis
Specific gravity 1030; 3+ hematuria, trace proteinuria, trace ketonuria, no glucosuria
Urinary microscopic examination shows a few erythrocytes, but no erythrocyte casts.
The lactate dehydrogenase level is elevated. Peripheral blood smear shows many
schistocytes. What is the most likely cause of this patient’s renal failure?
A) Thrombotic thrombocytopenic purpura
B) HIV-associated nephropathy
C) Surreptitious ingestion of antiretroviral drugs
D) Outpatient acute tubular necrosis
E) HIV-associated immune-mediated glomerulonephritis
ANSWER: A
A 38-year-old man with a 13-yr history of back pain, presented with acute pain and
swelling of one knee. On examination, the joint was tender and restricted in movement.
X-ray of the knee showed periarticular osteoporosis. On investigation, he had a raised
erythrocyte sedimentation rate of 32 mm/h, a mild anemia (Hb 106g/l) but no detectable
serum rheumatoid factor. The knee effusion was aspirated; the fluid contained a
polymorphonuclear leucocytosis but no organisms or rheumatoid factor. Fifteen months
later he developed an iritis in his left eye, low back pain and stiffness. His
peripheral joints were normal but pain could be elicited in both sacroiliac joints.
Choose the medications for patient with shown changes from the below list of options.
A) Methotrexate
B) Actovegin
C) Mydocalm
D) Warfarin
E) Pentoxifylline
ANSWER: A
A 38-year-old man with a 13-yr history of back pain, presented with acute pain and
swelling of one knee. On examination, the joint was tender and restricted in movement.
X-ray of the knee showed periarticular osteoporosis. On investigation, he had a raised
erythrocyte sedimentation rate of 102mm/h, a mild anaemia (Hb 106g/l) but no
detectable serum rheumatoid factor. The knee effusion was aspirated; the fluid
contained a polymorphonuclear leucocytosis but no organisms or rheumatoid factor.
Fifteen months later he developed an iritis in his left eye, low back pain and
stiffness. His peripheral joints were normal but pain could be elicited in both
sacroiliac joints. Choose the most likely diagnosis from the below list of options for
patient with shown changes
A) Ankylosing spondylitis
B) Reactive arthritis
C) Osteoarthritis
D) Gout
E) Psoriatic arthritis
ANSWER: A
A 38-year-old man with a 13-yr history of back pain,presented with acute pain and
swelling of one knee. On examination, the joint was tender and restricted in movement.
X-ray of the knee showed periarticular osteoporosis. On investigation, he had a raised
erythrocyte sedimentation rate of 32 mm/h, a mild anemia (Hb 106g/l) but no detectable
serum rheumatoid factor. The knee effusion was aspirated; the fluid contained a
polymorphonuclear leucocytosis but no organisms or rheumatoid factor. Fifteen months
later he developed an iritis in his left eye, low back pain and stiffness. His
peripheral joints were normal but pain could be elicited in both sacroiliac joints.
Which of the following medications is indicated for the treatment of the disease?
A) Sulfasalazine
B) Cyclosporine
C) Diclofenac
D) Pentoxifylline
E) Corticosteroids
ANSWER: A
A 39 year old female, hospitalized after a motor vehicle accident, develops acute
gastric stress ulcers. Increases in which of the following normal physiological
parameters may have contributed to this condition?
A) Pepsin production
B) Epithelial regenerative capacity
C) Mucosal blood flow
D) Mucus secretion
E) Bicarbonate transport
ANSWER: A
A 40 year-old male presents with jaundice. Total bilirubin is 91 mcmol/L, direct is 81
mcmol/L, alkaline phosphatase 3,2. units. Diagnosis is:
A) Obstructive jaundice
B) Viral hepatitis
C) Chronic active hepatitis
D) Hemolytic jaundice
E) Hemolytic anemia
ANSWER: A
A 41 year old male patient presented with recurrent episodes of bloody diarrhea &
mucus for 5 years. Despite regular treatment with adequate doses of sulfasalazine, he
has had several exacerbations of his disease and required several weeks of steroids
for the control for flares. What should be the next line of treatment for him?
A) Azathioprine
B) Methotrexate
C) Cyclosporine
D) Cyclophosphamide
E) None of above
ANSWER: A
A 41 year old patient presented with chronic diarrhoea for 3 months. A d-xylose
absorption test was ordered to look for:
A) Carbohydrate malabsorption due to mucosal disease.
B) Carbohydrate malabsorption due to chronic pancreatitis
C) Fat malabsorption due to mucosal disease.
D) Fat malabsorption due to chronic pancreatitis.
E) None of above
ANSWER: A
A 41 year-old male has jaundice. Blood examination shows total serum bilirubin 129
mcmol/L, direct bilirubin 120 mcmol/L and alkaline phosphatase 2,9. The Diagnosis is :
A) Obstructive jaundice
B) Viral hepatitis
C) Chronic active hepatitis
D) Haemolytic jaundice
E) Hemolytic anemia
ANSWER: A
A 43-yr-old woman presents with a hard, nodular midline neck mass. Blood tests reveal
the presence of antibodies to thyroglobulin. Put diagnosis, please.
A) Thyroglossal cyst
B) Hashimoto's thyroiditis
C) Acute thyroiditis
D) Graves' disease
E) Subacute thyroiditis
ANSWER: B
A 49-year-old man is brought to the emergency department after being found
unresponsive on a city street. His medical history is unknown. The patient is comatose,
with a Glasgow Coma Score of 3. Initial rectal temperature is 32 °C (89.6 °F),
systolic blood pressure 70mm Hg, respiratory rate 6/min, and pulse rate 120/min.
Funduscopy shows no hemorrhage or papilledema. The patient has numerous superficial
lacerations and ecchymoses on his extremities. There is no odor of alcoholic beverages.
The remainder of the physical examination is unremarkable. The patient is emergently
intubated and supported aggressively with intravenous fluids. Shortly thereafter, the
patient’s blood pressure increases to 207/131 mm Hg, requiring intravenous
antihypertensive medication. Urine output is 100 to 200 mL/h. Results of noncontrast
computed tomography of the head and portable chest radiography are normal. Laboratory
studies: Hematocrit41 %. Leukocyte count 32x109/L. Platelet count 422x109/L. Serum
sodium 151 mmol/L. Serum potassium 5.3 mmol/L. Serum chloride 112 mmol/L. Serum
bicarbonate - 5 mmol/L. Blood urea nitrogen 11 mmol/L. Serum creatinine 1.8 mmol/L.
Serum glucose 15,2 mmol/L. Serum lactate 4.3 mmol/L. Serum osmolality 375 mosmol/kg
H2O. Arterial blood gas pH 6.8, PCO2 16 mm Hg, Po2 159 mm Hg, SaO2 99%. Urinalysis pH
5.0, specific gravity 1.012, trace glucosuria, moderate hematuria, trace ketonuria,
proteinuria 100 mg/dL. Urine microscopy Numerous erythrocytes No salicylate,
acetaminophen, or ethanol is detected on toxicology screening. Intravenous infusion of
bicarbonate is begun, and the patient is transferred to the medical intensive care
unit. Repeated arterial blood gas analysis shows a pH of 6.8. What is the next most
appropriate step in the management of this patient?
A) Continue bicarbonate supplementation and add insulin to control blood glucose
B) Initiate ethanol drip and hemodialysis
C) Institute plasma exchange to treat acidemia
D) Initiate ethanol drip and continue bicarbonate and insulin supplementation
E) Perform emergency contrast computed tomography of the abdomen and pelvis
ANSWER: B
A 49-year-old man was well until 1 month ago, when he developed dyspnea, fatigue,
decreased appetite, and cough that produces blood-tinged sputum. Three days ago, he
had three episodes of frank hemoptysis. He has had no fever or chills. He has a 7-year
history of hypertension and has been a smoker for 15 years. He takes no medications.
On examination, the patient is alert and in no distress but is pallid. Blood pressure
is 155/88 mm Hg, pulse rate is 90/min, respiratory rate is 18/min, and temperature is
37 °C (98.6 °F). Skin examination is normal. Optic funduscopy shows mild sclerosis
and constriction. The carotids and thyroid are normal. The pharynx is clear. Pulmonary
examination reveals coarse crackles in the right mid-lung field. Cardiac examination
shows regular sinus rhythm, with no gallop; S1 and S2 are normal. The abdomen is soft,
with no organ enlargement. No redness or swelling of the joints is present. Laboratory
studies: Leukocyte count 9,2x109/L Hemoglobin - 87 g/L Hematocrit - 26%. Platelet
count - 443x109/L. Serum creatinine - 440 umol/L(was 130 mmol/L 3 months ago). Serum
sodium - 142 mmol/L. Serum chloride 103 mmol/L Serum potassium - 4.8 mmol/L. Serum
bicarbonate - 21 mmol/L. Serum total protein - 63 g/L. Serum albumin - 37 g/L. 24-hour
urine protein 3.5 g. Urinalysis 4+ proteinuria, 3+ hemoglobinuria. Urine microscopy
shows 25 to 50 erythrocytes/hpf, 5 to 15 leukocytes/hpf, dysmorphic erythrocytes, and
erythrocyte casts. Kidney biopsy reveals proliferative glomerulonephritis, with 50% of
glomeruli having crescents. Immunofluorescence shows linear staining with IgG. What
diagnostic term best describes the disease presentation in this patient?
A) Rapidly progressive glomerulonephritis
B) Nephrotic syndrome
C) Chronic renal failure
D) Acute tubular necrosis
E) none of them
ANSWER: A
A 49-yr-old woman complains of perioral paraesthesia, carpopedal spasm and generalised
seizures. Put diagnosis of metabolic disturbances, please.
A) Hypokalaemia
B) Hyperkalaemia
C) Hypocalcaemia
D) Hypercalcaemia
E) Hyponatraemia
ANSWER: C
A 50-year old male, working as a hotel cook, has four dependent family members. He has
been diagnosed with an early stage squamous cell cancer of anal canal. He has more
than 60% chances of cure. The best treatment option is:
A) Combined chemotherapy and radiotherapy
B) Combined surgery and radiotherapy
C) Abdomino-perineal resection
D) Chemotherapy alone
E) None of above
ANSWER: A
A 50-year-old male is evaluated because of pain in the right inguinal area, lower back,
and hands. The inguinal pain worsens as he walks, and all pain increases as the day
progresses. On physical examination, internal rotation of the right hip elicits the
groin pain. The second, third, and fourth metacarpophalangeal joints are swollen, warm,
and tender to pressure. Radiographs show severe osteoarthritis in the right hip. Which
of the following is the best test to confirm the diagnosis?
A) Rheumatoid factor
B) HLA-B27
C) Serum iron and iron-binding capacity
D) Fasting plasma glucose
E) Serum uric acid
ANSWER: C
A 50-year-old man who was undergoing maintenance haemodialysis was reviewed. His
medication comprised alfacalcidol 0.25 micrograms daily, calcium carbonate 500 mg
three times daily, ramipril 5 mg daily and simvastatin 10 mg daily. The dialysate
calcium concentration was 1.25 mmol/L. Investigations: serum corrected calcium 2.60
mmol/L (2.20–2.60), serum phosphate 1.67 mmol/L (0.8–1.4), plasma parathyroid hormone
28.0 pmol/L (0.9–5.4). What is the most appropriate next management step?
A) change calcium carbonate to sevelamer
B) dialyse against lower calcium dialysate
C) no change
D) omit alfacalcidol
E) prescribe cinacalcet
ANSWER: A
A 52 year old female patient was diagnosed to have chronic hepatitis C. He responded
to treatment with interferon. However, after one year of follow up he showed a relapse
of disease. Which of the following would be the next most appropriate choice?
A) Ribavarin and interferon
B) Lamivudine and interferon
C) Nevirapine and lamivudine
D) Indinavir and ribavarin.
E) None of above
ANSWER: A
A 55-year-old man presented to the clinic with a nodule on his elbow. He reported that
the nodule had been bothering him for 2 years and had steadily increased in size
recently. Further questioning revealed a history of repeated attacks of acute joint
pain. The physical examination revealed a rounded, subcutaneous nodule over the elbow,
which was tender and rubbery to the touch. The examination was also notable for a
subcutaneous nodule at the left metatarsal-phalangeal joint and left metacarpal-
phalangeal joint, as well as evidence of arthritis involving both hands. Which of the
following is characteristic of the arthritis?
A) Depositions of monosodium urate monohydrate crystals within the periarticular soft
tissues
B) Overgrowth of bones within joints
C) Autoimmune synovitis
D) Osteoporosis
E) Rheumatoid nodules over joints
ANSWER: A
A 58-year-old nun comes to your office because of lethargy, mild nausea, and weakness
for the past 2 weeks. Three years ago, pulmonary sarcoidosis was diagnosed by biopsy.
Three months ago, the patient began taking oral calcium (1500 mg/d) and 25-
hydroxyvitamin D as treatment for osteoporosis that was diagnosed by screening bone
density testing. She has chronic hypertension that is well controlled with metoprolol,
50 mg/day. On examination, the patient appears thin but well nourished and is in no
distress. She is oriented to time, date, and place. Blood pressure is 140/80 mm Hg,
pulse rate 80/min, temperature 37 °C (98.6°F). The thyroid is normal, and the neck
veins are not distended. The lungs are clear. Cardiac examination shows regular sinus
rhythm, no murmur, and normal first and second heart sounds. The abdomen is not tender,
the liver and spleen are not palpable, and no mass is present. There is no edema in
the lower extremities, and reflexes are 1 + and symmetrical. Laboratory studies:
Hemoglobin 138 g/L Hematocrit 38%. Leukocyte count 5,6x109/L. Blood urea nitrogen 24
mmol/L. Serum creatinine 220 mmol/L (was 100 mmol/L 3 months ago) Serum sodium 141
mmol/L. Serum potassium 4.4 mmol/L. Serum chloride 105 mmol/L. Serum bicarbonate 24
mmol/L. Serum calcium 12.8 mg/dL. Serum phosphorus 3.5 mg/dL. Serum parathyroid
hormone 18 pg/mL. Urinalysis pH 5.5; specific gravity 1.010; no proteinuria, hematuria,
or glucosuria; no cells on microscopy. Serum and urine immunoglobulins showed no
monoclonal protein. A polyclonal increase in IgG is present. Renal ultrasonography
demonstrates no hydronephrosis and no calculi. What is the most likely cause of this
patients acute renal failure?
A) Myeloma kidney
B) Acute interstitial nephritis
C) Hypercalcemia
D) Acute glomerulonephritis
E) Bilateral renal artery stenosis
ANSWER: B
A 64-year-old woman presents with a 1-year history of pain in her thumbs. A focused
examination reveals squaring at the base of both first digits, worse on the right, and
pain on pressure over the first carpal metacarpal joints. She also has non tender bony
overgrowth at the distal interphalangeal joints. The patient says that her mother had
the same fingers and she worries that she will become crippled. Choose the most likely
diagnosis from the below list of options.
A) Ankylosing spondylitis
B) Rheumatoid arthritis
C) Osteoarthritis
D) Gout
E) Psoriatic arthritis
ANSWER: C
A 66-year-old man has had a several-week history of fatigue and ankle swelling. His
appetite is normal, but his body weight has increased 4.5 kg (10 Ib). There is no
history of exertional dyspnea, paroxysmal nocturnal dyspnea, or orthopnea. He has had
hypertension for 10 years, and diabetes was diagnosed 4 months ago. He takes
hydrochlorothiazide, 25 mg/d, and metoprolol, 50 mg/d. On examination, the patient is
alert and in no distress. Blood pressure is 120/75 mm Hg, pulse rate is 82/min, and
temperature is 36.9 °C (98.4 °F). Skin examination is normal, and he has no
jugulovenous distention. Thyroid examination is normal. The lungs are clear. Cardiac
examination shows regular sinus rhythm, with no murmur or gallop. The abdomen is
slightly protuberant with shifting dullness, but no hepatosplenomegaly is noted. There
is 4+ edema of the legs and thighs and 1+ edema of the sacrum. Laboratory studies:
Complete blood count Normal Hemoglobin A1c 7.4%. Blood urea nitrogen - 20 mg/dL. Serum
creatinine - 0.9 mg/dL. Serum sodium - 141 meq/L. Serum chloride - 104 meq/L. Serum
potassium - 4.3 meq/L. Serum bicarbonate - 30 meq/L. Serum total protein - 4.4 g/dL.
Serum albumin - 1.7 g/dL. Serum cholesterol - 376 mg/dL. 24-hour urine protein 8.5 g.
Urinalysis pH 5.5; specific gravity 1 .020; protein 4+, trace hemoglobin Urine
microscopy shows many granular and hyaline casts and oval fat bodies. Monoclonal
protein is identified on urine immunoelectrophoresis. Plasma immunoelectrophoresis
shows an IgG level of 452 mg/dL, IgA of 284 mg/dL, 1gM of 122 mg/dL, K of 550 mg/dL,
and 2 of 193 mg/dL with a homogenous M band. What renal disease is most likely in this
patient?
A) Idiopathic membranous glomerulopathy
B) Diabetic nephropathy
C) Focal and segmental glomerulosclerosis
D) Hypertensive nephropathy
E) AL amyloidosis
ANSWER: E
A 68-year-old man is evaluated because of a 3-year history of bilateral knee pain and
low back pain. He has some stiffness for approximately 15 minutes when she awakens in
the morning, and during the afternoon her pain is worse. On physical examination, he
has slight swelling and tenderness to pressure of the distal interphalangeal joints 2-
5 on both hands. There is slight crepitus with motion of the right knee. Which of the
following treatment do use?
A) Methotrexat
B) Glucosamine
C) Colchicines
D) Tetracycline
E) Allopurinol
ANSWER: B
A 71-year-old man had non-Hodgkins lymphoma diagnosed 4 months ago. Treatment included
chemotherapy with rituximab and cyclosphosphamide, prednisone, and vincristine and
doxorubicin, followed by radiation therapy. He has had persistent anemia. He has had
hypertension for 15 years and prostatic hypertrophy, treated with doxazosin, for 10
years. He presents to the emergency department with polyuria, weakness, and lassitude.
On physical examination, blood pressure is 124/78 mm Hg, with no orthostatic changes;
pulse rate, 96/min; respiratory rate, 18/min; and temperature 36.8 °C (98.2 °F).
Mucous membranes are moist. There is no neck vein distention or hepatojugular reflux.
The cardiac examination is normal, and the chest is clear. The abdomen is benign. No
lower extremity edema is present. The electrocardiogram is normal. Hematocrit is 31%,
and leukocytosis is present, with a normal platelet count. The serum creatinine
concentration, previously normal, is now 2.4 mmol/L. Urinalysis shows a pH of 6.0 and
1 + proteinuria, but no hematuria or ketonuria. No formed elements appear on
microscopic examination. Urine sodium and osmolality values have been requested. What
is the most important next test to determine the reason for decreased renal function
in this patient?
A) Calculate the ratio of blood urea nitrogen to creatinine
B) Perform renal ultrasonography
C) Perform renal scanning
D) Administer 1.5 L of normal saline as a fluid trial
E) Perform renal biopsy
ANSWER: B
A criterion for the diagnosis of asthma is:
A) 15 % reversibility in spirography
B) 5% reversibility in spirography
C) 10% reversibility in spirography
D) X-ray changes
E) Sputum changes
ANSWER: A
A drug is to be delivered by a nebuliser. The size of a droplet for its humidification
(in mcm) is:
A) less than 5
B) 5-10
C) 10-15
D) 15-20
E) All of the above
ANSWER: A
A man aged 60 yrs has history of ischemic heart disease and atherosclerosis. He
presents with abdominal pain and maroon stools: likely diagnosis here is:
A) Acute mesenteric ischemia
B) Acute intestinal obstruction
C) Peritonitis
D) Appendicitis
E) None of above
ANSWER: A
A patient was operated for colonic carcinoma and later a solitary nodule was found in
the liver. Treatment of choice is:
A) Surgery
B) Radiation
C) Chemotherapy
D) Conservative treatment
E) None of above
ANSWER: A
A patient with BMI > 30 (kg/m2), serum glucose 24 mmol/L, urinary ketones ++++, he
requires:
A) Insulin
B) Glibenclamide
C) Metformin
D) Phenformin
E) Oral bicarbonate
ANSWER: A
A patient with BMI 34, serum glucose 26 mmol/l, urinary ketones 4+ requires:
A) Insulin
B) Glibenclamide
C) Metformin
D) Glimepiride
E) Acarbose
ANSWER: A
A patient with DKA has deep sighing respiration. The main cause of Kussmaul breathing
is:
A) Hyperglycemia
B) Metabolic acidosis
C) Dehydration
D) Hypokalemia
E) Ketonemia
ANSWER: B
Above the cavity of abscess it is possible to hear such breathing
A) Bronchial
B) Vesicular
C) Unclear
D) Hard
E) Weakened
ANSWER: A
Acid secretion phase, following food intake, defined by stimulation of mechanical and
chemical gastric wall receptors by luminal contents:
A) cephalic phase
B) gastric phase
C) intestinal phase
D) All of the above
E) None of above
ANSWER: B
Activation of these/this receptor(s) on basolateral parietal cells inhibit(s) gastric
acid secretion
A) prostaglandins
B) gastrin
C) histamine
D) acetylcholine
E) all of the above
ANSWER: A
Acute abscess and gangrene of lungs belong to
A) Pyogenic diseases of lungs
B) Chronic non-specific diseases of lungs
C) Congenital pathology of lungs
D) Obstructive diseases of lungs
E) Systemic diseases
ANSWER: A
Acute lymphocytic leukemia (ALL)
A) A 4-year-old patient with pancytopenia and circulating blasts
B) A 60-year-old patient with pancytopenia and circulating blasts
C) A 20%-30% long-term survival
D) A patient with bleeding and infection
E) A patient with gum and skin infiltration
ANSWER: A
Acute myelogenous leukemia (AML)
A) A 4-year-old patient with pancytopenia and circulating blasts
B) A 40-year-old patient with pancytopenia and circulating blasts
C) A 70%-80% long-term survival
D) A patient with bleeding and infection
E) A patient with gum and skin infiltration
ANSWER: B
Adenocarcinoma of esophagus is commonly found in :
A) Barret's oesophagus
B) Achalasia acardia
C) Plummer vinson syndrome
D) Chronic smoking
E) None of above
ANSWER: A
After undergoing surgery, for Carcinoma of colon a patient developed single liver
metastasis of 2cm. What you do next:
A) Resection
B) Chemo radiation
C) Acetic acid injection
D) Radio frequency ablation
E) None of above
ANSWER: A
All are precancerous for carcinoma colon except
A) carotene
B) Bile acids
C) Fats
D) crohn's disease
E) None of above
ANSWER: A
All are true about pseudopancreatic cyst of
A) Most common site is in head of pancreas
B) Presents as an abdominal mass
C) Serum amylase is increased
D) Common after acute pancreatitis
E) None of above
ANSWER: A
All are true regarding Helicobacter pylori except:
A) Less prevalent in developing countries
B) Toxicogenic strains usually causes ulcers
C) Urea breath test is positive
D) Gram negative organism
E) None of above
ANSWER: A
All are true regarding paroxysmal nocturnal haemoglobinuria, except:
A) Direct antiglobulin test (direct Coombs' test) is negative
B) Red urine
C) Presence of antibodies
D) Reticulocytosis
E) Raised bilirubin
ANSWER: C
All are true regarding Zollinger Ellison syndrome, except:
A) Decreased ratio of BAO to MAO
B) Recurrence after operation
C) Hypergastrinemia
D) Diarrhoea
E) None of above
ANSWER: A
All are used in treatment of Helicobactor pylori, EXCEPT:
A) Cisapride
B) Colloid bismuth
C) Clarithromycin
D) Metronidazole
E) Lansoprazol
ANSWER: A
All clinical signs can be seen in ophthalmic Grave's disease EXCEPT:
A) Lid retraction
B) Frequent blinking
C) Poor convergence
D) Upper lid "lad" on down gaze
E) Wide opened eyes
ANSWER: B
All medicines listed below are used in bronchial asthma, except:
A) Morphine
B) Salbutamol
C) Aminophylline
D) Steroids
E) Ipratropium
ANSWER: A
All of the following are principles of treatment of DM, EXCEPT:
A) Achievement and maintenance of normal or reasonable body weight.
B) Normalization of metabolism and achievement of DM compensation.
C) Education of the patients and self – control.
D) Maintenance (preservation) of working capacity.
E) Prophylaxis of acute and chronic complications.
ANSWER: C
All of the following are true regarding a patient with acid peptic disease except
A) Misoprostol is drug of choice in pregnant patients
B) Duodenal ulcer is preventable by the use of single night time H2 blockers
C) Omeprazole may help ulcers refractory to H2 blockers
D) Misoprostol is the drug of choice in patients on NSAIDS
E) None of above
ANSWER: A
All of the following are used in the management of uncomplicated thyrotoxicosis EXCEPT:
A) Propylthiouracil
B) Vasodilators
C) Radioactive iodine
D) Lugol's iodine
E) Propranolol
ANSWER: B
All of the following are useful for treating acute bronchial asthma except:
A) Sodium chromoglycate inhalation
B) 100% Oxygen
C) Hydrocortisone infusion
D) IV aminophylline
E) All of the above
ANSWER: A
All of the following drugs are useful in the treatment of a patient with acute
bronchial asthma except:
A) Montelukast
B) Ipratropium
C) Salbutamol
D) Hydrocortisone
E) All of the above
ANSWER: A
All of the following features are seen in the viral pneumonia except:
A) Presence of interstitial inflammation
B) Predominance of alveolar exudates
C) Bronchiolitis
D) Multinucleate giant cells in the bronchiolar wall
E) All features are present
ANSWER: B
All of the following statements about Leukotriene modifiers in the management of
bronchial asthma are true except:
A) May be used for acute asthma
B) May be used for exercise induced asthma
C) Zileuton is Leukotriene modifier
D) May uncover Churg Strauss syndrome
E) All of the above
ANSWER: A
All of the following statements about Pseudopancreatic cysts are true except:
A) Percutaneous aspiration is treatment of choice
B) Cystojejunostomy is treatment of choice
C) Serum amylase levels are increased
D) Presents as an epigastric mass
E) None of above
ANSWER: A
All of the following statements are true about sickle cell disease except
A) Vaso-occlusive crisis
B) Aplastic Crisis
C) Hypertensive crisis
D) Sequestration Crisis
E) Hyper hemolytic crisis
ANSWER: C
All of the following statements stand true for telangiectasia of colon except:
A) 50% involve rectum
B) May be seen in person more than 60 years of age
C) Common site is caecum
D) May be seen in person less than 40 years of age
E) None of above
ANSWER: A
All of these signs may be present in patient with pituitary insufficiency, EXCEPT:
A) Hypotension.
B) Hyperpigmentation.
C) Weight loss.
D) Hypogonadism.
E) Hypothyroidism.
ANSWER: B
All signs are associated with tetany EXCEPT:
A) Chvostek’s sign
B) Trousseau's sign
C) Erb's sign
D) Cole's sign
E) None above them
ANSWER: D
All statements are true in hyperparathyroidism EXCEPT:
A) May cause hypercalcemia
B) Commonly occurs after thyroidectomy
C) Solitary adenoma in parathyroid is a common cause
D) Nephrolithiasis is common
E) Frequent fractures
ANSWER: B
All the following are causes of Acute Pancreatitis except:
A) Hemochromatosis
B) Alcohol
C) Gall stones
D) Hypercalcemia
E) None of above
ANSWER: A
All the following can be indications for parathyroidectomy in patients with
hyperparathyroidism EXCEPT
A) Advanced age
B) Kidney stones
C) Osteoporosis
D) Calcium level &amp;amp;gt;2.9 mmol/L
E) Decreased creatinine clearance
ANSWER: A
All the following indicates early gastric cancer except
A) Involvement of mucosa, submucosa and muscularis
B) Involvement of mucosa and submucosa
C) Involvement of mucosa
D) Involvement of mucosa, submucosa and adjacent lymph nodes
E) None of above
ANSWER: A
All the following may be the causes of hemorrhagic pleural effusion except:
A) Congestive heart failure
B) Pulmonary embolism
C) Lung cancer
D) Tuberculosis
E) Severe chest trauma
ANSWER: A
Among the following, the most common site for Leiomyoma is
A) Stomach
B) Small Intestine
C) Duodenum
D) Colon
E) None of above
ANSWER: A
Antacid: effective, but associated with systemic alkalosis
A) sodium bicarbonate
B) calcium carbonate
C) magnesium hydroxide
D) aluminum hydroxide
E) All of above
ANSWER: A
Antacid: loose stools
A) magnesium hydroxide
B) aluminum hydroxide
C) All of above
D) None of above
E) sodium bicarbonate
ANSWER: A
Antacid: most likely to cause systemic phosphate depletion:
A) aluminum hydroxide
B) calcium carbonate
C) magnesium hydroxide
D) sodium bicarbonate
E) All of above
ANSWER: A
Antacid(s)
A) All of above
B) magnesium hydroxide
C) calcium carbonate
D) aluminum hydroxide
E) None of above
ANSWER: A
Aplastic anemia can be caused by all except:
A) ampicillini
B) chloramphenicol
C) quinine
D) phenytoin
E) carbamazepine
ANSWER: A
Aspirin sensitive asthma is associated with:
A) Nasal polyps
B) Extrinsic asthma
C) Urticaria
D) Obesity
E) All of the above
ANSWER: A
Attack of bronchial asthma differs from heart asthma by such sign:
A) By expiration dyspnea
B) By sudden development of attack
C) By orthostatic position of the patient
D) By duration of the attack
E) By inspiration dyspnea
ANSWER: A
Barrett's oesophagus is:
A) Lower oesophagus lined by columnar epithelium
B) Upper oesophagus lined by columnar epithelium
C) Lower oesophagus lined by ciliated epithelium
D) Lower oesophagus lined by pseudostretifide epithelium
E) None of above
ANSWER: A
Basal acid secretion is inhibited by:
A) all of the above
B) cimetidine (Tagamet)
C) nizatidine (Axid)
D) ranitidine (Zantac)
E) None of above
ANSWER: A
Basal acid secretion is stimulated by:
A) histamine
B) cimetidine (Tagamet)
C) nizatidine (Axid)
D) ranitidine (Zantac)
E) famotidine (Pepcid)
ANSWER: A
Basolateral parietal cell membranes contained these receptor types:
A) All of above
B) acetylcholine
C) prostaglandins
D) histamine
E) gastrin
ANSWER: A
Bence Jones protein may be found in the urine in
A) Chronic lymphocytic leukemia
B) Waldenstroms macroglobulinemia
C) Rheumatic fever
D) Multiple myeloma
E) AML
ANSWER: D
Best antithyroid drug to be used to pregnancy is:
A) Carbimazole
B) Thiamazole
C) Propylthiouracil
D) Radiocative I
E) Non of above
ANSWER: C
Best method for detecting of the minimal bronchiectasis is:
A) CT scan
B) Bronchogram
C) Radionuclide lung scan
D) Chest X-ray
E) Nothing of the above
ANSWER: A
Best position to reveal the small pleural effusions on chest X-ray is
A) Lateral decubitus view
B) AP view
C) PA view
D) Lateral view
E) All of the above
ANSWER: A
Best test for Small intestine malabsorption of carbohydrates is :
A) D-Xylose test
B) Shilling test
C) Lund meal test
D) Follacin test
E) None of above
ANSWER: A
Bilateral malignant pleural effusion is most often seen in:
A) Cancer of lung
B) Cancer of breast
C) Mesothelioma
D) Lymphoma
E) Nothing of the above
ANSWER: A
Bilateral pleural effusion is seen in:
A) Congestive cardiac failure
B) Nephritic syndrome
C) Constrictive pericarditis
D) All of the above
E) Nothing of the above
ANSWER: D
Bilateral subtotal of thyroidectomy for Graves’ disease most likely can cause
hypoparathyroidism as a result as:
A) Loss of thyrocalcitonin
B) Reduction in parathyroid stimulating hormone
C) Removal of the four parathyroids
D) Infarction of the parathyroids
E) Secondary hypopituitarism
ANSWER: D
Blocking of these structures can worsen the asthma
A) ?2 -adrenoreceptors
B) ?-adrenoreceptors
C) Mast cells
D) Neutrophils
E) Eosinophils
ANSWER: A
Bone marrow transplant is the treatment of choice in all, except:
A) Aplastic anemia
B) AML in 1st remission
C) ALL in second remission
D) ITP
E) Lympoma
ANSWER: D
Bronchial asthma is associated with raised levels of
A) Leukotrienes
B) Thromboxane
C) Antinuclear antibodies
D) Corticosteroids
E) All of the above
ANSWER: A
Bronchiectasis is the most common in such lung lobe:
A) Left lower
B) Right middle
C) Right upper
D) Left upper
E) Nothing of the above
ANSWER: A
Causes of haemorrhagic pleural effusion are all except:
A) Bronchial adenoma
B) Pulmonary infarction
C) Mesothelioma
D) Tuberculosis
E) Nothing of the above
ANSWER: A
CBC revealed a WBC of 72,000 and a differential that was 83% lymphocytes. What is the
MOST likely diagnosis?
A) Acute lymphocytic leukemia
B) Chronic lymphocytic leukemia
C) Acute prolymphocytic leukemia
D) Chronic prolymphocytic leukaemia
E) Lymphoma
ANSWER: B
Characteristic(s) of gastric mucosal acid secretion:
A) all of the above
B) from parietal cells founded in mucosal glands of fundusof the stomach
C) stimulated by muscarinic cholinergic system (parietal cell innervation)
D) oxidative dephosphorylation dependent
E) None of above
ANSWER: A
Charcot’s joint in diabetes is NOT seen in:
A) None of them
B) Knee
C) Hip
D) Tarsal
E) Elbow
ANSWER: E
Chose correct answer. Zollinger-Ellison syndrome -
A) gastrin-secreting islet cell tumor
B) it is associated with Helicobacter pylori
C) it is associated withLamblia
D) All of the above
E) None of above
ANSWER: A
Common sites for cushing ulcers include all of the following except:
A) Distal duodenum
B) Stomach
C) Is1 part of duodenum
D) Esophagus
E) None of above
ANSWER: A
Common X-ray finding of Staphylococcal pneumonia:
A) Bilateral hilar lymphadenopathy
B) Involvement of lower lobes
C) Dense homogenous consolidation
D) Visible air bronchogram
E) Nothing of the above
ANSWER: A
Commonest site of peptic ulcer is:
A) 1st part of Duodenum
B) Middle part of duodenum
C) Distal 1/3 of stomach
D) Pylorus of the stomach
E) None of above
ANSWER: A
Consider the following: Serum iron (1), MCV (2), serum total iron binding capacity (3),
serum ferritin (4). Which of these decreased in iron deficiency anaemia?
A) 1 and 2
B) 1, 3 and 4
C) 1, 2 and 4
D) 1, 2, 3 and 4
E) 2,3 and 4
ANSWER: C
Constipation is caused by
A) aluminum hydroxide
B) magnesium hydroxide
C) All of above
D) None of above
E) sodium bicarbonate
ANSWER: A
Contraindication for glucocorticosteroid therapy of bronchial asthma is:
A) Bleeding from gastric ulcer
B) Chronic persistant hepatitis
C) Exacerbation of respiratory-heart failure
D) Emphysema of lungs
E) Respiratory failure
ANSWER: A
Curschmann's spirals in sputum are seen in:
A) Bronchial asthma
B) Bronchiectasis
C) Chronic bronchitis
D) Wegenersgranulomatosis
E) All of the above
ANSWER: A
Dawn phenomenon refers to:
A) Early morning hyperglycemia followed by hypoglycemia
B) Hypoglycaemia followed by hyperglycemia
C) Early morning hypoglycemia
D) Early morning hyperglycemia
E) High insulin level
ANSWER: D
Deaths from lactic acidosis in diabetes mellitus are associated with therapy:
A) Glibenclamide
B) Gliquidone
C) Chlorpropamide
D) Phenformin
E) Glipizide
ANSWER: D
Decreased maximum mid-expiratory flow rate indicates obstruction in:
A) Small airways
B) Trachea
C) Large airways
D) Trachea &amp;amp;amp; bronchi both
E) Nothing of the above
ANSWER: A
Diabetic ketoacidosis characterized by:
A) Low serum potassium
B) Increased anion gap
C) Metabolic alkalosis
D) Respiratory acidosis
E) Hypoglycemia
ANSWER: B
Diagnostic tests for H. Pylori include all of the following except:
A) Nechyporenco test
B) Rapid urease test
C) Gastric biopsy &amp;amp;amp; Warthin-starry stain
D) Urea-breath test
E) None of above
ANSWER: A
Drug of choice for inhalation in acute asthma not responding to salbutamol is:
A) Ipratropium bromide
B) Atropine sulfate
C) Isopropamide
D) Hyoscine methyl bromide
E) All of the above
ANSWER: A
Drug of choice in asthma with heart disease is
A) Ipratropium
B) Rimiterol
C) Terbutaline
D) Cromolyn sodium
E) All of the above
ANSWER: A
Drug of choice in Zollinger Ellison syndrome is
A) Omeprazole
B) Ranitidine
C) Antacids
D) B-blocker
E) Atropin
ANSWER: A
Drug(s) used in eradication of Helicobacter pylori infection:
A) All of above
B) metronidazole (Flagyl)
C) clarithromycin (Biaxin)
D) bismuth compounds
E) amoxicillin (Amoxil Polymox)
ANSWER: A
During an epidemic of hepatitis E, Fatality is maximum in :
A) Pregnant women
B) Infants
C) Malnourished male
D) Adolescents
E) None of above
ANSWER: A
Early gastric cancer generally indicates
A) Gastric adenocarcinoma confined to the mucosa &amp;amp;amp; submucosa
B) Gastric adenocarcinoma confined to the mucosa
C) Gastric adencarcinoma detected early
D) Gastric adenocarcinoma less than 1 cm In size
E) None of above
ANSWER: A
Endoscopic biopsy from a case of H. pylori related duodenal ulcer is most likely to
reveal
A) Antral predominant gastritis
B) Multifocal atrophic gastritis
C) Acute erosive gastritis
D) Gastric atrophy
E) None of above
ANSWER: A
Epidemiological studies of H. pylori are done by using:
A) Serological markers
B) Urea-breath test
C) Culture
D) Gastric-biopsy urease test
E) None of above
ANSWER: A
Eradication of H. pylori effectiveness for bismuth compounds when using combination
with antibiotics:
A) 95%
B) 20%
C) 50%
D) 80%
E) 5%
ANSWER: A
Example(s) of "aggressive factors" in peptic ulcer disease:
A) pepsin
B) bicarbonate
C) prostaglandins
D) neither
E) All of above
ANSWER: A
Example(s) of "agressive factor(s)" in peptic ulcer disease:
A) Helicobacter pylori
B) prostaglandins
C) gastric mucus
D) bicarbonate
E) all of the above
ANSWER: A
Example(s) of "protective factor(s)" in peptic ulcer disease:
A) all of the above
B) prostaglandins
C) gastric mucus
D) bicarbonate
E) None of above
ANSWER: A
Examples of H2 (histamine receptor Type II) antagonists:
A) All of above
B) ranitidine (Zantac)
C) nizatidine (Axid)
D) famotidine (Pepcid)
E) cimetidine (Tagamet)
ANSWER: A
Exercise induced asthma is not precipitated by :
A) Swimming in hot water
B) High altitude climb and exercises
C) Cycling in cold weather
D) Swimming in cold water
E) All of the above
ANSWER: A
FEV1/FVC is reduced in case of:
A) Asthma
B) Pleural effusion
C) Lung fibrosis
D) All of the above
E) Nothing of the above
ANSWER: A
Findings in hemophilia A include:
A) Partial thromboplastin time increased
B) Increased clotting time
C) Increased prothrombin time
D) Bleeding time increased
E) Thrombocytopenia
ANSWER: A
Following are causes of unconjugated hyperbilirubinemia, except:
A) Rotor syndrome
B) Large hematoma
C) Hemolytic anemia
D) Megaloblastic anemia
E) Irondeficiency anemia
ANSWER: A
For the patient who has sustained tracheobronchial trauma, which assessment finding
alerts the physician to the possibility of tracheal lacerations?
A) Hypertympanic sound on affected side
B) Subcutaneous emphysema over the trachea
C) Hypotension and decreased capillary refill
D) Deviation of the trachea to the affected side
E) Non of above
ANSWER: B
Haemophilia A is characterised by the presence of following features, except
A) Bleeding into soft tissues
B) Reduced VIII levels
C) Prolonged bleeding time
D) Prolonged partial thromboplastin time
E) Bleeding into muscles &amp;amp;amp; joints
ANSWER: C
Helicobacter pylori is not associated with :
A) Gastric leiomyoma
B) Gastric cancer
C) Peptic ulcer of stomach
D) Peptic ulcer of duodenum
E) None of above
ANSWER: A
Histamine and gastric acid secretion:
A) all of the above
B) release enhanced by increased cholinergic activity
C) most important gastric acid secretion stimulant
D) released from enterochromaffin-like cells
E) None of above
ANSWER: A
How long is the usual course of drug treatment for a patient with active tuberculosis?
A) 7 to 10 days
B) 6 weeks
C) 6 months
D) 2 years
E) Non of above
ANSWER: C
Hypercalcemia may be treated by:
A) IV pamidronate
B) IV fluids + furosemide
C) Glucocorticoids
D) None above them
E) All of them
ANSWER: E
Hypergastrinemia with hypochlorhydria is seen in:
A) Pernicious anemia
B) All of above
C) Zollinger Ellison Syndrome
D) Glucagonoma
E) None of above
ANSWER: A
Hyperosmolar hyperglycemic non-ketonic coma is characterized by such level of glycemia:
A) 55 mmol/1
B) 20 mmol/1
C) 30 mmol/1
D) 5 mmol/1
E) 25 mmol/1
ANSWER: A
Hypoglycemic coma can be caused by:
A) Insulin deficiency
B) Dehydration
C) Extreme activity
D) Metabolic acidosis
E) Hypokalemia
ANSWER: C
Hypoglycemic effect of intermediate acting insulin lasts for:
A) 12-16 hours
B) 20 -30 hours
C) 0.5 – 1.0 hours
D) 4 – 6 hours
E) 6 – 8 hours
ANSWER: A
Hypoglycemic unawareness that occurs in diabetic patients when transferred from oral
hypoglycemic agents to insulin, is due to :
A) Autonomic neuropathy
B) Insulin resistance
C) Allergy
D) Somogi phenomenon
E) Down phenomenon
ANSWER: A
In a case of hypertrophic pyloric stenosis, the metabolic disturbance is
A) Metabolic alkalosis with paradoxical aciduria
B) Metabolic acidosis
C) Respiratory alkalosis
D) Metabolic alkalosis with alkaline urine
E) None of above
ANSWER: A
In a patient only Anti HBsAg is positive in serum, all other viral markers are
negative. It indicates :
A) Immunized person with hepatitis B vaccine
B) Chronic active hepatitis
C) Persistent carrier
D) Acute hepatitis
E) None of above
ANSWER: A
In bronchial asthma glucocorticoids
A) Reduce airway inflammation
B) Act as potent bronchodilators
C) Inhibit degranulation of mast cells
D) Block the action of humoral mediators
E) Nothing of the above
ANSWER: A
In endemic goiter such statement will be true:
A) Investigations invariably show some degree of hypothyroidism
B) There is a clear evidence of an increased incidence of carcinoma of the thyroid
C) Administration of thyroxine will often cause the goitre to shrink
D) Administration of iodine occasionally precipitates hyperthyroidism
E) All of them
ANSWER: E
In gastric outlet obstruction in a peptic ulcer patient, the site of obstruction is
most likely to be
A) Duodenum
B) Antrum
C) Pylorus
D) Pyloric canal
E) None of above
ANSWER: A
In patients with nonketotic hyperosmolar coma the level of glycemia is :
A) Slightly elevated
B) Mildly elevated
C) Moderately elevated
D) May be decreased
E) Grossly elevated
ANSWER: E
In pneumonia due to Mycoplasma all of the following are true except:
A) Bilateral infiltration on chest X-ray
B) Myalgia
C) Dry cough
D) Pleuritic chest pain
E) All of the above
ANSWER: A
In restrictive lung disease :
A) FEV1/FVC is high
B) FVC is high
C) FEV1 is high
D) All of the above
E) Nothing of the above
ANSWER: A
In severe bronchial asthma, it is present:
A) Hyperresonant chest with prolonged expiration
B) Infrequent rhonchi and absent breath sounds
C) Increased fremitus and crackles
D) Decreased fremitus and crepitation
E) All of the above
ANSWER: A
In the time of attack of bronchial asthma in lungs are heard
A) Dry loud wheezes
B) Moist loud wheezes
C) Moist soundless wheezes
D) Voice of pleural friction
E) Crepitation
ANSWER: A
In what specific way does the respiratory system function contribute to acid-base
balance?
A) Prevents excessive loss of hydrogen ions by evaporation
B) Increases the potassium ion content in bronchial secretions
C) Removes carbon dioxide generated as a result of metabolism
D) Maintains body water levels through mucous membrane function
E) Non of above
ANSWER: C
In what specific way does the respiratory system function contribute to acid-base
balance?
A) Prevents excessive loss of hydrogen ions by evaporation
B) Increases the potassium ion content in bronchial secretions
C) Removes carbon dioxide generated as a result of metabolism
D) Maintains body water levels through mucous membrane function
E) Non of above
ANSWER: C
In which case the dose of insulin in stable diabetics can be reduced:
A) Thyrotoxicosis
B) Steroid therapy
C) Pregnancy
D) Chronic renal failure
E) None above them
ANSWER: D
Increased amylase may be seen in all of the following except:
A) Appendicitis
B) Pancreatic pseudocyst
C) Perforated peptic ulcer
D) Ruptured ectopic pregnancy
E) None of above
ANSWER: A
Increased incidence of duodenal ulcer associated with:
A) All of above
B) alcoholic cirrhosis
C) renal transplantation
D) systemic mastocytosis
E) None of above
ANSWER: A
Increased incidence of duodenal ulcer associated with:
A) All of above
B) None of above
C) liver cirrhosis
D) chronic renal failure
E) hyperparathyroidism
ANSWER: A
Increased RBC level may be seen in:
A) Glomerulonephritis
B) Vitamin D excess
C) Cor pulmonale
D) Acute heart failure
E) Pneumonia
ANSWER: C
Insulin resistance is associated with all of the following status, EXCEPT:
A) Acanthosis nigricans
B) Lipodystrophy
C) Gout
D) Obesity
E) Metabolic syndrome
ANSWER: C
Intestinal hypomotility is seen in all the following except:
A) All of the above
B) Parkinsonism
C) Amyloidosis
D) Diabetes
E) None of above
ANSWER: A
Intrinsic factor for absorption of vitamin Bl2 is secreted by the :
A) Parietal cells
B) Peptic cells
C) Chief cells
D) Goblet cells
E) Endothelial cells
ANSWER: A
It is not Antacid
A) Histamine
B) magnesium hydroxide
C) calcium carbonate
D) sodium bicarbonate
E) All of above
ANSWER: A
Location(s) of histamine in gastric mucosa:
A) enterochromaffin-like cells (ECL) + mast cell cytoplasmic granules
B) mast cell cytoplasmic granules
C) enterochromaffin-like cells (ECL)
D) neither
E) are not localized in gastric mucosa
ANSWER: A
Macrophages containing large quantities of undigested and partial digested bacteria in
intestine are seen in
A) Whipple's disease
B) Amyloidosis
C) Immunoapproliferative small instetinal disease
D) Vibro cholerae infection
E) None of above
ANSWER: A
Massive bleeding per rectum in a 70 yr old patient is due to :
A) Diverticulosis
B) Carcinoma colon
C) Colitis
D) Polyps
E) None of above
ANSWER: A
Mechanism of action: cytoprotection, binds to the ulcer base, antibacterial effect:
A) bismuth compounds
B) tetracycline (Achromycin)
C) metronidazole (Flagyl)
D) ranitidine (Zantac)
E) omeprazole (Prilosec)
ANSWER: A
Mechanism of Idiopathic thrombocytopenic purpura is:
A) Vasculitis
B) Antibodies to vascular epithelium
C) Antibodies to platelets
D) Antibody to clotting factors
E) Fungal infectious
ANSWER: C
Mechanism(s) by which somatostatin reduces gastrin release:
A) inhibits parietal cells secretion+ inhibits histamine release by enterochromaffin-
like cells
B) inhibits histamine release by enterochromaffin-like cells
C) inhibits parietal cells secretion
D) stimulates parietal cells secretion
E) None of above
ANSWER: A
Melphalan is used in:
A) Wilm's tumor
B) Lymphoma
C) Multiple myeloma
D) Nephroblastoma
E) ALL
ANSWER: C
Metabolic changes associated with excessive vomiting includes the following:
A) Hypokalemia
B) Hyperchloremia
C) Metabolic acidosis
D) Decreases bicarbonates
E) None of above
ANSWER: A
Microcytic anemia is seen in:
A) Sprue
B) Pernicious anemia
C) Latun infestation
D) Iron deficiency anemia
E) Aplastic anemia
ANSWER: D
Monotherapy effectiveness in eradication of H. pylori-- bismuth compounds
A) 20%
B) 5%
C) 50%
D) 90%
E) 100%
ANSWER: A
More likely to affect the cytochrome P450 drug metabolizing system:
A) cimetidine (Tagamet)
B) All of above
C) nizatidine (Axid)
D) ranitidine (Zantac)
E) famotidine (Pepcid)
ANSWER: A
Most common CNS manifestation of Whipple's disease is :
A) Dementia
B) Supranuclear ophthalmoplegia
C) Seizure
D) Cerebellar ataxia
E) None of above
ANSWER: A
Most common complication of acute pancreatitis is:
A) Pseudocyst
B) Pancreatic abscess
C) Phlegmon
D) Pleural effusion
E) None of above
ANSWER: A
Most common extraintestinal manifestation of Crohn's disease of small bowel is.
A) Erythema Nodosum
B) Ankylosing Spondylitis
C) Iritis
D) Ureteral Obstruction
E) None of above
ANSWER: A
Most common sign of aspiration pneumonitis:
A) Tachypnoe
B) Bronchospasm
C) Cyanosis
D) Crepitations
E) Nothing of the above
ANSWER: A
Most sensitive and specific test for diagnosis of iron deficiency is:
A) Serum ferritin levels
B) Serum iron levels
C) Serum transferrin receptor population
D) Transferrin saturation
E) Hb, Ht
ANSWER: A
Most successful protocol for eradication of H. pylori/treatment of peptic ulcer
disease:
A) therapy using bismuth compounds, Clarythromycin (Flagyl), and amoxicillin (Amoxil
Polymox)
B) bismuth monotherapy
C) bismuth compounds and amoxicillin (Amoxil Polymox)
D) all of the above equally effective
E) None of above
ANSWER: A
Most widely used antacid in treating ulcer disease:
A) combination of aluminum hydroxide and magnesium hydroxide
B) calcium carbonate
C) magnesium hydroxide
D) aluminum hydroxide
E) sodium bicarbonate
ANSWER: A
Mycoplasma infection simulates:
A) Viral pneumonia
B) Pnemuococcal pneumonia
C) Pulmonary oedema
D) Pulmonary infarction
E) Nothing of the above
ANSWER: A
Non-parenteral hepatitis is :
A) Hepatitis E
B) Hep B
C) Hep C
D) Hep D
E) All of above
ANSWER: A
Omeprazole (Prilosec) and lansoprazole approved for clinical treatment of:
A) All of above
B) erosive gastritis
C) Zollinger-Ellison syndrome and other gastric acid hypersecretory states
D) None of above
E) duodenal ulcer
ANSWER: A
Only class of antiulcer drugs that can eradicate Helicobacter pylori and cure
associated gastritis:
A) colloidal bismuth
B) sucralfate (Carafate)
C) H2 blockers
D) Anticholinergic agents
E) None of above
ANSWER: A
Oral anticoagulant therapy is monitored with indexes
A) INR
B) PTT
C) TT
D) Clot lysis time
E) Bleeding time
ANSWER: A
Pancreatic pseudocyst most commonly occurs after
A) Pancreatitis
B) Trauma
C) Pancreatic surgery
D) Pancreatic malignancy
E) None of above
ANSWER: A
Paroxysmal nocturnal haemoglobinuria is due to:
A) Antibodies
B) Congenital membrane defect
C) Inherited autoimmune disorder
D) Complement mediated RBC lysis
E) Lysis of RBC in spleen
ANSWER: D
Pathogenic factor(s) in duodenal ulcer:
A) all of the above
B) genetic factor
C) cigarette smoking
D) alcoholic cirrhosis
E) COPD (chronic obstructive pulmonary disease)
ANSWER: A
Patient D. presents elevated serum T4 and increased radioactive iodine uptake. Put
diagnosis, please.
A) Non-toxic goiter
B) Hashimoto's thyroiditis
C) Subacute thyroiditis
D) Hypothyroidism
E) Graves' disease
ANSWER: E
Patient F., 29 yr. old, presents with subacute thyroiditis. Treatment will include:
A) Antibiotics
B) Vitamins
C) Antithyroid drugs
D) Iodine preparations
E) Corticosteroids
ANSWER: E
Patient on heparin therapy should be monitored with:
A) Clotting time
B) Bleeding time
C) Prothrombin time
D) Fibrinogen
E) PTT
ANSWER: A
Patient presents with recurrent duodenal ulcer of 2.5 cm size; procedure of choice
A) Truncal vagotomy and antrectomy
B) Truncal vagotomy and gastrojejunostomy
C) Highly selective vagotomy
D) Laparoscopic vagotomy and gastrojejunostomy
E) None of above
ANSWER: A
Peptic ulcer classification:
A) duodenal+ gastric
B) gastric
C) duodenal
D) neither
E) cardiac
ANSWER: A
Peptic ulcer should be differentiated with
A) All of above
B) Cholecystitis
C) Gastritis
D) Miocardial infarction
E) Pancreatitis
ANSWER: A
Pernicious anemia is seen in:
A) Iron deficiency
B) Chronic liver disease
C) Bleeding
D) Atrophic gastritis
E) Hemolysis
ANSWER: D
Platelet count is decreased in all of the following condition except:
A) Idiopathic thrombocytopenic purpura
B) Thrombotic thrombocytopenic purpura
C) Systemic lupus erythematosus
D) Haemophilia
E) Hemolytic uremic syndrom
ANSWER: D
Pneumothorax is a possible complication of
A) Staphylococcal pneumonia
B) Pneumococcal pneumonia
C) Klebsiella pneumonia
D) Viral pneumonia
E) All of the above
ANSWER: A
Presentation of hypothyroidism includes:
A) Menorrhagia
B) Oligomenorrhoea
C) Amenorrhea
D) All of them
E) None above them
ANSWER: A
Primary atypical pneumonia is caused by:
A) Mycoplasma
B) Mycobateriumkansasii
C) Str. pneumoniae
D) Pneumocystis carinii
E) Nothing of the above
ANSWER: A
Primary hyperparathyroidism most likely caused by:
A) Multiple parathyroid adenomas
B) Solitary parathyroid adenoma
C) Adrenal hyperplasia
D) Ectopic PTH production
E) None above them
ANSWER: B
Pseudopolyps are features of
A) Ulcerative colitis
B) Crohn's disease
C) Celiac sprue
D) Whipple's disease
E) None of above
ANSWER: A
pylori is known to cause all of the following except:
A) Fundal atrophic gastritis
B) Duodenal ulcer
C) Gastritis type B
D) Gastric ulcer
E) None of above
ANSWER: A
Raised serum amylase levels are used to diagnose
A) Acute pancreatitis
B) Degenerative diseases
C) Acute cholecystitis
D) Autoimmune disease
E) None of above
ANSWER: A
Ranitidine (Zantac) and gastric acid secretion:
A) inhibits basal acid secretion+ inhibits secretion in response to vagal stimulation
or feeding
B) inhibits secretion in response to vagal stimulation or feeding
C) inhibits basal acid secretion
D) neither
E) stimulates basal acid secretion
ANSWER: A
Relapse rate for duodenal ulcer following H. pylori eradication:
A) 15%
B) 75%
C) 50%
D) 90%
E) 5%
ANSWER: A
Reserve transcriptase of hepatitis B virus is coded on the following gene :
A) P gene
B) S gene
C) C gene
D) X gene
E) All of above
ANSWER: A
Risk factor for development of gastric Carcinoma
A) Intestinal metaplasia type III
B) Duodenal ulcer
C) Intestinal hyperplasia
D) Blood group O
E) None of above
ANSWER: A
Screening method for medullary carcinoma thyroid is:
A) Serum calcitonin
B) Serum calcium
C) Serum alkaline phosphate
D) Serum acid phosphatase
E) None above them
ANSWER: A
Secondary hyperparathyroidism is associated with:
A) Parathyroid adenoma
B) Marked hypercalcaemia
C) Chronic renal failure
D) Parathyroidectomy
E) All of them
ANSWER: C
Serum unconjugated bilirubin and urine urobilinogen concentration are elevated in
A) anemias of chronic disease
B) hemolytic anemias
C) anemia of acute hemorrhage
D) aplastic anemias
E) iron deficiency anemia
ANSWER: B
Sharko-Leiden crystals are seen in:
A) Bronchial asthma
B) Bronchiectasis
C) Chronic bronchitis
D) Wegenersgranulomatosis
E) All of the above
ANSWER: A
Side effect of antiulcer medication: urinary retention, blurred vision, xerostromia:
A) atropine
B) ranitidine (Zantac)
C) nifedipine (Procardia, Adalat)
D) sucralfate (Carafate)
E) None of above
ANSWER: A
Significant high ESR is seen in :
A) Polycythemia vera
B) CHF
C) Multiple myeloma
D) Sickle cell anemia
E) CLL
ANSWER: C
Skip granulomatous lesions are seen in
A) Crohn's disease
B) Ulcerative colitis
C) Whipple's disease
D) Reiter's disease
E) None of above
ANSWER: A
Small capillaries bleeds in:
A) Thrombocytopenia
B) Coagulation failure deficiency
C) Anemia
D) Agranulocytosis
E) Hemophilia
ANSWER: A
Somogi phenomenon characterized by :
A) Early morning hyperglycemia
B) Early morning hypoglycemia
C) Hypoglycemia followed by hyperglycemia
D) High insulin levels
E) Night hyperglycemia
ANSWER: A
Spherocytosis is best diagnosed by:
A) Splenic puncture
B) BM aspiration
C) Plasma
D) Peripheral blood smear
E) Phenotyping
ANSWER: D
Such radiological findings as "hour-glass" vertebrae and "triturated pelvis" are seen
in:
A) Thyrotoxicosis
B) Myxedema
C) Cretinism
D) Hyperparathyroidism
E) Hypothyroidism
ANSWER: D
Such results of investigations as positive Chvostek sign and elevated serum
parathyroid hormone (PTH) level are associated with:
A) Pseudohypoparathyroidism
B) Primary hypoparathyroidism
C) Primary hyperparathyroidism
D) Osteoporosis
E) Hypercalcemia of malignancy
ANSWER: A
The actions of insulin includes which one of the following?
A) Increased utilization of glucose by muscle
B) Increased hepatic output of glucose
C) Increased lipolysis
D) Increased amino acid flux from muscle due to protein breakdown
E) Activated glycogenolysis
ANSWER: A
The adenocarcinoma of esophagus-developed in
A) Barret's esophagus
B) Long standing achalasia
C) Corrosive structure
D) Alcohol abuse
E) None of above
ANSWER: A
The commonest acute presentation of sickle cell anaemia is:
A) Priapism
B) Bone pain
C) Fever
D) Splenomegaly
E) Pulmonary hypertension
ANSWER: B
The commonest hepatotropic virus progressing to chronicity is:
A) HCV
B) HAV
C) HBV
D) HEV
E) HAV+ HEV
ANSWER: A
The first virological marker following acute infection with HBV is :
A) HBs Ag
B) Anti HBs Ag
C) IgM anti HBc Ag
D) Anti HBe Ag
E) All of above
ANSWER: A
The following laboratory index is abnormally prolonged in ITP:
A) APTT
B) Bleeding time
C) Prothrombin time
D) Clotting time
E) None of above
ANSWER: B
The frequent causes of hypoglycemia in hospitalized diabetic patients include all the
following EXCEPT:
A) Insulin
B) Sulfonylureas
C) Adrenal insufficiency
D) Alcohol
E) Renal failure
ANSWER: C
The high level of C-peptide can be found in a case of:
A) Glucagonoma
B) Insulinoma
C) Gastrinoma
D) Hepatoma
E) In all of them
ANSWER: B
The histological features of coeliac disease include all of the following, except:
A) Increase in thickness of the mucosa
B) Crypt hyperplasia
C) Increase in intraepithelial lymphocytes
D) Increase in inflammatory cells in lamina propyria
E) None of above
ANSWER: A
The lowest recurrence of peptic ulcer is associated with
A) Vagotomy + Antrectomy
B) Vagotomy + drainage
C) Gastric resection
D) Highly selective vagotomy
E) Resection of stomach by Bilroth II
ANSWER: A
The main cause of hypercalcemic crisis is :
A) Parathyroid adenoma
B) CA Breast
C) Parathyroid hyperplasia
D) Paget's disease
E) Hyperthyroidism
ANSWER: A
The main cause of hyperosmolar coma is:
A) Insulin deficiency
B) Dehydration
C) Extreme activity
D) Hypoxia
E) Intercurrent infection
ANSWER: B
The main cause of lactic acidosis is
A) Insulin deficiency
B) Dehydration
C) Extreme activity
D) Hypoxia
E) Intercurrent infection
ANSWER: D
The major sign of hypoventilation is
A) Cyanosis
B) Hypoxia
C) Hypercapnia
D) Dyspnea
E) All of the above
ANSWER: A
The most common cause of aplastic anemia is:
A) Idiopathic
B) Chloramphenicol
C) Phenylbutazone
D) Petroleum products
E) Prednisolone
ANSWER: A
The most common cause of thyroid crisis is:
A) Thyroid surgery
B) 131I administration
C) Administration of Beta blockers
D) Thyrostatic therapy
E) All of the above
ANSWER: A
The most common cause of thyrotoxicosis is:
A) Grave's disease
B) Follicular adenoma
C) Multinodular goiter
D) Subacute thyroiditis
E) Iatrogenic
ANSWER: A
The most common community acquired infection is :
A) Streptococcal pneumonia
B) Hepatitis A
C) Cholera
D) Meningitis
E) Nothing of the above
ANSWER: A
The most common presentation of primary hyperparathyroidism is:
A) Bone fracture
B) Increased serum creatinine
C) Osteitis fibrosa cystica
D) Asymptomatic hypercalcemia
E) Kidney stones
ANSWER: D
The most common route of spread of hepatitis E is
A) Feco-oral
B) Blood transfusion
C) injection IV
D) perinatal transmission
E) All of above
ANSWER: A
The most effective correction of acidosis in diabetic acidosis is:
A) IV bicarbonate
B) IV Saline
C) Oral bicarbonate
D) IV Insulin
E) IV potassium
ANSWER: D
The most important regulator of serum 1,25(OH)2 vitamin D concentration is
A) Serum calcium
B) Serum magnesium
C) Serum 25(OH) vitamin D
D) TSH
E) Parathyroid hormone
ANSWER: E
The most pre-cancerous condition for carcinoma colon is
A) Familial polyposis
B) Hamartomatous polyps
C) Juvenile polyps
D) Hyperplastic polyps
E) None of above
ANSWER: A
The patient is taking enalapril (Vasotec), an angiotensin-converting enzyme (ACE)
inhibitor, for hypertension. Which respiratory side effect should you teach the
patient to expect?
A) Wheezing on exhalation
B) Increased nasal stuffiness
C) Chest pressure or pain
D) Persistent dry cough
E) Non of above
ANSWER: D
The place of 25-hydroxycholecalciferol formation is:
A) Liver
B) Kidney
C) Intestines
D) Pancreas
E) Brain
ANSWER: A
The principal role of the kidneys in the synthesis of the active form of vitamin D is:
A) Conversion of 7-dehydrocholesterol to vitamin D3
B) Absorption of vitamin D2
C) Conversion of vitamin D to 25-OH vitamin D
D) Absorption of vitamin D3
E) Conversion of 25-OH vitamin D to 1,25-(OH)2 vitamin D
ANSWER: E
The results of the glucose tolerance test: a fasting serum glucose is 6,5 mmol/l, 2-
hour postprandial serum glucose is 14,6 mmol/l. What is your diagnosis?
A) Normal.
B) Impaired glucose tolerance.
C) Impaired fasting glycemia.
D) Diabetes mellitus.
E) Neither.
ANSWER: D
The shortest period of the action is present in such group of preparations as:
A) Thiazolidinediones
B) Biguanides
C) Sulfonilureas
D) Acarbose
E) Glinides
ANSWER: E
The triad originally described by Zollinger-Ellison syndrome is characterized by:
A) Peptic ulceration, gastric hypersecretion, non beta cell tumour
B) Peptic ulceration, gastric hypersecretion, beta cell tumor
C) Peptic ulceration, achlorhydria, non beta cell tumour
D) Peptic ulceration, achlorhydria, beta cell tumour
E) None of above
ANSWER: A
The true statement of tertiary hyperparathyroidism is:
A) Primary hyperparathyroidism with decrease Ca+ level
B) Secondary hyperparathyroidism with chief cell adenoma
C) Secondary hyperparathyroidism following intestinal malabsorption
D) Metastasis with normal phosphate level
E) Secondary hyperparathyroidism following chronic renal failure
ANSWER: B
Thrombocytopenia, neutropenia are seen in
A) anemias of chronic disease
B) hemolytic anemias
C) anemia of acute hemorrhage
D) aplastic anemias
E) iron deficiency anemia
ANSWER: D
Thyroid carcinoma associated with hypocalcemia is
A) Follicular carcinoma
B) Medullary carcinoma
C) Anaplastic carcinoma
D) Papillary carcinoma
E) All of them
ANSWER: B
To remove onset of tetany you will order
A) Calcium chloride
B) Diuretics
C) Prednisolone
D) Potassium citrate
E) Magnesium sulphate
ANSWER: A
Toxic megacolon is most commonly associated with
A) Ulcerative colitis
B) Crohn's disease
C) Whipple's disease
D) Reiter's disease
E) None of above
ANSWER: A
Treatment of choice for aplastic anemia is:
A) Methotrexat
B) ampicillini
C) chloramphenicol
D) Bone marrow transplantation
E) Plasmaphoresis
ANSWER: D
Treatment of gastric ulcer disease
A) atropine is less effective than H2 receptor blockers
B) atropine is more effective than H2 receptor blockers
C) atropine and H2 receptor blockers are equally effective
D) atropine and H2 receptor blockers are not effective
E) Omeprazol is not used
ANSWER: A
Treatment of neonatal idiopathic thrombocytopenia is
A) Azathioprin
B) Dexamethasone
C) Immunoglobulins
D) Platelets transfusion
E) Splenectomy
ANSWER: C
True about idiopathic thrombocytopenic purpura is:
A) Increase megakaryocytes in bone marrow
B) Frequent joint haemorrhage
C) Most common in children
D) Males commonly affected
E) VIII factor deficiency
ANSWER: A
True about Ulcerative Colitis with malignancy
A) Is related to duration of ulcerative colitis
B) Is related to disease activity
C) It has a better prognosis
D) Malignancy is more in anorectal ulcerative colitis
E) None of above
ANSWER: A
True statement about Hypercalcemia :
A) Treatment of the primary cause is effective
B) Malignancy does not produce hypercalcemia
C) i.v. fluid with Furosemide is given
D) Pamidronate is not effective.
E) i.v. fluid with calcium chloride is effective
ANSWER: C
What is a possible level of platelets in the patient with Acute Lymphoblastic Leukemia?
A) 100.000 – 150.000/L
B) Over 200.000/L
C) 250.000– 300.000/L
D) Below 300.000/L
E) Absence of Pl
ANSWER: A
What is drug of choice for ulcerative colitis :
A) 5-amino salicylic acid
B) Prednisolone
C) Mercaptopurine
D) Salazopyrine
E) None of above
ANSWER: A
What is not true about blind loop syndrome?
A) Surgery is almost always required to correct small bowel syndrome
B) Megaloblastic anemia is commonly seen
C) It manifets as diarrhoea, weight loss and deficiency of fat soluble vitamins.
D) Broad spectrum antibiotics are the treatment of choice
E) None of above
ANSWER: A
What is not true about the immune mechanism in the small bowel?
A) Ig A acts by activating the complement pathway
B) Intestine contains more than 70% of IgA producing cells in the body
C) Ig A is produced by plasma cells in the lamina propria
D) Approximately 60% of the lymphoid cells are T cells
E) None of above
ANSWER: A
What is the Acute Lymphoblastic Leukemia?
A) Disorder of the blood-forming tissue (white cells)
B) Disorder of the blood-forming tissue (red cells)
C) Leukocytosis
D) Increasing of blood pressure
E) Decreasing of blood pressure
ANSWER: A
What is the best marker for prognosis of the multiple myeloma presents with bony
lesions:
A) Serum calcium level
B) Beta 1a microglobulin
C) Beta 1 microglobulin
D) Bone marrow plasma cell
E) Number of lytic lesions
ANSWER: D
What is the most necessary in order to diagnose bronchial asthma:
A) To notice the asthmatic attack of dyspnea
B) To find allergic antibodies
C) To find the presence of expiration dyspnea
D) To find the presence of signs of obstructive respiratory failure
E) To find the presence of eosinophylia in blood
ANSWER: A
What is true of haemophilia:
A) Increased PTT
B) Platelets level decreased
C) Increased factor VIII
D) Increased factor IX
E) BT decreased
ANSWER: A
What is true regarding congenital hypertrophic pyloric stenosis:
A) Hypochloremic alkalosis
B) More common in girls
C) Hellers myotomy is the procedure of choice.
D) Most often manifests at birth
E) None of above
ANSWER: A
Which among the following diseases may be causative agent for B12 deficiency anaemia?
A) Chronic gastritis of type B
B) Peptic gastric ulcer
C) Chronic gastritis of type A
D) Chronic pancreatitis
E) Chronic cholecystitis
ANSWER: C
Which amongst the following is the earliest indicator of hypothyroidism?
A) Decreased serum T4
B) Decreased serum T3
C) Increased relaxation phase of deep tendon reflexes
D) High serum TSH
E) High serum TRH
ANSWER: D
Which changes of a tongue are typical for vitamin B12 deficiency anaemia?
A) Geographic tongue
B) Red
C) Coated
D) Clean
E) Swollen
ANSWER: A
Which does not predispose to Carcinoma stomach?
A) Low Nitrate consumption
B) Salted meat and fish
C) Low fat and protein diet
D) HIgh Complex carbohydrate consumption
E) All of above
ANSWER: A
Which drug is mostly used for the treatment of chronic lymphatic leukemia?
A) Prednisone
B) Chlorambucil
C) Methotrexate
D) Myleran
E) Ampicilin
ANSWER: B
Which drug is not effective against H. pylori:
A) Erythromycin
B) Metronidazole
C) Amoxicillin
D) Colloidal Bismuth
E) None of above
ANSWER: A
Which from the following symptoms are characteristic for anaemia?
A) Petechia
B) Cyanosys
C) Pallor of skin and mucous membranes
D) Edema
E) Hyperpigmentation
ANSWER: C
Which index can differentiate factitious hyperinsulinemia from insulinoma:
A) C – peptides
B) Insulin antibodies
C) Serum glucose levels
D) None
E) All of them
ANSWER: A
Which is not a feature of multiple myeloma?
A) Cutaneous nodules
B) Bony lesions
C) Renal failure
D) 'M' Spike
E) Hypercalciemia
ANSWER: A
Which is not a paraneoplastic syndrome for Hepatocellular Carcinoma
A) Hyperglycemia
B) Hypoglycemia
C) Erythrocytosis
D) Hypercalcemia
E) None of above
ANSWER: A
Which of the following can help to put a diagnosis of subclinical hypothyroidism for a
patient?
A) Low thyroid hormone levels but no symptoms
B) Classic symptoms of hypothyroidism but normal thyroid function test results
C) Low free T4 levels but normal serum thyroid-stimulating hormone (TSH) level
D) Low serum TSH but normal free T4 levels
E) Increased serum TSH but normal free T4 levels
ANSWER: E
Which of the following conditions causes delayed, deep tissue-type bleeding?
A) Uremia
B) Hemophilia A
C) Therapy with aspirin
D) von Willebrand's disease (vWD)
E) Idiopathic thrombocytopenic purpura (ITP)
ANSWER: B
Which of the following conditions is LEAST likely to be associated with a low serum
25(OH) vitamin D level?
A) Dietary deficiency of vitamin D
B) Chronic severe cholestatic liver disease
C) Sedentary life-style
D) Chronic renal failure
E) High-dose glucocorticoid therapy
ANSWER: D
Which of the following has the weakest association with Hepatocellular Carcinoma (HCC)
A) oral contraceptives
B) Hepatitis C
C) Hepatitis B
D) Smoking
E) None of above
ANSWER: A
Which of the following hepatitis viruses have significant perinatal transmission
A) Hepatitis B virus
B) Hepatitis C virus
C) Hepatitis E virus
D) Hepatitis A virus
E) None of above
ANSWER: A
Which of the following is a feature of nephrocalcionsis:
A) Primary hyperparathyroidism
B) Medullary cystic kidney
C) Vitamin C intoxication
D) Pseudohypoparathyroidism
E) Primary hypothyroidism
ANSWER: A
Which of the following is False regarding H.Pylori infection
A) With chronic infection urease breath test become negative
B) H.Pylori infection remain lifelong if untreated
C) Endoscopy is diagnostic
D) Toxigenic strains usually causes ulcer
E) None of above
ANSWER: A
Which of the following is not a congenital abnormality associated with Juvenile polyps
A) Macrocephaly
B) Meckel's diverticulum
C) Malrotation
D) Mesenteric lymphangioma
E) None of above
ANSWER: A
Which of the following is NOT seen in Paroxysmal Nocturnal Hemoglobinuria:
A) Thrombosis
B) Hemosiderinuria
C) Increased haptoglobin
D) Thrombocytopenia
E) Anemia
ANSWER: C
Which of the following is NOT the contraindication for sulfonilureas?
A) Diabetic with excessive weight
B) Gestation diabetes
C) Diabetic with progressive weight loss
D) Type 2 DM, Non-proliferative rethinopathy
E) Pregnancy and lactation
ANSWER: D
Which of the following is not true about gastric lymphoma?
A) MALT lymphoma is the commonest variety.
B) Peak incidence of lymphomas is seen in 6th-7th decade
C) Endoscopy usually reveals gastritis like picture or gastric ulcer.
D) Stomach is the most common organ in the gi system which is involved in Lymphoma
E) All of above
ANSWER: A
Which of the following is not true about Pneumatosis intestinalis of small bowel?
A) Operative Procedures are required in most of the cases
B) Most common location is subserosa in the jejunum
C) It is seen equaly and males and females
D) It is associated with COPD and immunodeficiency states
E) None of above
ANSWER: A
Which of the following is not true for malignancy of Familial Adenomatous Polyposis
A) Astrocytomas
B) Thyroid
C) Adrenals
D) Hepatoblastomas
E) None of above
ANSWER: A
Which of the following is not used in treatment of leukemia?
A) Steroid
B) Pentostatin
C) Splenectomy
D) Alpha-interferon
E) Alendronic acid
ANSWER: E
Which of the following is the most common cause of death in Crohn's disease of small
bowel
A) Malignancy
B) Sepsis
C) Electrolyte Disorders
D) Thromboembolic Phenomenon
E) None of above
ANSWER: A
Which of the following is used in the treatment of the thyroid malignancy:
A) I131
B) I125
C) 99Tc
D) P32
E) MIBG
ANSWER: A
Which of the following may be seen in multiple myeloma?
A) Increased Alkaline phosphatase
B) Decreased IgA
C) Hypercalcemia
D) Hypouricemia
E) Fat bone marrow
ANSWER: C
Which of the following oral anti-diabetic drugs can be used in patients with renal
failure:
A) Tolbutamide
B) Chlorpropamide
C) Gliquidone
D) Glipizide
E) Metformin
ANSWER: C
Which of the following produces calcitonin?
A) A (alpha) cells
B) B (beta) cells
C) C (parafollicular) cells
D) D (delta) cells
E) F cells
ANSWER: C
Which of the following statements about peptic ulcer disease is true :
A) The incidence of complications has remained unchanged
B) Helicobacter pylori eradication increases the likelihood of occurrence of
complications.
C) The incidence of Helicobacter pylori reinfection in India is very low.
D) Helicobacter pylori eradication does not alter the recurrence ratio.
E) None of above
ANSWER: A
Which of the following structures do not form the external part of anal canal
mechanism?
A) Internal Sphincter
B) Levator Ani
C) Pubo rectalis
D) External Sphincter
E) None of above
ANSWER: A
Which of the following structures does not form the portal triad
A) Hepatic vein
B) Portal Vein
C) Hepatic Artery
D) Bile Duct
E) None of above
ANSWER: A
Which of these is true regarding CML?
A) Size of splenomegaly indicates prognosis
B) Phagocytic activity of WBC is reduced
C) Sudan black stain is specific for myeloblast
D) Myeloblast, granuloblast and lymphoblast become PH chromosome +–ve following
remission
E) Leukocytosis
ANSWER: A
Which one of the following is associated with increased bone resorbtion?
A) Estrogens
B) Calcitonin
C) Bisphosphonates
D) Hyperparathyroidism
E) Hypothyroidism
ANSWER: D
Which patient is at greatest risk for developing a “community-acquired” pneumonia?
A) The 40-year-old first-grade teacher
B) The 60-year-old smoker who is also an alcoholic
C) The 75-year-old with exercise-induced wheezing
D) The 35-year-old aerobics instructor who skips meals and eats only vegetables
E) None of above
ANSWER: B
Which signs are associated with tetany EXCEPT:
A) Chvostek’s sign
B) Trousseau's sign
C) Erb's sign
D) All of them
E) None above them
ANSWER: D
Which statement is true in diabetes mellitus type 2?
A) Insulinitis of beta- cells
B) Hyalinization of beta- cells
C) Atrophy of beta cells
D) Intact beta- cells
E) Degeneration of beta cells
ANSWER: D
Which test can you recommened as the most indicative of average recent blood glucose
levels:
A) Fasting serum glucose level
B) Random serum glucose level
C) Oral glucose tolerance test
D) Serum level of hemoglobin A1C
E) Urine glucose concentration
ANSWER: D
Which type diabetes is HLA associated:
A) Type 1 diabetes
B) Type 2 diabetes
C) Tropic diabetes
D) Gestation diabetes
E) All of them
ANSWER: A
With which of the following theophylline has an antagonistic interaction?
A) Histamine receptors.
B) Bradykinin receptors.
C) Adenosine receptors.
D) Imidazoline receptors.
E) Beta-adrenoreceptors
ANSWER: C
Zollinger-Ellison syndrome is characterized by all of the following except
A) Massive HCL in response to histamine injection
B) Recurrent duodenal ulcer
C) Severe diarrhea
D) Post bulbar ulcer
E) None of above
ANSWER: A
A 64-year-old woman presents with a 1-year history of pain in her thumbs. A focused
examination reveals squaring at the base of both first digits, worse on the right, and
pain on pressure over the first carpal metacarpal joints. She also has non tender bony
overgrowth at the distal interphalangeal joints. The patient says that her mother had
the same fingers and she worries that she will become crippled. Choose the most likely
diagnosis from the below list of options.
A) Ankylosing spondylitis
B) Rheumatoid arthritis
C) Osteoarthritis
D) Gout
E) Psoriatic arthritis
ANSWER: C
A 66-year-old man has had a several-week history of fatigue and ankle swelling. His
appetite is normal, but his body weight has increased 4.5 kg (10 Ib). There is no
history of exertional dyspnea, paroxysmal nocturnal dyspnea, or orthopnea. He has had
hypertension for 10 years, and diabetes was diagnosed 4 months ago. He takes
hydrochlorothiazide, 25 mg/d, and metoprolol, 50 mg/d. On examination, the patient is
alert and in no distress. Blood pressure is 120/75 mm Hg, pulse rate is 82/min, and
temperature is 36.9 °C (98.4 °F). Skin examination is normal, and he has no
jugulovenous distention. Thyroid examination is normal. The lungs are clear. Cardiac
examination shows regular sinus rhythm, with no murmur or gallop. The abdomen is
slightly protuberant with shifting dullness, but no hepatosplenomegaly is noted. There
is 4+ edema of the legs and thighs and 1+ edema of the sacrum. Laboratory studies:
Complete blood count Normal Hemoglobin A1c 7.4%. Blood urea nitrogen - 20 mg/dL. Serum
creatinine - 0.9 mg/dL. Serum sodium - 141 meq/L. Serum chloride - 104 meq/L. Serum
potassium - 4.3 meq/L. Serum bicarbonate - 30 meq/L. Serum total protein - 4.4 g/dL.
Serum albumin - 1.7 g/dL. Serum cholesterol - 376 mg/dL. 24-hour urine protein 8.5 g.
Urinalysis pH 5.5; specific gravity 1 .020; protein 4+, trace hemoglobin Urine
microscopy shows many granular and hyaline casts and oval fat bodies. Monoclonal
protein is identified on urine immunoelectrophoresis. Plasma immunoelectrophoresis
shows an IgG level of 452 mg/dL, IgA of 284 mg/dL, 1gM of 122 mg/dL, K of 550 mg/dL,
and 2 of 193 mg/dL with a homogenous M band. What renal disease is most likely in this
patient?
A) Idiopathic membranous glomerulopathy
B) Diabetic nephropathy
C) Focal and segmental glomerulosclerosis
D) Hypertensive nephropathy
E) AL amyloidosis
ANSWER: E
A 68-year-old man is evaluated because of a 3-year history of bilateral knee pain and
low back pain. He has some stiffness for approximately 15 minutes when she awakens in
the morning, and during the afternoon her pain is worse. On physical examination, he
has slight swelling and tenderness to pressure of the distal interphalangeal joints 2-
5 on both hands. There is slight crepitus with motion of the right knee. Which of the
following treatment do use?
A) Methotrexat
B) Glucosamine
C) Colchicines
D) Tetracycline
E) Allopurinol
ANSWER: B
A 71-year-old man had non-Hodgkins lymphoma diagnosed 4 months ago. Treatment included
chemotherapy with rituximab and cyclosphosphamide, prednisone, and vincristine and
doxorubicin, followed by radiation therapy. He has had persistent anemia. He has had
hypertension for 15 years and prostatic hypertrophy, treated with doxazosin, for 10
years. He presents to the emergency department with polyuria, weakness, and lassitude.
On physical examination, blood pressure is 124/78 mm Hg, with no orthostatic changes;
pulse rate, 96/min; respiratory rate, 18/min; and temperature 36.8 °C (98.2 °F).
Mucous membranes are moist. There is no neck vein distention or hepatojugular reflux.
The cardiac examination is normal, and the chest is clear. The abdomen is benign. No
lower extremity edema is present. The electrocardiogram is normal. Hematocrit is 31%,
and leukocytosis is present, with a normal platelet count. The serum creatinine
concentration, previously normal, is now 2.4 mmol/L. Urinalysis shows a pH of 6.0 and
1 + proteinuria, but no hematuria or ketonuria. No formed elements appear on
microscopic examination. Urine sodium and osmolality values have been requested. What
is the most important next test to determine the reason for decreased renal function
in this patient?
A) Calculate the ratio of blood urea nitrogen to creatinine
B) Perform renal ultrasonography
C) Perform renal scanning
D) Administer 1.5 L of normal saline as a fluid trial
E) Perform renal biopsy
ANSWER: B
A criterion for the diagnosis of asthma is:
A) 15 % reversibility in spirography
B) 5% reversibility in spirography
C) 10% reversibility in spirography
D) X-ray changes
E) Sputum changes
ANSWER: A
A drug is to be delivered by a nebuliser. The size of a droplet for its humidification
(in mcm) is:
A) less than 5
B) 5-10
C) 10-15
D) 15-20
E) All of the above
ANSWER: A
A man aged 60 yrs has history of ischemic heart disease and atherosclerosis. He
presents with abdominal pain and maroon stools: likely diagnosis here is:
A) Acute mesenteric ischemia
B) Acute intestinal obstruction
C) Peritonitis
D) Appendicitis
E) None of above
ANSWER: A
A patient was operated for colonic carcinoma and later a solitary nodule was found in
the liver. Treatment of choice is:
A) Surgery
B) Radiation
C) Chemotherapy
D) Conservative treatment
E) None of above
ANSWER: A
A patient with BMI > 30 (kg/m2), serum glucose 24 mmol/L, urinary ketones ++++, he
requires:
A) Insulin
B) Glibenclamide
C) Metformin
D) Phenformin
E) Oral bicarbonate
ANSWER: A
A patient with BMI 34, serum glucose 26 mmol/l, urinary ketones 4+ requires:
A) Insulin
B) Glibenclamide
C) Metformin
D) Glimepiride
E) Acarbose
ANSWER: A
A patient with DKA has deep sighing respiration. The main cause of Kussmaul breathing
is:
A) Hyperglycemia
B) Metabolic acidosis
C) Dehydration
D) Hypokalemia
E) Ketonemia
ANSWER: B
Above the cavity of abscess it is possible to hear such breathing
A) Bronchial
B) Vesicular
C) Unclear
D) Hard
E) Weakened
ANSWER: A
Acid secretion phase, following food intake, defined by stimulation of mechanical and
chemical gastric wall receptors by luminal contents:
A) cephalic phase
B) gastric phase
C) intestinal phase
D) All of the above
E) None of above
ANSWER: B
Activation of these/this receptor(s) on basolateral parietal cells inhibit(s) gastric
acid secretion
A) prostaglandins
B) gastrin
C) histamine
D) acetylcholine
E) all of the above
ANSWER: A
Acute abscess and gangrene of lungs belong to
A) Pyogenic diseases of lungs
B) Chronic non-specific diseases of lungs
C) Congenital pathology of lungs
D) Obstructive diseases of lungs
E) Systemic diseases
ANSWER: A
Acute lymphocytic leukemia (ALL)
A) A 4-year-old patient with pancytopenia and circulating blasts
B) A 60-year-old patient with pancytopenia and circulating blasts
C) A 20%-30% long-term survival
D) A patient with bleeding and infection
E) A patient with gum and skin infiltration
ANSWER: A
Acute myelogenous leukemia (AML)
A) A 4-year-old patient with pancytopenia and circulating blasts
B) A 40-year-old patient with pancytopenia and circulating blasts
C) A 70%-80% long-term survival
D) A patient with bleeding and infection
E) A patient with gum and skin infiltration
ANSWER: B
Adenocarcinoma of esophagus is commonly found in :
A) Barret's oesophagus
B) Achalasia acardia
C) Plummer vinson syndrome
D) Chronic smoking
E) None of above
ANSWER: A
After undergoing surgery, for Carcinoma of colon a patient developed single liver
metastasis of 2cm. What you do next:
A) Resection
B) Chemo radiation
C) Acetic acid injection
D) Radio frequency ablation
E) None of above
ANSWER: A
All are precancerous for carcinoma colon except
A) carotene
B) Bile acids
C) Fats
D) crohn's disease
E) None of above
ANSWER: A
All are true about pseudopancreatic cyst of
A) Most common site is in head of pancreas
B) Presents as an abdominal mass
C) Serum amylase is increased
D) Common after acute pancreatitis
E) None of above
ANSWER: A
All are true regarding Helicobacter pylori except:
A) Less prevalent in developing countries
B) Toxicogenic strains usually causes ulcers
C) Urea breath test is positive
D) Gram negative organism
E) None of above
ANSWER: A
All are true regarding paroxysmal nocturnal haemoglobinuria, except:
A) Direct antiglobulin test (direct Coombs' test) is negative
B) Red urine
C) Presence of antibodies
D) Reticulocytosis
E) Raised bilirubin
ANSWER: C
All are true regarding Zollinger Ellison syndrome, except:
A) Decreased ratio of BAO to MAO
B) Recurrence after operation
C) Hypergastrinemia
D) Diarrhoea
E) None of above
ANSWER: A
All are used in treatment of Helicobactor pylori, EXCEPT:
A) Cisapride
B) Colloid bismuth
C) Clarithromycin
D) Metronidazole
E) Lansoprazol
ANSWER: A
All clinical signs can be seen in ophthalmic Grave's disease EXCEPT:
A) Lid retraction
B) Frequent blinking
C) Poor convergence
D) Upper lid "lad" on down gaze
E) Wide opened eyes
ANSWER: B
All medicines listed below are used in bronchial asthma, except:
A) Morphine
B) Salbutamol
C) Aminophylline
D) Steroids
E) Ipratropium
ANSWER: A
All of the following are principles of treatment of DM, EXCEPT:
A) Achievement and maintenance of normal or reasonable body weight.
B) Normalization of metabolism and achievement of DM compensation.
C) Education of the patients and self – control.
D) Maintenance (preservation) of working capacity.
E) Prophylaxis of acute and chronic complications.
ANSWER: C
All of the following are true regarding a patient with acid peptic disease except
A) Misoprostol is drug of choice in pregnant patients
B) Duodenal ulcer is preventable by the use of single night time H2 blockers
C) Omeprazole may help ulcers refractory to H2 blockers
D) Misoprostol is the drug of choice in patients on NSAIDS
E) None of above
ANSWER: A
All of the following are used in the management of uncomplicated thyrotoxicosis EXCEPT:
A) Propylthiouracil
B) Vasodilators
C) Radioactive iodine
D) Lugol's iodine
E) Propranolol
ANSWER: B
All of the following are useful for treating acute bronchial asthma except:
A) Sodium chromoglycate inhalation
B) 100% Oxygen
C) Hydrocortisone infusion
D) IV aminophylline
E) All of the above
ANSWER: A
All of the following drugs are useful in the treatment of a patient with acute
bronchial asthma except:
A) Montelukast
B) Ipratropium
C) Salbutamol
D) Hydrocortisone
E) All of the above
ANSWER: A
All of the following features are seen in the viral pneumonia except:
A) Presence of interstitial inflammation
B) Predominance of alveolar exudates
C) Bronchiolitis
D) Multinucleate giant cells in the bronchiolar wall
E) All features are present
ANSWER: B
All of the following statements about Leukotriene modifiers in the management of
bronchial asthma are true except:
A) May be used for acute asthma
B) May be used for exercise induced asthma
C) Zileuton is Leukotriene modifier
D) May uncover Churg Strauss syndrome
E) All of the above
ANSWER: A
All of the following statements about Pseudopancreatic cysts are true except:
A) Percutaneous aspiration is treatment of choice
B) Cystojejunostomy is treatment of choice
C) Serum amylase levels are increased
D) Presents as an epigastric mass
E) None of above
ANSWER: A
All of the following statements are true about sickle cell disease except
A) Vaso-occlusive crisis
B) Aplastic Crisis
C) Hypertensive crisis
D) Sequestration Crisis
E) Hyper hemolytic crisis
ANSWER: C
All of the following statements stand true for telangiectasia of colon except:
A) 50% involve rectum
B) May be seen in person more than 60 years of age
C) Common site is caecum
D) May be seen in person less than 40 years of age
E) None of above
ANSWER: A
All of these signs may be present in patient with pituitary insufficiency, EXCEPT:
A) Hypotension.
B) Hyperpigmentation.
C) Weight loss.
D) Hypogonadism.
E) Hypothyroidism.
ANSWER: B
All signs are associated with tetany EXCEPT:
A) Chvostek’s sign
B) Trousseau's sign
C) Erb's sign
D) Cole's sign
E) None above them
ANSWER: D
All statements are true in hyperparathyroidism EXCEPT:
A) May cause hypercalcemia
B) Commonly occurs after thyroidectomy
C) Solitary adenoma in parathyroid is a common cause
D) Nephrolithiasis is common
E) Frequent fractures
ANSWER: B
All the following are causes of Acute Pancreatitis except:
A) Hemochromatosis
B) Alcohol
C) Gall stones
D) Hypercalcemia
E) None of above
ANSWER: A
All the following can be indications for parathyroidectomy in patients with
hyperparathyroidism EXCEPT
A) Advanced age
B) Kidney stones
C) Osteoporosis
D) Calcium level &amp;amp;gt;2.9 mmol/L
E) Decreased creatinine clearance
ANSWER: A
All the following indicates early gastric cancer except
A) Involvement of mucosa, submucosa and muscularis
B) Involvement of mucosa and submucosa
C) Involvement of mucosa
D) Involvement of mucosa, submucosa and adjacent lymph nodes
E) None of above
ANSWER: A
All the following may be the causes of hemorrhagic pleural effusion except:
A) Congestive heart failure
B) Pulmonary embolism
C) Lung cancer
D) Tuberculosis
E) Severe chest trauma
ANSWER: A
Among the following, the most common site for Leiomyoma is
A) Stomach
B) Small Intestine
C) Duodenum
D) Colon
E) None of above
ANSWER: A
Antacid: effective, but associated with systemic alkalosis
A) sodium bicarbonate
B) calcium carbonate
C) magnesium hydroxide
D) aluminum hydroxide
E) All of above
ANSWER: A
Antacid: loose stools
A) magnesium hydroxide
B) aluminum hydroxide
C) All of above
D) None of above
E) sodium bicarbonate
ANSWER: A
Antacid: most likely to cause systemic phosphate depletion:
A) aluminum hydroxide
B) calcium carbonate
C) magnesium hydroxide
D) sodium bicarbonate
E) All of above
ANSWER: A
Antacid(s)
A) All of above
B) magnesium hydroxide
C) calcium carbonate
D) aluminum hydroxide
E) None of above
ANSWER: A
Aplastic anemia can be caused by all except:
A) ampicillini
B) chloramphenicol
C) quinine
D) phenytoin
E) carbamazepine
ANSWER: A
Aspirin sensitive asthma is associated with:
A) Nasal polyps
B) Extrinsic asthma
C) Urticaria
D) Obesity
E) All of the above
ANSWER: A
Attack of bronchial asthma differs from heart asthma by such sign:
A) By expiration dyspnea
B) By sudden development of attack
C) By orthostatic position of the patient
D) By duration of the attack
E) By inspiration dyspnea
ANSWER: A
Barrett's oesophagus is:
A) Lower oesophagus lined by columnar epithelium
B) Upper oesophagus lined by columnar epithelium
C) Lower oesophagus lined by ciliated epithelium
D) Lower oesophagus lined by pseudostretifide epithelium
E) None of above
ANSWER: A
Basal acid secretion is inhibited by:
A) all of the above
B) cimetidine (Tagamet)
C) nizatidine (Axid)
D) ranitidine (Zantac)
E) None of above
ANSWER: A
Basal acid secretion is stimulated by:
A) histamine
B) cimetidine (Tagamet)
C) nizatidine (Axid)
D) ranitidine (Zantac)
E) famotidine (Pepcid)
ANSWER: A
Basolateral parietal cell membranes contained these receptor types:
A) All of above
B) acetylcholine
C) prostaglandins
D) histamine
E) gastrin
ANSWER: A
Bence Jones protein may be found in the urine in
A) Chronic lymphocytic leukemia
B) Waldenstroms macroglobulinemia
C) Rheumatic fever
D) Multiple myeloma
E) AML
ANSWER: D
Best antithyroid drug to be used to pregnancy is:
A) Carbimazole
B) Thiamazole
C) Propylthiouracil
D) Radiocative I
E) Non of above
ANSWER: C
Best method for detecting of the minimal bronchiectasis is:
A) CT scan
B) Bronchogram
C) Radionuclide lung scan
D) Chest X-ray
E) Nothing of the above
ANSWER: A
Best position to reveal the small pleural effusions on chest X-ray is
A) Lateral decubitus view
B) AP view
C) PA view
D) Lateral view
E) All of the above
ANSWER: A
Best test for Small intestine malabsorption of carbohydrates is :
A) D-Xylose test
B) Shilling test
C) Lund meal test
D) Follacin test
E) None of above
ANSWER: A
Bilateral malignant pleural effusion is most often seen in:
A) Cancer of lung
B) Cancer of breast
C) Mesothelioma
D) Lymphoma
E) Nothing of the above
ANSWER: A
Bilateral pleural effusion is seen in:
A) Congestive cardiac failure
B) Nephritic syndrome
C) Constrictive pericarditis
D) All of the above
E) Nothing of the above
ANSWER: D
Bilateral subtotal of thyroidectomy for Graves’ disease most likely can cause
hypoparathyroidism as a result as:
A) Loss of thyrocalcitonin
B) Reduction in parathyroid stimulating hormone
C) Removal of the four parathyroids
D) Infarction of the parathyroids
E) Secondary hypopituitarism
ANSWER: D
Blocking of these structures can worsen the asthma
A) ?2 -adrenoreceptors
B) ?-adrenoreceptors
C) Mast cells
D) Neutrophils
E) Eosinophils
ANSWER: A
Bone marrow transplant is the treatment of choice in all, except:
A) Aplastic anemia
B) AML in 1st remission
C) ALL in second remission
D) ITP
E) Lympoma
ANSWER: D
Bronchial asthma is associated with raised levels of
A) Leukotrienes
B) Thromboxane
C) Antinuclear antibodies
D) Corticosteroids
E) All of the above
ANSWER: A
Bronchiectasis is the most common in such lung lobe:
A) Left lower
B) Right middle
C) Right upper
D) Left upper
E) Nothing of the above
ANSWER: A
Causes of haemorrhagic pleural effusion are all except:
A) Bronchial adenoma
B) Pulmonary infarction
C) Mesothelioma
D) Tuberculosis
E) Nothing of the above
ANSWER: A
CBC revealed a WBC of 72,000 and a differential that was 83% lymphocytes. What is the
MOST likely diagnosis?
A) Acute lymphocytic leukemia
B) Chronic lymphocytic leukemia
C) Acute prolymphocytic leukemia
D) Chronic prolymphocytic leukaemia
E) Lymphoma
ANSWER: B
Characteristic(s) of gastric mucosal acid secretion:
A) all of the above
B) from parietal cells founded in mucosal glands of fundusof the stomach
C) stimulated by muscarinic cholinergic system (parietal cell innervation)
D) oxidative dephosphorylation dependent
E) None of above
ANSWER: A
Charcot’s joint in diabetes is NOT seen in:
A) None of them
B) Knee
C) Hip
D) Tarsal
E) Elbow
ANSWER: E
Chose correct answer. Zollinger-Ellison syndrome -
A) gastrin-secreting islet cell tumor
B) it is associated with Helicobacter pylori
C) it is associated withLamblia
D) All of the above
E) None of above
ANSWER: A
Common sites for cushing ulcers include all of the following except:
A) Distal duodenum
B) Stomach
C) Is1 part of duodenum
D) Esophagus
E) None of above
ANSWER: A
Common X-ray finding of Staphylococcal pneumonia:
A) Bilateral hilar lymphadenopathy
B) Involvement of lower lobes
C) Dense homogenous consolidation
D) Visible air bronchogram
E) Nothing of the above
ANSWER: A
Commonest site of peptic ulcer is:
A) 1st part of Duodenum
B) Middle part of duodenum
C) Distal 1/3 of stomach
D) Pylorus of the stomach
E) None of above
ANSWER: A
Consider the following: Serum iron (1), MCV (2), serum total iron binding capacity (3),
serum ferritin (4). Which of these decreased in iron deficiency anaemia?
A) 1 and 2
B) 1, 3 and 4
C) 1, 2 and 4
D) 1, 2, 3 and 4
E) 2,3 and 4
ANSWER: C
Constipation is caused by
A) aluminum hydroxide
B) magnesium hydroxide
C) All of above
D) None of above
E) sodium bicarbonate
ANSWER: A
Contraindication for glucocorticosteroid therapy of bronchial asthma is:
A) Bleeding from gastric ulcer
B) Chronic persistant hepatitis
C) Exacerbation of respiratory-heart failure
D) Emphysema of lungs
E) Respiratory failure
ANSWER: A
Curschmann's spirals in sputum are seen in:
A) Bronchial asthma
B) Bronchiectasis
C) Chronic bronchitis
D) Wegenersgranulomatosis
E) All of the above
ANSWER: A
Dawn phenomenon refers to:
A) Early morning hyperglycemia followed by hypoglycemia
B) Hypoglycaemia followed by hyperglycemia
C) Early morning hypoglycemia
D) Early morning hyperglycemia
E) High insulin level
ANSWER: D
Deaths from lactic acidosis in diabetes mellitus are associated with therapy:
A) Glibenclamide
B) Gliquidone
C) Chlorpropamide
D) Phenformin
E) Glipizide
ANSWER: D
Decreased maximum mid-expiratory flow rate indicates obstruction in:
A) Small airways
B) Trachea
C) Large airways
D) Trachea &amp;amp;amp; bronchi both
E) Nothing of the above
ANSWER: A
Diabetic ketoacidosis characterized by:
A) Low serum potassium
B) Increased anion gap
C) Metabolic alkalosis
D) Respiratory acidosis
E) Hypoglycemia
ANSWER: B
Diagnostic tests for H. Pylori include all of the following except:
A) Nechyporenco test
B) Rapid urease test
C) Gastric biopsy &amp;amp;amp; Warthin-starry stain
D) Urea-breath test
E) None of above
ANSWER: A
Drug of choice for inhalation in acute asthma not responding to salbutamol is:
A) Ipratropium bromide
B) Atropine sulfate
C) Isopropamide
D) Hyoscine methyl bromide
E) All of the above
ANSWER: A
Drug of choice in asthma with heart disease is
A) Ipratropium
B) Rimiterol
C) Terbutaline
D) Cromolyn sodium
E) All of the above
ANSWER: A
Drug of choice in Zollinger Ellison syndrome is
A) Omeprazole
B) Ranitidine
C) Antacids
D) B-blocker
E) Atropin
ANSWER: A
Drug(s) used in eradication of Helicobacter pylori infection:
A) All of above
B) metronidazole (Flagyl)
C) clarithromycin (Biaxin)
D) bismuth compounds
E) amoxicillin (Amoxil Polymox)
ANSWER: A
During an epidemic of hepatitis E, Fatality is maximum in :
A) Pregnant women
B) Infants
C) Malnourished male
D) Adolescents
E) None of above
ANSWER: A
Early gastric cancer generally indicates
A) Gastric adenocarcinoma confined to the mucosa &amp;amp;amp; submucosa
B) Gastric adenocarcinoma confined to the mucosa
C) Gastric adencarcinoma detected early
D) Gastric adenocarcinoma less than 1 cm In size
E) None of above
ANSWER: A
Endoscopic biopsy from a case of H. pylori related duodenal ulcer is most likely to
reveal
A) Antral predominant gastritis
B) Multifocal atrophic gastritis
C) Acute erosive gastritis
D) Gastric atrophy
E) None of above
ANSWER: A
Epidemiological studies of H. pylori are done by using:
A) Serological markers
B) Urea-breath test
C) Culture
D) Gastric-biopsy urease test
E) None of above
ANSWER: A
Eradication of H. pylori effectiveness for bismuth compounds when using combination
with antibiotics:
A) 95%
B) 20%
C) 50%
D) 80%
E) 5%
ANSWER: A
Example(s) of "aggressive factors" in peptic ulcer disease:
A) pepsin
B) bicarbonate
C) prostaglandins
D) neither
E) All of above
ANSWER: A
Example(s) of "agressive factor(s)" in peptic ulcer disease:
A) Helicobacter pylori
B) prostaglandins
C) gastric mucus
D) bicarbonate
E) all of the above
ANSWER: A
Example(s) of "protective factor(s)" in peptic ulcer disease:
A) all of the above
B) prostaglandins
C) gastric mucus
D) bicarbonate
E) None of above
ANSWER: A
Examples of H2 (histamine receptor Type II) antagonists:
A) All of above
B) ranitidine (Zantac)
C) nizatidine (Axid)
D) famotidine (Pepcid)
E) cimetidine (Tagamet)
ANSWER: A
Exercise induced asthma is not precipitated by :
A) Swimming in hot water
B) High altitude climb and exercises
C) Cycling in cold weather
D) Swimming in cold water
E) All of the above
ANSWER: A
FEV1/FVC is reduced in case of:
A) Asthma
B) Pleural effusion
C) Lung fibrosis
D) All of the above
E) Nothing of the above
ANSWER: A
Findings in hemophilia A include:
A) Partial thromboplastin time increased
B) Increased clotting time
C) Increased prothrombin time
D) Bleeding time increased
E) Thrombocytopenia
ANSWER: A
Following are causes of unconjugated hyperbilirubinemia, except:
A) Rotor syndrome
B) Large hematoma
C) Hemolytic anemia
D) Megaloblastic anemia
E) Irondeficiency anemia
ANSWER: A
For the patient who has sustained tracheobronchial trauma, which assessment finding
alerts the physician to the possibility of tracheal lacerations?
A) Hypertympanic sound on affected side
B) Subcutaneous emphysema over the trachea
C) Hypotension and decreased capillary refill
D) Deviation of the trachea to the affected side
E) Non of above
ANSWER: B
Haemophilia A is characterised by the presence of following features, except
A) Bleeding into soft tissues
B) Reduced VIII levels
C) Prolonged bleeding time
D) Prolonged partial thromboplastin time
E) Bleeding into muscles &amp;amp;amp; joints
ANSWER: C
Helicobacter pylori is not associated with :
A) Gastric leiomyoma
B) Gastric cancer
C) Peptic ulcer of stomach
D) Peptic ulcer of duodenum
E) None of above
ANSWER: A
Histamine and gastric acid secretion:
A) all of the above
B) release enhanced by increased cholinergic activity
C) most important gastric acid secretion stimulant
D) released from enterochromaffin-like cells
E) None of above
ANSWER: A
How long is the usual course of drug treatment for a patient with active tuberculosis?
A) 7 to 10 days
B) 6 weeks
C) 6 months
D) 2 years
E) Non of above
ANSWER: C
Hypercalcemia may be treated by:
A) IV pamidronate
B) IV fluids + furosemide
C) Glucocorticoids
D) None above them
E) All of them
ANSWER: E
Hypergastrinemia with hypochlorhydria is seen in:
A) Pernicious anemia
B) All of above
C) Zollinger Ellison Syndrome
D) Glucagonoma
E) None of above
ANSWER: A
Hyperosmolar hyperglycemic non-ketonic coma is characterized by such level of glycemia:
A) 55 mmol/1
B) 20 mmol/1
C) 30 mmol/1
D) 5 mmol/1
E) 25 mmol/1
ANSWER: A
Hypoglycemic coma can be caused by:
A) Insulin deficiency
B) Dehydration
C) Extreme activity
D) Metabolic acidosis
E) Hypokalemia
ANSWER: C
Hypoglycemic effect of intermediate acting insulin lasts for:
A) 12-16 hours
B) 20 -30 hours
C) 0.5 – 1.0 hours
D) 4 – 6 hours
E) 6 – 8 hours
ANSWER: A
Hypoglycemic unawareness that occurs in diabetic patients when transferred from oral
hypoglycemic agents to insulin, is due to :
A) Autonomic neuropathy
B) Insulin resistance
C) Allergy
D) Somogi phenomenon
E) Down phenomenon
ANSWER: A
In a case of hypertrophic pyloric stenosis, the metabolic disturbance is
A) Metabolic alkalosis with paradoxical aciduria
B) Metabolic acidosis
C) Respiratory alkalosis
D) Metabolic alkalosis with alkaline urine
E) None of above
ANSWER: A
In a patient only Anti HBsAg is positive in serum, all other viral markers are
negative. It indicates :
A) Immunized person with hepatitis B vaccine
B) Chronic active hepatitis
C) Persistent carrier
D) Acute hepatitis
E) None of above
ANSWER: A
In bronchial asthma glucocorticoids
A) Reduce airway inflammation
B) Act as potent bronchodilators
C) Inhibit degranulation of mast cells
D) Block the action of humoral mediators
E) Nothing of the above
ANSWER: A
In endemic goiter such statement will be true:
A) Investigations invariably show some degree of hypothyroidism
B) There is a clear evidence of an increased incidence of carcinoma of the thyroid
C) Administration of thyroxine will often cause the goitre to shrink
D) Administration of iodine occasionally precipitates hyperthyroidism
E) All of them
ANSWER: E
In gastric outlet obstruction in a peptic ulcer patient, the site of obstruction is
most likely to be
A) Duodenum
B) Antrum
C) Pylorus
D) Pyloric canal
E) None of above
ANSWER: A
In patients with nonketotic hyperosmolar coma the level of glycemia is :
A) Slightly elevated
B) Mildly elevated
C) Moderately elevated
D) May be decreased
E) Grossly elevated
ANSWER: E
In pneumonia due to Mycoplasma all of the following are true except:
A) Bilateral infiltration on chest X-ray
B) Myalgia
C) Dry cough
D) Pleuritic chest pain
E) All of the above
ANSWER: A
In restrictive lung disease :
A) FEV1/FVC is high
B) FVC is high
C) FEV1 is high
D) All of the above
E) Nothing of the above
ANSWER: A
In severe bronchial asthma, it is present:
A) Hyperresonant chest with prolonged expiration
B) Infrequent rhonchi and absent breath sounds
C) Increased fremitus and crackles
D) Decreased fremitus and crepitation
E) All of the above
ANSWER: A
In the time of attack of bronchial asthma in lungs are heard
A) Dry loud wheezes
B) Moist loud wheezes
C) Moist soundless wheezes
D) Voice of pleural friction
E) Crepitation
ANSWER: A
In what specific way does the respiratory system function contribute to acid-base
balance?
A) Prevents excessive loss of hydrogen ions by evaporation
B) Increases the potassium ion content in bronchial secretions
C) Removes carbon dioxide generated as a result of metabolism
D) Maintains body water levels through mucous membrane function
E) Non of above
ANSWER: C
In what specific way does the respiratory system function contribute to acid-base
balance?
A) Prevents excessive loss of hydrogen ions by evaporation
B) Increases the potassium ion content in bronchial secretions
C) Removes carbon dioxide generated as a result of metabolism
D) Maintains body water levels through mucous membrane function
E) Non of above
ANSWER: C
In which case the dose of insulin in stable diabetics can be reduced:
A) Thyrotoxicosis
B) Steroid therapy
C) Pregnancy
D) Chronic renal failure
E) None above them
ANSWER: D
Increased amylase may be seen in all of the following except:
A) Appendicitis
B) Pancreatic pseudocyst
C) Perforated peptic ulcer
D) Ruptured ectopic pregnancy
E) None of above
ANSWER: A
Increased incidence of duodenal ulcer associated with:
A) All of above
B) alcoholic cirrhosis
C) renal transplantation
D) systemic mastocytosis
E) None of above
ANSWER: A
Increased incidence of duodenal ulcer associated with:
A) All of above
B) None of above
C) liver cirrhosis
D) chronic renal failure
E) hyperparathyroidism
ANSWER: A
Increased RBC level may be seen in:
A) Glomerulonephritis
B) Vitamin D excess
C) Cor pulmonale
D) Acute heart failure
E) Pneumonia
ANSWER: C
Insulin resistance is associated with all of the following status, EXCEPT:
A) Acanthosis nigricans
B) Lipodystrophy
C) Gout
D) Obesity
E) Metabolic syndrome
ANSWER: C
Intestinal hypomotility is seen in all the following except:
A) All of the above
B) Parkinsonism
C) Amyloidosis
D) Diabetes
E) None of above
ANSWER: A
Intrinsic factor for absorption of vitamin Bl2 is secreted by the :
A) Parietal cells
B) Peptic cells
C) Chief cells
D) Goblet cells
E) Endothelial cells
ANSWER: A
It is not Antacid
A) Histamine
B) magnesium hydroxide
C) calcium carbonate
D) sodium bicarbonate
E) All of above
ANSWER: A
Location(s) of histamine in gastric mucosa:
A) enterochromaffin-like cells (ECL) + mast cell cytoplasmic granules
B) mast cell cytoplasmic granules
C) enterochromaffin-like cells (ECL)
D) neither
E) are not localized in gastric mucosa
ANSWER: A
Macrophages containing large quantities of undigested and partial digested bacteria in
intestine are seen in
A) Whipple's disease
B) Amyloidosis
C) Immunoapproliferative small instetinal disease
D) Vibro cholerae infection
E) None of above
ANSWER: A
Massive bleeding per rectum in a 70 yr old patient is due to :
A) Diverticulosis
B) Carcinoma colon
C) Colitis
D) Polyps
E) None of above
ANSWER: A
Mechanism of action: cytoprotection, binds to the ulcer base, antibacterial effect:
A) bismuth compounds
B) tetracycline (Achromycin)
C) metronidazole (Flagyl)
D) ranitidine (Zantac)
E) omeprazole (Prilosec)
ANSWER: A
Mechanism of Idiopathic thrombocytopenic purpura is:
A) Vasculitis
B) Antibodies to vascular epithelium
C) Antibodies to platelets
D) Antibody to clotting factors
E) Fungal infectious
ANSWER: C
Mechanism(s) by which somatostatin reduces gastrin release:
A) inhibits parietal cells secretion+ inhibits histamine release by enterochromaffin-
like cells
B) inhibits histamine release by enterochromaffin-like cells
C) inhibits parietal cells secretion
D) stimulates parietal cells secretion
E) None of above
ANSWER: A
Melphalan is used in:
A) Wilm's tumor
B) Lymphoma
C) Multiple myeloma
D) Nephroblastoma
E) ALL
ANSWER: C
Metabolic changes associated with excessive vomiting includes the following:
A) Hypokalemia
B) Hyperchloremia
C) Metabolic acidosis
D) Decreases bicarbonates
E) None of above
ANSWER: A
Microcytic anemia is seen in:
A) Sprue
B) Pernicious anemia
C) Latun infestation
D) Iron deficiency anemia
E) Aplastic anemia
ANSWER: D
Monotherapy effectiveness in eradication of H. pylori-- bismuth compounds
A) 20%
B) 5%
C) 50%
D) 90%
E) 100%
ANSWER: A
More likely to affect the cytochrome P450 drug metabolizing system:
A) cimetidine (Tagamet)
B) All of above
C) nizatidine (Axid)
D) ranitidine (Zantac)
E) famotidine (Pepcid)
ANSWER: A
Most common CNS manifestation of Whipple's disease is :
A) Dementia
B) Supranuclear ophthalmoplegia
C) Seizure
D) Cerebellar ataxia
E) None of above
ANSWER: A
Most common complication of acute pancreatitis is:
A) Pseudocyst
B) Pancreatic abscess
C) Phlegmon
D) Pleural effusion
E) None of above
ANSWER: A
Most common extraintestinal manifestation of Crohn's disease of small bowel is.
A) Erythema Nodosum
B) Ankylosing Spondylitis
C) Iritis
D) Ureteral Obstruction
E) None of above
ANSWER: A
Most common sign of aspiration pneumonitis:
A) Tachypnoe
B) Bronchospasm
C) Cyanosis
D) Crepitations
E) Nothing of the above
ANSWER: A
Most sensitive and specific test for diagnosis of iron deficiency is:
A) Serum ferritin levels
B) Serum iron levels
C) Serum transferrin receptor population
D) Transferrin saturation
E) Hb, Ht
ANSWER: A
Most successful protocol for eradication of H. pylori/treatment of peptic ulcer
disease:
A) therapy using bismuth compounds, Clarythromycin (Flagyl), and amoxicillin (Amoxil
Polymox)
B) bismuth monotherapy
C) bismuth compounds and amoxicillin (Amoxil Polymox)
D) all of the above equally effective
E) None of above
ANSWER: A
Most widely used antacid in treating ulcer disease:
A) combination of aluminum hydroxide and magnesium hydroxide
B) calcium carbonate
C) magnesium hydroxide
D) aluminum hydroxide
E) sodium bicarbonate
ANSWER: A
Mycoplasma infection simulates:
A) Viral pneumonia
B) Pnemuococcal pneumonia
C) Pulmonary oedema
D) Pulmonary infarction
E) Nothing of the above
ANSWER: A
Non-parenteral hepatitis is :
A) Hepatitis E
B) Hep B
C) Hep C
D) Hep D
E) All of above
ANSWER: A
Omeprazole (Prilosec) and lansoprazole approved for clinical treatment of:
A) All of above
B) erosive gastritis
C) Zollinger-Ellison syndrome and other gastric acid hypersecretory states
D) None of above
E) duodenal ulcer
ANSWER: A
Only class of antiulcer drugs that can eradicate Helicobacter pylori and cure
associated gastritis:
A) colloidal bismuth
B) sucralfate (Carafate)
C) H2 blockers
D) Anticholinergic agents
E) None of above
ANSWER: A
Oral anticoagulant therapy is monitored with indexes
A) INR
B) PTT
C) TT
D) Clot lysis time
E) Bleeding time
ANSWER: A
Pancreatic pseudocyst most commonly occurs after
A) Pancreatitis
B) Trauma
C) Pancreatic surgery
D) Pancreatic malignancy
E) None of above
ANSWER: A
Paroxysmal nocturnal haemoglobinuria is due to:
A) Antibodies
B) Congenital membrane defect
C) Inherited autoimmune disorder
D) Complement mediated RBC lysis
E) Lysis of RBC in spleen
ANSWER: D
Pathogenic factor(s) in duodenal ulcer:
A) all of the above
B) genetic factor
C) cigarette smoking
D) alcoholic cirrhosis
E) COPD (chronic obstructive pulmonary disease)
ANSWER: A
Patient D. presents elevated serum T4 and increased radioactive iodine uptake. Put
diagnosis, please.
A) Non-toxic goiter
B) Hashimoto's thyroiditis
C) Subacute thyroiditis
D) Hypothyroidism
E) Graves' disease
ANSWER: E
Patient F., 29 yr. old, presents with subacute thyroiditis. Treatment will include:
A) Antibiotics
B) Vitamins
C) Antithyroid drugs
D) Iodine preparations
E) Corticosteroids
ANSWER: E
Patient on heparin therapy should be monitored with:
A) Clotting time
B) Bleeding time
C) Prothrombin time
D) Fibrinogen
E) PTT
ANSWER: A
Patient presents with recurrent duodenal ulcer of 2.5 cm size; procedure of choice
A) Truncal vagotomy and antrectomy
B) Truncal vagotomy and gastrojejunostomy
C) Highly selective vagotomy
D) Laparoscopic vagotomy and gastrojejunostomy
E) None of above
ANSWER: A
Peptic ulcer classification:
A) duodenal+ gastric
B) gastric
C) duodenal
D) neither
E) cardiac
ANSWER: A
Peptic ulcer should be differentiated with
A) All of above
B) Cholecystitis
C) Gastritis
D) Miocardial infarction
E) Pancreatitis
ANSWER: A
Pernicious anemia is seen in:
A) Iron deficiency
B) Chronic liver disease
C) Bleeding
D) Atrophic gastritis
E) Hemolysis
ANSWER: D
Platelet count is decreased in all of the following condition except:
A) Idiopathic thrombocytopenic purpura
B) Thrombotic thrombocytopenic purpura
C) Systemic lupus erythematosus
D) Haemophilia
E) Hemolytic uremic syndrom
ANSWER: D
Pneumothorax is a possible complication of
A) Staphylococcal pneumonia
B) Pneumococcal pneumonia
C) Klebsiella pneumonia
D) Viral pneumonia
E) All of the above
ANSWER: A
Presentation of hypothyroidism includes:
A) Menorrhagia
B) Oligomenorrhoea
C) Amenorrhea
D) All of them
E) None above them
ANSWER: A
Primary atypical pneumonia is caused by:
A) Mycoplasma
B) Mycobateriumkansasii
C) Str. pneumoniae
D) Pneumocystis carinii
E) Nothing of the above
ANSWER: A
Primary hyperparathyroidism most likely caused by:
A) Multiple parathyroid adenomas
B) Solitary parathyroid adenoma
C) Adrenal hyperplasia
D) Ectopic PTH production
E) None above them
ANSWER: B
Pseudopolyps are features of
A) Ulcerative colitis
B) Crohn's disease
C) Celiac sprue
D) Whipple's disease
E) None of above
ANSWER: A
pylori is known to cause all of the following except:
A) Fundal atrophic gastritis
B) Duodenal ulcer
C) Gastritis type B
D) Gastric ulcer
E) None of above
ANSWER: A
Raised serum amylase levels are used to diagnose
A) Acute pancreatitis
B) Degenerative diseases
C) Acute cholecystitis
D) Autoimmune disease
E) None of above
ANSWER: A
Ranitidine (Zantac) and gastric acid secretion:
A) inhibits basal acid secretion+ inhibits secretion in response to vagal stimulation
or feeding
B) inhibits secretion in response to vagal stimulation or feeding
C) inhibits basal acid secretion
D) neither
E) stimulates basal acid secretion
ANSWER: A
Relapse rate for duodenal ulcer following H. pylori eradication:
A) 15%
B) 75%
C) 50%
D) 90%
E) 5%
ANSWER: A
Reserve transcriptase of hepatitis B virus is coded on the following gene :
A) P gene
B) S gene
C) C gene
D) X gene
E) All of above
ANSWER: A
Risk factor for development of gastric Carcinoma
A) Intestinal metaplasia type III
B) Duodenal ulcer
C) Intestinal hyperplasia
D) Blood group O
E) None of above
ANSWER: A
Screening method for medullary carcinoma thyroid is:
A) Serum calcitonin
B) Serum calcium
C) Serum alkaline phosphate
D) Serum acid phosphatase
E) None above them
ANSWER: A
Secondary hyperparathyroidism is associated with:
A) Parathyroid adenoma
B) Marked hypercalcaemia
C) Chronic renal failure
D) Parathyroidectomy
E) All of them
ANSWER: C
Serum unconjugated bilirubin and urine urobilinogen concentration are elevated in
A) anemias of chronic disease
B) hemolytic anemias
C) anemia of acute hemorrhage
D) aplastic anemias
E) iron deficiency anemia
ANSWER: B
Sharko-Leiden crystals are seen in:
A) Bronchial asthma
B) Bronchiectasis
C) Chronic bronchitis
D) Wegenersgranulomatosis
E) All of the above
ANSWER: A
Side effect of antiulcer medication: urinary retention, blurred vision, xerostromia:
A) atropine
B) ranitidine (Zantac)
C) nifedipine (Procardia, Adalat)
D) sucralfate (Carafate)
E) None of above
ANSWER: A
Significant high ESR is seen in :
A) Polycythemia vera
B) CHF
C) Multiple myeloma
D) Sickle cell anemia
E) CLL
ANSWER: C
Skip granulomatous lesions are seen in
A) Crohn's disease
B) Ulcerative colitis
C) Whipple's disease
D) Reiter's disease
E) None of above
ANSWER: A
Small capillaries bleeds in:
A) Thrombocytopenia
B) Coagulation failure deficiency
C) Anemia
D) Agranulocytosis
E) Hemophilia
ANSWER: A
Somogi phenomenon characterized by :
A) Early morning hyperglycemia
B) Early morning hypoglycemia
C) Hypoglycemia followed by hyperglycemia
D) High insulin levels
E) Night hyperglycemia
ANSWER: A
Spherocytosis is best diagnosed by:
A) Splenic puncture
B) BM aspiration
C) Plasma
D) Peripheral blood smear
E) Phenotyping
ANSWER: D
Such radiological findings as "hour-glass" vertebrae and "triturated pelvis" are seen
in:
A) Thyrotoxicosis
B) Myxedema
C) Cretinism
D) Hyperparathyroidism
E) Hypothyroidism
ANSWER: D
Such results of investigations as positive Chvostek sign and elevated serum
parathyroid hormone (PTH) level are associated with:
A) Pseudohypoparathyroidism
B) Primary hypoparathyroidism
C) Primary hyperparathyroidism
D) Osteoporosis
E) Hypercalcemia of malignancy
ANSWER: A
The actions of insulin includes which one of the following?
A) Increased utilization of glucose by muscle
B) Increased hepatic output of glucose
C) Increased lipolysis
D) Increased amino acid flux from muscle due to protein breakdown
E) Activated glycogenolysis
ANSWER: A
The adenocarcinoma of esophagus-developed in
A) Barret's esophagus
B) Long standing achalasia
C) Corrosive structure
D) Alcohol abuse
E) None of above
ANSWER: A
The commonest acute presentation of sickle cell anaemia is:
A) Priapism
B) Bone pain
C) Fever
D) Splenomegaly
E) Pulmonary hypertension
ANSWER: B
The commonest hepatotropic virus progressing to chronicity is:
A) HCV
B) HAV
C) HBV
D) HEV
E) HAV+ HEV
ANSWER: A
The first virological marker following acute infection with HBV is :
A) HBs Ag
B) Anti HBs Ag
C) IgM anti HBc Ag
D) Anti HBe Ag
E) All of above
ANSWER: A
The following laboratory index is abnormally prolonged in ITP:
A) APTT
B) Bleeding time
C) Prothrombin time
D) Clotting time
E) None of above
ANSWER: B
The frequent causes of hypoglycemia in hospitalized diabetic patients include all the
following EXCEPT:
A) Insulin
B) Sulfonylureas
C) Adrenal insufficiency
D) Alcohol
E) Renal failure
ANSWER: C
The high level of C-peptide can be found in a case of:
A) Glucagonoma
B) Insulinoma
C) Gastrinoma
D) Hepatoma
E) In all of them
ANSWER: B
The histological features of coeliac disease include all of the following, except:
A) Increase in thickness of the mucosa
B) Crypt hyperplasia
C) Increase in intraepithelial lymphocytes
D) Increase in inflammatory cells in lamina propyria
E) None of above
ANSWER: A
The lowest recurrence of peptic ulcer is associated with
A) Vagotomy + Antrectomy
B) Vagotomy + drainage
C) Gastric resection
D) Highly selective vagotomy
E) Resection of stomach by Bilroth II
ANSWER: A
The main cause of hypercalcemic crisis is :
A) Parathyroid adenoma
B) CA Breast
C) Parathyroid hyperplasia
D) Paget's disease
E) Hyperthyroidism
ANSWER: A
The main cause of hyperosmolar coma is:
A) Insulin deficiency
B) Dehydration
C) Extreme activity
D) Hypoxia
E) Intercurrent infection
ANSWER: B
The main cause of lactic acidosis is
A) Insulin deficiency
B) Dehydration
C) Extreme activity
D) Hypoxia
E) Intercurrent infection
ANSWER: D
The major sign of hypoventilation is
A) Cyanosis
B) Hypoxia
C) Hypercapnia
D) Dyspnea
E) All of the above
ANSWER: A
The most common cause of aplastic anemia is:
A) Idiopathic
B) Chloramphenicol
C) Phenylbutazone
D) Petroleum products
E) Prednisolone
ANSWER: A
The most common cause of thyroid crisis is:
A) Thyroid surgery
B) 131I administration
C) Administration of Beta blockers
D) Thyrostatic therapy
E) All of the above
ANSWER: A
The most common cause of thyrotoxicosis is:
A) Grave's disease
B) Follicular adenoma
C) Multinodular goiter
D) Subacute thyroiditis
E) Iatrogenic
ANSWER: A
The most common community acquired infection is :
A) Streptococcal pneumonia
B) Hepatitis A
C) Cholera
D) Meningitis
E) Nothing of the above
ANSWER: A
The most common presentation of primary hyperparathyroidism is:
A) Bone fracture
B) Increased serum creatinine
C) Osteitis fibrosa cystica
D) Asymptomatic hypercalcemia
E) Kidney stones
ANSWER: D
The most common route of spread of hepatitis E is
A) Feco-oral
B) Blood transfusion
C) injection IV
D) perinatal transmission
E) All of above
ANSWER: A
The most effective correction of acidosis in diabetic acidosis is:
A) IV bicarbonate
B) IV Saline
C) Oral bicarbonate
D) IV Insulin
E) IV potassium
ANSWER: D
The most important regulator of serum 1,25(OH)2 vitamin D concentration is
A) Serum calcium
B) Serum magnesium
C) Serum 25(OH) vitamin D
D) TSH
E) Parathyroid hormone
ANSWER: E
The most pre-cancerous condition for carcinoma colon is
A) Familial polyposis
B) Hamartomatous polyps
C) Juvenile polyps
D) Hyperplastic polyps
E) None of above
ANSWER: A
The patient is taking enalapril (Vasotec), an angiotensin-converting enzyme (ACE)
inhibitor, for hypertension. Which respiratory side effect should you teach the
patient to expect?
A) Wheezing on exhalation
B) Increased nasal stuffiness
C) Chest pressure or pain
D) Persistent dry cough
E) Non of above
ANSWER: D
The place of 25-hydroxycholecalciferol formation is:
A) Liver
B) Kidney
C) Intestines
D) Pancreas
E) Brain
ANSWER: A
The principal role of the kidneys in the synthesis of the active form of vitamin D is:
A) Conversion of 7-dehydrocholesterol to vitamin D3
B) Absorption of vitamin D2
C) Conversion of vitamin D to 25-OH vitamin D
D) Absorption of vitamin D3
E) Conversion of 25-OH vitamin D to 1,25-(OH)2 vitamin D
ANSWER: E
The results of the glucose tolerance test: a fasting serum glucose is 6,5 mmol/l, 2-
hour postprandial serum glucose is 14,6 mmol/l. What is your diagnosis?
A) Normal.
B) Impaired glucose tolerance.
C) Impaired fasting glycemia.
D) Diabetes mellitus.
E) Neither.
ANSWER: D
The shortest period of the action is present in such group of preparations as:
A) Thiazolidinediones
B) Biguanides
C) Sulfonilureas
D) Acarbose
E) Glinides
ANSWER: E
The triad originally described by Zollinger-Ellison syndrome is characterized by:
A) Peptic ulceration, gastric hypersecretion, non beta cell tumour
B) Peptic ulceration, gastric hypersecretion, beta cell tumor
C) Peptic ulceration, achlorhydria, non beta cell tumour
D) Peptic ulceration, achlorhydria, beta cell tumour
E) None of above
ANSWER: A
The true statement of tertiary hyperparathyroidism is:
A) Primary hyperparathyroidism with decrease Ca+ level
B) Secondary hyperparathyroidism with chief cell adenoma
C) Secondary hyperparathyroidism following intestinal malabsorption
D) Metastasis with normal phosphate level
E) Secondary hyperparathyroidism following chronic renal failure
ANSWER: B
Thrombocytopenia, neutropenia are seen in
A) anemias of chronic disease
B) hemolytic anemias
C) anemia of acute hemorrhage
D) aplastic anemias
E) iron deficiency anemia
ANSWER: D
Thyroid carcinoma associated with hypocalcemia is
A) Follicular carcinoma
B) Medullary carcinoma
C) Anaplastic carcinoma
D) Papillary carcinoma
E) All of them
ANSWER: B
To remove onset of tetany you will order
A) Calcium chloride
B) Diuretics
C) Prednisolone
D) Potassium citrate
E) Magnesium sulphate
ANSWER: A
Toxic megacolon is most commonly associated with
A) Ulcerative colitis
B) Crohn's disease
C) Whipple's disease
D) Reiter's disease
E) None of above
ANSWER: A
Treatment of choice for aplastic anemia is:
A) Methotrexat
B) ampicillini
C) chloramphenicol
D) Bone marrow transplantation
E) Plasmaphoresis
ANSWER: D
Treatment of gastric ulcer disease
A) atropine is less effective than H2 receptor blockers
B) atropine is more effective than H2 receptor blockers
C) atropine and H2 receptor blockers are equally effective
D) atropine and H2 receptor blockers are not effective
E) Omeprazol is not used
ANSWER: A
Treatment of neonatal idiopathic thrombocytopenia is
A) Azathioprin
B) Dexamethasone
C) Immunoglobulins
D) Platelets transfusion
E) Splenectomy
ANSWER: C
True about idiopathic thrombocytopenic purpura is:
A) Increase megakaryocytes in bone marrow
B) Frequent joint haemorrhage
C) Most common in children
D) Males commonly affected
E) VIII factor deficiency
ANSWER: A
True about Ulcerative Colitis with malignancy
A) Is related to duration of ulcerative colitis
B) Is related to disease activity
C) It has a better prognosis
D) Malignancy is more in anorectal ulcerative colitis
E) None of above
ANSWER: A
True statement about Hypercalcemia :
A) Treatment of the primary cause is effective
B) Malignancy does not produce hypercalcemia
C) i.v. fluid with Furosemide is given
D) Pamidronate is not effective.
E) i.v. fluid with calcium chloride is effective
ANSWER: C
What is a possible level of platelets in the patient with Acute Lymphoblastic Leukemia?
A) 100.000 – 150.000/L
B) Over 200.000/L
C) 250.000– 300.000/L
D) Below 300.000/L
E) Absence of Pl
ANSWER: A
What is drug of choice for ulcerative colitis :
A) 5-amino salicylic acid
B) Prednisolone
C) Mercaptopurine
D) Salazopyrine
E) None of above
ANSWER: A
What is not true about blind loop syndrome?
A) Surgery is almost always required to correct small bowel syndrome
B) Megaloblastic anemia is commonly seen
C) It manifets as diarrhoea, weight loss and deficiency of fat soluble vitamins.
D) Broad spectrum antibiotics are the treatment of choice
E) None of above
ANSWER: A
What is not true about the immune mechanism in the small bowel?
A) Ig A acts by activating the complement pathway
B) Intestine contains more than 70% of IgA producing cells in the body
C) Ig A is produced by plasma cells in the lamina propria
D) Approximately 60% of the lymphoid cells are T cells
E) None of above
ANSWER: A
What is the Acute Lymphoblastic Leukemia?
A) Disorder of the blood-forming tissue (white cells)
B) Disorder of the blood-forming tissue (red cells)
C) Leukocytosis
D) Increasing of blood pressure
E) Decreasing of blood pressure
ANSWER: A
What is the best marker for prognosis of the multiple myeloma presents with bony
lesions:
A) Serum calcium level
B) Beta 1a microglobulin
C) Beta 1 microglobulin
D) Bone marrow plasma cell
E) Number of lytic lesions
ANSWER: D
What is the most necessary in order to diagnose bronchial asthma:
A) To notice the asthmatic attack of dyspnea
B) To find allergic antibodies
C) To find the presence of expiration dyspnea
D) To find the presence of signs of obstructive respiratory failure
E) To find the presence of eosinophylia in blood
ANSWER: A
What is true of haemophilia:
A) Increased PTT
B) Platelets level decreased
C) Increased factor VIII
D) Increased factor IX
E) BT decreased
ANSWER: A
What is true regarding congenital hypertrophic pyloric stenosis:
A) Hypochloremic alkalosis
B) More common in girls
C) Hellers myotomy is the procedure of choice.
D) Most often manifests at birth
E) None of above
ANSWER: A
Which among the following diseases may be causative agent for B12 deficiency anaemia?
A) Chronic gastritis of type B
B) Peptic gastric ulcer
C) Chronic gastritis of type A
D) Chronic pancreatitis
E) Chronic cholecystitis
ANSWER: C
Which amongst the following is the earliest indicator of hypothyroidism?
A) Decreased serum T4
B) Decreased serum T3
C) Increased relaxation phase of deep tendon reflexes
D) High serum TSH
E) High serum TRH
ANSWER: D
Which changes of a tongue are typical for vitamin B12 deficiency anaemia?
A) Geographic tongue
B) Red
C) Coated
D) Clean
E) Swollen
ANSWER: A
Which does not predispose to Carcinoma stomach?
A) Low Nitrate consumption
B) Salted meat and fish
C) Low fat and protein diet
D) HIgh Complex carbohydrate consumption
E) All of above
ANSWER: A
Which drug is mostly used for the treatment of chronic lymphatic leukemia?
A) Prednisone
B) Chlorambucil
C) Methotrexate
D) Myleran
E) Ampicilin
ANSWER: B
Which drug is not effective against H. pylori:
A) Erythromycin
B) Metronidazole
C) Amoxicillin
D) Colloidal Bismuth
E) None of above
ANSWER: A
Which from the following symptoms are characteristic for anaemia?
A) Petechia
B) Cyanosys
C) Pallor of skin and mucous membranes
D) Edema
E) Hyperpigmentation
ANSWER: C
Which index can differentiate factitious hyperinsulinemia from insulinoma:
A) C – peptides
B) Insulin antibodies
C) Serum glucose levels
D) None
E) All of them
ANSWER: A
Which is not a feature of multiple myeloma?
A) Cutaneous nodules
B) Bony lesions
C) Renal failure
D) 'M' Spike
E) Hypercalciemia
ANSWER: A
Which is not a paraneoplastic syndrome for Hepatocellular Carcinoma
A) Hyperglycemia
B) Hypoglycemia
C) Erythrocytosis
D) Hypercalcemia
E) None of above
ANSWER: A
Which of the following can help to put a diagnosis of subclinical hypothyroidism for a
patient?
A) Low thyroid hormone levels but no symptoms
B) Classic symptoms of hypothyroidism but normal thyroid function test results
C) Low free T4 levels but normal serum thyroid-stimulating hormone (TSH) level
D) Low serum TSH but normal free T4 levels
E) Increased serum TSH but normal free T4 levels
ANSWER: E
Which of the following conditions causes delayed, deep tissue-type bleeding?
A) Uremia
B) Hemophilia A
C) Therapy with aspirin
D) von Willebrand's disease (vWD)
E) Idiopathic thrombocytopenic purpura (ITP)
ANSWER: B
Which of the following conditions is LEAST likely to be associated with a low serum
25(OH) vitamin D level?
A) Dietary deficiency of vitamin D
B) Chronic severe cholestatic liver disease
C) Sedentary life-style
D) Chronic renal failure
E) High-dose glucocorticoid therapy
ANSWER: D
Which of the following has the weakest association with Hepatocellular Carcinoma (HCC)
A) oral contraceptives
B) Hepatitis C
C) Hepatitis B
D) Smoking
E) None of above
ANSWER: A
Which of the following hepatitis viruses have significant perinatal transmission
A) Hepatitis B virus
B) Hepatitis C virus
C) Hepatitis E virus
D) Hepatitis A virus
E) None of above
ANSWER: A
Which of the following is a feature of nephrocalcionsis:
A) Primary hyperparathyroidism
B) Medullary cystic kidney
C) Vitamin C intoxication
D) Pseudohypoparathyroidism
E) Primary hypothyroidism
ANSWER: A
Which of the following is False regarding H.Pylori infection
A) With chronic infection urease breath test become negative
B) H.Pylori infection remain lifelong if untreated
C) Endoscopy is diagnostic
D) Toxigenic strains usually causes ulcer
E) None of above
ANSWER: A
Which of the following is not a congenital abnormality associated with Juvenile polyps
A) Macrocephaly
B) Meckel's diverticulum
C) Malrotation
D) Mesenteric lymphangioma
E) None of above
ANSWER: A
Which of the following is NOT seen in Paroxysmal Nocturnal Hemoglobinuria:
A) Thrombosis
B) Hemosiderinuria
C) Increased haptoglobin
D) Thrombocytopenia
E) Anemia
ANSWER: C
Which of the following is NOT the contraindication for sulfonilureas?
A) Diabetic with excessive weight
B) Gestation diabetes
C) Diabetic with progressive weight loss
D) Type 2 DM, Non-proliferative rethinopathy
E) Pregnancy and lactation
ANSWER: D
Which of the following is not true about gastric lymphoma?
A) MALT lymphoma is the commonest variety.
B) Peak incidence of lymphomas is seen in 6th-7th decade
C) Endoscopy usually reveals gastritis like picture or gastric ulcer.
D) Stomach is the most common organ in the gi system which is involved in Lymphoma
E) All of above
ANSWER: A
Which of the following is not true about Pneumatosis intestinalis of small bowel?
A) Operative Procedures are required in most of the cases
B) Most common location is subserosa in the jejunum
C) It is seen equaly and males and females
D) It is associated with COPD and immunodeficiency states
E) None of above
ANSWER: A
Which of the following is not true for malignancy of Familial Adenomatous Polyposis
A) Astrocytomas
B) Thyroid
C) Adrenals
D) Hepatoblastomas
E) None of above
ANSWER: A
Which of the following is not used in treatment of leukemia?
A) Steroid
B) Pentostatin
C) Splenectomy
D) Alpha-interferon
E) Alendronic acid
ANSWER: E
Which of the following is the most common cause of death in Crohn's disease of small
bowel
A) Malignancy
B) Sepsis
C) Electrolyte Disorders
D) Thromboembolic Phenomenon
E) None of above
ANSWER: A
Which of the following is used in the treatment of the thyroid malignancy:
A) I131
B) I125
C) 99Tc
D) P32
E) MIBG
ANSWER: A
Which of the following may be seen in multiple myeloma?
A) Increased Alkaline phosphatase
B) Decreased IgA
C) Hypercalcemia
D) Hypouricemia
E) Fat bone marrow
ANSWER: C
Which of the following oral anti-diabetic drugs can be used in patients with renal
failure:
A) Tolbutamide
B) Chlorpropamide
C) Gliquidone
D) Glipizide
E) Metformin
ANSWER: C
Which of the following produces calcitonin?
A) A (alpha) cells
B) B (beta) cells
C) C (parafollicular) cells
D) D (delta) cells
E) F cells
ANSWER: C
Which of the following statements about peptic ulcer disease is true :
A) The incidence of complications has remained unchanged
B) Helicobacter pylori eradication increases the likelihood of occurrence of
complications.
C) The incidence of Helicobacter pylori reinfection in India is very low.
D) Helicobacter pylori eradication does not alter the recurrence ratio.
E) None of above
ANSWER: A
Which of the following structures do not form the external part of anal canal
mechanism?
A) Internal Sphincter
B) Levator Ani
C) Pubo rectalis
D) External Sphincter
E) None of above
ANSWER: A
Which of the following structures does not form the portal triad
A) Hepatic vein
B) Portal Vein
C) Hepatic Artery
D) Bile Duct
E) None of above
ANSWER: A
Which of these is true regarding CML?
A) Size of splenomegaly indicates prognosis
B) Phagocytic activity of WBC is reduced
C) Sudan black stain is specific for myeloblast
D) Myeloblast, granuloblast and lymphoblast become PH chromosome +–ve following
remission
E) Leukocytosis
ANSWER: A
Which one of the following is associated with increased bone resorbtion?
A) Estrogens
B) Calcitonin
C) Bisphosphonates
D) Hyperparathyroidism
E) Hypothyroidism
ANSWER: D
Which patient is at greatest risk for developing a “community-acquired” pneumonia?
A) The 40-year-old first-grade teacher
B) The 60-year-old smoker who is also an alcoholic
C) The 75-year-old with exercise-induced wheezing
D) The 35-year-old aerobics instructor who skips meals and eats only vegetables
E) None of above
ANSWER: B
Which signs are associated with tetany EXCEPT:
A) Chvostek’s sign
B) Trousseau's sign
C) Erb's sign
D) All of them
E) None above them
ANSWER: D
Which statement is true in diabetes mellitus type 2?
A) Insulinitis of beta- cells
B) Hyalinization of beta- cells
C) Atrophy of beta cells
D) Intact beta- cells
E) Degeneration of beta cells
ANSWER: D
Which test can you recommened as the most indicative of average recent blood glucose
levels:
A) Fasting serum glucose level
B) Random serum glucose level
C) Oral glucose tolerance test
D) Serum level of hemoglobin A1C
E) Urine glucose concentration
ANSWER: D
Which type diabetes is HLA associated:
A) Type 1 diabetes
B) Type 2 diabetes
C) Tropic diabetes
D) Gestation diabetes
E) All of them
ANSWER: A
With which of the following theophylline has an antagonistic interaction?
A) Histamine receptors.
B) Bradykinin receptors.
C) Adenosine receptors.
D) Imidazoline receptors.
E) Beta-adrenoreceptors
ANSWER: C
Zollinger-Ellison syndrome is characterized by all of the following except
A) Massive HCL in response to histamine injection
B) Recurrent duodenal ulcer
C) Severe diarrhea
D) Post bulbar ulcer
E) None of above
ANSWER: A
4 weeks after myocardial infarction a 56-year-old patient developed acute heart pain,
pronounced dyspnea. Objectively: the patient’s condition is extremely grave, there is
marked cyanosis of the face, swelling and throbbing of the neck veins, peripheral
pulse is absent, carotid artery pulse is rhythmic, 130/min., BP is 60/20 mm Hg.
Auscultation of the heart reveals extremely muffled sounds, percussion reveals heart
border extension in both directions. What is the optimal treatment tactis for this
patient?
A) Pericardiocentesis and immediate thoracotomy
B) Oxygen inhalation
C) Puncture of the pleural cavity on the left
D) Conservative treatment, infusion ofadrenomimetics
E) Pleural cavity drainage
ANSWER: A
A 14-year-old girl has been presenting with irritability and tearfulness for about a
year. A year ago she was also found to have diffuse enlargement of the thyroid gland
(II grade). This condition was regarded as a pubertal manifestation, the girl did not
undergo any treatment. The girl’s irritability gradually gave place to a complete
apathy. The girl developed puffy face, soft tissues pastosity, bradycardia,
constipations. Skin pallor and gland density progressed, the skin developed a waxen
hue. What disease can be suspected?
A) Autoimmune thyroiditis
B) Diffuse toxic goiter
C) Thyroid carcinoma
D) Subacute thyroiditis
E) Juvenile basophilism
ANSWER: A
A 15-year-old patient complains of excessive body weight, headache, irritability,
rapid fatigability. Significant increase of body weight occurred at the age of 14.
Objectively: weight is 90 kg; height is 160 sm, proportional body built. Fatty tissue
is distributed evenly. There are thin pink striae (stretch marks) on the thighs,
abdomen and mammary glands. BP - 145/90 mm Hg. Provisional diagnosis is:
A) Pubertate dyspituitarism
B) Alimentary constitutive obesity
C) Somatoform autonomic dysfunction
D) Itsenko-Cushing’s disease
E) Cushing’s syndrome
ANSWER: A
A 15-year-old patient suffers from headache, nasal haemorrhages, sense of lower
extremity coldness. Objectively: muscles of shoulder girdle are developed, lower
extremi-ties are hypotrophied. Pulsation on the pedal and femoral arteries is sharply
dampened. BP is 150/90 mm Hg, 90/60 in the legs. Systolic murmur can be auscultated
above carotid arteries. What is the most probable diagnosis?
A) Aorta coarctation
B) Aorta aneurism
C) Aortal stenosis
D) Aortal insufficiency
E) Coarctation of pulmonary artery
ANSWER: A
A 16-year-old adolescent has been hospitalized with complaints of unceasing nasal
hemorrhage and unbearable pain in his ri-ght elbow joint. Objectively: the large joint
is enlarged and defigurated, the skin over the joint is hyperemic. Arthropathy signs
can be observed in the other joints. Ps- 90/min. Blood test: erythrocytes - 3, 9 •
1012/l, Нb- 130 g/l, color index - 1,0, leukocytes - 5, 6 • 109/l, platelets - 220 •
109/l, ESR- 6 mm/hour. Lee-White coagulation time: start- 24, end-27_10. What drug
would be most efficient in the treatment of this patient?
A) Cryoprecipitate
B) Calcium chloride
C) Concentrated red cells
D) Aminocapronic acid
E) Vicasol (Menadione)
ANSWER: A
A 16-year-old girl addressed a doctor with complaints of fatigability and dizziness.
On heart auscultation: systolic murmur in the II intercostal area along the breastbone
edge on the left. ECG revealed signs of the right ventricular hypertrophy. X-ray
revealed dilatation of the the pulmonary artery trunk, enlargement of the right heart.
What heart disorder is it?
A) Pulmonary artery outflow stenosis
B) Fallot’s tetrad
C) Defect of the interatrial septum
D) Coarctation of aorta
E) Pulmonary artery valve failure
ANSWER: A
A 19-year-old male patient complains of intense painin the left knee joint.
Objectively: the left knee joint is enlarged, the overlying skin is hyperemic,the
joint is painful on palpation. Blood test results: RBC- 3, 8•1012/l, Hb- 122g/l,
lymphocytes- 7, 4 • 109/l, platelets - 183 • 109/l. ESR-10 mm/h. Duke bleeding time
is 4 minutes, Lee-White clottingtime-24 minutes. A-PTT is 89sec. Rheumatoid factor is
negative.What is the most likely diagnosis?
A) Hemophilia, hemarthrosis
B) Werlhof’s disease
C) Rheumatoid arthritis
D) Thrombocytopathy
E) Hemorrhagic vasculitis, articular form
ANSWER: A
A 19-year-old patient complains of dyspnea during physical exertion. He often has
bronchitis and pneumonia. Cardiac murmur has been observed since his childhood. On
auscultation: there is splitting of II heart sound over pulmonary arteria, systolic
murmur in the third intercostal space near the left sternum margin. ECG test shows
right bundle-branch block. What is the provisional diagnosis?
A) Interatrial septum defect
B) Open arterial duct
C) Coarctation of aorta
D) Aortic stenosis
E) Mitral valve insufficiency
ANSWER: A
A 19-year-old patient complains of dyspnea during physical exertion. He often has
bronchitis and pneumonia. Cardiac murmur has been observed since his chi-ldhood. On
auscultation: there is splitting of the II heart sound over pulmonary arteria,
systolic murmur in the third intercostal space near the left sternum margin. ECG test
shows right bundle-branch block. What is the provi-sional diagnosis?
A) Interatrial septum defect
B) Open arterial duct
C) Coarctation of aorta
D) Aortic stenosis
E) Mitral valve insufficiency
ANSWER: A
A 20-year-old patient was delivered to a surgical unit complaining of an incised wound
on his right forearm that has been bleeding for 1,5 days. Suffers from general
weakness, vertigo, cold sweat, opplotentes. Skin and visi- ble mucous membranes are
pale. Heart rate is 110/min, BP is 100/70 mm Hg. Blood test: Hb is 100g/l,
erythrocytes 2,5•1012/l.What i sthe cause for the paient’s genera lcondition?
A) Posthemorrhagic anemia
B) Aplastic anemia
C) Wound infection
D) Concomitant disease
E) Acute trombophlebitis
ANSWER: A
A 20-year-old patient was delivered to a surgical unit complaining of an incised wound
on his right forearm that has been bleeding for 1,5 days. Suffers from general
weakness, vertigo, cold sweat, opplotentes. Skin and visible mucous membranes are pale.
Heart rate is 110/min, BP is 100/70 mm Hg. Blood test: Hb is 100 g/l, erythrocytes 2,
5 • 1012/l. What is the cause for the patient’s general condition?
A) Posthemorrhagic anemia
B) Aplastic anemia
C) Wound infection
D) Concomitant disease
E) Acute trombophlebitis
ANSWER: A
A 22-year-old patient complainsof 8-months-long delay of menstruation. Anamnesis:
menarche since the age of 12,5. Since the age of 18 menstruations are irregular. No
pregnancies. Mammary glands have normal development; when the nipples are pressed,
milk drops are discharged. On gynecological examination: moderate uterus hypoplasia.
On hormonal examination: prolactin level exceeds the norm two times. On computed
tomogram of the sellar region: a space-occupying lesion 4 mm in diameter is detected.
The most likely diagnosis is:
A) Pituitary tumor
B) Lactation amenorrhea
C) Stein–Leventhal syndrome (Polycystic ovarysyndrome)
D) Sheehan’s syndrome
E) Cushing’s disease
ANSWER: A
A 22-year-old woman on a reduced diet, vegetarian, attended a hospital with complaints
of distorted smell and taste perception and lesions in the angles of her mouth.
Objecti- vely: sclera is distinctly blue. Diagnosis: iron- deficiency anemia. What
clinical syndrome is expressed primarily?
A) Sideropenic
B) Anemic
C) Hemorrhagic
D) Hemolytic
E) Myelodysplasic
ANSWER: A
A 23-year-old patient after intake of brake fluid has developed anuria that has been
lasting for 5 days already. Creatinine level increased up to 0,769 mmol/l. What
treatment tactics should be chosen in the given case?
A) Hemodialysis
B) Detoxification therapy
C) Antidotal therapy
D) Diuretics
E) Plasmapheresis
ANSWER: A
A 23-year-old patient had taken 1 g of aspirin to treat acute respiratory infection.
After that he developed an asthmatic attack with labored expiration it was arrested by
introduction of aminophylline. The patient’s medical history is not burdened with
allergies. The patient has undergone two surgeries for nasal polyposis in the past.
What diagnosis is most likely?
A) Aspirin-induced asthma
B) Atopic bronchial asthma
C) Infectious allergic bronchial asthma
D) Exercise-induced asthma
E) Symptomatic bronchospasm
ANSWER: A
A 23-year-old patient had taken 1 g ofaspirin to treat acute respiratory infection.
After that he developed an asthmatic fit wi-th labored expiration that was arrested by
introduction of aminophylline. The patient’s medical history is not burdened with
allergies. The patient has undergone two surgeries for nasal polyposis in the past.
What diagnosis is most likely?
A) Aspirin-induced asthma
B) Atopic bronchial asthma
C) Infectious allergic bronchial asthma
D) Exercise-induced asthma
E) Symptomatic bronchospasm
ANSWER: A
A 23-year-old woman after stress has developed thirst, polydipsia, polyuria, weight
loss, increasing fatigue. Later she developed nausea and somnolence, lost
consciousness and was hospitalised. Glycemia is 27 mmol/l, acetone in urine is sharply
positive. Treatment for ketoacidotic coma was initiated. When would it be advisable to
start preventive treatment of hypoglycemia by introduction of 5% glucose solution?
A) After glycemia rate drops to13-14 mmol/l
B) 2 hours after beginning of insulin therapy
C) When patient becomes conscious
D) After glycemia rate becomes normal
E) If glycemia decreases with the rate over 5 mmol/l perhour
ANSWER: A
A 23-year-old woman, who works as a milk and dairy inspector, after the miscarriage
suffers from high fever up to 38, 6oC , recurring chills, excessive sweating.
Objectively: polyadenitis, pain in the lumbosacral spine, swollen left knee joint,
enlarged liver and spleen. What diagnosis is the most likely?
A) Brucellosis
B) Sepsis
C) Toxoplasmosis
D) Polyarticular rheumatoid arthritis
E) Yersiniosis
ANSWER: A
A 24-year-old male patient had been diagnosed with class III diffuse toxic
goiter.There is moderate hyperthyroidism. A surgery was suggested,and the patient
agreed to it.What preoperative measures should be taken for prevention of thyrotoxic
crisis in the postoperative period?
A) Administration of antithyroid drugs
B) Minimally invasive surgical techniques
C) Bed rest
D) Detoxification therapy
E) Administration of corticosteroids
ANSWER: A
A 24-year-old patient visited a doctor complaining of enlargement of his submaxillary
lymph nodes. Objectively: submaxillary, axillary and inguinal lymph nodes are enlarged.
Chest X-ray shows: enlarged lymph nodes of mediastinum. Blood test: erythrocytes - 3,
4 • 1012/l, Hb- 100 g/l, blood colour index - 0,88, platelets - 190 • 109/l,
leucocytes - 7, 5•109 /l, eosinophiles - 8%, band neutrophiles - 2%, segmented
neutrophiles - 67%, lymphocytes - 23%, ESR- 22 mm/hour. What test must be prescribed
to verify the cause of lymphadenopathy?
A) Open biopsy of lymph nodes
B) Ultrasonography of abdominal cavity
C) Mediastinum tomography
D) Puncture biopsy of lymph nodes
E) Sternal puncture
ANSWER: A
A 24-year-old woman, teacher by profession, complains of dizziness and heart pain
irradiating to the left nipple. Pain is not associated with physical activity and
cannot be relieved by nitroglycerin, it abates after taking Valocordin and lasts for
an hour or more. The patient has a nearly 2-year history of this di-sease. Objectively:
Heart rate - 76 bpm. BP - 110/70 mm Hg. Heart borders are normal, heart sounds are
clear. The ECG shows respiratory arrhythmia. X-ray of the cervicothoracic spine shows
no pathology. Lungs, abdomen are unremarkable. What changes in blood formula can be
expected?
A) No changes
B) Leukocytosis
C) Thrombocytopenia
D) Leukemic hiatus
E) Increased ESR
ANSWER: A
A 25-year-old female patient complains of marked weakness, sleepiness, blackouts,
dizziness, taste disorder.The patient has a history of menorrhagia.Objectively:the
patient as marked weakness, paleskin, cracksin the corners of mouth, peelingnails,
systolic apical murmur. Blood test results: RBC- 3, 4 • 1012/l, Hb-70g/l, colorindex-
0,75, platelets - 140 • 109/l, WBC- 6, 2 • 109/l. What is the most likely diagnosis?
A) Chronic posthemorrhagic anemia
B) Acute leukemia
C) Acute post hemorrhagic anemia
D) B12-deficiency anemia
E) Werlhof’s disease
ANSWER: A
A 25-year-old patient has been admitted to the hospital with the following problems:
weakness, sweating, itching, weight loss, enlarged submandibular, cervical, axillary,
inguinal lymph nodes. Objectively: hepatomegaly. Lymph node biopsy revealed giant
Berezovsky-Reed-Sternberg cells, polymorphocellular granuloma composed of lymphocytes,
reticular cells, neutrophils, eosinophils, fibrous tissue, plasma cells. What is the
most likely diagnosis?
A) Lymphogranulomatosis
B) Lymph node tuberculosis
C) Lymphoreticulosarcoma
D) Cancer metastases to lymph nodes
E) Macofollicular reticulosis
ANSWER: A
A 26-year-old female patient has an11- year history of rheumatism.Four years ago she
suffered 2 rheumatic attacks. Over the last 6 months there have been paroxysms of
atrial fibrillation every 2-3 months. What option of antiarrhythmic therapy or tactics
should be proposed?
A) Immediate hospitalization
B) Defibrillation
C) Lidocaine administration
D) Heparin administration
E) Prophylactic administration of cordarone
ANSWER: E
A 26-year-old male patient complains of piercing pain during breathing, cough, dyspnea.
Objectively: to- 37, 3oC , respiration rate is 19/min, heart rate is 92/min; BP is
120/80 mm Hg. Vesicular respiration is observed. In the inferolateral parts of chest
auscultation in both inspiration and expiration phase revealed noise that was getting
stronger at phonendoscope pressing and could be still heard after cough. ECG showed no
pathological changes. What is the most likely diagnosis?
A) Acute pleuritis
B) Intercostal neuralgia
C) Subcutaneous emphysema
D) Spontaneous pneumothorax
E) Pericarditis sicca
ANSWER: A
A 27-year-old woman complains of bleeding gums, nasal hemorrhages, multiple hematomas
on the skin of her limbs and on the front of her torso, extreme general fatigue. Blood
test: Hb- 64 g/l, erythrocytes - 2, 5 • 1012/l, reticulocytes - 16%, platelets 30 •
109/l, ESR- 22 mm/hour. What approach would be most efficient for treatment of this
pathology?
A) Splenectomy
B) Dicynone (Etamsylate)
C) Platelet concentrate transfusion
D) Cytostatics
E) Group B vitamins
ANSWER: A
A 28-year-old woman complains of increased intermenstrual periods up to 2 months,
hirsutism. Gynaecological examinati-on revealed that the ovaries were enlarged,
painless, compact, uterus had no pecularities. Pelvic ultrasound revealed that the
ovari-es were 4-5 cm in diameter and had multiple enlarged follicles on periphery. X-
ray of skull base showed that sellar region was dilated. What is the most probable
diagnosis?
A) Stein-Leventhal syndrome (Polycystic ovary syndrome)
B) Algodismenorrhea
C) Sheehan’s syndrome
D) Premenstrual syndrome
E) Morgagni-Stewart syndrome
ANSWER: A
A 29-year-old female patient complains of dyspnea and palpitations on exertion.
According to her mother, as a child she had heart murmur, did not undergo any
examinations. Objectively: the patient has pale skin, Ps- 94/min, rhythmic. AP-
120/60mmHg. In the II intercostal space on the left auscultation reveals a continuous
rasping systolodiastolic murmur, diastolic-above the pulmonary artery. Blood and urine
are unremarkable. What is the most likely diagnosis?
A) Patent ductus arteriosus
B) Atrial septal defect
C) Ventricular septal defect
D) Aortarctia
E) Tetralogy of Fallot
ANSWER: A
A 29-year-old patient works as a motor mechanic. Anamnes is shows frequent exposure to
cold, exacerbation of chronic bronchitis attended by cough with relativly small amount
of mucopurulent sputum, subfebrility, sometimes joined by hemoptysis and pain in the
right side of chest. Breathing is vesicular. X-ray shows darkening and sharp decrease
in size of the lower lobe distinctly visible on the X-ray image as a streak 2-3 cm
wide situated at the angle from lung root to the frontal costodiaphragmatic recess.
The most likely diagnosis is:
A) Peripheral lung cancer
B) Bronchiectasis
C) Pneumonia
D) Middle lobe syndrome
E) Inter lobular pleurisy
ANSWER: A
A 30-year-old male patient complains of inertness, low-gradefever, bleeding gums,
frequent quinsies, aching bones. Objectively: the patient has pale skin and mucous
membranes, sternalgia,+2 cm liver, +5cm painless spleen. Blood test results: RBC- 2, 7
• 1012/l, Нb-80g/l,WBC- 3 • 109/l, eosinophils -4%, basophils-5%, blasts- 4 segmented
neutrophils-17%, lymphocytes- 29%, myelocytes-25%, promyelocytes-12%, monocytes-2%,
platelets- 80 • 109/l, ESR- 57 mm/h.What test should be performed to verify the
diagnosis?
A) Sternal puncture
B) Trephinebiopsy
C) Lymphnodebiopsy
D) Lumbar puncture
E) Chest X-ray
ANSWER: A
A 30-year-old woman with a long history of chronic pyelonephritis complains of
considerable weakness, sleepiness, decrease in diuresis down to 100 ml per day. BP is
200/120 mmHg.In blood:creatinine-0,62millimole/l, hypoproteinemia, albumines - 32 g/l,
potassi- um - 6,8 millimole/l, hypochromic anemia, increased ESR. What is the first
step in the patient treatment tactics?
A) Haemodialysis
B) Antibacterialtherapy
C) Enterosorption
D) Haemosorption
E) Blood transfusion
ANSWER: A
A 30-year-old woman with a long history of chronic pyelonephritis complains of
considerable weakness, sleepiness, decrease in di-uresis down to 100 ml per day. BP is
200/120 mm Hg. In blood: creatinine - 0,62 millimole/l, hypoproteinemia, albumines -
32 g/l, potassium - 6,8 millimole/l, hypochromic anemia, increased ESR. What is the
first step in the patient treatment tactics?
A) Haemodialysis
B) Antibacterial therapy
C) Enterosorption
D) Haemosorption
E) Blood transfusion
ANSWER: A
A 32 year old patient complains of cardiac irregularities, dizziness, dyspnea at
physical exertion. He has never suffered from such condition before. Objectively: Ps-
74/min., rhythmic. BP- 130/80 mm Hg. Auscultation revealed systolic murmur above aorta,
the first heart sound was normal. ECG showed hypertrophy of the left ventricle, signs
of repolarization disturbance in the I , V5 and V6 leads. Echocardiogram revealed that
interventricular septum was 2 cm. What is the most probable diagnosis?
A) Hypertrophic cardiomyopathy
B) Aortic stenosis
C) Essential hypertension
D) Myocardium infarction
E) Coarctation of aorta
ANSWER: A
A 32-year-old welder complains of weakness and fever. His illness initially presented
as tonsillitis one month earlier. On examination: BT- 38, 9oC , RR- 24/min., HR-
100/min., BP- 100/70 mm Hg, hemorrhages on the legs, enlargement of the lymph nodes.
CBC shows Hb- 70 g/l, RBC- 2, 2 • 1012/l,WBC- 3, 0 • 109/l with 32% of blasts, 1% of
eosinophiles, 3% of bands, 36% of segments, 20% of lymphocytes, and 8% of monocytes,
ESR- 47 mm/hour. What is the cause of anemia?
A) Acute leukemia
B) Chronic lympholeukemia
C) Aplastic anema
D) B12-deficient anemia
E) Chronic hemolytic anemia
ANSWER: A
A 32-year-old woman complains of di-zziness, headache, palpitation, tremor. For the
last several months she has been under outpatient observation for increased arteri-al
pressure. Since recently such attacks have become more frequent and severe.
Objectively: the skin is covered with clammy sweat, tremor of the extremities is
present. HR-110/min., BP- 220/140 mm Hg. Heart sounds are muffled. Blood test results:
WBCs- 9, 8 •/l, ESR- 22 mm/hour. Blood glucose - 9,8 millimole/l. What disease is the
most likely cause of this crisis?
A) Pheochromocytoma
B) Essential hypertension
C) Preeclampsia
D) Primary hyperaldosteronism
E) Diabetic glomerulosclerosis
ANSWER: A
A 32-year-old woman complains of dizziness, headache,palpitation, tremor. For the last
several months she has been under outpatient monitoring for increased arterial
pressure. Recently such attacks have become more frequent and severe. Objectively:skin
is covered with clammy sweat, tremor of the extremitiesispresent.Heartrate-
110/min,BP220/140 mm Hg. Heart sounds are weakened. In blood: WBCs - 9, 8 • 109/l, ESR
- 22 mm/h. Blood glucose - 9,8 millimole/l. What disease is the most likely cause of
this crisis?
A) Pheochromocytoma
B) Essentialhypertension
C) Preeclampsia
D) Primary hyper aldosteronism
E) Diabetic glomerulosclerosis
ANSWER: A
A 34-year-old patient after vacation in Crimea has developed severe pain in her elbow
joints, dyspnea and weakness. Body temperature is 37, 6o C, skin pallor anderythema on
the cheeks and bridge of nose are observed, lip mucosa is ulcerated. The joints are
not visibly deformed, movement of the right elbow joint is restricted. Pleura friction
sound is detected in the lungs on the right side below the angle of scapula. Heart
sounds are dull, tachycardia, gallop rhythm, heart rate - 114/min, BP - 100/60 mm Hg.
The most likely diagnosisis:
A) Systemic lupus erythematosus
B) Rheumatic carditis
C) Rheumatoid arthritis
D) Infectious allergic myocarditis
E) Dry pleurisy
ANSWER: A
A 34-year-old patient complains of profuse sweating at night, skin itching, weight
loss (9 kg within the last 3 months). Examination revealed malnutrition, skin pallor.
Palpation of neck and inguinal areas revealed dense elastic lymph nodes of about 1 cm
in diameter, nonmobile, non-adhering to skin. What is the most probable diagnosis?
A) Lymphogranulomatosis
B) Chronic lymphadenitis
C) Lymphosarcoma
D) Burkitt’s lymphoma
E) Cancer metastases
ANSWER: A
A 35-year-old female patient has gained 20 kg weight within a year with the normal
diet. She complains of chill,sleepiness,dyspnea. The patient’s mother and sister are
corpulent. Objectively: height - 160 cm, weight - 92 kg, BMI - 35,9. Obesity is
uniform, there are no striae. The face is amimic. The skin is dry. The tongue is
thickened. Heart sounds are muffled. Heart rate - 56/min, BP - 140/100 mm Hg. The
patient has constipations, amenorrhea for 5 months.TSH-28mkME/l(normal rate-0,32- 5).
Craniogram shows no pathology. What is the etiology ofobesity?
A) Hypothyroid
B) Hypoovarian
C) Hypothalamic-pituitary
D) Alimentary andconstitutive
E) Hypercorticoid
ANSWER: A
A 35-year-old female patient has gained 20 kg weight within a year with the normal
diet. She complains of chill, sleepi-ness, dyspnea. The patient’s mother and sister
are corpulent. Objectively: height - 160 cm, weight - 92 kg, BMI- 35,9. Obesity is
uniform, there are no striae. The face is amimic. The skin is dry. The tongue is
thickened. Heart sounds are muffled. Heart rate - 56/min., BP- 140/100 mm Hg. The
patient has consti-pations, amenorrhea for 5 months. TSH- 28 mkME/l (normal rate -
0,32-5). Craniogram shows no pathology. What is the etiology of obesity in this case?
A) Hypothyroid
B) Hypo-ovarian
C) Hypothalamic-pituitary
D) Alimentary and constitutive
E) Hypercorticoid
ANSWER: A
A 37-year-old man suffers from attacks of unconsciousness, dyspnea during physical
exertion, periodical sensations of heart rate disorder. Father of the patient died
suddenly at the age of 45. Objectively: heart rate is 90/min., BP is 140/90 mm Hg. On
heart US: ejection fraction - 49%, significant myocardium thickening of the left
ventricle and interventricular septum. What drug should be prescribed for the
treatment
A) Bisoprolol
B) Enalapril
C) Phenyhydinum (Nifedipine)
D) Hydrochlorothiazide
E) Furosemide
ANSWER: A
A 39-year-old patient complains of morning headache, appetite loss, nausea, morning
vomi-ting, periodic nasal haemorrhages. The patient had acute glomerulonephritis at
the age of 15. Examination revealed rise of arterial pressure up to 220/130 mm Hg,
skin haemorrhages on his arms and legs, pallor of skin and mucous membranes. What
biochemical parameter is the most important for making diagnosis in this case?
A) Blood creatinine
B) Blood bilirubin
C) Blood sodium
D) Uric acid
E) Fibrinogen
ANSWER: A
A 39-year-old patient complains of morning headache, appetite loss, nausea, morning
vomiting, periodic nasal haemorrhages. The patient had a case of acute
glomerulonephritis at the age of 15. Examination revealed rise of arterial pressure up
to 220/130 mm Hg, skin haemorrhages on his arms and legs, pallor of skin and mucous
membranes. What biochemical parameter is most important for making diagnosis in this
case?
A) Blood creatinine
B) Blood bilirubin
C) Blood sodium
D) Uric acid
E) Fibrinogen
ANSWER: A
A 40-year-old female patient complains of having a bulge on the anterior surface of
neck for 5 years. Objectively: Heart rate - 72 bpm, arterial pressure - 110/70 mm Hg,
in the right lobe of thyroid gland palpation reveals a mobile 4x2cm node,the left lobe
is not palpable, the basal metabolicrate is 6%.What is the most likely diagnosis?
A) Nodular euthyroidgoiter
B) Nodular hyperthyroidgoiter
C) Riedel’sthyroiditis
D) Mixed euthyroidgoiter
E) The median cervicalcyst
ANSWER: A
A 40-year-old female patient complains of having a bulge on the anterior surface of
neck for 5 years. Objectively: Heart rate - 72 bpm, arterial pressure - 110/70 mm Hg,
in the right lobe of thyroid gland palpation reveals a mobile 4x2 cm node, the left
lobe is not palpable, the basal metabolic rate is 6%. What is the most likely
diagnosis?
A) Nodular euthyroid goiter
B) Nodular hyperthyroid goiter
C) Riedel’s thyroiditis
D) Mixed euthyroid goiter
E) The median cervical cyst
ANSWER: A
A 40-year-old female patient complains of having a bulge on the anterior surface of
neck for 5 years. Objectively: Heart rate - 72 bpm, arterial pressure - 110/70 mm Hg,
in the right lobe of thyroid gland palpation reveals a mobi-le 4x2 cm node, the left
lobe is not palpable, the basal metabolic rate is 6%. What is the most likely
diagnosis?
A) Nodular euthyroid goiter
B) Nodular hyperthyroid goiter
C) Riedel’s thyroiditis
D) Mixed euthyroid goiter
E) The median cervical cyst
ANSWER: A
A 40-year-old female patient complains of having a bulge on the anterior surface of
neck for 5 years. Objectively: Ps- 72/min., arterial pressure - 110/70 mm Hg, in the
ri-ght lobe of thyroid gland palpation reveals a mobile node 4x2 cm in size, the left
lobe is not palpable, the basal metabolic rate is 6%. What is the most likely
diagnosis?
A) Nodular euthyroid goiter
B) Nodular hyperthyroid goiter
C) Riedel’s thyroiditis
D) Mixed euthyroid goiter
E) Median cervical cyst
ANSWER: A
A 40-year-old patient has acute onset of disease caused by overexposure to cold.
Temperature has increased up to 39oC . Foul-smelling sputum is expectorated during
coughig. Various moist crackles can be auscultated above the 3rd segment on the ri-ght.
Blood test: leukocytes - 15, 0 • 109/l, stab neutrophils - 12%, ESR- 52 mm/hour. On X-
ray: in the 3rd segment on the right there is a focus of shadow 3 cm in diameter, low
density, with fuzzy smooth margins and a clearing in its center. What disease is most
likely in the given case?
A) Pneumonia complicated by an abscess
B) Infiltrative tuberculosis
C) Peripheral pulmonary cancer
D) Cystic echinococcosis
E) Pulmonary cyst
ANSWER: A
A 40-year-old patient has acute onsetof disease caused by overexposure to cold.
Temperature has increased up to 39oC . Foul-smelling sputum is expectorated during
coughig. Various moist crackles can be auscultated above the 3rd segment on the right.
Blood test: leukocytes - 15, 0 • 109/l, stab neutrophils - 12%, ESR- 52 mm/hour. On X-
ray: in the 3rd segment on the right there is a focus of shadow 3 cm in diameter, low
density, with fuzzy smooth margins and a clearing in its center. What disease is most
likely in the given case?
A) Pneumonia complicated by an abscess
B) Infiltrative tuberculosis
C) Peripheral pulmonary cancer
D) Cystic echinococcosis
E) Pulmonary cyst
ANSWER: A
A 40-year-old patient suffers from influenza. On the 5th day of illness there are pain
behind sternum, cough with sputum, inertness. Temperature is 39, 5o C. Faceispale.
Mucosa of conjunctivas and pharynx is hyperemic. Heart rate is 120/min, breathing rate
is 38/min. In the lower lung segments shortening of percussion sound and moist rales
(crackles) can be detected. What additional investigation should be performed first of
all to specify the diagnosis?
A) Lung X-ray
B) ECG
C) Heart US
D) Mantoux test
E) Spirography
ANSWER: A
A 40-year-old patient suffers from influenza. On the 5th day of illness there are pain
behind sternum, cough with sputum, inertness. Temperature is 39, 5 oC. Face is pale.
Mucosa of conjunctivas and pharynx is hyperemic. Heart rate is 120/min, breathing rate
is 38/min. In the lower lung segments shortening of percussion sound and moist rales
(crackles) can be detected. What additional investigation should be performed first of
all to specify the diagnosis?
A) Lung X-ray
B) ECG
C) Heart US
D) Mantoux test
E) Spirography
ANSWER: A
A 40-year-old woman with a history of combined mitral valve disease with predominant
stenosis complains of dyspnea, asthma attacks at night, heart problems. At present,
she is unable to do even easy housework. What is the optimal tactics of the patient
treatment?
A) Mitral commissurotomy
B) Implantation of an artificial valve
C) Antiarrhythmic therapy
D) Treatment of heart failure
E) Antirheumatic therapy
ANSWER: A
A 41-year-old male patient was delivered to a hospital unconscious. During the
previous 7 days he had been taking large doses of biseptolum for a cold.The night
before, he began complaining of dyspnea, especially when lying down, swollen legs, 2-
day urinary retention. In the morning he had seizures and lost consciousness.
Objctively: noisy breathing at the rate of 30/min,edematous legs and lumbar region,
Ps-50/min. Plasma creatinine is 0,586mmol/l, plasma potassium-7,2mmol/l. What
treatment is necessary for this patient?
A) Hemodialysis
B) Large doses of verospiron
C) Plasma volumeex panders
D) Glucocorticosteroids
E) Heparin
ANSWER: A
A 41-year-old patient with Addison’s disease had influenza. After that he developed
adynamia, depression, nausea, vomiting, diarrhea and hypoglycemia. BP is 75/50 mm Hg.
Blood test: decreased content of corticosterone, hydrocortisone, 13-oxycorticosteroids,
17-oxycorticosteroids. What condition has developed in thepatient?
A) Acute adrenal gland insufficiency
B) Acute gastritis
C) Acute enterocolitis
D) Collapse
E) Diabetes mellitus
ANSWER: A
A 43-year-old female patient complains of dyspnea, swelling of legs, abdomen
enlargement, pricking heart pain. She has a history of tuberculousbronchoadenitis,
quinsies.The patient’s condition deteriorated 6 months ago. Objectively: cyanosis,
bulging neck veins, vesicular breathing. Heart borders are not displaced. Heart sounds
are muffled, Ps- 106/min, liver is +4cm, ascites is present. Low voltage on the ECG
has been revealed. Radiograph shows a thin layer of calcium deposits along the left
contour of heart. What treatment should be recommended to the patient?
A) Treatment by a cardiac surgeon
B) Digitalis preparations
C) Anti-TB drugs
D) Diuretics
E) Vasodilators, nitrates
ANSWER: A
A 44-year-old man complains of dyspnea with sensation of lack of air on inhale,
palpitations occurring during slight physical exertion, and shin edemas that appear in
evening and resolve in morning. His condition has been lasting for 5 months already,
deterioration is gradual. What method of instrumental diagnostics allows to verify
decrease of systolic function in this patient?
A) Echocardiography
B) Electrocardiography
C) Computed tomogram
D) Phonocardiography
E) Holter blood pressure monitoring
ANSWER: A
A 45-year-old man was delivered to a hospital with complaints of vomiting wi-th
streaks of blood, loss of weight. On esophagofiberscopy a cauliflower-shaped mucosal
growth was detected in the abdominal esophagus. The mucosa there bleeds on contact.
What preliminary diagnosis can be made?
A) Esophageal tumor
B) Barrett esophagus
C) Abdominal esophagitis
D) Esophageal diverticulum
E) Esophageal achalasia
ANSWER: A
A 45-year-old patient complains of pain in the epigastric region, left subcostal area,
abdominal distension, diarrhea, loss of wei-ght. He has been suffering from this
condition for 5 years. Objectively: tongue is moist with white coating near the root;
deep palpation of abdomen reveals slight pain in the epigastric region and Мауо-
Robson’s point. Liver is pai-nless and protrudes 1 cm from the costal arch. Spleen
cannot be palpated. What disease can be primarily suspected?
A) Chronic pancreatitis
B) Atrophic gastritis
C) Peptic stomach ulcer
D) Chronic cholecystitis
E) Acute cholecystitis
ANSWER: A
A 45-year-old patient complains of pain in the epigastric region, left subcostal area,
abdominal distension, diarrhea, loss of wei-ght. He has been suffering from this
condition for 5 years. Objectively: tongue is moist with white coating near the root;
deep palpation of abdomen reveals slight pain in the epigastric region and Мауо-
Robson’s point. Liver is pai-nless and protrudes 1 cm from the costal arch. Spleen
cannot be palpated. What disease can be primarily suspected?
A) Chronic pancreatitis
B) Atrophic gastritis
C) Peptic stomach ulcer
D) Chronic cholecystitis
E) Chronic enteritis
ANSWER: A
A 46-year-old male patient complains of periodic epigastric painthat occurs at night.
Objectively: HR-70/min, AP-125/75 mm Hg, tenderness in the epigastric region is
present. EGD confirms duodenal ulcer of 0,6cm in
diameter.TestforH.Pyloriispositive.Which of the given antisecretory drugs will be a
compulsory element of the treatment regimen?
A) Omeprazole
B) Famotidine
C) Pirenzepine
D) Atropine
E) Maalox
ANSWER: A
A 46-year-old patient with temporarily undetermined diagnosis was prescribed
pleurocentesis based on the results of the X-ray. The puncture yielded 1000 ml of a
liquid with the following properties: clear, specific gravity - 1,010, protein content
- 1%, Rivalta’s test is negative, erythrocytes - 2-3 in the field of vision. What
disorder are these pathologic changes characteristic of?
A) Cardiac failure
B) Pleuropneumonia
C) Pleural mesothelioma
D) Pulmonary tuberculosis
E) Pulmonary cancer
ANSWER: A
A 47-year-old female patient complains of cough with purulent sputum, pain in the
lower left chest, periodical body temperature rise. She has been suffering from this
condition for about 10 years. Objectively: "drumstick" distal phalanges. What
examination would be the most informative for making a diagnosis?
A) Bronchography
B) Bronchoscopy
C) Survey radiograph of lungs
D) Pleural puncture
E) Bacteriological analysis of sputum
ANSWER: A
A 47-year-old female patient complains of cough with purulent sputum, pain in the
lower left chest, periodical body temperature rise. She has been suffering from this
condition for about 10 years. Objectively: "drumstick"distal phalanges. What
examination would be the most informative for making a diagnosis?
A) Bronchography
B) Bronchoscopy
C) Survey radiograph of lungs
D) Pleural puncture
E) Bacteriological analysis of sputum
ANSWER: A
A 47-year-old male patient complains of compressive chest pain that occurs both at
rest and during light physical activity; irregular heartbeat. These problems arose 3
months ago. The patient’s brother died suddenly at the age of 30. Objectively: Ps-
84/min ,arrhythmic, AP- 130/80 mm Hg. ECG confirms signs of left ventricular
hypertrophy, abnormal Q- waves in V 4 - V 6 leads. EchoCG revealst hat
interventricular septum is 1,7cm, left ventricular wall thickness is 1,2cm.What is the
most likely diagnosis?
A) Hypertrophic cardiomyopathy
B) Neurocirculatoryasthenia
C) Exertional angina
D) Myocarditis
E) Pericarditis
ANSWER: A
A 47-year-old male patient has been lately complaining of compressing chest pain that
occurs when he walks a distance of 700-800 m. Once a week, he drinks 2 liters of beer.
Rise in arterial pressure has been observed for the last 7 years. Objectively: Ps-
74/min, AP- 120/80 mm Hg. The bicycle ergometry performed at workload of 75 watts
shows 2mm ST-segment depressionin V 4?V6 leads.What is the most likely diagnosis?
A) Exertional stenocardia, II functional class
B) Exertional stenocardia, III functional class
C) Exertional stenocardia, IV functional class
D) Vegetative-vascular dystonia of hypertensive type
E) Alcoholic cardiomyopathy
ANSWER: A
A 48-year-old man complains of constant pain in the upper abdomen, predominantly on
the left, which aggravates after eating, diarrhea, loss of weight. The patient has
alcohol use disorder. Two years ago he had a case of acute pancreatitis. Blood amylase
is 4 g/hour•l. Feces analysis: steatorrhea, creatorrhea. Blood sugar is 6,0 mmol/l.
What treatment should be prescribed?
A) Panzinorm forte (Pancreatin)
B) Insulin
C) Gastrozepin (Pirenzepine)
D) Contrykal (Aprotinin)
E) No-Spa (Drotaverine)
ANSWER: A
A 48-year-old patient has the following symptoms: diffuse enlargement of thyroid
gland,exophthalmus,weight loss up to 4kg wi- thin 2months,excessive
sweating.Objectively: heartrate 105/min,BP-180/70mmHg. Stool is normal. What therapy
is advisable in the given case?
A) Mercazolil (Thiamazole)
B) Potassium iodide
C) Propranolol
D) Iodomarin
E) Thyroxin
ANSWER: A
A 48-year-old patient was found to have diffuse enlargement of the thyroid gland,
exophthalmia, weight loss of 4 kg in 2 months, sweating. Objectively: HR- 105/min.,
BP-140/70 mm Hg. Defecation act is normal. What kind of therapy is recommended in this
case?
A) Mercazolil
B) Radioiodine
C) Propranolol
D) Lugol’s solution
E) Thyroxine
ANSWER: A
A 48-year-old woman has been suffering from chronic pancreatitis for the last 7 years.
Lately she has been noticing an increase in daily feces with foul smell, abdominal di-
stention, gurgling. The patient complains of diarrhea, weakness, fatigability, loss of
appeti-te, loss of weight. What syndrome can be suspected in this case?
A) Malabsorption
B) Irritable colon
C) Maldigestion
D) Exudative enteropathy
E) Endocrine gland failure
ANSWER: A
A 49-year-old female patient has type 1 diabetes of moderate severity.The disease is
complicated by retinopathy and polyneuropathy. Besides that, repeated analyses of the
daily urinary excretion of albumin revealed microalbuminuria(200-
300mg/day).Glomerular filtration rate is 105ml/min. Blood pressure is within normal
range. Normalization of the following indicator should be the first-priority task in
the secondary prevention of diabetic nephropathy:
A) Glycosylated hemoglobin
B) C-peptide
C) Blood insulin
D) Fastingglucose
E) Glycemia 2hours after a meal
ANSWER: A
A 49-year-old male patient complains of retrosternal pain, heartburn,weight loss of 8
kg over the last year,constipation,weakness. The patient has been a smoker for 20years,
and has a10-year history of gastroesophageal reflux disease.The patient is
asthenic,has dry skin. EGD revealed an ulcer in the lower third of the sophagus and
esophageal stricture accompanied by edema,hyperemia and multiple erosions of the
mucosa.What study is required for more accurate diagnosis?
A) Biopsy of the esophageal mucosa
B) X-rayexamination of the esophagus
C) Respiratory test for Helico bacter pylori
D) pH-metry of the esophagus and the stomach
E) Fecal occult blood test
ANSWER: A
A 49-year-old patient 2 years ago was diagnosedwithstage1silicosis. He complains of
increased dyspnea and pain in the infrascapular regions. X-ray shows diffuse
intensification and distortion of lung pattern with numerous nodularshadows2-
4mmindiameter.Hardening of right interlobar pleura is detected. Lung roots are
hardened and expanded. What X-ray type of pneumosclerosis does the patient have?
A) Nodular
B) Interstitial
C) Interstitial-nodular
D) Nodal
E) Tumor-like
ANSWER: A
A 49-year-old patient complains of swallowing disorder that intensifies duri-ng eating
solid food, hiccups, hoarse voice, nausea, regurgitation, significant weight loss (15
kg within 2,5 months). Objectively: body weight is reduced; the skin is pale and dry;
vesicular respiration; heart sounds are suffi-ciently sonorous; heart rate is rhythmic.
The abdomen is soft, no pain on palpation. The liver is not enlarged. What
investigation is most necessary for making the diagnosis in this case?
A) Esophagoduodenoscopy with biopsy
B) Clinical blood analysis
C) X-ray of the gastrointestinal tract
D) X-ray in the Trendelenburg position
E) Investigation of gastric secretion
ANSWER: A
A 50-year-old man, who works as a polisher at a combine-building factory, addressed
the factory’s sectorial doctor with complaints of general fatigue, sensations of
numbness and pain in his fingers. Objectively: the skin of his fingers is pale.
Reaction to pain, tactile and thermal stimuli was revealed to be slightly disrupted.
No disruptions can be observed within the other organs and systems. What disorder is
the most likely?
A) Pneumatic hammer disease
B) Multiple neuritis
C) Raynaud’s disease
D) Syringomyelia
E) Deforming arthrosis
ANSWER: A
A 50-year-old patient complains about having pain attacks in the right subcostal area
for about a year. The pain arises mainly after taking fatty food. Over the last week
the attacks occurred daily and became more pai-nful. On the 3rd day of hospitalization
the pati-ent presented with icteritiousness of skin and scleras, light-coloured feces
and dark urine. In blood: neutrophilic leukocytosis - 13, 1 • 109/l, ESR - 28 mm/h.
What is the most likely diagnosis?
A) Chronic calculous cholecystitis
B) Chronic recurrent pancreatitis
C) Fatty degeneration of liver
D) Chronic cholangitis, exacerbation stage
E) Hypertensive dyskinesia of gallbladder
ANSWER: A
A 51-year-old female patient complains of frequent defecation and liquid blood-
streaked stools with mucus admixtures, diffuse pain in the inferolateral abdomen, 6 kg
weight loss within the previous month. Objectively: body temperature - 37, 4oC ,
malnutrition, skin is pale and dry. Abdomen is soft, sigmoid is pai-nful and spasmodic,
makes a rumbling sound. Liver is dense, painful, extends 3 cm below the costal margin.
What is the most likely di-agnosis?
A) Non-specific ulcerative colitis
B) Bacillary dysentery
C) Sprue
D) Intestinal enzymopathy
E) Helminthic invasion
ANSWER: A
A 52-year-old patient complains of pain in the right part of her chest, dyspnea, cough
with a lot of albuminoid sputum emitting foul smell of "meat slops". Objectively: the
patient’s condition is grave, cyanosis is observed, breathing rate is 31/min,
percussion sound above the right lung is shortened, auscultation revealed various
moist rales (crackles). What is the most probable diagnosis?
A) Lung gangrene
B) Lung abscess
C) Pleura empyema
D) Multiple bronchiectasis
E) Chronic pneumonia
ANSWER: A
A 52-year-old patient, who has been suffering from angina pectoris for 2 weeks, has
more and more frequent pain attacks in the area behind his sternum and his need for
nitroglycerine increased. Objectively: the condition is of moderate severity. Skin is
pale. Heart sounds are weakened, rhythmic. Heart rate is 76 per minute. ECG shows no
signs of focal myocardial damage. What is the most likely diagnosis?
A) Progressive angina pectoris
B) First-time angina pectoris
C) Stable FC II angina pectoris
D) Variant angina pectoris
E) Acute cardiac infarction
ANSWER: A
A 53-year-old woman complains of weight loss up to 10 kg within the last 2 years,
liquid foul-smelling stool two times a day that poorly washes off the toilet, periodic
bouts of nausea, girdle pain in the upper abdomen. Objecti-vely: pain in Gubergrits
zone (on the right from navel) and Mayo-Robson’s point. Biochemical blood analysis:
glucose - 3,2 mmol/l, bilirubin - 16,5 micromole/l, crude protein - 56,4 g/l. Urine
diastase - 426 g/h/l. D-xylose test (oral admi-nistration of 25 g of d-xylose) after 5
hours reveals 3 g of xylose in urine. The most likely diagnosis is:
A) Pancreatitis. Malabsorption syndrome
B) Pseudomembranous colitis
C) Nonspecific ulcerative colitis
D) Irritable bowel syndrome
E) Chronic gastritis
ANSWER: A
A 54-year-old man had been drowning at sea, when he was found and evacuated to the
shore. Objectively: unconscious, pale face, no breathing can be auscultated, thready
pulse. Resuscitation measures allowed to save the man. What complication can develop
in him in the nearest future?
A) Pulmonary edema
B) Respiratory arrest
C) Encephalopathy
D) Cardiac arrest
E) Bronchial spasm
ANSWER: A
A 54-year-old patient complains of weakness, jaundice, itching skin. Disease onset was
1,5 months ago: fever up to 39oC appeared at first, with progressive jaundice
developed 2 weeks later. On hospitalisation jaundice was severely progressed. Liver
cannot be palpated. Gall bladder is enlarged and painless. Blood bilirubin is 190
micromole/l (accounting for direct bilirubin). Stool is acholic. What is the most
likely jaundice genesis in this patient?
A) Mechanical jaundice
B) Hepatocellular jaundice
C) Hemolytic jaundice
D) Caroli syndrome
E) Gilbert’s syndrome
ANSWER: A
A 54-year-old patient complains of weakness, weight loss despite the unchanged
appetite, frequent urination, skin itch for six months. Some time ago the patient
underwent treatment for furunculosis. She has not been examined recently. Objectively:
malnutrition, dry skin with scratch marks. Small lymph nodes can be palpated in the
axillary regions. Changes in the internal organs are absenr. What test must be
performed in the first place?
A) Fasting blood sugar
B) Complete blood count
C) Endoscopy of stomach
D) Lymph node biopsy
E) Blood sterility testing
ANSWER: A
A 54-year-old patient complains of weakness, weight loss despite the unchanged
appetite, frequent urination, skin itch for six months. Some time ago the patient
underwent treatment for furunculosis. She has not been examined recently. Objectively:
malnutrition, dry skin with scratch marks. Small lymph nodes can be palpated in the
axillary regions. Changes in the internal organs are absenr. What test must be
performed in the first place?
A) Fasting blood sugar
B) Complete blood count
C) Endoscopy of stomach
D) Lymph node biopsy
E) Blood sterility testing
ANSWER: A
A 54-year-old patient has been suffering from diabetes mellitus for 5 years, with diet
being his only treatment. Within the last half a year he lost 7 kg of body weight,
complains of thirst, vertigo when raising from bed, decrease of erectile function,
frequent stool, especially at night. Objectively: malnutrition, dry skin. BP in lying
position is 160/90 mm Hg; BP in standing position is 170/85 mm Hg. No edemas. Fasting
plasma glucose level is 12 mmol/l. Glycated hemoglobin accounts for 11%. Albumin
excreted with urine is 20mg per day.The most likely diagnosis is:
A) Diabetes mellitus type 2 with visceral neuropathy
B) Diabetes mellitus type 1with ketoacidosis
C) Diabetes mellitus type 1 with encephalomyelopathy
D) Diabetes mellitus type 2 with nephropathy
E) Diabetes mellitus type 2 with polyneuropathy
ANSWER: A
A 56-year-old patient complains of pain in the epigastrium after eating, eructation,
loss of appetite, slight loss of weight, fatigabili-ty. The patient smokes; no
excessive alcohol consumption. Objectively: pale mucosa, BP-110/70 mm Hg. The tongue
is ”lacquered”. The abdomen is soft, sensitive in the epi-gastric area. Blood test:
erythrocytes - 3,0 T/l, Hb- 110 g/l, color index - 1,1; macrocytosis; leukocytes - 5,5
g/l, ESR- 13 mm/hour. On fibrogastroduodenoscopy: atrophy of fundic mucosa. What
pathogenesis does this disorder have?
A) Producing antibodies to parietal cells
B) Н.pyloripersistence
C) Alimentary factor
D) Chemical factor
E) Gastropathic effect
ANSWER: A
A 56-year-old patient with diffuse toxic goiter has ciliary arrhythmia, heart rate is
110-120/min., arterial hypertension, BP is 165/90 mm Hg. What drug besides Mercazolil
(Thi-amazole) should be prescribed in this case?
A) Propranolol
B) Radioactive iodine
C) Novocainamide (procainamide)
D) Verapamil
E) Corinfar
ANSWER: A
A 56-year-old patient with diffuse toxic goiter has ciliary arrhythmia, heart rate is
110-120/min., arterial hypertension, BP is 165/90 mm Hg. What drug besides Mercazolil
(Thi-amazole) should be prescribed in this case?
A) Propranolol
B) Radioactive iodine
C) Novocainamide (procainamide)
D) Verapamil
E) Corinfar
ANSWER: A
A 56-year-old woman complains of itchi-ng skin of her torso, constant nausea,
constipation, sensation of heaviness and pain in the right subcostal area, extreme
general fatigue. The patient suffers from biliary cirrhosis. The skin is pale icteric.
The abdomen is soft, the liver protrudes 2,0 cm from under the margin of the right
costal arch, sensitive on palpation. Biochemical investigation: total bilirubin -
142,0 mcmol/l, conjugated bilirubin - 139,0 mcmol/l, alanine aminotransferase - 0,98
mmol/hour•l, aspartate aminotransferase - 0,82 mmol/hour•l, alkaline phosphatase -
8,7 mmol/hour•l. What drug should be prescribed in the first place?
A) Ursodeoxycholic acid
B) Sirepar
C) Allochol
D) Essentiale forte (Phospholipides)
E) Livolin forte
ANSWER: A
A 57-year-old male patient had an attack of retrosternal pain that lasted more than
1,5 hours. Objectively: the patient is inert, adynamic, has pale skin, cold
extremities, poor volume pulse, heart rate-120/min, AP-70/40 mm Hg. ECG shows ST
elevationin II, III, aVF leads. What condition are these changes typical for?
A) Cardiogenic shock
B) Arrhythmogenic shock
C) Perforated gastriculcer
D) Acute pericarditis
E) Acute pancreatitis
ANSWER: A
A 57-year-old woman complains of havi-ng a sensation of esophageal compresion,
palpitation, difficult breathing during eating solid food, occasional vomiting with a
full mouth, ”wet pillow” sign at night for the last 6 months. Objectively: body
tempearture - 39oC , height - 168 cm, weight - 72 kg, Ps-76/min., BP- 120/80 mm Hg. X-
ray revealed a considerable dilation of the esophagus and its constriction in the
cardial part. What pathology is most likely to have caused dysphagia in this patient?
A) Achalasia cardiae
B) Primary esophagism
C) Hiatal hernia
D) Esophageal carcinoma
E) Reflux esophagitis
ANSWER: A
A 57-year-old woman complains of having a sensation of esophageal compresion, palpi-
tation, breathing difficulties when eating solid food, occasional vomiting with a full
mouth, "wet pillow"sign at night for the last 6 months. Objectively: body tempearture
- 39oC , height - 172cm, weight - 72 kg, heart rate - 76/min, BP-120/80 mm Hg. X-ray
revealed considerable dilation of esophagus and its constriction in the cardial part.
What pathology is most likely to have caused dysphagia in this patient?
A) Achalasia cardiae
B) Primary esophagism
C) Hiatal hernia
D) Esophageal carcinoma
E) Reflux esophagitis
ANSWER: A
A 58-year-old female patient complains of spontaneous bruises, weakness, bleeding
gums,dizziness.Objectively:the mucous membranes and skin are pale with numerous
hemorrhages of various time of origin. Lymph nodes are not enlarged.Ps-100/min,AP-
110/70 mmHg.There are no changes of internal organs. Blood test results:RBC-
3,0•1012/l, Нb-92g/l,colour index-0,9, anisocytosis, poikilocytosis,WBC-10•109/l,
eosinophils -2%,stab neutrophils-12%, segmented neutrophils-68%,lymphocytes - 11%,
monocytes-7%, ESR-12mm/h.What laboratory test is to be determined next for making
adiagnosis?
A) Platelets
B) Reticulocytes
C) Clottingtime
D) Osmotic resistance of erythrocytes
E) Fibrinogen
ANSWER: A
A 58-year-old patient complains of general weakness, weight loss up to 10 kg within
the last 1,5 months, progressive pain in the small of the back, raise of blood
pressure to 220/160 mm Hg, subfebrile body temperature. Objectively: tuberous slightly
movable lump can be palpated in the right subcostal area; veins of spermatic cord and
scrotum are dilated. Blood test: Hb - 86 g/l, ESR - 44 mm/h. Urine test: specific
gravity - 1020, proteine - 0,99 g/l,erythrocytes-all field of vision, leukocytes4-6 in
the field of vision. The provisional diagnosisis:
A) Kidney tumor
B) Urolithiasis
C) Acute pyelonephritis
D) Acute glomerulonephritis
E) Nephroptosis
ANSWER: A
A 58-year-old patient complains of headache in the occipital region, nausea, choking,
opplotentes. The presentations appeared after a physical exertion. Objectively: the
patient is excited. Face is hyperemic. Skin is pale. Heart sounds are regular, the 2nd
aortic sound is accentuated. BP - 240/120 mm Hg, heart rate - 92/min. Auscultation
reveals some fine moist rales (crackles) in the lower parts of the lungs. Liver is not
enlarged. ECG shows signs of hypertrophy and left ventricular overload. What is the
most likely diagnosis?
A) Complicated hypertensive crisis
B) Acute myocardial infarction, pulmonaryedema
C) Bronchial asthma exacerbation
D) Uncomplicated hypertensic crisis
E) Community-acquired pneumonia
ANSWER: A
A 58-year-old patient was delivered to an admission room with complaints of pain in
the thorax on the left. On clinical examination: aside from tachycardia (102/min.) no
other changes. On ECG: pathologic wave Q in I, аVL, QS in V1, V2, V3 leads and ’domed’
ST elevation with negative T. What diagnosis is the most likely?
A) Acute left ventricular anterior myocardialinfarction
B) Variant angina pectoris
C) Aortic dissection
D) Acute left ventricular posterior myocardialinfarction
E) Exudative pericarditis
ANSWER: A
A 60-year-old woman developed weakness, vertigo, rapid fatigability during the last
year. Recently there have been dyspnea and paresthesia observed. Objectively: skin and
mucous membranes are pale and icteric. Lingual papillae are smoothed out. Liver and
spleen are situated at the edge of costal arch. Blood test: Hb- 70 g/l, erythrocytes -
1, 7•1012 /l, blood color index - 1,2, macrocytes. What drug can be prescribed on
pathogenetic grounds?
A) VitaminB12
B) VitaminB6
C) Ascorbic acid
D) Iron preparations
E) VitaminB1
ANSWER: A
A 60-year-old woman started feeling weakness, vertigo, rapid fatigability during the
last year. Recently there have been dyspnea and paresthesia observed. Objectively:
skin and mucous membranes are pale and icteric. Lingual papillae are smoothed out.
Liver and spleen are situated at the edge of costal arch. Bloodtest: Hb-
70g/l,erythrocytes-1,7•1012/l, blood colorindex-1,2, macrocytes.What drug can be
prescribed on pathogenetic grounds?
A) Vitamin B12
B) Vitamin B6
C) Ascorbic acid
D) Iron preparations
E) Vitamin B1
ANSWER: A
A 62-year-old man addresed a urologist with complaints of frequent urination at night
(5-6 times per night), sensation of incomplete voiding of the urinary bladder, pain in
the lower abdomen, slow urination. Anamnesis: the II degree essential hypertension
(peak BP is 160/100 mm Hg). Current case: the II degree enlargement of the prostate
gland, PSA is 2,2 ng/ml. Select the drug suitable for long-term therapy of the
patient’s combined pathology:
A) Doxazosin
B) Propranolol
C) Indapamide
D) Amlodipine
E) Captopril
ANSWER: A
A 63-year-old female complains of general weakness, a feeling of heaviness,
compression in the epigastrium, postprandial fullness, nausea, belching after
meals.These symptoms have been observed for about 15 years. Objectively: body
temperatureis 36, 4oC, respiratory rate-20/min, Ps-88/min, blood pressure-115/75 mm Hg.
Skin and mucous membranes are pale. Blood test results:RBC- 2,0•1012/l, Hb-
100g/l.Tests revealed parietal- cell antibodies. What is the most likely reason for
the development of anemia in this patient?
A) Production of antibodies to intrinsic factor
B) Disruption of hemoglobin synthesis
C) Disruption of erythropoietin synthesis
D) Impaired iron absorption
E) Increased lossofiron
ANSWER: A
A 63-year-old male patient with persistent atrial fibrillation complains of moderate
dyspnea. Objectively: peripheral edemata are absent, vesicular breathing is present,
heart rate-72/min, AP-140/90 mm Hg.What combination of drugs will be most effective
for the secondary prevention of heart failure?
A) Beta-blockers, ACE inhibitors
B) Beta-blockers, cardiac glycosides
C) Cardiac glycosides, diuretics
D) Cardiac glycosides, ACE inhibitors
E) Diuretics, beta-blockers
ANSWER: A
A 64-year-old patient has been hospi-talised with complaints of progressive jaundi-ce
that developed over 3 weeks ago without pain syndrome, along with general weakness,
loss of appetite. Objectively: temperature is 36, 8oC , heart rate is 78/min., abdomen
is soft and painless, peritoneum irritation symptoms are not detected, palpation
reveals sharply enlarged tense gallbladder. What disease can be characterised with
these symptoms?
A) Cancer of pancreas head
B) Duodenal ulcer
C) Acute cholecystitis
D) Chronic cholecystitis
E) Cholecystitis caused by lambliasis
ANSWER: A
A 64-year-old patient has been hospitalised with complaints of progressive jaundice
that developed over 3 weeks ago without pain syndrome, general weakness, loss of
appetite. Objectively: temperature is 36, 8oC , heart rate is 78/min, abdomen is soft
and painless, peri-toneum irritation symptoms are not detected, palpation reveals
sharply enlarged tense gallbladder. What disease can be characterised with these
symptoms?
A) Cancer of pancreas head
B) Duodenal ulcer
C) Acute cholecystitis
D) Chronic cholecystitis
E) Cholecystitis caused by lambliasis
ANSWER: A
A 57-year-old woman complains of having a sensation of esophageal compresion, palpi-
tation, breathing difficulties when eating solid food, occasional vomiting with a full
mouth, "wet pillow"sign at night for the last 6 months. Objectively: body tempearture
- 39oC , height - 172cm, weight - 72 kg, heart rate - 76/min, BP-120/80 mm Hg. X-ray
revealed considerable dilation of esophagus and its constriction in the cardial part.
What pathology is most likely to have caused dysphagia in this patient?
A) Achalasia cardiae
B) Primary esophagism
C) Hiatal hernia
D) Esophageal carcinoma
E) Reflux esophagitis
ANSWER: A
A 58-year-old female patient complains of spontaneous bruises, weakness, bleeding
gums,dizziness.Objectively:the mucous membranes and skin are pale with numerous
hemorrhages of various time of origin. Lymph nodes are not enlarged.Ps-100/min,AP-
110/70 mmHg.There are no changes of internal organs. Blood test results:RBC-
3,0•1012/l, Нb-92g/l,colour index-0,9, anisocytosis, poikilocytosis,WBC-10•109/l,
eosinophils -2%,stab neutrophils-12%, segmented neutrophils-68%,lymphocytes - 11%,
monocytes-7%, ESR-12mm/h.What laboratory test is to be determined next for making
adiagnosis?
A) Platelets
B) Reticulocytes
C) Clottingtime
D) Osmotic resistance of erythrocytes
E) Fibrinogen
ANSWER: A
A 58-year-old patient complains of general weakness, weight loss up to 10 kg within
the last 1,5 months, progressive pain in the small of the back, raise of blood
pressure to 220/160 mm Hg, subfebrile body temperature. Objectively: tuberous slightly
movable lump can be palpated in the right subcostal area; veins of spermatic cord and
scrotum are dilated. Blood test: Hb - 86 g/l, ESR - 44 mm/h. Urine test: specific
gravity - 1020, proteine - 0,99 g/l,erythrocytes-all field of vision, leukocytes4-6 in
the field of vision. The provisional diagnosisis:
A) Kidney tumor
B) Urolithiasis
C) Acute pyelonephritis
D) Acute glomerulonephritis
E) Nephroptosis
ANSWER: A
A 58-year-old patient complains of headache in the occipital region, nausea, choking,
opplotentes. The presentations appeared after a physical exertion. Objectively: the
patient is excited. Face is hyperemic. Skin is pale. Heart sounds are regular, the 2nd
aortic sound is accentuated. BP - 240/120 mm Hg, heart rate - 92/min. Auscultation
reveals some fine moist rales (crackles) in the lower parts of the lungs. Liver is not
enlarged. ECG shows signs of hypertrophy and left ventricular overload. What is the
most likely diagnosis?
A) Complicated hypertensive crisis
B) Acute myocardial infarction, pulmonaryedema
C) Bronchial asthma exacerbation
D) Uncomplicated hypertensic crisis
E) Community-acquired pneumonia
ANSWER: A
A 58-year-old patient was delivered to an admission room with complaints of pain in
the thorax on the left. On clinical examination: aside from tachycardia (102/min.) no
other changes. On ECG: pathologic wave Q in I, аVL, QS in V1, V2, V3 leads and ’domed’
ST elevation with negative T. What diagnosis is the most likely?
A) Acute left ventricular anterior myocardialinfarction
B) Variant angina pectoris
C) Aortic dissection
D) Acute left ventricular posterior myocardialinfarction
E) Exudative pericarditis
ANSWER: A
A 60-year-old woman developed weakness, vertigo, rapid fatigability during the last
year. Recently there have been dyspnea and paresthesia observed. Objectively: skin and
mucous membranes are pale and icteric. Lingual papillae are smoothed out. Liver and
spleen are situated at the edge of costal arch. Blood test: Hb- 70 g/l, erythrocytes -
1, 7•1012 /l, blood color index - 1,2, macrocytes. What drug can be prescribed on
pathogenetic grounds?
A) VitaminB12
B) VitaminB6
C) Ascorbic acid
D) Iron preparations
E) VitaminB1
ANSWER: A
A 60-year-old woman started feeling weakness, vertigo, rapid fatigability during the
last year. Recently there have been dyspnea and paresthesia observed. Objectively:
skin and mucous membranes are pale and icteric. Lingual papillae are smoothed out.
Liver and spleen are situated at the edge of costal arch. Bloodtest: Hb-
70g/l,erythrocytes-1,7•1012/l, blood colorindex-1,2, macrocytes.What drug can be
prescribed on pathogenetic grounds?
A) Vitamin B12
B) Vitamin B6
C) Ascorbic acid
D) Iron preparations
E) Vitamin B1
ANSWER: A
A 62-year-old man addresed a urologist with complaints of frequent urination at night
(5-6 times per night), sensation of incomplete voiding of the urinary bladder, pain in
the lower abdomen, slow urination. Anamnesis: the II degree essential hypertension
(peak BP is 160/100 mm Hg). Current case: the II degree enlargement of the prostate
gland, PSA is 2,2 ng/ml. Select the drug suitable for long-term therapy of the
patient’s combined pathology:
A) Doxazosin
B) Propranolol
C) Indapamide
D) Amlodipine
E) Captopril
ANSWER: A
A 63-year-old female complains of general weakness, a feeling of heaviness,
compression in the epigastrium, postprandial fullness, nausea, belching after
meals.These symptoms have been observed for about 15 years. Objectively: body
temperatureis 36, 4oC, respiratory rate-20/min, Ps-88/min, blood pressure-115/75 mm Hg.
Skin and mucous membranes are pale. Blood test results:RBC- 2,0•1012/l, Hb-
100g/l.Tests revealed parietal- cell antibodies. What is the most likely reason for
the development of anemia in this patient?
A) Production of antibodies to intrinsic factor
B) Disruption of hemoglobin synthesis
C) Disruption of erythropoietin synthesis
D) Impaired iron absorption
E) Increased lossofiron
ANSWER: A
A 63-year-old male patient with persistent atrial fibrillation complains of moderate
dyspnea. Objectively: peripheral edemata are absent, vesicular breathing is present,
heart rate-72/min, AP-140/90 mm Hg.What combination of drugs will be most effective
for the secondary prevention of heart failure?
A) Beta-blockers, ACE inhibitors
B) Beta-blockers, cardiac glycosides
C) Cardiac glycosides, diuretics
D) Cardiac glycosides, ACE inhibitors
E) Diuretics, beta-blockers
ANSWER: A
A 64-year-old patient has been hospi-talised with complaints of progressive jaundi-ce
that developed over 3 weeks ago without pain syndrome, along with general weakness,
loss of appetite. Objectively: temperature is 36, 8oC , heart rate is 78/min., abdomen
is soft and painless, peritoneum irritation symptoms are not detected, palpation
reveals sharply enlarged tense gallbladder. What disease can be characterised with
these symptoms?
A) Cancer of pancreas head
B) Duodenal ulcer
C) Acute cholecystitis
D) Chronic cholecystitis
E) Cholecystitis caused by lambliasis
ANSWER: A
A 64-year-old patient has been hospitalised with complaints of progressive jaundice
that developed over 3 weeks ago without pain syndrome, general weakness, loss of
appetite. Objectively: temperature is 36, 8oC , heart rate is 78/min, abdomen is soft
and painless, peri-toneum irritation symptoms are not detected, palpation reveals
sharply enlarged tense gallbladder. What disease can be characterised with these
symptoms?
A) Cancer of pancreas head
B) Duodenal ulcer
C) Acute cholecystitis
D) Chronic cholecystitis
E) Cholecystitis caused by lambliasis
ANSWER: A
A 65-year-old male patient complains of dyspnea that is getting worse with exertion,
morning cough with expectoration of mucous sputum. For about15years, he has been
subject to regular medical check-up for chronic bronchitis. The patient takes berodual
(16inhaled doses per day). Objectively: body temperatureis 36, 8 oC, RR-24/min, Ps-
110/min, AP-145/90 mm Hg. Auscultation reveals a lot of dry rales above the lungs.
FEV1-65%. What is the optimal tactics of further management of the patient?
A) To administer inhalation of corticosteroids
B) To administer antibiotics
C) To administer theophylline
D) Toincreasethedailydoseofberodual
E) To include short-acting ? 2-agonists into the therapy
ANSWER: A
A 67-year-old female patient with hypertensive crisis has asthma, cough with
expectoration of frothypink sputum, moist rales in the lungs. The patient stays in
sitting position, respiratory rate is 40/min, AP- 214/136 mm Hg, heart rate-
102/min.What is the most rational tactics of this patient management?
A) Intravenous administration of furosemide
B) Urgent pneumography
C) Bed rest, lying position
D) Intravenous administration of a ?-blocker
E) Tactics can be determined after ECG and chest radiography
ANSWER: A
A 67-year-old man complains of dyspnea on exertion, attacks of retrosternal pain,
dizziness. He has no history of rheumatism. Objectively: pale skin, acrocyanosis.
There are crackles in the lower lungs. There is systolic thrill in the II intercostal
space on the right, coarse systolic murmur conducted to the vessels of neck. BP-
130/90 mm Hg, heart rate - 90/min., regular rhythm. The liver extends 5 cm from under
the edge of costal arch, shin edemas are present. Specify the suspected valvular
defect:
A) Aortic stenosis
B) Pulmonary artery stenosis
C) Mitral insufficiency
D) Ventricular septal defect
E) Tricuspid regurgitation
ANSWER: A
A 70-year-old patient consulted a doctor about arrhythmic cardiac activity, dyspnea.
Objectively: BP - 150/90 mm Hg, extrasystole arrhythmia (10-12 beats per minute), left
ventricular systolic dysfunction (ejection fraction at the rate of 42%). Which of
antiarrhythmic drugs should be administered as initial therapy in this case?
A) Amiodarone
B) Flecainide
C) Encainide
D) Moracizine
E) Digoxin
ANSWER: A
A patient complains of fatigue, lack of appetite, pain and burning sensation in the
tongue, numbness of the distal limbs, diarrhea. Objectively: pale skin with lemon-
yellow tint, face puffiness, brown pigmentation in the form of a "butterfly", bright
red areas on the tongue. The liver is 3 cm below the costal margin, soft. Blood count:
RBCs - 1, 5 • 1012/l, colour index -1,2, WBCs - 3, 8•109 /l, thrombocytes - 180•109
/l, eosinophiles - 0%, stab neutrophiles - 1%, segmented neutrophiles - 58%,
lymphocytes - 38% monocytes - 3%, RBC macrocytosis. ESR - 28 mm/h. What diagnosis are
these presentations typical for?
A) B12-deficiency anemia
B) Iron deficiency anemia
C) Aplastic anemia
D) Acute erythromyelosis
E) Chronic adrenal failure
ANSWER: A
A patient complains of weight gain, chill, edemas, xeroderma, somnolence, difficulties
with focusing. Objectively: height is 165 cm; weightis90kg;body proportions are of
female type, to- 35, 8o C, heart rate - 58/min, BP -105/60 mm Hg. Heart sounds are
weakened, bradycardia is observed. Other internal organs have no changes. Thyroid
gland cannot be palpated. Milk secretion from mammary glands is observed. Hormone
investigation revealed increased levels of thyroid-stimulating and level of
thyroxine(T4).Which one is the cause for obesity?
A) Primary hypothyroidism
B) Secondary hypothyroidism
C) Prolactinoma
D) Hypopituitarism
E) Adiposogenitaldystrophy
ANSWER: A
A patient complains of weight gain, chill, edemas, xeroderma, somnolence, difficulties
with focusing. Objectively: height is 165 cm; weight is 90 kg; body proportions are of
female type, to- 35, 8oC , heart rate - 58/min, BP - 105/60 mm Hg. Heart sounds are
weakened, bradycardia is observed. Other internal organs have no changes. Thyroid
gland cannot be palpated. Milk secretion from mammary glands is observed. Hormone
investigation revealed increased levels of thyroid-stimulating hormone (TSH) and
prolactin, and decreased level of thyroxine (T4). Which one is the cause for obesity?
A) Primary hypothyroidism
B) Secondary hypothyroidism
C) Prolactinoma
D) Hypopituitarism
E) Adiposogenital dystrophy
ANSWER: A
A patient has been provisionally di-agnosed with pheochromocytoma at the stage of
intermission. BP is within norm, there is a tendency towards tachycardia. No urine
pathologies. The decision has been made to perform a provocative test with histamine.
What drug should be kept close at hand for emergency aid in case of positive test
result?
A) Phentolamine
B) Pipolphen
C) Nifedipine
D) Mesaton (Phenylephrine)
E) Prednisolone
ANSWER: A
A patient with autoimmune thyroiditis accompanied by multinodular goiter underwent the
right lobeectomy and subtotal resection of the left lobe.What drug should be
administered to prevent postoperative hypothyroidism?
A) L-thyroxine
B) Merkazolil
C) Iodomarin
D) Lithium drugs
E) Insulin
ANSWER: A
A patient’s X-ray image (anteroposterior projection) shows deformation of lung pattern,
pneumofibrosis, reticular (honeycomb) lung pattern of lower pulmonary
segments,cylindric and fusiform lumps. The most likely diagnosis is:
A) Multiple bronchiectasis
B) Right lungabscess
C) Non-hospital-acquired pneumonia
D) Lung small development
E) Diffuse pulmonary fibrosis
ANSWER: A
A patient’s X-ray image (anteroposterior projection) shows deformation of lung pattern,
pneumofibrosis, reticular (honey comb) lung pattern of lower pulmonary segments,
cylindric and fusiform lumps. The most likely diagnosis is:
A) Multiple bronchiectasis
B) Right lung abscess
C) Non-hospital-acquired pneumonia
D) Lungs maldevelopment
E) Diffuse pulmonary fibrosis
ANSWER: A
A tractor driver with the record of service of 24 years has undergone palestesiometry
test (Vibration Sensitivity Measurement). Test revealed increased vibration
sensitivity threshold at the frequencies of 63-125-259 Hz to 25 dB. Dynamometry is 20
kg on the right and 16 kg on the left. Cold stimulustestis positive, time of hand
temeperature restoration is 52 minutes. Blanching at pressure symptom is positive and
equals 21. Hypesthesia of upper and lower limbs is observed and can be classified as
"gloves"and "socks"polyneuritic pattern. Make the provisional diagnosis.
A) Vibration disease, I stage, caused by combinedvibration
B) Vibration disease, I stage, caused by local vibration
C) Vibration disease, II stage, caused by combined vibration
D) Vibration disease, II stage, caused by local vibration
E) Vibration disease, I stage, caused by general vibration
ANSWER: A
A woman complains of muscle weakness and general fatigue, dyspnea, vertigo,
brittleness of her hair and nails, an urge to eat chalk. Anamnesis states uterine
fibroid. Common blood analysis: erythrocytes - 2,8 Т/l, Hb- 80 g/l, color index - 0,78,
anisocytosis, poikilocythemia, serum iron - 10 mcmol/l. What diagnosis is most likely?
A) Iron-deficiency anemia
B) B12-deficient anemia
C) Autoimmune hemolytic anemia
D) Aplastic anemia
E) Hypoplastic anemia
ANSWER: A
A woman consulted a therapeutist about fatigability, significant weight loss, weakness,
loss of appetite. She has been having amenorrhea for 8 months. A year ago she born a
full-term child. Haemorrhage during labour made up 2 l. She got blood and blood
substi-tute transfusions. What is the most probable diagnosis?
A) Sheehan’s syndrome
B) Stein-Leventhal syndrome
C) Shereshevsky-Turner’s syndrome
D) Homological blood syndrome
E) Vegetovascular dystonia
ANSWER: A
A woman has developed sudden thoracic pain on the right with expectoration of pink
sputum and body temperature rise up to 37, 7oC on the 4th day after the surgery for
cystoma of the right ovary. On lung examination: dullness of the lung sound on the
lower right is observed. Isolated moist crackles can be auscultated in the same area.
What complication is the most likely?
A) Pulmonary infarction
B) Pneumonia
C) Pulmonary abscess
D) Exudative pleurisy
E) Pneumothorax
ANSWER: A
A woman has developed sudden thoracicpain on the right with expectoration of pi-nk
sputum and body temperature rise up to 37, 7oC on the 4th day after the surgery for
cystoma of the right ovary. On lung examination: dullness of the lung sound on the
lower right is observed. Isolated moist crackles can be auscultated in the same area.
What complication is the most likely?
A) Pulmonary infarction
B) Pneumonia
C) Pulmonary abscess
D) Exudative pleurisy
E) Pneumothorax
ANSWER: A
A worker, who was involved in fire fighting inside the building that stored 2 kg of
mercury, has been delivered to a hospital with complaints of emotional expansiveness,
palpitations, excessive sweating, body tremor, heart pain. Within one day his
condition aggravated. Objectively: the skin is pale and moist. The patient is
depressed. Permanent red dermographism, erethism, unstable BP are observed. What drug
is necessary in this case?
A) Unithiol
B) Atropine sulfate
C) Calcium tetacine
D) Amyl nitrite
E) Dipyroxime
ANSWER: A
After having the flu, a 39-year-old male patient with a history of Addison’s disease
developed a condition manifested by
weakness,depression,nausea,vomiting,diarrhea,hypoglycemia.AP-75/50 mm Hg.Blood tes
results:low corticosterone and cortisol,13-oxycorticosteroids, 17-oxycorticosteroids
levels.What condition developed in the patient?
A) Acute adrenal insufficiency
B) Acute gastritis
C) Acute enterocolitis
D) Collapse
E) Diabetes mellitus
ANSWER: A
Against the background of angina a patient has developed pain in tubular bones.
Examination revealed generalized enlargement of lymph nodes, hepatolienal syndrome,
sternalgia. In blood: RBCs - 3, 6 • 1012/l, Hb-87 g/l, thrombocytes - 45 • 109/l,
WBCs -13•109/l,blasts-87%, stabneutrophiles-1%, segmented neutrophiles - 7%,
lymphocytes - 5%, ESR - 55 mm/h. What is the most likely diagnosis?
A) Acuteleukemia
B) Erythremia
C) Chronic lymphocytic leukemia
D) Chronic myeloid leukemia
E) Multiplemyeloma
ANSWER: A
An 8-year-old girl periodically has sudden short-term heart pain, sensation of chest
compression, epigastric pain, dizziness, vomiting. Objectively: the patient is pale,
respiratory rate - 40/min, jugular pulse is present. Heart rate - 185 bpm, of poor
volume. BP - 75/40 mm Hg. ECG taken during an attack shows ectopic P waves, QRS wave
is not deformed. At the end of an attack a compensatory pause is observed. The most
likely cause of the attack is:
A) Paroxysmal atrial tachycardia
B) Sinus tachycardia
C) Paroxysmal ventricular tachycardia
D) Complete AV-block
E) Atrial fibrillation
ANSWER: A
An electrogas welding operator working at a machine workshop performs welding and
cutting of metal, which is accompanied by intense UV-radiation. His welding station is
equipped with efficient mechanical venti-lation. What occupational disease is most
likely to develop in an electro-gas welding operator?
A) Photoelectric ophthalmia
B) Heatstroke
C) Vegetative-vascular dystonia
D) Chronic overheating
E) Pneumoconiosis
ANSWER: A
During examination a patient is unconsci-ous, his skin is dry and hot, face hyperemia
is present. The patient has Kussmaul’s respi-ration, there is also smell of acetone in
the air. Symptoms of peritoneum irritation are positive. Blood sugar is 33 millimole/l.
What emergency actions should be taken?
A) Intravenous infusion of short-acting insulin
B) Intravenous infusion of glucose along with insulin
C) Introduction of long-acting insulin
D) Intravenous infusion of neohaemodesum along with glutamic acid
E) Intravenous infusion of sodium chloride saline
ANSWER: A
Examination of an electric welder with 15 years of servicerecordrevealed dry rales in
the lower lung fields. Radiograph shows diffuse nodules sized 3-4 mm in the middle and
lower lung fields. What disease can be suspected?
A) Heavy-metalconiosis
B) Silicosis
C) Silicatosis
D) Carbon pneumoconiosis
E) Bronchitis
ANSWER: A
On the 4th day after recovering from acold a patient was hospitalized with complai-nts
of solitary spittings of mucoid sputum. On the 2nd day there was a single discharge of
about 250 ml of purulent blood-streaked sputum. Objectively: the patient’s condition
is moderately severe. Respiratory rate - 28-30/min., Ps- 96/min., BP- 110/70 mm Hg.
Respiration above the left lung is vesicular, weak above the right lung. There are
various moist crackles above the lower lobe and amphoric breath near the angle of
scapula. What is the most likely diagnosis?
A) Acute pulmonary abscess
B) Exudative pleuritis
C) Acute focal pneumonia
D) Pleural empyema
E) Pyopneumothorax
ANSWER: A
Residents of an industrial community situated near a plant suffer from increased
morbidity rate caused by nervous and endocrine system conditions and kidney diseases.
Blood test: decrease of sulfhydric groups content in blood. The pathologies developed
can be caused by environment being polluted by the following:
A) Mercury
B) Cadmium
C) Boron
D) Chromium
E) Lead
ANSWER: A
Residents of an industrial community situated near a plant suffer from increased
morbidity rate caused by nervous and endocrine system conditions and kidney diseases.
Blood test: decrease of sulfhydric groups content in blood. The pathologies developed
can be caused by environment being polluted by the following:
A) Mercury
B) Cadmium
C) Boron
D) Chromium
E) Lead
ANSWER: A
Survey radiograph of a 52-year-old worker of an agglomeration plant (28 years of
experience, the concentration of metal dust is 22-37mg/m3) shows mild lypronounced
interstitial fibrosis with diffused contrast well-defined small nodular shadows. The
patient has no complaints. Pulmonary function is not compromised. What is the
provisional diagnosis?
A) Siderosis
B) Silicosis
C) Anthraco-silicatosis
D) Silicatosis
E) Anthracosis
ANSWER: A
Survey radiograph of a 52-year-old worker of an agglomeration plant (28 years of
experi-ence, the concentration of metal dust is 22-37 mg/m3) shows mildly pronounced
interstitial fi-brosis with diffused contrast well-defined small nodular shadows. The
patient has no complaints. Pulmonary function is not compromised. What is the
provisional diagnosis?
A) Siderosis
B) Silicosis
C) Anthraco-silicatosis
D) Silicatosis
E) Anthracosis
ANSWER: A
Example(s) of "protective factor(s)" in peptic ulcer disease:
A) all of the above
B) prostaglandins
C) gastric mucus
D) bicarbonate
E) None of above
ANSWER: A
Examples of H2 (histamine receptor Type II) antagonists:
A) All of above
B) ranitidine (Zantac)
C) nizatidine (Axid)
D) famotidine (Pepcid)
E) cimetidine (Tagamet)
ANSWER: A
Exercise induced asthma is not precipitated by :
A) Swimming in hot water
B) High altitude climb and exercises
C) Cycling in cold weather
D) Swimming in cold water
E) All of the above
ANSWER: A
FEV1/FVC is reduced in case of:
A) Asthma
B) Pleural effusion
C) Lung fibrosis
D) All of the above
E) Nothing of the above
ANSWER: A
Findings in hemophilia A include:
A) Partial thromboplastin time increased
B) Increased clotting time
C) Increased prothrombin time
D) Bleeding time increased
E) Thrombocytopenia
ANSWER: A
Following are causes of unconjugated hyperbilirubinemia, except:
A) Rotor syndrome
B) Large hematoma
C) Hemolytic anemia
D) Megaloblastic anemia
E) Irondeficiency anemia
ANSWER: A
For the patient who has sustained tracheobronchial trauma, which assessment finding
alerts the physician to the possibility of tracheal lacerations?
A) Hypertympanic sound on affected side
B) Subcutaneous emphysema over the trachea
C) Hypotension and decreased capillary refill
D) Deviation of the trachea to the affected side
E) Non of above
ANSWER: B
Haemophilia A is characterised by the presence of following features, except
A) Bleeding into soft tissues
B) Reduced VIII levels
C) Prolonged bleeding time
D) Prolonged partial thromboplastin time
E) Bleeding into muscles &amp;amp;amp; joints
ANSWER: C
Helicobacter pylori is not associated with :
A) Gastric leiomyoma
B) Gastric cancer
C) Peptic ulcer of stomach
D) Peptic ulcer of duodenum
E) None of above
ANSWER: A
Histamine and gastric acid secretion:
A) all of the above
B) release enhanced by increased cholinergic activity
C) most important gastric acid secretion stimulant
D) released from enterochromaffin-like cells
E) None of above
ANSWER: A
How long is the usual course of drug treatment for a patient with active tuberculosis?
A) 7 to 10 days
B) 6 weeks
C) 6 months
D) 2 years
E) Non of above
ANSWER: C
Hypercalcemia may be treated by:
A) IV pamidronate
B) IV fluids + furosemide
C) Glucocorticoids
D) None above them
E) All of them
ANSWER: E
Hypergastrinemia with hypochlorhydria is seen in:
A) Pernicious anemia
B) All of above
C) Zollinger Ellison Syndrome
D) Glucagonoma
E) None of above
ANSWER: A
Hyperosmolar hyperglycemic non-ketonic coma is characterized by such level of glycemia:
A) 55 mmol/1
B) 20 mmol/1
C) 30 mmol/1
D) 5 mmol/1
E) 25 mmol/1
ANSWER: A
Hypoglycemic coma can be caused by:
A) Insulin deficiency
B) Dehydration
C) Extreme activity
D) Metabolic acidosis
E) Hypokalemia
ANSWER: C
Hypoglycemic effect of intermediate acting insulin lasts for:
A) 12-16 hours
B) 20 -30 hours
C) 0.5 – 1.0 hours
D) 4 – 6 hours
E) 6 – 8 hours
ANSWER: A
Hypoglycemic unawareness that occurs in diabetic patients when transferred from oral
hypoglycemic agents to insulin, is due to :
A) Autonomic neuropathy
B) Insulin resistance
C) Allergy
D) Somogi phenomenon
E) Down phenomenon
ANSWER: A
In a case of hypertrophic pyloric stenosis, the metabolic disturbance is
A) Metabolic alkalosis with paradoxical aciduria
B) Metabolic acidosis
C) Respiratory alkalosis
D) Metabolic alkalosis with alkaline urine
E) None of above
ANSWER: A
In a patient only Anti HBsAg is positive in serum, all other viral markers are
negative. It indicates :
A) Immunized person with hepatitis B vaccine
B) Chronic active hepatitis
C) Persistent carrier
D) Acute hepatitis
E) None of above
ANSWER: A
In bronchial asthma glucocorticoids
A) Reduce airway inflammation
B) Act as potent bronchodilators
C) Inhibit degranulation of mast cells
D) Block the action of humoral mediators
E) Nothing of the above
ANSWER: A
In endemic goiter such statement will be true:
A) Investigations invariably show some degree of hypothyroidism
B) There is a clear evidence of an increased incidence of carcinoma of the thyroid
C) Administration of thyroxine will often cause the goitre to shrink
D) Administration of iodine occasionally precipitates hyperthyroidism
E) All of them
ANSWER: E
In gastric outlet obstruction in a peptic ulcer patient, the site of obstruction is
most likely to be
A) Duodenum
B) Antrum
C) Pylorus
D) Pyloric canal
E) None of above
ANSWER: A
In patients with nonketotic hyperosmolar coma the level of glycemia is :
A) Slightly elevated
B) Mildly elevated
C) Moderately elevated
D) May be decreased
E) Grossly elevated
ANSWER: E
In pneumonia due to Mycoplasma all of the following are true except:
A) Bilateral infiltration on chest X-ray
B) Myalgia
C) Dry cough
D) Pleuritic chest pain
E) All of the above
ANSWER: A
In restrictive lung disease :
A) FEV1/FVC is high
B) FVC is high
C) FEV1 is high
D) All of the above
E) Nothing of the above
ANSWER: A
In severe bronchial asthma, it is present:
A) Hyperresonant chest with prolonged expiration
B) Infrequent rhonchi and absent breath sounds
C) Increased fremitus and crackles
D) Decreased fremitus and crepitation
E) All of the above
ANSWER: A
In the time of attack of bronchial asthma in lungs are heard
A) Dry loud wheezes
B) Moist loud wheezes
C) Moist soundless wheezes
D) Voice of pleural friction
E) Crepitation
ANSWER: A
In what specific way does the respiratory system function contribute to acid-base
balance?
A) Prevents excessive loss of hydrogen ions by evaporation
B) Increases the potassium ion content in bronchial secretions
C) Removes carbon dioxide generated as a result of metabolism
D) Maintains body water levels through mucous membrane function
E) Non of above
ANSWER: C
In what specific way does the respiratory system function contribute to acid-base
balance?
A) Prevents excessive loss of hydrogen ions by evaporation
B) Increases the potassium ion content in bronchial secretions
C) Removes carbon dioxide generated as a result of metabolism
D) Maintains body water levels through mucous membrane function
E) Non of above
ANSWER: C
In which case the dose of insulin in stable diabetics can be reduced:
A) Thyrotoxicosis
B) Steroid therapy
C) Pregnancy
D) Chronic renal failure
E) None above them
ANSWER: D
Increased amylase may be seen in all of the following except:
A) Appendicitis
B) Pancreatic pseudocyst
C) Perforated peptic ulcer
D) Ruptured ectopic pregnancy
E) None of above
ANSWER: A
Increased incidence of duodenal ulcer associated with:
A) All of above
B) alcoholic cirrhosis
C) renal transplantation
D) systemic mastocytosis
E) None of above
ANSWER: A
Increased incidence of duodenal ulcer associated with:
A) All of above
B) None of above
C) liver cirrhosis
D) chronic renal failure
E) hyperparathyroidism
ANSWER: A
Increased RBC level may be seen in:
A) Glomerulonephritis
B) Vitamin D excess
C) Cor pulmonale
D) Acute heart failure
E) Pneumonia
ANSWER: C
Insulin resistance is associated with all of the following status, EXCEPT:
A) Acanthosis nigricans
B) Lipodystrophy
C) Gout
D) Obesity
E) Metabolic syndrome
ANSWER: C
Intestinal hypomotility is seen in all the following except:
A) All of the above
B) Parkinsonism
C) Amyloidosis
D) Diabetes
E) None of above
ANSWER: A
Intrinsic factor for absorption of vitamin Bl2 is secreted by the :
A) Parietal cells
B) Peptic cells
C) Chief cells
D) Goblet cells
E) Endothelial cells
ANSWER: A
It is not Antacid
A) Histamine
B) magnesium hydroxide
C) calcium carbonate
D) sodium bicarbonate
E) All of above
ANSWER: A
Location(s) of histamine in gastric mucosa:
A) enterochromaffin-like cells (ECL) + mast cell cytoplasmic granules
B) mast cell cytoplasmic granules
C) enterochromaffin-like cells (ECL)
D) neither
E) are not localized in gastric mucosa
ANSWER: A
Macrophages containing large quantities of undigested and partial digested bacteria in
intestine are seen in
A) Whipple's disease
B) Amyloidosis
C) Immunoapproliferative small instetinal disease
D) Vibro cholerae infection
E) None of above
ANSWER: A
Massive bleeding per rectum in a 70 yr old patient is due to :
A) Diverticulosis
B) Carcinoma colon
C) Colitis
D) Polyps
E) None of above
ANSWER: A
Mechanism of action: cytoprotection, binds to the ulcer base, antibacterial effect:
A) bismuth compounds
B) tetracycline (Achromycin)
C) metronidazole (Flagyl)
D) ranitidine (Zantac)
E) omeprazole (Prilosec)
ANSWER: A
Mechanism of Idiopathic thrombocytopenic purpura is:
A) Vasculitis
B) Antibodies to vascular epithelium
C) Antibodies to platelets
D) Antibody to clotting factors
E) Fungal infectious
ANSWER: C
Mechanism(s) by which somatostatin reduces gastrin release:
A) inhibits parietal cells secretion+ inhibits histamine release by enterochromaffin-
like cells
B) inhibits histamine release by enterochromaffin-like cells
C) inhibits parietal cells secretion
D) stimulates parietal cells secretion
E) None of above
ANSWER: A
Melphalan is used in:
A) Wilm's tumor
B) Lymphoma
C) Multiple myeloma
D) Nephroblastoma
E) ALL
ANSWER: C
Metabolic changes associated with excessive vomiting includes the following:
A) Hypokalemia
B) Hyperchloremia
C) Metabolic acidosis
D) Decreases bicarbonates
E) None of above
ANSWER: A
Microcytic anemia is seen in:
A) Sprue
B) Pernicious anemia
C) Latun infestation
D) Iron deficiency anemia
E) Aplastic anemia
ANSWER: D
Monotherapy effectiveness in eradication of H. pylori-- bismuth compounds
A) 20%
B) 5%
C) 50%
D) 90%
E) 100%
ANSWER: A
More likely to affect the cytochrome P450 drug metabolizing system:
A) cimetidine (Tagamet)
B) All of above
C) nizatidine (Axid)
D) ranitidine (Zantac)
E) famotidine (Pepcid)
ANSWER: A
Most common CNS manifestation of Whipple's disease is :
A) Dementia
B) Supranuclear ophthalmoplegia
C) Seizure
D) Cerebellar ataxia
E) None of above
ANSWER: A
Most common complication of acute pancreatitis is:
A) Pseudocyst
B) Pancreatic abscess
C) Phlegmon
D) Pleural effusion
E) None of above
ANSWER: A
Most common extraintestinal manifestation of Crohn's disease of small bowel is.
A) Erythema Nodosum
B) Ankylosing Spondylitis
C) Iritis
D) Ureteral Obstruction
E) None of above
ANSWER: A
Most common sign of aspiration pneumonitis:
A) Tachypnoe
B) Bronchospasm
C) Cyanosis
D) Crepitations
E) Nothing of the above
ANSWER: A
Most sensitive and specific test for diagnosis of iron deficiency is:
A) Serum ferritin levels
B) Serum iron levels
C) Serum transferrin receptor population
D) Transferrin saturation
E) Hb, Ht
ANSWER: A
Most successful protocol for eradication of H. pylori/treatment of peptic ulcer
disease:
A) therapy using bismuth compounds, Clarythromycin (Flagyl), and amoxicillin (Amoxil
Polymox)
B) bismuth monotherapy
C) bismuth compounds and amoxicillin (Amoxil Polymox)
D) all of the above equally effective
E) None of above
ANSWER: A
Most widely used antacid in treating ulcer disease:
A) combination of aluminum hydroxide and magnesium hydroxide
B) calcium carbonate
C) magnesium hydroxide
D) aluminum hydroxide
E) sodium bicarbonate
ANSWER: A
Mycoplasma infection simulates:
A) Viral pneumonia
B) Pnemuococcal pneumonia
C) Pulmonary oedema
D) Pulmonary infarction
E) Nothing of the above
ANSWER: A
Non-parenteral hepatitis is :
A) Hepatitis E
B) Hep B
C) Hep C
D) Hep D
E) All of above
ANSWER: A
Omeprazole (Prilosec) and lansoprazole approved for clinical treatment of:
A) All of above
B) erosive gastritis
C) Zollinger-Ellison syndrome and other gastric acid hypersecretory states
D) None of above
E) duodenal ulcer
ANSWER: A
Only class of antiulcer drugs that can eradicate Helicobacter pylori and cure
associated gastritis:
A) colloidal bismuth
B) sucralfate (Carafate)
C) H2 blockers
D) Anticholinergic agents
E) None of above
ANSWER: A
Oral anticoagulant therapy is monitored with indexes
A) INR
B) PTT
C) TT
D) Clot lysis time
E) Bleeding time
ANSWER: A
Pancreatic pseudocyst most commonly occurs after
A) Pancreatitis
B) Trauma
C) Pancreatic surgery
D) Pancreatic malignancy
E) None of above
ANSWER: A
Paroxysmal nocturnal haemoglobinuria is due to:
A) Antibodies
B) Congenital membrane defect
C) Inherited autoimmune disorder
D) Complement mediated RBC lysis
E) Lysis of RBC in spleen
ANSWER: D
Pathogenic factor(s) in duodenal ulcer:
A) all of the above
B) genetic factor
C) cigarette smoking
D) alcoholic cirrhosis
E) COPD (chronic obstructive pulmonary disease)
ANSWER: A
Patient D. presents elevated serum T4 and increased radioactive iodine uptake. Put
diagnosis, please.
A) Non-toxic goiter
B) Hashimoto's thyroiditis
C) Subacute thyroiditis
D) Hypothyroidism
E) Graves' disease
ANSWER: E
Patient F., 29 yr. old, presents with subacute thyroiditis. Treatment will include:
A) Antibiotics
B) Vitamins
C) Antithyroid drugs
D) Iodine preparations
E) Corticosteroids
ANSWER: E
Patient on heparin therapy should be monitored with:
A) Clotting time
B) Bleeding time
C) Prothrombin time
D) Fibrinogen
E) PTT
ANSWER: A
Patient presents with recurrent duodenal ulcer of 2.5 cm size; procedure of choice
A) Truncal vagotomy and antrectomy
B) Truncal vagotomy and gastrojejunostomy
C) Highly selective vagotomy
D) Laparoscopic vagotomy and gastrojejunostomy
E) None of above
ANSWER: A
Peptic ulcer classification:
A) duodenal+ gastric
B) gastric
C) duodenal
D) neither
E) cardiac
ANSWER: A
Peptic ulcer should be differentiated with
A) All of above
B) Cholecystitis
C) Gastritis
D) Miocardial infarction
E) Pancreatitis
ANSWER: A
Patient K., 50 years old, entered the hospital with complaints of pain in the right
lower quadrant, nausea, general weakness. Laparoscopy was appointed. When carrying out
a diagnostic laparoscopy on suspicion of abscess of Douglas space, the second port
input is typically carried out
A. On the midline of the abdomen in hypogastrium
B. In the periumbilical area
C. On the midline of the abdomen in the epigastrium
D. At McBurney point
E. At Volkovych-Kocher point
ANSWER: A

Patient K., 59 years old, entered the hospital with complaints of pain in the right
lower quadrant, nausea, general weakness. Laparoscopy was appointed. To perform a
diagnostic laparoscopy on suspicion of acute appendicitis, setting the working port is
typically carried out
A. On The median line below the umbilicus in hypogastrium
B. On the midline of the abdomen in mesogastrium
C. At midline abdominal
D. Epigastrium midline of the abdomen 2-3 cm below the xiphoid processus
E. At Volkovych-Kocher point
ANSWER: A

Patient M., 30 years old, entered the hospital with complaints of pain in the right
upper quadrant, nausea, general weakness. Laparoscopy was appointed. When carrying
out a diagnostic laparoscopy on suspicion of acute cholecystitis, second port is
typically inserted
A. Epigastrium midline of the abdomen 2-3 cm below the xiphoid processus
B. At Kerr point
C. On the midline of the abdomen in hypogastrium
D. At McBurney point
E. At Volkovych-Kocher point
ANSWER: A

Patient J., 50 years old, entered the hospital with complaints of pain in the right
lower quadrant, nausea, general weakness. Laparoscopy was appointed. When carrying
out a diagnostic laparoscopy on suspicion of acute salpingal disorders, setting the
working port is typically carried out
A. On the midline of the abdomen in hypogastrium
B. In the periumbilical area
C. On the midline of the abdomen in the epigastrium
D. At McBurney point
E. At Volkovych-Kocher point
ANSWER: A

Patient K., 40 years old, entered the hospital with complaints of pain in the right
upper quadrant, nausea, general weakness. Laparoscopy was appointed. To perform a
diagnostic laparoscopy on suspected perforated ulcer, the introduction of instrumental
conducting port is done
A. On the midline of the abdomen in the epigastrium
B. In the periumbilical area
C. At Kerr point
D. At McBurney point
E. At Volkovych-Kocher point
ANSWER: A

Patient L., 27 years old, entered the hospital with complaints of pain in the right
lower quadrant, nausea, general weakness. Laparoscopy was appointed. In the
implementation of laparoscopy over suspected salpingitis, preparing the second port
site is carried out
A. On the midline of the abdomen below the navel in hypogastrium
B. In mesogastrium 1 cm above or below the navel the median line
C. On the midline of the abdomen in the epigastrium 10 cm above the navel
D. At McBurney point
E. At Kerr point
ANSWER: A

Patient R., 45 years old, entered the hospital with complaints of pain in the right
upper quadrant, nausea, general weakness. Laparoscopy was appointed. At
laparoscopy on suspicion of extrahepatic bile duct cancer, the second port input is
carried out
A. In the midline epigastrium
B. At Volkovych-Kocher point
C. On the midline of the abdomen in hypogastrium 10 cm below the navel
D. At McBurney point
E. At Kerr point
ANSWER: A

Patient M., 55 years old, entered the hospital with complaints of pain in the right
upper quadrant, nausea, general weakness. Laparoscopy was
appointed. For laparoscopy on suspicion of liver echinococcosis, setting the
second port is carried out
A. Epigastrium
B. At Volkovych-Kocher point
C. On the midline of the abdomen in hypogastrium 10 cm below the navel
D. At McBurney point
E. At Volkovych-Kocher point
ANSWER: A

Patient R., 78 years old, entered the hospital with complaints of pain in the right
upper quadrant, nausea, general weakness. Laparoscopy was appointed. For laparoscopy
on limited local peritonitis, instrumental port setting is done
A. Depending on the localization of process
B. On the midline of the abdomen in the epigastrium 10 cm above the navel
C. On the midline of the abdomen in hypogastrium 10 cm below the navel
D. At Volkovych-Kocher point
E. At Kerr point
ANSWER: A
After laparoscopic surgery for cholelithiasis, the whole set of equipment requires
laparoscopic sterilization. What method of sterilization is used for video cameras?
A. liquid
B. gas
C. autoclaving
D. dry-air
E. All of the above
ANSWER: A

Female, 28 years old, complains of increasing abdominal weakness over the last year.
OBJECTIVE: abdomen is enlarged symmetrically, with percussion of the abdomen in
lateral canals and above the pubis - obtuse sound, slight pain at all abdomen, no
muscle tension and signs of peritoneal irritation. According to ultrasound there is a
free fluid in the abdomen in a large quantity. Recommended to diagnostic laparoscopy.
Endovideosurgical complex consists of:
A. Videocomplex
B. Systems of aspiration and irrigation
C. Systems of insufflation
D. Coagulation Systems
E. All answers are correct
ANSWER: E

Patient I., 46 years old, entered the hospital with complaints of pain in the right
lower quadrant, nausea, general weakness. Laparoscopy was
appointed. In the implementation of laparoscopy on suspicion of pelvic
tumor, setting the working port is carried out
A. On the midline of the abdomen in hypogastrium 10 cm below the navel
B. In mesogastrium 1 cm above or below the navel the median line
C. On the midline of the abdomen in the epigastrium 10 cm above the navel
D. At Volkovych-Kocher point
E. At Kerr point
ANSWER: A

Patient '47 entered complaining of mild abdominal pain. Based on the review, the
results of clinical tests and examinations exhibited a preliminary diagnosis - liver
disease. The surgeon plans to perform aspiration biopsy of the liver. What tool use to
do a liver biopsy in this case?
A. Veresha Needle
B. Deschamps Needle
C. Biopsy Needle
D. Plain needle
E. biopsy forceps
ANSWER: C

Patient '67 arrived with complaints of frequent constipation, bloating. When


performing colonoscopy the endoscopist failed to pass the rectal-sigmoid angle of
colon. Was decided to perform the diagnostic laparoscopy. What tool should impose the
Colon-Colon anastomosis?
A. Linear stapler
B. Circular stapler
C. System of electrocoagulation Ligasure
D. Morselator
E. There is no right answer
ANSWER: B

Patient '33 entered complaining on severe abdominal pain, which originally appeared in
the epigastrium and in 2 hours migrated to the right iliac area. The patient exhibited
the diagnosis of acute appendicitis. What laparoscopic instrument the surgeon may
apply to ligate the appendix?
A. clipsator
B. Anatomical clamp
C. monopolar coagulation
D. Needle of Deschamps
E. Instrument for loading of needles
ANSWER: E

Patient '42 delivered in urgent order with complaints on pain epigastricm area during
the last 10 hours. Complaints appeared after the initiation of diet and alcohol abuse.
On examination, the abdomen is slightly swollen, involved in breathing, during
palpation he feeels the acute pain in the epigastrium and left upper quadrant.
Positive symptom of Mayo-Robson. According to ultrasound there is a liquid in the
omental bag. Which tool is recommended to disconnect the tissues?
A. L-shaped monopolar electrode
B. "universal" Clip
C. dissector
D. curved scissors
E. bipolar forceps
ANSWER: C

Patient '43, is prepared for the diagnostic laparoscopy to clarify the diagnosis. What
is considered to be the least traumatic tool for capturing intestine?
A. dissector
B. Babcock clamp type
C. Anatomical clip
D. 5-petaled Retractor
E. None of the above
ANSWER: B

32 year old patient during the recent week marks fever, pain in the chest. preliminary
diagnosis – abscess of the right lung. Differential diagnosis of chronic lung abscess
is made with:
A. atelectasis
B. bronchiectasis
C. pneumonia
D. lymphadenitis
E. emphisema
ANSWER: A
Patient B., 64 years old, entered the hospital with complaints of pain in the right
upper quadrant, nausea, general weakness. Laparoscopy was appointed. Chronic calculous
cholecystitis under review is characterized by the following features
A. gallbladder glued with surrounding organs, constricted, scars or sclerosis
B. the gallbladder is enlarged, streched, white and blue color
C. the gallbladder is enlarged, dark purple color, through extensive serous membrane
translucent mesh of blood vessels and capillaries
D. the gallbladder has whitish dense bulbs on the surface
E. none of the listed
ANSWER: A

Patient '62 delivered to the hospital complaining on persistent pain in the left upper
quadrant for 6 months. On examination, the abdomen is moderately tender in the left
upper quadrant. Symptoms of peritoneal irritation are absent. When a routine
splenectomy which of these methods are allowed for coagulation of splenic artery?
A. monopolar coagulation
B. Bipolar coagulation
C. System "Harmonica"
D. System Ligasure
E. monopolar coagulation using endoloop
ANSWER: D

Patient 53 years old, taken to hospital with complaints of abdominal pain, radiating
to the rectum, weakness, significant bleeding during menstruation, during the last 2
years. The abdomen is tense, slightly painful in the lower regions, negative symptoms
of peritoneal irritation, palpable round lesion above the vagina. What tool is used to
do fixing the uterus in its laparoscopic extirpation?
A. retainer of the "corkscrew"
B. Morselator
C. uterine manipulator
D. clamp type "crocodile"
E. The correct answer is missing
ANSWER: C

Patient B., 56years old, admitted with complaints of persistent pain in the right
upper quadrant during the day. The complaints came after a diet violation and abuse of
fatty foods. On examination the abdomen is sharply painful in the right upper quadrant.
Positive symptom of Ortner. The laparoscopic cholecystectomy is performed to patient.
To stop bleeding from the liver during the allocation of the gallbladder can use this
tool:
A. monopolar electrode type "scoop"
B. Surgical clamp
C. anatomical clip
D. Bipolar Forceps
E. clip of the "universal"
ANSWER: A
Patient B., 65 years old, entered the hospital with complaints of pain in the right
upper quadrant, nausea, general weakness. Laparoscopy was appointed. Chronic calculous
cholecystitis under review is characterized by the following features
A. gallbladder glued with surrounding organs, constricted, scars or sclerosis
B. the gallbladder is enlarged, streched, white and blue color
C. the gallbladder is enlarged, dark purple color, through extensive serous membrane
translucent mesh of blood vessels and capillaries
D. the gallbladder has whitish dense bulbs on the surface
E. none of the listed
ANSWER: A

Patient H., '29, who entered with complaints on icteric skin and sclera, and pain in
the right upper quadrant. After the survey was diagnosed calculous cholecystitis and
laparoscopic cholecystectomy performed using laparoscopic trocar and plastic tools.
What method of sterilization is used for this type of instrument?
A. liquid
B. gas
C. autoclaving
D. dry-air
E. None of the above
ANSWER: E

Patient K. '43, after the surgery the on chronic calculous cholecystitis was used
dissector, 2 anatomical clips, surgical clamp crocodile type and monopolar electrode.
At what temperature is for the presterilizing processing of the laparoscopic
instruments?
A. 50 ° C
B. 20 ° C
C. 30 ° C.
D. 90 ° C
E. 120 ° C
ANSWER: A

Patient K. '43, after the surgery the on chronic calculous cholecystitis was used
dissector, 2 anatomical clips, surgical clamp crocodile type and monopolar electrode.
It is necessary to conduct sterilization of used instruments. Which solution is
carried out for sterilization of instruments with dielectric coating?
A. alcohol
B. hydrogen peroxide
C. iodine
D. «Saydeks"
E. «Plivasept"
ANSWER: D

Patient C., 23 years old, entered the hospital with complaints of pain in the right
upper quadrant, nausea, general weakness. Laparoscopy was appointed. The gallbladder
is enlarged, dark purple color, through extensive serous membrane translucent mesh of
blood vessels and capillaries is in such pathologies as
A. Empyema of the gallbladder
B. Edema of the gallbladder
C. Chronic calculous cholecystitis
D. Cancer of the gallbladder
E. None of the
ANSWER: A

Patient K., 43 years old, was admitted with complaints of persistent pain in the right
upper quadrant during the day. The complaints came after a diet violation and abuse of
fatty foods. On examination the abdomen is sharply painful in the right upper quadrant.
Positive symptom of Ortner. The laparoscopic cholecystectomy is performed to patient.
Which tool is used to overlay clips on cystic artery?
A. System Ligasure
B. Linear stapler
C. Circular Stapler
D. clipator
E. All answers are correct
ANSWER: D

Patient M., '42 was performed arthroscopic surgery for meniscal injury of the knee
joint. After the surgery instruments require sterilization. What method of
sterilization is used for fiber optic cable?
A. liquid
B. gas
C. autoclaving
D. dry-air
E. All of the above
ANSWER: B

Patient P., '48, when performing laparoscopic surgery on chronic calculous


cholecystitis complications arose such as perforation gallbladder. There was a need to
wash the abdominal cavity. What is the solution used in the suction-irrigator?
A. 5% glucose
B. furatsillina
C. Physiological soloution
D. A solution with using antibiotic
E. chlorhexidine
ANSWER: C

Patient R., 58r., Appealed with complaints on violations of urination, hematuria.


After a physical and MRI study was diagnosed the prostate cancer with Mts to the
regional lymph nodes. There was a robot-assisted laparoscopy. What method of
sterilization of instruments used for laparoscopic robot da Vinci?
A. liquid
B. gas
C. autoclaving
D. dry-air
E. no correct answer
ANSWER: E
Patient, 23 years old, was taken to hospital urgently, complaining on pain in the
abdomen, more to the right, radiating to the rectum, dizziness. The above symptoms
appeared suddenly at night. Last menstrual period was 2 weeks ago. Objectively: skin
is pale, pulse - 92 for 1 min, body temperature - 36.6 0C, blood pressure - 100/60 mm
Hg. Art. Abdomen is slightly tense, slightly painful in the lower regions, the
symptoms of peritoneal irritation are weakly-positive. Analysis of blood: hemoglobin -
98 g / l. By which of these models of laparoscopes is the best to perform the
observing laparoscopy in this case?
A. 0 ° laparoscope 5 mm
B. laparoscope 30 ° 5 mm
C. 45 ° laparoscope 5 mm
D. 0 ° laparoscope 10mm
E. 45 ° laparoscope 10mm
ANSWER: B

patient, 43 years old, taken to hospital with complaints of abdominal pain, radiating
to the rectum, weakness, significant bleeding during menstruation, during the last 2
years. The abdomen is tense, slightly painful in the lower regions, negative symptoms
of peritoneal irritation, palpable round lesion above the vagina. Which tool is used
to remove the uterus when its laparoscopic extirpation?
A. Lock type "corkscrew"
B. Mortelator
C. uterine manipulator
D. Clamp type "crocodile"
E. The correct answer is missing
ANSWER: B

patient, 53 years old, taken to hospital with complaints of abdominal pain, radiating
to the rectum, weakness, significant bleeding during menstruation, during the last 2
years. The abdomen is tense, slightly painful in the lower regions, negative symptoms
of peritoneal irritation, palpable round lesion above the vagina. What tool is used
for fixation of the fibromatous node?
A. Morselyatom
B. retainer of the "corkscrew"
C. scissors
D. monopolar electrode endoloop
E. dissector
ANSWER: B

patients, 53 years old, taken to hospital with complaints on abdominal pain, radiating
to the rectum, weakness, significant bleeding during menstruation, during the last 2
years. Last menstrual period lasted 8 days. Objectively: skin is pale, pulse - 102 for
1 min, body temperature - 36.6 0C, blood pressure - 100/60 mm Hg. Art. The abdomen is
tense, slightly painful in the lower regions, negative symptoms of peritoneal
irritation, palpable the round lesion above the vagina. Analysis of blood: hemoglobin
- 88 g / l. What tool should perform removal of fibromatous site?
A. Moselator
B. retainer of the "corkscrew"
C. scissors
D. monopolar electrode endoloop
E. dissector
ANSWER: A

The patient, '22, that works as model, complained of abdominal pain in the right iliac
region during the last 2-years. Symptoms of peritoneal irritation are weakly positive.
After the observing laparoscopy revealed the inflammation of the appendix. What method
of laparoscopic surgery is the best in this case?
A. multiportal laparoscopy
B. Robot-assisted laparoscopy
C. one-port laparoscopy
D. laparoscopy through natural openings
E. Laparolifting
ANSWER: C

The patient, '22, that works as model, complained of abdominal pain in the right iliac
region during the last 2-years. Symptoms of peritoneal irritation are weakly positive.
Using clinical and laboratory studies were diagnosed catarrhal appendicitis. What
method of laparoscopic surgery is the best?
A. multiportal laparoscopy
B. Robot-assisted laparoscopy
C. one-port laparoscopy
D. laparoscopy through natural openings
E. Laparolifting
ANSWER: D

The surgeon performs a diagnostic laparoscopy to the patient with unclear diagnosis.
What kit of laparoscope should prepare for surgery?
A. 5 mm 0 ° and 30 ° 5 mm
B. 10 mm 0 ° and 45 ° 5 mm
C. 30 ° C. and 5 mm 5 mm 45 °
D. 5 mm 0 ° and 0 ° 10 mm
E. 10 mm 0 ° and 45 ° 10 mm
ANSWER: A

To patient F., '43, was performed the diagnostic laparoscopy with suspected ovarian
apoplexy, the gynecologist diagnosed the catarrhal appendicitis and performed
laparoscopic surgery. What is the name of the doctor and in what year was the first in
the world laparoscopic appendectomy?
A. Zemm in 1983
B. Tracing in 1929
C. Frederick in 1930
D. Hes in 1937
E. tapes in 1945
ANSWER: A

When performing diagnostic laparoscopy surgeon damaged the small intestine during
trocar introduction. By which the tool is applied the seam?
A. Instrument for applying continuous suture
B. Linear stapler
C. The standard laparoscopic needle holder
D. clipator
E. sticks for loading of nodes
ANSWER: A

Patient K., 40 years old, entered the hospital with complaints of pain in the right
lower quadrant, nausea, general weakness. Laparoscopy was appointed. To perform a
diagnostic laparoscopy on suspicion of acute appendicitis, port for laparoscope is
typically placed
A. In the periumbilical area
B. On the midline of the abdomen in the epigastrium
C. At midline of abdominal wall 10 cm upwards the navel
D. At midline in hypogastrium 10 cm below the navel
E. At Volkovych-Kocher point
ANSWER: A

Patient K., 50 years old, entered the hospital with complaints of pain in the right
upper quadrant, nausea, general weakness. For additional examination surgeon conducted
laparoscopy. In diagnostic laparoscopy surgeon pays attention to
A. signs of inflammation, tumor metastasis and free fluid in the abdominal
cavity, the shape and size of their color, vascular pattern
B. shape and size of organs, vascular pattern, signs of inflammation, tumor
metastasis and free fluid in the abdomen
C. shape and size of their color, signs of inflammation, tumor metastasis and free
fluid in the abdomen
D. shape and size of their color, vascular pattern, signs of inflammation, free fluid
in the abdomen
E. shape and size of their color, vascular pattern, signs of inflammation, tumors
ANSWER: A

Patient K., 50 years old, entered the hospital with complaints of pain in the right
upper quadrant, nausea, general weakness. Laparoscopy was appointed. Liver is granular
and nodular. This characteristic of laparoscopy picture indicates
A. Primary biliary cirrhosis
B. Hemochromatosis of liver
C. Obstructive cholangitis
D. Norm
E. Viral hepatitis
ANSWER: A

Patient K., 60 years old, came to the department complaining of pain in the right
upper quadrant, nausea, general weakness. Laparoscopy was appointed. Diagnostic review
is conducted
A. in the Trendelenburg position and horizontal position on the right and left side
B. in a horizontal position and the Trendelenburg position, on the right side
C. in a horizontal position and the Trendelenburg position, on the left side
D. horizontally on the right and left side
E. in the Trendelenburg position, on the right and left side
ANSWER: A

Patient K., 60 years old, came to the department complaining of pain in the right
upper quadrant, nausea, general weakness. Laparoscopy was appointed. The surface of
liver is smooth, dark brown in color with a greenish tinge. This is characteristic for
laparoscopy at
A. Pigmentary cirrhosis of liver
B. Primary biliary cirrhosis
C. Obstructive cholangitis
D. Norm
E. Viral hepatitis
ANSWER: A

Patient L., 55 years old, entered the hospital with complaints of pain in the both
upper quadrant, nausea, general weakness. Laparoscopy was appointed. To perform a
diagnostic laparoscopy on suspected perforated ulcer, input of optical port is
typically carried out
A. In the periumbilical area
B. On the midline of the abdomen in the epigastrium
C. At Kerr point
D. At McBurney point
E. At Volkovych-Kocher point
ANSWER: A

Patient M., 46 years old, entered the hospital with complaints of pain in the right
upper quadrant, nausea, general weakness. Laparoscopy was appointed. Edema of the
gallbladder under viewed characterized by the following features
A. the gallbladder is enlarged, streched, white and blue color
B. the gallbladder is enlarged, dark purple color, through extensive serous membrane
translucent mesh of blood vessels and capillaries
C. gallbladder glued with surrounding organs, constricted, scars or sclerosis
D. the gallbladder has whitish dense bulbs on the surface
E. none of the listed
ANSWER: A

Patient N., 32 years old, entered the hospital with complaints of pain in the right
upper quadrant, nausea, general weakness. Laparoscopy was appointed. Empyema of the
gallbladder under review is characterized by the following features
A. the gallbladder is enlarged, dark purple color, through extensive serous membrane
translucent mesh of blood vessels and capillaries
B. the gallbladder is enlarged, streched, white and blue color
C. gallbladder glued with surrounding organs, constricted, scars or sclerosis
D. the gallbladder has whitish dense bulbs on the surface
E. none of the listed
ANSWER: A

Patient N., 40 years old, was admitted to the hospital complaining of pain in the
right upper quadrant, nausea, general weakness. After additional examination surgeon
conducted laparoscopy. At diagnostic laparoscopy
A. You can change the position of the patient
B. You can not change the position of the patient
C. You can change the position of the patient in the sagittal plane
D. You can change the position of the patient in the frontal plane
E. preferably not to confuse the patient
ANSWER: A

Patient N., 40 years old, was admitted to the hospital complaining of pain in the
right upper quadrant, nausea, general weakness. After additional examination surgeon
conducted laparoscopy. The surface of liver is smooth, dark brown in color with a
greenish tinge. This characteristic of laparoscopy in
A. Hemochromatosis of liver
B. Norm
C. Obstructive cholangitis
D. Obstructive jaundice
E. Viral hepatitis
ANSWER: A

Patient N., 40 years old, was admitted to the hospital complaining of pain in the
right upper quadrant, nausea, general weakness. After additional examination surgeon
conducted laparoscopy. Greenish tint in the liver characterized
A. Obstructive cholangitis
B. Hemochromatosis of liver
C. Primary biliary cirrhosis
D. Obstructive jaundice
E. Norm
ANSWER: A

Patient S., aged 49, was admitted to hospital with complaints of pain in the right
upper quadrant, nausea, general weakness. After additional examination surgeon
conducted laparoscopy. Thin capillary net of blood vessels and soft whitish net of
hepatic lymph vessels is characteristic for
A. Viral hepatitis B
B. Hemochromatosis of liver
C. Primary biliary cirrhosis
D. Obstructive cholangitis
E. Obstructive jaundice
ANSWER: A

Patient S., aged 49, was admitted to hospital with complaints of pain in the right
upper quadrant, nausea, general weakness. After additional examination surgeon
conducted laparoscopy. Diagnostic laparoscopy with regulations is performed by means
of
A. move table to which the patient is fixed
B. manipulation of patients , not the table
C. manipulation by table to which the patient is fixed only sagittally
D. move table to which the patient is fixed only frontally
E. move table to which the patient is fixed only by height
ANSWER: A
In the surgical ward 82 yrs old patient enrolled. An examination showed suspected
acute cholecystitis abscess. What research method should be used primarily for further
diagnosis?
A. An ultrasound scan of the abdominal cavity
B. Infusion cholecystography
C. Laparoscopy
D. Retrograde panctreatocholangiography
E. Percutaneous cholecystography
ANSWER: A

In hospital delivered Male 42 years in critical condition: acrocyanosis, dyspnea,


subcutaneous emphysema in the neck and upper torso. Complains of severe chest pain,
epigastric pain. The body temperature of 38.9 (C, pulse 130 beats / min, blood
pressure = 80/50 mm Hg. Known that 6 hours ago after meals and alcohol appeared
vomiting and began to grow above mentioned phenomenon. Which is the preliminary
diagnosis?
A. Pinching paraesophageal hernia
B. Spontaneous pneumothorax
C. Pulmonary artery trombemboly
D. Spontaneous rupture of the esophagus
E. perforated ulcers of the stomach
ANSWER: D

In the department of thoracic surgery was hospitalized 43 year old patient with a
preliminary diagnosis of piotoraks. The fluid level reaches the bifurcation of the
trachea. Define standard Skeletopy of bifurcation of the trachea at the thoracic
vertebrae in adults:
A. IV vertebra
B. III vertebra
C. V vertebra
D. VI vertebra
E. VII vertebra
ANSWER: A

In the surgical ward taken patient of 38 years with stab wounds of the chest.
Patient's condition is serious. Consciousness confused, pale skin, cardiac deaf. Pulse
120 for 1 min., BP 70/40 mm Hg. Breathing - 32 for 1 min. On the front surface of the
chest in the third intercostal space on the left parasternal line a stab wound with
the length of 3cm. covered with blood clots is observed. Percussion – the border of
cardiac dullness is extended. What complication does occur?
A. Pericardial tamponade
B. Traumatic shock
C. Pulmonary artery
D. Acute cerebrovascular accident
E. Acute myocardial infarction
ANSWER: A
Patient B., 54 years old, was admitted to hospital with complaints of pain in the left
upper quadrant, nausea, general weakness. After additional examination surgeon
conducted laparoscopy. During conducting diagnostic laparoscopy surgeon diagnosed
hemorrhagic pancreonecrosis characterized by
A. presence of hemorrhagic fluid in the abdomen
B. presence of "stearic plaques"
C. dark red, brown or black pancreas
D. visual picture of acute cholecystitis
E. gland lobules are displayed clearly
ANSWER: A

In thoracic surgery there is patient of 37 years old with an abscess of the right lung.
For the second period, lung abscess characteristic such X ray picture:
A. eclipse of lung tissue without clear contours;
B. enlightenment of lung tissue without clear contours;
C. enlightenment of lung tissue with clear contours;
D. enlightenment of lung tissue specific cells;
E. lung tissue blackout with clear contours.
ANSWER: C

Patient B., 56 years old, was admitted to hospital with complaints of pain in the
right upper quadrant, nausea, general weakness. After additional examination
laparoscopy for treatment was appointed. For the question of urgency of surgery for
acute cholecystitis the most important is:
A. The presence of peritonitis
B. The intensity of pain
C. Duration of disease
D. The number of attacks in history
E. The presence of gallstones
ANSWER: A

Male 40 years with complaints of dizziness, pain in the left side of the chest,
shortness of breath. Got sick for 1 hour ago after hitting the left side on the edge
of the table. Pulse 122 beats/min. Respiratory rate 38 per minute. BP 90/60 mm Hg The
left half of the chest is behind during breathing, percussion over the lower part to 8
intercostal space (determined the dull sound, above – tympanic sound, auscultation -
breath sharply weakened. Diagnosis?
A. Pneumothorax
B. Hemothorax
C. Hemopericard
D. Hemopneumothorax
E. Pneumoempyema
ANSWER: D

Male 40 years, delivered by an ambulance after the accident, complaining of pronounced


shortness of breath, pain in the right half of the chest and sternum, cough with a
small amount of bright frothy blood. OBJECTIVE: condition of the patient is extremely
difficult, cyanosis of the face, severe subcutaneous emphysema of the chest wall, neck
and face. pulse - rhythmic, 110 for 1 min., BP - 90/60 mm Auscultation of the lungs -
breathing is rapidly weakened throughout, left side - satisfactory. What method of
examination is necessary to diagnose pathology for this patient?
A. Radiography of the chest and fibrobronchoscopy
B. Electrocardiography
C. Computed tomography of the brain
D. EFGDS
E. Ultrasound
ANSWER: A

Patient B., 56 years old, was admitted to hospital with complaints of pain in the
right upper quadrant, nausea, general weakness. After additional examination
laparoscopy for treatment was appointed. During the day after laparoscopic drainage of
choledochus bile in average is measured
A. Up to 700 -1000ml
B. Up to 5 ml
C. Up to 10 ml
D. Up to 15ml
E. Up to 2-3l
ANSWER: A

Patient '32 entered the surgical department within 6 hours from the time of disease
diagnosis: spontaneous pneumothorax. Treatment with passive drainage of the pleural
cavity for 3 days gave no effect. What is the most reliable studies will reveal the
cause of no effect of treatment?
A. Bronchography
B. Plain radiography of the chest
C. Thoracoscopy
D. Bronchoscopy
E. Ultrasound
ANSWER: C

Patient '39 hospitalized in thoracic surgery, complaining of chest pain, hard


breathing, fever. Diagnosed pulmonary gangrene. Gangrene of the lungs accompanied by
sputum:
A. Color of meat slops
B. light color
C. dark color
D. the blood
E. foam
ANSWER: A

Patient '40. Suddenly appeared pain in the left half of the chest, choking. State of
moderate severity, pulse - 110 per minute, blood pressure 90/60 mm Hg. Breathing in
the left side does not listen. When radiography of the chest - the collapse of the
left lung on one half. What treatment you want to assign to the patient?
A. rest, resolution therapy
B. Pleural puncture
C. Surgical treatment
D. Passive drainage of the pleural cavity
E. Active thoracostomy
ANSWER: D

Patient G., 37 years old, was admitted to hospital with complaints of pain in the
right upper quadrant, nausea, general weakness. After additional examination surgeon
conducted laparoscopy. Laparoscopy observation is carried out
A. sequentially from left to right and top to bottom
B. consistently better left to right and top to bottom
C. not consistently better from right to left and from top to bottom
D. not consistently better from right to left
E. consistently better from top to bottom
ANSWER: A

Patient '47 treated in thoracic surgery on an abscess of the right lung. Available
respiratory failure. In the I degree of dyspnea the respiratory failure is:
A. under load
B. in calm state
C. constant;
D. in horizontal body position
E. in upright body position
ANSWER: A

Patient '59 for a long time suffers from the lung abscess, available repeated
pulmonary hemorrhages. Repeated pulmonary hemorrhage in chronic abscesses is treated:
A. Blood Transfusion
B. transfusion of blood products
C. Surgically
D. input coagulants
E. entering antycoagulants
ANSWER: C

Patient 23 years old held X-ray with contrast. Diagnosed esophageal diverticulum.
Radiographic evidence of esophageal diverticulum is:
A. symptom of "wet bag"
B. symptom of bell
C. symptom of "shoe laces "
D. obtuse angle of His;
E. lack of gas bubble of the stomach
ANSWER: A

Patient 28 years old, accidentally drank acid solution. Got burns of the esophagus.
First aid for burns of the esophagus by concentrated acid (gastric lavage):
A. 0,25% solution of novocaine
B. 0,9% solution of sodium chlorid
C. 5% glucose solution
D. 2% sodium bicarbonate
E. 1: 1000 solution of potassium manganese
ANSWER: D
Patient 37 years, the clinic delivered in 50 minutes. after receiving stab wounds, the
wound is located in the heart area. The patient is pale, sharply inhibited, BP 60/20
mm Hg. century. expanded the boundaries of the heart, auscultation - warm tone deaf.
Your actions?
A. Conducting intensive antishock therapy
B. Emergency thoracotomy with excision and suture haemopericardium heart.
C. Conducting intensive antishock therapy followed by surgery
D. Conduct antishock therapy with simultaneous execution thoracoscopy
E. X-ray of the chest
ANSWER: B

Patient 40 years is treated within two weeks due to an acute abscess of the upper
right lung. The treatment: antibiotics, sulfanilamide preparations, vitamin, calcium
chloride, infusion therapy. However, the patient continued the high fever, cough it
bother with the periodic discharge of large number of purulent sputum. What treatment
can effectively complement the rehabilitation of an abscess?
A. Microtracheostomy
B. Tracheostomy
C. Therapeutic bronchoscopy
D. Postural drainage
E. Drainage of pleural cavity
ANSWER: A

Patient 48 years old was hospitalized in thoracic surgery with a diagnosis of Zenker
diverticulum. Being prepared for surgery. Access for Zenker diverticulum?
A. cervical access;
B. lateral thoracotomy in the third intercostal space
C. lateral thoracotomy in the fourth intercostal space
D. lateral thoracotomy in the seventh intercostal space
E. Upper midline laparotomy
ANSWER: A

Patient 58 years old after the X-ray contrast the epiphrenal diverticulum was
diagnosed. The proposed surgery. Access to the epiphrenal diverticulum?
A. right-lateral thoracotomy in IV intercostal space;
B. left-sided lateral thoracotomy in IV intercostal space; l
C. eft-sided thoracotomy in the lateral VII intercostal space;
D. right-lateral thoracotomy in VII intercostal space;
E. Upper midline laparotomy.
ANSWER: A

Patient 62 years old, who chronically abused ba alcohol delivered to the thoracic
department with a diagnosis of esophageal burns. The need for gastrostomy in case of
burns of the esophagus occurs when:
A. esophageal perforation
B. First degree;
C. Second degree;
D. the third degree;
E. burns of the mouth.
ANSWER: A

Patient complains of discomfort behind the breastbone. Available symptom of ‘wet bag’.
Symptom "wet bag" is characteristic for:
A. diverticulum of the esophagus
B. sliding hiatal hernia
C. paraesophageal hiatal hernia
D. iatrogenic perforation of the esophagus
E. relaxation of the diaphragm
ANSWER: A

Patient K., 30 years old, entered the hospital with complaints of pain in the right
upper quadrant, nausea, general weakness. Additional examination found a previous
diagnosis of bile-stone disease. Laparoscopy was prescribed. Gallbladder has smooth
surface, white with blue color, enlarged and stretched. In this case surgeon has to
conduct
A. Laparoscopic cholecystectomy
B. Complete diagnostic laparoscopy
C. open cholecystectomy
D. Cholecystostomy
E. None of the listed
ANSWER: A

Patient K. '34 was hospitalized in serious condition with a wound to the left of the
chest. Patient's condition is severe, skin pale, his face is bluish-purple, swollen
neck veins. Percussion notes expanding the boundaries of the heart, auscultation -
voiceless heart tones. Your diagnosis?
A. Injury to the heart with the development of tamponade
B. Injuries of the left lung with the development of hemopleura
C. Injury of the left lung with the development of pneumothorax
D. Injury of the left lung with the development of stump-hemopleura
E. Injury of chest, bleeding from intercostal arteries
ANSWER: A

Patient of 64 years of age was hospitalized in the department of thoracic surgery


patient with a second period of lung abscess. The second period of lung abscess is
characterized by:
A. increase toxicity
B. deterioration of the general condition of the patien
C. fever
D. allocating a significant amount of pus
E. hectic body temperature
ANSWER: A

Patient S., 27 years old, went to hospital in serious condition, 50 minutes after
receiving penetrating wounds of the chest to the left. OBJECTIVE: consciousness -
stupor, pale skin, acrocyanosis. Pulse 120 beats per minute, poor volume and tension.
BP - mm 80/40 Cardiac tone is deaf, dramatically expanded boundaries. In the III
intercostal space on the left parasternal line is the stab wound. On plain film chest
expansion is defined shadows and smoothing the waist of the heart, left hemothorax to
V edges. What is the most possible reason of severity of the patient?
A. cardiac tamponade
B. Acute heart failure
C. cardiac arrhythmias
D. loss
E. hemothorax and acute respiratory failure.
ANSWER: A

Patient, taken after the accident, complaining of pain in the left chest, cough,
hemoptysis, dyspnea. Subcutaneous emphysema, cyanosis of the skin, heart rate 98 per 1
min. Auscultation - breath weakened in the left. Rib hull damage is not detected,
radiographically observed air strip along the trachea, left-sided pneumothorax. Your
diagnosis?
A. Rupture of esophagus
B. Lung abscess with perforation
C. Rupture of the left main bronchus
D. Rupture of the left dome of the diaphragm
E. Pneumoempyema
ANSWER: C

The patient complains on heartburn, feeling of bitterness in the mouth. Diagnosed


reflux esophagitis. Reflux esophagitis is a manifestation of:
A. diverticulum of the esophagus
B. paraesophageal hiatal hernia
C. iatrogenic perforation of the esophagus
D. Relaxation of the diaphragm
E. sliding hiatal hernia
ANSWER: E

The patient delivered unconscious. Skin and mucous membranes are pale, cyanotic,
breathing is weakened, in the lower regions does not listened, at the level of 6 rib
on the anterior axillary line there is a wound hole with moderate bleeding and passage
of air to insufficiently. Radiological findings: a bullet in the pleural cavity. What
medical tactic?
A. Emergency thoracotomy
B. Thoracoscopy with removal of bullet
C. Transfer valve into open pneumothorax
D. Drainage of pleural cavity
E. Tosca bandage on a wound
ANSWER: A

The patient is being treated in the thoracic surgery on empyema. existing respiratory
failure. The number of respiratory movements during the first stage of respiratory
failure (choose the wrong answer):
A. 14-15
B. 16-18
C. 19-20
D. 23-24
E. 21-22
ANSWER: D

The patient of '54 complains on poor passage of food through the esophagus. The last
two years saw the neck protrusion in the left after eating, vomiting with food. He
began to lose weight. At night, during sleep there is cough. When X-ray contrast study
of the esophagus at the level of the collarbone appears a depot of barium, size and
shape of an egg. What is the most likely diagnosis?
A. Esophagic-tracheal fistula
B. Cancer of the esophagus
C. diverticulum of the esophagus
D. Stenosis
E. Esophagic spasm
ANSWER: C

The patient received a wound in the chest spot in the projection of the heart.
Patient's condition is serious, complaining of shortness of breath, a tendency to
hypotension, muffled heart tones. The boundaries of the heart with percussion extended
to the left. In Plain radiography of the chest cavity – expanding of hearts shadow to
the left, reducing the oscillation amplitude contour of the heart. According to
electrocardiography - reduced voltage complex QRS. Set the diagnosis?
A. Pericardial tamponade
B. Pneumonia
C. Myocardial infarction
D. Pneumothorax
E. Hemothorax
ANSWER: A

To the toracal surgery was hospitalised a patient with suspected pulmonary gangrene.
When percussion in case of gangrene of lung is determined:
A. clear lung sounds
B. The box sound
C. wide area of dull sound
D. tympanic sound
E. The narrow plot of dull sound
ANSWER: C

Patient K., 50 years old, entered the hospital with complaints of pain in the left
upper quadrant, nausea, general weakness. After additional examination for treatment
of pancreatitis laparoscopy was appointed. During conducting laparoscopic drainage of
the abdominal cavity of the patient with acute pancreatitis, port for laparoscope
typically is placed
A. In the periumbilical area
B. On the midline of the abdomen in the epigastrium
C. On the midline of the abdomen in hypogastrium
D. In mesogastrium on the affected side
E. At Volkovych- Kocher point
ANSWER: A
Patient K., 50 years old, entered the hospital with complaints of pain in the right
upper quadrant, nausea, general weakness. After additional examination for traetment
for acute pancreatitis laparoscopy was appointed. For laparoscopy for the treatment
of acute pancreatitis, the second port input is carried out
A. For midline epigastrium
B. In mesogastrium 1 cm above or below the navel the median line
C. On the midline of the abdomen in hypogastrium 10 cm below the navel
D. At McBurney point
E. At Mayo point
ANSWER: A

Patient K., 50 years old, entered the hospital with complaints of pain in the right
upper quadrant, nausea, general weakness. After additional examination laparoscopy for
treatment was appointed. Which among these is the final stage in laparoscopic
cholecystectomy?
A. subserose extraction of gallbladder
B. ligation of cystic duct and artery
C. removal of the gall bladder
D. Removal of calculus
E. suturing the gallbladder bed
ANSWER: A

Patient K., 50 years old, entered the hospital with complaints of pain in the left and
right upper quadrant, nausea, general weakness. The laparoscopy was appointed. During
conducting diagnostic laparoscopy mixed biliary pancreatic necrosis was diagnosed
characterized by
A. all specified
B. presence of "stearic plaques"
C. presence of hemorrhagic fluid
D. dark red, brown or black pancreas
E. visual picture of acute cholecystitis
ANSWER: A

Patient K., 50 years old, entered the hospital with complaints of pain in the left
upper quadrant, nausea, general weakness. The laparoscopy was appointed. Hemorrhagic
necrotizing pancreatitis is characterized by
A. presence of hemorrhagic fluid in the free abdominal cavity
B. presence of scattered on the surface of large and small omentum or elsewhere
"stearic plaques"
C. dark red, brown or black pancreatic cancer, the presence of hemorrhagic fluid in
the free abdominal cavity, the presence of "stearic plaques"
D. visual picture of acute cholecystitis plus characteristic signs of inflammation of
the pancreas
E. There is no specific sign
ANSWER: A

Patient K., 50 years old, entered the hospital with complaints of pain in the right
upper quadrant, nausea, general weakness. Laparoscopy was appointed. Streched
enlarged gallbladder characterizes
A. acute simple cholecystitis
B. normal gallbladder
C. acute gangrenous cholecystitis
D. acute gangrenous perforated cholecystitis
E. perforated cholecystitis
ANSWER: A

Patient K., 54 years old, came to the department complaining of pain in the right
upper quadrant, nausea, general weakness. After additional examination laparoscopy for
treatment was appointed. During conducting therapeutic laparoscopy for acute
obstructive cholecystitis, operational port is carried out
A. Epigastrium 2-3 cm below the xiphoid processus
B. At Mayo point
C. On the midline of the abdomen in hypogastrium
D. At McBurney point
E. At Volkovych- Kocher point
ANSWER: A

Patient K., 54 years old, entered the department complaining of pain in the right
upper quadrant, nausea, general weakness. After additional examination laparoscopy for
treatment was appointed. Retrograde cholecystectomy is performed in one of the
following cases:
A. In the presence of inflammatory infiltrate in the cervical region of the
gallbladder
B. In elderly patients
C. In the presence of phenomena of cholangitis
D. When contracted gallbladder
E. When stone in cervix of the gallbladder
ANSWER: A

Patient K., 54 years old, entered the department complaining of pain in the left upper
quadrant, nausea, general weakness. Was laparoscopy was appointed. During conducting
diagnostic laparoscopy focal fatty pancreatic necrosis was diagnosed characterized by
A. presence of "stearic plaques"
B. presence of hemorrhagic fluid
C. dark red, brown or black pancreas
D. visual picture of acute cholecystitis
E. gland lobules are displayed clearly
ANSWER: A

Patient K., 54 years old, entered the department complaining of pain in the left upper
quadrant, nausea, general weakness. The laparoscopy was appointed. During conducting
diagnostic laparoscopy scattering on the surface of large and small omentum or
elsewhere of "stearic plaques" indicates
A. Fatty pancreatic necrosis
B. Hemorrhagic necrotizing pancreatitis
C. Mixed pancreatic necrosis
D. Biliary pancreatitis
E. Intact pancreas
ANSWER: A

Patient K., 70 years old, entered the hospital with complaints of pain in the right
upper quadrant, nausea, general weakness. It has a pronounced respiratory
failure. Additional examination found a previous diagnosis of bile-stone
disease. Laparoscopy was prescribed. Gallbladder has smooth surface, white with blue
color, enlarged and stretched. One of the complications of acute cholecystitis was
diagnosed. It is
A. Edema of the gallbladder
B. Empyema of the gallbladder
C. Chronic calculous cholecystitis
D. Cancer of the gallbladder
E. None of the listed
ANSWER: A

Patient K., 74 years old, entered the department with complaints of pain in the left
upper quadrant, nausea, general weakness. The laparoscopy was appointed. Acute
biliary pancreatitis in a patient is characterized by
A. visual picture of acute cholecystitis accompanying characteristic signs of
inflammation of the pancreas
B. presence of scattered on the surface of large and small omentum or elsewhere
"stearic plaques"
C. presence of hemorrhagic fluid in the free abdominal cavity
D. dark red, brown or black pancreatic cancer, the presence of hemorrhagic fluid in
the free abdominal cavity, the presence of "stearic plaques"
E. There is no specific sign
ANSWER: A

Patient K., 74 years old, entered the department with complaints of pain in the right
upper quadrant, nausea, general weakness. Llaparoscopy was appointed. In acute
gangrenous perforated cholecystitis, characteristic appearance of the gallbladder is
following
A. the presence of black spots with bile contents in the free abdominal cavity
B. the presence of black spots with fibrinous layers
C. hyperemia on the background, fibrinous layerings can be observed
D. enlarged stretched gallbladder
E. omentum tightly wrapped around gallbladder, as a result-poor visualization
ANSWER: A

Patient N., 40 years old, was admitted to hospital complaining of marked pain in the
right upper quadrant, nausea, general weakness. After additional examination surgeon
conducted laparoscopy. The characteristic appearance of the gallbladder at perforated
cholecystitis is following
A. omentum tightly wrapped around gallbladder, as a result-poor visualization
B. the presence of black spots with bile contents in the free abdominal cavity
C. the presence of black spots with fibrinous layers
D. hyperemia on the background, fibrinous layerings can be observed
E. enlarged stretched gallbladder
ANSWER: A
Patient N., 40 years old, was admitted to the hospital complaining of pain in the
right upper quadrant, nausea, general weakness. After additional examination
laparoscopy for treatment was appointed. In conducting laparoscopic cholecystectomy,
port for laparoscope is typically placed
A. In the periumbilical area
B. On the midline of the abdomen in the epigastrium
C. On the midline of the abdomen in hypogastrium
D. In mesogastrium on the affected side
E. At Volkovych- Kocher point
ANSWER: A

Patient N., 40 years old, was admitted to the hospital complaining of pain in the
right upper quadrant, nausea, general weakness. After additional examination
laparoscopy for treatment was appointed. For diagnosis of uncomplicated cholelithiasis
surgeon should prefer:
A. Ultrasonography
B. Endoscopic retrograde cholangiopancreatography
C. Laparoscopy
D. Percutaneous cholangiography
E. Fractional duodenal intubation
ANSWER: A

Patient N., 40 years old, was admitted to the hospital complaining of pain in the
right upper quadrant, nausea, general weakness. After additional examination
laparoscopy for treatment was appointed. Stages of laparoscopic cholecystostomy are:
A. puncture of the gallbladder, cholecystostomy, fixing drainage tube into the gall
bladder and the parietal peritoneum
B. blending metal clip on cystic duct, recutting artery, suturing the gallbladder bed
C. ligation of cystic duct, gallbladder removal, drainage of subhepatic space
D. allocation subserously , gallbladder puncture of the gallbladder, cystic duct
transection
E. puncture of the gallbladder, holetsystostomy
ANSWER: A

Patient N., 40 years old, was admitted to the hospital complaining of pain in the left
and right upper quadrant, nausea, general weakness. After additional examination
surgeon conducted laparoscopy. During conducting diagnostic laparoscopy with a picture
of acute cholecystitis there are signs of inflammation of the pancreas that occurs in
this state as
A. Biliary acute pancreatitis
B. Focal necrotizing pancreatitis
C. Focal hemorrhagic pancreatitis
D. Mixed pancreatic necrosis
E. Intact pancreas
ANSWER: A
Patient N., 40 years old, was admitted to the hospital complaining of pain in the
right upper quadrant, nausea, general weakness. After additional examination doctor
conducted laparoscopy. Acute catharral cholecystitis appearance is characterized by
A. enlarged stretched gallbladder
B. hyperemia on the background, fibrinous layering can be observed
C. the presence of black spots with fibrinous layers
D. the presence of black spots with bile contents in the free abdominal cavity
E. omentum tightly wrapped around gallbladder, as a result-poor visualization
ANSWER: A

Patient S., 57 years old, entered the department with complaints of pain in the left
upper quadrant, nausea, general weakness. After additional examination surgeon
conducted laparoscopy. Mixed necrotizing pancreatitis in a patient is characterized by
A. dark red, brown or black pancreatic cancer, the presence of hemorrhagic fluid in
the free abdominal cavity, the presence of "stearic plaques"
B. presence of scattered on the surface of large and small omentum or elsewhere
"stearic plaques"
C. presence of hemorrhagic fluid in the free abdominal cavity
D. visual picture of acute cholecystitis andcharacteristic signs of inflammation of
the pancreas
E. There is no specific sign
ANSWER: A

Patient S., 57 years old, entered the department with complaints of pain in the right
upper quadrant, nausea, general weakness. After additional examination surgeon
conducted laparoscopy. In acute gangrenous cholecystitis gallbladder looks as
A. the presence of black spots with fibrinous layers
B. hyperemia on the background, fibrinous layerings can be observed
C. enlarged stretched gallbladder
D. the presence of black spots with bile contents in the free abdominal cavity
E. omentum tightly wrapped around gallbladder, as a result-poor visualization
ANSWER: A

The patient K., 40 yrs.old is undergoing GERD surgery. At Toupet surgery


A. Place the camera above the umbilicus, one third of the distance to the xiphoid
process.
B. Place the camera above the umbilicus, one half of the distance to the xiphoid
process.
C. Place the camera under the umbilicus, one third of the distance to the pubis.
D. Place the camera above the xiphoid process.
E. Place the camera aside the umbilicus.
ANSWER: A

The patient K., 40 yrs.old is undergoing GERD surgery. At Toupet surgery


A. Place two lateral retracting ports in the right and left anterior
axillary lines, respectively.
B. Place two lateral retracting ports in the right and left posterior
axillary lines, respectively.
C. Place two lateral retracting ports in the right and left middle axillary
lines, respectively.
D. Place two lateral retracting ports in the right and left clavicular
lines, respectively.
E. Place two lateral retracting ports in the right and left pararectal
lines, respectively.
ANSWER: A

The patient K., 40 yrs.old is undergoing GERD surgery. At Toupet surgery one uses 30%
to 45% of reverse Trendelenburg to displace the transverse colon and small bowel
inferiorly,
A. keeping them from obstructing the view of the video camera
B. keeping them from obstructing the view of the hiatus
C. keeping them from obstructing the view of liver
D. keeping them from obstructing the view of the stomach
E. it is traditional
ANSWER: A

The patient K., 40 yrs.old is undergoing GERD surgery. Laparoscopic fundoplication is


indicated for
A. the treatment of objectively documented, relatively severe gastroesophageal reflux
disease
B. the treatment of relatively severe gastroesophageal reflux disease
C. the treatment of appendicitis
D. the treatment of objectively severe gastroesophageal reflux disease
E. the treatment of reflux
ANSWER: A

The patient K., 40 yrs.old is undergoing GERD surgery. Patient position at GERD
surgery is…
A. patient is placed supine, the knees only slightly ?exed
B. patient is placed supine with the head elevated 75 degrees
C. patient is placed with the head elevated 15 degrees in the modi?ed lithotomy
position
D. patient is placed with the head elevated 45 degrees, the knees only slightly
extended
E. patient is placed
ANSWER: A

The patient K., 40 yrs.old is undergoing GERD surgery. Patient position at Nissen
procedure is…
A. patient is placed supine, the knees only slightly ?exed
B. patient is placed supine with the head elevated 75 degrees
C. patient is placed with the head elevated 15 degrees in the modi?ed lithotomy
position
D. patient is placed with the head elevated 45 degrees, the knees only slightly
extended
E. patient is placed
ANSWER: A
The patient K., 40 yrs.old is undergoing GERD surgery. Patients with gastroesophageal
re?ux may be considered candidates for the procedure of laparoscopic fundoplication at:
A. Esophageal complications such as erosive esophagitis, stricture, and/or Barrett’s
esophagus
B. Non-Respiratory complications
C. Dependence upon diet for relief of symptoms
D. Laryngeal symptoms with a good response to diet
E. None of the listed
ANSWER: A

The patient K., 40 yrs.old is undergoing GERD surgery. Patients with gastroesophageal
re?ux may be considered candidates for the procedure of laparoscopic fundoplication at:
A. Esophageal stricture
B. Non-Respiratory complications
C. Dependence upon diet for relief of symptoms
D. Laryngeal symptoms with a good response to diet
E. None of the listed
ANSWER: A

The patient K., 40 yrs.old is undergoing GERD surgery. Patients with gastroesophageal
re?ux may be considered candidates for the procedure of laparoscopic fundoplication at:
A. Respiratory complication such as bronchiectasis
B. Non-erosive esophagitis
C. Dependence upon diet for relief of symptoms
D. Laryngeal symptoms with a good response to diet
E. None of the listed
ANSWER: A

The patient K., 40 yrs.old is undergoing GERD surgery. Patients with gastroesophageal
re?ux may be considered candidates for the procedure of laparoscopic fundoplication at:
A. Dependence upon proton pump inhibitors (PPIs) for relief of symptoms, particularly
if dose escalation is required
B. Non-erosive esophagitis
C. Dependence upon diet for relief of symptoms
D. Laryngeal symptoms with a good response to diet
E. None of the listed
ANSWER: A

The patient K., 40 yrs.old is undergoing GERD surgery. The surgeon stands at Nissen
fundoplication
A. between the patient’s legs
B. to the right of the patient
C. to the left of the patient
D. either side of the patient
E. at the patient’s head
ANSWER: A
The patient K., 40 yrs.old is undergoing GERD surgery. The surgeon stands at Toupet
fundoplication
A. between the patient’s legs
B. to the right of the patient
C. to the left of the patient
D. either side of the patient
E. at the patient’s head
ANSWER: A

The patient K., 40 yrs.old is undergoing GERD surgery. The surgeon stands at GERD
surgery…
A. The surgeon stands between the legs and works with both hands. This allows the
rightand left-handed instruments to approach the hiatus from the respective upper
abdominal quadrants.
B. The surgeon stands between the legs and works with right hand.
C. The surgeon stands between the legs and works with left hand. This allows the
right and left-handed instruments to approach the hiatus from the respective upper
abdominal quadrants.
D. The surgeon stands aside the patient and works with both hands. This allows the
rightand left-handed instruments to approach the hiatus from the respective upper
abdominal quadrants.
E. The surgeon stands between the legs and works with left hand. This allows the
left-handed instruments to approach the hiatus.
ANSWER: A

The patient K., 40 yrs.old is undergoing GERD surgery. The workup for laparoscopic
treatment of GERD consists of one of the following stages.
A. 24-hour pH monitoring
B. Manometric ultrasound
C. Assessment of esophageal width
D. Selection of a partial fundoplication technique
E. None of the listed
ANSWER: A

The patient K., 40 yrs.old is undergoing GERD surgery. The workup for laparoscopic
treatment of GERD consists of one of the following stages.
A. Selection of a partial versus complete fundoplication
B. Manometric ultrasound
C. Assessment of esophageal width
D. Selection of a partial fundoplication technique
E. None of the listed
ANSWER: A
Who had first time in medical practice performed cholecystectomy
A. Courvoisier L.
B. Lanhenbuh K.
C. The monastic ND
D. Fedorov SP
E. Coeur G.
ANSWER: B

With the patient lying on his back at the lowered terminal end of the table well to
examine:
A. Gallbladder
B. Stomach
C. Recto-sigmoid colon angle
D. Uterus
E. Spleen
ANSWER: B

With the patient lying on his back at the lowered terminal end of the table well to
examine:
A. Gallbladder
B. Abdominal esophagus
C. the right lobe of the liver
D. Uterus
E. Spleen
ANSWER: B

With the patient lying on his back in a horizontal position table well to examine:
A. Gallbladder
B. Pancreas
C. small intestine
D. sigmoid
E. Spleen
ANSWER: C

With the patient lying on his back with lowered head end of the table well to examine:
A. Gallbladder
B. Pancreas
C. Recto-sigmoid colon angle
D. Uterus
E. Spleen
ANSWER: D

With the patient lying on his back with lowered head end of the table well to examine:
A. Gallbladder
B. Pancreas
C. Recto-sigmoid colon angle
D. bladder
E. Spleen
ANSWER: D
With the patient lying on his back with lowered head end of the table well to examine:
A. Gallbladder
B. Pancreas
C. Recto-sigmoid colon angle
D. Fallopian tubes
E. Spleen
ANSWER: D

With the patient lying on his back with lowered head end of the table well to examine:
A. Gallbladder
B. Pancreas
C. Recto-sigmoid colon angle
D. Ovaries
E. Spleen
ANSWER: D

With the patient lying on his right side with horizontal table well to examine:
A. Spleen
B. Pancreas
C. the right lobe of the liver
D. Uterus
E. Right side channel
ANSWER: A

With the patient lying on his right side with lowered head end table well to examine:
A. Gallbladder
B. Pancreas
C. Sigmoid
D. The right ovary
E. Spleen
ANSWER: C

With the patient lying on his right side with lowered head end table well to examine:
A. Gallbladder
B. Pancreas
C. Recto-sigmoid colon angle
D. The right ovary
E. Spleen
ANSWER: C

With the patient lying on the left side at the lowered terminal end of the table well
to examine:
A. Spleen
B. Pancreas
C. Gallbladder
D. Uterus
E. Ovaries
ANSWER: C
With the patient lying on the left side at the lowered terminal end of the table well
to examine:
A. Spleen
B. Pancreas
C. Hepatic flexure of colon
D. Uterus
E. Ovaries
ANSWER: C

With the patient lying on the left side in the horizontal position of the table is
well to examine:
A. Spleen
B. Pancreas
C. the right lobe of the liver
D. Uterus
E. Right side channel
ANSWER: E

With the patient lying on the right side at the lowered terminal end of the table well
to examine:
A. greater curvature of the stomach
B. Pancreas
C. Kidneys
D. Uterus
E. Spleen
ANSWER: A

With the patient lying on the right side at the lowered terminal end of the table well
to examine:
A. splenic flexure of the colon
B. Pancreas
C. Kidneys
D. Uterus
E. Spleen
ANSWER: A

What diameter of trocar is used in most surgeries in modern laparoscopy?


A. 2 and 4 mm
B. 4 and 8 mm
C. 5 and 10 mm
D. 15 and 20 mm
E. 10 and 15 mm
ANSWER: C

What group of instruments does “Babcock” clamp belong?


A. surgical clamp
B. atraumatic clamp
C. dissector
D. instrument for bipolar coagulation
E. for applying continuous sutures
ANSWER: B

What is culdoscopy?
A. Overview of the abdominal cavity using a special endoscope
B. Overview of the chest cavity using a special endoscope
C. Review of the mediastinum using a special endoscope
D. Review the joint cavity using a special endoscope
E. Pelvic exam using a special endoscope
ANSWER: E

What is laparoscopy?
A. Overview of the abdominal cavity using a special endoscope
B. Overview of the chest cavity using a special endoscope
C. Review of the mediastinum using a special endoscope
D. Review the joint cavity using a special endoscope
E. Pelvic exam using a special endoscope
ANSWER: A

What is needed to perform arthroscopy?


A. Introduction of air into the abdominal cavity
B. Intubation of main bronchi
C. The introduction of air into the chest cavity
D. The introduction of air into the joint cavity
E. The introduction of fluid into the joint cavity
ANSWER: E

What process represents the IV type of smear at oncocytological examination?


A. the unchanged epithelium.
B. mild or moderate dysplasia.
C. cancer.
D. inflammatory process.
E. intraepithelial cancer.
ANSWER: E

At diagnostic laparoscopy
A. You can change the position of the patient
B. You can not change the position of the patient
C. You can change the position of the patient in the sagittal plane
D. You can change the position of the patient in the frontal plane
E. preferably is not to confuse the patient
ANSWER: A

Pelvis should be examined with the patient:


A. On the back of the horizontal position of the table
B. On the back of the head at the lowered end of the table
C. On the right side of the raised head end of table
D. On the left side of the raised head end of table
E. on the left side of the horizontal position of the table
ANSWER: D

At diagnostic laparoscopy dark red, brown or black pancreas, the presence of


hemorrhagic fluid "stearic plaques" indicates
A. Mixed pancreatic necrosis
B. Focal fatty pancreatic necrosis
C. Focal hemorrhagic necrotizing pancreatitis
D. Biliary pancreatitis
E. Intact pancreas
ANSWER: A

Planned therapeutic laparoscopy is indicated for all of the states except:


A. pain caused by adhesions in the abdominal cavity
B. cirrhosis
C. retention of ovarian cysts
D. pancreatic pseudocyst
E. papillary serous ovarian cysts
ANSWER: E

Preparing the patient for emergency laparoscopy includes all of the listed except:
A. Placing stomach tube
B. Identifying the blood group and Rh
C. drink methylene blue
D. Explain to the patient that help is needed from him during the investigation
E. Enter the intra-muscular 1 ml of 0.1% solution of atropine sulfate and 2%
promedol
ANSWER: C

Preparing the patient for routine laparoscopy includes all of the listed except:
A. cleansing enemas
B. sedatives
C. teaching the patient to "inflate" the abdomen
D. emptying the bladder
E. abdominal X-rays
ANSWER: E

Round ligament of the liver is projected onto the line connecting:


A. Navel and right upper quadrant
B. Navel and the left upper quadrant
C. Navel and xyphoid
D. Umbilicus and symphysis
E. Navel and left inguinal quadrant
ANSWER: A

Sigmoid colon is available to view when the patient is:


A. On the back of the horizontal position of the table
B. On the back of the head end of the table lowered
C. On the left side of the horizontal position of the table
D. On the left side of the head at the closed end of the table
E. on the right side with slightly lowered head end of the table
ANSWER: E

Survey of appendix should be at the position of the patient:


A. On the back of the horizontal position of the table
B. On the back of the head end of the table announced
C. On the right side of the lowered head end of table
D. On the left side of the lowered head end of table
E. on the right side
ANSWER: D

The following areas of the abdomen are not neighboring:


A. mesogastrium and epigastrium
B. mesogastrium and hypogastrium
C. hypogastrium and epigastrium
D. All answers are correct
E. There is no right answer
ANSWER: C

The line connecting the pelvic spines divides abdomen on


A. mesogastrium and epigastrium
B. mesogastrium and hypogastrium
C. hypogastrium and epigastrium
D. All answers are correct
E. There is no right answer
ANSWER: B

The right kidney can be easily seen when:


A. summing the lower edge of the right lobe of the liver
B. shifting down transverse angle of the colon
C. shifting upwards hepatic angle of the colon
D. shifted laterally ascending colon
E. shifted medially ascending colon
ANSWER: A

To better view the gallbladder additional trocar introduced in


A. left upper quadrant
B. umbilical area
C. the left side of the area
D. the right upper quadrant
E. above the vagina
ANSWER: D

To view appendix the additional trocar is injected into the peritoneal cavity in
A. the upper right point of Tracing
B. the upper left point of Tracing
C. in the suprapubic area
D. the white line of the abdomen 5 cm above the umbilicus
E. At the point Mc BURNEO
ANSWER: C

Trocar deeply entered in the left iliac area may damage:


A. The left fallopian tube
B. liver
C. Stomach
D. The ascending colon
E. bladder
ANSWER: B

Trocar deeply entered in the left iliac area may damage:


A. Left ovary
B. liver
C. Stomach
D. The ascending colon
E. bladder
ANSWER: B

Trocar deeply entered in the left subcostal area may damage:


A. Ovaries
B. liver
C. Stomach
D. Uterus
E. bladder
ANSWER: C

Trocar deeply entered in the left subcostal area may damage:


A. Ovaries
B. liver
C. The tail of the pancreas
D. Uterus
E. bladder
ANSWER: C

Trocar deeply entered in the left subcostal area may damage:


A. Ovaries
B. liver
C. Left adrenal
D. Uterus
E. bladder
ANSWER: C

Trocar deeply entered the left side of the area can damage:
A. Ovaries
B. liver
C. Stomach
D. the descending colon
E. bladder
ANSWER: D

Trocar deeply entered the left upper quadrant can damage:


A. Ovaries
B. liver
C. Stomach
D. small intestine
E. bladder
ANSWER: D

Trocar deeply introduced in suprapubic area may damage:


A. The left iliac vessels
B. liver
C. bladder
D. The ascending colon
E. Stomach
ANSWER: C

Trocar deeply introduced in suprapubic area may damage:


A. The left iliac vessels
B. liver
C. the uterus
D. The ascending colon
E. Stomach
ANSWER: C

Trocar deeply introduced in the epigastric region may damage:


A. Stomach
B. Spleen
C. Kidneys
D. Uterus
E. bladder
ANSWER: A

Trocar deeply introduced in the epigastric region may damage:


A. pancreas
B. Spleen
C. Kidneys
D. Uterus
E. bladder
ANSWER: A

Trocar deeply introduced in the epigastric region may damage:


A. The left lobe of the liver
B. Spleen
C. Kidneys
D. Uterus
E. bladder
ANSWER: A
Trocar deeply introduced in the right subcostal area may damage:
A. Stomach
B. Gallbladder
C. Ovaries
D. Uterus
E. bladder
ANSWER: B

Trocar deeply introduced in the right subcostal area may damage:


A. Stomach
B. The structures of hepato-duodenal links
C. Ovaries
D. Uterus
E. bladder
ANSWER: B

Trocar deeply introduced in the right subcostal area may damage:


A. Stomach
B. The upper pole of the right kidney
C. Ovaries
D. Uterus
E. bladder
ANSWER: B

Trocar deeply introduced in the umbilical area may damage:


A. Kidneys
B. liver
C. Stomach
D. small intestine
E. bladder
ANSWER: D

At diagnostic laparoscopy with a picture of acute cholecystitis there are signs of


inflammation of the pancreas at such state as
A. Biliary acute pancreatitis
B. Focal necrotizing pancreatitis
C. Focal hemorrhagic necrotizing pancreatitis
D. Mixed pancreatic necrosis
E. Intact pancreas
ANSWER: A

Trocar deeply put into the right iliac area may damage:
A. Right ovary
B. liver
C. Stomach
D. The ascending colon
E. bladder
ANSWER: A
At initial stages liver surface is smooth, dark brown in color with a greenish
tinge. This statement is typical for laparoscopy at
A. Hemochromatosis of liver
B. Primary biliary cirrhosis
C. Obstructive cholangitis
D. Obstructive jaundice
E. Viral hepatitis
ANSWER: A

Trocar deeply put into the right iliac area may damage:
A. Right iliac vessels
B. liver
C. Stomach
D. The ascending colon
E. bladder
ANSWER: A

At laparoscopic examination
A. You can change the position of the patient
B. You can not change the position of the patient
C. You can change the position of the patient in the sagittal plane
D. You can change the position of the patient in the frontal plane
E. No need to change the position of the patient
ANSWER: A

Trocar deeply put into the right side of the area can damage:
A. Ovaries
B. liver
C. Stomach
D. the right kidney
E. bladder
ANSWER: D

Trocar introduced along the linea alba below the umbilicus on its way passes the
following layers of the abdominal wall:
A. The skin, the front piece aponeurosis, rectus muscle, peritoneum
B. The skin, subcutaneous tissue, anterior leaflet aponeurosis, rectus muscle,
peritoneum
C. The skin, subcutaneous fat, aponeurosis and peritoneum
D. skin, aponeurosis, preperitoneal fat and peritoneum
E. The skin, subcutaneous fat, anterior leaflet aponeurosis, rectus abdominis muscle,
posterior leaflet aponeurosis, preperitoneal fat and peritoneum
ANSWER: E

Trocar put on the left or right iliac areas is on its way passes the following layers
of the abdominal wall:
A. The skin, the front piece aponeurosis, rectus muscle, peritoneum
B. The skin, subcutaneous tissue, anterior leaflet aponeurosis, rectus muscle,
peritoneum
C. The skin, subcutaneous fat, aponeurosis and peritoneum
D. The skin, subcutaneous fat, aponeurosis of external oblique muscle, the internal
oblique and transverse muscles preperitoneal tissue, peritoneum
E. The skin, subcutaneous fat, aponeurosis, preperitoneal fat and peritoneum
ANSWER: D

Trocar put on the left or right subcostal areas on its way passes the following layers
of the abdominal wall:
A. The skin, the front piece aponeurosis, rectus muscle, peritoneum
B. The skin, subcutaneous tissue, anterior leaflet aponeurosis, rectus muscle,
peritoneum
C. The skin, subcutaneous fat, aponeurosis and peritoneum
D. The skin, subcutaneous fat, external, internal oblique and transverse muscles
preperitoneal tissue, peritoneum
E. The skin, subcutaneous fat, aponeurosis, preperitoneal fat and peritoneum
ANSWER: D

Urgent Care laparoscopy is indicated for all these states, except:


A. mesenteric thrombosis
B. hepatic subcapsular hematoma
C. acute pancreatitis
D. acute cholecystitis
E. jaundice
ANSWER: B

At laparoscopic examination enlarged and streched by contents gallbladder can be


seen at
A. acute simple cholecystitis
B. acute phlegmonous cholecystitis
C. acute gangrenous cholecystitis
D. acute gangrenous perforated cholecystitis
E. perforated cholecystitis
ANSWER: A

Vertical lines running along the outer edge of the rectus abdominis muscle separate
mesogastrium on following areas:
A. The left and right side
B. The left and right iliac
C. The left and right subcostal
D. All answers are correct
E. There is no right answer
ANSWER: A

At laparoscopic examination on the background of hyperemia fibrin layers can be


observed at
A. acute phlegmonous cholecystitis
B. acute cholecystitis simple
C. acute gangrenous cholecystitis
D. acute gangrenous cholecystitis, perforated
E. perforated cholecystitis
ANSWER: A

When planning therapeutic manipulation at acute pancreatitis laparoscope should be


introduced into the abdominal cavity through a:
A. upper right point of Calc
B. upper left point of Calc
C. lower left point of Calc
D. the white line of the abdomen above the pubis
E. at the point of McBurney
ANSWER: C

At laparoscopic examination at acute gangrenous cholecystitis characteristic


appearance of the gallbladder is following
A. the presence of black spots with fibrinous layers
B. enlarged and streched by contents gallbladder
C. hyperemia on the background and fibrinous layerings
D. the presence of black spots and leak of bile contents into abdominal cavity
E. Tightly wrapped bigger omentum around gallbladder
ANSWER: A

With the patient lying on his back at the lowered terminal end of the table well to
examine:
A. Gallbladder
B. Stomach
C. Recto-sigmoid colon angle
D. Uterus
E. Spleen
ANSWER: B

With the patient lying on his back at the lowered terminal end of the table well to
examine:
A. Gallbladder
B. Abdominal esophagus
C. the right lobe of the liver
D. Uterus
E. Spleen
ANSWER: B

With the patient lying on his back in a horizontal position table well to examine:
A. Gallbladder
B. Pancreas
C. small intestine
D. sigmoid
E. Spleen
ANSWER: C
With the patient lying on his back with lowered head end of the table well to examine:
A. Gallbladder
B. Pancreas
C. Recto-sigmoid colon angle
D. Uterus
E. Spleen
ANSWER: D

With the patient lying on his back with lowered head end of the table well to examine:
A. Gallbladder
B. Pancreas
C. Recto-sigmoid colon angle
D. bladder
E. Spleen
ANSWER: D

With the patient lying on his back with lowered head end of the table well to examine:
A. Gallbladder
B. Pancreas
C. Recto-sigmoid colon angle
D. uterine-bladder space
E. Spleen
ANSWER: D

With the patient lying on his back with lowered head end of the table well to examine:
A. Gallbladder
B. Pancreas
C. Recto-sigmoid colon angle
D. Ovaries
E. Spleen
ANSWER: D

With the patient lying on his right side with horizontal table well to examine:
A. Spleen
B. Pancreas
C. the right lobe of the liver
D. Uterus
E. Right side channel
ANSWER: A

At laparoscopic examination at acute simple cholecystitis characteristic appearance of


the gallbladder is following
A. enlarged and streched by contents gallbladder
B. hyperemia on the background and fibrinous layerings
C. the presence of black spots with fibrinous layers
D. the presence of black spots and leak of bile contents into abdominal cavity
E. tightly wrapped bigger omentum around gallbladder
ANSWER: A
With the patient lying on his right side with lowered head end table well to examine:
A. Gallbladder
B. Pancreas
C. Recto-sigmoid colon angle
D. The right ovary
E. Spleen
ANSWER: C

With the patient lying on the left side at the lowered terminal end of the table helps
to examine:
A. Spleen
B. Pancreas
C. duodenum
D. Uterus
E. Ovaries
ANSWER: C

With the patient lying on the left side in the horizontal position of the table helps
to examine:
A. Spleen
B. Pancreas
C. the right lobe of the liver
D. Uterus
E. The ascending colon
ANSWER: E

With the patient lying on the right side at the lowered terminal end of the table well
to examine:
A. Spleen
B. Pancreas
C. Kidneys
D. Uterus
E. Ovaries
ANSWER: A

At laparoscopy on suspicion of extrahepatic bile duct cancer, the second port input is
carried out
A. In the midline epigastrium
B. At Volkovych-Kocher point
C. On the midline of the abdomen in hypogastrium 10 cm below the navel
D. At McBurney point
E. At Kerr point
ANSWER: A

At laparoscopy on suspicion of liver cancer, the introduction of instrumental port is


performed
A. On the midline of the abdomen in the epigastrium 10 cm above the navel
B. In mesogastrium 1 cm above or below the navel the median line
C. At Volkovych-Kocher point
D. At McBurney point
E. At Kerr point
ANSWER: A

At laparoscopy on suspicion of liver cysts, the second input port is carried out at
A. Epigastrium
B. At Volkovych-Kocher point
C. On the midline of the abdomen in hypogastrium 10 cm below the navel
D. At McBurney point
E. At Kerr point
ANSWER: A

At laparoscopy on suspicion of right adnexitis, the first port site is


A. In mesogastrium 1 cm above or below the navel the median line
B. On the midline of the abdomen in the epigastrium 10 cm above the navel
C. On the midline of the abdomen in hypogastrium 10 cm below the navel
D. At McBurney point
E. At Volkovych-Kocher point
ANSWER: A

For laparoscopy for acute pancreatitis second port input is made


A. On the midline of the abdomen in the epigastrium
B. In mesogastrium 1 cm above or below the navel the median line
C. On the midline of the abdomen in hypogastrium 10 cm below the navel
D. At McBurney point
E. At Kerr point
ANSWER: A

For laparoscopy for acute simple appendicitis, instrumental port is set


A. Below the navel in hypogastrium
B. In mesogastrium 1 cm above or below the navel the median line
C. At midline abdominal epigastrium 5 cm above the umbilicus
D. At McBurney point
E. At Volkovich point
ANSWER: A

For laparoscopy for acute simple appendicitis, setting the port to perform laparoscopy
is conducted
A. At McBurney point
B. At Volkovich point
C. In mesogastrium 1 cm above or below the navel at the median line
D. At midline abdominal epigastrium 5 cm above the umbilicus
E. On the midline of the abdomen in hypogastrium 5 cm below the navel
ANSWER: C

A specific antyamebic medicine is:


A. amynoglicozides
B. metronidazol
C. emetine, chloroquine, diphosphate
D. quinine
E. cephalosporines
ANSWER: C

What solution is used for conducting of liquid hysteroscopy?


A. a 5% solution of glucose.
B. a 10% solution of chloride of sodium.
C. gemodez.
D. a 40% glucose solution.
E. polygluлin.
ANSWER: E

At laparoscopic examination of the presence of black spots with fibrinous layers


can be observed in
A. acute gangrenous cholecystitis
B. acute cholecystitis simple
C. acute phlegmonous cholecystitis
D. acute gangrenous cholecystitis, perforated
E. perforated cholecystitis
ANSWER: A

What sounding of uterine cavity is not used for?


A. for determination of permeability of cervical canal.
B. for determination straight of cervical canal.
C. for determination of length of uterine cavity.
D. for the exposure of tumors in the uterine cavity.
E. for the exposure of tumors of ovaries.
ANSWER: E

Cancer of the gallbladder under review is characterized by the following features


A. the gallbladder has whitish dense bulbs on the surface
B. the gallbladder is enlarged, streched, white and blue color
C. the gallbladder is enlarged, dark purple color, through extensive serous membrane
translucent mesh of blood vessels and capillaries
D. gallbladder glued with surrounding organs, constricted, scars or sclerosis
E. none of the listed
ANSWER: A

Chronic calculous cholecystitis under review is characterized by the following


features
A. gallbladder glued with surrounding organs, constricted, scars or sclerosis
B. the gallbladder is enlarged, streched, white and blue color
C. the gallbladder is enlarged, dark purple color, through extensive serous membrane
translucent mesh of blood vessels and capillaries
D. the gallbladder has whitish dense bulbs on the surface
E. none of the listed
ANSWER: A

During laparoscopic examination, surgeon pays attention to


A. shape and size of organs, their color, vascular pattern, signs of inflammation,
tumor, metastasis and free fluid in the abdomen
B. shape and size of organs, vascular pattern, signs of inflammation, tumor
metastasis and free fluid in the abdomen
C. shape and size of their color, signs of inflammation, tumor, metastasis and free
fluid in the abdomen
D. shape and size of their color, vascular pattern, signs of inflammation, free
fluid in the abdomen
E. shape and size of their color, vascular pattern, signs of inflammation, tumors
ANSWER: A

Edema of the gallbladder under viewed characterized by the following features


A. the gallbladder is enlarged, streched, white and blue color
B. the gallbladder is enlarged, dark purple color, through extensive serous membrane
translucent mesh of blood vessels and capillaries
C. gallbladder glued with surrounding organs, constricted, scars or sclerosis
D. the gallbladder has whitish dense bulbs on the surface
E. none of the listed
ANSWER: A

Enlarged and streched by contents gallbladder can be seen at


A. acute simple cholecystitis
B. normal gallbladder
C. acute gangrenous cholecystitis
D. acute gangrenous cholecystitis, perforated
E. perforated cholecystitis
ANSWER: A

Fibrinous layers can be observed in


A. acute phlegmonous cholecystitis
B. acute cholecystitis simple
C. norm
D. acute perforated gangrenous cholecystitis
E. perforated cholecystitis
ANSWER: A

Focal fatty pancreatic necrosis is characterized by


A. presence of scattered on the surface of large and small omentum or elsewhere
"stearic plaques"
B. presence of hemorrhagic fluid in the free abdominal cavity
C. dark red, brown or black pancreas, the presence of hemorrhagic fluid in the free
abdominal cavity, the presence of "stearic plaques"
D. visual picture of acute cholecystitis together with characteristic signs of
inflammation of the pancreas
E. There is no specific sign
ANSWER: A

For hemochromatosis of liver during laparoscopy is typical


A. liver surface smooth, dark brown
B. fine capillary net of blood vessels
C. liver is smooth gray-brown.
D. intense brownish-green color
E. moderate greenish tint
ANSWER: A

Greenish tint of liver is characteristic for laparoscopy at


A. Obstructive cholangitis
B. Hemochromatosis of liver
C. Primary biliary cirrhosis
D. Obstructive jaundice
E. Normally
ANSWER: A

In acute gangrenous cholecystitis characteristic appearance of the gallbladder is


following
A. the presence of black spots with fibrinous layers
B. enlarged and streched by contents gallbladder
C. hyperemia on the background and fibrinous layerings
D. the presence of black spots and leak of bile contents into abdominal cavity
E. Tightly wrapped bigger omentum around gallbladder
ANSWER: A

In acute phlegmonous cholecystitis characteristic appearance of the gallbladder is


following
A. hyperemia on the background and fibrinous layerings
B. enlarged and streched by contents gallbladder
C. the presence of black spots with fibrinous layers
D. the presence of black spots and leak of bile contents into abdominal cavity
E. Tightly wrapped bigger omentum around gallbladder
ANSWER: A

In obstructive cholangitis liver characterized


A. moderate greenish tint
B. fine capillary net of blood vessels and soft whitish mesh lymphatic vessels
C. during laparoscopy liver is enlarged, thick consistency. The surface of the liver
is smooth gray-brown
D. intense brownish-green color
E. at early stages of liver surface is smooth, dark brown in color with a greenish
tinge
ANSWER: A

In viral hepatitis for liver during laparoscopy most characteristic is


A. fine capillary net of blood vessels and soft whitish mesh of lymphatic vessels
B. liver enlarged, thick consistency, liver is smooth gray-brown.
C. intense brownish-green color
D. moderate greenish tint
E. liver surface is smooth, dark brown in color with a greenish tinge
ANSWER: A
Intact pancreas is characterized by
A. lobular gland structure
B. at the surface "of stearic plaques"
C. presence of hemorrhagic fluid in the free abdominal cavity
D. dark red pancreas
E. none of the listed
ANSWER: A

Laparoscopic diagnostic review is conducted


A. consistently better from right to left and top to bottom
B. consistently better left to right and top to bottom
C. not consistently better from right to left and from top to bottom
D. consistently better from right to left
E. consistently better from top to bottom
ANSWER: A

After the Veres needle introduction one must perform tests that indicate:
A. The pressure in the abdomen
B. Location of the distal end of the needle
C. Depth of anesthesia
D. All answers are correct
E. There is no right answer
ANSWER: B

Mixed pancreatic necrosis is characterized by


A. dark red, brown or black pancreas, the presence of hemorrhagic fluid in the free
abdominal cavity, the presence of "stearic plaques"
B. presence of scattered on the surface of large and small omentum or elsewhere
"stearic plaques"
C. presence of hemorrhagic fluid in the free abdominal cavity
D. visual picture of acute cholecystitis together with characteristic signs of
inflammation of the pancreas
E. There is no specific signs
ANSWER: A

Overview laparoscopy is conducted


A. sequentially from left to right and top to bottom
B. consistently better left to right and top to bottom
C. not consistently better from right to left and from top to bottom
D. not consistently better from right to left
E. consistently better from top to bottom
ANSWER: A

Primary biliary cirrhosis is characterized by the following at review


A. during laparoscopy liver is enlarged, thick consistency, the surface of the liver
is smooth gray-brown
B. fine capillary net of blood vessels and soft whitish mesh lymphatic vessels
C. intense brownish-green color
D. moderate greenish tint
E. surface of the liver is smooth, dark brown in color with a greenish tinge
ANSWER: A

Surface of liver is smooth, dark brown in color with a greenish tinge. This statement
is typical for laparoscopy at
A. Hemochromatosis of liver
B. Norm
C. Obstructive cholangitis
D. Obstructive jaundice
E. Viral hepatitis
ANSWER: A

The gallbladder is enlarged, dark purple color, through extensive serous membrane
translucent mesh of blood vessels and capillaries is in such pathologies as
A. Empyema of the gallbladder
B. Edema of the gallbladder
C. Chronic calculous cholecystitis
D. Cancer of the gallbladder
E. None of the
ANSWER: A

Where is a smear taken for oncocytology before hysteroscopy?


A. from the lateral fornix of vagina and cervical canal.
B. from vaginal part of uterine cervix.
C. from the back fornix of vagina and cervical canal.
D. from a cervical canal.
E. from vaginal part of uterine cervix and cervical canal.
ANSWER: E

The presence of black spots and leak of bile contents into abdominal cavity can be
seen at
A. acute perforated gangrenous cholecystitis
B. acute cholecystitis simple
C. acute phlegmonous cholecystitis
D. norm
E. perforated cholecystitis
ANSWER: A

The presence of hemorrhagic fluid in the free abdominal cavity indicates


A. Hemorrhagic necrotizing pancreatitis
B. Focal fatty pancreatic necrosis
C. Mixed pancreatic necrosis
D. Biliary pancreatitis
E. Intact pancreas
ANSWER: A

Tightly wrapped bigger omentum around gallbladder can prevent overview at


A. probable bladder perforation
B. acute cholecystitis simple
C. acute phlegmonous cholecystitis
D. acute gangrenous cholecystitis
E. acute gangrenous cholecystitis, perforated
ANSWER: A

To perform a diagnostic laparoscopy on suspected perforated ulcer, input of optical


port is typically carried out
A. In the periumbilical area
B. On the midline of the abdomen in the epigastrium
C. At Kerr point
D. At McBurney point
E. At Volkovych-Kocher point
ANSWER: A

To perform a diagnostic laparoscopy on suspicion of acute appendicitis, port for


laparoscope is typically placed
A. In the periumbilical area
B. On the midline of the abdomen in the epigastrium
C. At midline of abdominal wall 10 cm upwards the navel
D. At midline in hypogastrium 10 cm below the navel
E. At Volkovych-Kocher point
ANSWER: A

Together with visual picture of acute cholecystitis characteristic signs of


inflammation of the pancreas indicate
A. Biliary pancreatitis
B. Focal fatty pancreatic necrosis
C. Focal hemorrhagic necrotizing pancreatitis
D. Mixed pancreatic necrosis
E. Intact pancreas
ANSWER: A

When carrying out a diagnostic laparoscopy on suspicion of acute cholecystitis, input


of optical port is typically carried out
A. In the periumbilical area
B. At Kerr point
C. On the midline of the abdomen in hypogastrium
D. At McBurney point
E. At Volkovych-Kocher point
ANSWER: A

When carrying out a diagnostic laparoscopy on suspicion of cirrhosis, enter of the


first port is conducted
A. In the periumbilical area
B. On the midline of the abdomen in the epigastrium
C. On the midline of the abdomen in hypogastrium
D. At McBurney point
E. At Volkovych-Kocher point
ANSWER: A

When diagnostic laparoscopic examination in obstructive jaundice liver has


A. brownish-green intensive color
B. soft whitish mesh of lymphatic vessels
C. liver enlarged, thick consistency. The surface of the liver is smooth gray-brown
D. moderate greenish tint
E. liver surface is smooth, dark brown in color with a greenish tinge
ANSWER: A

When diagnostic laparoscopy attention is always payed to


A. signs of inflammation, tumor metastasis and free fluid in the abdominal
cavity, the shape and size of their color, vascular pattern
B. shape and size of organs, vascular pattern, signs of inflammation, tumor
metastasis and free fluid in the abdomen
C. shape and size of their color, signs of inflammation, tumor metastasis and free
fluid in the abdomen
D. shape and size of their color, vascular pattern, signs of inflammation, free
fluid in the abdomen
E. shape and size of their color, vascular pattern, signs of inflammation, tumors
ANSWER: A

When diagnostic laparoscopy body surface is smooth, the gallbladder is enlarged,


streched, white and blue color at
A. Edema of the gallbladder
B. Empyema of the gallbladder
C. Chronic non-calculous cholecystitis
D. Cancer
E. Norm
ANSWER: A

When diagnostic laparoscopy gallbladder empyema under review is as follows


A. the gallbladder is enlarged, dark purple color, through serous membrane there is
translucent mesh of blood vessels and capillaries
B. the gallbladder is enlarged, streched, white and blue color
C. gallbladder glued with surrounding organs, constricted, scars or sclerosis
D. the gallbladder has whitish dense bulbs on the surface
E. none of the listed
ANSWER: A

When diagnostic laparoscopy gallbladder edema during the inspection is visualized as


follows
A. the gallbladder is enlarged, streched, whitish and blue color
B. gallbladder of dark purple color extensive vascular net shines through the serosa
C. gallbladder glued with surrounding organs, constricted, scars or sclerosis
D. the gallbladder has whitish dense bulbs on the surface
E. none of the listed
ANSWER: A
When diagnostic laparoscopy, overview is achieved by
A. manipulation of table the patient is fixed to
B. manipulation of patient
C. manipulation of table to which the patient is fixed only sagittally
D. manipulation of table to which the patient is fixed only frontally
E. manipulation of table to which the patient is fixed only by height
ANSWER: A

At diagnostic laparoscopy the gallbladder is enlarged, dark purple color, through


extensive serous membrane translucent mesh of blood vessels and capillaries. It
characterizes
A. Empyema of the gallbladder
B. Edema of the gallbladder
C. Chronic non-calculous cholecystitis
D. Cancer
E. Normal gallbladder
ANSWER: A

Antibiotic therapy of liver abscesses should be based on the following principles


A. the use of broad-spectrum antibiotics
B. a combination of antibiotics
C. antibiotics only after determining the nature of its flora
D. degree in accounting bile excretion of antibiotics
E. all listed
ANSWER: E

Clinic extrahepatic portal hypertension does not include:


A. Splenomegaly with hypersplenism or without
B. Splenomegaly with hypersplenism
C. varicose veins of the esophagus
D. ascites
E. cholesterosis of gallbladder
ANSWER: D
Complaints of cough with purulent sputum, increased body temperature to 39°C, pain in
the left half of the chest. Has been ill for 2 weeks, the onset is caused by
undercooling. The lag of the left half of the chest during breathing, the shortening
of percussion sound over the lower lobe, by auscultation weakened breathing with
amphoric sound, crackling rales. On X-ray of the chest expressed infiltration of lung
tissue with enlightenment in the center with fluid level. What is the primary
diagnosis?
A. Acute lung abscess.
B. Pleural empyema.
C. Abscessing pneumonia.
D. Pyopneumothorax.
E. Bronchiectatic disease.
ANSWER: A

Complaints of cough with purulent sputum, increased body temperature to 39°C, pain in
the left half of the chest. Has been ill for 2 months, the onset is caused by
undercooling. The lag of the left half of the chest during breathing, the shortening
of percussion sound over the lower lobe, by auscultation weakened breathing with
amphoric sound. On X-ray of the chest the destruction cavity with the fibrous capsule
in the projection of lower lobe of the left lung, infiltration of lung tissue is not
determined. What is the primary diagnosis?
A. Chronic lung abscess.
B. Pleural empyema.
C. Acute lung abscess
D. Abscessing pneumonia.
E. Pyopneumothorax.
ANSWER: A

Complaints of cough with purulent sputum, increased body temperature to 39°C, pain in
the left half of the chest. Has been ill for 2 weeks, the onset is caused by
undercooling. The lag of the left half of the chest during breathing, over the left
lobe a dull percussion sound, by auscultation the breathing is absent. On X-ray of the
chest the shadow in the basal parts of the left lung with an oblique upper level along
Damuazo's line. What is the primary diagnosis?
A. Pleural empyema.
B. Acute lung abscess
C. Chronic lung abscess.
D. Suppurative cyst of the lung.
E. Pyopneumothorax.
ANSWER: A

Complaints of cough with foul-smelling purulent sputum with streaks of blood,


increased body temperature to 40°C, pain in the left half of the chest, dyspnea at
rest. Has been ill for 2 weeks, the onset is caused by undercooling. The lag of the
left half of the chest during breathing, with a shortening of the pulmonary percussion
sound, by auscultation moist rales over the lower lobe of the left lung. On X-ray of
the chest expressed infiltration of left lung tissue with a giant cavity in the lower
lobe with the level of the fluid. What is the primary diagnosis?
A. Gangrenous abscess of lung.
B. Pleural empyema.
C. Acute lung abscess
D. Abscessing pneumonia.
E. Gangrene of the lung.
ANSWER: A

Complaints of cough with purulent sputum, increased body temperature to 39°C, pain in
the left half of the chest. Has been ill for 2 weeks, the onset is caused by
undercooling. The lag of the left half of the chest during breathing, the shortening
of percussion sound over the lower lobe, by auscultation weakened breathing. On X-ray
of the chest paracostal fusiform shadow in the projection of the left lower lobe of
the lung. What is the primary diagnosis?
A. Limited empyema.
B. Wide-spread pleural empyema.
C. Acute lung abscess
D. Chronic lung abscess.
E. Pyopneumothorax.
ANSWER: A

The patient, 78 years old, entered with complaints of pain in the left half of the
chest, coughing, with daily 80 ml of mucopurulent sputum, fever to 37,2°C. The X-rays
of the lower lobe of right lung revealed a cavity with irregular internal border and
outside spicules with minor infiltration around. What is the primary diagnosis?
A. Hollow form of lung cancer
B. Chronic lung abscess
C. Suppuration cyst of lung
D. Fibro-cavernous tuberculosis
E. Limited empyema
ANSWER: A

The patient has the pyogenic lung abscess, which was complicated by bleeding. What
medicines are the most suitable to stop the bleeding?
A. Vitamin K.
B. Anticoagulants.
C. Antibiotics.
D. Antiaggregants.
E. Prostaglandins.
ANSWER: A

The patient received chest trauma 2 hours ago. Complains of the severe pain in the
right half of the chest, dyspnea at rest. On examination: the lag of the right half of
the chest during breathing, crepitation along the V-VI ribs on the right side, by
percussion - tympanic sound, by auscultation - breathing is absent. The primary
diagnosis: Closed chest trauma. Fractures of V-VI ribs on the right side.
Posttraumatic pneumothorax. What is the treatment of pneumothorax?
A. Pleural drainage
B. Pleural puncture
C. Thoracotomy
D. Pneumonectomy, bilobectomy, lobectomy
E. Conservative treatment
ANSWER: A

Was pressed by the truck to the wall. Complains of the expressed dyspnea, difficult
breathing, chest pain. On examination the expressed cyanosis. The frequency of
respiratory movements - 26-28 per 1 min. Unstable hemodynamics. The chest is deformed,
abnormal mobility of the front wall. The swelling of soft tissues of the neck with
crepitation. What is the primary diagnosis?
A. Mediastinal emphysema.
B. Posttraumatic pneumothorax.
C. Posttraumatic hemothorax.
D. Posttraumatic pneumonia
E. Subcutaneous emphysema.
ANSWER: A

Female patient, 62 years old, was got in accident. On examination was detected the
region of the right half of the chest, which disengages during inspiration. What are
the most appropriate therapeutic measures?
A. External fixation of a floating area
B. Introduction of narcotic analgetics
C. Vagosympathetic block by Vishnevsky
D. Tight chest bandage
E. Paravertebral blockade
ANSWER: A

In the patient on the fourth day after the chest trauma on X-ray - heterogeneous
shadow in the lower lobe. By puncture received a small amount of light yellow fluid
with blood clots. What treatment are the best for the patient?
A. Drainage of the pleural cavity
B. Operational - lung decortication
C. Daily puncture
D. Resorbed therapy
E. Antibacterial therapy
ANSWER: A

Patient S., 35 years old, entered the hospital after the chest trauma. During clinical
and X-ray examination was diagnosed the left-side hemothorax. Where the drainage of
pleural space in hemothorax is performed?
A. VII intercostal space, scapular line
B. II intercostal space, midclavicular line
C. II intercostal space, scapular line
D. IV intercostal space, anterior axillary line
E. VII intercostal space, midclavicular line
ANSWER: A

Patient S., 35 years old, entered the hospital after the chest trauma. During clinical
and X-ray examination was diagnosed the left-side hemothorax. What method is the most
informative in the diagnostic of hemothorax?
A. Pleural puncture
B. General blood analysis
C. Sputum analysis
D. Auscultation
E. X-ray examination
ANSWER: A

54 years old patient complains of dysphagia. Two years ago noticed on the left side of
neck the appearance of protrusion after eating, vomiting by food, night cough. Began
to lose his weight. On X-ray of esophagus with barium at the level of the clavicle was
revealed the depot of barium like chicken egg by the size and shape. What operation is
performed in this disease?
A. Resection of diverticulum
B. Esophagomyotomy
C. Esophagogastric anastomosis
D. Extirpation of esophagus
E. Esophageal plastics by intestine
ANSWER: A

On X-ray of the esophagus in the right lateral projection in the middle third on the
front wall was found out the additional shadow, of round shape with smooth contours to
2 cm in diameter. What is the most probable diagnosis?
A. Diverticulum of the esophagus
B. Achalasia of the esophagus
C. Esophageal cancer
D. Chemical burn of the esophagus
E. Diaphragmatic hernia
ANSWER: A

Among the methods of esophageal plastic the most physiologic and safe modern method is:
A. Isoperistaltic plastic by tube of the greater curvature of the stomach after the
extirpation of the esophagus through a cervical-laparotomy access.
B. Large intestine plastic in antiperistaltic position of the transplant.
C. Large intestine plastic in isoperistaltic position of the transplant.
D. Large intestine plastic with a skin flap.
E. Large intestine plastic by ileocecal segment.
ANSWER: A

Complaints of the pain behind the breastbone, difficult passage of solid food, weight
loss, dizziness. Has been ill for 3 months. Last 2 days disturbs the vomiting after
fluid food, the stagnation of fluid food. On EGDS severe narrowing of the esophagus,
rigidity of the walls, hyperemic mucosa without folds. What is the most probable
diagnosis?
A. Esophageal cancer
B. Sliding esophageal hernia
C. Paraesophageal hernia
D. Reflux esophagitis
E. Varicose veins of the esophagus
ANSWER: A
53 year old man complains of recurrent pain behind the breastbone, heartburn,
especially in the horizontal position. Sometimes the burning pain behind the sternum
occurs after hot or spicy food. Two weeks ago was vomiting by blood and lost of
consciousness. Has entered the hospital after repeated gastric bleeding. What drugs
are used for the treatment of this pathology?
A. Blockers of proton pomp
B. Spasmolytics
C. Adrenoblockers
D. Blockers of calcium channel
E. Anticoagulants
ANSWER: A

What is the most wide-spread cause of the peptic stricture of esophagus associated
with reflux-esophagitis?
A. Sliding esophageal hernia.
B. Prolonged nasogastric intubation in the esophagus.
C. Short stay nasogastric intubation.
D. Frequent vomiting of pregnancy.
E. Achalasia of the esophagus
ANSWER: A

Complications paraesophageal hernias of esophagus:


A. Incarceration
B. Malignancy.
C. Esophageal stricture.
D. Dysphagia.
E. Reflux esophagitis.
ANSWER: A

A 42-year-old male comes in for a routine physical examination. He is noted to have


impaired glucose methabolism, and a BP of 135/85 mmHg. What is the best treatment plan
for this individual?
A. Aggressive lifestyle modification
B. Institute thiazide diuretic regimen
C. No treatment at this time
D. Initiate an ACE-inhibitor
E. Initiate a beta-blocker
ANSWER: A

For the case below, select the most significant adverse effect of the antihypertensive
and/or cardiac agent in question: a 45-year old female has been on diuretic, but BP
remains elevated at 145/95, leading to the proposed addition of lisinopril. Which key
potential adverse effect should be discussed?
A. Increased triglyceride levels
B. Peripheral edema
C. Lupus-like syndrome
D. Cough
E. Gynecomastia
ANSWER: D
The initial antihypertensive medication recommended for patients who have no
compelling indications or contraindications is
A. ACE-inhibitor
B. Calcium-channel blocker
C. Diuretics
D. Beta blocker
E. Any of the above
ANSWER: E

What is the appropriate course of action regarding the patient’s antihypertensive


therapy?
A. Advise a low-sodium diet
B. Finish doxazosin therapy and consider an alternative agent
C. Advise high dietary intake of calcium and potassium
D. Increase the doxazosin to 4 mg a day
E. Advise magnesium supplements
ANSWER: B

At a patient with the acute heart attack of myocardium best of all to warn relapsing
fibrillation of ventricles with :
A. cordaronum
B. lidocainum
C. ornidinum
D. electrocardiostimulation
E. there is no right answer
ANSWER: D

At a patient, that is found on medical treatment in the therapeutic department, the


sudden stopping of circulation of blood happene Medical personnel begun the
reanimation measure Define the most rational way of injection of adrenalin for
renewal of heart abbreviations in default of vein access:
A. To enter to a 3 ml solution of adrenalin in a trache
B. To enter to a 1 ml solution of adrenalin in muscl
C. To enter adrenalin in muscle, multiplying a dose in 3 time
D. The intracardial injection.
E. Adrenalin can be not entere
ANSWER: A

At a patient, that is found under the permanent electrocardioscopic supervision,


microwave fibrillation of myocardium and diagnosed clinical death develope It is
necessary to do:
A. to inject the solution of calcium in cor
B. to conduct high-voltage electric defibrillation
C. to inject solution of atropine in cor
D. to inject solution of adrenalin in cor
E. All answers are wrong
ANSWER: B
At the ventilated patient with the edema of lungs at low pressure and septic shock
intravenous infusion 7,5 mcg/kg/min dopamini will increase
A. RaO2, saturation of oxygen of the mixed vein blood, consumption of oxygen
B. diuresis
C. cardiac systolic volume
D. right A) and B)
E. faithful all answers
ANSWER: C

In 2 hours after renewal of cardial activity at a patient, that carried the sudden
stop of heart on a background hemorhagic shock (blood lost near 2,5 l) and is found on
ALV, unstable hemorhagia (AP – 80/40 – 90/60 mm of mer item, tahycardia)is marked,
central vein pressure – 5 mm wt.st. It is related to:
A. By Hypovolume syndrome
B. By cardia insufficiency
C. By the inadequate interchange of gases
D. Vasoplegia
E. By the inadequate anaesthetizing
ANSWER: A

In a clinic a patient with the traumatic tearing of both lower extremities off at the
level of knee-joints is delivere A patient is extremly inert, languid, pale, pulse
140 bmin, threadlike, AP 500. On both lower extremities there are the imposed plait
Bleeding at the receipt is not present. From the words of doctor of first-aid, lost
about 3 litres of blood in place of event. What principal reason of heavy of the state
of Patient?
A. acute hemorrhag
B. Pain shock.
C. Ishemia of extremities as a result of application of tourniquet
D. Fatty embolism
E. acute kidney insufficiency
ANSWER: A

It is typical for:
A. Negative CVT
B. Erroneous punction of artery
C. Right-side pmeumothorax
D. Edema of lungs
E. Hypodermic emphysema
ANSWER: A

Patient 20 years for verification of the functional state of kidneys the X- ray
examination with v/v injection of cardiotrast is conducte At the end of injection the
state of patient acutely became worse, the shortness of breath, hyperemia of skin,
itch appeare AP – 60/20 mm of mer item, HBA – 132/min. A similar research was
conducted 3 months ago, such effects were not observe What most reliable diagnosis?
A. Medicinal anafilactic shock
B. Acute kidney insufficiency
C. Tromboembolism of pulmonary artery
D. Stress on the conducted manipulation
E. Heart attack miocardium
ANSWER: A

Patient 38 years, native plasma was poure At the end of infusion the state became
worse: Patient confused, excited, cyanosys, hypersalivation. Breathing frequency 36 on
1 min., AP – 70/40 mm of mer item, whistling dry wheeze Which from the following
mediceni must be injected firstly?
A. Adrenalin.
B. Eufilin.
C. Suprastin.
D. Noradrenalin.
E. Prednizolon.
ANSWER: A

Patient 40 years with the acute gastro - intestinal bleeding a canned blood was poured
in a volume 400 ml after conducting of all tests on compatibility. After
hemotransfusion the state of patient became worse, appeared head pains and pains in
muscles athe temperature of body rose to 38,8 What can explaine the state of
patient?
A. Pirogenic reaction of middle heavy
B. By development of hemotransfusion shock
C. Alergic reaction
D. By development of bacterial-toxic shock
E. By air embolism
ANSWER: B

Patient 62 years the third day of presense in department of intensive therapy


concerning the acute transmural heart attack of myocardium of front-partition
localization. At night woked up from a suffocating cough, feeling of fear and troubl
At a review: cyanosys, FB – 30 after 1 min., HBA – 132/ min., a rhythm is correct,
tones of heart are deaf, accent ІІ tone above a pulmonary artery, AP – 180/110 mm
mercury. There is the loosened breathing above lungs with the far of moist wheezes in
lower fate What probably became the reason of worsening of the state?
A. Edema of lungs
B. Embolism of pulmonary artery
C. The repeated heart attack miocardium
D. Hypertensive crisis
E. Attack of bronchial asthma
ANSWER: A

Patient N., 28 years ol 6 day after the complicated birth The clinical hematological
signs of subacute disseminate intravascular coagulation syndrome developed after skin
hemorrhage and uterine bleeding. The state of patient is very ba blood: Er-2,7 of T/l,
Hb-78 of gm/l, CI - 0,93, L-4,7 of Gm/l, thrombocytes-88 of gm/l, time of blood
cloating - 16 min, prothrombin time - 25 sec, ethanol test +, fibrinogen-1,4 gramme/l,
What preparations should be prescribed ?
A. freezed plasma
B. Heparinum
C. Reopoliglycin
D. Cryoprecipitate
E. U-aminokapric acid
ANSWER: A

Patient N., 40 years, groom. In anamnesis there is an allergy to nonsteroidal


antyinflammatory. After injection of antytetanus on a method Besredco concerning the
hammered wound of right shin, through 20 mines, there was a acute weakness, labouring
breath, through 10 mines, loss of consciousnes What mechanism of development of
anafilactic form of illness?
A. Sensitization to the albumen of horse whey
B. Low quality of horse whey
C. Breach of the technique of PPS injection
D. Presence in anamnesis of medical allergy
E. Infection of whey
ANSWER: A

Pharmacological medicine, that diminish the (afterload) left ventricle at a patient


with the acute heart attack of myocardium, are not included
A. nitroglycerine
B. strophantine
C. nitroproussid sodium
D. esmolol (brevibloc)
E. nifedipinum
ANSWER: B

Sick 46 years treated oneself in a therapeutic department with pneumonia of lower


dole of right lung. Planned antibacterial therapy - amoxiklav. After 40 min after
intramuscular injection of duty dose, the patients feeled dizziness, pain behind a
breastbon AT 60/40 mm mer , pulse, - 120 a min., rhythmical. During examination of
lungs: wheezes under both lung Temperature is 38,5 What is worsening of the
condition related to?
A. Anaphylactic shock
B. Infectious toxic shock
C. Collapse
D. Tromboembolia of pulmonary artery
E. Infectious shock
ANSWER: A

The patient 32 years have infusion of native plasm At the end of infusion the state
became worse: disorientation, cyanosys, excitation, appeared hypersalivation, tahypnoe,
AP =70/40 mm mer st., in lungs – the dissipated dry wheeze What medicine must be
injected firstly?
A. Adrenalin.
B. Suprastin.
C. Gidrocortizon.
D. Dopamin.
E. Eufilin.
ANSWER: A
To patient P., 50 years, with an unspecific ulcerous colitis with the purpose of
correction of anaemia transfusion of selfgroup blood 500 ml A(ІІ) the Rh(-) was
conducte A doctor went out from a chamber after conducting of necessary tests before
hemotransfusion. In 20 minutes he was quickly asked to the patient. Patient without
consciousnes The cyanosys of upper body part. Irregular breathing with the selection
of a plenty of foamy, with the admixtures of blood, phlegm. Pulse on peripheries and
arterial pressure are not determine Tones of heart are deaf, unrhythmical. An ampoule
and transfusion system is empty. What complication arose up as a result of
hemotransfusion?
A. Air embolism of pulmonary artery
B. Tromboembolism of pulmonary artery
C. Edema of lungs
D. Heart attack of myocardium
E. Syndrome of massive hemotransfusion
ANSWER: A

A man 50 years of asymptomatic mixed goiter. At the first stage, it should appoint
A. only observation
B. thyroid hormones to suppress the function of cancer
C. propylthiouracil
D. subtotal thyroidectomy
E. radioiodine
ANSWER: A

In patient K, aged 26, noted a relapse of hyperthyroidism after medical treatment.


Your tactics.
A. Surgical treatment
B. Continue medical treatment
C. Outpatient
D. Treatment is not required
E. Is no right answer
ANSWER: A

On examination, the patient was 32 years reveal the formation of the left lobe of the
thyroid gland size 4x6 cm, painless at palpation. What additional diagnostic method to
assign?
A. Thyroid gland
B. Radiography of the neck
C. Doppler
D. Rheovasography
E. EEG
ANSWER: A

On examination, the patient was 32 years reveal the formation of the left lobe of the
thyroid gland size 4x6 cm, painless at palpation. What analysis should be performed in
order to clarify the diagnosis?
A. Thyroid hormones
B. Total blood
C. Urinalysis
D. Immunogram
E. Protein fraction
ANSWER: A

Patient J., 57, was admitted to the surgical clinic with a diagnosis: euthyroid
nodular goiter. Choose the correct treatment option.
A. resection of the thyroid gland with maximal preservation of healthy tissue and
routine histological examination
B. enucleation
C. medication
D. subtotal resection of the thyroid gland
E. excision of the node with the routine histological examination
ANSWER: A

Patient K, aged 49, was admitted to the hospital with the diagnosis: diffuse toxic
goiter. What study be done.
A. Scanning of the thyroid gland
B. EFGDS
C. Rheovasography
D. Doppler
E. There is no correct answer
ANSWER: A

Patient M, 39 years old, was admitted to the hospital with the diagnosis: diffuse
toxic goiter. What study be done.
A. Thyroid gland
B. EFGDS
C. Rheovasography
D. Doppler
E. Is no right answer
ANSWER: A

Patient M., aged 35, lives in the area of iodine deficiency, was admitted to the
clinic with complaints of enlarged thyroid gland. What is the most likely diagnosis in
a patient?
A. there is no right answer
B. acute strumitis
C. sporadic goiter
D. epidemic goiter
E. mass thyrotoxicosis
ANSWER: A

Patient O., aged 39, on the diffuse toxic goiter performed subtotal resection of the
thyroid gland. One day the patient became restless, twitching of facial muscles
appeared convulsive reduction of hands. What mated complication of surgery?
A. Removal of parathyroid glands
B. Iodine deficiency
C. Lack of thyroid tissue
D. Increased thyroid hormone
E. Increased parathyroid hormone
ANSWER: A

?Patient S., 43, in the last 5 months of worry tearfulness, irritability, fatigue,
progressive weight loss. On palpation the thyroid gland increased to III class.,
painless. Pulse 110-120 in minute, regular, blood pressure - 150/80. At USD: tissue
homogeneous, tissue hypertrophy hyper. The most likely diagnosis
A. Toxic goiter
B. Hashimoto struma
C. Acute thyroiditis
D. nodular goiter
E. goiter De Quervain
ANSWER: A

Patient S., 43, in the last 5 months of worry tearfulness, irritability, fatigue,
progressive weight loss. On palpation the thyroid gland increased to III class.,
painless. Pulse 110-120 in minute, regular, blood pressure - 150/80. What additional
diagnostic method to assign?
A. Thyroid gland
B. Radiography of the neck
C. Doppler
D. Reovazography
E. EEG
ANSWER: A

Patient S., 43, in the last 5 months of worry tearfulness, irritability, fatigue,
progressive weight loss. On palpation the thyroid gland increased to III class.,
painless. Pulse 110-120 in minute, regular, blood pressure - 150/80. What analysis
should be performed in order to clarify the diagnosis?
A. Total blood
B. Urinalysis
C. Thyroid hormones
D. Protein fraction
E. Immunogram
ANSWER: C

Patients after resection of the thyroid having convulsions, symptoms by Chvostek and
Trousseau. What a complication arose in a patient?
A. there is no right answer
B. laryngeal nerve injury
C. residual effects of hyperthyroidism
D. thyrotoxic crisis
E. hypothyroidism
ANSWER: A

Patients after resection of the thyroid having convulsions, symptoms by Chvostek and
Trousseau. What a complication is arose in a patient?
A. hypoparathyreosis
B. laryngeal nerve injury
C. residual effects of hyperthyroidism
D. thyrotoxic crisis
E. hypothyroidism
ANSWER: A

The patient diagnosed nodular nontoxic goiter. What operation is indicated the patient?
A. resection of the affected lobe with histological examination
B. conservative treatment of thyroxine
C. removal of the affected lobe, isthmus and central lymph node dissection
D. enucleation site
E. subtotal thyroidectomy
ANSWER: A

The patient diagnosed nodular nontoxic goiter. What operation is indicated the patient?
A. No right answer
B. conservative treatment of thyroxine
C. removal of the affected lobe, isthmus and central lymph node dissection
D. enucleation site
E. subtotal thyroidectomy
ANSWER: A

The patient diagnosed thyrotoxicosis. Which of the following symptoms suggests the
pathology?
A. Graefe symptom
B. Peritoneal signs
C. Trophic ulcer
D. Limb gangrene
E. No right answer
ANSWER: A

The patient diagnosed thyrotoxicosis. Which of the following symptoms suggests the
pathology?
A. Mobius symptom
B. Peritoneal signs
C. Trophic ulcer
D. Limb gangrene
E. No right answer
ANSWER: A

The patient diagnosed thyrotoxicosis. Which of the following symptoms suggests the
pathology?
A. Tremor of the upper limbs
B. Peritoneal signs
C. Trophic ulcer
D. Limb gangrene
E. No right answer
ANSWER: A
The patient diagnosed with an aberrant goiter. Refine the definition of aberrant
goiter.
A. cancer of the thyroid gland
B. metastases of thyroid cancer in the liver
C. atypical location of the thyroid gland
D. all true
E. all wrong
ANSWER: A

The patient diagnosed with nodular euthyroid goiter left lobe of the thyroid gland.
What is the optimal treatment option.
A. hemistrumectomy or resection of the lobe of the thyroid gland
B. excision of the node with the routine histological examination
C. enucleation site
D. subtotal thyroidectomy
E. conservative treatment thyroidin
ANSWER: A

The patient lives in the area of endemic iodine. What can be used to prevent goitre?
A. merkasalil
B. vaccination
C. iodine
D. improving the social life of the population
E. iodination salt
ANSWER: E

The patient made thyreoidectomy. Which of the following complications may occur in a
patient?
A. Damage to the trachea
B. Onychomycosis
C. Trophic ulcer
D. Limb gangrene
E. No right answer
ANSWER: A

The patient made thyreoidectomy. Which of the following complications may occur in a
patient?
A. Damage to the recurrent laryngeal nerve
B. Onychomycosis
C. Trophic ulcer
D. Limb gangrene
E. No right answer
ANSWER: A

The patient S., 546 years old, diagnosed with goiter II degree. What is characteristic
of the III degree of increase in thyroid gland?
A. visible swallowing
B. giant goiter
C. determined only by palpation
D. visible only when swallowing
E. determined only on ultrasound
ANSWER: A

The patient S., 61, suffering for 2 years nodular goiter, recently noted a rapid
increase in the node. On scanning image identified a "cold node". Specify the most
probable cause of this condition
A. node malignancy
B. cystic degeneration of the node
C. autoimmune
D. all true
E. hemorrhage site
ANSWER: A

The patient underwent surgery on the thyroid gland. Which of the following post-
operative complications can sports a patient?
A. No right answer
B. Ischemia of the upper extremities
C. Ischemia of lower extremities
D. Ileus
E. Gastric
ANSWER: A

The patient underwent surgery on the thyroid gland. Which of the following
postoperative complications may be sports in the patient?
A. Hypocalcemia
B. Ischemia of the upper extremities
C. Ischemia of lower extremities
D. Ileus
E. Gastric
ANSWER: A

The patient underwent surgery on the thyroid gland. Which of the following
postoperative complications may be sports in the patient?
A. Thyrotoxic crisis
B. Ischemia of the upper extremities
C. Ischemia of lower extremities
D. Ileus
E. Gastric
ANSWER: A

The patient V., age 56, diagnosed with goiter of third degree. What is characteristic
of the III degree of increase in thyroid gland?
A. visible without swallowing
B. giant goiter
C. determined only by palpation
D. visible only when swallowing
E. determined only on ultrasound
ANSWER: A
The patient was admitted with the diagnosis: diffuse toxic goiter. What operation is
indicated the patient?
A. No right answer
B. thyroidectomy
C. hemistrumectomy
D. strumectomy
E. no right answer
ANSWER: A

The patient was planned to study the thyroid gland with radioactive iodine-131. How
much time should not use iodine and thyreostatics?
A. 30 days
B. 50 days
C. 40 days
D. 10 days
E. 20 days
ANSWER: A

The patient's 43 years revealed an increase in the left lobe of the thyroid gland.
When scanning in this region found a hot site. Diagnosis.
A. nodular toxic goiter
B. diffuse non-toxic goiter
C. toxic goiter
D. multinodular toxic goiter
E. non-toxic nodular goiter
ANSWER: A

The patient's 60 years in the last 3 months has been rapidly increasing dense mass in
the left lobe of the thyroid gland. Effects of hyperthyroidism is not. At thyroid scan
revealed a cold junction. Preliminary diagnosis
A. thyroid cancer
B. lipoma of the thyroid gland
C. cyst
D. metastasis of lung cancer
E. thyroid cyst
ANSWER: A

A patient admitted to the clinic with a diagnosis: obliterating atherosclerosis of


lower extremities, chronic arterial ischemia I stage. The most informative type of
research vessels are:
A. Arteriography
B. Rheovasography
C. Thermometry
D. Palpation identification of artery pulsation
E. Venogram
ANSWER: A
A patient admitted to the clinic with a diagnosis: obliterating atherosclerosis of
lower extremities, chronic arterial ischemia I stage. The most informative type of
research vessels are:
A. No right answer
B. Rheovasography
C. Thermometry
D. Palpation identification of artery pulsation
E. Venogram
ANSWER: A

A patient admitted with complaints of pain in the lower extremities during the passage
of 300 - 400 m. What is the stage of chronic arterial insufficiency in the patient?
A. II A
B. I
C. II B
D. III
E. IV
ANSWER: A

A patient admitted with complaints of pain in the lower extremities during the passage
of 200 m. What is the stage of chronic arterial insufficiency is the patient?
A. II B
B. I
C. II A
D. III
E. IV
ANSWER: A

A patient admitted with complaints of pain from intermittent claudication, and


impotence. What is the diagnosis can be suspected in a patient?
A. Leriche syndrome
B. Obliterating endarteritis
C. Deep vein thrombosis leg
D. Iliacfemoral thrombosis
E. Coarctation of aorta
ANSWER: A

A patient admitted with complaints of pain in the muscles of the buttocks and waist.
What is the diagnosis can be suspected in a patient?
A. Leriche syndrome
B. Obliterating endarteritis
C. Deep vein thrombosis leg
D. Iliacfemoral thrombosis
E. Coarctation of aorta
ANSWER: A

The patient 53 years old admitted to hospital with suspected atherosclerosis arteries
of lower extremities. Differential diagnosis should be with:
A. Obliterative endarteritis
B. Deep vein thrombosis of lower leg
C. Iliac vein thrombosis
D. Varicosity
E. Thrombophlebitis superficial veins
ANSWER: A

The patient 53 years old admitted to hospital with suspected atherosclerosis of


arteries of lower extremities. Differential diagnosis should be with:
A. Diabetic angiopathy
B. Deep vein thrombosis of lower leg
C. Iliac vein thrombosis
D. Varicosity
E. Thrombophlebitis superficial veins
ANSWER: A

The patient admitted to hospital with complaints of pain at rest. What should be used
to block pain?
A. Epidural block
B. No-shpa
C. Trental
D. Rheopolyglucin
E. Solcosery
ANSWER: A

The patient admitted to hospital with complaints of pain at rest. What should be used
to block pain?
A. Paravertebral sympathetic blockade
B. No shpa
C. Trental
D. Rheopolyglucine
E. Solcoseryl
ANSWER: A

Patient is in hospital with a diagnosis: arterial occlusive disease of the lower


extremities, chronic arterial insufficiency II stage. To improve the rheological
properties of blood should be used:
A. Trental
B. Seduksen
C. Actovegin
D. Niacin
E. Papaverine
ANSWER: A

Patient is in hospital with a diagnosis: obliterating atherosclerosis of arteries


lower extremities, chronic arterial insufficiency II stage. To remove vasospasm should
be used:
A. Papaverine
B. Rheopoliglikin
C. Seduksen
D. Actovegin
E. Niacin
ANSWER: A

Patient to detect arterial ischemia need to be functional tests:


A. Goldflam
B. Troyanov
C. Sidorenko
D. Stepanova
E. Kokket
ANSWER: A

Patient to detect arterial ischemia need to be functional tests:


A. Panchenko
B. Troyanov
C. Sidorenko
D. Stepanova
E. Kokket
ANSWER: A

At doppler ultrasound study in the patients revealed atherosclerotic changes in


arteries. What method of diagnosis must be pursued to clarify the localization process?
A. Arteriography
B. Thermometry
C. Radiography of limbs
D. Radiography of the chest cavity
E. ECG
ANSWER: A

The patient admitted to hospital with a diagnosis: embolism the left popliteal artery.
What diseases can cause embolism?
A. Rheumatic heart disease
B. Acute pancreatitis
C. Acute cholecystitis
D. Ulcer
E. Obliterate endarteritis
ANSWER: A

The patient admitted to hospital with a diagnosis: embolism the right popliteal
artery . What is the most probable cause of embolism?
A. Cardiac aneurysm
B. Acute pancreatitis
C. Acute cholecystitis
D. Ulcer
E. Obliterate endarteritis
ANSWER: A

The patient admitted to hospital with a diagnosis: embolism the right popliteal
artery . What is the most probable cause of embolism?
A. Aneurysm of femoral artery
B. Acute pancreatitis
C. Acute cholecystitis
D. Ulcer
E. Obliterate endarteritis
ANSWER: A

The patient admitted to hospital with suspected embolism the left popliteal artery.
What is the symptom will testify in favor of embolism?
A. Severe pain in the limbs
B. Filling saphenous veins
C. Trophic ulcer leg
D. Varicose saphenous veins
E. Reticular varicose
ANSWER: A

*The patient admitted to hospital with suspected embolism right popliteal artery. What
is the symptom will testify in favor of embolism?
A. Cold extremities
B. Filling saphenous veins
C. Trophic ulcer leg
D. Varicose saphenous veins
E. Reticular varicose
ANSWER: A

The patient admitted to hospital with suspected embolism left popliteal artery. What
is the symptom will testify in favor of embolism?
A. Pale skin limbs
B. Filling saphenous veins
C. Trophic ulcer leg
D. Varicose saphenous veins
E. Reticular varicose
ANSWER: A

The patient admitted to hospital with suspected embolism, right femoral artery. What
is the symptom will testify in favor of embolism?
A. The deterioration of the picture subcutaneous veins of the affected limb
B. Filling saphenous veins
C. Trophic ulcer leg
D. Varicose saphenous veins
E. Reticular varicose
ANSWER: A

The patient admitted to hospital with suspected embolism the right femoral artery.
What is the symptom will testify in favor of embolism?
A. Changing a deep sensitivity to the affected limb
B. Filling saphenous veins
C. Trophic ulcer leg
D. Varicose saphenous veins
E. Reticular varicose
ANSWER: A

In the patient admitted to hospital with suspected embolism the right femoral artery.
What is the symptom will testify in favor of embolism?
A. The disappearance of pulsation with the level of the femoral artery
B. Filling saphenous veins
C. Trophic ulcer leg
D. Varicose saphenous veins
E. Reticular varicose
ANSWER: A

In the patient zone of ischemia grabs both legs and lower abdomen. What are level of
occlusion?
A. Bifurcation of the aorta
B. Iliac arteries
C. Femoral artery
D. Popliteal artery
E. Arteries foot
ANSWER: A

In the patient zone of ischemia captures the lower extremity to the inguinal ligament.
What are level of occlusion?
A. Iliac arteries
B. Bifurcation of the aorta
C. Femoral artery
D. Popliteal artery
E. Arteries foot
ANSWER: A

In the patient zone of ischemia captures the lower extremity to the knee joint. What
are level of occlusion?
A. Popliteal artery
B. Bifurcation of the aorta
C. Iliac arteries
D. Femoral artery
E. Arteries foot
ANSWER: A

In the patient zone of ischemia captures the whole hand. What are level of occlusion?
A. Subclavian artery
B. Axillary artery
C. Brachial artery
D. Ulnar artery
E. Arteries hand
ANSWER: A

In the patient zone of ischemia grabs his hand to the middle third of the shoulder.
What are level of occlusion?
A. Brachial artery
B. Axillary artery
C. Subclavian artery
D. Ulnar artery
E. Arteries hand
ANSWER: A

In the patient zone of ischemia grabs his hand to the elbow joint. What are level of
occlusion?
A. Ulnar artery
B. Axillary artery
C. Subclavian artery
D. Brachial artery
E. Arteries hand
ANSWER: A

A patient admitted to hospital complaining of pain in the limbs with a gradual


increase in intensity. This symptom is typical for
A. Thrombosis
B. Embolism
C. Atherosclerotic lesions
D. Varicose
E. Postthrombotic syndrome
ANSWER: A

The patient admitted to hospital with a stab wound femoral artery. Which type of
injury is this damage?
A. Open injury
B. Closed injury
C. Mixed damage
D. Combined damage
E. Fire damage
ANSWER: A

The patient admitted to hospital with chopped wound in the femoral artery. Which type
of injury is this damage?
A. Open injury
B. Closed injury
C. Mixed damage
D. Combined damage
E. Fire damage
ANSWER: A

The patient admitted to hospital with a shattered wound in the femoral artery. Which
type of injury is this damage?
A. Open injury
B. Closed injury
C. Mixed damage
D. Combined damage
E. Through damage
ANSWER: A

The patient admitted to hospital with external bleeding in the femoral artery. Which
type of injury is this damage?
A. Fresh wounds
B. Closed injury
C. Mixed damage
D. Combined damage
E. Through damage
ANSWER: A

The patient was admitted to hospital in a state of shock and damage in the femoral
artery in history. Which type of injury is this damage?
A. Fresh wounds
B. Closed injury
C. Mixed damage
D. Combined damage
E. Through damage
ANSWER: A

The patient admitted to hospital pulsating hematoma and injuries to the femoral artery
in history. Which type of injury is this damage?
A. Complicated wounds
B. Closed injury
C. Mixed damage
D. Combined damage
E. Through damage
ANSWER: A

The patient was admitted to hospital with bruising and damage to suppuration in the
femoral artery in history. Which type of injury is this damage?
A. Complicated wounds
B. Closed injury
C. Mixed damage
D. Combined damage
E. Through damage
ANSWER: A

The patient admitted to hospital with a compression of the femoral artery in history.
Which type of injury is this damage?
A. Closed injury
B. Open injury
C. Mixed damage
D. Combined damage
E. Through damage
ANSWER: A
The patient admitted to the hospital with damage to only the femoral artery. Which
type of injury is this damage?
A. Isolated damage
B. Open injury
C. Mixed damage
D. Combined damage
E. Through damage
ANSWER: A

The patient admitted to the hospital with damage to the popliteal artery in the first
day after injury. To what this type of injury is damage?
A. Recent damage
B. Open injury
C. Mixed damage
D. Combined damage
E. Through damage
ANSWER: A

The patient admitted to the hospital with damage to the popliteal artery on the second
day after injury. To what this type of injury is damage?
A. Recent damage
B. Open injury
C. Mixed damage
D. Combined damage
E. Through damage
ANSWER: A

The patient admitted to the hospital with damage to the popliteal artery on the
seventh day after injury. To what this type of injury is damage?
A. Complicated injuries
B. Open injury
C. Mixed damage
D. Combined damage
E. Through damage
ANSWER: A

The patient admitted to the hospital with damage to the popliteal artery in the second
week after injury. To what this type of injury is damage?
A. Complicated injuries
B. Open injury
C. Mixed damage
D. Combined damage
E. Through damage
ANSWER: A

A patient admitted to hospital with clinical symptoms of external bleeding in axillary


artery. To whom this type of injury is damage?
A. Open injury
B. Complicated injuries
C. Mixed damage
D. Combined damage
E. Through damage
ANSWER: A

A patient admitted to hospital with knife wounds in the superficial femoral artery.
Which research method can give the most accurate information?
A. Vascular ultrasound
B. Thermometry
C. Radiography of the lower extremity
D. Radiography of abdominal
E. Rheovasography
ANSWER: A

A patient admitted to hospital with knife wounds in the femoral artery. Which research
method can give the most accurate information?
A. There is no correct answer
B. Thermometry
C. Radiography of the lower extremity
D. Radiography of abdominal
E. Rheovasography
ANSWER: A

A patient admitted to hospital with a diagnosis: Varicose veins the left lower
extremity. At the examination revealed only varicose saphenous veins of legs without
edema. What stage of varicose veins is in a patient?
A. I
B. II A
C. II B
D. III
E. IV
ANSWER: A

A patient admitted to hospital with a diagnosis: Varicose veins left lower extremity.
At the examination revealed varicose saphenous veins with leg edema, pasty, and
induration of the skin ulcer in the lower third of the leg. What stage of varicose
veins is in a patient?
A. III
B. II A
C. I
D. II B
E. IV
ANSWER: A

A patient admitted to hospital with a diagnosis: Varicose veins left lower limb,
chronic venous insufficiency I degree. What kind of treatment the patient is the best?
A. Elastic compression
B. Surgical treatment
C. Antihypertensive therapy
D. Anticoagulant therapy
E. Anticholesterol therapy
ANSWER: A

A patient admitted to hospital with a diagnosis: Varicose veins left lower limb,
chronic venous insufficiency III degree. What kind of treatment the patient is the
best?
A. Surgical treatment
B. Elastic compression
C. Antihypertensive therapy
D. Anticoagulant therapy
E. Anticholesterol therapy
ANSWER: A

A patient admitted to hospital with a diagnosis: Varicose veins left lower limb,
chronic venous insufficiency of II degree. What drug with venotonics properties should
be appoint?
A. Detralex
B. Aspirin
C. Pentoksifilin
D. Heparin
E. Warphrin
ANSWER: A

A patient admitted to hospital with a diagnosis: Varicose veins left lower limb,
chronic venous insufficiency of II degree. Which drug with venotonics properties
should be appoint?
A. Detralex
B. Aspirin
C. Pentoksifilin
D. Heparin
E. Warphrin
ANSWER: A

A patient admitted to hospital with a diagnosis: Varicose veins left lower limb,
chronic venous insufficiency of II degree. Which drug with antiplatelet properties
should be designated?
A. Aspirin
B. Detralex
C. Pentoksifilin
D. Heparin
E. Warphrin
ANSWER: A

The patient admitted to the hospital with a diagnosis: Varicose disease of the right
lower limb, chronic venous insufficiency III degree. Which of the following methods
gives the most accurate information about the state of the venous system?
A. Vascular ultrasound
B. Radiography of the affected limb
C. Rheovasography
D. Thermometry
E. ECG
ANSWER: A

The patient admitted to the hospital with a diagnosis: Varicose disease of the right
lower limb chronic venous insufficiency III degree. Which of the following methods
gives the most accurate information about the state of the venous system?
A. Phlebography
B. Radiography of the affected limb
C. Rheovasography
D. Thermometry
E. ECG
ANSWER: A

The patient admitted to the hospital with a diagnosis: Varicose disease of the right
lower limb chronic venous insufficiency III degree. Which of the following methods
gives the most accurate information about the state of communicative veins?
A. No right answer
B. Radiography of the affected limb
C. Rheovasography
D. Thermometry
E. ECG
ANSWER: A

The patient admitted to the hospital with a diagnosis: Varicose disease of the right
lower limb chronic venous insufficiency III degree. Which of the following methods
gives the most accurate information about the state of the venous system?
A. Flebotonometry
B. Radiography of the affected limb
C. Rheovasography
D. Thermometry
E. ECG
ANSWER: A

The patient admitted to hospital with a diagnosis: Varicose disease, right lower
extremity, chronic venous insufficiency III drgree. Which diseases should be
differentiate this disease?
A. Congenital arteriovenous fistula
B. Diabetic angiopathy
C. Atherosclerosis obliterans
D. Obliterative endarteritis
E. Raynaud's disease
ANSWER: A

The patient admitted to hospital with a diagnosis: Varicose disease, right lower
extremity, chronic venous insufficiency III degree. Which diseases should be
differentiate this disease?
A. Obtained arteriovenous fistula
B. Diabetic angiopathy
C. Atherosclerosis obliterans
D. Obliterative endarteritis
E. Raynaud's disease
ANSWER: A

The patient admitted to hospital with a diagnosis: Varicose disease, right lower
extremity, chronic venous insufficiency III degree. Which diseases should be
differentiate this disease?
A. No right answer
B. Diabetic angiopathy
C. Atherosclerosis obliterans
D. Obliterative endarteritis
E. Raynaud's disease
ANSWER: A

The patient admitted to hospital with a diagnosis: Varicose disease left lower limb,
chronic venous insufficiency II drgree. What is a contraindication to surgical
treatment?
A. Obstruction of deep veins
B. Chronic bronchitis
C. Past history of pneumonia
D. Past history of angina
E. Cholelithiasis
ANSWER: A

The patient admitted to hospital with a diagnosis: Varicose disease left lower limb,
chronic venous insufficiency II degree. What is a contraindication to surgical
treatment?
A. Heart failure
B. Chronic bronchitis
C. Past history of pneumonia
D. Past history of angina
ANSWER: A

The patient complains on burning pain in the limbs and the presence of painful dense
strand of progress varicose veins in the leg. What is the most likely diagnosis in a
patient?
A. Thrombophlebitis of subcutaneous veins
B. Deep vein thrombosis
C. Obliterating atherosclerosis
D. Occlusive disease
E. Diabetic angiopathy
ANSWER: A

The patient complains of burning pain in the limbs and the presence of painful dense
strand of progress varicose veins in the leg with the transition to the lower and
middle third of the thigh. What is the most likely diagnosis in a patient?
A. Ascending thrombophlebitis of subcutaneous veins
B. Deep vein thrombosis
C. Obliterating atherosclerosis
D. Occlusive disease
E. Diabetic angiopathy
ANSWER: A

The patient complains of burning pain in the limbs and the presence of painful dense
strand of progress varicose veins on the back of the tibia. What is the most likely
diagnosis in a patient?
A. Small saphenous vein thrombophlebitis
B. Deep vein thrombosis
C. Obliterating atherosclerosis
D. Occlusive disease
E. Diabetic angiopathy
ANSWER: A

The patient complains on frequent thrombophlebitis in the zone of varicose great


saphenous vein. What is the most likely diagnosis?
A. No right answer
B. Ascending thrombophlebitis
C. Deep vein thrombosis
D. Postthrombotic syndrome
E. Erysipelas
ANSWER: A

304. The patient admitted to hospital with a diagnosis: Varicose veins left lower
extremity. Acute ascending thrombophlebitis of subcutaneous veins. Tactics?
A. Surgical treatment
B. Conservative treatment
C. Physiotherapy
D. Sanatorium treatment
E. Treatment is not required
ANSWER: A

The patient underwent surgery for acute thrombophlebitis of subcutaneous veins. Which
of the following drugs should be appoint after surgery?
A. Ciprinol
B. Vitamin C
C. Vitamin
D. Atenolol
E. Vasilip
ANSWER: A

The patient underwent surgery for acute thrombophlebitis of subcutaneous veins. Which
of the following drugs should appoint after surgery?
A. Analgetics
B. Vitamin C
C. Vitamin
D. Atenolol
E. Vasilip
ANSWER: A

The patient underwent surgery for acute thrombophlebitis of subcutaneous veins. How
long a patient should be advised to use elastic compression?
A. 3 months.
B. 1 month.
C. 1 year
D. 6 months.
E. 2 weeks
ANSWER: A

The patient underwent surgery for acute thrombophlebitis of subcutaneous veins. How
long a patient should be advised to use elastic compression?
A. There is no correct answer.
B. 1 month.
C. 1 year
D. 6 months.
E. 2 weeks
ANSWER: A

The patient complains on pain in the right leg, increases with foot movements. What
research method is most reliable for further diagnosis?
A. Vascular ultrasound
B. Radiography of the affected limb
C. Ultrasonography of the abdomen
D. Radiography of the chest cavity
E. ECG
ANSWER: A

The patient complains on pain in the right leg, increases with foot movements. What
research method is most reliable for further diagnosis?
A. Phlebography
B. Radiography of the affected limb
C. Ultrasonography of the abdomen
D. Radiography of the chest cavity
E. ECG
ANSWER: A

The patient complains on pain in the right leg hip, increases with movements in the
foot and the knee joint. What research method is most reliable for further diagnosis?
A. Vascular ultrasound
B. Radiography of the affected limb
C. Ultrasonography of the abdomen
D. Radiography of the chest cavity
E. ECG
ANSWER: A
The patient complains on pain in the right leg hip, increases with movements in the
foot and the knee joint. What research method is most reliable for further diagnosis?
A. Phlebography
B. Radiography of the affected limb
C. Ultrasonography of the abdomen
D. Radiography of the chest cavity
E. ECG
ANSWER: A

The patient complains on pain in the right leg and thigh, reinforced at moving foot
and the knee joint. There is hypercyanotic color skin limbs. What research method is
most reliable for further diagnosis?
A. Phlebography
B. Radiography of the affected limb
C. Ultrasonography of the abdomen
D. Radiography of the chest cavity
E. ECG
ANSWER: A

The patient complains on pain in the right leg and thigh, reinforced at moving foot
and the knee joint. There is hypercyanotic color skin limbs. What research method is
most reliable for further diagnosis?
A. Vascular ultrasound
B. Radiography of the affected limb
C. Ultrasonography of the abdomen
D. Radiography of the chest cavity
E. ECG
ANSWER: A

The patient complains on pain in the right leg and thigh, reinforced at moving foot
and the knee joint. There is hypercyanotic color skin limbs. Swelling of limbs
throughout and on the buttocks. What research method is most reliable for further
diagnosis?
A. Vascular ultrasound
B. Radiography of the affected limb
C. Ultrasonography of the abdomen
D. Radiography of the chest cavity
E. ECG
ANSWER: A

The patient admitted to the hospital with a diagnosis deep vein thrombosis left lower
extremity. The patient suddenly began to worry shortness of breath, retrosternal pain.
What complication can be suspected in a patient?
A. Pulmonary embolism
B. Superficial thrombophlebitis
C. Ulcer
D. Pharyngitis
E. Pleurisy
ANSWER: A
The patient admitted to the hospital with a diagnosis deep vein thrombosis left lower
extremity. The patient suddenly began to worry shortness of breath, pain with
localized behind the breastbone, a feeling of fear. What complication can be suspected
in a patient?
A. Pulmonary embolism
B. Superficial thrombophlebitis
C. Ulcer
D. Pharyngitis
E. Pleurisy
ANSWER: A

The patient admitted to the hospital with a diagnosis deep vein thrombosis left lower
extremity. The patient suddenly began to worry shortness of breath, retrosternal pain.
What method diagnosis should be used to refine the diagnosis?
A. Radiography of the chest
B. Reovazogrfiya
C. Densitometry
D. Ultrasound of internal organs
E. Ultrasound vascular limb
ANSWER: A

In the patient suspected pulmonary embolism. Which of the following methods


investigation is required to clarify the diagnosis?
A. Study of blood coagulation
B. Complete blood
C. Urinalysis
D. Determine the level of uric acid
E. Confirmation is not required
ANSWER: A

In the patient suspected pulmonary embolism. Which of the following methods


investigation is required to clarify the diagnosis?
A. ECG
B. Complete blood
C. Urinalysis
D. Determine the level of uric acid
E. Confirmation is not required
ANSWER: A

In the patient suspected pulmonary embolism. Which of the following methods


investigation is required to clarify the diagnosis?
A. Angiopulmonography
B. Complete blood
C. Urinalysis
D. Determine the level of uric acid
E. Confirmation is not required
ANSWER: A
In the patient suspected pulmonary embolism. Which of the following methods
investigation is required to clarify the diagnosis?
A. Lung scintigraphy
B. Complete blood
C. Urinalysis
D. Determine the level of uric acid
E. Confirmation is not required
ANSWER: A

In the patient suspected pulmonary embolism. Done radiography of the chest cavity.
Which of the following symptoms are indicate pulmonary embolism?
A. The rise of the dome diaphragm
B. Pulmonary fibrosis root
C. Availability Calcinates
D. Increased vascular pattern
E. No right answer
ANSWER: A

In the patient suspected pulmonary embolism. Done radiography of the chest cavity.
Which of the following symptoms are indicate pulmonary embolism?
A. Pleural effusion in the sinuses
B. Pulmonary fibrosis root
C. Availability Calcinates
D. Increased vascular pattern
E. No right answer
ANSWER: A

In the patient suspected pulmonary embolism. Done radiography of the chest cavity.
Which of the following symptoms are indicate pulmonary embolism?
A. Breaking vessels near the root of the lungs
B. Pulmonary fibrosis root
C. Availability Calcinates
D. Increased vascular pattern
E. No right answer
ANSWER: A

In the patient suspected pulmonary embolism. Done radiography of the chest cavity.
Which of the following symptoms are indicate pulmonary embolism?
A. Hyperperfusion of the contralateral lung
B. Pulmonary fibrosis root
C. Availability Calcinates
D. Increased vascular pattern
E. No right answer
ANSWER: A

In the patient suspected pulmonary embolism. Echocardiography. Which of the following


symptoms are indicate pulmonary embolism?
A. Advanced, hypokinetic right ventricle
B. Normal right ventricle
C. Spasm of the proximal pulmonary arteries
D. Relationships right ventricle / left ventricle is not broken
E. No right answer
ANSWER: A

In the patient suspected pulmonary embolism. Echocardiography. Which of the following


symptoms are indicate pulmonary embolism?
A. Increased ratio of right ventricle / left ventricle
B. Normal right ventricle
C. Spasm of the proximal pulmonary arteries
D. Relationships right ventricle / left ventricle is not broken
E. No right answer
ANSWER: A

In the patient suspected pulmonary embolism. Echocardiography. Which of the following


symptoms are indicate pulmonary embolism?
A. Increasing the speed of blood tricuspid regurgitation> 3.7 m / s
B. Normal right ventricle
C. Spasm of the proximal pulmonary arteries
D. Relationships right ventricle / left ventricle is not broken
E. No right answer
ANSWER: A

In the patient suspected pulmonary embolism. Echocardiography. Which of the following


symptoms are indicate pulmonary embolism?
A. Violation of blood flow from the right ventricle
B. Normal right ventricle
C. Spasm of the proximal pulmonary arteries
D. Relationships right ventricle / left ventricle is not broken
E. No right answer
ANSWER: A

Patients with suspected pulmonary embolism made angiopulmonography. Which of the


following symptoms is direct?
A. Vascular filling defects
B. Asymmetry of vascular contrast material filling
C. Slow progress or stasis of contrast
D. Increased pulmonary artery trunk and its major branches
E. No right answer
ANSWER: A

A patient complains of a pain in a right iliac area. At first pain appeared in


epigastrium, then relocated in a right iliac area. During examination muscular tension
of anterior abdominal wall and symptoms of peritoneal irritation are detected in a
right iliac area. Leucocytes-8?109. General urine analysis within norm. Your diagnosis.
A. Acute appendicitis
B. Right-side renal colic
C. Appendicular infiltrate
D. Meckel's diverticulum
E. Perforative ulcer
ANSWER: A

A patient was operated 9 days ago for destructive appendicitis with pelvic position.
As a complication – the abscess of Duglas space was developed. At digital rectal
examination the infiltrate was detected with softening in center. Tactic of surgeon?
A. Punction of the abscess with draining
B. Relaparotomy, draining of the abscess
C. Presacral antibiotic-novocaine blockade
D. Warm small enemas, suppositories with antibiotics
E. Electrophoresis with antibiotics on abdomen
ANSWER: A

A patient complains of a pain in the lower abdomen, mostly on the right side,
dizziness. The mentioned signs appeared suddenly, at night. Last menstruation 2 weeks
ago. During examination the skin is pale, pulse 92 /min., t -36°C, AP 100/60 mm Hg.
The abdomen is slightly tense, painful in lower areas. The signs of peritoneal
irritation are insignificant. Нв-98 g/l. Your diagnosis?
A. Ovarian apoplexy
B. Renal colic
C. Acute appendicitis
D. Intestinal obstruction
E. Bleeding gastric ulcer
ANSWER: A

In patient during the operation for acute destructive appendicitis the appendicular
infiltrate was diagnosed. What the further tactic of surgeon?
A. Appendectomy not performed, draining the abdominal cavity
B. Appendectomy not performed, to suit the wound
C. Performed appendectomy
D. Performed a cystostomy
E. To remove the infiltrate
ANSWER: A

Among the ways of penetration of infection in appendix in acute appendicitis the most
frequent is enterogenic. What microorganisms are revealed more frequent?
A. Colli
B. Staphylococci
C. Streptococci
D. Bacteroids
E. Koch's mycobacterii
ANSWER: A

A diarrhea is not obligatory, but possible in acute appendicitis. In what cases


diarrhea confirms the inflammation of appendix?
A. In pelvic appendicitis
B. On the first day of the disease
C. In fever
D. In retrocaecal appendicitis
E. In retroperitoneal appendicitis
ANSWER: A

The pregnant woman (22 weeks) is delivered in urgent clinic with complaints of
abdominal pain. The onset was acute, a day ago. Pain appeared in epigastrium, then in
the right half of abdomen. There was a single vomiting. The abdomen painful above the
right iliac area. Signs of peritoneal irritation are insignificant. Leucocytosis –
21?109. What is the most probable diagnosis?
A. Acute appendicitis
B. Toxemia of pregnancy
C. Threatened abortion
D. Acute pancreatitis
E. Mesenteric thrombosis
ANSWER: A

35 y.o. woman was admitted to thoracic surgery department with fever up to 400C, onset
of pain in the side caused by deep breathing, cough with considerable quantity of
purulent sputum and blood with bad smell. What is the most likely diagnosis?
A. Complication of liver echinococcosis
B. Bronchiectatic disease
C. Actinomycosis of lungs
D. Pulmonary tuberculosis
E. Abscess of the lung
ANSWER: E

The patient aged 55, complains of acute pain in the epigastric region, frequent
vomiting, without any relief, a general weakness. On examination: pale skin, tongue is
dry. Pulse rate A 110/min. AP - 80/40 mm Hg. The abdomen is hollowed, soft by
palpation, moderately tense in the epigastrium, signs of peritoneal irritation are
absent. On X-ray - Kloibers cups in the left half. Has been ill for a day. What kind
of pathology should be suspected?
A. Acute cholecystitis
B. Acute pancreatitis
C. Perforated duodenal ulcer
D. Large intestine obstruction
E. High intestinal obstruction
ANSWER: E

In patient 18 years old, appeared cramping abdominal pain, vomiting, constipation and
gas delay. On abdominal X-ray revealed Kloibers cups. What kind of acute abdominal
disease can we speak about?
A. Helminthic invasion
B. Enterocolitis
C. Bad preparation of the patient for examination
D. Flatulence
E. Intestinal obstruction
ANSWER: E
Patient 55 years complains of severe cramping abdominal pain, nausea, vomiting,
constipation and gas delay. Has been ill for 6 hours. Had been operated for traumatic
injury of spleen. On examination: pulse 84/min. The tongue is moderately wet. The
abdomen distended with asymmetry due to the increase in the left half. Defined the
"splashing sound". Peristaltic sounds periodically amplified. On X-ray of the
abdominal cavity - Kloibers cups. What is the primary diagnosis?
A. Sigmoid volvulus
B. Ulcerative colitis
C. Thrombosis of mesenteric vessels
D. Obstructive ileus (tumor genesis)
E. Acute adhesive intestinal obstruction
ANSWER: E

On abdominal X-ray of the patient is determined several sites of enlightenment of


hemispheric shape with the clear horizontal level. What causes such X-ray picture?
A. Acute pancreatitis
B. Colon cancer
C. Flatulence
D. Perforated ulcer
E. Intestinal obstruction
ANSWER: E

The patient 74 years old, complains of the severe cramping pain in the left side of
the abdomen, nausea, constipation and gas delay. Has been ill for 8 hours. During the
last 20 years is suffering from constipations. Refuses admixtures of mucus and blood
in the stool. Pulse - 82/min. The tongue is moderately wet. Abdomen greatly distended,
asymmetrical due to the increase of the left half. Determined the splashing sound. On
X-ray of the abdominal cavity Kloibers cups in the left abdomen with a dilatated colon
loop over them. On rectal examination the sign Obukhovsky hospital. What is the
primary diagnosis?
A. Ulcerative colitis
B. Thrombosis of mesenteric vessels
C. Adhesive ileus
D. Obstructive ileus (tumor genesis)
E. Sigmoid volvulus
ANSWER: E

In the patient A., 79 years old, was diagnosed acute intestinal obstruction and
concomitant cardiac pathology. Pulse 103/min AP - 95/60 mm. Hg. After stabilization of
hemodynamic parameters the patient was taken for operation, where. revealed a volvulus
of sigmoid colon. After detorsion the bowel is dark, peristalsis and vascular
pulsation is absent. What is the optimal operation should be done in this case?
A. Colectomy
B. Drainage of the abdominal cavity
C. Transanal intubation of the colon
D. Resection of the sigmoid colon with primary anastomosis
E. Resection of the sigmoid colon with colostomy
ANSWER: E
Patient M, 66 years old, arrived from the district hospital at 10 o'clock with
complaints of severe abdominal pain, nausea, constipation and gas delay. The disease
started suddenly, at 4 o'clock in the morning, with strong cramping abdominal pain.
After one hour of the onset the patient entered to the district hospital, and after 5
hours transferred to the regional surgical department. After properly preparing the
patient underwent the operation the acute intestinal obstruction. After wide
laparotomy, the nodulus was found out. Intestinal loops in knot of a dark-brown color,
peristalsis and vascular pulsation is absent. What kind of acute intestinal
obstruction you deal with?
A. Paralytic
B. Mixed
C. Spastic
D. Obstructive
E. Strangulated
ANSWER: E

A 33 y.o. male patient was admi-tted to a hospital. A patient is pale, at an attempt


to stand up he complains of strong dizziness. There was vomiting like coffee-grounds
approximately hour ago. BP- 90/60 mm Hg., pulse- 120 b/min. In anamnesis, a patient
has suffered from ulcer of the stomach, painless form during 4 years. An ulcer was
exposed at gastrofi-beroscopy. Your diagnosis:
A. Ulcer of duodenum, complicated with bleeding
B. Erosive gastritis
C. Acute pleurisy
D. Acute myocardial infarction, abdominal form
E. Ulcer of stomach, complicated with bleeding
ANSWER: E

In the university hospital entered the patient with a diagnosis of intestinal


obstruction. From anamnesis: weight loss (10 kg for 3 months), weakness and
occasionally blood in the stool. What is the cause of obstruction?
A. Coprostasis
B. Ulcerative colitis
C. Helminthic invasion
D. Adhesive disease of peritoneum
E. Colon cancer
ANSWER: E

Patient 43 years old, who had been operated for acute cholecystitis, entered the
clinic with complaints of intense cramping abdominal pain, repeated vomiting and
absence of stool. AP - 90/60 mm Hg. The abdomen is moderately distended, soft,
slightly painful. SklyarovТs sign is positive Shchetkin - Blumberg sign is slightly
positive. What is the primary diagnosis?
A. Perforation of colon intestine
B. Acute pancreatitis
C. Mesenteric thrombosis
D. Perforation of malignant tumor of the stomach
E. Acute adhesive intestinal obstruction
ANSWER: E
The patient 18 years old, was hospitalized in a university hospital in urgent way with
the signs of acute intestinal obstruction. From anamnesis 1 year ago she was operated
for acute destructive appendicitis. Recently bothered abdominal distension and pain.
Which of the following etiological factors resulted in intestinal obstruction?
A. Helminthic invasion
B. Food disorders
C. Diverticulum of ileum
D. Dolihosigma
E. Adhesions in the abdominal cavity
ANSWER: E

Female 50 years old, who had been operated for uterine fibroma (hysterectomy), entered
with complaints of abdominal pain, vomiting. On the X-ray of abdomen revealed the
signs of small bowel obstruction. What is the cause of obstruction?
A. Diverticulitis
B. Cancer of the small intestine
C. Carcinoma of the colon
D. Strangulated inguinal hernia
E. Adhesive disease
ANSWER: E

The patient entered with complaints of cramping abdominal pain, vomiting, delay of
stool and gases. The patient's condition is severe. Pulse - 105 per 1 min. The tongue
is dry. Abdomen distended, asymmetrical. By percussion - tympanitis, visible by eye
peristalsis. Positive Valyas sign. Shchetkin-Blumberg sign is slightly positive. What
is your diagnosis?
A. Acute appendicitis
B. Acute cholecystitis
C. Mesenteric thrombosis
D. Acute pancreatitis
E. Intestinal obstruction
ANSWER: E

Patient M., aged 25, was hospitalized with complaints of nausea, vomiting, cramping
pain in the right half of the abdomen. In anamnesis appendectomy. Pulse 96 per minute,
AP 110/70 mm Hg. The abdomen is distended, asymmetrical due to protrusion of the right
half, where determined a sign of intestinal splash. The peristalsis is strengthened.
Signs of peritoneal irritation are absent. Rectal examination: sphincter tone is
lowered, ampoule dilated. What is the primary diagnosis?
A. Food poisoning
B. Acute adnexitis
C. Acute pancreatitis
D. Acute obstructive ileus
E. Acute adhesive intestinal obstruction
ANSWER: E

In the surgical department entered the patient with complaints of sudden cramping pain
in the abdomen, its asymmetry, which arose suddenly after a food abuse, delay of stool
and gases. By palpation the abdomen is soft, without signs of peritoneal irritation.
In the abdominal cavity in the left half detected the movable, painful, tumor
formation, by elastic consistency. What is the primary diagnosis?
A. Rectal cancer
B. Stomach tumor
C. Acute pancreatitis
D. Obstipation syndrome
E. Sigmoid volvulus
ANSWER: E

A 42 y.o. man died in a road accident after the haemorrhage on the site, due to acute
hemorrhagic anaemia. What minimum percentage volume of the total blood loss would
result in death at acute haemorrhage?
A. 6-9%
B. 10-14%
C. 15-20%
D. 35-50%
E. 25-30%
ANSWER: E

A 36 y.o. patient is diagnosed with right sided pneumothorax. What method of treatment
is indicated to the patient?
A. Antiinflammation therapy
B. Symptomatic therapy
C. Pleural puncture
D. Thoracotomy
E. Surgical drainage of the pleural cavity
ANSWER: E

In the department of surgery entered the patient who complained of cramping abdominal
pain, which appeared suddenly 3 hours ago, repeated vomiting. 6 hours ago ate a pound
of walnuts. On examination: abdomen is distended symmetrically, on auscultation -
strengthening the peristaltic tones. On X-ray of the abdominal cavity multiple
horizontal air-fluid levels (the width of the horizontal level of the fluid is greater
than the height of the gas). What is your primary diagnosis?
A. Chronic paralytic intestinal obstruction
B. Acute large intestine paralytic ileus
C. Acute paralytic intestinal obstruction
D. Acute obstructive large intestine obstruction
E. Acute obstructive intestinal obstruction
ANSWER: E

For the differential diagnosis of acute appendicitis with a perforated duodenal ulcer
you apply?
A. X-ray of the stomach with barium sulfate
B. Laparocentesis, colonoscopy
C. Irrigoscopy, irrigography
D. X-ray of gastrointestinal tract, irrigoscopy
E. The examination abdominal radiography
ANSWER: E

On the 3rd day after appendectomy you made the diagnosis - the diffuse peritonitis.
What is your tactic?
A. To perform laparoscopy
B. To perform angiographic examination
C. To prescribe spasmolytics, analgetics
D. To enhance antibacterial therapy
E. To perform laparotomy
ANSWER: E

In the patient, who was operated for gangrenous appendicitis, on the 2nd day after the
operation developed paresis of the intestine, jaundice, appeared fever and pain in the
right side of abdomen. No signs of peritoneal irritation. What kind of complication of
the disease developed?
A. Subhepatic abscess
B. Intraintestinal abscess
C. Peritonitis
D. Subphrenic abscess
E. Pylephlebitis
ANSWER: E

You have diagnosed the appendicular infiltrate in the patient. What is the most
expedient conservative treatment in first three days?
A. Parenteral use of proteolytic enzymes
B. The warm on right iliac region
C. U.H.F.-therapy
D. Analgesics
E. Antibiotics
ANSWER: E

During the operation for acute appendicitis, you revealed a dense appendicular
infiltrate. What is your tactic?
A. All the answers wrong
B. Perform disclosure and drainage of the infiltrate
C. To mobilize the appendix from inflammatory infiltrate and perform appendectomy
D. Under general anesthesia perform a median laparotomy and appendectomy
E. To close the wound of the abdominal wall and prescribe antibiotics and local cold
ANSWER: E

In the patient on the 7th day after the operation for gangrenous appendicitis, was
developed a fever, pain in the rectum, tenesmi, frequent and painful urination. During
rectal examination revealed the infiltrate in pelvis. During 3 days after the
treatment, which included warm enemas and antibiotics, the patient's condition had not
improved. Repeat examination revealed a softening of rectal infiltration. Temperature
had hectic nature. What is the treatment?
A. Laparoscopic drainage of abscess
B. Laparotomy, opening abscess
C. Drainage of pelvic abscess through retroperitoneal access
D. Antibiotics, physiotherapy, hyperbaric oxygen therapy
E. Drainage of pelvic abscess through the rectum
ANSWER: E

You diagnosed and prescribed conservative treatment of appendicular infiltrate. What


are the signs of abscessing of appendicular infiltrate?
A. Increase of leukocytosis with a shift to the left, the increase of ESR
B. Signs of peritoneal irritation
C. Restoring or increase of pain
D. Maintaining a high hectic temperature
E. All answers correct
ANSWER: E

In the patient, aged 18, on the 7th day after the appendectomy developed a pelvic
abscess. What will you do?
A. Drain the abscess by a right-side extraperitoneal access
B. Drain the abscess through the right iliac region
C. Drain the abscess by low-median access and drainage of small pelvis
D. Massive antibiotic therapy
E. Drain the abscess through the front wall of the rectum
ANSWER: E

In the patient, aged 40, on the 2nd day after the appendectomy for gangrenous
appendicitis, developed intestinal paresis, fever, appeared pain in the right half of
abdomen, enlarged liver and jaundice, signs of peritoneal irritation. What
complications of acute appendicitis you can think about?
A. Subhepatic abscess
B. Intraintestinal abscess
C. Subphrenic abscess
D. Peritonitis
E. Pylephlebitis
ANSWER: E

In the patient after appendectomy on the 5th day appeared the hectic fever,
leucocytosis. In the depths of the pelvis appeared moderate pain, disuric phenomenon,
tenesmi. What additional method of investigation you will begin with to diagnose the
complication?
A. Repeat irrigoscopy
B. Rectoromanoscopy
C. Repeat tests of blood and urine
D. Cystochromoscopy
E. A digital rectal examination
ANSWER: E

You suspect the abscess of Douglas' pouch in the patient. What investigations should
be carried out for its diagnosis?
A. Laparocentesis
B. Laparoscopy
C. Colonoscopy
D. Rectoromanoscopy
E. A digital rectal examination
ANSWER: E

You suspected acute appendicitis in patient. Diagnostic program includes:


A. Laboratory tests
B. Rectal examination in men and vaginal additional examination in women
C. To exclude all diseases that can simulate acute abdominal pathology
D. Thorough anamnesis
E. All answers correct
ANSWER: E

The patient, is pregnant for 32 weeks, has stayed in the surgical department for 18
hours. During dynamic observation it is impossible to rule out acute appendicitis.
What is your tactic?
A. To perform laparoscopy and to operate if the diagnosis confirms
B. With gynecologist to perform abortion, and then appendectomy
C. Carry out abdominal ultrasonography
D. Continue to observe the patient
E. Patient need to undergo the operation
ANSWER: E

On examination of the patient, the therapeutist suspected acute appendicitis, but is


not confident in the diagnosis. The disease began six hours ago. What to do?
A. Dynamic observation of patient in outpatient department
B. As for the short time since the beginning of the disease, to recommend conservative
treatment. The calm, local cold, antibiotics
C. Urgently send the patient to the clinic for further laboratory examination
D. To recommend the surgeon consultation
E. Urgent hospitalization of the patient in the surgical department
ANSWER: E

Patient K. has the diagnosis: diffuse purulent peritonitis of appendicular origin. For
the patient is indicated:
A. Total parenteral nutrition for 1-2 days after operation
B. Antibacterial therapy
C. Correction of fluid and electrolyte disorders
D. Appendectomy and sanation of the abdominal cavity
E. All mentioned is correct
ANSWER: E

The patient was diagnosed the diffuse peritonitis of appendicular origin. In this case
used:
A. Drainage of the abdominal cavity
B. Abdominal lavage
C. Appendectomy
D. Median laparotomy
E. All mentioned
ANSWER: E
You have to operate the patient with a typical picture of acute phlegmonous
appendicitis. What access for appendectomy is the most suitable in this case?
A. Transverse laparotomy above navel
B. Right-side transrectal incision
C. Right-side pararectal access
D. Low-median laparotomy
E. Volkovych-Diakonov's access
ANSWER: E

The patient's T, at the ambulatory reception is suspected for acute appendicitis. What
is advisable to do?
A. Observed the patient ambulatory and hospitalized in case of deterioration
B. To prescribe the anti-inflammatory therapy and repeat examination on the next day
C. The control of the dynamics of body temperature and leucocytosis for 12-24 hours
D. To prescribe spasmolytics and repeat examination of the patient in 4-6 hours
E. Immediately hospitalization of the patient
ANSWER: E

The patient, 76 years old, with myocardial infarction, is suspected for acute
destructive appendicitis. What is your tactic?
A. All the answers are wrong
B. Operation if the diagnosis confirms by laparoscopy
C. Prescription of massive doses of broad-spectrum antibiotics and if this therapy is
ineffective A operation
D. Monitoring and operation if the peritonitis develops
E. Urgent operation
ANSWER: E

The patient operated on for acute appendicitis at day 4 after surgery developed
peritonitis. Your tactics?
A. Laparoscopy
B. Laparocentesis
C. Assign analgesics
D. Antibiotic
E. Operation
ANSWER: E

Patient after surgery for diffuse peritonitis antibiotics. What is the way the drug is
best?
A. intraperitoneal
B. intramuscular
C. subcutaneous
D. oral
E. intravenous
ANSWER: E

The patient during surgery revealed fibropurulent peritonitis. What activities should
be undertaken?
A. drainage of the abdominal cavity.
B. decompression of the intestine.
C. reorganization of the abdominal cavity.
D. removal or delimitation source of peritonitis.
E. all answers are correct
ANSWER: E

Patient transported to hospital with a diagnosis: sub-diaphragmatic abscess. What


tactics of treatment is most correct?
A. all of the above is true
B. laparotomy, dissection, and backfilling the cavity
C. extraperitoneal incision and drainage
D. conservative treatment
E. thick needle puncture of an abscess under ultrasound control
ANSWER: E

Patient transported to hospital with suspected sub-diaphragmatic abscess. What method


of research can clarify the diagnosis?
A. irrigoscopy
B. contrast fluoroscopy GIT
C. doppler
D. ECG
E. computer tomography
ANSWER: E

Patient transported to hospital with a diagnosis: subdiaphragmatic abscess. What


access to the abscess should be used?
A. Fedorovs laparotomy
B. two-stage transpleural approach
C. lumbotomy
D. thoracolaparotomy
E. Extrapleural extraperitoneal method
ANSWER: E

Patients 2 hours ago was hit in the stomach. Fetched in emergency hospital in serious
condition. When radiography diagnosed gap cavity body. Your tactics.
A. antibiotic
B. laparoscopy
C. laparocentesis
D. cold on the stomach
E. operation
ANSWER: E

Patients 2 hours ago was hit in the stomach. Fetched in emergency hospital in serious
condition with a suspected rupture cavity body. What method of research will clarify
the diagnosis?
A. irrigoscopy
B. contrast fluoroscopy GIT
C. doppler
D. ECG
E. laparocentesis
ANSWER: E

Patients 2 hours ago fell from the second floor. His condition is grave. There are
signs of diffuse peritonitis. What investigation method can eliminate the damage of a
hollow organ?
A. thermometry
B. doppler
C. rheovasography
D. ECG
E. radiography of the abdominal cavity
ANSWER: E

Patients 2 hours ago fell from the second floor. His condition is grave. There are
signs of diffuse peritonitis. Completed radiography abdomen. Which of the radiological
signs confirm the diagnosis of rupture cavity body?
A. infiltration in the abdominal cavity
B. free fluid in the abdominal cavity
C. Kloibers bowls
D. highstand dome of the diaphragm
E. free gas in the abdominal cavity
ANSWER: E

The patient admitted with blunt abdominal trauma. Examination - positive symptoms of
irritation of the peritoneum. Suspected damage to internal organs. What method of
diagnosis should be used?
A. EEG
B. ECG
C. rheovasography
D. angiography
E. laparocentesis
ANSWER: E

Patient received an abdominal trauma 3 hours ago. Fetched in emergency clinic


complaining of acute pain in stomach. Positive symptoms irritation of the peritoneum.
When the review X-rye revealed free gas under the dome of the diaphragm. Installed
diagnosis: breaking the body cavity. Treatment.
A. antibiotic
B. laparoscopy
C. laparocentesis
D. cold on the stomach
E. operation
ANSWER: E

The patient admitted in emergency hospital in serious condition. Pulse 112 per minute.,
AP - 110 \ 60, T - 35,1 C. Installed diagnosis diffuse peritonitis. Which research
method should be used to clarify the source of peritonitis?
A. Thermometry
B. Doppler
C. Rheovasography
D. ECG
E. Ultrasound
ANSWER: E

The patient admitted in emergency hospital in serious condition. Pulse 112 per minute.,
AP - 110 \ 60., T-35, 1 C. Installed diagnosis diffuse peritonitis. What treatment is
indicated the patient?
A. puncture
B. antibiotic
C. laparoscopy
D. laparocentesis
E. surgery
ANSWER: E

The patient complains of severe abdominal pain, which occurred suddenly 6 hourse ago.
Installed diagnosis of peritonitis. Which symptom is most typical for this diagnosis?
A.
ANSWER: A

tension of the muscles of the abdominal wall


A. leukocytosis
B. tachycardia
C. temperature rise
D. irritation of the peritoneum
ANSWER: D

Patient N., aged 45, was admitted to the surgical department with the diagnosis:
biliary peritonitis. Treatment.
A. Fluid management
B. Treatment is not required
C. Outpatient care
D. Conservative treatment
E. Operative treatment
ANSWER: E

Patient D., aged 39, was admitted to the surgical department with the diagnosis:
diffuse peritonitis. What does determines the severity of peritonitis?
A. All true
B. Body mass index
C. The growth of the patient
D. Patient weight
E. No right answer
ANSWER: E

Patient S., aged 35, was admitted to the surgical department with the diagnosis:
appendicular peritonitis. Treatment.
A. Fluid management
B. Treatment is not required
C. Outpatient care
D. Conservative treatment
E. Operative treatment
ANSWER: E

The patient was diagnosed during surgery iatrogenic injury extrarenal bile ducts. What
operation is indicated?
A. imposition biliodigistiv anastomosis
B. closure of the duct on shaped drainage
C. suturing of the duct on the T-shaped drainage
D. closure of the injury duct separate atraumatic suture needle
E. any of the above
ANSWER: E

Patient S., 41, was admitted to hospital with a diagnosis: obstructive jaundice. To
diagnose the causes of jaundice is the subject of a more reliable:
A. direct hepatosplenography
B. liver scintigraphy
C. intravenous cholecystocholangiography
D. oral cholecystography
E. retrograde cholangiography
ANSWER: E

For patients with suspected calculous cholecystitis for diagnosing of stones in the
gallbladder is indicated:
A. vulvocentesis
B. laparocentesis
C. doppler
D. angiography
E. intravenous cholecystocholangiography, ERCP, sonography
ANSWER: E

Patients with suspected calculous cholecystitis for diagnosing stones in the


gallbladder indicate:
A. vulvocentesis
B. laparocentesis
C. doppler
D. angiography
E. endoscopic retrograde cholangiography
ANSWER: E

Patients with suspected calculous cholecystitis for diagnosing stones in the


gallbladder indicate:
A. vulvotsentez
B. celiocentesis
C. Doppler
D. angiography
E. abdominal X-ray
ANSWER: E

The patient with suspected narrowing of the distal common bile duct for diagnosis you
must:
A. definition level of blood flow in truncus coeliacus
B. certain fraction of bilirubin
C. laparocentesis
D. echo kardioskopiyu
E. there is no right answer
ANSWER: E

Patients after cholecystectomy in the immediate postoperative period gradually


increases jaundice, these operating cholangiography not indicate the pathology of the
bile ducts. The most likely cause of jaundice
A. all of the above
B. hemolytic jaundice
C. common bile duct stone
D. serum hepatitis
E. operating choledochal injury (ligation)
ANSWER: E

Patient V., aged 45, was admitted to hospital with a diagnosis: chronic cholecystitis.
What is a reliable radiological signs of chronic cholecystitis?
A. shadow of suspicion on concretions in the gall bladder at a reduced by 1/3 after
the administration of choleretic breakfast
B. shadow of the gall bladder
C. sharply increased, not reduced by giving choleretic breakfast
D. weak fluoroscopic shadow of the gall bladder
E. "disconnected" gallbladder
ANSWER: E

Patient S., aged 56, was admitted to hospital with a diagnosis: acute catarrhal
cholecystitis. What symptoms are not typical for this diagnosis?
A. Mussi symptom
B. lack of muscle tension in the right hypochondrium
C. symptom Kera
D. nausea
E. all true
ANSWER: E

The patient admitted to hospital with suspected choledocolithiasis. What method of


diagnosis is not used in this case?
A. transhepatic holegraphy
B. ERPHG
C. intravenous cholecystocholangiography
D. ultrasound
E. endoscopy
ANSWER: E
The patient admitted to hospital with suspected choledocolithiasis. What method of
diagnosis is not used in this case?
A. transhepatic holegraphy
B. ERPHG
C. intravenous cholecystocholangiography
D. ultrasound
E. all answer are right
ANSWER: E

Patient A., aged 45, was admitted to hospital with jaundice in the background
cholelithias. The patient is shown:
A. plasmapheresis
B. catheterization of the celiac artery
C. conservative treatment
D. emergency surgery
E. there is no right answer
ANSWER: E

Patient A., aged 45, was admitted to hospital with jaundice in the background
cholelithias. The patient is indicate:
A. plasmapheresis
B. catheterization of the celiac artery
C. conservative treatment
D. emergency surgery
E. emergency operation after the preoperative
ANSWER: E

Patient M., aged 56, was admitted to the hospital with a diagnosis: acute gangrenous
cholecystitis. The patient is shown:
A. operation with no effect of conservative treatment
B. decision depends on the age of the patient
C. delayed operation
D. conservative treatment
E. there is no right answer
ANSWER: E

Patient M., aged 56, was admitted to the hospital with a diagnosis: acute gangrenous
cholecystitis. The patient is shown:
A. operation with no effect of conservative treatment
B. decision depends on the age of the patient
C. delayed operation
D. conservative treatment
E. Emergency operation
ANSWER: E

Patient S., 42 years old, entered the clinic with a diagnosis: acute calculary
cholecystitis. What analgesics is contraindicated?
A. spazmalgon, baralgin
B. spazgan
C. no-spa
D. omnopon
E. no right answer
ANSWER: E

Patient S., 42 years old, entered the clinic with a diagnosis: acute calculary
cholecystitis. What analgesics is contraindicated?
A. spazmalgon, baralgin
B. spazgan
C. no-spa
D. omnopon
E. morphine hydrochloride
ANSWER: E

The patient K., 50 years old, six months after cholecystectomy performed at the
calculouse chronic cholecystitis, again began to appear in the right hypochondrium
pain, occasionally accompanied by yellowing of the sclera. Which of the following
methods is most informative for diagnosis in this case?
A. computed tomography
B. scanning of the liver,
C. oral cholecystography,
D. infusion holegraphy,
E. USD
ANSWER: E

The patient K., 50 years old, six months after cholecystectomy performed at the
calculouse chronic cholecystitis, again began to appear in the right hypochondrium
pain, occasionally accompanied by yellowing of the sclera. Ultrasound examination of
the abdominal cavity obvious pathology of the extrahepatic bile duct could not be
detected. Which of the following methods is most informative for diagnosis in this
case?
A. computed tomography
B. scanning of the liver,
C. oral cholecystography,
D. infusion holegraphy,
E. endoscopic retrograde cholangiopancreatography
ANSWER: E

Patient T., age 37, was admitted to hospital with a diagnosis: acute pancreatitis. For
the detoxication patient displayed:
A. Using antispasmodics
B. Epidural anesthesia
C. Application antisecretion drugs
D. Perirenal blockade
E. Plasmapheresis
ANSWER: E

Patient K., aged 39, admitted to hospital with clinical acute pancreatitis. Which
analgesic should not be used?
A. Keterolak
B. Ketanov
C. Baralgin
D. Analgin
E. Can all
ANSWER: E

Patient P., aged 68, was admitted to hospital with clinical acute pancreatitis. Which
complications might arise?
A. Urination disorders
B. Wrapping Sigma
C. Pyelonephritis
D. Thyrotoxicosis
E. No right answer
ANSWER: E

Patient P., aged 78, was admitted to hospital with clinical acute pancreatitis. Which
complications might arise?
A. Urination disorders
B. Wrapping Sigma
C. Pyelonephritis
D. Thyrotoxicosis
E. Retroperitoneal phlegmon
ANSWER: E

Patient P., aged 58, was admitted to hospital with clinical acute pancreatitis. Which
complications might arise?
A. Urination disorders
B. Wrapping Sigma
C. Pyelonephritis
D. Thyrotoxicosis
E. Omental abscess
ANSWER: E

The patient admitted to hospital with a diagnosis: biliary pancreatitis. What type of
treatment is shown to the patient.
A. Treatment not indicated
B. Observation of the surgeon at home
C. Outpatient
D. Conservative treatment in hospital
E. No right answer
ANSWER: E

Patient R., 67 years old, enrolled in urgent hospital with a diagnosis: acute
pancreatitis. What medications is necessary for patient?
A. Hypnotics
B. Peripheral vasodilators
C. Prostaglandins
D. Venotonics
E. Cytostatics
ANSWER: E

Patient R., 67 years old, enrolled in urgent hospital with a diagnosis: acute
pancreatitis. What medications is necessary to appoint a patient?
A. Hypnotics
B. Peripheral vasodilators
C. Prostaglandins
D. Venotonics
E. Spasmodicals
ANSWER: E

Patient S., aged 65, went to urgent hospital with a diagnosis: acute pancreatitis.
Which disease is necessary to differentiate this pathology?
A. Cystitis
B. Hernia
C. Thrombophlebitis of subcutaneous veins
D. Thyrotoxicosis
E. Perforated ulcer
ANSWER: E

Patient S., aged 65, went to urgent hospital with a diagnosis: acute pancreatitis.
Which disease is necessary to differentiate this pathology?
A. Cystitis
B. Hernia
C. Thrombophlebitis of subcutaneous veins
D. Thyrotoxicosis
E. Thrombosis of mesenteric vessels
ANSWER: E

Patient K., aged 45, entered the reception room with suspected acute pancreatitis.
What kind of research required to clarify the diagnosis?
A. Scintigraphy
B. Doppler
C. MRI
D. CT
E. there is no right answer
ANSWER: E

Patient K, aged 45, entered the reception room with suspected acute pancreatitis. What
kind of research required to clarify the diagnosis?
A. Scintigraphy
B. Doppler
C. MRI
D. CT
E. Ultrasound
ANSWER: E
The patient with acute pancreatitis showed a yellow color of the skin near the navel.
It is a symptom of the author?
A. Gray-Turners
B. Halsteads
C. Mondors
D. Lagermfs
E. Kulens
ANSWER: E

Patient with acute pancreatitis revealed cyanosis lateral parts of abdominal skin. It
is a symptom of the author?
A. Kulens
B. Halsteads
C. Mondors
D. Lagermfs
E. Gray-Turners
ANSWER: E

Patient with acute pancreatitis revealed a sharp cyanosis of the face. It is a symptom
of the author?
A. Kulens
B. Gray-Turners
C. Halsteads
D. Mondors
E. Lagermfs
ANSWER: E

The patient with acute pancreatitis revealed purple spots on the face and body. It is
a symptom of the author?
A. Kulens
B. Gray-Turners
C. Halsteads
D. Lagermfs
E. Mondors
ANSWER: E

In operation for acute abdomen intraoperatively revealed a hemorrhagic effusion and


foci of fat necrosis in the peritoneum. What kind of illness should think?
A. perforated gastric ulcer
B. mesenteric thrombosis
C. hollow organ injury
D. rupture of the liver
E. acute pancreatitis
ANSWER: E

The patient after intake of alcohol appeared repeated vomiting, epigastric pain of
herpes character. Diagnosis
A. Crohn diseases
B. bleeding from varices of the esophagus
C. Gastric ulcer
D. Mallory-Weiss syndrome
E. there is no right answer
ANSWER: E

For a patient F., 44 years., erosive proctitis is diagnosed. What is preparation of


choice in treatment of disease?
A. Vitamins
B. Enzymes preparations
C. Hormonal preparations
D. Antibiotics
E. Sulphosalaso-drugs
ANSWER: E

For a patient an unspecific ulcerative colitis is diagnosed. Prescribed anti-recurrent


treatment. What is preparation of choice in treatment of disease?
A. Salbutamolum
B. Smecta
C. Lipofundinum
D. NSAID
E. Sulfasalasin
ANSWER: E

The patient of B., 47 years, entered to surgical department with complaints for pain
in the left inguinal area, weight lost. Objectively: T- 38 F. Stomach troubles,
painful at palpation in the left inguinal area. At rectoscopy hyperaemia and deep
cracks of mucous membrane, ulcer, stenosis is founded. What is preparation of choice
in treatment of disease?
A. Smecta, Imodium.
B. mercaptopurine
C. NSAID
D. Anticoagulants
E. 5-ASK, sulfasalasin, NSAID
ANSWER: E

The patient W., 38 years, entered to surgical department with complaints for eriodic
pain in a right inguinal area, which arises up after the reception of meal, weight
lost. Objectively: T- 38 F. Stomach is subinflated, painful at palpation in the left
inguinal area. At rectoscopy is hyperaemia and deep cracks of mucous membrane, ulcer,
stenosis. What is the diagnosis of patient?
A. Salmonellas
B. Unspecific ulcerative colitis
C. Disease of Hirshprung.
D. Dysentery
E. Crohns disease
ANSWER: E

For a patient., 45., it is diagnosed unspecific ulcerative colitis, phase of remission.


What is preparation of choice in treatment of disease?
A. hormones
B. vitamins of group In
C. cytostatics
D. antiagregant
E. preparations of 5-aminosalicile acid
ANSWER: E

For a patient., 45., a unspecific ulcerative colitis is diagnosed. It is conducted


irrigography. On a sciagram the characteristic sign of this disease is founded. It is
a symptom:
A. "niches"
B. "half of month"
C. "roadways"
D. "index finger"
E. "water-pipe"
ANSWER: E

Patient R. 32 years, complains for periodic pain in a right inguinal area, increase of
temperature to 38C., general weakness, diarrhoea with the admixtures of blood. At a
rectoscopy an edema, hyperaemia, plural erosions, ulcers, festering and necrotizing
raid of mucous membrane of intestine, is founded. What possible diagnosis?
A. Diverticulitis.
B. Crohns disease
C. Poliposis
D. Enterocolitis
E. Unspecific ulcerative colitis
ANSWER: E

Patient, 70 years, during the last 3 months disturb constipation. During the last 2
weeks was absent of stool during 3-4 days. Patient is used purgative drugs. One week
ago was bleeding from rectum up to 200 ml of the fresh blood. Lost of weight up to 10
kg, an appetite is bad. At examination the general state is middle weight. During
palpation of abdomen tumour-like formation is palpated in the left iliac area by size
68 cm. At percussion - tympanitis. It is not founded any pathology at digital
examination of rectum. What previous diagnosis?
A. No right answer.
B. Volvulus of sigmoid bowel
C. Megacolon
D. Diverticulitis of sigmoid bowel
E. Tumour of the left half of colon
ANSWER: E

The patient F., in 46 years, entered with complaints of pain in the left inguinal area,
diarrhoea up to 20-30 times on twenty-four hours, with the admixtures of blood and
mucus. T - 38 C. Per rectum is a mucous with hyperaemia, swelling, covered by the
ulcers of different size, with bleeding. What disease?
A. Diverticulosis
B. Poliposis
C. Polyps
D. Diverticulums
E. Unspecific ulcerative colitis
ANSWER: E

To the patient G., 62 years, roentgenologically and on colonoscopy is diagnosed


unspecific ulcerative colitis. What method of treatment of this patient?
A. enemas with a celandine
B. enemas with cytostatics
C. enemas with NSAID
D. enemas with smecta
E. enemas with sulfasalasin
ANSWER: E

Patient of B., 38 years, entered with complaints of moderate stomach-ache, diarrhoea


up to 17 times on twenty-four hours with blood, pus, mucus. Temperature of body is 38
F. Stomach is inflated, painful in the colon projection. Per rectum: dark blood is
founded. It is anaemia, ESR - a 42 mm/hour. What is diagnosis?
A. Specific ulcerative colitis
B. Diverticulosis
C. Crohns disease.
D. Enterocolitis
E. Unspecific ulcerative colitis
ANSWER: E

Patient of H., 30 years, a previous diagnosis is: Poliposis of colon. What most
reliable method of diagnostics of polypusТs of colon is indicated to the patient?:
A. Laparoscopy
B. Examination of excrement
C. Irrigoscopy
D. X-Ray with sulphate of barium
E. Colonoscopy
ANSWER: E

For a patient, 39Ё., an unspecific ulcerative colitis is diagnosed. In the case of


development of complications, which complication does not need operative treatment?
A. Perforation
B. Malignancy
C. Toxic megacolon
D. Bleeding
E. Water-electrolyte changes
ANSWER: E

Patient of A, 45 years, diagnosis: unspecific ulcerative colitis. What treatment is


indicated to the patient?
A. All are false
B. Subtotal colectomy with ileostomy
C. Total colectomy with an ileostomy
D. Complete parenterally feed
E. All are true
ANSWER: E

A patient T., 35 years, complaints for a periodic pain in abdomen, frequent (up to 16
- 18 on twenty-four hours) unexecuted stool with mucus and blood. An unspecific
ulcerative colitis is diagnosed. The sudden stopping of diarrhoea came on a background
of heavy intoxication. The indicated changes of signs of disease means:
A. hypovolemia
B. perforation
C. misdiagnosis
D. efficiency of conservative therapy
E. toxic dilatation
ANSWER: E

Patient S., 45 years, passed irrigography. Clinically and on the sciagram a toxic
megacolon is founded. For what disease these changes is characteristic?
A. To the syndrome of Paits-Egers
B. To the syndrome of Gardner
C. Disease of Hirshprung
D. Crohns disease
E. Unspecific ulcerative colitis
ANSWER: E

A patient 38 years complain for pain in the area of anus, fever. He is ill a 5 days. A
disease began acutely. In anamnesis an operation is concerning a acute paraproctitis 2
years ago. At a examination on the left from anus slight swelling, turning, pain at
palpation. In the area of scar of 2cm from anus point opening with festering
excretions. What diagnosis?
A. thrombosis
B. submucous paraproctitis
C. pelviorectal paraproctitis
D. ishiorectal paraproctitis
E. recrudescent paraproctitis
ANSWER: E

A patient 38 years complain for pain in the area of anus, fever. He is ill a 5 days. A
disease began acutely. In anamnesis an operation is concerning a acute paraproctitis 2
years ago. At a examination on the left from anus slight swelling, turning, pain at
palpation. In the area of scar of 2cm from anus point opening with festering
excretions. What treatment is indicated to the patient?
A. an operation is after Kenu-Mailce
B. operation after Ryzhykh- 1
C. conservative
D. opening of paraproctitis
E. operation after Habriel
ANSWER: E

A patient 60 years complain for pain in an anal area, high temperature of body,
diarrhoea with constipations, periodically appearance of blood in incandescence. What
sign testify in behalf on a acute paraproctitis in this case ?
A. only pain
B. constipations
C. diarrhoea
D. enterorrhagia
E. pain, high temperature of body
ANSWER: E

A patient 36 years complain for pain in a crotch, fever, high temperature. He is ill a
5 days. A disease began acutely. State of middle weight. At a examination slight
swelling of buttock on the right, pain at palpation. At the finger inspection of
rectum acutely sickly compression, that swelling in a rectum. What is the ?
A. thrombosis
B. extrasphincteric fistula
C. acute pelviorectal paraproctitis
D. acute submucous paraproctitis
E. acute ishiorectal paraproctitis
ANSWER: E

The patient of 33 years complain for the expressed pains in the area of anus, hat
arise up at the end of act of defecation, admixtures of blood in incandescence. He is
ill for a year. Pains were intensive at first, intensity diminished then. At a
examination on the back commissure of anal ring longitudinal linear wound a 2*1 cm,
pale-grey, with the hypertrophied roller. On a 2 cm higher line of comb polypus on a
narrow leg diameter by 0.5 cm. What operations at posterior anal fissure with polypus?
A. Operation Blinnichev.
B. For Habriel.
C. For Milligan-Morgan.
D. Cut off of fissure.
E. Cut off of fissure and removal of polypus
ANSWER: E

The patient 32 years complain for the expressed pain in the area of anus, which arises
up at the end of act of defecation, admixtures of blood in incandescence. He is ill
for a year. Pains were intensive at first, intensity diminished then. At a examination
on the back commissure of anal ring longitudinal linear wound a 2*1 cm, pale-grey,
with the hypertrophied roller. Operation, that indicated to the patient?
A. Operation Noblja.
B. Operation Habriel.
C. Operation Milligan-Morgan.
D. Suturing of fissure.
E. Cut off the fissure with dosed sphincterotomy.
ANSWER: E

A patient 30 years complain for the unpleasant feeling in the area of rectum and
periodic bleeding at the end of act of defecation, fall off haemorrhoidal knots at
defecation. He is ill a few years. The state is satisfactory. At anoscopy enlarged
haemorrhoidal knots are determined at 11 hour. What operation is radical in this case?
A. Conservative treatment
B. Sclerosing injections
C. Operation of Habriel
D. Ligation of haemorrhoidal knots
E. Operation of Milligan-Morgan
ANSWER: E

For a patient 29 years a chronic extrasphincteric paraproctitis is diagnosed. What


operation is indicated ?
A. Conservative treatment
B. Excision of fistula is with moving of mucous membrane of distal part of rectum
C. Excision of fistula (as Habriel)
D. Excision of fistula is with imposition of deaf sutures
E. Excision of fistula is with realization of ligature
ANSWER: E

For a patient 27 years a acute subcutaneus paraproctitis is diagnosed. What operation


is indicated?
A. Conservative treatment
B. Excision of abscess is with moving of mucous membrane of distal part of rectum
C. Excision of abscess is with realization of ligature
D. Excision of abscess is with imposition of sutures
E. Excision of abscess (as Habriel)
ANSWER: E

For a patient 26 years a acute ishiorectal paraproctitis is diagnosed. What operation


is indicated ?
A. Conservative treatment
B. Excision of abscess is with moving of mucous membrane of distal part of rectum
C. Excision of abscess (by Habriel)
D. Excision of abscess is with imposition of sutures
E. Excision of abscess is with realization of ligature
ANSWER: E

For the patient of 24 years on posterior anal is founded fissure of mucous rectum.
Pregnancy of 16 weeks. Tactic of surgeon.
A. operation after Kenu-Mailce
B. cut off fissure
C. operation after Habriel
D. operation after Milligan-Morgan
E. Cut off fissure after birth of child
ANSWER: E

A 19 y.o. man was admitted to the reception department in 20 minutes after being
wounded with the knife to the left chest. The patient is confused. The heart rate is
96 bpm and BP - 80/60 mm Hg. There are the dilated neck veins, sharply diminished
apical beat and evident heart enlargement What kind of penetrative chest wound
complications has developed in patient?
A. Massive hemothorax
B. Open pneumothorax
C. Closed pneumothorax
D. Valve-likes pneumothorax
E. Pericardium tamponade
ANSWER: E

For a patient 25 years three month ago the ulcer of rectum is diagnosed. At a complex
examination A Wassermann test is positive. What is diagnosis
A. crack of rectum
B. fistula
C. paraproctitis
D. hemorroids
E. venereal limphogranuloma
ANSWER: E

The patient of 43 after the operation of haemorrhoidectomy had cicatricle stricture of


anus. What operation is indicated?
A. colostomy
B. hemorrhoidectomy
C. dosed sphincterotomy
D. bougie of rectum from suturing of mucous rectum to perianal skin
E. dosed sphincterotomy and suturing of mucous rectum to perianal skin
ANSWER: E

For a patient the combined hemorroids, complicated by a thrombosis, is diagnosed. A


patient is disturbed by pain, discomfort, itch. What is the indication to the
operation in this case
A. discomfort
B. itch
C. pain
D. age
E. thrombosis of haemorrhoidal nodes
ANSWER: E

For a patient intersphincteric fistula of i is 30 years diagnosed external hemorroids.


He is ill about 5 years. What operation is indicated?
A. Operation Ryzhykh- 1.
B. Operation Blinnichev.
C. By Habriel.
D. By Milligan-Morgan.
E. After Milligan-Morgan and Habriel
ANSWER: E

The patient of 28 years complain for the unpleasant feeling in the area of rectum and
periodic bleeding at the end of act of defecation. He is ill a few years. The state is
satisfactory. Palpation of stomach - without pathology. At anoscopy the haemorrhoidal
knot are determined at 11 hour, blood with the edema of mucous membrane. Haemoglobin
is 100 /L, red corpuscles 2,7 * 1012.What is a diagnosis?
A. Poliposis, bleeding, anaemia.
B. Unspecific ulcerative colitis, bleeding, anaemia.
C. Cancer of rectum, bleeding, anaemia.
D. Gastric ulcer, bleeding, anaemia.
E. Internal haemorrhoid, bleeding, anaemia.
ANSWER: E

The patient of 29 years complain for the unpleasant feeling in the area of rectum and
periodic bleeding at the end of act of defecation. He is ill a few years. The state is
satisfactory. Palpation of stomach - without pathology. It is not founded at the
examination of anus and finger inspection of pathological structure. At anoscopy the
haemorrhoidal knot are determined at 11 hour, blood with the edema of mucous membrane.
Haemoglobin is 100 /L; red corpuscles 2,7 * 1012. Treatment?
A. Operation Blinnichev.
B. Operation Habriel.
C. Ryzhykh- 2.
D. Ryzhykh- 1.
E. Operation Milligan-Morgan.
ANSWER: E

Patient during the act of defecation has pain in anal channel, red blood in the stool.
What disease?
A. Proctopolypus
B. Cancer of rectum
C. Acute paraproctitis
D. Haemorrhoids
E. Anal fissure
ANSWER: E

A 35 y.o. patient complains of a difficult swallowing, pain behind the reastbone. He


can eat only liquid food. While swallowing sometimes he has attacks of cough and
dyspnea. Above mentioned complaints are progressing. It is known that the patient has
had a chemical burn of esophagus one month ago. What complication does the patient
have?
A. Esophagitis
B. Esophageal diverticula
C. Cardiac achalasia
D. Cardiac insufficiency
E. Corrosive esophagitis and stricture
ANSWER: E

A 64 y.o. patient has developed of squeering substernal pain which had appeared 2
hours ago and irradiated to the left shoulder, marked weakness. On examination: pale
skin, cold sweat. Pulse- 108 bpm, AP- 70/50 mm Hg, heart sound are deaf, vesicular
breathing, soft abdomen, painless, varicouse vein on the left shin, ECG: synus rhythm,
heart rate is 100 bmp, ST-segment is sharply elevated in II, III aVF leads. What is
the most likely disorder?
A. Cardiac asthma
B. Pulmonary artery thromboembolia
C. Disquamative aortic aneurizm
D. Cardiac tamponade
E. Cardiogenic shock
ANSWER: E

A 51 y.o. woman complains of dull pain in the right subcostal area and epigastric area,
nausea, appetite decline during 6 months. There is a history of gastric peptic ulcer.
On examination: weight loss, pulse is 70 bpm, AP is 120/70 mm Hg. Diffuse tenderness
and resistance of muscles on palpation.There is a hard lymphatic node 1x1cm in size
over the left clavicle. What method of investigation will be the most useful?
A. Ultrasound examination of abdomen
B. pH-metry
C. Ureatic test
D. Stomach X-ray
E. Esophagogastroduodenoscopy with biopsy
ANSWER: E

Patient 27 years old has stomach-ache, liquid stool up to 10 times per days with mucus
and blood, weakness, weight lost. On the irrigigraphy is narrowing of transvers colon.
What diagnosis?
A. Unspecific ulcerative colitis
B. Spastic colitis.
C. Polypus of small intestine.
D. Dysentery.
E. Cancer of transvers colon.
ANSWER: E

Patient 50 years old has permanent pain in the anus, frequent defecation with blood,
lost of appetite, weight lost, weakness. What examination is prescribed?
A. Sonography
B. Doplerography
C. Selective angiography
D. Radio-active scan
E. A biopsy with histological examination
ANSWER: E

Patient has a general weakness, presence of dark blood in the stool. At a rectoscopy
on 11 cm from anus is found the circular narrowing of rectum. What diagnosis?
A. Acute proctitis
B. Chronic paraproctitis
C. Acute paraproctitis
D. Proctopolypus
E. Cancer of rectum
ANSWER: E

Patient 59 years old has suspicion of the tumor of ascending part of colon. What
method of examination is the best?
A. Endoscopy
B. Sonography
C. Survey sciagraphy
D. Irrigography
E. A colonoscopy with a biopsy
ANSWER: E

The patient diagnosed with nodular euthyroid goiter left lobe of the thyroid gland.
What is the optimal treatment option.
A. conservative treatment thyroidin
B. subtotal thyroidectomy
C. enucleation site
D. excision of the node with the routine histological examination
E. hemistrumectomy or resection of the lobe of the thyroid gland
ANSWER: E

The patient S., 546 years old, diagnosed with goiter II degree. What is characteristic
of the III degree of increase in thyroid gland?
A. determined only on ultrasound
B. visible only when swallowing
C. determined only by palpation
D. giant goiter
E. visible swallowing
ANSWER: E

A man 50 years of asymptomatic mixed goiter. At the first stage, it should appoint
A. radioiodine
B. subtotal thyroidectomy
C. propylthiouracil
D. thyroid hormones to suppress the function of cancer
E. only observation
ANSWER: E

Patient K, aged 49, was admitted to the hospital with the diagnosis: diffuse toxic
goiter. What study be done.
A. There is no correct answer
B. Doppler
C. Rheovasography
D. EFGDS
E. Scanning of the thyroid gland
ANSWER: E

Patient M, 39 years old, was admitted to the hospital with the diagnosis: diffuse
toxic goiter. What study be done.
A. Is no right answer
B. Doppler
C. Rheovasography
D. EFGDS
E. Thyroid gland
ANSWER: E

The patient complaints of excessive sweating, hand tremor, exophthalmos. The most
likely diagnosis.
A. Is no right answer
B. Thyrotoxicosis
C. Thyreodit
D. Goiter
E. Strumit
ANSWER: E

Patient M., aged 35, lives in the area of iodine deficiency, was admitted to the
clinic with complaints of enlarged thyroid gland. What is the most likely diagnosis in
a patient?
A. mass thyrotoxicosis
B. epidemic goiter
C. sporadic goiter
D. acute strumitis
E. there is no right answer
ANSWER: E

Patient K., aged 45, lives in the area of iodine deficiency, was admitted to the
clinic with complaints of enlarged thyroid gland. What is the most likely diagnosis in
a patient?
A. mass thyrotoxicosis
B. epidemic goiter
C. sporadic goiter
D. acute strumitis
E. goiter
ANSWER: E

The patient's 43 years revealed an increase in the left lobe of the thyroid gland.
When scanning in this region found a hot site. Diagnosis.
A. non-toxic nodular goiter
B. multinodular toxic goiter
C. toxic goiter
D. diffuse non-toxic goiter
E. nodular toxic goiter
ANSWER: E

A patient complains of pathological lump, appearing in the right inguinal region on


exercise. The lump is round-shaped, 4 cm in diameter, on palpation: soft elastic
consistency, positions near the medial part of PoupartтАЩs ligament. The lump is
situated inwards from the spermatic cord. What is the most probable preliminary
diagnosis?
A. Right-sided oblique inguinal hernia
B. Right-sided femoral hernia
C. Varicose veins of the right hip
D. Lipoma of the right inguinal area
E. Right-sided direct inguinal hernia
ANSWER: E

Patients after resection of the thyroid having convulsions, symptoms by Chvostek and
Trousseau. What a complication arose in a patient?
A. hypothyroidism
B. thyrotoxic crisis
C. residual effects of hyperthyroidism
D. laryngeal nerve injury
E. there is no right answer
ANSWER: E

Patients after resection of the thyroid having convulsions, symptoms by Chvostek and
Trousseau. What a complication is arose in a patient?
A. hypothyroidism
B. thyrotoxic crisis
C. residual effects of hyperthyroidism
D. laryngeal nerve injury
E. hypoparathyreosis
ANSWER: E

On examination, the patient was 32 years reveal the formation of the left lobe of the
thyroid gland size 4x6 cm, painless at palpation. What additional diagnostic method to
assign?
A. EEG
B. Rheovasography
C. Doppler
D. Radiography of the neck
E. Thyroid gland
ANSWER: E

On examination, the patient was 32 years reveal the formation of the left lobe of the
thyroid gland size 4x6 cm, painless at palpation. What analysis should be performed in
order to clarify the diagnosis?
A. Protein fraction
B. Immunogram
C. Urinalysis
D. Total blood
E. Thyroid hormones
ANSWER: E

Patient O., aged 39, on the diffuse toxic goiter performed subtotal resection of the
thyroid gland. One day the patient became restless, twitching of facial muscles
appeared convulsive reduction of hands. Treatment.
A. introduction of calcium chloride oral
B. infusion therapy
C. introduction of iodine
D. introduction Seduxen
E. the introduction of calcium chloride intravenously
ANSWER: E

Patient S., 43, in the last 5 months of worry tearfulness, irritability, fatigue,
progressive weight loss. On palpation the thyroid gland increased to III class.,
painless. Pulse 110-120 in minute, regular, blood pressure - 150/80. What analysis
should be performed in order to clarify the diagnosis?
A. Immunogram
B. Protein fraction
C. Urinalysis
D. Total blood
E. Thyroid hormones
ANSWER: E

Patient S., 43, in the last 5 months of worry tearfulness, irritability, fatigue,
progressive weight loss. On palpation the thyroid gland increased to III class.,
painless. Pulse 110-120 in minute, regular, blood pressure - 150/80. At USD: tissue
homogeneous, tissue hypertrophy hyper. The most likely diagnosis.
A. goiter De Quervain
B. nodular goiter
C. Acute thyroiditis
D. Hashimoto's thyroiditis
E. there is no right answer
ANSWER: E

Patient S., 43, in the last 5 months of worry tearfulness, irritability, fatigue,
progressive weight loss. On palpation the thyroid gland increased to III class.,
painless. Pulse 110-120 in minute, regular, blood pressure - 150/80. At USD: tissue
homogeneous, tissue hypertrophy hyper. The most likely diagnosis
A. goiter De Quervain
B. nodular goiter
C. Acute thyroiditis
D. Hashimoto struma
E. toxic goiter
ANSWER: E

The patient, aged 39, suffering from stomach ulcer, recently had a feeling of
heaviness in the epigastric region, which is usually only after vomiting. The patient
has lost much weight. What is the treatment in this patient?
A. Translate patient infectious disease clinic
B. Transfer the patient in therapy department
C. Conservative treatment
D. Outpatient
E. Surgical treatment
ANSWER: E

The patient, aged 39, suffering from stomach ulcer, recently had a feeling of
heaviness in the epigastric region, which is usually only after vomiting. The patient
has lost much weight. What method of diagnosis should be used to refine the diagnosis?
A. MRI
B. CT
C. Ultrasound
D. ECG
E. X-ray study of the abdominal cavity with the passage of barium
ANSWER: E

The patient, aged 48, suffering for years with stomach ulcer. Diagnosed malignancy
ulcers. Tactics.
A. Translate patient infectious disease clinic
B. Transfer the patient in therapy department
C. Conservative treatment
D. Outpatient
E. Surgical treatment
ANSWER: E

The patient, aged 48, suffering for years with stomach ulcer, said that the pain had
not become dependent on food intake, have become permanent and less intense. Noted
weakness, malaise, fatigue. Lost 5 kg. ESR 40 mm / hour. What method of diagnosis
should be used to refine the diagnosis?
A. MRI
B. CT
C. X-ray study
D. ECG
E. EFGDS
ANSWER: E

In the ward the patient brought in an unconscious state, with periodic bouts of clonic
seizures. According to relatives, the patient for many years suffered from a disease
of the stomach. Last month the patient had copious vomiting every day, and he lost
considerable weight. On examination, the patient exhausted, dehydrated, in the
epigastric region has pigmentation, is determined by the splashing. What method of
research to clarify the diagnosis shows the patient after stabilization of the state?
A. Ultrasound
B. MRI
C. CT
D. ECG
E. X-ray study of the abdominal cavity with the passage of barium
ANSWER: E

In the ward the patient brought in an unconscious state, with periodic bouts of clonic
seizures. According to relatives, the patient for many years suffered from a disease
of the stomach. Last month the patient had copious vomiting every day, and he lost
considerable weight. On examination, the patient exhausted, dehydrated, in the
epigastric region has pigmentation, is determined by the splashing. What is the
diagnosis can be made sick?
A. All answers are correct
B. Ulcer Penetration
C. Malignancy ulcers
D. Perforated ulcer
E. Pyloristenosis
ANSWER: E
The patient, 30 years old, admitted to the surgical department after 2 hours from the
moment of perforation of gastric ulcer. The diagnosis beyond doubt, but the patient
categorically refused the operation. Your tactics.
A. Translate patient infectious disease clinic
B. Transfer the patient in therapy department
C. Forced to operate on a patient
D. Outpatient
E. Taylors method
ANSWER: E

The patient, 35 years old, was admitted to the surgical department with ulcerative
gastric hemorrhage. What medications is necessary to appoint a patient?
A. Veintonics
B. Prostaglandins
C. Vasodilators
D. Antibiotics
E. Preparations of blood
ANSWER: E

The patient, 33 years old, with no ulcer history admitted to the surgical department
with the diagnosis: perforated gastric ulcer. In the operation revealed perforation
hole 0,5 ? 0,5 cm on the lesser curvature of the stomach, without inflammatory
infiltration around. What should be the volume of transactions?
A. Gastrostomy
B. The operation is not shown
C. Resection of gastric Billroth II
D. Resection of gastric Billroth I
E. Suturing of perforated holes
ANSWER: E

The patient, 35 years old, was admitted to the surgical department with ulcerative
gastric hemorrhage. What medications is necessary to appoint a patient?
A. Veintonics
B. Prostaglandins
C. Vasodilators
D. Antibiotics
E. Saline infusion solutions
ANSWER: E

The patient with a history of ulcerative while examining the abdomen indicated the
disappearance of liver dullness. What is the diagnosis should be suspected in a
patient?
A. All answers are correct
B. Ulcer Penetration
C. Malignancy ulcers
D. Perforated ulcer
E. Ulcer bleeding
ANSWER: E
The patient, 35 years old, admitted to emergency room, diagnosed a bleeding stomach
ulcer. Where the patient must treatment?
A. Day hospital
B. Orthopedics
C. Surgery
D. Therapeutic department
E. Intensive Care Unit
ANSWER: E

The patient, 40 years old, long-suffering with stomach ulcer, said that the last 2
days the pain had become less intense, but at the same time it has been increasing
weakness, dizziness. In morning for a few seconds to lose consciousness. On
examination, pulse 100 in 1 min. The patient was pale. In the epigastric region is
very little pain. No symptoms of irritation of the peritoneum. What additional
research you can apply for further diagnosis?
A. MRI
B. CT
C. X-ray study
D. ECG
E. EFGDS
ANSWER: E

A 44-year-old male patient complains of severe non-localized abdominal pain, pain in


the right shoulder girdle, repeated vomiting, red urine. The onset of the disease is
associ-ated with alcohol consumption. The face is hyperemic. AP- 70/40 mm Hg.
Abdominal radiography reveals no pathological shadows. Hemodiastase is 54 mg/h/l.
Prothrombin is 46%. What is the provisional diagnosis?
A. Acute pancreatitis
B. Acute myocardial infarction
C. Perforated gastric ulcer
D. Thrombosis of mesenteric vessels
E. Aneurysm of the abdominal aorta
ANSWER: A

A 24-year-old male patient had been diagnosed with class III diffuse toxic goi-ter.
There is moderate hyperthyroidism. A surgery was suggested, and the patient agreed to
it. What preoperative measures should be taken for prevention of thyrotoxic crisis in
the postoperative period?
A. Administration of antithyroid drugs
B. Minimally invasive surgical techniques
C. Bed rest
D. Detoxification therapy
E. Administration of corticosteroids
ANSWER: A

A 47-year-old female patient has an 8-year history of ulcerative colitis, has been
treated with glucocorticoids. She complai-ns of cramping pain in the umbilical regi-on
and left iliac region which has signifi-cantly increased during the past 2 weeks, di-
arrhea with mucus and blood 4-6 times a day, elevated body temperature up to 38 -39o
C , headache and pain in the knee joints. Objecti-vely: the patient is in moderate
condition, Ps- 108/min, AP- 90/60 mm Hg; heart and lungs are unremarkable; the tongue
is moi-st; abdominal muscle tone is significantly decreased; peristaltic noises are
absent. What complication developed in the patient?
A. Toxic dilatation of the colon
B. Perforation of the colon
C. Enterorrhagia
D. Stricture of the colon
E. Colon carcinoma
ANSWER: B

A 40-year-old male patient has had heavi-ness in the epigastric region for the last 6
months. He has not undergone any exami-nations. The night before, he abused vodka. In
the morning there was vomiting, and 30 mi-nutes after physical activity the patient
experi-enced dizziness and profuse hematemesis. What pathology should be suspected in
the first place?
A. Mallory-Weiss syndrome
B. Menetriers disease
C. Gastric ulcer
D. Perforated ulcer
E. Zollinger-Ellison syndrome
ANSWER: A

It has been suspected that a newborn has congenital diaphragmatic hernia (asphyctic
incarceration). What study will allow to confi-rm the diagnosis?
A. Plan radiography of the chest cavity
B. Plan abdominal radiography
C. Irrigography
D. Pneumoirrigoscopy
E. Fibroesophagogastroduodenoscopy
ANSWER: A

As a result of a road accident a 45-year-old male patient got multiple injuries,


namely closed fractures of the right humerus and the left antebrachial bones with a
di-splacement of bone fragments, a closed blunt abdominal injury. The patient was
delivered to the emergency department 30 minutes after the injury. Objectively: the
skin is pale. AP-90/20 mm Hg, there is pain and deformati-on at the fracture sites.
Abdomen is tense, palpation causes severe pain, there is rebound tenderness (positive
Blumbergs sign). What is the treatment tactics of choice?
A. Urgent diagostic laparotomy
B. Infusion therapy to stabilize blood pressure
C. Fracture immobilization, analgesia
D. Local anesthetic blockade of fractures
E. Additional tests to specify the diagnosis
ANSWER: A

4 weeks after a myocardial infarction, a 56-year-old male patient developed acute


heart pain, pronounced dyspnea. Objectively: the patients condition is extremely grave,
there is marked cyanosis of face, swelling and throbbing of neck veins, peripheral
pulse is absent, the carotid artery pulse is rhythmic,bpm, AP- 60/20 mm Hg.
Auscultation of heart reveals extremely muffled sounds, percussion reveals heart
border extension in both directions. What is the optimal treatment tactis for this
patient?
A. Pericardiocentesis and immediate thoracotomy
B. Oxygen inhalation
C. Puncture of the pleural cavity on the left
D. Conservative treatment, infusion of adrenomimetics
E. Pleural cavity drainage
ANSWER: A

A 49-year-old male patient consulted a doctor about difficult swallowing, voice


hoarseness, weight loss. These symptoms have been gradually progressing for the last 3
months. Objectively: the patient is exhausted, supraclavicular lymph nodes are
enlarged. Esophagoscopy revealed no esophageal pathology. Which of the following
studies is most appropriate in this case?
A. Computed tomography of chest and medi-astinum
B. X-ray of lungs
C. Multiplanar imaging of esophagus
D. Radioisotope investigation of chest
E. Ultrasound investigation of mediastinum
ANSWER: A

A 45-year-old male patient complains of acute pain in his right side irradiating to
the right thigh and crotch. The patient claims also to have frequent urination with
urine which resembles a meat slops. The patient has no previous history of this
condition. There is costovertebral angle tenderness on the right (positive
Pasternatskys symptom). What is the most likely diagnosis?
A. Acute glomerulonephritis
B. Acute appendicitis
C. Acute pyelonephritis
D. Acute cholecystitis. Renal colic
E. Acute pancreatitis
ANSWER: A

A 42-year-old male patient has been delivered to a hospital in a grave condition with
dyspnea, cough with expectoration of purulent sputum, fever up to 39, 5o C. The first
symptoms appeared 3 weeks ago. Two weeks ago, a local therapist diagnosed him wi-th
acute right-sided pneumonia. Over the last 3 days, the patients condition deteriorated:
there was a progress of dyspnea, weakness, lack of appetite. Chest radiography
confirms a rounded shadow in the lower lobe of the right lung with a horizontal fluid
level, the right si-nus is not clearly visualized. What is the most likely diagnosis?
A. Abscess of the right lung
B. Acute pleuropneumonia
C. Right pulmonary empyema
D. Atelectasis of the right lung
E. Pleural effusion
ANSWER: A
A patient was delivered to a surgical department after a road accident with a closed
trauma of chest and right-sided rib fracture. The patient was diagnosed with right-
sided pneumothorax; it is indicated to perform drainage of pleural cavity. Pleural
puncture should be made:
A. In the 2nd intercostal space along the middle clavicular line
B. In the 7th intercostal space along the scapular line
C. In the point of the greatest dullness on percussion
D. In the projection of pleural sinus
E. In the 6th intercostal space along the posterior axillary line
ANSWER: A

A 60-year-old woman, mother of 6 children, developed a sudden onset of upper abdominal


pain radiating to the back, accompanied by nausea, vomiting, fever and chills.
Subsequently, she noticed yellow discoloration of her sclera and skin. On physical
examination the patient was found to be febrile with temperature of 38, 9oC, along
with right upper quadrant tenderness. The most likely diagnosis is:
A. Choledochal cyst
B. Choledocholithiasis
C. Malignant biliary stricture
D. Benign biliary stricture
E. Carcinoma of the head of the pancreas
ANSWER: B

A 29-year-old patient works as a motor mechanic. Anamnesis shows frequent exposure to


cold, exacerbation of chronic bronchitis attended by cough with relativly small amount
of mucopurulent sputum, subfebrility, sometimes joined by hemoptysis and pain in the
right side of chest. Breathing is vesicular. X-ray shows darkening and sharp decrease
in size of the lower lobe distinctly visible on the X-ray image as a streak 2-3 cm
wide situated at the angle from lung root to the frontal costodiaphragmatic recess.
The most likely diagnosis is:
A. Middle lobe syndrome
B. Interlobular pleurisy
C. Bronchiectasis
D. Pneumonia
E. Peripheral lung cancer
ANSWER: E

A 29-year-old female patient complains of dyspnea, heaviness and chest pain on the
right, body temperature rise up to 37, 2oC. The disease is associated with a chest
trauma received 4 days ago. Objectively: skin is pale and moist. Heart rate is 90 bpm,
regular. Palpation reveals a dull sound on the right, auscultation reveals
significantly weakened vesicular breathing. In blood: RBCs - 2, 8 В· 1012/l, colour
index - 0,9, Hb- 100 g/l,WBCs - 8, 0В·109/l, ESR - 17 mm/h. What results of
diagnostic puncture of the pleural cavity can be expected?
A. Haemorrhagic punctate
B. Exudate
C. Purulent punctate
D. Transudate
E. Chylous liquid
ANSWER: A

A 40-year-old woman with a history of combined mitral valve disease with predominant
stenosis complains of dyspnea, asthma attacks at night, heart problems. At present,
she is unable to do even easy housework. What is the optimal tactics of the patient
treatment?
A. Mitral commissurotomy
B. Antiarrhythmia therapy
C. Antirheumatic therapy
D. Treatment of heart failure
E. Implantation of an artificial valve
ANSWER: A

A 43-year-old patient had right-sided deep vein thrombosis of iliofemoral segment 3


years ago. Now he is suffering from the sense of heaviness, edema of the lower right
extremity. Objectively: moderate edema of shin, brown induration of skin in the lower
third of shin, varix dilatation of superficial shin veins are present. What is the
most probable diagnosis?
A. Lymphedema of lower right extremity
B. Acute thrombosis of right veins
C. Postthrombophlebitic syndrome, varicose form
D. Parkes-Weber syndrome
E. Acute thrombophlebitis of superficial veins
ANSWER: C

Patient K., 50 years old, entered the hospital with complaints of pain in the right
lower quadrant, nausea, general weakness. Laparoscopy was appointed. When carrying out
a diagnostic laparoscopy on suspicion of abscess of Douglas space, the second port
input is typically carried out
A. On the midline of the abdomen in hypogastrium
B. In the periumbilical area
C. On the midline of the abdomen in the epigastrium
D. At McBurney point
E. At Volkovych-Kocher point
ANSWER: A

Patient K., 59 years old, entered the hospital with complaints of pain in the right
lower quadrant, nausea, general weakness. Laparoscopy was appointed. To perform a
diagnostic laparoscopy on suspicion of acute appendicitis, setting the working port is
typically carried out
A. On The median line below the umbilicus in hypogastrium
B. On the midline of the abdomen in mesogastrium
C. At midline abdominal
D. Epigastrium midline of the abdomen 2-3 cm below the xiphoid processus
E. At Volkovych-Kocher point
ANSWER: A

Patient M., 30 years old, entered the hospital with complaints of pain in the right
upper quadrant, nausea, general weakness. Laparoscopy was appointed. When carrying
out a diagnostic laparoscopy on suspicion of acute cholecystitis, second port is
typically inserted
A. Epigastrium midline of the abdomen 2-3 cm below the xiphoid processus
B. At Kerr point
C. On the midline of the abdomen in hypogastrium
D. At McBurney point
E. At Volkovych-Kocher point
ANSWER: A

Patient J., 50 years old, entered the hospital with complaints of pain in the right
lower quadrant, nausea, general weakness. Laparoscopy was appointed. When carrying
out a diagnostic laparoscopy on suspicion of acute salpingal disorders, setting the
working port is typically carried out
A. On the midline of the abdomen in hypogastrium
B. In the periumbilical area
C. On the midline of the abdomen in the epigastrium
D. At McBurney point
E. At Volkovych-Kocher point
ANSWER: A

Patient K., 40 years old, entered the hospital with complaints of pain in the right
upper quadrant, nausea, general weakness. Laparoscopy was appointed. To perform a
diagnostic laparoscopy on suspected perforated ulcer, the introduction of instrumental
conducting port is done
A. On the midline of the abdomen in the epigastrium
B. In the periumbilical area
C. At Kerr point
D. At McBurney point
E. At Volkovych-Kocher point
ANSWER: A

Patient L., 27 years old, entered the hospital with complaints of pain in the right
lower quadrant, nausea, general weakness. Laparoscopy was appointed. In the
implementation of laparoscopy over suspected salpingitis, preparing the second port
site is carried out
A. On the midline of the abdomen below the navel in hypogastrium
B. In mesogastrium 1 cm above or below the navel the median line
C. On the midline of the abdomen in the epigastrium 10 cm above the navel
D. At McBurney point
E. At Kerr point
ANSWER: A

Patient R., 45 years old, entered the hospital with complaints of pain in the right
upper quadrant, nausea, general weakness. Laparoscopy was appointed. At
laparoscopy on suspicion of extrahepatic bile duct cancer, the second port input is
carried out
A. In the midline epigastrium
B. At Volkovych-Kocher point
C. On the midline of the abdomen in hypogastrium 10 cm below the navel
D. At McBurney point
E. At Kerr point
ANSWER: A

Patient M., 55 years old, entered the hospital with complaints of pain in the right
upper quadrant, nausea, general weakness. Laparoscopy was
appointed. For laparoscopy on suspicion of liver echinococcosis, setting the
second port is carried out
A. Epigastrium
B. At Volkovych-Kocher point
C. On the midline of the abdomen in hypogastrium 10 cm below the navel
D. At McBurney point
E. At Volkovych-Kocher point
ANSWER: A

Patient R., 78 years old, entered the hospital with complaints of pain in the right
upper quadrant, nausea, general weakness. Laparoscopy was appointed. For laparoscopy
on limited local peritonitis, instrumental port setting is done
A. Depending on the localization of process
B. On the midline of the abdomen in the epigastrium 10 cm above the navel
C. On the midline of the abdomen in hypogastrium 10 cm below the navel
D. At Volkovych-Kocher point
E. At Kerr point
ANSWER: A

After laparoscopic surgery for cholelithiasis, the whole set of equipment requires
laparoscopic sterilization. What method of sterilization is used for video cameras?
A. liquid
B. gas
C. autoclaving
D. dry-air
E. All of the above
ANSWER: A

Female, 28 years old, complains of increasing abdominal weakness over the last year.
OBJECTIVE: abdomen is enlarged symmetrically, with percussion of the abdomen in
lateral canals and above the pubis - obtuse sound, slight pain at all abdomen, no
muscle tension and signs of peritoneal irritation. According to ultrasound there is a
free fluid in the abdomen in a large quantity. Recommended to diagnostic laparoscopy.
Endovideosurgical complex consists of:
A. Videocomplex
B. Systems of aspiration and irrigation
C. Systems of insufflation
D. Coagulation Systems
E. All answers are correct
ANSWER: E

Patient I., 46 years old, entered the hospital with complaints of pain in the right
lower quadrant, nausea, general weakness. Laparoscopy was
appointed. In the implementation of laparoscopy on suspicion of pelvic
tumor, setting the working port is carried out
A. On the midline of the abdomen in hypogastrium 10 cm below the navel
B. In mesogastrium 1 cm above or below the navel the median line
C. On the midline of the abdomen in the epigastrium 10 cm above the navel
D. At Volkovych-Kocher point
E. At Kerr point
ANSWER: A

Patient '47 entered complaining of mild abdominal pain. Based on the review, the
results of clinical tests and examinations exhibited a preliminary diagnosis - liver
disease. The surgeon plans to perform aspiration biopsy of the liver. What tool use to
do a liver biopsy in this case?
A. Veresha Needle
B. Deschamps Needle
C. Biopsy Needle
D. Plain needle
E. biopsy forceps
ANSWER: C

Patient '67 arrived with complaints of frequent constipation, bloating. When


performing colonoscopy the endoscopist failed to pass the rectal-sigmoid angle of
colon. Was decided to perform the diagnostic laparoscopy. What tool should impose the
Colon-Colon anastomosis?
A. Linear stapler
B. Circular stapler
C. System of electrocoagulation Ligasure
D. Morselator
E. There is no right answer
ANSWER: B

Patient '33 entered complaining on severe abdominal pain, which originally appeared in
the epigastrium and in 2 hours migrated to the right iliac area. The patient exhibited
the diagnosis of acute appendicitis. What laparoscopic instrument the surgeon may
apply to ligate the appendix?
A. clipsator
B. Anatomical clamp
C. monopolar coagulation
D. Needle of Deschamps
E. Instrument for loading of needles
ANSWER: E

Patient '42 delivered in urgent order with complaints on pain epigastricm area during
the last 10 hours. Complaints appeared after the initiation of diet and alcohol abuse.
On examination, the abdomen is slightly swollen, involved in breathing, during
palpation he feeels the acute pain in the epigastrium and left upper quadrant.
Positive symptom of Mayo-Robson. According to ultrasound there is a liquid in the
omental bag. Which tool is recommended to disconnect the tissues?
A. L-shaped monopolar electrode
B. "universal" Clip
C. dissector
D. curved scissors
E. bipolar forceps
ANSWER: C

Patient '43, is prepared for the diagnostic laparoscopy to clarify the diagnosis. What
is considered to be the least traumatic tool for capturing intestine?
A. dissector
B. Babcock clamp type
C. Anatomical clip
D. 5-petaled Retractor
E. None of the above
ANSWER: B

32 year old patient during the recent week marks fever, pain in the chest. preliminary
diagnosis – abscess of the right lung. Differential diagnosis of chronic lung abscess
is made with:
A. atelectasis
B. bronchiectasis
C. pneumonia
D. lymphadenitis
E. emphisema
ANSWER: A

Patient B., 64 years old, entered the hospital with complaints of pain in the right
upper quadrant, nausea, general weakness. Laparoscopy was appointed. Chronic calculous
cholecystitis under review is characterized by the following features
A. gallbladder glued with surrounding organs, constricted, scars or sclerosis
B. the gallbladder is enlarged, streched, white and blue color
C. the gallbladder is enlarged, dark purple color, through extensive serous membrane
translucent mesh of blood vessels and capillaries
D. the gallbladder has whitish dense bulbs on the surface
E. none of the listed
ANSWER: A

Patient '62 delivered to the hospital complaining on persistent pain in the left upper
quadrant for 6 months. On examination, the abdomen is moderately tender in the left
upper quadrant. Symptoms of peritoneal irritation are absent. When a routine
splenectomy which of these methods are allowed for coagulation of splenic artery?
A. monopolar coagulation
B. Bipolar coagulation
C. System "Harmonica"
D. System Ligasure
E. monopolar coagulation using endoloop
ANSWER: D

Patient 53 years old, taken to hospital with complaints of abdominal pain, radiating
to the rectum, weakness, significant bleeding during menstruation, during the last 2
years. The abdomen is tense, slightly painful in the lower regions, negative symptoms
of peritoneal irritation, palpable round lesion above the vagina. What tool is used to
do fixing the uterus in its laparoscopic extirpation?
A. retainer of the "corkscrew"
B. Morselator
C. uterine manipulator
D. clamp type "crocodile"
E. The correct answer is missing
ANSWER: C

Patient B., 56years old, admitted with complaints of persistent pain in the right
upper quadrant during the day. The complaints came after a diet violation and abuse of
fatty foods. On examination the abdomen is sharply painful in the right upper quadrant.
Positive symptom of Ortner. The laparoscopic cholecystectomy is performed to patient.
To stop bleeding from the liver during the allocation of the gallbladder can use this
tool:
A. monopolar electrode type "scoop"
B. Surgical clamp
C. anatomical clip
D. Bipolar Forceps
E. clip of the "universal"
ANSWER: A

Patient B., 65 years old, entered the hospital with complaints of pain in the right
upper quadrant, nausea, general weakness. Laparoscopy was appointed. Chronic calculous
cholecystitis under review is characterized by the following features
A. gallbladder glued with surrounding organs, constricted, scars or sclerosis
B. the gallbladder is enlarged, streched, white and blue color
C. the gallbladder is enlarged, dark purple color, through extensive serous membrane
translucent mesh of blood vessels and capillaries
D. the gallbladder has whitish dense bulbs on the surface
E. none of the listed
ANSWER: A

Patient H., '29, who entered with complaints on icteric skin and sclera, and pain in
the right upper quadrant. After the survey was diagnosed calculous cholecystitis and
laparoscopic cholecystectomy performed using laparoscopic trocar and plastic tools.
What method of sterilization is used for this type of instrument?
A. liquid
B. gas
C. autoclaving
D. dry-air
E. None of the above
ANSWER: E

Patient K. '43, after the surgery the on chronic calculous cholecystitis was used
dissector, 2 anatomical clips, surgical clamp crocodile type and monopolar electrode.
At what temperature is for the presterilizing processing of the laparoscopic
instruments?
A. 50 ° C
B. 20 ° C
C. 30 ° C.
D. 90 ° C
E. 120 ° C
ANSWER: A

Patient K. '43, after the surgery the on chronic calculous cholecystitis was used
dissector, 2 anatomical clips, surgical clamp crocodile type and monopolar electrode.
It is necessary to conduct sterilization of used instruments. Which solution is
carried out for sterilization of instruments with dielectric coating?
A. alcohol
B. hydrogen peroxide
C. iodine
D. «Saydeks"
E. «Plivasept"
ANSWER: D

Patient C., 23 years old, entered the hospital with complaints of pain in the right
upper quadrant, nausea, general weakness. Laparoscopy was appointed. The gallbladder
is enlarged, dark purple color, through extensive serous membrane translucent mesh of
blood vessels and capillaries is in such pathologies as
A. Empyema of the gallbladder
B. Edema of the gallbladder
C. Chronic calculous cholecystitis
D. Cancer of the gallbladder
E. None of the
ANSWER: A

Patient K., 43 years old, was admitted with complaints of persistent pain in the right
upper quadrant during the day. The complaints came after a diet violation and abuse of
fatty foods. On examination the abdomen is sharply painful in the right upper quadrant.
Positive symptom of Ortner. The laparoscopic cholecystectomy is performed to patient.
Which tool is used to overlay clips on cystic artery?
A. System Ligasure
B. Linear stapler
C. Circular Stapler
D. clipator
E. All answers are correct
ANSWER: D

Patient M., '42 was performed arthroscopic surgery for meniscal injury of the knee
joint. After the surgery instruments require sterilization. What method of
sterilization is used for fiber optic cable?
A. liquid
B. gas
C. autoclaving
D. dry-air
E. All of the above
ANSWER: B

Patient P., '48, when performing laparoscopic surgery on chronic calculous


cholecystitis complications arose such as perforation gallbladder. There was a need to
wash the abdominal cavity. What is the solution used in the suction-irrigator?
A. 5% glucose
B. furatsillina
C. Physiological soloution
D. A solution with using antibiotic
E. chlorhexidine
ANSWER: C

Patient R., 58r., Appealed with complaints on violations of urination, hematuria.


After a physical and MRI study was diagnosed the prostate cancer with Mts to the
regional lymph nodes. There was a robot-assisted laparoscopy. What method of
sterilization of instruments used for laparoscopic robot da Vinci?
A. liquid
B. gas
C. autoclaving
D. dry-air
E. no correct answer
ANSWER: E

Patient, 23 years old, was taken to hospital urgently, complaining on pain in the
abdomen, more to the right, radiating to the rectum, dizziness. The above symptoms
appeared suddenly at night. Last menstrual period was 2 weeks ago. Objectively: skin
is pale, pulse - 92 for 1 min, body temperature - 36.6 0C, blood pressure - 100/60 mm
Hg. Art. Abdomen is slightly tense, slightly painful in the lower regions, the
symptoms of peritoneal irritation are weakly-positive. Analysis of blood: hemoglobin -
98 g / l. By which of these models of laparoscopes is the best to perform the
observing laparoscopy in this case?
A. 0 ° laparoscope 5 mm
B. laparoscope 30 ° 5 mm
C. 45 ° laparoscope 5 mm
D. 0 ° laparoscope 10mm
E. 45 ° laparoscope 10mm
ANSWER: B

patient, 43 years old, taken to hospital with complaints of abdominal pain, radiating
to the rectum, weakness, significant bleeding during menstruation, during the last 2
years. The abdomen is tense, slightly painful in the lower regions, negative symptoms
of peritoneal irritation, palpable round lesion above the vagina. Which tool is used
to remove the uterus when its laparoscopic extirpation?
A. Lock type "corkscrew"
B. Mortelator
C. uterine manipulator
D. Clamp type "crocodile"
E. The correct answer is missing
ANSWER: B
patient, 53 years old, taken to hospital with complaints of abdominal pain, radiating
to the rectum, weakness, significant bleeding during menstruation, during the last 2
years. The abdomen is tense, slightly painful in the lower regions, negative symptoms
of peritoneal irritation, palpable round lesion above the vagina. What tool is used
for fixation of the fibromatous node?
A. Morselyatom
B. retainer of the "corkscrew"
C. scissors
D. monopolar electrode endoloop
E. dissector
ANSWER: B

patients, 53 years old, taken to hospital with complaints on abdominal pain, radiating
to the rectum, weakness, significant bleeding during menstruation, during the last 2
years. Last menstrual period lasted 8 days. Objectively: skin is pale, pulse - 102 for
1 min, body temperature - 36.6 0C, blood pressure - 100/60 mm Hg. Art. The abdomen is
tense, slightly painful in the lower regions, negative symptoms of peritoneal
irritation, palpable the round lesion above the vagina. Analysis of blood: hemoglobin
- 88 g / l. What tool should perform removal of fibromatous site?
A. Moselator
B. retainer of the "corkscrew"
C. scissors
D. monopolar electrode endoloop
E. dissector
ANSWER: A

The patient, '22, that works as model, complained of abdominal pain in the right iliac
region during the last 2-years. Symptoms of peritoneal irritation are weakly positive.
After the observing laparoscopy revealed the inflammation of the appendix. What method
of laparoscopic surgery is the best in this case?
A. multiportal laparoscopy
B. Robot-assisted laparoscopy
C. one-port laparoscopy
D. laparoscopy through natural openings
E. Laparolifting
ANSWER: C

The patient, '22, that works as model, complained of abdominal pain in the right iliac
region during the last 2-years. Symptoms of peritoneal irritation are weakly positive.
Using clinical and laboratory studies were diagnosed catarrhal appendicitis. What
method of laparoscopic surgery is the best?
A. multiportal laparoscopy
B. Robot-assisted laparoscopy
C. one-port laparoscopy
D. laparoscopy through natural openings
E. Laparolifting
ANSWER: D
The surgeon performs a diagnostic laparoscopy to the patient with unclear diagnosis.
What kit of laparoscope should prepare for surgery?
A. 5 mm 0 ° and 30 ° 5 mm
B. 10 mm 0 ° and 45 ° 5 mm
C. 30 ° C. and 5 mm 5 mm 45 °
D. 5 mm 0 ° and 0 ° 10 mm
E. 10 mm 0 ° and 45 ° 10 mm
ANSWER: A

To patient F., '43, was performed the diagnostic laparoscopy with suspected ovarian
apoplexy, the gynecologist diagnosed the catarrhal appendicitis and performed
laparoscopic surgery. What is the name of the doctor and in what year was the first in
the world laparoscopic appendectomy?
A. Zemm in 1983
B. Tracing in 1929
C. Frederick in 1930
D. Hes in 1937
E. tapes in 1945
ANSWER: A

When performing diagnostic laparoscopy surgeon damaged the small intestine during
trocar introduction. By which the tool is applied the seam?
A. Instrument for applying continuous suture
B. Linear stapler
C. The standard laparoscopic needle holder
D. clipator
E. sticks for loading of nodes
ANSWER: A

Patient K., 40 years old, entered the hospital with complaints of pain in the right
lower quadrant, nausea, general weakness. Laparoscopy was appointed. To perform a
diagnostic laparoscopy on suspicion of acute appendicitis, port for laparoscope is
typically placed
A. In the periumbilical area
B. On the midline of the abdomen in the epigastrium
C. At midline of abdominal wall 10 cm upwards the navel
D. At midline in hypogastrium 10 cm below the navel
E. At Volkovych-Kocher point
ANSWER: A

Patient K., 50 years old, entered the hospital with complaints of pain in the right
upper quadrant, nausea, general weakness. For additional examination surgeon conducted
laparoscopy. In diagnostic laparoscopy surgeon pays attention to
A. signs of inflammation, tumor metastasis and free fluid in the abdominal
cavity, the shape and size of their color, vascular pattern
B. shape and size of organs, vascular pattern, signs of inflammation, tumor
metastasis and free fluid in the abdomen
C. shape and size of their color, signs of inflammation, tumor metastasis and free
fluid in the abdomen
D. shape and size of their color, vascular pattern, signs of inflammation, free fluid
in the abdomen
E. shape and size of their color, vascular pattern, signs of inflammation, tumors
ANSWER: A

Patient K., 50 years old, entered the hospital with complaints of pain in the right
upper quadrant, nausea, general weakness. Laparoscopy was appointed. Liver is granular
and nodular. This characteristic of laparoscopy picture indicates
A. Primary biliary cirrhosis
B. Hemochromatosis of liver
C. Obstructive cholangitis
D. Norm
E. Viral hepatitis
ANSWER: A

Patient K., 60 years old, came to the department complaining of pain in the right
upper quadrant, nausea, general weakness. Laparoscopy was appointed. Diagnostic review
is conducted
A. in the Trendelenburg position and horizontal position on the right and left side
B. in a horizontal position and the Trendelenburg position, on the right side
C. in a horizontal position and the Trendelenburg position, on the left side
D. horizontally on the right and left side
E. in the Trendelenburg position, on the right and left side
ANSWER: A

Patient K., 60 years old, came to the department complaining of pain in the right
upper quadrant, nausea, general weakness. Laparoscopy was appointed. The surface of
liver is smooth, dark brown in color with a greenish tinge. This is characteristic for
laparoscopy at
A. Pigmentary cirrhosis of liver
B. Primary biliary cirrhosis
C. Obstructive cholangitis
D. Norm
E. Viral hepatitis
ANSWER: A

Patient L., 55 years old, entered the hospital with complaints of pain in the both
upper quadrant, nausea, general weakness. Laparoscopy was appointed. To perform a
diagnostic laparoscopy on suspected perforated ulcer, input of optical port is
typically carried out
A. In the periumbilical area
B. On the midline of the abdomen in the epigastrium
C. At Kerr point
D. At McBurney point
E. At Volkovych-Kocher point
ANSWER: A
Patient M., 46 years old, entered the hospital with complaints of pain in the right
upper quadrant, nausea, general weakness. Laparoscopy was appointed. Edema of the
gallbladder under viewed characterized by the following features
A. the gallbladder is enlarged, streched, white and blue color
B. the gallbladder is enlarged, dark purple color, through extensive serous membrane
translucent mesh of blood vessels and capillaries
C. gallbladder glued with surrounding organs, constricted, scars or sclerosis
D. the gallbladder has whitish dense bulbs on the surface
E. none of the listed
ANSWER: A

Patient N., 32 years old, entered the hospital with complaints of pain in the right
upper quadrant, nausea, general weakness. Laparoscopy was appointed. Empyema of the
gallbladder under review is characterized by the following features
A. the gallbladder is enlarged, dark purple color, through extensive serous membrane
translucent mesh of blood vessels and capillaries
B. the gallbladder is enlarged, streched, white and blue color
C. gallbladder glued with surrounding organs, constricted, scars or sclerosis
D. the gallbladder has whitish dense bulbs on the surface
E. none of the listed
ANSWER: A

Patient N., 40 years old, was admitted to the hospital complaining of pain in the
right upper quadrant, nausea, general weakness. After additional examination surgeon
conducted laparoscopy. At diagnostic laparoscopy
A. You can change the position of the patient
B. You can not change the position of the patient
C. You can change the position of the patient in the sagittal plane
D. You can change the position of the patient in the frontal plane
E. preferably not to confuse the patient
ANSWER: A

Patient N., 40 years old, was admitted to the hospital complaining of pain in the
right upper quadrant, nausea, general weakness. After additional examination surgeon
conducted laparoscopy. The surface of liver is smooth, dark brown in color with a
greenish tinge. This characteristic of laparoscopy in
A. Hemochromatosis of liver
B. Norm
C. Obstructive cholangitis
D. Obstructive jaundice
E. Viral hepatitis
ANSWER: A

Patient N., 40 years old, was admitted to the hospital complaining of pain in the
right upper quadrant, nausea, general weakness. After additional examination surgeon
conducted laparoscopy. Greenish tint in the liver characterized
A. Obstructive cholangitis
B. Hemochromatosis of liver
C. Primary biliary cirrhosis
D. Obstructive jaundice
E. Norm
ANSWER: A

Patient S., aged 49, was admitted to hospital with complaints of pain in the right
upper quadrant, nausea, general weakness. After additional examination surgeon
conducted laparoscopy. Thin capillary net of blood vessels and soft whitish net of
hepatic lymph vessels is characteristic for
A. Viral hepatitis B
B. Hemochromatosis of liver
C. Primary biliary cirrhosis
D. Obstructive cholangitis
E. Obstructive jaundice
ANSWER: A

Patient S., aged 49, was admitted to hospital with complaints of pain in the right
upper quadrant, nausea, general weakness. After additional examination surgeon
conducted laparoscopy. Diagnostic laparoscopy with regulations is performed by means
of
A. move table to which the patient is fixed
B. manipulation of patients , not the table
C. manipulation by table to which the patient is fixed only sagittally
D. move table to which the patient is fixed only frontally
E. move table to which the patient is fixed only by height
ANSWER: A

In the surgical ward 82 yrs old patient enrolled. An examination showed suspected
acute cholecystitis abscess. What research method should be used primarily for further
diagnosis?
A. An ultrasound scan of the abdominal cavity
B. Infusion cholecystography
C. Laparoscopy
D. Retrograde panctreatocholangiography
E. Percutaneous cholecystography
ANSWER: A

In hospital delivered Male 42 years in critical condition: acrocyanosis, dyspnea,


subcutaneous emphysema in the neck and upper torso. Complains of severe chest pain,
epigastric pain. The body temperature of 38.9 (C, pulse 130 beats / min, blood
pressure = 80/50 mm Hg. Known that 6 hours ago after meals and alcohol appeared
vomiting and began to grow above mentioned phenomenon. Which is the preliminary
diagnosis?
A. Pinching paraesophageal hernia
B. Spontaneous pneumothorax
C. Pulmonary artery trombemboly
D. Spontaneous rupture of the esophagus
E. perforated ulcers of the stomach
ANSWER: D
In the department of thoracic surgery was hospitalized 43 year old patient with a
preliminary diagnosis of piotoraks. The fluid level reaches the bifurcation of the
trachea. Define standard Skeletopy of bifurcation of the trachea at the thoracic
vertebrae in adults:
A. IV vertebra
B. III vertebra
C. V vertebra
D. VI vertebra
E. VII vertebra
ANSWER: A

In the surgical ward taken patient of 38 years with stab wounds of the chest.
Patient's condition is serious. Consciousness confused, pale skin, cardiac deaf. Pulse
120 for 1 min., BP 70/40 mm Hg. Breathing - 32 for 1 min. On the front surface of the
chest in the third intercostal space on the left parasternal line a stab wound with
the length of 3cm. covered with blood clots is observed. Percussion – the border of
cardiac dullness is extended. What complication does occur?
A. Pericardial tamponade
B. Traumatic shock
C. Pulmonary artery
D. Acute cerebrovascular accident
E. Acute myocardial infarction
ANSWER: A

Patient B., 54 years old, was admitted to hospital with complaints of pain in the left
upper quadrant, nausea, general weakness. After additional examination surgeon
conducted laparoscopy. During conducting diagnostic laparoscopy surgeon diagnosed
hemorrhagic pancreonecrosis characterized by
A. presence of hemorrhagic fluid in the abdomen
B. presence of "stearic plaques"
C. dark red, brown or black pancreas
D. visual picture of acute cholecystitis
E. gland lobules are displayed clearly
ANSWER: A

In thoracic surgery there is patient of 37 years old with an abscess of the right lung.
For the second period, lung abscess characteristic such X ray picture:
A. eclipse of lung tissue without clear contours;
B. enlightenment of lung tissue without clear contours;
C. enlightenment of lung tissue with clear contours;
D. enlightenment of lung tissue specific cells;
E. lung tissue blackout with clear contours.
ANSWER: C

Patient B., 56 years old, was admitted to hospital with complaints of pain in the
right upper quadrant, nausea, general weakness. After additional examination
laparoscopy for treatment was appointed. For the question of urgency of surgery for
acute cholecystitis the most important is:
A. The presence of peritonitis
B. The intensity of pain
C. Duration of disease
D. The number of attacks in history
E. The presence of gallstones
ANSWER: A

Male 40 years with complaints of dizziness, pain in the left side of the chest,
shortness of breath. Got sick for 1 hour ago after hitting the left side on the edge
of the table. Pulse 122 beats/min. Respiratory rate 38 per minute. BP 90/60 mm Hg The
left half of the chest is behind during breathing, percussion over the lower part to 8
intercostal space (determined the dull sound, above – tympanic sound, auscultation -
breath sharply weakened. Diagnosis?
A. Pneumothorax
B. Hemothorax
C. Hemopericard
D. Hemopneumothorax
E. Pneumoempyema
ANSWER: D

Male 40 years, delivered by an ambulance after the accident, complaining of pronounced


shortness of breath, pain in the right half of the chest and sternum, cough with a
small amount of bright frothy blood. OBJECTIVE: condition of the patient is extremely
difficult, cyanosis of the face, severe subcutaneous emphysema of the chest wall, neck
and face. pulse - rhythmic, 110 for 1 min., BP - 90/60 mm Auscultation of the lungs -
breathing is rapidly weakened throughout, left side - satisfactory. What method of
examination is necessary to diagnose pathology for this patient?
A. Radiography of the chest and fibrobronchoscopy
B. Electrocardiography
C. Computed tomography of the brain
D. EFGDS
E. Ultrasound
ANSWER: A

Patient B., 56 years old, was admitted to hospital with complaints of pain in the
right upper quadrant, nausea, general weakness. After additional examination
laparoscopy for treatment was appointed. During the day after laparoscopic drainage of
choledochus bile in average is measured
A. Up to 700 -1000ml
B. Up to 5 ml
C. Up to 10 ml
D. Up to 15ml
E. Up to 2-3l
ANSWER: A

Patient '32 entered the surgical department within 6 hours from the time of disease
diagnosis: spontaneous pneumothorax. Treatment with passive drainage of the pleural
cavity for 3 days gave no effect. What is the most reliable studies will reveal the
cause of no effect of treatment?
A. Bronchography
B. Plain radiography of the chest
C. Thoracoscopy
D. Bronchoscopy
E. Ultrasound
ANSWER: C

Patient '39 hospitalized in thoracic surgery, complaining of chest pain, hard


breathing, fever. Diagnosed pulmonary gangrene. Gangrene of the lungs accompanied by
sputum:
A. Color of meat slops
B. light color
C. dark color
D. the blood
E. foam
ANSWER: A

Patient '40. Suddenly appeared pain in the left half of the chest, choking. State of
moderate severity, pulse - 110 per minute, blood pressure 90/60 mm Hg. Breathing in
the left side does not listen. When radiography of the chest - the collapse of the
left lung on one half. What treatment you want to assign to the patient?
A. rest, resolution therapy
B. Pleural puncture
C. Surgical treatment
D. Passive drainage of the pleural cavity
E. Active thoracostomy
ANSWER: D

Patient G., 37 years old, was admitted to hospital with complaints of pain in the
right upper quadrant, nausea, general weakness. After additional examination surgeon
conducted laparoscopy. Laparoscopy observation is carried out
A. sequentially from left to right and top to bottom
B. consistently better left to right and top to bottom
C. not consistently better from right to left and from top to bottom
D. not consistently better from right to left
E. consistently better from top to bottom
ANSWER: A

Patient '47 treated in thoracic surgery on an abscess of the right lung. Available
respiratory failure. In the I degree of dyspnea the respiratory failure is:
A. under load
B. in calm state
C. constant;
D. in horizontal body position
E. in upright body position
ANSWER: A

Patient '59 for a long time suffers from the lung abscess, available repeated
pulmonary hemorrhages. Repeated pulmonary hemorrhage in chronic abscesses is treated:
A. Blood Transfusion
B. transfusion of blood products
C. Surgically
D. input coagulants
E. entering antycoagulants
ANSWER: C

Patient 23 years old held X-ray with contrast. Diagnosed esophageal diverticulum.
Radiographic evidence of esophageal diverticulum is:
A. symptom of "wet bag"
B. symptom of bell
C. symptom of "shoe laces "
D. obtuse angle of His;
E. lack of gas bubble of the stomach
ANSWER: A

Patient 28 years old, accidentally drank acid solution. Got burns of the esophagus.
First aid for burns of the esophagus by concentrated acid (gastric lavage):
A. 0,25% solution of novocaine
B. 0,9% solution of sodium chlorid
C. 5% glucose solution
D. 2% sodium bicarbonate
E. 1: 1000 solution of potassium manganese
ANSWER: D

Patient 37 years, the clinic delivered in 50 minutes. after receiving stab wounds, the
wound is located in the heart area. The patient is pale, sharply inhibited, BP 60/20
mm Hg. century. expanded the boundaries of the heart, auscultation - warm tone deaf.
Your actions?
A. Conducting intensive antishock therapy
B. Emergency thoracotomy with excision and suture haemopericardium heart.
C. Conducting intensive antishock therapy followed by surgery
D. Conduct antishock therapy with simultaneous execution thoracoscopy
E. X-ray of the chest
ANSWER: B

Patient 40 years is treated within two weeks due to an acute abscess of the upper
right lung. The treatment: antibiotics, sulfanilamide preparations, vitamin, calcium
chloride, infusion therapy. However, the patient continued the high fever, cough it
bother with the periodic discharge of large number of purulent sputum. What treatment
can effectively complement the rehabilitation of an abscess?
A. Microtracheostomy
B. Tracheostomy
C. Therapeutic bronchoscopy
D. Postural drainage
E. Drainage of pleural cavity
ANSWER: A

Patient 48 years old was hospitalized in thoracic surgery with a diagnosis of Zenker
diverticulum. Being prepared for surgery. Access for Zenker diverticulum?
A. cervical access;
B. lateral thoracotomy in the third intercostal space
C. lateral thoracotomy in the fourth intercostal space
D. lateral thoracotomy in the seventh intercostal space
E. Upper midline laparotomy
ANSWER: A

Patient 58 years old after the X-ray contrast the epiphrenal diverticulum was
diagnosed. The proposed surgery. Access to the epiphrenal diverticulum?
A. right-lateral thoracotomy in IV intercostal space;
B. left-sided lateral thoracotomy in IV intercostal space; l
C. eft-sided thoracotomy in the lateral VII intercostal space;
D. right-lateral thoracotomy in VII intercostal space;
E. Upper midline laparotomy.
ANSWER: A

Patient 62 years old, who chronically abused ba alcohol delivered to the thoracic
department with a diagnosis of esophageal burns. The need for gastrostomy in case of
burns of the esophagus occurs when:
A. esophageal perforation
B. First degree;
C. Second degree;
D. the third degree;
E. burns of the mouth.
ANSWER: A

Patient complains of discomfort behind the breastbone. Available symptom of ‘wet bag’.
Symptom "wet bag" is characteristic for:
A. diverticulum of the esophagus
B. sliding hiatal hernia
C. paraesophageal hiatal hernia
D. iatrogenic perforation of the esophagus
E. relaxation of the diaphragm
ANSWER: A

Patient K., 30 years old, entered the hospital with complaints of pain in the right
upper quadrant, nausea, general weakness. Additional examination found a previous
diagnosis of bile-stone disease. Laparoscopy was prescribed. Gallbladder has smooth
surface, white with blue color, enlarged and stretched. In this case surgeon has to
conduct
A. Laparoscopic cholecystectomy
B. Complete diagnostic laparoscopy
C. open cholecystectomy
D. Cholecystostomy
E. None of the listed
ANSWER: A

Patient K. '34 was hospitalized in serious condition with a wound to the left of the
chest. Patient's condition is severe, skin pale, his face is bluish-purple, swollen
neck veins. Percussion notes expanding the boundaries of the heart, auscultation -
voiceless heart tones. Your diagnosis?
A. Injury to the heart with the development of tamponade
B. Injuries of the left lung with the development of hemopleura
C. Injury of the left lung with the development of pneumothorax
D. Injury of the left lung with the development of stump-hemopleura
E. Injury of chest, bleeding from intercostal arteries
ANSWER: A

Patient of 64 years of age was hospitalized in the department of thoracic surgery


patient with a second period of lung abscess. The second period of lung abscess is
characterized by:
A. increase toxicity
B. deterioration of the general condition of the patien
C. fever
D. allocating a significant amount of pus
E. hectic body temperature
ANSWER: A

Patient S., 27 years old, went to hospital in serious condition, 50 minutes after
receiving penetrating wounds of the chest to the left. OBJECTIVE: consciousness -
stupor, pale skin, acrocyanosis. Pulse 120 beats per minute, poor volume and tension.
BP - mm 80/40 Cardiac tone is deaf, dramatically expanded boundaries. In the III
intercostal space on the left parasternal line is the stab wound. On plain film chest
expansion is defined shadows and smoothing the waist of the heart, left hemothorax to
V edges. What is the most possible reason of severity of the patient?
A. cardiac tamponade
B. Acute heart failure
C. cardiac arrhythmias
D. loss
E. hemothorax and acute respiratory failure.
ANSWER: A

Patient, taken after the accident, complaining of pain in the left chest, cough,
hemoptysis, dyspnea. Subcutaneous emphysema, cyanosis of the skin, heart rate 98 per 1
min. Auscultation - breath weakened in the left. Rib hull damage is not detected,
radiographically observed air strip along the trachea, left-sided pneumothorax. Your
diagnosis?
A. Rupture of esophagus
B. Lung abscess with perforation
C. Rupture of the left main bronchus
D. Rupture of the left dome of the diaphragm
E. Pneumoempyema
ANSWER: C

The patient complains on heartburn, feeling of bitterness in the mouth. Diagnosed


reflux esophagitis. Reflux esophagitis is a manifestation of:
A. diverticulum of the esophagus
B. paraesophageal hiatal hernia
C. iatrogenic perforation of the esophagus
D. Relaxation of the diaphragm
E. sliding hiatal hernia
ANSWER: E

The patient delivered unconscious. Skin and mucous membranes are pale, cyanotic,
breathing is weakened, in the lower regions does not listened, at the level of 6 rib
on the anterior axillary line there is a wound hole with moderate bleeding and passage
of air to insufficiently. Radiological findings: a bullet in the pleural cavity. What
medical tactic?
A. Emergency thoracotomy
B. Thoracoscopy with removal of bullet
C. Transfer valve into open pneumothorax
D. Drainage of pleural cavity
E. Tosca bandage on a wound
ANSWER: A

The patient is being treated in the thoracic surgery on empyema. existing respiratory
failure. The number of respiratory movements during the first stage of respiratory
failure (choose the wrong answer):
A. 14-15
B. 16-18
C. 19-20
D. 23-24
E. 21-22
ANSWER: D

The patient of '54 complains on poor passage of food through the esophagus. The last
two years saw the neck protrusion in the left after eating, vomiting with food. He
began to lose weight. At night, during sleep there is cough. When X-ray contrast study
of the esophagus at the level of the collarbone appears a depot of barium, size and
shape of an egg. What is the most likely diagnosis?
A. Esophagic-tracheal fistula
B. Cancer of the esophagus
C. diverticulum of the esophagus
D. Stenosis
E. Esophagic spasm
ANSWER: C

The patient received a wound in the chest spot in the projection of the heart.
Patient's condition is serious, complaining of shortness of breath, a tendency to
hypotension, muffled heart tones. The boundaries of the heart with percussion extended
to the left. In Plain radiography of the chest cavity – expanding of hearts shadow to
the left, reducing the oscillation amplitude contour of the heart. According to
electrocardiography - reduced voltage complex QRS. Set the diagnosis?
A. Pericardial tamponade
B. Pneumonia
C. Myocardial infarction
D. Pneumothorax
E. Hemothorax
ANSWER: A

To the toracal surgery was hospitalised a patient with suspected pulmonary gangrene.
When percussion in case of gangrene of lung is determined:
A. clear lung sounds
B. The box sound
C. wide area of dull sound
D. tympanic sound
E. The narrow plot of dull sound
ANSWER: C

Patient K., 50 years old, entered the hospital with complaints of pain in the left
upper quadrant, nausea, general weakness. After additional examination for treatment
of pancreatitis laparoscopy was appointed. During conducting laparoscopic drainage of
the abdominal cavity of the patient with acute pancreatitis, port for laparoscope
typically is placed
A. In the periumbilical area
B. On the midline of the abdomen in the epigastrium
C. On the midline of the abdomen in hypogastrium
D. In mesogastrium on the affected side
E. At Volkovych- Kocher point
ANSWER: A

Patient K., 50 years old, entered the hospital with complaints of pain in the right
upper quadrant, nausea, general weakness. After additional examination for traetment
for acute pancreatitis laparoscopy was appointed. For laparoscopy for the treatment
of acute pancreatitis, the second port input is carried out
A. For midline epigastrium
B. In mesogastrium 1 cm above or below the navel the median line
C. On the midline of the abdomen in hypogastrium 10 cm below the navel
D. At McBurney point
E. At Mayo point
ANSWER: A

Patient K., 50 years old, entered the hospital with complaints of pain in the right
upper quadrant, nausea, general weakness. After additional examination laparoscopy for
treatment was appointed. Which among these is the final stage in laparoscopic
cholecystectomy?
A. subserose extraction of gallbladder
B. ligation of cystic duct and artery
C. removal of the gall bladder
D. Removal of calculus
E. suturing the gallbladder bed
ANSWER: A

Patient K., 50 years old, entered the hospital with complaints of pain in the left and
right upper quadrant, nausea, general weakness. The laparoscopy was appointed. During
conducting diagnostic laparoscopy mixed biliary pancreatic necrosis was diagnosed
characterized by
A. all specified
B. presence of "stearic plaques"
C. presence of hemorrhagic fluid
D. dark red, brown or black pancreas
E. visual picture of acute cholecystitis
ANSWER: A

Patient K., 50 years old, entered the hospital with complaints of pain in the left
upper quadrant, nausea, general weakness. The laparoscopy was appointed. Hemorrhagic
necrotizing pancreatitis is characterized by
A. presence of hemorrhagic fluid in the free abdominal cavity
B. presence of scattered on the surface of large and small omentum or elsewhere
"stearic plaques"
C. dark red, brown or black pancreatic cancer, the presence of hemorrhagic fluid in
the free abdominal cavity, the presence of "stearic plaques"
D. visual picture of acute cholecystitis plus characteristic signs of inflammation of
the pancreas
E. There is no specific sign
ANSWER: A

Patient K., 50 years old, entered the hospital with complaints of pain in the right
upper quadrant, nausea, general weakness. Laparoscopy was appointed. Streched
enlarged gallbladder characterizes
A. acute simple cholecystitis
B. normal gallbladder
C. acute gangrenous cholecystitis
D. acute gangrenous perforated cholecystitis
E. perforated cholecystitis
ANSWER: A

Patient K., 54 years old, came to the department complaining of pain in the right
upper quadrant, nausea, general weakness. After additional examination laparoscopy for
treatment was appointed. During conducting therapeutic laparoscopy for acute
obstructive cholecystitis, operational port is carried out
A. Epigastrium 2-3 cm below the xiphoid processus
B. At Mayo point
C. On the midline of the abdomen in hypogastrium
D. At McBurney point
E. At Volkovych- Kocher point
ANSWER: A

Patient K., 54 years old, entered the department complaining of pain in the right
upper quadrant, nausea, general weakness. After additional examination laparoscopy for
treatment was appointed. Retrograde cholecystectomy is performed in one of the
following cases:
A. In the presence of inflammatory infiltrate in the cervical region of the
gallbladder
B. In elderly patients
C. In the presence of phenomena of cholangitis
D. When contracted gallbladder
E. When stone in cervix of the gallbladder
ANSWER: A

Patient K., 54 years old, entered the department complaining of pain in the left upper
quadrant, nausea, general weakness. Was laparoscopy was appointed. During conducting
diagnostic laparoscopy focal fatty pancreatic necrosis was diagnosed characterized by
A. presence of "stearic plaques"
B. presence of hemorrhagic fluid
C. dark red, brown or black pancreas
D. visual picture of acute cholecystitis
E. gland lobules are displayed clearly
ANSWER: A

Patient K., 54 years old, entered the department complaining of pain in the left upper
quadrant, nausea, general weakness. The laparoscopy was appointed. During conducting
diagnostic laparoscopy scattering on the surface of large and small omentum or
elsewhere of "stearic plaques" indicates
A. Fatty pancreatic necrosis
B. Hemorrhagic necrotizing pancreatitis
C. Mixed pancreatic necrosis
D. Biliary pancreatitis
E. Intact pancreas
ANSWER: A

Patient K., 70 years old, entered the hospital with complaints of pain in the right
upper quadrant, nausea, general weakness. It has a pronounced respiratory
failure. Additional examination found a previous diagnosis of bile-stone
disease. Laparoscopy was prescribed. Gallbladder has smooth surface, white with blue
color, enlarged and stretched. One of the complications of acute cholecystitis was
diagnosed. It is
A. Edema of the gallbladder
B. Empyema of the gallbladder
C. Chronic calculous cholecystitis
D. Cancer of the gallbladder
E. None of the listed
ANSWER: A

Patient K., 74 years old, entered the department with complaints of pain in the left
upper quadrant, nausea, general weakness. The laparoscopy was appointed. Acute
biliary pancreatitis in a patient is characterized by
A. visual picture of acute cholecystitis accompanying characteristic signs of
inflammation of the pancreas
B. presence of scattered on the surface of large and small omentum or elsewhere
"stearic plaques"
C. presence of hemorrhagic fluid in the free abdominal cavity
D. dark red, brown or black pancreatic cancer, the presence of hemorrhagic fluid in
the free abdominal cavity, the presence of "stearic plaques"
E. There is no specific sign
ANSWER: A

Patient K., 74 years old, entered the department with complaints of pain in the right
upper quadrant, nausea, general weakness. Llaparoscopy was appointed. In acute
gangrenous perforated cholecystitis, characteristic appearance of the gallbladder is
following
A. the presence of black spots with bile contents in the free abdominal cavity
B. the presence of black spots with fibrinous layers
C. hyperemia on the background, fibrinous layerings can be observed
D. enlarged stretched gallbladder
E. omentum tightly wrapped around gallbladder, as a result-poor visualization
ANSWER: A

Patient N., 40 years old, was admitted to hospital complaining of marked pain in the
right upper quadrant, nausea, general weakness. After additional examination surgeon
conducted laparoscopy. The characteristic appearance of the gallbladder at perforated
cholecystitis is following
A. omentum tightly wrapped around gallbladder, as a result-poor visualization
B. the presence of black spots with bile contents in the free abdominal cavity
C. the presence of black spots with fibrinous layers
D. hyperemia on the background, fibrinous layerings can be observed
E. enlarged stretched gallbladder
ANSWER: A

Patient N., 40 years old, was admitted to the hospital complaining of pain in the
right upper quadrant, nausea, general weakness. After additional examination
laparoscopy for treatment was appointed. In conducting laparoscopic cholecystectomy,
port for laparoscope is typically placed
A. In the periumbilical area
B. On the midline of the abdomen in the epigastrium
C. On the midline of the abdomen in hypogastrium
D. In mesogastrium on the affected side
E. At Volkovych- Kocher point
ANSWER: A

Patient N., 40 years old, was admitted to the hospital complaining of pain in the
right upper quadrant, nausea, general weakness. After additional examination
laparoscopy for treatment was appointed. For diagnosis of uncomplicated cholelithiasis
surgeon should prefer:
A. Ultrasonography
B. Endoscopic retrograde cholangiopancreatography
C. Laparoscopy
D. Percutaneous cholangiography
E. Fractional duodenal intubation
ANSWER: A
Patient N., 40 years old, was admitted to the hospital complaining of pain in the
right upper quadrant, nausea, general weakness. After additional examination
laparoscopy for treatment was appointed. Stages of laparoscopic cholecystostomy are:
A. puncture of the gallbladder, cholecystostomy, fixing drainage tube into the gall
bladder and the parietal peritoneum
B. blending metal clip on cystic duct, recutting artery, suturing the gallbladder bed
C. ligation of cystic duct, gallbladder removal, drainage of subhepatic space
D. allocation subserously , gallbladder puncture of the gallbladder, cystic duct
transection
E. puncture of the gallbladder, holetsystostomy
ANSWER: A

Patient N., 40 years old, was admitted to the hospital complaining of pain in the left
and right upper quadrant, nausea, general weakness. After additional examination
surgeon conducted laparoscopy. During conducting diagnostic laparoscopy with a picture
of acute cholecystitis there are signs of inflammation of the pancreas that occurs in
this state as
A. Biliary acute pancreatitis
B. Focal necrotizing pancreatitis
C. Focal hemorrhagic pancreatitis
D. Mixed pancreatic necrosis
E. Intact pancreas
ANSWER: A

Patient N., 40 years old, was admitted to the hospital complaining of pain in the
right upper quadrant, nausea, general weakness. After additional examination doctor
conducted laparoscopy. Acute catharral cholecystitis appearance is characterized by
A. enlarged stretched gallbladder
B. hyperemia on the background, fibrinous layering can be observed
C. the presence of black spots with fibrinous layers
D. the presence of black spots with bile contents in the free abdominal cavity
E. omentum tightly wrapped around gallbladder, as a result-poor visualization
ANSWER: A

Patient S., 57 years old, entered the department with complaints of pain in the left
upper quadrant, nausea, general weakness. After additional examination surgeon
conducted laparoscopy. Mixed necrotizing pancreatitis in a patient is characterized by
A. dark red, brown or black pancreatic cancer, the presence of hemorrhagic fluid in
the free abdominal cavity, the presence of "stearic plaques"
B. presence of scattered on the surface of large and small omentum or elsewhere
"stearic plaques"
C. presence of hemorrhagic fluid in the free abdominal cavity
D. visual picture of acute cholecystitis andcharacteristic signs of inflammation of
the pancreas
E. There is no specific sign
ANSWER: A
Patient S., 57 years old, entered the department with complaints of pain in the right
upper quadrant, nausea, general weakness. After additional examination surgeon
conducted laparoscopy. In acute gangrenous cholecystitis gallbladder looks as
A. the presence of black spots with fibrinous layers
B. hyperemia on the background, fibrinous layerings can be observed
C. enlarged stretched gallbladder
D. the presence of black spots with bile contents in the free abdominal cavity
E. omentum tightly wrapped around gallbladder, as a result-poor visualization
ANSWER: A

The patient K., 40 yrs.old is undergoing GERD surgery. At Toupet surgery


A. Place the camera above the umbilicus, one third of the distance to the xiphoid
process.
B. Place the camera above the umbilicus, one half of the distance to the xiphoid
process.
C. Place the camera under the umbilicus, one third of the distance to the pubis.
D. Place the camera above the xiphoid process.
E. Place the camera aside the umbilicus.
ANSWER: A

The patient K., 40 yrs.old is undergoing GERD surgery. At Toupet surgery


A. Place two lateral retracting ports in the right and left anterior
axillary lines, respectively.
B. Place two lateral retracting ports in the right and left posterior
axillary lines, respectively.
C. Place two lateral retracting ports in the right and left middle axillary
lines, respectively.
D. Place two lateral retracting ports in the right and left clavicular
lines, respectively.
E. Place two lateral retracting ports in the right and left pararectal
lines, respectively.
ANSWER: A

The patient K., 40 yrs.old is undergoing GERD surgery. At Toupet surgery one uses 30%
to 45% of reverse Trendelenburg to displace the transverse colon and small bowel
inferiorly,
A. keeping them from obstructing the view of the video camera
B. keeping them from obstructing the view of the hiatus
C. keeping them from obstructing the view of liver
D. keeping them from obstructing the view of the stomach
E. it is traditional
ANSWER: A

The patient K., 40 yrs.old is undergoing GERD surgery. Laparoscopic fundoplication is


indicated for
A. the treatment of objectively documented, relatively severe gastroesophageal reflux
disease
B. the treatment of relatively severe gastroesophageal reflux disease
C. the treatment of appendicitis
D. the treatment of objectively severe gastroesophageal reflux disease
E. the treatment of reflux
ANSWER: A

The patient K., 40 yrs.old is undergoing GERD surgery. Patient position at GERD
surgery is…
A. patient is placed supine, the knees only slightly ?exed
B. patient is placed supine with the head elevated 75 degrees
C. patient is placed with the head elevated 15 degrees in the modi?ed lithotomy
position
D. patient is placed with the head elevated 45 degrees, the knees only slightly
extended
E. patient is placed
ANSWER: A

The patient K., 40 yrs.old is undergoing GERD surgery. Patient position at Nissen
procedure is…
A. patient is placed supine, the knees only slightly ?exed
B. patient is placed supine with the head elevated 75 degrees
C. patient is placed with the head elevated 15 degrees in the modi?ed lithotomy
position
D. patient is placed with the head elevated 45 degrees, the knees only slightly
extended
E. patient is placed
ANSWER: A

The patient K., 40 yrs.old is undergoing GERD surgery. Patients with gastroesophageal
re?ux may be considered candidates for the procedure of laparoscopic fundoplication at:
A. Esophageal complications such as erosive esophagitis, stricture, and/or Barrett’s
esophagus
B. Non-Respiratory complications
C. Dependence upon diet for relief of symptoms
D. Laryngeal symptoms with a good response to diet
E. None of the listed
ANSWER: A

The patient K., 40 yrs.old is undergoing GERD surgery. Patients with gastroesophageal
re?ux may be considered candidates for the procedure of laparoscopic fundoplication at:
A. Esophageal stricture
B. Non-Respiratory complications
C. Dependence upon diet for relief of symptoms
D. Laryngeal symptoms with a good response to diet
E. None of the listed
ANSWER: A

The patient K., 40 yrs.old is undergoing GERD surgery. Patients with gastroesophageal
re?ux may be considered candidates for the procedure of laparoscopic fundoplication at:
A. Respiratory complication such as bronchiectasis
B. Non-erosive esophagitis
C. Dependence upon diet for relief of symptoms
D. Laryngeal symptoms with a good response to diet
E. None of the listed
ANSWER: A

The patient K., 40 yrs.old is undergoing GERD surgery. Patients with gastroesophageal
re?ux may be considered candidates for the procedure of laparoscopic fundoplication at:
A. Dependence upon proton pump inhibitors (PPIs) for relief of symptoms, particularly
if dose escalation is required
B. Non-erosive esophagitis
C. Dependence upon diet for relief of symptoms
D. Laryngeal symptoms with a good response to diet
E. None of the listed
ANSWER: A

The patient K., 40 yrs.old is undergoing GERD surgery. The surgeon stands at Nissen
fundoplication
A. between the patient’s legs
B. to the right of the patient
C. to the left of the patient
D. either side of the patient
E. at the patient’s head
ANSWER: A

The patient K., 40 yrs.old is undergoing GERD surgery. The surgeon stands at Toupet
fundoplication
A. between the patient’s legs
B. to the right of the patient
C. to the left of the patient
D. either side of the patient
E. at the patient’s head
ANSWER: A

The patient K., 40 yrs.old is undergoing GERD surgery. The surgeon stands at GERD
surgery…
A. The surgeon stands between the legs and works with both hands. This allows the
rightand left-handed instruments to approach the hiatus from the respective upper
abdominal quadrants.
B. The surgeon stands between the legs and works with right hand.
C. The surgeon stands between the legs and works with left hand. This allows the
right and left-handed instruments to approach the hiatus from the respective upper
abdominal quadrants.
D. The surgeon stands aside the patient and works with both hands. This allows the
rightand left-handed instruments to approach the hiatus from the respective upper
abdominal quadrants.
E. The surgeon stands between the legs and works with left hand. This allows the
left-handed instruments to approach the hiatus.
ANSWER: A
The patient K., 40 yrs.old is undergoing GERD surgery. The workup for laparoscopic
treatment of GERD consists of one of the following stages.
A. 24-hour pH monitoring
B. Manometric ultrasound
C. Assessment of esophageal width
D. Selection of a partial fundoplication technique
E. None of the listed
ANSWER: A

The patient K., 40 yrs.old is undergoing GERD surgery. The workup for laparoscopic
treatment of GERD consists of one of the following stages.
A. Selection of a partial versus complete fundoplication
B. Manometric ultrasound
C. Assessment of esophageal width
D. Selection of a partial fundoplication technique
E. None of the listed
ANSWER: A
Where is located the second anatomical narrowing of esophagus?
A. The site of crossing with left bronchus
B. The site of pharyngoesophageal junction
C. The site of crossing with aorta
D. The site of passing through diaphragm
E. The site of cardia
ANSWER: A

The site of passing through diaphragm is:


A. The third anatomical narrowing of esophagus
B. The first anatomical narrowing of esophagus
C. The second anatomical narrowing of esophagus
D. The first physiological narrowing of esophagus
E. The second physiological narrowing of esophagus
ANSWER: A

The site of cardia is:


A. The second physiological narrowing of esophagus
B. The first anatomical narrowing of esophagus
C. The second anatomical narrowing of esophagus
D. The third anatomical narrowing of esophagus
E. The first physiological narrowing of esophagus
ANSWER: A

What is the mechanism of formation of pulsion diverticula?


A. Herniation of the esophageal wall proximal to anatomical narrowing
B. Inflammatory changes of paraesophageal tissues
C. Dilatation of esophagus caused by achalasia
D. Changes of esophagus caused by chemical burns
E. Changes of esophagus caused by reflux esophagitis
ANSWER: A

What is the mechanism of formation of traction diverticula?


A. Inflammatory changes of paraesophageal tissues
B. Herniation of the esophageal wall proximal to anatomical narrowing
C. Dilatation of esophagus caused by achalasia
D. Changes of esophagus caused by chemical burns
E. Changes of esophagus caused by reflux esophagitis
ANSWER: A

What can bifurcational diverticula result in?


A. Esophago-bronchial fistula with aspiration pneumonia
B. Signs of achalasia
C. Cyanosis of the upper part of body
D. Compressible mass on the left side of the neck
E. Coarctation of aorta
ANSWER: A

What is the main objective manifestation of epiphrenal diverticula?


A. Signs of achalasia
B. Cyanosis of the upper part of body
C. Compressible mass on the left side of the neck
D. Esophago-bronchial fistula with aspiration pneumonia
E. Lung atelectasis
ANSWER: A

What is the main method of diagnostic of esophageal diverticula?


A. Esophagogastroduodenoscopy
B. Pleural punctere
C. Ultrasound examination
D. Plain X-ray examination of the chest
E. Irrigoscopy
ANSWER: A

What operation is performed in esophageal diverticula?


A. Resection of diverticulum
B. Esophagomyotomy
C. Esophagogastric anastomosis
D. Extirpation of esophagus
E. Esophageal plastics by intestine
ANSWER: A

What surgical access should be applied for Zenker's diverticula?


A. Cervical access along the anterior border of the sternocleidomastoid muscle
B. Left-sided anterolateral thoracotomy in ІV intercostal space
C. Right-sided posterolateral thoracotomy in ІV intercostal space
D. Left-sided posterolateral thoracotomy in VІІ intercostal space
E. Right-sided anterolateral thoracotomy in VІІ intercostal space
ANSWER: A

What surgical access should be applied for bifurcational diverticula?


A. Right-sided posterolateral thoracotomy in ІV intercostal space
B. Cervical access along the anterior border of the sternocleidomastoid muscle
C. Left-sided anterolateral thoracotomy in ІV intercostal space
D. Left-sided posterolateral thoracotomy in VІІ intercostal space
E. Right-sided anterolateral thoracotomy in VІІ intercostal space
ANSWER: A

What surgical access should be applied for epiphrenal diverticula?


A. Left-sided posterolateral thoracotomy in VІІ intercostal space
B. Cervical access along the anterior border of the sternocleidomastoid muscle
C. Left-sided anterolateral thoracotomy in ІV intercostal space
D. Right-sided posterolateral thoracotomy in ІV intercostal space
E. Right-sided anterolateral thoracotomy in VІІ intercostal space
ANSWER: A

What is the characteristic feature of achalasia of the cardia?


A. Failure of the lower esophageal sphincter to relax
B. Spasm of the lower esophageal sphincter
C. Cicatrical changes after the burn
D. Anorexia
E. Esophageal gaping
ANSWER: A

What is the characteristic feature of the II stage of achalasia?


A. Constant spasm with a moderate esophageal dilation and maintained peristalsis
B. Asymptomatic
C. Functional spasm without esophageal dilation
D. Cicatrical changes with expressed esophageal dilation, the peristalsis is absent
E. Considerable esophageal dilation with S-shaped elongation.
ANSWER: A

For the clinical manifestation of esophageal achalasia is typical:


A. Dysphagia
B. Dyspnea
C. Cyanosis of the upper part of body
D. Retention of stool and gases
E. Vomiting by "coffee masses"
ANSWER: A

For the clinical manifestation of esophageal achalasia is typical:


A. Loss of weight
B. Dyspnea
C. Cyanosis of the upper part of body
D. Retention of stool and gases
E. Vomiting by "coffee masses"
ANSWER: A

What does dysphagia mean?


A. Disturbances of swallowing
B. Pain behind breastbone
C. Absence of appetite
D. Esophageal vomiting
E. Vomiting with blood
ANSWER: A

What is the I stage of dysphagia?


A. Disturbances of solid food passage
B. Asymptomatic
C. Disturbances of semisolid food passage
D. Disturbances of liquid food passage
E. No passage of food
ANSWER: A

What is the II stage of dysphagia?


A. Disturbances of semisolid food passage
B. Asymptomatic
C. Disturbances of solid food passage
D. Disturbances of liquid food passage
E. No passage of food
ANSWER: A

What is the I stage of morphological changes of esophageal burns?


A. Stage of acute esophagitis
B. Asymptomatic
C. Stage of chronic esophagitis
D. Stage of cicatrical stricture of esophagus
E. Stage of late complications
ANSWER: A

What is the II stage of morphological changes of esophageal burns?


A. Stage of chronic esophagitis
B. Asymptomatic
C. Stage of acute esophagitis
D. Stage of cicatrical stricture of esophagus
E. Stage of late complications
ANSWER: A

What is the III stage of morphological changes of esophageal burns?


A. Stage of cicatrical stricture of esophagus
B. Asymptomatic
C. Stage of acute esophagitis
D. Stage of chronic esophagitis
E. Stage of late complications
ANSWER: A

What is the I degree of esophageal burns?


A. Superficial burn with the damage of epithelial layer of esophagus;
B. The burn with the damage of entire mucosa of esophagus;
C. The burn damage of all layers of esophagus;
D. The spread of postburn necrosis on paraesophageal tissue and adjacent organs.
E. Asymptomatic
ANSWER: A

What is the III degree of esophageal burns?


A. The burn damage of all layers of esophagus;
B. Superficial burn with the damage of epithelial layer of esophagus;
C. The burn with the damage of entire mucosa of esophagus;
D. The spread of postburn necrosis on paraesophageal tissue and adjacent organs.
E. Asymptomatic
ANSWER: A

What is the IV degree of esophageal burns?


A. The spread of postburn necrosis on paraesophageal tissue and adjacent organs.
B. Superficial burn with the damage of epithelial layer of esophagus;
C. The burn with the damage of entire mucosa of esophagus;
D. The burn damage of all layers of esophagus;
E. Asymptomatic
ANSWER: A

What is the roentgenological sign of the esophageal burn of mild degree?


A. Free passage of barium with maintained peristalsis
B. Filling defects without peristalsis
C. "Rat tail" sign with stagnation of barium
D. "Bird-beak" sign without peristalsis
E. Dilated esophagus with sites of constriction and weak peristalsis
ANSWER: A

What is the main clinical manifestation of the esophageal burn of severe degree?
A. Clinic of shock
B. Clinic of reflux-esophagitis
C. Clinic of acute abdomen
D. Clinic of hepatic insufficiency
E. Clinic of respiratory insufficiency
ANSWER: A

What solution is used for washing out of acid esophageal burn?


A. Sodium hydrocarbonatis solution
B. Antiseptic solution
C. Antibiotic solution
D. Glucose solution
E. Vinegar solution
ANSWER: A

For the clinical manifestation of sliding diaphragmatic hernia is typical:


A. Heartburn
B. Dyspnea
C. Cyanosis of the upper part of body
D. Retention of stool and gases
E. Vomiting by "coffee masses"
ANSWER: A

For the clinical manifestation of sliding diaphragmatic hernia is typical:


A. Regurgitation
B. Dyspnea
C. Cyanosis of the upper part of body
D. Retention of stool and gases
E. Vomiting by "coffee masses"
ANSWER: A

What type of operation is used for paraesophageal diaphragmatic hernia?


A. Cruroplasty
B. Esophagostomy
C. Resection of the esophagus
D. Resection of the stomach
E. Cruroplasty with Nissen's fundoplication
ANSWER: A

What type of operation is used for sliding diaphragmatic hernia?


A. Cruroplasty with Nissen's fundoplication
B. Esophagostomy
C. Cruroplasty
D. Resection of the esophagus
E. Resection of the stomach
ANSWER: A

What is the roentgenological sign of acute mediastinitis?


A. Widening of mediastinum, shadowing of its anterior
B. Filling defect
C. The sign of "bell"
D. Lack of air bubble of the stomach
E. High standing of diaphragmatic dome
ANSWER: A

What sign is typical for anterior mediastinitis?


A. Intensifying of pain when head is unbent back
B. Throbbing chest pain with irradiation in interscapular region
C. Intensifying of pain at swallowing
D. Swelling above clavicle
E. Sign of compression of azygos and hemiazygos veins
ANSWER: A

What sign is typical for anterior mediastinitis?


A. Signs of compression of superior vena cava
B. Throbbing chest pain with irradiation in interscapular region
C. Intensifying of pain at swallowing
D. Swelling above clavicle
E. Sign of compression of azygos and hemiazygos veins
ANSWER: A

What sign is typical for posterior mediastinitis?


A. Throbbing chest pain with irradiation in interscapular region
B. Intensifying of pain during percussion of breast bone
C. Intensifying of pain when head is unbent back
D. Occurrence of swelling in the region of jugular fossa
E. Signs of compression of superior vena cava
ANSWER: A

What sign is typical for posterior mediastinitis?


A. Intensifying of pain during vertebral pressing
B. Intensifying of pain during percussion of breast bone
C. Intensifying of pain when head is unbent back
D. Occurrence of swelling in the region of jugular fossa
E. Signs of compression of superior vena cava
ANSWER: A

The leading signs in acute intestinal obstruction are:


A. Wave-like pain, vomiting, delay of gases and stool
B. „Knife-like” pain, wooden abdomen, proper anamnesis
C. „Knife-like” pain, wooden abdomen, vomiting
D. Wave-like pain, anaemia
E. Nausea, loss of appetite, metallic taste in the mouth
ANSWER: A

The Sklyarov's sign in acute intestinal obstruction is:


A. Noise of splash
B. Good heard cardiac tones during auscultation of the abdomen
C. Dullness in the lower regions
D. Sound of falling drop
E. Gaping of anus
ANSWER: A

The Grekov's sign in acute intestinal obstruction is:


A. Gaping of anus
B. Good heard cardiac tones during auscultation of the abdomen
C. Dullness in the lower regions
D. Sound of falling drop
E. Noise of splash
ANSWER: A

In acute intestinal obstruction the basic X-ray sign is:


A. Air-fluid levels, Kloiber's cups
B. Expressed limitation of mobility of the right dome of diaphragm
C. Diffusely dilated loops of bowels
D. Free gas in the abdomen
E. Sklyarov's sign
ANSWER: A

What are the Kloiber's cups?


A. Horizontal air-fluid levels
B. Gas bubble of the stomach
C. Folds of intestine
D. Gas sickles under the domes of diaphragm
E. None of mentioned
ANSWER: A

What does not belong to conservative therapy of acute intestinal obstruction?


A. Liquidation of hypervolemia
B. Decompression of gastrointestinal tract
C. The struggle against abdominal-pain shock
D. Detoxication
E. Correction of microcirculation
ANSWER: A
What does not belong to the fight against abdominal-pain shock?
A. Performing of siphon enema
B. Paranephral novocaine blockade
C. Neuroleptanalgesia
D. Peridural anaesthesia
E. Spasmolytic therapy
ANSWER: A

The decompression of gastrointestinal tract includess everything, except:


A. Lavage of abdominal cavity
B. Endoscopic intubation
C. Enterotomy with aspiration
D. Washing of the stomach
E. Performing of siphon enema
ANSWER: A

The criteria of the efficiency of gastrointestinal tract passage renewal during


conservative therapy of acute intestinal obstruction is:
A. Pulling of gases and stool
B. Normalization of rectal temperature
C. Absence of Shchotkin-Blumberg's sign
D. Feeling of heartburn
E. None of mentioned
ANSWER: A

To the criteria of permanent renewal of the gastrointestinal tract passage as


efficiency of conservative treatment belongs:
A. Absence of stagnant content in the stomach
B. Absence of Shchotkin-Blumberg's sign
C. Normalization of rectal temperature
D. Feeling of heartburn
E. None of mentioned
ANSWER: A

The indication for operative treatment of acute intestinal obstruction is:


A. Mechanical acute intestinal obstruction in inefficient conservative treatment
B. I phase of the course of acute intestinal obstruction
C. II phase of the course of acute intestinal obstruction
D. The prolonged anamnesis of acute intestinal obstruction
E. Mechanical acute intestinal obstruction
ANSWER: A

Choose the correct algorithm of the operative intervation for the II stage of acute
intestinal obstruction :
A. Laparotomy, liquidation of obstruction, intestinal intubation, sanation of
abdominal cavity, suturing of the abdomen
B. Laparotomy, liquidation of the source of peritonitis, sanation of abdominal cavity,
suturing of the abdomen
C. Laparotomy, liquidation of obstruction, sanation of abdominal cavity, suturing of
the abdomen
D. Laparotomy, liquidation of obstruction, intestinal intubation, sanation of
abdominal cavity, laparostomy
E. Laparotomy, liquidation of obstruction, liquidation of the source of peritonitis,
intestinal intubation, sanation of abdominal cavity, suturing of the abdomen
ANSWER: A

What treatment is indicated in gall-stones intestinal obturation?


A. Only operative
B. Only conservative
C. Operative in the case of the development of peritonitis
D. Treatment is not required
E. Tactic depends on the size of stone
ANSWER: A

What treatment tactic of acute intestinal obstruction, caused by a tumour obturation


is required?
A. Operative intervation
B. Liquidation of tumour by a chemotherapy
C. Liquidations of tumour by radiotherapy
D. Operative intervation only after chemotherapy
E. Only symptomatic treatment
ANSWER: A

The most frequently the sygmoid volvulus arises in:


A. Elderly patients with frequent constipations
B. Females with menstrual arrest
C. Children
D. Elderly patients people with permanent diarrhea
E. New-borns
ANSWER: A

The nodulus requires:


A. Untie the knot, if impossible – resection of the bowel
B. Resection of the bowel
C. Untie the knot
D. To perform the stoma. The second stage the resection of the bowel
E. None of mentioned
ANSWER: A

The contributory factor of the development of obturation is:


A. Stool stones
B. Long intestinal mesentery
C. Adhesions in abdominal cavity
D. All of mentioned
E. None of mentioned
ANSWER: A
What dyspeptic manifestations are typical for acute appendicitis?
A. Single nausea and vomiting
B. Constant vomiting and nausea without any relief
C. Vomiting by bile without any relief
D. Absence of peristalsis
E. Constant diarrhea
ANSWER: A

What does the Rozdolsky’s sign mean?


A. Painfulness in a right iliac area during percussion
B. Pain in right lower quadrant during palpation of left lower quadrant
C. Increase of pain in a right iliac area when the patient lies on the left side
D. Increased pain with coughing
E. Migration of pain to the right iliac area from epigastric
ANSWER: A

Who usually suffer from gangrenous appendicitis?


A. People of old age
B. Newborns
C. Children
D. Pregnant women
E. Young men
ANSWER: B

What does the psoas-sign mean?


A. Pain on extension of right thigh
B. Painfulness during palpation of Petit triangle
C. Migration of pain to the right iliac area from epigastric
D. Tapping of lumbar region cause the pain
E. Increase of pain in a right iliac area when the patient lies on the left side
ANSWER: A

What manifestation is predominant for retroperitoneal appendicitis?


A. Clinic of retroperitoneal phlegmon
B. Clinic of acute abdomen
C. Dyspeptic syndrome
D. Clinic of acute intestinal obstruction
E. Clinic of acute pancreatitis
ANSWER: A

What manifestation is predominant for pelvic appendicitis?


A. Clinic of irritation of pelvic organs (dysuria, pulling rectal pain, tenesmi)
B. Clinic of acute abdomen
C. Clinic of retroperitoneal phlegmon
D. Clinic of acute intestinal obstruction
E. Clinic of acute pancreatitis
ANSWER: A

What objective manifestations are typical for retrocaecal appendicitis?


A. Painfulness of anterior rectal wall and posterior vaginal vault
B. Abdominal distension
C. Absence of hepatic dullness
D. Clinic of retroperitoneal phlegmon
E. Rigidity of anterior abdominal wall
ANSWER: A

What does the Rovsing's sign mean?


A. Pain in right lower quadrant during palpation of left lower quadrant
B. Increase of pain in a right iliac area when the patient lies on the left side
C. The increase of pain intensity during the palpation of right iliac area when the
patient lies on the left side.
D. Increased pain with coughing
E. Migration of pain to the right iliac area from epigastric
ANSWER: A

What does the Dunphy's sign mean?


A. Increased pain with coughing
B. Pain in right lower quadrant during palpation of left lower quadrant
C. Increase of pain in a right iliac area when the patient lies on the left side
D. The increase of pain intensity during the palpation of right iliac area when the
patient lies on the left side.
E. Migration of pain to the right iliac area from epigastric
ANSWER: A

What does the Voskresenky’s sign mean?


A. The increase of pain during quick sliding movements by the tips of fingers from
epigastric to right iliac area.
B. Increased pain with coughing
C. Pain in right lower quadrant during palpation of left lower quadrant
D. Increase of pain in a right iliac area when the patient lies on the left side
E. Migration of pain to the right iliac area from epigastric
ANSWER: A

What does the Yaure-Rozanov sign mean?


A. Painfulness during palpation of Petit triangle
B. Increased pain with coughing
C. Pain in right lower quadrant during palpation of left lower quadrant
D. Increase of pain in a right iliac area when the patient lies on the left side
E. Migration of pain to the right iliac area from epigastric
ANSWER: A

What does the psoas sign mean?


A. Pain on extension of right thigh
B. Increased pain with coughing
C. Pain in right lower quadrant during palpation of left lower quadrant
D. Increase of pain in a right iliac area when the patient lies on the left side
E. Migration of pain to the right iliac area from epigastric
ANSWER: A
The expressed pain in a right lumbar area is typical for:
A. Retrocecal appendicitis
B. Phlegmonous appendicitis
C. Simple appendicitis
D. Left-side appendicitis appendicitis
E. Pelvic appendicitis
ANSWER: A

The dysuria is typical for:


A. Pelvic appendicitis
B. Retrocecal appendicitis
C. Phlegmonous appendicitis
D. Simple appendicitis
E. Left-side appendicitis appendicitis
ANSWER: A

The absence of muscular tenderness is typical for:


A. Pelvic appendicitis
B. Retrocecal appendicitis
C. Phlegmonous appendicitis
D. Simple appendicitis
E. Left-side appendicitis appendicitis
ANSWER: A

The painfulness of anterior rectal wall is typical for:


A. Pelvic appendicitis
B. Retrocecal appendicitis
C. Phlegmonous appendicitis
D. Simple appendicitis
E. Left-side appendicitis appendicitis
ANSWER: A

For the retroperitoneal appendicitis is typical:


A. Flank tenderness in right lower quadrant
B. Peritoneal signs
C. The painfulness of the left iliac region
D. Clinic of irritation of pelvic organs
E. Painfulness of anterior rectal wall and posterior vaginal vault
ANSWER: A

For the left-side appendicitis is typical:


A. The painfulness of the left iliac region
B. Expressed pain in a right lumbar area
C. Flank tenderness in right lower quadrant
D. Clinic of irritation of pelvic organs
E. Painfulness of anterior rectal wall and posterior vaginal vault
ANSWER: A
For the pelvic appendicitis is typical:
A. Clinic of irritation of pelvic organs
B. The painfulness of the left iliac region
C. Expressed pain in a right lumbar area
D. Flank tenderness in right lower quadrant
E. Peritoneal signs
ANSWER: A

For the pelvic appendicitis is typical:


A. Painfulness of anterior rectal wall
B. The painfulness of the left iliac region
C. Expressed pain in a right lumbar area
D. Flank tenderness in right lower quadrant
E. Peritoneal signs
ANSWER: A

For the pelvic appendicitis is typical:


A. Painfulness of posterior vaginal vault
B. The painfulness of the left iliac region
C. Expressed pain in a right lumbar area
D. Flank tenderness in right lower quadrant
E. Peritoneal signs
ANSWER: A

Appendicular infiltrate is treated:


A. antibiotics, paranephral blockade, detoxication therapy
B. antiseptics, analgesia, antibiotics, anti-inflammatory therapy
C. antibiotics, diuretics, antispasmodic, anti-inflammatory therapy
D. analgesia, antibiotics, diuretics, anti-inflammatory therapy
E. anti-inflammatory drugs, paranephral blockade, detoxication therapy
ANSWER: A

The removal of appendix from the base is:


A. Retrograde appendectomy
B. antegrade appendectomy
C. retrocecal appendectomy
D. antececal appendectomy
E. laparoscopic appendectomy
ANSWER: A

Tumour with fluctuation are the main clinical manifestation of:


A. appendicular abscess
B. appendicular peritonitis
C. appendicular infiltrate
D. appendicular mesadenitis
E. appendicular typhlitis
ANSWER: A

The peculiarities of the clinical course of appendicitis in children are caused:


A. by the bailer form of appendix
B. by the tubular form of appendix
C. by hypertrophy of appendix
D. by atrophy of appendix
E. by the spherical form of appendix
ANSWER: A

The changes of clinical manifestations of acute appendicitis in pregnancy are caused


by:
A. distension of anterior abdominal wall by uterus
B. inflammation of uterus
C. irritation of anterior abdominal wall by uterus
D. compression of appendix by uterus
E. inflammation of the right ovarium
ANSWER: A

The changes in clinical manifestation of acute appendicitis in pregnancy is


characterized :
A. by the absence of signs of peritoneal irritation
B. by the presence of signs of peritoneal irritation
C. by the presence of expressed signs of peritoneal irritation
D. by displacement of the signs of peritoneal irritation
E. by the change of the character of signs of peritoneal irritation
ANSWER: A

What form of appendicitis the signs of peritoneal irritation are absent in?
A. chronic
B. calculous
C. perforative
D. appendicular infiltrate
E. appendicular abscess
ANSWER: A

Black colour, fibrino-purulent fur, pus in the lumen are the signs of:
A. gangrenous appendicitis
B. phlegmonous appendicitis
C. catarrhal appendicitis
D. gangreno-perforative appendicitis
E. dystrophic appendicitis
ANSWER: A

Black colour, fibrino-purulent fur, perforation are the signs of:


A. gangreno-perforative appendicitis
B. phlegmonous appendicitis
C. gangrenous appendicitis
D. catarrhal appendicitis
E. dystrophic appendicitis
ANSWER: A
Only during the operation is possible the differential diagnostics of acute
appendicitis with:
A. terminal ileitis
B. renal colic
C. acute pyelonephritis
D. acute paraproctitis
E. acute pancreatitis
ANSWER: A

The distinctive peculiarities of acute appendicitis in the second half of pregnancy


are:
A. Weak express of pain syndrome, similar to the ligamentary tension of uterus
B. Absence of Volkovcha-Kocher's sign
C. Expressed signs of peritoneal irritation
D. The express local muscular tension in a right iliac area
E. Expressed of Obraztsov's sign
ANSWER: A

In the diagnostics of pelvic appendicitis the most valuable is:


A. rectal and vaginal examination
B. laboratory analyses
C. laparocentesis
D. laparoscopy
E. colonoscopy
ANSWER: A

For the retrocaecal appendicitis is not typical:


A. Volkovcha-Kocher's sign
B. delayed diagnostics
C. late entrance of patients in the hospital
D. frequent development of destructive forms
E. weak expressed signs of peritoneal irritation
ANSWER: A

As the first period flowing of acute pancreatitis is named:


A. Haemodynamic violations and pancretic shock
B. To functional insufficiency of parenchymatous organs
C. Degenerative and festerings complications
D. All answers are faithful
E. A right answer is not present
ANSWER: A

As the third period flowing of acute pancreatitis is named:


A. Haemodynamic violations and pancretic shock
B. To functional insufficiency of parenchymatous organs
C. Degenerative and festerings complications
D. All answers are faithful
E. A right answer is not present
ANSWER: B
What complications at a acute pancreatitis is behave to late:
A. Peritonitis
B. Phlegmon retroperitoneum space
C. Formation of pseudocysts
D. Development of saccharine diabetes
E. Intestinal impassability
ANSWER: B

What a clinical flow can be at acute pancreatitis:


A. Easy, middle, heavy
B. Acute, chronic
C. Abortive, making progress
D. Edema, necrosis
E. Any variant
ANSWER: C

For the abortive flowing characteristically:


A. A process limited to the acute edema with convalescence in 7-8 days
B. A process limited to tearing away of the pathologically changed gland
C. A process limited to tearing away of the pathologically unchanged gland
D. A disease completed so not attaining clinical displays
E. Changes from the side of organ are not present
ANSWER: A

Specify the optimum volume operation at acute biliar pancreatitis:


A. Deleting exsudate from abdominal region
B. Decapsulation pancreas
C. Pancreatectomy
D. Sanitization and draining bilious ways
E. Draining pancreas channel
ANSWER: D

Peritonitis is divided into:


A. reactive, toxic, terminal
B. compensated, subkompensovanyy, terminal;
C. stage (I; II; III);
D. upper and lower half of the abdomen;
E. Early and late
ANSWER: A

Duration of preoperative preparation of patients with peritonitis limited:


A. conduct necessary surveys;
B. stabilization of homeostasis;
C. Willingness of Surgeons;
D. their preparation for anesthesia;
E. established ongoing all causes
F. neostigmine methylsulfate
ANSWER: E
To be carried out by blood tests?
A. now the whole blood transfusion is therefore not necessary to conduct tests;
B. determining blood type, biological samples;
C. to save time and Rh blood group-membership can be estimated by documented data (in
passport), to conduct a biological sample;
D. blood group O (I) Rh (-) is universal for transfusions for any recipient;
E. blood grouping, Rh accessories, group and individual compatibility of biological
samples
ANSWER: E

What is the abscessing pneumonia characterized by?


A. Multiple destructive foci 0,3-0,5 cm in size within 1-2 segments of lungs
B. Purulent destruction of pulmonary tissue within 1 segment with formation of cavity,
filled by pus
C. Purulent, necrosis of a pulmonary tissue within 2-3 segments, detached from
adjacent pulmonary parenchyma
D. Diffuse purulent, ichorous necrosis more than lobe without the tendency to defined
demarcation
E. Accumulation of pus in a pleural cavity
ANSWER: A

What is the predominant factor which causes the lung abscess?


A. Disturbances of bronchial permeability with the development of atelectasis
B. Increased cholesterol, dyslipoproteinemia
C. Pulmonary hypertension
D. Rheumatism, endocarditis
E. Myocardial infarction
ANSWER: A

The blunted sound by percussion is typical for:


A. Lung abscess
B. Bronchitis
C. Lung emphysema
D. Pneumothorax
E. Lung cyst
ANSWER: A

The blunted sound by percussion is typical for:


A. Lung gangrene
B. Bronchitis
C. Lung emphysema
D. Pneumothorax
E. Lung cyst
ANSWER: A

One or several cavities with a thick, dense pyogenic sheath on X-ray is typical for:
A. Chronic lung abscess
B. Lung gangrene
C. Acute lung abscess
D. Lung emphysema
E. Lung cyst
ANSWER: A

What kind of X-ray shadow is typical for acute lung abscess before draining?
A. Rounded shadow with considerable perifocal infiltration
B. Homogeneous spherical shadow with regular edge on the background of intact
pulmonary tissue
C. Heterogeneous shadow with calcifications, excentric destruction and regular edge
D. Homogeneous spherical shadow with irregular edge and phenomena of lymphangitis
(corona maligna)
E. Heterogeneous shadow with destruction, displaced in the upper lobes, with fibrosis,
petrifactions in adjacent tissue, peribronchial lymphadenitis
ANSWER: A

What kind of X-ray shadow is typical for lung cyst?


A. Homogeneous spherical shadow with regular edge on the background of intact
pulmonary tissue
B. Rounded shadow with considerable perifocal infiltration
C. Heterogeneous shadow with calcifications, excentric destruction and regular edge
D. Homogeneous spherical shadow with irregular edge and phenomena of lymphangitis
(corona maligna)
E. Heterogeneous shadow with destruction, displaced in the upper lobes, with fibrosis,
petrifactions in adjacent tissue, peribronchial lymphadenitis
ANSWER: A

What kind of X-ray shadow is typical for tuberculoma?


A. Heterogeneous shadow with calcifications and regular edge
B. Homogeneous spherical shadow with regular edge on the background of intact
pulmonary tissue
C. Rounded shadow with considerable perifocal infiltration
D. Homogeneous spherical shadow with irregular edge and phenomena of lymphangitis
(corona maligna)
E. Heterogeneous shadow with destruction, displaced in the upper lobes, with fibrosis,
petrifactions in adjacent tissue, peribronchial lymphadenitis
ANSWER: A

What kind of X-ray shadow is typical for tubercular cavern?


A. Heterogeneous shadow with destruction, displaced in the upper lobes, with fibrosis,
petrifactions in adjacent tissue, peribronchial lymphadenitis
B. Homogeneous spherical shadow with regular edge on the background of intact
pulmonary tissue
C. Rounded shadow with considerable perifocal infiltration
D. Heterogeneous shadow with calcifications and regular edge
E. Homogeneous spherical shadow with irregular edge and phenomena of lymphangitis
(corona maligna)
ANSWER: A
Heterogeneous shadow with destruction, displaced in the upper lobes, with fibrosis,
petrifactions in adjacent tissue, peribronchial lymphadenitis on X-ray is typical for:
A. Tubercular cavern
B. Peripheral lung cancer
C. Tuberculoma
D. Lung cyst
E. Lung emphysema
ANSWER: A

The clinical dynamics of lung abscess which is characterized by prompt positive


clinical, roentgenological and laboratory dynamics and recovery after the adequate
treatment regards to the:
A. Favorable course
B. Non-progressive course
C. Progressing course
D. Incapsulated process
E. Complicated course
ANSWER: A

The clinical dynamics of lung abscess which is characterized by transforming of the


process into the chronic form due to poor drainage of the suppurative focus and
permanent purulent intoxication regards to the:
A. Non-progressive course
B. Favorable course
C. Progressing course
D. Incapsulated process
E. Complicated course
ANSWER: A

The clinical dynamics of lung abscess which is characterized by the partial or


complete obstruction of the draining bronchus combined with satisfactory resistance of
the organism regards to the:
A. Incapsulated process
B. Progressing course
C. Non-progressive course
D. Favorable course
E. Complicated course
ANSWER: A

What is the indication for operative treatment of acute abscess of lungs?


A. Pulmonary bleeding of ІІ- ІІІ degree
B. Decompensation of the vital organs
C. Bilateral purulent destruction of lungs
D. Incurable malignant tumours
E. Pulmonary hypertension
ANSWER: A

What is the indication for operative treatment of acute abscess of lungs?


A. Tense pyopneumothorax, which is failed to liquidate by the draining of pleural
space
B. Decompensation of the vital organs
C. Bilateral purulent destruction of lungs
D. Incurable malignant tumours
E. Pulmonary hypertension
ANSWER: A

What is the indication for operative treatment of acute abscess of lungs?


A. Impossibility to rule out the suspicion on a malignant tumour
B. Decompensation of the vital organs
C. Bilateral purulent destruction of lungs
D. Incurable malignant tumours
E. Pulmonary hypertension
ANSWER: A

What is revealed in pleural empyema by auscultation?


A. The breathing isn't auscultated
B. Vesicular breathing
C. Amphoric breathing with moist rales
D. Bronchial breathing with moist rales
E. Harsh breathing with dry rales
ANSWER: A

The absence of breathing sounds by auscultation is typical for:


A. Pleural empyema
B. Bronchitis
C. Lung emphysema
D. Pulmonary hypertension
E. Pneumonia
ANSWER: A

What is the most informative in differential diagnostic of pleural empyema with


pleuropneumonia?
A. Pleural puncture
B. X-ray examination
C. Auscultation
D. Clinical manifestation
E. Sputum analysis
ANSWER: A

The swelled soft tissues of supraclavicular region are typical for the:
A. Apical empyema
B. Paracostal empyema
C. Paramediastinal empyema
D. Basal empyema
E. Postoperative empyema
ANSWER: A
The restricted thoracic excursion with severe chest pain are typical for the:
A. Paracostal empyema
B. Apical empyema
C. Paramediastinal empyema
D. Basal empyema
E. Postoperative empyema
ANSWER: A

The heart pain is typical for the:


A. Paramediastinal empyema
B. Paracostal empyema
C. Apical empyema
D. Basal empyema
E. Postoperative empyema
ANSWER: A

The pain in subcostal area, which increases at respiration is typical for the:
A. Paramediastinal empyema
B. Paracostal empyema
C. Apical empyema
D. Basal empyema
E. Postoperative empyema
ANSWER: A

What is the typical method of treatment of spread empyema?


A. Drainage of pleural space
B. Pleural puncture
C. Thoracotomy
D. Pneumonectomy, bilobectomy, lobectomy
E. Conservative treatment
ANSWER: A

What is the typical method of treatment of chronic empyema?


A. Pleurectomy, decortication of lung
B. Drainage of pleural space
C. Thoracotomy
D. Pneumonectomy, bilobectomy, lobectomy
E. Conservative treatment
ANSWER: A

What is the cause of pyopneumothorax?


A. Bronchiectatic disease
B. Obstructive bronchitis
C. Pulmonary embolism
D. Bronchial asthma
E. Pulmonary emphysema
ANSWER: A

What is the early complication of chest trauma?


A. Pneumothorax
B. Posttraumatic pneumonia
C. Posttraumatic pleurisy
D. Lung abscess
E. Pleural empyema
ANSWER: A

What is the early complication of chest trauma?


A. Hemothorax
B. Lung abscess
C. Pleural empyema
D. Posttraumatic pneumonia
E. Posttraumatic pleurisy
ANSWER: A

What is the late complication of chest trauma?


A. Pleural empyema
B. Pneumothorax
C. Hemothorax
D. Mediastinal emphysema
E. Traumatic shock, asphyxia
ANSWER: A

What is the chief clinical manifestation of noncomplicated rib fracture?


A. Pain
B. Dyspnea
C. Hemoptysis
D. Shock
E. Vomiting
ANSWER: A

What kind of X-ray picture is typical for noncomplicated rib fracture?


A. Break in continuity of bone fragments of ribs
B. Exudate in pleural space
C. Collapse of the lung
D. Lung athelectasis
E. Heterogeneous lung shadow with destruction
ANSWER: A

What is the chief clinical manifestation of floating rib fracture?


A. Shock
B. Pain
C. Dyspnea
D. Hemoptysis
E. Vomiting
ANSWER: A

What is the chief objective sign of floating rib fracture?


A. Paradoxical respiratory movements of chest
B. Crepitation of ribs
C. Hematoma of the chest wall
D. Hemoptysis
E. Subcutaneous emphysema
ANSWER: A

What kind of X-ray picture is typical for floating rib fracture?


A. Multiple rib fracture with deformity of the chest
B. Lung emphysema
C. Spheric shadow of the lung
D. Lung athelectasis
E. Heterogeneous lung shadow with destruction
ANSWER: A

What type of Novocaine block is used for the treatment of floating rib fracture?
A. Vagosympathetic block
B. Paranephral block
C. Spinal block
D. Epidural anesthesia
E. Lung root dlock
ANSWER: A

Subtotal pneumothorax means:


A. Collapse of lung to 2/3 of its volume
B. No collapse of lung
C. Collapse of lung to 1/3 of its volume
D. Collapse of lung more than 2/3 of its volume
E. Total collapse of lung
ANSWER: A

The collapse of lung in pneumothorax from 1/3 to 2/3 of its volume is called:
A. Subtotal pneumothorax
B. Partial pneumothorax
C. Total pneumothorax
D. Bilateral pneumothorax
E. Paradoxal pneumothorax
ANSWER: A

The collapse of lung in pneumothorax more than 2/3 of its volume is called:
A. Total pneumothorax
B. Partial pneumothorax
C. Subtotal pneumothorax
D. Bilateral pneumothorax
E. Paradoxal pneumothorax
ANSWER: A

What is revealed in pneumothorax by X-ray?


A. Lung collapse
B. Lung atelectasis
C. Rounded cavity with air-fluid level
D. Intensive homogeneous shadow in a basal parts with horizontal upper contour
E. Intensive homogeneous shadow in a basal parts with oblique upper contour
ANSWER: A

What is revealed in hemothorax by X-ray?


A. Intensive homogeneous shadow in a basal parts with oblique upper contour
B. Lung atelectasis
C. Rounded shadow with irregular contour
D. Rounded cavity with air-fluid level
E. Intensive homogeneous shadow in a basal parts with horizontal upper contour
ANSWER: A

Where is the level of the X-ray shadow in moderate hemothorax?


A. Up to scapular angle
B. Shadow observed only in the region of sinus
C. Up to ІІІ rib
D. Complete shadow of a pleural space
E. The shadow is absent
ANSWER: A

The Revilour-Greguar's test is used in the diagnostics of:


A. Pleural bleeding
B. Lung abscess
C. Pleural empyema
D. Pneumothorax
E. Deep vein thrombosis
ANSWER: A

What pathology is the indication for operative treatment?


A. Great and total hemothorax
B. Noncomplicated rib fracture
C. Closed partial pneumothorax
D. Closed total pneumothorax
E. Subcutaneous emphysema
ANSWER: A

What pathology is the indication for operative treatment?


A. Continuing hemothorax
B. Closed partial pneumothorax
C. Closed total pneumothorax
D. Noncomplicated rib fracture
E. Subcutaneous emphysema
ANSWER: A

What is the main cause of mediastinal emphysema?


A. Disruptions of trachea, bronchi
B. Rib fracture
C. Pneumothorax
D. Hemothorax
E. Mediastinal tumours
ANSWER: A

What does the mediastinal emphysema result in?


A. Cardiac tamponade
B. Hemoptysis
C. Pleural empyema
D. Pneumothorax
E. Lung atelectasis
ANSWER: A

Where is located the first anatomical narrowing of esophagus?


A. The site of pharyngoesophageal junction
B. The site of crossing with left bronchus
C. The site of crossing with aorta
D. The site of passing through diaphragm
E. The site of cardia
ANSWER: A

Hectic fever is possible at


A. to the uncomplicated ulcer
B. cicatrical-ulcerous stenosis
C. penetration
D. perforations of ulcer in the first clock
E. bleeding from an ulcer
ANSWER: C

Change in the analysis of blood at a perforete ulcer


A. leucopenia
B. anaemia
C. eosinophilia
D. leucocytosis with a neutrophilic change
E. there is not a right answer
ANSWER: D

At the perforete ulcer of duodenum used more frequent


A. sewing up of the perforate opening
B. sewing up gastroenteroanastomosis
C. resection of stomach
D. resection of stomach for a shutdown
E. different types of vagotomy in combination with the economy resection of stomach
and other draining operations
ANSWER: B

The ways distribution of gastroenteric content during the perforation of ulcer depend
on
A. anatomic structure of the lateral ductings
B. locations of stomach
C. localizations of the perforate opening
D. only transferred
E. forms and locations of transversal rim bowel
ANSWER: D

Sudden and painful pain with localization in the middle departments of stomach with an
irradiation in the back more characteristic for
A. heart attack the myocardium
B. break aneurysm the aorta
C. bilious colic
D. perforate ulcers
E. nephrocolic
ANSWER: B

What primary purpose treatment the patients with the heavy form of hemorragic
pancreatonecrosis to the operation is:
A. Liquidations the pain
B. Disintoxication the organism
C. Liquidations crampy the big duodenal papilla
D. Declines secretory activity the pancreas
E. Improvements microcirculation
ANSWER: B

ERCP apply at:


A. Postcholecystectomy syndrome
B. Stenosing papillitis
C. Stenosis the supraduodenal department of choledoch
D. Stricture the terminal department of choledoch
E. Mechanical icterus
ANSWER: C

At pancreatitis abscesses and infected necrosises execute such operations, except for:
A. Opening of abscess with draining
B. Pancreaticnecrsekvestrektomy
C. Pancreaticsekvestrektomy
D. Pancreaticsekvestrektomy with laparostomy
E. Total pancreatotomy
ANSWER: E

What most effective treatment the unformed complicated cyst is:


A. Conservative treatment
B. External draining cyst
C. Resection cyst within the limits of the unchanged gland
D. Cysticenterostomy
E. Cystogastrostomy
ANSWER: B

All surgical interferences at the destructive forms of acute pancreatitis divide on:
A. Early, late, deferred operations
B. Primary, second, repeated operations
C. Invasion, not invasion operations
D. Complicated, operations are not complicated
E. Not divided
ANSWER: A

What is necrectomy:
A. Delete the necrotic area within the limits of nonviable fabrics
B. Delete the necrotic area within the limits of healthy fabrics
C. Delete part of organ with his transversal cutting within the limits of the changed
fabrics
D. Total delete of organ
E. There is not a faithful answer
ANSWER: B

What is resection the pancreas:


A. Delete the necrotic area within the limits of nonviable fabrics
B. Delete the necrotic area within the limits of healthy fabrics
C. Delete part of organ with his transversal cutting within the limits of the changed
fabrics
D. Total delete of organ
E. There is not a faithful answer
ANSWER: C

Kulen’s sing at acute pancreatitis:


A. Violet spots on face and trunk
B. Cyanosis sides of stomach and trunk
C. Cyanosis skin of stomach
D. Icteritiousness round a belly-button
E. Cyanosys of hands
ANSWER: D

Voskresenskiy’s sing at acute pancreatitis:


A. Absence pulsation the abdominal aorta
B. Sickliness in left costal-vertebral coal
C. Sickliness and proof tension the muscles in an epigastrium with passing to left
subcostal area
D. Icteritiousness round a belly-button
E. Skin hyperesthesia in projection the gland
ANSWER: A

Meyo-Robson’s sing at acute pancreatitis:


A. Absence pulsation the abdominal aorta
B. Sickliness in left costal-vertebral coal
C. Sickliness and proof tension the muscles in an epigastrium with passing to left
subcostal area
D. Icteritiousness round a belly-button
E. Skin hyperesthesia in projection the gland
ANSWER: B
The clinical picture the aneurysm of the thoracic aorta distinguish syndromes:
A. Compression
B. Venous hypertension
C. Respiratory failure
D. Chronic arterial ischemia
E. Eating Disorders
ANSWER: A

The clinical picture the aneurysm of the thoracic aorta distinguish syndromes:
A. Hemodynamic
B. Venous hypertension
C. Respiratory failure
D. Chronic arterial ischemia
E. Eating Disorders
ANSWER: A

The clinical picture the aneurysm of the thoracic aorta distinguish syndromes:
A. No right answer
B. Venous hypertension
C. Respiratory failure
D. Chronic arterial ischemia
E. Eating Disorders
ANSWER: A

Aortoalgya found at:


A. No right answer
B. Deepvein thrombosis
C. Thrombosis of the inferior vena cava
D. Chronic arterial ischemia
E. Vena cava superior syndrome
ANSWER: A

Artalgesia must be differentiated from


A. Syndrome of the anterior chest wall
B. Appendicitis
C. Chronic bronchitis
D. Asthmatic
E. Ileus
ANSWER: A

Artalgya must be differentiated from


A. No right answer
B. Appendicitis
C. Chronic bronchitis
D. Asthmatic
E. Ileus
ANSWER: A
Type I Coarctation of aorta are:
A. Isolated contraction in the transition region of the aortic arch to the descending
division
B. The combination of this defect with ductus arteriosus and arterial or venous
shunting of blood
C. The combination of coarctation of the aorta with other congenital or acquired
defects of the cardiovascular system
D. Multiple or atypical localization of coarctation of the aorta
E. All answers are correct
ANSWER: A

Type II Coarctation of aorta are:


A. The combination of this defect with ductus arteriosus and arterial or venous
shunting of blood
B. Isolated contraction in the transition region of the aortic arch to the descending
division
C. The combination of coarctation of the aorta with other congenital or acquired
defects of the cardiovascular system
D. Multiple or atypical localization of coarctation of the aorta
E. All answers are correct
ANSWER: A

For coarctation of the aorta is characterized


A. Poor development of the lower limb muscles
B. Poor development of the musculature of the shoulder girdle
C. Well muscled legs
D. Good development of the pelvic belt
E. All answers are correct
ANSWER: A

For coarctation of the aorta is characterized


A. Pulse on the radial artery a full and busy
B. Pulse on the radial artery weakened
C. Pulse on the radial artery is absent
D. Pulse on the radial artery thready
E. All answers are correct
ANSWER: A

For coarctation of the aorta is characterized


A. Pulse on the posterior tibial artery is absent
B. Pulse on the posterior tibial artery weakened
C. Pulse on the posterior tibial artery is determined
D. Pulse on the posterior tibial artery satisfactory
E. All answers are correct
ANSWER: A

For coarctation of the aorta is characterized


A. No right answer
B. Pulse on the femoral artery is absent
C. Pulse on the femoral artery is defined clearly
D. Pulse on the femoral artery satisfactory
E. All answers are correct
ANSWER: A

Tortuosity and increased intercostals arteries is characteristic


A. Aortarctia
B. Aneurysms of the thoracic aorta
C. Abdominal aortic aneurysm
D. Vena cava superior syndrome
E. Vena cava inferior syndrome
ANSWER: A

Tortuosity and increased intercostals arteries is characteristic


A. No right answer
B. Aneurysms of the thoracic aorta
C. Abdominal aortic aneurysm
D. Vena cava superior syndrome
E. Vena cava inferior syndrome
ANSWER: A

Strong pulsation of the carotid, brachial, intercostal arteries is characteristic


A. No right answer
B. Aneurysms of the thoracic aorta
C. Abdominal aortic aneurysm
D. Vena cava superior syndrome
E. Vena cava inferior syndrome
ANSWER: A

For coarctation of the aorta is characterized


A. Blood pressure on the legs lowered
B. Blood pressure on the legs elevated
C. Blood pressure on the legs in normal
D. The difference of pressure on the right and left leg
E. All answers are correct
ANSWER: A

Obliterating atherosclerosis of the carotid arteries is more common in:


A. No right answer
B. In men younger than 40 years
C. In the men's 20 - 30 years
D. Not found
E. All answers are correct
ANSWER: A

The most important mechanism compensation in lesions of brachiocephalic arteries is:


A. Circle of Willis
B. Aortic arch
C. Brachial artery
D. Abdominal aorta
E. Vena cava superior
ANSWER: A

The most important mechanism of compensation in lesions of brachiocephalic arteries is:


A. No right answer
B. Aortic arch
C. Brachial artery
D. Abdominal aorta
E. Vena cava superior
ANSWER: A

Steal syndrome characteristic:


A. Occlusion of the proximal segment of the clavicular artery
B. Occlusion of the aortic arch
C. Occlusion of the brachial artery
D. Occlusion of the abdominal aorta
E. Occlusion of the superior vena cava
ANSWER: A

At Steal syndrome is characterized rob:


A. Brain
B. Right upper extremity
C. Left upper limb
D. Both limbs
E. All answers are correct
ANSWER: A

The absence pulsations the left superficial temporal characteristic for:


A. Lesions of the left common carotid artery
B. Lesions of the aortic arch
C. Occlusion of the brachial artery
D. Lesions of the abdominal aorta
E. Lesions of the superior vena cava
ANSWER: A

The absence pulsation on the left radial artery characteristic:


A. Lesion of the left subclavian artery
B. Lesions of the aortic arch
C. Occlusion of the brachial artery
D. Lesions of the abdominal aorta
E. Lesions of the superior vena cava
ANSWER: A

Reducing pulsation the left brachial artery is characteristic:


A. Lesion of the left subclavian artery
B. Lesions of the aortic arch
C. Occlusion of the brachial artery
D. Lesions of the abdominal aorta
E. Lesions of the superior vena cava
ANSWER: A

Reducing pulsation left brachial artery is characteristic:


A. No right answer
B. Lesions of the aortic arch
C. Occlusion of the brachial artery
D. Lesions of the abdominal aorta
E. Lesions of the superior vena cava
ANSWER: A

Nonspecific aortoarteriitis more striking:


A. Brachiocephalic trunk
B. Brachial artery
C. Abdominal aorta
D. Thoracic aorta
E. Coronary arteries
ANSWER: A

For lesions the subclavian artery is characterized by:


A. Chill hand
B. Dermahemia hands
C. Increased filling of subcutaneous veins of the upper extremities
D. The pulsation of the arteries of the upper extremities is not broken
E. All true
ANSWER: A

For lesions the subclavian artery is characterized by:


A. Cooling of the skin brushes
B. Dermahemia hands
C. Increased filling of subcutaneous veins of the upper extremities
D. The pulsation of the arteries of the upper extremities is not broken
E. All true
ANSWER: A

For lesions the subclavian artery is characterized by:


A. Hypotrophy of muscles of hands
B. Dermahemia hands
C. Increased filling of subcutaneous veins of the upper extremities
D. The pulsation of the arteries of the upper extremities is not broken
E. All true
ANSWER: A

For lesions the subclavian artery is characterized by:


A. The weakening of the pulse at the radial artery
B. Dermahemia hands
C. Increased filling of subcutaneous veins of the upper extremities
D. The pulsation of the arteries of the upper extremities is not broken
E. All true
ANSWER: A

In the diagnosis of atherosclerotic lesions an important role plays:


A. Ultrasound of neck vessels
B. Thermometry
C. Rheovasography
D. Radiological examination of the neck
E. Ultrasonography of the abdomen
ANSWER: A

In the diagnosis of atherosclerotic lesions an important role plays:


A. No right answer
B. Thermometry
C. Rheovasography
D. Radiological examination of the neck
E. Ultrasonography of the abdomen
ANSWER: C

The linear velocity of blood flow in carotid arteries can be determined using
A. Ultrasonic Doppler
B. CT
C. Radiography neck
D. Rheovasography
E. Thermometry
ANSWER: A

The volumetric blood flow rate on the carotid arteries can be determined using
A. Ultrasonic Doppler
B. CT
C. Radiography neck
D. Rheovasography
E. Thermometry
ANSWER: A

The linear velocity of blood flow in carotid arteries can be determined using
A. No right answer
B. CT
C. Radiography neck
D. Rheovasography
E. Thermometry
ANSWER: A

To diagnose the circle of Willis terms use:


A. Transcranial Doppler
B. Thermometry
C. Rheovasography
D. Radiological examination of the neck
E. Ultrasonography of the abdomen
ANSWER: A
Skalenus syndrome is
A. Extravessel compression of the subclavian artery at the exit from the thorax
B. Atherosclerosis of the subclavian artery
C. Aneurysm of the subclavian artery
D. Acute thrombosis of the subclavian artery
E. Axillaries artery aneurysm
ANSWER: A

Skalenus syndrome is
A. No right answer
B. Atherosclerosis of the subclavian artery
C. Aneurysm of the subclavian artery
D. Acute thrombosis of the subclavian artery
E. Axillaries artery aneurysm
ANSWER: A

Indications for carotid endarterctomy based on


A. Clinical manifestations of vascular insufficiency
B. Limitation of atherosclerosis
C. Prescription treatment of atherosclerosis
D. Patient's wishes
E. Want doctor
ANSWER: A

Indications for carotid endarterctomy based on


A. State of the surface of atherosclerotic plaque
B. Limitation of atherosclerosis
C. Prescription treatment of atherosclerosis
D. Patient's wishes
E. Want doctor
ANSWER: A

Carotid endarterctomy contraindicated


A. Up to 6 weeks after stroke
B. Up to 8 weeks after stroke
C. Up to 10 weeks after stroke
D. Contraindications No
E. All true
ANSWER: A

Carotid endarterctomy contraindicated


A. 2-3 months. myocardial infarction
B. 4-5 months. myocardial infarction
C. Up to 10 weeks after stroke
D. Contraindications No
E. All true
ANSWER: A
The clinical picture the aneurysm of the thoracic aorta distinguish syndromes:
A. Compression
B. Venous hypertension
C. Respiratory failure
D. Chronic arterial ischemia
E. Eating Disorders
ANSWER: A

The clinical picture the aneurysm of the thoracic aorta distinguish syndromes:
A. No right answer
B. Venous hypertension
C. Respiratory failure
D. Chronic arterial ischemia
E. Eating Disorders
ANSWER: A

Aortoalgya found at:


A. No right answer
B. Deepvein thrombosis
C. Thrombosis of the inferior vena cava
D. Chronic arterial ischemia
E. Vena cava superior syndrome
ANSWER: A

For aneurysms the thoracic aorta is characterized


A. Dry unproductive cough
B. Moist cough
C. The presence of asthma
D. The presence of pleural effusion
E. Respiratory disorders are absent
ANSWER: A

For artalgya is characteristic


A. Increased blood pressure
B. Lowering blood pressure
C. Changes in blood pressure is not
D. Fluctuations in blood pressure
E. Reducing pulse pressure
ANSWER: A

For artalgya is characteristic


A. Lack of effect of nitrate
B. The presence of the effect of nitrate
C. The presence of the effect of statins
D. The presence of the effect of antibiotic
E. The presence of the effect of aspirin
ANSWER: A

Artalgya must be differentiated from


A. Angina
B. Appendicitis
C. Chronic bronchitis
D. Asthmatic
E. Ileus
ANSWER: A

For the initial part of the aortic arch aneurysm is characterized by


A. No right answer
B. The decrease and delay pulse on the radial artery on the left
C. The increase in size and acceleration of the pulse on the radial artery on the
right
D. The increase in size and acceleration of the pulse on the radial artery on the left
E. All answers are correct
ANSWER: A

For aneurysms of the distal aortic arch is characterized by


A. The decrease and delay the pulse on the radial artery on the left
B. The decrease and delay the pulse on the radial artery on the right
C. The increase in size and acceleration of the pulse on the radial artery on the
right
D. The increase in size and acceleration of the pulse on the radial artery on the left
E. All answers are correct
ANSWER: A

Pulsating tumor-like formation with a reddish tinge of the skin over his right or left
from the front of the sternum is characteristic:
A. Aneurysm
B. Aneurysms of the descending aorta
C. Abdominal aortic aneurysms
D. Embolism of aorta
E. Thrombosis of the aorta
ANSWER: A

Type II Coarctation of aorta are:


A. The combination of this defect with ductus arteriosus and arterial or venous
shunting of blood
B. Isolated contraction in the transition region of the aortic arch to the descending
division
C. The combination of coarctation of the aorta with other congenital or acquired
defects of the cardiovascular system
D. Multiple or atypical localization of coarctation of the aorta
E. All answers are correct
ANSWER: A

For coarctation of the aorta is characterized


A. Poor development of the pelvic girdle muscles
B. Poor development of the musculature of the shoulder girdle
C. Well muscled legs
D. Good development of the pelvic belt
E. All answers are correct
ANSWER: A

For coarctation of the aorta is characterized


A. No right answer
B. Poor development of the musculature of the shoulder girdle
C. Well muscled legs
D. Good development of the pelvic belt
E. All answers are correct
ANSWER: A

For coarctation of the aorta is characterized


A. Pulse on the dorsal artery of foot missing
B. Pulse on the back of a weakened artery of foot
C. Pulse on the dorsal artery of foot is determined
D. Pulse on the dorsal artery of foot satisfactory
E. All answers are correct
ANSWER: A

For coarctation of the aorta is characterized


A. Pulse on the femoral artery weakened
B. Pulse on the femoral artery is absent
C. Pulse on the femoral artery is defined clearly
D. Pulse on the femoral artery satisfactory
E. All answers are correct
ANSWER: A

Tortuosity and increased intercostals arteries is characteristic


A. No right answer
B. Aneurysms of the thoracic aorta
C. Abdominal aortic aneurysm
D. Vena cava superior syndrome
E. Vena cava inferior syndrome
ANSWER: A

Strong pulsation of the carotid, brachial, intercostal arteries is characteristic


A. No right answer
B. Aneurysms of the thoracic aorta
C. Abdominal aortic aneurysm
D. Vena cava superior syndrome
E. Vena cava inferior syndrome
ANSWER: A

For coarctation of the aorta is characterized


A. Blood pressure on the legs lowered
B. Blood pressure on the legs elevated
C. Blood pressure on the legs in normal
D. The difference of pressure on the right and left leg
E. All answers are correct
ANSWER: A

For diagnostic coarctation of the aorta used:


A. Contrast aortography
B. Pulse Oximetry
C. Radiography limbs
D. Radiography abdominal
E. All answers are correct
ANSWER: A

In the etiology of stenosis brachycephalic arteries plays a role


A. Obliterating atherosclerosis
B. Endarteritis
C. Viral diseases
D. Intoxication
E. Supercooling
ANSWER: A

Steal syndrome characteristic:


A. Occlusion of the proximal segment of the clavicular artery
B. Occlusion of the aortic arch
C. Occlusion of the brachial artery
D. Occlusion of the abdominal aorta
E. Occlusion of the superior vena cava
ANSWER: A

Syndrome of vertebrobasilar insufficiency characteristic:


A. No right answer
B. Lesions of the internal carotid arteries
C. Lost external carotid arteries
D. Lesions of brachial artery
E. Lesions of the aortic arch
ANSWER: A

In the diagnosis pathology of the carotid arteries leading place is:


A. Contrast angiography
B. Thermometry
C. Radiography of the neck
D. Radiography of the chest cavity
E. Ultrasonography of the abdomen
ANSWER: A

In the diagnosis pathology of vertebral arteries leading place is:


A. Contrast angiography
B. Thermometry
C. Radiography of the neck
D. Radiography of the chest cavity
E. Ultrasonography of the abdomen
ANSWER: A

In the diagnosis pathology of the subclavian artery leading place is:


A. Contrast angiography
B. Thermometry
C. Radiography of the neck
D. Radiography of the chest cavity
E. Ultrasonography of the abdomen
ANSWER: A

Nonspecific aortoarteriitis more striking:


A. Brachiocephalic trunk
B. Brachial artery
C. Abdominal aorta
D. Thoracic aorta
E. Coronary arteries
ANSWER: A

For lesions the subclavian artery is characterized by:


A. Chill hand
B. Dermahemia hands
C. Increased filling of subcutaneous veins of the upper extremities
D. The pulsation of the arteries of the upper extremities is not broken
E. All true
ANSWER: A

For lesions the subclavian artery is characterized by:


A. Hypotrophy of muscles of hands
B. Dermahemia hands
C. Increased filling of subcutaneous veins of the upper extremities
D. The pulsation of the arteries of the upper extremities is not broken
E. All true
ANSWER: A

For atherosclerotic carotid arteries is characterized by:


A. Loss of consciousness
B. Dermahemia neck
C. Increased filling saphenous veins neck
D. The pulsation of the carotid artery is not broken
E. All true
ANSWER: A

For atherosclerotic carotid arteries is characterized by:


A. Dysopia
B. Dermahemia neck
C. Increased filling saphenous veins neck
D. The pulsation of the carotid artery is not broken
E. All true
ANSWER: A
For atherosclerotic carotid arteries is characterized by:
A. Ischemic stroke with hemiparesis
B. Dermahemia neck
C. Increased filling saphenous veins neck
D. The pulsation of the carotid artery is not broken
E. All true
ANSWER: A

In the diagnosis of atherosclerotic lesions an important role plays:


A. Ultrasound of neck vessels
B. Thermometry
C. Rheovasography
D. Radiological examination of the neck
E. Ultrasonography of the abdomen
ANSWER: A

In the diagnosis of atherosclerotic lesions an important role plays:


A. No right answer
B. Thermometry
C. Rheovasography
D. Radiological examination of the neck
E. Ultrasonography of the abdomen
ANSWER: C

In the diagnosis of atherosclerotic lesions an important role plays:


A. Nuclear Magnetic Resonance
B. Thermometry
C. Rheovasography
D. Radiological examination of the neck
E. Ultrasonography of the abdomen
ANSWER: A

To diagnose the circle of Willis terms use:


A. Transcranial Doppler
B. Thermometry
C. Rheovasography
D. Radiological examination of the neck
E. Ultrasonography of the abdomen
ANSWER: A

Skalenus syndrome is
A. Extravessel compression of the subclavian artery at the exit from the thorax
B. Atherosclerosis of the subclavian artery
C. Aneurysm of the subclavian artery
D. Acute thrombosis of the subclavian artery
E. Axillaries artery aneurysm
ANSWER: A
Indications for carotid endarterctomy based on
A. Clinical manifestations of vascular insufficiency
B. Limitation of atherosclerosis
C. Prescription treatment of atherosclerosis
D. Patient's wishes
E. No right answer
ANSWER: A

Indications for carotid endarterctomy based on


A. State of the surface of atherosclerotic plaque
B. Limitation of atherosclerosis
C. Prescription treatment of atherosclerosis
D. Patient's wishes
E. No right answer
ANSWER: A

Carotid endarterctomy contraindicated


A. Up to 6 weeks after stroke
B. Up to 8 weeks after stroke
C. Up to 10 weeks after stroke
D. Contraindications No
E. All true
ANSWER: A

Carotid endarterctomy contraindicated


A. 2-3 months. myocardial infarction
B. 4-5 months. myocardial infarction
C. Up to 10 weeks after stroke
D. Contraindications No
E. All true
ANSWER: A

Carotid endarterctomy contraindicated


A. At liver failure
B. 4-5 months. myocardial infarction
C. Up to 10 weeks after stroke
D. Contraindications No
E. All true
ANSWER: A

The advantages of local anesthesia at carotid endarterectomy are


A. Precise control of the state of consciousness of the patient
B. Muscle relaxation
C. Lack of consciousness of the patient
D. The possibility of movements
E. All true
ANSWER: A

The advantages of local anesthesia at carotid endarterectomy are


A. Possibility to save spontaneous respiration
B. Muscle relaxation
C. Lack of consciousness of the patient
D. The possibility of movements
E. All true
ANSWER: A

After carotid endarterectomy is necessary to monitor for:


A. The wounds on the neck
B. The muscle of the upper extremities
C. The muscle of the upper extremities
D. Possibility of movement
E. All true
ANSWER: A

Early complication after carotid endarterectomy:


A. Thrombosis segment reconstruction
B. Aneurysm segment reconstruction
C. Arteriovenous fistula
D. Suppurating wounds
E. All true
ANSWER: A

Where are falls the great saphenous vein?


A. Thigh Vienna
B. Calf veins
C. Popliteal vein
D. External iliac vein
E. V. cava inf.
ANSWER: A

Localization relapsing small saphenous vein are


A. In the popliteal fossa
B. In the upper third of the lower extremity
C. In the lower third of the thigh
D. 2-3 cm below the inguinal ligament
E. In the lumbar region
ANSWER: A

What is the typical sign for IIA stage of varicose veins?


A. Transient edema
B. Feeling gravity
C. Permanent edema
D. Polychromia
E. Trophic ulcer
ANSWER: A

What is the typical sign for IIB stage of varicose veins?


A. Permanent edema
B. Feeling gravity
C. Temporary swelling
D. Open trophic ulcer
E. Repairer trophic ulcer
ANSWER: A

For varicose veins of the lower extremities is characterized :


A. Feeling gravity
B. Intermittent claudication
C. Pain at rest
D. Loss of sensitivity
E. Loss of movement
ANSWER: A

For varicose veins of the lower extremities is characterized:


A. Extension of saphenous veins
B. Cyanosis of lower limb
C. The pallor of the lower extremity
D. Loss of sensitivity
E. Loss of movement
ANSWER: A

For varicose veins of the lower extremities is characterized:


A. Trophic ulcer leg
B. Cyanosis of lower limb
C. The pallor of the lower extremity
D. Loss of sensitivity
E. Loss of movement
ANSWER: A

Feeling the gravity of the lower extremities is characteristic for:


A. Varicose
B. Atherosclerotic lesions
C. Obliterative endarteritis
D. Leriche
E. Femoral artery embolism
ANSWER: A

Hyperpigmentation of the lower third of the leg is characteristic:


A. Varicose
B. Atherosclerotic lesions
C. Obliterative endarteritis
D. Leriche
E. Femoral artery embolism
ANSWER: A

What is the test used to determine valvular insufficiency superficial veins?


A. Troyanov-Trendelenburg’s test
B. Thalman’s test
C. Pratt’s test
D. Mayo Pratt’s test
E. Delba Perthes’s test (sustainer test)
ANSWER: A

What is the test used to assess the patency of deep veins?


A. Delba Perthes’s test (sustainer test)
B. Troyanov-Trendelenburg’s test
C. Hakenbruh’s test
D. Pratt’s test
E. Thalmann’s test
ANSWER: A

Thalmann’s test used in the diagnosis :


A. Valvular insufficiency communicative veins
B. Valvular insufficiency of superficial veins
C. Passing deep vein
D. Deep phlebothrombosis
E. Extremity lymphedema
ANSWER: A

Mayo Pratt’s test used in the diagnosis:


A. Patency of deep veins
B. Valvular insufficiency communicative veins
C. Valvular insufficiency of superficial veins
D. Deep phlebothrombosis
E. Extremity lymphedema
ANSWER: A

What is typical complication of varicose veins?


A. Subcutaneous thrombophlebitis
B. Arterial thrombosis
C. Paresis
D. Lymphostasis
E. Gangrene
ANSWER: A

What is typical complication of varicose veins?


A. Trophic ulcer
B. Arterial thrombosis
C. Paresis
D. Plegia
E. Gangrene
ANSWER: A

What is the main method diagnosis еру venous angiodysplasia?


A. Phlebography
B. Сoagulogramm
C. Delba Perthes’s test (sustainer test)
D. Ultrasound
E. Arteriography
ANSWER: A

Phlebography is used to diagnose:


A. Venous angiodysplasia
B. Atherosclerotic lesions
C. Obliterative endarteritis
D. Lymphedema
E. Gynecology
ANSWER: A

Narath’s operation is:


A. Treatment of saphenous vein collaterals
B. Ligation sapheno-femoral ejection
C. Removal of the main stem saphenous vein
D. Extrafascial ligation perforating veins
E. Subfascial ligation perforating veins
ANSWER: A

Coccet’s operation is:


A. Extrafascial ligation perforating veins
B. Treatment of saphenous vein collaterals
C. Ligation sapheno-femoral ejection
D. Removal of the main stem saphenous vein
E. Subfascial ligation of perforating veins
ANSWER: A

Narath’s operation is performed at :


A. Varicose
B. Deep vein thrombosis
C. Lymphedema
D. Atherosclerosis obliterans
E. Occlusive disease
ANSWER: A

Indications for sclerotherapy is:


A. Telangiectasia
B. Obliterating atherosclerosis
C. Occlusive disease
D. Deep vein thrombosis
E. Extremity lymphedema
ANSWER: A

The method of choice in treating varicose reticulum is:


A. Sclerotherapy
B. Operation Linton
C. Saphenectomy
D. Intimectomy
E. Autogenous vein bypass
ANSWER: A

The method of choice in treating recurrent varicose veins are:


A. Sclerotherapy
B. Operation Linton
C. Saphenectomy
D. Intimectomy
E. Autogenous vein bypass
ANSWER: A

Why saphenectomy is always starts ligation safeno-femoral ejection?


A. To prevent pulmonary embolism
B. To prevent bleeding
C. To prevent safeno-femoral reflux
D. To easily insert a venous extractor
E. To perform sclerotherapy
ANSWER: A

Painful bundle along the saphenous vein is characteristic for:


A. Subcutaneous thrombophlebitis
B. Lymphedema
C. Atherosclerotic lesions
D. Obliterative endarteritis
E. Deep vein thrombosis
ANSWER: A

What indicators prothrombin index should be in the treatment of venous thrombosis?


A. 50-70%
B. 10-20%
C. 30-40%
D. 85-100%
E. 100-120%
ANSWER: A

What is the normal level of plasma fibrinogen?


A. 2-4 g / l
B. 6-8 g / l
C. 10-16 grams / liter
D. 30-50 grams / liter
E. 75-100 g / l
ANSWER: A

What is the clinical form of postthrombotic syndrome does not exist?


A. Gangrenous
B. Sclerotic
C. Varicose
D. Oedema
E. Peptic
ANSWER: A

What are the clinical manifestations of postthrombotic syndrome?


A. Symptoms of venous insufficiency
B. Symptoms of nerve damage
C. Symptoms of vascular dystony
D. Signs of arterial insufficiency
E. Restriction of movement of the lower extremity
ANSWER: A

What is the typical sign for stage I postthrombotic syndrome?


A. Transient edema
B. Permanent edema
C. Polychromia
D. Trophic ulcer
E. Gangrene
ANSWER: A

What is the typical sign for stage II postthrombotic syndrome?


A. Permanent edema
B. Feeling gravity
C. Transient edema
D. Open trophic ulcer
E. Healing of trophic ulcers
ANSWER: A

What treatment is indicated in postthrombotic syndrome?


A. Saphenectomy
B. Conservative treatment
C. Sclerotherapy
D. Troyanov-Trendelenburg’s operation
E. Saphenectomy with subfascial ligation communicative veins (Linton’s operation)
ANSWER: A

Localization relapsing great saphenous vein are


A. 2-3 cm below the inguinal ligament
B. In the upper third of the lower extremity
C. In the popliteal fossa
D. In the lower third of the thigh
E. In the lumbar region
ANSWER: A

What veins belong to a surface system?


A. Small and large subcutaneous vein
B. Veins
C. Superficial and deep femoral vein
D. Humeral vein
E. Elbow and radial veins
ANSWER: A
Which factor are dominates in the development of primary varicose veins?
A. The weakness of the connective tissue blood vessels
B. Arteriovenous fistula
C. Venous hypoplasia
D. Diabetes mellitus
E. Obliterating atherosclerosis
ANSWER: A

What is the pathological basis for the development of chronic venous insufficiency?
A. Venous hypertension
B. Block lymph drainage
C. Arterial ischemia
D. Arterial hypertension
E. Innervation
ANSWER: A

What are causes venous valve insufficiency?


A. Venous hypertension
B. Arterial ischemia
C. Arterial hypertension
D. Innervation
E. Violation of lymph drainage
ANSWER: A

For varicose veins of the lower extremities is characterized :


A. Feeling gravity
B. Intermittent claudication
C. Pain at rest
D. Loss of sensitivity
E. Loss of movement
ANSWER: A

For varicose veins of the lower extremities is characterized:


A. Extension of saphenous veins
B. Cyanosis of lower limb
C. The pallor of the lower extremity
D. Loss of sensitivity
E. Loss of movement
ANSWER: A

For varicose veins of the lower extremities is characterized:


A. Trophic ulcer leg
B. Cyanosis of lower limb
C. The pallor of the lower extremity
D. Loss of sensitivity
E. Loss of movement
ANSWER: A
What is the test used to assess the patency of deep veins?
A. Delba Perthes’s test (sustainer test)
B. Troyanov-Trendelenburg’s test
C. Hakenbruh’s test
D. Pratt’s test
E. Thalmann’s test
ANSWER: A

Thalmann’s test used in the diagnosis :


A. Valvular insufficiency communicative veins
B. Valvular insufficiency of superficial veins
C. Passing deep vein
D. Deep phlebothrombosis
E. Extremity lymphedema
ANSWER: A

Mayo Pratt’s test used in the diagnosis:


A. Patency of deep veins
B. Valvular insufficiency communicative veins
C. Valvular insufficiency of superficial veins
D. Deep phlebothrombosis
E. Extremity lymphedema
ANSWER: A

What is typical complication of varicose veins?


A. Subcutaneous thrombophlebitis
B. Arterial thrombosis
C. Paresis
D. Lymphostasis
E. Gangrene
ANSWER: A

What is the main method diagnosis еру venous angiodysplasia?


A. Phlebography
B. Сoagulogramm
C. Delba Perthes’s test (sustainer test)
D. Ultrasound
E. Arteriography
ANSWER: A

What operation is carried out at varicose veins?


A. Saphenectomy
B. Thrombectomy
C. Vein ligation
D. Intimectomy
E. Femoropopliteal bypass
ANSWER: A

What treatment is indicated for uncomplicated varicose veins?


A. Saphenectomy
B. Conservative treatment
C. Sclerotherapy
D. Troyanov-Trendelenburg’s operation
E. Saphenectomy with subfascial ligation communicative veins (Linton’s operation)
ANSWER: A

In what sequence being saphenectomy


A. Operation Troyanov-Trendelenburg’s, Babcock’s, Narath’s
B. Operation Linton’s. Cockett’s, Babcock’s
C. Operation Narath’s, Babcock’s, Cockett’s
D. Operation Babcock’s, Linton’s, Troyanov-Trendelenburg’s
E. Operation Babcock’s, Narath’s, Cokket’s
ANSWER: A

Contraindication to saphenectomy with varicose veins are:


A. Obstruction deep veins
B. Incompetence of venous valves communicative vein
C. Incompetence ostial valve
D. Loose type of the great saphenous vein
E. Valvular insufficiency sural veins
ANSWER: A

What is the normal level of plasma fibrinogen?


A. 2-4 g / l
B. 6-8 g / l
C. 10-16 grams / liter
D. 30-50 grams / liter
E. 75-100 g / l
ANSWER: A

What are the clinical manifestations of postthrombotic syndrome?


A. Symptoms of venous insufficiency
B. Symptoms of nerve damage
C. Symptoms of vascular dystony
D. Signs of arterial insufficiency
E. Restriction of movement of the lower extremity
ANSWER: A

What is the main cause of atherosclerotic lesions?


A. Hypercholesterolemia, dyslipoproteinemia
B. Infection
C. Trauma
D. Rheumatism, endocarditis
E. Myocardial infarction
ANSWER: A

What does belongs to the second stage of atherosclerotic lesions according to the
classification by Fontane?
A. Functional circulatory insufficiency
B. Asymptomatic ran
C. Full compensation
D. Limb ischemia at rest
E. Destruction of tissue
ANSWER: A

What does belongs to the fourth stage of atherosclerotic lesions according to the
classification by Fontane?
A. Destruction of tissue
B. Asymptomatic ran
C. Full compensation
D. Functional circulatory insufficiency
E. Limb ischemia at rest
ANSWER: A

What is the most typical sign of the second stage of atherosclerotic lesions?
A. Intermittent claudication
B. Cooling of the lower extremities
C. Fever
D. Gangrene
E. Pain at rest
ANSWER: A

What is the most typical feature of the fourth stage of atherosclerotic lesions?
A. Gangrene
B. Pain at rest
C. Fever
D. Cooling of the lower extremities
E. Intermittent claudication
ANSWER: A

Intermittent claudication is characterized by:


A. Pain in the lower extremities
B. Heartache
C. Arthralgia
D. Dizziness
E. Edema of lower extremities
ANSWER: A

For atherosclerotic lesions at arteriography is characterized by:


A. Segmental occlusion of the arteries
B. Occlusion of terminal part of the inferior vena cava
C. Occlusion of the superior vena cava
D. Dysplasia arteries
E. Diffuse stenosis of small arteries
ANSWER: A
What method of research is the most informative in the differential diagnosis between
atherosclerosis and endarteritis obliterans?
A. Angiography
B. ECG
C. Biochemical analysis of blood
D. Complete blood
E. Target biopsy
ANSWER: A

At what level is no ripple at lumbosacral radiculitis?


A. Stored at all levels of
B. Calf arteries
C. Popliteal artery
D. Femoral artery
E. Aorta
ANSWER: A

What are the indications for conservative therapy of obliterative atherosclerosis?


A. I-II stage of chronic arterial insufficiency
B. Not shown at all
C. III-IV stage of chronic arterial insufficiency
D. Leriche syndrome
E. Arterial thrombosis
ANSWER: A

Blanching of the skin foot is characteristic:


A. Obliterative endarteritis
B. Deep thrombophlebitis
C. Surface thrombophlebitis
D. Postthrombotic syndrome
E. Phlegmon of the foot
ANSWER: A

What is the typical sign for the I stage of obliterating endarteritis?


A. Cooling of the lower extremities
B. Intermittent claudication
C. Fever
D. Pain at rest
E. Gangrene
ANSWER: A

What is the typical sign for the III stage of obliterating endarteritis?
A. Pain at rest
B. Cooling of the lower extremities
C. Fever
D. Intermittent claudication
E. Gangrene
ANSWER: A
For obliterative endarteritis is characterized by:
A. Intermittent claudication
B. Angina
C. Dizziness
D. Oedema of lower extremities
E. Extension of saphenous veins
ANSWER: A

At obliterative endarteritis first affected:


A. Peripheral arteries.
B. Inguinal artery.
C. Aorta.
D. Ventral trunk.
E. The upper and lower mesenteric artery.
ANSWER: A

With the defeat of what artery atherosclerosis can develops Leriche syndrome?
A. Bifurcation of the aorta, common iliac arteries.
B. Popliteal artery.
C. Arteries of the lower leg.
D. Ventral trunk.
E. Inferior mesenteric artery.
ANSWER: A

What kind of reconstructive operations on the major arteries are carried out with
obliterating atherosclerosis?
A. Endarterectomy, bypass surgery or prosthetic arteries.
B. Leriche's operation.
C. Lumbar sympathectomy.
D. Palm’s operation .
E. Troyanov-Trendelenburg’s operation.
ANSWER: A

What kind of manipulation to be done vascular prostheses ifection?


A. Remove the prosthesis.
B. Catheterization subclavian vein.
C. Catheterization great saphenous vein.
D. Fasciotomy.
E. Necrectomy.
ANSWER: A

Aorto-occlusive disease at arteriogram characterized by:


A. Uniform narrowing of the lumen of the arteries of the lower extremities.
B. Occlusion of peripheral arteries.
C. Uneven narrowing of the lumen of the arteries of the lower extremities.
D. Occlusion of collateral arteries.
E. Occlusion of capillaries.
ANSWER: A
The most severe complication after reconstructive operations on the major arteries are:
A. Bleeding
B. Suppuration
C. Thrombosis
D. Chylorrhea
E. Phlebeurysm
ANSWER: A

What are the indications for lumbar sympathectomy at obliterating endartereite?


A. Stage II
B. Stage IV
C. Gangrene of the lower extremity
D. Deep venous thrombosis
E. The duration of reactive hyperemia was more than 3 minutes
ANSWER: A

At segmental occlusion of the superficial femoral artery what operation is performed:


A. Autogenous vein bypass
B. Endarterectomy of the femoral artery
C. Iliac-femoral bypass surgery
D. Saphenectomy
E. Artery ligation
ANSWER: A

At segmental occlusion of the iliac artery what operation is performed:


A. Iliac-femoral aloshuntirovanie
B. Autogenous vein bypass
C. Endarterectomy of the femoral artery
D. Saphenectomy
E. Artery ligation
ANSWER: A

Agents, which cause peritonitis, can be all except for:


A. urines at the break of urinary bladder
B. tables of contents the stomach during the perforation of ulcer
C. to blood at the trauma of stomach
D. biles during the perforation of gall-bladder
E. air in an abdominal region after laparoscopy research
ANSWER: E

In classic motion of peritonitis select the stages:


A. early, intermediate, late
B. reactive, intermediate, late
C. toxic, intoxication, terminal
D. reactive, toxic, terminal
E. reactive, toxic, late
ANSWER: D

To the local isolated peritonitis does not attribute:


A. subdiaphragmatic abscess
B. subhepatic abscess
C. interintestinal abscess
D. primary idiopathic peritonitis
E. abscess cystic-rectal spaces
ANSWER: D

The complex treatment of festering peritonitis does not provide for:


A. delete of primary hearth
B. vagotomy
C. correction the metabolic violations
D. adequate therapy by antibiotics
E. struggle of paresis the intestine
ANSWER: B

The diagnostics criteria of the anaerobic peritonitis is


A. stormy progress of disease
B. heavy festering intoxication
C. expressed enteroplegia
D. abundant amount of exsudate green-brown colors
E. all is transferred
ANSWER: E

For the reactive stage of festering peritonitis not characteristically:


A. acute sickliness the stomach at palpation
B. positive Shchetkin-Blyumberg’s symptom
C. tension of muscles the front abdominal wall
D. face of «Hippocrates»
E. tachycardia
ANSWER: D

Postoperative peritonitis is characterized a flow:


A. typical
B. atypical
C. stormy
D. with the expressed pain reaction
E. with the expressed intoxication
ANSWER: E

The most informing method the instrumental diagnostics of peritonitis is:


A. survey sciagraphy of abdominal region
B. lasparoscopy
C. angiography
D. gastroscopy
E. colonoscopy
ANSWER: A

Decision role in differential diagnostics the peritonitis and dissecting aneurysm of


aorta , there is a symptom:
A. aperistalsis
B. acute pain in a stomach
C. systole noise above the abdominal department of aorta
D. absence of pulsation of abdominal department of aorta
E. melena at rectal research
ANSWER: C

Preoperated complication of acute appendicitis


A. diffusive peritonitis
B. intra-abdominal bleeding
C. suppuration of wound
D. eventeration wounds
E. there are not a right answer
ANSWER: A

For perforative appendicitis characteristically


A. tension the muscles of front abdominal wall
B. there is the sudden strengthening of stomach-aches
C. rapid growth of clinical picture the peritonitis
D. Razdolskuy’s symptom
E. all transferred
ANSWER: E

Tension of abdominal wall and stage peritonitis at acute appendicitis


A. absents, a stomach is swollen - terminal
B. absents, a stomach is not swollen - terminal
C. expressed, a stomach is not swollen - terminal
D. absents, a stomach is swollen - toxic
E. absents, a stomach is swollen – initial
ANSWER: C

Intraoperative cholangiograpy at cholecystectomy used for


A. Researches of peristalsis the general bilious channel
B. Retrograde filling of intrahepatic bilious channels
C. Researches tone the sphincter Oddi
D. Exceptions concrements in channels
E. Exposures cholangitis
ANSWER: D

Most widespread laparoscopic operation at bile-stone illness:


A. Cholecystectomy with revision the bilious ways
B. Ideal cholecysectomy
C. Choledoholitotomy
D. Cholecystectomy
E. All answers are right
ANSWER: C

The special research extrahepatic bilious ways is absolutely indicated at:


A. Shallow stone in common bile duct, suspicion on stenosis the large duodenal
papilla, expansion of the common bile duct, mechanical icterus in the moment of
operation
B. Suspicion on stenosis of large duodenal papilla, expansion of the common bile duct,
mechanical icterus in the moment of operation
C. At expansion of the common bile duct
D. All right
E. All not right
ANSWER: D

Acute cholecystitis usually begins with


A. Increases the temperature
B. Appearances the vomiting
C. Pains under a rib on the right
D. Disorders of chair
E. Weights are in a epigastria area
ANSWER: C

Characteristic laboratory sign of the acute uncomplicated cholecystitis


A. Diastasuria
B. Leykocytosis
C. Hypoglycemia
D. Glucosuria
E. Hyperbillirubinemia
ANSWER: B

What operation is used for suppuration the pseudocysts of pancreas:


A. Cystojejunostomy on the eliminated loop
B. External draining the cyst
C. Cystogastrostomy
D. Cystoduodenostomy
E. Cystoenteroanastomosis
ANSWER: B

What operation is most often used for localization the formed pseudocyst in the area
of tail the pancreas:
A. Cystojejunostomy on the eliminated loop
B. External draining the cyst
C. Cystogastrostomy
D. Cystoduodenostomy
E. Cystoenteroanastomosis
ANSWER: D

What most effective blocker secretion of pancreas at acute pancreatitis:


A. Cyanocobalamin
B. Ubretid
C. Arginine
D. Sandostatin
E. Benzogeksoniy
ANSWER: D

Indication to early operative interference at acute pancreatitis is:


A. Acute oedematous pancreatitis
B. Acute pancreatolysis
C. Acute traumatic pancreatitis at the „fresh” break the gland
D. Acute fatty pancreatitis
E. A faithful answer is not present
ANSWER: C

What from operations does not execute at surgical treatment complicated acute
pancreatitis:
A. Through draining the stuffing-box bag
B. Abdominisation the pancreas
C. Omentopankreatopeksiy
D. Left-side resection of gland
E. Pancreatojejunostomy
ANSWER: E

What most effective treatment the unformed uncomplicated cyst is:


A. Conservative treatment
B. External draining cyst
C. Resection cyst within the limits of the unchanged gland
D. Cysticenterostomy
E. Cystogastrostomy
ANSWER: A

What most effective treatment the formed uncomplicated cyst is:


A. External draining the cyst
B. Marsupialization
C. Resection the cyst within the limits of the unchanged gland
D. Cysticenterostomy
E. Cystogastrostomy
ANSWER: D

What is sequestrotomy:
A. Delete the necrotic area within the limits of nonviable fabrics
B. Delete the necrotic area within the limits of healthy fabrics
C. Delete part of organ with his transversal cutting within the limits of the changed
fabrics
D. Total delete of organ
E. There is not a faithful answer
ANSWER: A

At percusion in the first clock after perforation the ulcer more possibly
A. dulling in the gently sloping places of stomach
B. expansion of percusion border of liver
C. tympanitis in left subcosctal area
D. expansion of percusion border the spleen
E. there is not a right answer
ANSWER: A

Relative absolute indication to operative treatment ulcerous illness is


A. penetration of ulcer
B. ulcerous anamnesis more than 15 years
C. malignization ulcers
D. perforation of ulcer
E. relapses more than 3 times per a year
ANSWER: A

Absolute sign of unstable hemostasis


A. profluvium blood from a vessel;
B. absence blood in a stomach and bulb of duodenum;
C. presence light blood and faltungs of blood in a stomach;
D. all answers are correct;
E. all answers are not correct
ANSWER: A

Absolute indication to operative treatment the ulcerous illness is


A. voluminous bleeding
B. callous ulcers
C. relapses more than 2 one time per a year
D. ulcerous anamnesis more than 10 years
E. heavy pain syndrome
ANSWER: A

Absolute indication to operative treatment the ulcerous illness is


A. relapses more than 2 one time per a year
B. ulcerous anamnesis more than 10 years
C. relapse ulcer after the resection of stomach
D. relapses more than 3 times per a year
E. cicatrical-ulcerous stenosis of pylorus
ANSWER: E

Patient which the gastroenteric bleeding in house is necessary


A. To send a patient in surgical permanent establishment
B. To appoint rest, enter Cacl, vicasol
C. To wash a stomach, appoint a cold, rest of supervision
D. To send a patient in a therapeutic gastroenterology separation
E. A right answer absents
ANSWER: A

Esophagogastroduodenoscopy can find out next changes in a stomach, except for


A. tumours
B. ulcers
C. bleeding polypuses
D. erosions
E. changes of evacuation function
ANSWER: E

To absolute indication to operative interference at ulcerous illness does not belong


A. scarry-ulcerous stenosis
B. perforation of ulcer
C. profuse bleeding
D. diameter ulcer a more than 3 cm
E. bleeding what does not stopped with conservative
ANSWER: A

What from the transferred operations does not belong to organ protective
A. trunk vagotomy
B. resection by Bilrot II
C. selective vagotomy
D. selective proximal vagotomy
E. the all transferred does not belong
ANSWER: B

At a subhepatic abscess can take place all, except for:


A. pains in a thorax with an irradiation in a supraclavicular area
B. reactive pleurisy
C. Courvoisier's symptom
D. Senator’s symptom
E. Dyushen’s symptom
ANSWER: E

The leading symptom of peritonitis is:


A. stomach-ache
B. enteroplegia
C. swelling of stomach
D. symptoms the irritation of peritoneum
E. symptoms of the intestinal impassability
ANSWER: A

Specify obligatory measures which are conducted during an operation concerning


widespread fibrinopurulent peritonitis:
A. removal the source of peritonitis
B. sanitization of the abdominal region.
C. decompression of intestine.
D. draining of abdominal region.
E. all answers are faithful
ANSWER: A

For what purpose in treatment of diffusive festering peritonitis does execute


nasointestinal intubation?
A. account of losses the liquid through a gastroenteric highway.
B. control of electrolyte composition the intestinal maintenance
C. prophylaxis of intestinal impassability
D. stimulation of the intestinal peristalsis
E. suppression of the intestinal peristalsis
ANSWER: D

For perforeted ulcer characteristically


A. tension the muscles of front abdominal wall
B. melena
C. vomiting by coffee-grounds
D. high intestinal impassability
E. vomiting stagnant gastric maintenance
ANSWER: A

For motion of disease ulcerous illness of middle weight characteristically


A. development of complications
B. relapses 1-2 times per a year
C. 4 and anymore relapses on a year
D. 5 and more relapses are on a year
E. 3 and anymore relapses on a year
ANSWER: B

Who had first time in medical practice performed cholecystectomy


A. Courvoisier L.
B. Lanhenbuh K.
C. The monastic ND
D. Fedorov SP
E. Coeur G.
ANSWER: B

With the patient lying on his back at the lowered terminal end of the table well to
examine:
A. Gallbladder
B. Stomach
C. Recto-sigmoid colon angle
D. Uterus
E. Spleen
ANSWER: B

With the patient lying on his back at the lowered terminal end of the table well to
examine:
A. Gallbladder
B. Abdominal esophagus
C. the right lobe of the liver
D. Uterus
E. Spleen
ANSWER: B

With the patient lying on his back in a horizontal position table well to examine:
A. Gallbladder
B. Pancreas
C. small intestine
D. sigmoid
E. Spleen
ANSWER: C

With the patient lying on his back with lowered head end of the table well to examine:
A. Gallbladder
B. Pancreas
C. Recto-sigmoid colon angle
D. Uterus
E. Spleen
ANSWER: D

With the patient lying on his back with lowered head end of the table well to examine:
A. Gallbladder
B. Pancreas
C. Recto-sigmoid colon angle
D. bladder
E. Spleen
ANSWER: D

With the patient lying on his back with lowered head end of the table well to examine:
A. Gallbladder
B. Pancreas
C. Recto-sigmoid colon angle
D. Fallopian tubes
E. Spleen
ANSWER: D

With the patient lying on his back with lowered head end of the table well to examine:
A. Gallbladder
B. Pancreas
C. Recto-sigmoid colon angle
D. Ovaries
E. Spleen
ANSWER: D

With the patient lying on his right side with horizontal table well to examine:
A. Spleen
B. Pancreas
C. the right lobe of the liver
D. Uterus
E. Right side channel
ANSWER: A

With the patient lying on his right side with lowered head end table well to examine:
A. Gallbladder
B. Pancreas
C. Sigmoid
D. The right ovary
E. Spleen
ANSWER: C

With the patient lying on his right side with lowered head end table well to examine:
A. Gallbladder
B. Pancreas
C. Recto-sigmoid colon angle
D. The right ovary
E. Spleen
ANSWER: C

With the patient lying on the left side at the lowered terminal end of the table well
to examine:
A. Spleen
B. Pancreas
C. Gallbladder
D. Uterus
E. Ovaries
ANSWER: C

With the patient lying on the left side at the lowered terminal end of the table well
to examine:
A. Spleen
B. Pancreas
C. Hepatic flexure of colon
D. Uterus
E. Ovaries
ANSWER: C

With the patient lying on the left side in the horizontal position of the table is
well to examine:
A. Spleen
B. Pancreas
C. the right lobe of the liver
D. Uterus
E. Right side channel
ANSWER: E

With the patient lying on the right side at the lowered terminal end of the table well
to examine:
A. greater curvature of the stomach
B. Pancreas
C. Kidneys
D. Uterus
E. Spleen
ANSWER: A

With the patient lying on the right side at the lowered terminal end of the table well
to examine:
A. splenic flexure of the colon
B. Pancreas
C. Kidneys
D. Uterus
E. Spleen
ANSWER: A

What diameter of trocar is used in most surgeries in modern laparoscopy?


A. 2 and 4 mm
B. 4 and 8 mm
C. 5 and 10 mm
D. 15 and 20 mm
E. 10 and 15 mm
ANSWER: C

What group of instruments does “Babcock” clamp belong?


A. surgical clamp
B. atraumatic clamp
C. dissector
D. instrument for bipolar coagulation
E. for applying continuous sutures
ANSWER: B

What is culdoscopy?
A. Overview of the abdominal cavity using a special endoscope
B. Overview of the chest cavity using a special endoscope
C. Review of the mediastinum using a special endoscope
D. Review the joint cavity using a special endoscope
E. Pelvic exam using a special endoscope
ANSWER: E

What is laparoscopy?
A. Overview of the abdominal cavity using a special endoscope
B. Overview of the chest cavity using a special endoscope
C. Review of the mediastinum using a special endoscope
D. Review the joint cavity using a special endoscope
E. Pelvic exam using a special endoscope
ANSWER: A

What is needed to perform arthroscopy?


A. Introduction of air into the abdominal cavity
B. Intubation of main bronchi
C. The introduction of air into the chest cavity
D. The introduction of air into the joint cavity
E. The introduction of fluid into the joint cavity
ANSWER: E

What process represents the IV type of smear at oncocytological examination?


A. the unchanged epithelium.
B. mild or moderate dysplasia.
C. cancer.
D. inflammatory process.
E. intraepithelial cancer.
ANSWER: E

At diagnostic laparoscopy
A. You can change the position of the patient
B. You can not change the position of the patient
C. You can change the position of the patient in the sagittal plane
D. You can change the position of the patient in the frontal plane
E. preferably is not to confuse the patient
ANSWER: A

Pelvis should be examined with the patient:


A. On the back of the horizontal position of the table
B. On the back of the head at the lowered end of the table
C. On the right side of the raised head end of table
D. On the left side of the raised head end of table
E. on the left side of the horizontal position of the table
ANSWER: D

At diagnostic laparoscopy dark red, brown or black pancreas, the presence of


hemorrhagic fluid "stearic plaques" indicates
A. Mixed pancreatic necrosis
B. Focal fatty pancreatic necrosis
C. Focal hemorrhagic necrotizing pancreatitis
D. Biliary pancreatitis
E. Intact pancreas
ANSWER: A

Planned therapeutic laparoscopy is indicated for all of the states except:


A. pain caused by adhesions in the abdominal cavity
B. cirrhosis
C. retention of ovarian cysts
D. pancreatic pseudocyst
E. papillary serous ovarian cysts
ANSWER: E

Preparing the patient for emergency laparoscopy includes all of the listed except:
A. Placing stomach tube
B. Identifying the blood group and Rh
C. drink methylene blue
D. Explain to the patient that help is needed from him during the investigation
E. Enter the intra-muscular 1 ml of 0.1% solution of atropine sulfate and 2%
promedol
ANSWER: C

Preparing the patient for routine laparoscopy includes all of the listed except:
A. cleansing enemas
B. sedatives
C. teaching the patient to "inflate" the abdomen
D. emptying the bladder
E. abdominal X-rays
ANSWER: E

Round ligament of the liver is projected onto the line connecting:


A. Navel and right upper quadrant
B. Navel and the left upper quadrant
C. Navel and xyphoid
D. Umbilicus and symphysis
E. Navel and left inguinal quadrant
ANSWER: A

Sigmoid colon is available to view when the patient is:


A. On the back of the horizontal position of the table
B. On the back of the head end of the table lowered
C. On the left side of the horizontal position of the table
D. On the left side of the head at the closed end of the table
E. on the right side with slightly lowered head end of the table
ANSWER: E

Survey of appendix should be at the position of the patient:


A. On the back of the horizontal position of the table
B. On the back of the head end of the table announced
C. On the right side of the lowered head end of table
D. On the left side of the lowered head end of table
E. on the right side
ANSWER: D

The following areas of the abdomen are not neighboring:


A. mesogastrium and epigastrium
B. mesogastrium and hypogastrium
C. hypogastrium and epigastrium
D. All answers are correct
E. There is no right answer
ANSWER: C

The line connecting the pelvic spines divides abdomen on


A. mesogastrium and epigastrium
B. mesogastrium and hypogastrium
C. hypogastrium and epigastrium
D. All answers are correct
E. There is no right answer
ANSWER: B

The right kidney can be easily seen when:


A. summing the lower edge of the right lobe of the liver
B. shifting down transverse angle of the colon
C. shifting upwards hepatic angle of the colon
D. shifted laterally ascending colon
E. shifted medially ascending colon
ANSWER: A

To better view the gallbladder additional trocar introduced in


A. left upper quadrant
B. umbilical area
C. the left side of the area
D. the right upper quadrant
E. above the vagina
ANSWER: D

To view appendix the additional trocar is injected into the peritoneal cavity in
A. the upper right point of Tracing
B. the upper left point of Tracing
C. in the suprapubic area
D. the white line of the abdomen 5 cm above the umbilicus
E. At the point Mc BURNEO
ANSWER: C

Trocar deeply entered in the left iliac area may damage:


A. The left fallopian tube
B. liver
C. Stomach
D. The ascending colon
E. bladder
ANSWER: B

Trocar deeply entered in the left iliac area may damage:


A. Left ovary
B. liver
C. Stomach
D. The ascending colon
E. bladder
ANSWER: B

Trocar deeply entered in the left subcostal area may damage:


A. Ovaries
B. liver
C. Stomach
D. Uterus
E. bladder
ANSWER: C

Trocar deeply entered in the left subcostal area may damage:


A. Ovaries
B. liver
C. The tail of the pancreas
D. Uterus
E. bladder
ANSWER: C

Trocar deeply entered in the left subcostal area may damage:


A. Ovaries
B. liver
C. Left adrenal
D. Uterus
E. bladder
ANSWER: C

Trocar deeply entered the left side of the area can damage:
A. Ovaries
B. liver
C. Stomach
D. the descending colon
E. bladder
ANSWER: D

Trocar deeply entered the left upper quadrant can damage:


A. Ovaries
B. liver
C. Stomach
D. small intestine
E. bladder
ANSWER: D

Trocar deeply introduced in suprapubic area may damage:


A. The left iliac vessels
B. liver
C. bladder
D. The ascending colon
E. Stomach
ANSWER: C

Trocar deeply introduced in suprapubic area may damage:


A. The left iliac vessels
B. liver
C. the uterus
D. The ascending colon
E. Stomach
ANSWER: C

Trocar deeply introduced in the epigastric region may damage:


A. Stomach
B. Spleen
C. Kidneys
D. Uterus
E. bladder
ANSWER: A

Trocar deeply introduced in the epigastric region may damage:


A. pancreas
B. Spleen
C. Kidneys
D. Uterus
E. bladder
ANSWER: A

Trocar deeply introduced in the epigastric region may damage:


A. The left lobe of the liver
B. Spleen
C. Kidneys
D. Uterus
E. bladder
ANSWER: A

Trocar deeply introduced in the right subcostal area may damage:


A. Stomach
B. Gallbladder
C. Ovaries
D. Uterus
E. bladder
ANSWER: B

Trocar deeply introduced in the right subcostal area may damage:


A. Stomach
B. The structures of hepato-duodenal links
C. Ovaries
D. Uterus
E. bladder
ANSWER: B

Trocar deeply introduced in the right subcostal area may damage:


A. Stomach
B. The upper pole of the right kidney
C. Ovaries
D. Uterus
E. bladder
ANSWER: B

Trocar deeply introduced in the umbilical area may damage:


A. Kidneys
B. liver
C. Stomach
D. small intestine
E. bladder
ANSWER: D

At diagnostic laparoscopy with a picture of acute cholecystitis there are signs of


inflammation of the pancreas at such state as
A. Biliary acute pancreatitis
B. Focal necrotizing pancreatitis
C. Focal hemorrhagic necrotizing pancreatitis
D. Mixed pancreatic necrosis
E. Intact pancreas
ANSWER: A

Trocar deeply put into the right iliac area may damage:
A. Right ovary
B. liver
C. Stomach
D. The ascending colon
E. bladder
ANSWER: A

At initial stages liver surface is smooth, dark brown in color with a greenish
tinge. This statement is typical for laparoscopy at
A. Hemochromatosis of liver
B. Primary biliary cirrhosis
C. Obstructive cholangitis
D. Obstructive jaundice
E. Viral hepatitis
ANSWER: A

Trocar deeply put into the right iliac area may damage:
A. Right iliac vessels
B. liver
C. Stomach
D. The ascending colon
E. bladder
ANSWER: A

At laparoscopic examination
A. You can change the position of the patient
B. You can not change the position of the patient
C. You can change the position of the patient in the sagittal plane
D. You can change the position of the patient in the frontal plane
E. No need to change the position of the patient
ANSWER: A

Trocar deeply put into the right side of the area can damage:
A. Ovaries
B. liver
C. Stomach
D. the right kidney
E. bladder
ANSWER: D

Trocar introduced along the linea alba below the umbilicus on its way passes the
following layers of the abdominal wall:
A. The skin, the front piece aponeurosis, rectus muscle, peritoneum
B. The skin, subcutaneous tissue, anterior leaflet aponeurosis, rectus muscle,
peritoneum
C. The skin, subcutaneous fat, aponeurosis and peritoneum
D. skin, aponeurosis, preperitoneal fat and peritoneum
E. The skin, subcutaneous fat, anterior leaflet aponeurosis, rectus abdominis muscle,
posterior leaflet aponeurosis, preperitoneal fat and peritoneum
ANSWER: E

Trocar put on the left or right iliac areas is on its way passes the following layers
of the abdominal wall:
A. The skin, the front piece aponeurosis, rectus muscle, peritoneum
B. The skin, subcutaneous tissue, anterior leaflet aponeurosis, rectus muscle,
peritoneum
C. The skin, subcutaneous fat, aponeurosis and peritoneum
D. The skin, subcutaneous fat, aponeurosis of external oblique muscle, the internal
oblique and transverse muscles preperitoneal tissue, peritoneum
E. The skin, subcutaneous fat, aponeurosis, preperitoneal fat and peritoneum
ANSWER: D

Trocar put on the left or right subcostal areas on its way passes the following layers
of the abdominal wall:
A. The skin, the front piece aponeurosis, rectus muscle, peritoneum
B. The skin, subcutaneous tissue, anterior leaflet aponeurosis, rectus muscle,
peritoneum
C. The skin, subcutaneous fat, aponeurosis and peritoneum
D. The skin, subcutaneous fat, external, internal oblique and transverse muscles
preperitoneal tissue, peritoneum
E. The skin, subcutaneous fat, aponeurosis, preperitoneal fat and peritoneum
ANSWER: D

Urgent Care laparoscopy is indicated for all these states, except:


A. mesenteric thrombosis
B. hepatic subcapsular hematoma
C. acute pancreatitis
D. acute cholecystitis
E. jaundice
ANSWER: B

At laparoscopic examination enlarged and streched by contents gallbladder can be


seen at
A. acute simple cholecystitis
B. acute phlegmonous cholecystitis
C. acute gangrenous cholecystitis
D. acute gangrenous perforated cholecystitis
E. perforated cholecystitis
ANSWER: A

Vertical lines running along the outer edge of the rectus abdominis muscle separate
mesogastrium on following areas:
A. The left and right side
B. The left and right iliac
C. The left and right subcostal
D. All answers are correct
E. There is no right answer
ANSWER: A

At laparoscopic examination on the background of hyperemia fibrin layers can be


observed at
A. acute phlegmonous cholecystitis
B. acute cholecystitis simple
C. acute gangrenous cholecystitis
D. acute gangrenous cholecystitis, perforated
E. perforated cholecystitis
ANSWER: A

When planning therapeutic manipulation at acute pancreatitis laparoscope should be


introduced into the abdominal cavity through a:
A. upper right point of Calc
B. upper left point of Calc
C. lower left point of Calc
D. the white line of the abdomen above the pubis
E. at the point of McBurney
ANSWER: C

At laparoscopic examination at acute gangrenous cholecystitis characteristic


appearance of the gallbladder is following
A. the presence of black spots with fibrinous layers
B. enlarged and streched by contents gallbladder
C. hyperemia on the background and fibrinous layerings
D. the presence of black spots and leak of bile contents into abdominal cavity
E. Tightly wrapped bigger omentum around gallbladder
ANSWER: A

With the patient lying on his back at the lowered terminal end of the table well to
examine:
A. Gallbladder
B. Stomach
C. Recto-sigmoid colon angle
D. Uterus
E. Spleen
ANSWER: B
With the patient lying on his back at the lowered terminal end of the table well to
examine:
A. Gallbladder
B. Abdominal esophagus
C. the right lobe of the liver
D. Uterus
E. Spleen
ANSWER: B

With the patient lying on his back in a horizontal position table well to examine:
A. Gallbladder
B. Pancreas
C. small intestine
D. sigmoid
E. Spleen
ANSWER: C

With the patient lying on his back with lowered head end of the table well to examine:
A. Gallbladder
B. Pancreas
C. Recto-sigmoid colon angle
D. Uterus
E. Spleen
ANSWER: D

With the patient lying on his back with lowered head end of the table well to examine:
A. Gallbladder
B. Pancreas
C. Recto-sigmoid colon angle
D. bladder
E. Spleen
ANSWER: D

With the patient lying on his back with lowered head end of the table well to examine:
A. Gallbladder
B. Pancreas
C. Recto-sigmoid colon angle
D. uterine-bladder space
E. Spleen
ANSWER: D

With the patient lying on his back with lowered head end of the table well to examine:
A. Gallbladder
B. Pancreas
C. Recto-sigmoid colon angle
D. Ovaries
E. Spleen
ANSWER: D
With the patient lying on his right side with horizontal table well to examine:
A. Spleen
B. Pancreas
C. the right lobe of the liver
D. Uterus
E. Right side channel
ANSWER: A

At laparoscopic examination at acute simple cholecystitis characteristic appearance of


the gallbladder is following
A. enlarged and streched by contents gallbladder
B. hyperemia on the background and fibrinous layerings
C. the presence of black spots with fibrinous layers
D. the presence of black spots and leak of bile contents into abdominal cavity
E. tightly wrapped bigger omentum around gallbladder
ANSWER: A

With the patient lying on his right side with lowered head end table well to examine:
A. Gallbladder
B. Pancreas
C. Recto-sigmoid colon angle
D. The right ovary
E. Spleen
ANSWER: C

With the patient lying on the left side at the lowered terminal end of the table helps
to examine:
A. Spleen
B. Pancreas
C. duodenum
D. Uterus
E. Ovaries
ANSWER: C

With the patient lying on the left side in the horizontal position of the table helps
to examine:
A. Spleen
B. Pancreas
C. the right lobe of the liver
D. Uterus
E. The ascending colon
ANSWER: E

With the patient lying on the right side at the lowered terminal end of the table well
to examine:
A. Spleen
B. Pancreas
C. Kidneys
D. Uterus
E. Ovaries
ANSWER: A

At laparoscopy on suspicion of extrahepatic bile duct cancer, the second port input is
carried out
A. In the midline epigastrium
B. At Volkovych-Kocher point
C. On the midline of the abdomen in hypogastrium 10 cm below the navel
D. At McBurney point
E. At Kerr point
ANSWER: A

At laparoscopy on suspicion of liver cancer, the introduction of instrumental port is


performed
A. On the midline of the abdomen in the epigastrium 10 cm above the navel
B. In mesogastrium 1 cm above or below the navel the median line
C. At Volkovych-Kocher point
D. At McBurney point
E. At Kerr point
ANSWER: A

At laparoscopy on suspicion of liver cysts, the second input port is carried out at
A. Epigastrium
B. At Volkovych-Kocher point
C. On the midline of the abdomen in hypogastrium 10 cm below the navel
D. At McBurney point
E. At Kerr point
ANSWER: A

At laparoscopy on suspicion of right adnexitis, the first port site is


A. In mesogastrium 1 cm above or below the navel the median line
B. On the midline of the abdomen in the epigastrium 10 cm above the navel
C. On the midline of the abdomen in hypogastrium 10 cm below the navel
D. At McBurney point
E. At Volkovych-Kocher point
ANSWER: A

For laparoscopy for acute pancreatitis second port input is made


A. On the midline of the abdomen in the epigastrium
B. In mesogastrium 1 cm above or below the navel the median line
C. On the midline of the abdomen in hypogastrium 10 cm below the navel
D. At McBurney point
E. At Kerr point
ANSWER: A

For laparoscopy for acute simple appendicitis, instrumental port is set


A. Below the navel in hypogastrium
B. In mesogastrium 1 cm above or below the navel the median line
C. At midline abdominal epigastrium 5 cm above the umbilicus
D. At McBurney point
E. At Volkovich point
ANSWER: A

For laparoscopy for acute simple appendicitis, setting the port to perform laparoscopy
is conducted
A. At McBurney point
B. At Volkovich point
C. In mesogastrium 1 cm above or below the navel at the median line
D. At midline abdominal epigastrium 5 cm above the umbilicus
E. On the midline of the abdomen in hypogastrium 5 cm below the navel
ANSWER: C

A specific antyamebic medicine is:


A. amynoglicozides
B. metronidazol
C. emetine, chloroquine, diphosphate
D. quinine
E. cephalosporines
ANSWER: C

What solution is used for conducting of liquid hysteroscopy?


A. a 5% solution of glucose.
B. a 10% solution of chloride of sodium.
C. gemodez.
D. a 40% glucose solution.
E. polygluлin.
ANSWER: E

At laparoscopic examination of the presence of black spots with fibrinous layers


can be observed in
A. acute gangrenous cholecystitis
B. acute cholecystitis simple
C. acute phlegmonous cholecystitis
D. acute gangrenous cholecystitis, perforated
E. perforated cholecystitis
ANSWER: A

What sounding of uterine cavity is not used for?


A. for determination of permeability of cervical canal.
B. for determination straight of cervical canal.
C. for determination of length of uterine cavity.
D. for the exposure of tumors in the uterine cavity.
E. for the exposure of tumors of ovaries.
ANSWER: E

Cancer of the gallbladder under review is characterized by the following features


A. the gallbladder has whitish dense bulbs on the surface
B. the gallbladder is enlarged, streched, white and blue color
C. the gallbladder is enlarged, dark purple color, through extensive serous membrane
translucent mesh of blood vessels and capillaries
D. gallbladder glued with surrounding organs, constricted, scars or sclerosis
E. none of the listed
ANSWER: A

Chronic calculous cholecystitis under review is characterized by the following


features
A. gallbladder glued with surrounding organs, constricted, scars or sclerosis
B. the gallbladder is enlarged, streched, white and blue color
C. the gallbladder is enlarged, dark purple color, through extensive serous membrane
translucent mesh of blood vessels and capillaries
D. the gallbladder has whitish dense bulbs on the surface
E. none of the listed
ANSWER: A

During laparoscopic examination, surgeon pays attention to


A. shape and size of organs, their color, vascular pattern, signs of inflammation,
tumor, metastasis and free fluid in the abdomen
B. shape and size of organs, vascular pattern, signs of inflammation, tumor
metastasis and free fluid in the abdomen
C. shape and size of their color, signs of inflammation, tumor, metastasis and free
fluid in the abdomen
D. shape and size of their color, vascular pattern, signs of inflammation, free
fluid in the abdomen
E. shape and size of their color, vascular pattern, signs of inflammation, tumors
ANSWER: A

Edema of the gallbladder under viewed characterized by the following features


A. the gallbladder is enlarged, streched, white and blue color
B. the gallbladder is enlarged, dark purple color, through extensive serous membrane
translucent mesh of blood vessels and capillaries
C. gallbladder glued with surrounding organs, constricted, scars or sclerosis
D. the gallbladder has whitish dense bulbs on the surface
E. none of the listed
ANSWER: A

Enlarged and streched by contents gallbladder can be seen at


A. acute simple cholecystitis
B. normal gallbladder
C. acute gangrenous cholecystitis
D. acute gangrenous cholecystitis, perforated
E. perforated cholecystitis
ANSWER: A

Fibrinous layers can be observed in


A. acute phlegmonous cholecystitis
B. acute cholecystitis simple
C. norm
D. acute perforated gangrenous cholecystitis
E. perforated cholecystitis
ANSWER: A

Focal fatty pancreatic necrosis is characterized by


A. presence of scattered on the surface of large and small omentum or elsewhere
"stearic plaques"
B. presence of hemorrhagic fluid in the free abdominal cavity
C. dark red, brown or black pancreas, the presence of hemorrhagic fluid in the free
abdominal cavity, the presence of "stearic plaques"
D. visual picture of acute cholecystitis together with characteristic signs of
inflammation of the pancreas
E. There is no specific sign
ANSWER: A

For hemochromatosis of liver during laparoscopy is typical


A. liver surface smooth, dark brown
B. fine capillary net of blood vessels
C. liver is smooth gray-brown.
D. intense brownish-green color
E. moderate greenish tint
ANSWER: A

Greenish tint of liver is characteristic for laparoscopy at


A. Obstructive cholangitis
B. Hemochromatosis of liver
C. Primary biliary cirrhosis
D. Obstructive jaundice
E. Normally
ANSWER: A

In acute gangrenous cholecystitis characteristic appearance of the gallbladder is


following
A. the presence of black spots with fibrinous layers
B. enlarged and streched by contents gallbladder
C. hyperemia on the background and fibrinous layerings
D. the presence of black spots and leak of bile contents into abdominal cavity
E. Tightly wrapped bigger omentum around gallbladder
ANSWER: A

In acute phlegmonous cholecystitis characteristic appearance of the gallbladder is


following
A. hyperemia on the background and fibrinous layerings
B. enlarged and streched by contents gallbladder
C. the presence of black spots with fibrinous layers
D. the presence of black spots and leak of bile contents into abdominal cavity
E. Tightly wrapped bigger omentum around gallbladder
ANSWER: A
In obstructive cholangitis liver characterized
A. moderate greenish tint
B. fine capillary net of blood vessels and soft whitish mesh lymphatic vessels
C. during laparoscopy liver is enlarged, thick consistency. The surface of the liver
is smooth gray-brown
D. intense brownish-green color
E. at early stages of liver surface is smooth, dark brown in color with a greenish
tinge
ANSWER: A

In viral hepatitis for liver during laparoscopy most characteristic is


A. fine capillary net of blood vessels and soft whitish mesh of lymphatic vessels
B. liver enlarged, thick consistency, liver is smooth gray-brown.
C. intense brownish-green color
D. moderate greenish tint
E. liver surface is smooth, dark brown in color with a greenish tinge
ANSWER: A

Intact pancreas is characterized by


A. lobular gland structure
B. at the surface "of stearic plaques"
C. presence of hemorrhagic fluid in the free abdominal cavity
D. dark red pancreas
E. none of the listed
ANSWER: A

Laparoscopic diagnostic review is conducted


A. consistently better from right to left and top to bottom
B. consistently better left to right and top to bottom
C. not consistently better from right to left and from top to bottom
D. consistently better from right to left
E. consistently better from top to bottom
ANSWER: A

After the Veres needle introduction one must perform tests that indicate:
A. The pressure in the abdomen
B. Location of the distal end of the needle
C. Depth of anesthesia
D. All answers are correct
E. There is no right answer
ANSWER: B

Mixed pancreatic necrosis is characterized by


A. dark red, brown or black pancreas, the presence of hemorrhagic fluid in the free
abdominal cavity, the presence of "stearic plaques"
B. presence of scattered on the surface of large and small omentum or elsewhere
"stearic plaques"
C. presence of hemorrhagic fluid in the free abdominal cavity
D. visual picture of acute cholecystitis together with characteristic signs of
inflammation of the pancreas
E. There is no specific signs
ANSWER: A

Overview laparoscopy is conducted


A. sequentially from left to right and top to bottom
B. consistently better left to right and top to bottom
C. not consistently better from right to left and from top to bottom
D. not consistently better from right to left
E. consistently better from top to bottom
ANSWER: A

Primary biliary cirrhosis is characterized by the following at review


A. during laparoscopy liver is enlarged, thick consistency, the surface of the liver
is smooth gray-brown
B. fine capillary net of blood vessels and soft whitish mesh lymphatic vessels
C. intense brownish-green color
D. moderate greenish tint
E. surface of the liver is smooth, dark brown in color with a greenish tinge
ANSWER: A

Surface of liver is smooth, dark brown in color with a greenish tinge. This statement
is typical for laparoscopy at
A. Hemochromatosis of liver
B. Norm
C. Obstructive cholangitis
D. Obstructive jaundice
E. Viral hepatitis
ANSWER: A

The gallbladder is enlarged, dark purple color, through extensive serous membrane
translucent mesh of blood vessels and capillaries is in such pathologies as
A. Empyema of the gallbladder
B. Edema of the gallbladder
C. Chronic calculous cholecystitis
D. Cancer of the gallbladder
E. None of the
ANSWER: A

Where is a smear taken for oncocytology before hysteroscopy?


A. from the lateral fornix of vagina and cervical canal.
B. from vaginal part of uterine cervix.
C. from the back fornix of vagina and cervical canal.
D. from a cervical canal.
E. from vaginal part of uterine cervix and cervical canal.
ANSWER: E
The presence of black spots and leak of bile contents into abdominal cavity can be
seen at
A. acute perforated gangrenous cholecystitis
B. acute cholecystitis simple
C. acute phlegmonous cholecystitis
D. norm
E. perforated cholecystitis
ANSWER: A

The presence of hemorrhagic fluid in the free abdominal cavity indicates


A. Hemorrhagic necrotizing pancreatitis
B. Focal fatty pancreatic necrosis
C. Mixed pancreatic necrosis
D. Biliary pancreatitis
E. Intact pancreas
ANSWER: A

Tightly wrapped bigger omentum around gallbladder can prevent overview at


A. probable bladder perforation
B. acute cholecystitis simple
C. acute phlegmonous cholecystitis
D. acute gangrenous cholecystitis
E. acute gangrenous cholecystitis, perforated
ANSWER: A

To perform a diagnostic laparoscopy on suspected perforated ulcer, input of optical


port is typically carried out
A. In the periumbilical area
B. On the midline of the abdomen in the epigastrium
C. At Kerr point
D. At McBurney point
E. At Volkovych-Kocher point
ANSWER: A

To perform a diagnostic laparoscopy on suspicion of acute appendicitis, port for


laparoscope is typically placed
A. In the periumbilical area
B. On the midline of the abdomen in the epigastrium
C. At midline of abdominal wall 10 cm upwards the navel
D. At midline in hypogastrium 10 cm below the navel
E. At Volkovych-Kocher point
ANSWER: A

Together with visual picture of acute cholecystitis characteristic signs of


inflammation of the pancreas indicate
A. Biliary pancreatitis
B. Focal fatty pancreatic necrosis
C. Focal hemorrhagic necrotizing pancreatitis
D. Mixed pancreatic necrosis
E. Intact pancreas
ANSWER: A

When carrying out a diagnostic laparoscopy on suspicion of acute cholecystitis, input


of optical port is typically carried out
A. In the periumbilical area
B. At Kerr point
C. On the midline of the abdomen in hypogastrium
D. At McBurney point
E. At Volkovych-Kocher point
ANSWER: A

When carrying out a diagnostic laparoscopy on suspicion of cirrhosis, enter of the


first port is conducted
A. In the periumbilical area
B. On the midline of the abdomen in the epigastrium
C. On the midline of the abdomen in hypogastrium
D. At McBurney point
E. At Volkovych-Kocher point
ANSWER: A

When diagnostic laparoscopic examination in obstructive jaundice liver has


A. brownish-green intensive color
B. soft whitish mesh of lymphatic vessels
C. liver enlarged, thick consistency. The surface of the liver is smooth gray-brown
D. moderate greenish tint
E. liver surface is smooth, dark brown in color with a greenish tinge
ANSWER: A

When diagnostic laparoscopy attention is always payed to


A. signs of inflammation, tumor metastasis and free fluid in the abdominal
cavity, the shape and size of their color, vascular pattern
B. shape and size of organs, vascular pattern, signs of inflammation, tumor
metastasis and free fluid in the abdomen
C. shape and size of their color, signs of inflammation, tumor metastasis and free
fluid in the abdomen
D. shape and size of their color, vascular pattern, signs of inflammation, free
fluid in the abdomen
E. shape and size of their color, vascular pattern, signs of inflammation, tumors
ANSWER: A

When diagnostic laparoscopy body surface is smooth, the gallbladder is enlarged,


streched, white and blue color at
A. Edema of the gallbladder
B. Empyema of the gallbladder
C. Chronic non-calculous cholecystitis
D. Cancer
E. Norm
ANSWER: A
When diagnostic laparoscopy gallbladder empyema under review is as follows
A. the gallbladder is enlarged, dark purple color, through serous membrane there is
translucent mesh of blood vessels and capillaries
B. the gallbladder is enlarged, streched, white and blue color
C. gallbladder glued with surrounding organs, constricted, scars or sclerosis
D. the gallbladder has whitish dense bulbs on the surface
E. none of the listed
ANSWER: A

When diagnostic laparoscopy gallbladder edema during the inspection is visualized as


follows
A. the gallbladder is enlarged, streched, whitish and blue color
B. gallbladder of dark purple color extensive vascular net shines through the serosa
C. gallbladder glued with surrounding organs, constricted, scars or sclerosis
D. the gallbladder has whitish dense bulbs on the surface
E. none of the listed
ANSWER: A

When diagnostic laparoscopy, overview is achieved by


A. manipulation of table the patient is fixed to
B. manipulation of patient
C. manipulation of table to which the patient is fixed only sagittally
D. manipulation of table to which the patient is fixed only frontally
E. manipulation of table to which the patient is fixed only by height
ANSWER: A

At diagnostic laparoscopy the gallbladder is enlarged, dark purple color, through


extensive serous membrane translucent mesh of blood vessels and capillaries. It
characterizes
A. Empyema of the gallbladder
B. Edema of the gallbladder
C. Chronic non-calculous cholecystitis
D. Cancer
E. Normal gallbladder
ANSWER: A

Antibiotic therapy of liver abscesses should be based on the following principles


A. the use of broad-spectrum antibiotics
B. a combination of antibiotics
C. antibiotics only after determining the nature of its flora
D. degree in accounting bile excretion of antibiotics
E. all listed
ANSWER: E

Clinic extrahepatic portal hypertension does not include:


A. Splenomegaly with hypersplenism or without
B. Splenomegaly with hypersplenism
C. varicose veins of the esophagus
D. ascites
E. cholesterosis of gallbladder
ANSWER: D
A 28-year-old patient has been taken to a hospital withe acute pain in the lower
abdomen. There was a brief syncope. The delay of menstruation is 2 months. Objectively:
the patient has pale skin, BP- 90/50 mm Hg, Ps- 110/min. Lower abdomen is extremely
painful. Vaginal examination reveals uterus enlargement. There is positive Promtov’s
sign. Right adnexa are enlarged and very painful. What is the most likely diagnosis?
A. Right – sided tubal pregnancy
B. Right ovary apoplexy
C. Acute right-sided salpingoophoritis
D. Pelvioperitionitis
E. Incipient abortion
ANSWER: A

A 58-year-old female patient came to the antenatal clinic complaining of bloody


light-red discharges from the genital tracts. Menopause is 12 years. Gynaecological
examination revealed age involution of externalia and vagina; uterine cervix was
unchanged, there were scant bloody discharges from uterine cervix, uterus was of
normal size; uterine adnexa were not palpable; parametria were free. What is the most
likely diagnosis?
A. Uterine carcinoma
B. . Atrophic colpitic
C. Abnormalities of menstrual cycle of climacteric nature
D. Cervical carcinoma
E. Granulosa cell tumor of ovary
ANSWER: A

Full-term pregnancy. Body weight of the pregnant woman is 62 kg. The fetus has the
longitudinal lie, the fetal head is engaged to pelvic inlet. Abdominal circumference
is 100 cm. Fundal height is 35 cm. What is the approximate weight of the fetus?
A. 3kg 500 g
B. 4 kg
C. 2 kg 500 g
D. 3 kg
E. 4 kg 500 g
ANSWER: A

A newborn’s head is of dolichocephalic shape, that is front-to-back elongated.


Examination of the occipital region revealed a labour tumour located in the middle
between the prefontanel and posterior fontanel. Specify the type of fetal presentation:
A. Posterior vertex presentation
B. Anterior vertex presentation
C. Presentation of the bregma
D. Brow presentation
E. Face presentation
ANSWER: A

A 30-year-old multigravida has been in labour for 18 hours. 2 hours ago pushing stage
began. Fetal heart rate is clear, rhythmic, 136/min. Vaginal examination reveals the
complete cervical dilatation, the fetal head is in the pelvic outlet. Sagittal suture
corresponds with anteroposteror diemeter of outlet the occipital fontanel is near the
pubis. The patient has been diagnosed with primary uterine inertia. What is the
further tactics of labour management?
A. Outlet forceps
B. Labour stimulation
C. Cesarean section
D. Skin- head Ivanov`s forceps
E. Tsovianov method
ANSWER: A

A baby was born by a young smoker. The labour was complicated by uterine inertia,
difficult delivery of the baby’s head and shoulders. The baby’s Apgar score was 4.
Which of the following is a risk factor for a spinal cord injury?
A. Diffiult delivery of the head and shoulders
B. Young age of the mother
C. Pernicious habits
D. Uterine inertia
E. Chronic hypoxia
ANSWER: A

A patient complains of being unable to get pregnant for 5 years. A complete clinical
examination gave the following results: hormonal function is not impaired, urogenital
infection hasn’t been found, on hysterosalpingography both tubes were filled with the
contrast medium up to the isthmic segment, abdominal contrast was not visualized. The
patient’s husband is healthy. What tactics will be most effective?
A. In-vitro fertilization
B. Insemination with husband`s sperm
C. ICSI within in-vitro fertilization program
D. Hydrotubation
E. Laperascopic tubal plasty
ANSWER: A

A 55-year-old patient whose menstruation stopped 5 years ago complains of vaginal


dryness, frequent and painful urination. Gynecologist revealed signs of atrophic
colpitis. Urine analysis revealed no peculiarities. Which locally acting product will
provide the proper therapeutic effect?
A. Vaginal suppositories “Ovestin”
B. Vaginal tablets “Tergynan”
C. Vaginal cream “Meratin Combi”
D. Vaginal gel “Metronidazole”
E. Vaginal cream “Dalacin”
ANSWER: A

A 49-year-old patient complains of itching, burning in the external genitals, frequent


urination. The symptoms has been present for the last 7 months. The patient has
irregular menstruation, once every 3-4 months. Over the last 2 years she presents with
hot flashes, sweating, sleep disturbance. Examination revealed no pathological changes
of the internal reproductive organs. Complete blood count and urinalysis showed no
pathological changes. Vaginal smear contained 20-25 leukocytes in the field of vision,
mixed flora. What is the most likely diagnosis?
A. Menopauzual syndrome
B. Cystitis
C. Trichomonas colpitis
D. Vulvitis
E. Bacterial vaginosis
ANSWER: A

Examination of a Rh-negative pregnant woman at 32 weeks of gestation revealed a four-


time rise of Rh-antibody titer within 2 weeks, the titer was 1:64. In the first two
pregnancies the patient had experienced antenatal fetal death due to hemolytic disease.
What is the optimal tactics of pregnancy management?
A. Early delivery
B. delivery at 37 weeks of gestation
C. Screening for Rh-antibodies 2 weeks later and early delivery in case of further
titer rise
D. Introdution of anti-Rh (D) immunoglobulin
E. Ultrasound for signs of hemolytic disease of the fetus
ANSWER: A

3 months after the first labor a 24- year-old patient complained of amenorrhea.
Pregnancy ended by Caesarian section because of premature separation of normally
located placenta. Blood loss is 2000 ml due to disturbance of blood clotting.
Choose the most suitable investigation:
A. Estimation of gonadotropin level
B. US of small pelvis
C. Progesteron assay
D. Computer tomography of head
E. Estimation of testosteron rate in blood serum
ANSWER: A

A 24-year-old primipara was hospitalised with complaints of discharge of the amniotic


waters. The uterus is tonic on palpation. The position of the fetus is longitudinal,
it is pressed with the head to pelvic outlet. Palpitation of the fetus is rhythmical,
140 bpm, auscultated on the left below the navel. Internal examination: cervix of the
uterus is 2,5 cm long, dense, the external opening is closed, light amniotic waters
are discharged. Point out the correct component of the diagnosis:
A. Antenatal discharge of the amniotic waters
B. Early discharge of the amniotic waters
C. The beginning of the 1st stage of labour
D. The end of the 1st stage of labour
E. Pathological preterm labour
ANSWER: A

A 59-year-old female patient admitted to the maternity clinic with complains of


bloody discharge from the vagina. Menopause is 12 years. Speculum examination: uterus
cervix was not erosive, small amount of bloody discharge came from the cervical canal.
Uterus is of normal size, adnexa are not palpable. Fornices were deep and painless.
What method should be applied for the diagnosis specification?
A. Diuatation and curatage
B. Laparoscopy
C. Culdocentesis
D. Extensive colposcopy
E. Culdoscopy
ANSWER: A

A 28-year-old woman has bursting pain in the lower abdomen during menstruation;
chocolate-like discharges from vagina are observed. It is known from the anamnesis
that the patient suffers from chronic adnexitis. Bimanual examination revealed a
tumour-like formation of heterogenous consistency 7х7 cm large to the left from the
uterus. The formation is restrictedly movable, painful when moved. What is the most
probable diagnosis?
A. Endometrioid cyst of the left ovary
B. Follicular cyst of the left ovary
C. Fibromatous node
D. Exacerbation of chronic adnexitis
E. Tumour of sigmoid colon
ANSWER: A

A 30-year-old parturient woman was delivered to a maternity hospital with full-term


pregnancy. She complains of severe lancinating pain in the uterus that started 1 hour
ago, nausea, vomiting, cold sweat. Anamnesis states cesarean section 2 years ago.
Uterine contractions stopped. Skin and mucous membranes are pale. Heart rate is
100/min, BP is 90/60 mm Hg. Uterus has no clear margins, is sharply painful. No
heartbeat can be auscultated in the fetus. Moderate bloody discharge from the uterus
can be observed. Uterus cervix is 4 cm open. Presenting part is not visible. The most
likely diagnosis is:
A. Uterine rupture
B. Initial uterine rupture
C. Threatened uterine rupture
D. Premature detachment of normally positioned placenta
E. Compression of inferior pudendal vein
ANSWER: A

A 26-year-old woman has attended maternity center complaining of her inability to


become pregnant despite 3 years of regular sex life. Examination revealed the
following: increased body weight; male-type pubic hair; excessive pilosis of thighs;
ovaries are dense and enlarged; basal body temperature is monophasic. The most likely
diagnosis is:
A. Ovaries sclerocystosis
B. Inflammation of uterine appendages
C. Adrenogenital syndrome
D. Premenstrual syndrome
E. Gonadal dysgenesis
ANSWER: A

28-year-old woman complains of increased intermenstrual periods up to 2 months,


hirsutism. Gynaecological examination revealed that the ovaries were enlarged,
painless, compact, uterus had no pecularities. Pelvic ultrasound revealed that the
ovaries were 4-5 cm in diameter and had multiple enlarged follicles on periphery. X-
ray of skull base showed that sellar region was dilated. What is the most probable
diagnosis?
A. Stein-Leventhal syndrome (Polycystic ovary syndrome)
B. Algodismenorrhea
C. Sheehan’s syndrome
D. Premenstrual syndrome
E. Morgagni-Stewart syndrome
ANSWER: A

A woman consulted a therapeutist about fatigability, significant weight loss, weakness,


loss of appetite. She has been having amenorrhea for 8 months. A year ago she born a
full-term child. Haemorrhage during labour made up 2 l. She got blood and blood
substitute transfusions. What is the most probable diagnosis?
A. Sheehan’s syndrome
B. Stein-Leventhal syndrome
C. Shereshevsky-Turner’s syndrome
D. Homological blood syndrome
E. Vegetovascular dystonia
ANSWER: A

A parturient woman is 27 year old, it was her second labour, delivery was at full-term,
normal course. On the 3rd day of postpartum period body temperature is 36,8 0C, heart
rate - 72/min, BP - 120/80 mm Hg. Mammary glands are moderately swollen, nipples are
clean. Abdomen is soft and painless. Fundus of uterus is 3 fingers below the umbilicus.
Lochia are bloody, moderate. What is the most probable diagnosis?
A. Physiological course of postpartum period
B. Subinvolution of uterus
C. Postpartum metroendometritis
D. Remnants of placental tissue after labour
E. Lactostasis
ANSWER: A

A 37-year-old woman complains of sharp pains in her external genitalia, edema of the
vulvar lips, pain when walking. Objectively: body temperature is 38, 7 0C, heart rate
is 98/min. Inside the right lover part of labic major there is a dense, painful,
tumor-like growth 5,0х4,5 cm in size; skin and mucosa of the external genitalia are
hyperemic, copious foul-smelling discharge is observed. The most likely diagnosis is:
A. Acute bartholinitis
B. Furuncle of outer labia
C. Acute vulvovaginitis
D. Bartholin’s cyst
E. Carcinoma of vulva
ANSWER: A

Examination of placenta revealed a defect. An obstetrician performed manual


investigation of uterine cavity, uterine massage. Prophylaxis of endometritis in the
postpartum period should involve the following actions:
A. Antibacterial therapy
B. Instrumental revision of uterine cavity
C. Haemostatic therapy
D. Contracting agents
E. Intrauterine instillation of dioxine
ANSWER: A

On the 10th day postpartum a puerperant woman complains of pain and heaviness in the
left mammary gland. Body temperature is 38, 8 0C, Ps- 94 bpm. The left mammary gland
is edematic, the upper external quadrant of skin is hyperemic. Fluctuation symptom is
absent. The nipples discharge drops of milk when pressed. What is a doctor’s further
tactics?
A. Antibiotic therapy, immobilization and expression of breast milk
B. Compress to both mammary glands
C. Inhibition of lactation
D. Physiotherapy
E. Opening of the abscess and drainage of the mammary gland 180
ANSWER: A

A 25-year-old patient during selfexamination detected a tumor in the upper external


quadrant of the right mammary gland. On palpation: painless, dense, mobile growth 2 cm
in diameter is detected in the mammary gland; no changes in the peripheral lymph nodes
are observed. On mammary glands US: in the upper external quadrant of the right
mammary gland there is a space-occupying lesion of increased echogenicity 21х18 mm in
size. The most likely diagnosis is:
A. Fibrous adenoma
B. Lacteal cyst
C. Diffuse mastopathy
D. Breast cancer
E. Mastitis
ANSWER: A

A maternity patient breastfeeding for 1,5 weeks has attended a doctor. She considers
the onset of her disease to be when proportional breast engorgement occurred. Mammary
glands are painful. Body temperature is 36, 6 0C. Expression of breast milk is
hindered. The most likely diagnosis is:
A. Lactostasis
B. Infiltrative mastitis
C. Suppurative mastitis
D. Chronic cystic mastitis
E. Gangrenous mastitis
ANSWER: A

An Rh-negative woman with 32-weeklong term of pregnancy has been examined. It was
observed that Rh-antibodies titer had increased four times within the last 2 weeks and
was 1:64. First two pregnancies ended in antenatal death of fetus caused by hemolytic
disease. What tactics of pregnancy management should be chosen?
A. Preterm delivery
B. Delivery at 37 weeks term
C. Rh-antibody test in 2 weeks; if Rh-antibodies increase in number conduct delivery
D. Introduction of anti-Rh immunoglobulin
E. US examination to determine signs of fetal erythroblastosis
ANSWER: A

A 48-year-old patient was delivered to a hospital in-patient unit with uterine


bleeding that occurred after the 2-week-long delay of menstruation. Anamnesis states
single birth. Examination of the uterine cervix with speculum mirrors revealed no
pathologies. On bimanual examination: uterus is of normal size, painless, mobile;
uterine appendages have no changes. Discharge is bloody and copious. What primary
hemostatic measure should be taken in the given case?
A. Fractional curettage of uterine cavity
B. Hormonal hemostasis
C. Hemostatics
D. Uterine tamponade
E. Uterotonics
ANSWER: A

A 30-year-old woman complains of infertility during her 10-year-long married life.


Menstruations occur since she was 14 and are irregular, with delays up to a month and
longer. Body mass is excessive. Hirsutism is observed. On bimanual examination:
uterine body is decreased in size; ovaries are increased in size, dense, painless, and
mobile. The most likely diagnosis is:
A. Stein–Leventhal syndrome (Polycystic ovary syndrome)
B. Follicular cyst of ovaries
C. Genital endometriosis
D. Genital tuberculosis
E. Inflammatory tumor of ovaries
ANSWER: A

.A 50-year-old female patient complains of aching pain in the lower abdomen. She has a
history of normal menstrual cycle. At the age of 40, the patient underwent a surgery
for gastric ulcer. Examination findings: abdomen is soft, in the hypogastrium there is
a well-defined nodular tumor of limited mobility. Vaginal examination findings: the
cervix is clean, of cylindrical shape. Body of the uterus cannot be palpated
separately. On both sides of the uterus palpation reveals tight tumors with an uneven
surface. The tumors are immobile andl fill the whole pelvic cavity. What is the most
likely diagnosis?
A. Krukenberg tumor
B. Ovarian fibroid
C. Ovarian granulose cell tumor
D. Bilateral pioovarium
E. Subserous metrofibrioma
ANSWER: A

A 13-year-old girl was admitted to the gynecology department for having a significant
bleeding from the vagina for 10 days. The patient has a history of irregular menstrual
cycle since menarche. Menarche occurred at the age of 11. Recto-abdominal examination
revealed no pathology. What is the provisional diagnosis?
A. Juvenile uterine bleeding
B. Adenomyosis
C. Injury of the external genitalia
D. Werlhof’s disease
E. Endometrial polyp
ANSWER: A

A 21-year-old female patient consulted a gynecologist about itching, burning, watery


vaginal discharges with a fish-like smell. Speculum examination revealed that the
cervical and vaginal mucosa was of a normal pink color. Vaginal examination revealed
no alterations of the uterus and adnexa. Gram-stained smears included clue cells. What
is the most likely pathology?
A. Bacterial vaginosis (gardnerellosis)
B. Chlamydiosis
C. Gonorrhea
D. Trichomoniasis
E. Candidiasis
ANSWER: A

A 26-year-old secundipara at 40 weeks of gestation arrived at the maternity ward after


the beginning of labor activity. 2 hours before, bursting of waters occurred. The
fetus was in a longitudinal lie with cephalic presentation. Abdominal circumference
was 100 cm, fundal height - 42 cm. Contractions occurred every 4-5 minutes and lasted
25 seconds each. Internal obstetric examination revealed cervical effacement, opening
by 4 cm. Fetal bladder was absent. Fetal head was pressed against the pelvic inlet.
What complication arose in childbirth?
A. Early amniorrhea
B. Primary uterine inertia
C. Secondary uterine inertia
D. Discoordinated labor
E. Clinically narrow pelvis
ANSWER: A

A 39-year-old female patient complains on dyspnea when walking, palpitation, edemata


in the evening. The patient’s height is 164 cm, weight - 104 kg. Objectively:
overnutrition. Heart sounds are weak, and tachycardia is present. The menstrual cycle
is not broken. Blood sugar is 5,6 mmol/l, ACTH-response tests revealed no alterations.
X-ray of the Turkish saddle revealed no pathology. What disease is it?
A. Alimentary obesity
B. Climax
C. Pituitary obesity
D. Diabetes mellitus
E. Cushing’s syndrome (primary hypercorti-solism)
ANSWER: A

A 28-year-old female patient has been admitted to the gynecology department for
abdominal pain, spotting before and after menstruation for 5 days. The disease is
associated with the abortion which she had 2 years ago. Antiinflammatory treatment had
no effect. Bimanual examination findings: the uterus is enlarged, tight, painful,
smooth. Hysteroscopy reveals dark red holes in the fundus with dark blood coming out
of them. What diagnosis can be made on the grounds of these clinical presentations?
A. Internal endometriosis
B. Polymenorrhea
C. Hypermenorrhea
D. Submucous fibromatous node
E. Dysfunctional uterine bleeding
ANSWER: A

A woman at 30 weeks pregnant has had an attack of eclampsia at home. On admission to


the maternity hospital BP- 150/100 mm Hg. Estimated fetal weight is 1500 g. There is
face and shin edema. Urine protein is 0,66g /l. Uterine cervix is 5 poin by Bishop
scale. An intensive complex therapy has been started. What is the correct tactics of
this case management?
A. Delivery by cesarean section
B. Continue therapy and prolong pregnancy for 1-2 weeks
C. Continue therapy and prolong pregnancy for 3-4 weeks
D. Labor induction by intravenous oxytocin or prostaglandins
E. Treat preeclampsia and achieve the delivery by way of conservative management
ANSWER: A

A pregnant 26-year-old woman was admitted to a hospital for abdominal pain and
bleeding from the genital tract. Bimanual examination revealed that uterus was the
size of 9 weeks of pregnancy, the cervical canal let a finger through. Fetal tissues
could be palpated in the orifice. There was moderate vaginal bleeding. What is the
tactics of choice?
A. Instrumental extraction of fetal tissue
B. Surveillance
C. Administration of hormones
D. Hemostatic and antianemic therapy
E. Therapy for the maintenance of pregnancy
ANSWER: A

A 36-year-old female pesented to a gynecological hospital with a significant bleeding


from the genital tract and a 1-month delay of menstruation. Bimanual examination
revealed soft barrel-shaped cervix. Uterus was of normal size, somewhat softened.
Appendages were unremarkable on both sides. Speculum examination revealed that the
cervix was cyanotic, enlarged, with the the external orifice disclosed up to 0,5 cm.
Urine hCG test was positive. What is the most likely diagnosis?
A. Cervical pregnancy
B. Uterogestation
C. Abortion in progress
D. Threatened miscarriage
E. Ectopic pregnancy
ANSWER: A

An 18-year-old girl complains of breast pain and engorgement, headaches, irritability,


swelling of the lower extremities. These symptoms have been observed since menarche
and occur 3-4 days before the regular menstruation. Gynecological examination revealed
no pathology. Make a diagnosis:
A. Premenstrual syndrome
B. Neurasthenia
C. Renal disease
D. Mastopathy
E. Cardiovascular disorder
ANSWER: A

A 25-year-old female presented to a women’s welfare clinic and reported the inability
to get pregnant within 3 years of regular sexual activity. Examination revealed
increased body weight, male pattern of pubic hair growth, excessive pilosis of thighs,
dense enlarged ovaries, monophasic basal temperature. What is the most likely
diagnosis?
A. Polycystic ovarian syndrome
B. Adnexitis
C. Adrenogenital syndrome
D. Premenstrual syndrome
E. Gonadal dysgenesis
ANSWER: A

A 23-year-old female consulted a gynecologist on the 20th day postpartum period about
pain in the left breast, purulent discharge from the nipple. Objectively: Ps-120/min,
t - 39 0C . The left breast is painful, larger than the right one, hyperemic. In the
upper quadrant there is an infiltrate sized 10x15 cm with a softening inside. Blood
test results: ESR- 50 mm/h, WBC- 15, 0 109/l. What is the tactics of choice?
A. Refer to the surgical department for operati-ve treatment
B. Refer to the gynecology department
C. Refer to the postpartum department
D. Refer to a polyclinic surgeon for conservati-ve treatment
E. Lance the breast abscess in the women’s health clinic
ANSWER: A

A puerperant is 28 years old. It’s the 3rd day post-partum after a second, normal,
term delivery. The body temperature is of 36, 8 0 C , Ps- 72/min, BP- 120/80 mm Hg.
Mammary glands are moderately engorged, the nipples are clean. Abdomen is soft,
painless. The fundus is 3 fingers’ breadth below the umbilicus. Moderate bloody lochia
are present. What diagnosis can be made?
A. Physiological course of the postpartum period
B. Subinvolution of uterus
C. Postpartum metroendometritis
D. Remains of placental tissue after childbirth
E. Lactostasis
ANSWER: A

A puerperant is 32 years old, it’s her first childbirth, term precipitate labor, the
III period is unremarkable, the uterus is contracted, tight. Examination of the birth
canal revealed a rupture in the left posterior vaginal wall that was gutured by catgut.
Two hours later, the patient complained on a feeling of pressure on the anus, pain in
the perineum, minor vaginal discharges, edema of the vulva. These clinical
presentations are indicative most likely of:
A. Vaginal hematoma
B. Hysterocervicorrhexis
C. Hemorrhoids
D. Hysterorrhesis
E. Hypotonic bleeding
ANSWER: A

A 31-year-old female patient complains of infertility, amenorrhea for 2 years after


the artificial abortion that was complicated by endometritis. Objective: examination
of the external genitalia reveals no pathology, there is female pattern of hair
distribution. According to the functional tests, the patient has biphasic ovulatory
cycle. What form of infertility is the case?
A. Uterine
B. Ovarian
C. Pituitary
D. Hypothalamic
E. Immunological
ANSWER: A

A female patient complains of being unable to get pregnant for 5 years. A complete
clinical examination brought the following results: hormonal function is not impaired,
urogenital infection hasn’t been found, on hysterosalpingography both tubes were
filled with the contrast medium up to the isthmic segment, abdominal contrast was not
visualized. The patient’s husband is healthy. What tactics will be most effective?
A. In-vitro fertilization
B. Insemination with husband’s sperm
C. ICSI within in-vitro fertilization program
D. Hydrotubation
E. Laparoscopic tubal plasty
ANSWER: A

A 19-year-old primiparous woman with a body weight of 54,5 kg gave birth at 38 weeks
gestation to a full-term live girl after a normal vaginal delivery. The girl’s weight
was 2180,0 g, body length - 48 cm. It is known from history that the woman has been a
smoker for 8 years, and kept smoking during pregnancy. Pregnancy was complicated by
moderate vomiting of pregnancy from 9 to 12 weeks pregnant, edemata of pregnancy from
32 to 38 weeks. What is the most likely cause of low birth weight?
A. Fetoplacental insufficiency
B. Low weight of the woman
C. Woman’s age
D. First trimester preeclampsia
E. Third trimester preeclampsia
ANSWER: A

A 23-year-old primigravida at 39 weeks gestation has been admitted to the maternity


ward with irregular contractions. The intensity of uterine contractions is not
changing, the intervals between them stay long. Bimanual examination reveals that the
cervix is centered, soft, up to 1,5 cm long. There is no cervical dilatation. What
diagnosis should be made?
A. Pregnancy I, 39 weeks, preliminary period
B. Pregnancy I, 39 weeks, labor I, 1 period, the latent phase
C. Pregnancy I, 39 weeks, labor I, period 1, the active phase
D. Pregnancy I, 39 weeks, birth I, 1 period, the acceleration phase
E. Pregnancy I, 39 weeks, pathological prelimi-nary period
ANSWER: A

On the 10th day postpartum a puerperant woman complains of pain and heaviness in the
left breast. Body temperature is 38, 8 0C , Ps- 94 bpm. The left breast is edematic,
the supero-external quadrant of skin is hyperemic. Fluctuation symptom is absent. The
nipples di-scharge drops of milk when pressed. What is a doctor’s further tactics?
A. Antibiotic therapy, immobilization and expression of breast milk
B. Compress to both breasts
C. Inhibition of lactation
D. Physiotherapy
E. Opening of the abscess and drainage of the breast
ANSWER: A

During the breast self-exam a 37-year-old female patient revealed a lump in the lower
inner quadrant of her left breast. Palpation confirms presence of a mobile well-
defined neoplasm up to 2 cm large. Peripheral lymph nodes are not changed. What is the
way of further management?
A. Ultrasound examination, mammography, fine-needle aspiration bi-opsy
B. Anti-inflammatory therapy, physiotherapy
C. Radical mastectomy
D. Ultrasound monitoring of genitals during the entire course of antiestrogens therapy,
systemic enzyme therapy, phytotherapy
E. Case follow-up
ANSWER: A

A 25-year-old female has a self-detected tumor in the upper outer quadrant of her
right breast. On palpation there is a painless, firm, mobile lump up to 2 cm in
diameter, peripheral lymph nodes are not changed. In the upper outer quadrant of the
right breast ultrasound revealed a massive neoplasm with increased echogenicity sized
21x18 mm. What is the most likely diagnosis?
A. Fibroadenoma
B. Lactocele
C. Diffuse mastopathy
D. Mammary cancer
E. Mastitis
ANSWER: A

On dmission a 35-year-old female reports acute abdominal pain, fever up to 38, 8 0C,
mucopurulent discharges. The patient is nulliparous, has a history of 2 artificial
abortions. The patient is unmarried, has sexual contacts. Gynecological examination
reveals no uterus changes. Appendages are enlarged, bilaterally painful. There is
profuse purulent vaginal discharge. What study is required to confirm the diagnosis?
A. Bacteriologic and bacteriascopic studies
B. Hysteroscopy
C. Curettage of uterine cavity
D. Vaginoscopy
E. Laparoscopy
ANSWER: A

A 20-year-old female consulted a gynecologist about not having menstrual period for 7
months. History abstracts: early childhood infections and frequent tonsillitis,
menarche since 13 years, regular monthly menstrual cycle of 28 days, painless
menstruation lasts 5-6 days. 7 months ago the patient had an emotional stress.
Gynecological examination revealed no alterations in the uterus. What is the most
likely diagnosis?
A. Secondary amenorrhea
B. Primary amenorrhea
C. Algomenorrhea
D. Spanomenorrhea
E. Cryptomenorrhea
ANSWER: A

A 48-year-old female has been admitted to the gynecology department for pain in the
lower right abdomen and low back pain, constipations. Bimanual examination reveals:
the uterus is immobile, the size of a 10-week pregnancy, has irregular surface.
Aspirate from the uterine cavity contains atypical cells. What is the most appropriate
diagnosis?
A. Hysterocarcinoma
B. Cervical cancer
C. Metrofibroma
D. Colon cancer
E. Chorionepithelioma
ANSWER: A

A pregnant woman is 28 years old. Anamnesis: accelerated labor complicated by the II


degree cervical rupture. The following two pregnancies resulted in spontaneous
abortions at the terms of 12 and 14 weeks. On speculum examination: the uterine cervix
is scarred from previous ruptures at 9 and 3 hours, the cervical canal is gaping. On
vaginal examination: the cervix is 2 cm long, the external orifice is open 1 cm wide,
the internal orifice is half-open; the uterus is enlarged to the 12th week of
pregnancy, soft, mobile, painless, the appendages are without changes. What diagnosis
would you make?
A. Isthmico-cervical insufficiency, habitual noncarrying of pregnancy
B. Threatened spontaneous abortion
C. Incipient abortion, habitual noncarrying of pregnancy
D. Cervical hysteromyoma, habitual noncarryi-ng of pregnancy
E. Cervical pregnancy, 12 weeks
ANSWER: A

An infant has been born at the 41st week of gestation. The pregnancy was complicated
with severe gestosis of the second semester. The weight of the baby is 2400 g, the
height is 50 cm. Objectively: the skin is flabby, the layer of subcutaneous fat is
thin, hypomyotonia, neonatal reflexes are weak. The internal organs are without
pathologic changes. This newborn can be estimated as a:
A. Full-term infant with prenatal growth retardation
B. Premature infant
C. Immature infant
D. Postmature infant
E. Full-term infant with normal body weight
ANSWER: A

A full term baby born from the 1st noncomplicated pregnancy with complicated labor was
diagnosed with cephalohematoma. On the 2nd day of life the child developed jaundice;
on the 3rd day of life there appeared neurological changes: nystagmus, Graefe syndrome.
Urine is yellow, feces are golden-yellow. The mother’s blood group is А (II) Rh-, the
child’s - А (II) Rh+. On the 3rd day the results of the child’s blood test are as
follows: Hb- 200 g/l, erythrocytes - 6, 1 1012/l, blood bilirubin - 58 mcmol/l due to
the presence of its unconjugated fraction, Ht- 0,57. In this case the jaundice is
caused by:
A. Craniocerebral birth injury
B. Physiologic jaundice
C. Hemolytic disease of newborn
D. Atresia of bile passages
E. Fetal hepatitis
ANSWER: A

A 26-year-old woman, who gave birth 7 months ago, has been suffering from nausea,
morning sickness, somnolence for the last 2 weeks. The patient breasfeeds; no
menstruation. She has been using no means of contraception. What method would be most
efficient in clarification of the diagnosis?
A. Ultrasound
B. Small pelvis radiography
C. Palpation of mammary glands and squeezi-ng out colostrum
D. Bimanual abdominovaginal examination
E. Mirror examination
ANSWER: A

A 30-year-old parturient woman was deli-vered to a maternity hospital with full-term


pregnancy. She complains on severe lancinating pain in the uterus that started 1 hour
ago, nausea, vomiting, cold sweat. Anamnesis states cesarean section 2 years ago.
Uterine contractions stopped. Skin and mucous membranes are pale. Heart rate is
100/min., BP is 90/60 mm Hg. Uterus has no clear margins, is sharply painful. No
heartbeat can be auscultated in the fetus. Moderate bloody discharge from the uterus
can be observed. Uterus cervix is 4 cm open. Presenting part is not visible. The most
likely diagnosis is:
A. Uterine rupture
B. Initial uterine rupture
C. Threatened uterine rupture
D. Premature detachment of normally positi-oned placenta
E. Compression of inferior pudendal vein
ANSWER: A

During the dynamic observation of a parturient woman in the second stage of labor it
was registered that the fetal heart rate decreased to 90-100/min. and did not normali-
ze after contractions. Vaginal examination revealed the complete cervical dilatation,
the fetal head filling the entire posterior surface of the pubic symphysis and sacral
hollow; the sagittal suture was in the anteroposteri-or diameter of the pelvic outlet,
the posterior fontanelle was in front under the pubic arch. What plan for further
labour management should be recommended?
A. Application of forceps minor
B. Caesarean section
C. Episiotomy
D. Application of cavity forceps
E. Stimulation of labour activity through intravenous injection of oxytocin
ANSWER: A

A 16-year-old girl has primary amenorrhea, no pubic hair growth, normally developed
mammary glands; her genotype is 46 ХY; uterus and vagina are absent. What is your
diagnosis?
A. Testicular feminization syndrome
B. Mayer- Rokitansky- Kuster-Hauser syndrome
C. Cushing’s syndrome
D. Sheehan syndrome
E. Cushing’s disease
ANSWER: A

A 27-year-old sexually active woman complains of numerous vesicles on the right sex
lip, itch and burning. Eruptions regularly appear before menstruation and disappear 8-
10 days later. What is the most likely di-agnosis?
A. Herpes simplex virus
B. Bartholinitis
C. Primary syphilis
D. Cytomegalovirus infection
E. Genital condylomata
ANSWER: A

A 35-year-old woman addressed a gynecological in-patient department with complaints of


regular pains in her lower abdomen, which increase during menstruation, and dark-brown
sticky discharge from the genital tracts. On bimanual examination: the uterine body is
slightly enlarged, the appendages are not palpated. Speculum examination of the
uterine cervix reveals bluish spots. What diagnosis is most likely?
A. Cervical endometriosis
B. Cervical erosion
C. Cervical polyp
D. Cervical cancer
E. Cervical fibroid
ANSWER: A
40 years old primapara is presented on 42-43 week of pregnancy. Uterine contractions
are weak. Fetal head is arrested to pelvic inlet. Vasten sign is positive. Fetal
heart rate is 140 bmp, rhythmic. Cervical dilation is 6 cm. Cranial bones are dense,
sagittal suture is not palpated. What is the management of labor?
A. Caesarean section
B. Amniotomy
C. Forceps delivery
D. Labor induction
E. Vacuum application
ANSWER: A

40 years old primapara is presented on 42-43 week of pregnancy. Uterine contractions


are weak. Fetal head is arrested to pelvic inlet. Vasten sign is positive. Fetal
heart rate is 140 bmp, rhythmic. Cervical dilation is 6 cm. Cranial bones are dense,
sagittal suture is not palpated. Which complication is presented?
A. Cephalo-pelvic disproportion
B. Fetal distress
C. Uterine inertia
D. False labor
E. Hypertonic uterine dysfunction
ANSWER: A

40 years old primapara is presented on 42-43 week of pregnancy. Uterine contractions


are weak. Fetal head is arrested to pelvic inlet. Vasten sign is positive. Fetal
heart rate is 140 bmp, rhythmic. Cervical dilation is 7 cm. Cranial bones are dense,
sagittal suture is not palpated. What is the reason of labor complication?
A. Postterm pregnancy
B. Deflexed presentation
C. Breech presentation
D. Multiple pregnancy
E. Polyhydramnios
ANSWER: A

39 years old primapara at 40 week of pregnancy is admitted to the hospital with


complaints of regular uterine contractions every 2- 3minutes by 45 seconds.
Longitudinal fetal lie, cephalic presentation was diagnosed. Vasten sign is positive.
Fetal heart rate is 142 in 1min, clear, rhythmic. The sizes of pelvis are normal.
Probable fetal weight is 4200 g. Uterine cervix is dilated to 8 cm, edematous.
Amniotic membranes are absent. Fetal head is above the pelvic inlet. Promontorium is
not reached. What is the adequate management of labor?
A. Immediate cesarean section
B. Fetal destroying operation
C. Medical treatment
D. Augmentation of labor
E. Spasmolytic prescription
ANSWER: A

39 years old primapara at 40 week of pregnancy is admitted to the hospital with


complaints of regular uterine contractions every 2- 3minutes by 45 seconds.
Longitudinal fetal lie, cephalic presentation were diagnosed. Vasten sign is positive.
Fetal heart rate is 142 in 1min, clear, rhythmic. The sizes of pelvis are normal.
Probable fetal weight is 4200 g. Uterine cervix is dilated to 8 cm, edematous.
Amniotic membranes are absent. Fetal head is above the pelvic inlet. Promontorium is
not reached. Which of the below pelvic sizes are considered to be normal for such
woman?
A. 20-22-24-29cm
B. 22-22-24-28cm
C. 25-28-30-20cm
D. 28-28-30-20cm
E. 23-24-25-26cm
ANSWER: C

37 years old primapara at 41 week of pregnancy is admitted to the hospital with


complaints of regular uterine contractions every 2- 3minutes by 45 seconds.
Longitudinal fetal lie, cephalic presentation were diagnosed. Vasten sign is positive.
Fetal heart rate is 142 in 1min, clear, rhythmic. The sizes of pelvis are normal.
Probable fetal weight is 4200 g. Uterine cervix is dilated to 8 cm, edematous.
Amniotic membranes are absent. Fetal head is above the pelvic inlet. Promontorium is
not reached. Positive Vasten sign suggest about:
A. Adequate cervical dilation
B. Cephalo-pelvic disproportion
C. Uterine inertia
D. Uterine rupture
E. Hypertonic uterine dysfunction
ANSWER: B

39 years old primapara at 39 week of gestation is admitted with complaints of regular


uterine contractions every 2- 3minutes by 45 seconds. Lie of the fetus is
longitudinal, cephalic presentation. Vasten sign is positive. Fetal heart rate is 136
beats per minute, rhythmic. Pelvic sizes are normal. Probable fetal weight is 4100g.
Uterine cervix is dilated to 9 cm, edematous. Amniotic membranes are absent. Fetal
head is above the pelvic inlet. Promontorium is not reached. Which complication has
been occurred in labor?
A. Clinical contracted pelvis
B. Uterine rupture
C. Fetal distress
D. Uterine inertia
E. General contracted pelvis
ANSWER: A

A 29 years old woman at 38 weeks of gestation complaints of regular contractions.


Pelvic sizes: 25-28-30-21cm. The patient is found to have 100% effaced cervix and 3
cm cervical dilation. Diagonal conjugate has 13cm. Which type of the pelvis is
presented in the patient?
A. Normal pelvis
B. General contracted pelvis
C. Simple flat pelvis
D. Flat rachitic pelvis
E. Transverse contracted pelvis
ANSWER: A

A 21 years old woman at 40 weeks of gestation woman presents with complaints of pain
in the lower abdomen. Pelvic sizes: 26 – 26 – 30 - 21 cm. Uterine contractions are
regular and last every 5 minutes by 20seconds. The patient is found to have 100 %
effaced cervix for 1 cm dilated. Diagonal conjugate has 11cm. Which type of the pelvis
is presented in this patient?
A. Normal pelvis
B. General contracted pelvis
C. Simple flat pelvis
D. Flat rachitic pelvis
E. Transverse contracted pelvis
ANSWER: D

29 years old woman at 32 week of gestation woman presents with complaints of regular
uterine contractions. Pelvic sizes: 23 – 25 – 29 - 18 cm. Uterine contractions are
regular and last every 5 minutes by 20seconds. The patient is found to have 100 %
effaced cervix for 2 cm dilated. Diagonal conjugate has 11cm. Which type of the pelvis
is presented in this patient?
A. Normal pelvis
B. General contracted pelvis
C. Simple flat pelvis
D. Flat rachitic pelvis
E. Transverse contracted pelvis
ANSWER: B

29 years old woman at 35 week of gestation woman presents with complaints of regular
uterine contractions. Pelvic sizes: 23 – 25 – 29 - 18 cm. Solovjov index is 14 cm.
Uterine contractions are regular and last every 5 minutes by 20seconds. The patient is
found to have 100 % effaced cervix for 2 cm dilated. How many centimeters do obstetric
conjugate have?
A. 15cm
B. 14cm
C. 13cm
D. 11cm
E. 9cm
ANSWER: E

29 years old woman at 35 week of gestation woman presents with complaints of regular
uterine contractions. Pelvic sizes: 23 – 25 – 29 - 18 cm. Solovjov index is 14 cm.
Uterine contractions are regular and last every 5 minutes by 20seconds. The patient is
found to have 100 % effaced cervix for 2 cm dilated. Which degree of pelvic
contraction does the patient have?
A. I
B. II
C. III
D. IV
E. V
ANSWER: A

A 21 years old woman at 40 weeks of gestation woman presents with complaints of pain
in the lower abdomen. Pelvic sizes: 26 – 26 – 30 - 21 cm. Uterine contractions are
regular and last every 5 minutes by 20seconds. Solovjov index is 15cm. The patient is
found to have 100 % effaced cervix for 1 cm dilated. Diagonal conjugate has 13cm. How
much centimeters does obstetric conjugate have?
A. 15cm
B. 14cm
C. 13cm
D. 11cm
E. 9cm
ANSWER: D

A 24 years old primipara was hospitalized with complaints of irregular painful


uterine contractions. The uterus is tonic on palpation. The position of the fetus is
longitudinal, fetal head is fixated to pelvic inlet. Fetal heart rate is rhythmic, 140
bpm. Vertical size of Michaelis rhomb has 11cm, and transverse has 10cm. In obstetric
exam cervix of the uterus is 2,5 cm long, dense, the external cervical os is closed.
How much centimeters does obstetric conjugate have?
A. 11
B. 10
C. 8
D. 7
E. 6
ANSWER: A

A 24 years old primipara was hospitalized with complaints of irregular painful uterine
contractions. The uterus is tonic on palpation. The position of the fetus is
longitudinal, fetal head is fixated to pelvic inlet. Fetal heart rate is rhythmic, 140
bpm. Vertical size of Michaelis rhomb has 10cm, and transverse has 9cm. In obstetric
exam cervix of the uterus is 2,5 cm long, dense, the external cervical os is closed.
How much centimeters does obstetric conjugate have?
A. 11
B. 10
C. 8
D. 7
E. 6
ANSWER: B

N., 21 years old, primapara, 38 weeks of pregnancy. The labor started 5 hours ago. The
membranes ruptured 2 hours ago. Pelvic sizes: 22,24,29,19 cm. Solovjov index is 14 cm.
Fetal head rate 140 per minute with satisfactory characteristics. Per vaginum: the
cervix is 6 cm dilated. The amniotic sac is absent. Diagonal conjugate is 10 cm. Fetal
buttocks are palpated in the pelvic inlet. Which type of the pelvis does the woman
have?
A. Simple flat pelvis
B. Flat rachitic pelvis
C. General contracted pelvis
D. Osteomalatic pelvis
E. Transverse contracted pelvis
ANSWER: C

K., 23 years old, primapara, 39 weeks of pregnancy. The labor started 5 hours ago. The
membranes ruptured 2 hours ago. Pelvic sizes: 22,24,29,19 cm. Solovjov index is 14 cm.
Fetal head rate 140 per minute with satisfactory characteristics. Cervix is 6 cm
dilated in vaginal examination. The amniotic sac is absent. Diagonal conjugate is 10
cm. Fetal buttocks are palpated in the pelvic inlet. Which degree of pelvic
contraction does the woman have?
A. I
B. II
C. III
D. IV
E. Normal pelvic sizes
ANSWER: A

N., 21 years old, primapara, 39 weeks of pregnancy. The labor has been started 6 hours
ago. The membranes ruptured 2 hours ago. Pelvic sizes: 25,28,31,20 cm. Solovjov index
is 14 cm. Fetal heart rate is 140 per minute with satisfactory characteristics. Cervix
is 6 cm dilated in vaginal examination. The amniotic sac is presented. Diagonal
conjugate is 13 cm. Fetal head is presented at – 1 station. Which type of the pelvis
does the woman have?
A. Simple flat pelvis
B. Flat rachitic pelvis
C. General contracted pelvis
D. Osteomalatic pelvis
E. Normal pelvis
ANSWER: E

39-years old primapara at 38 week of gestation complaints of regular uterine


contractions every 3 minutes by 25 seconds. Fetal head is presented at – 2 station.
Fetal head rate 140 per minute with satisfactory characteristics. Uterine cervix is 6
cm dilated in vaginal examination. The amniotic sac is presented. Diagonal conjugate
is 13 cm. At the sacrococcygeal region 2cm exostosis is presented. What is management
of labor?
A. Spontaneous vaginal delivery
B. Cesarean section
C. Obstetric forceps
D. Vacuum extraction
E. Fetal destroying operation
ANSWER: B

A woman with obliquely dislocated pelvis in her 40th week of pregnancy, first labor
has regular birth activity. Uterine contractions are every 3 minutes by 25 seconds.
Fetal head is presented at – 2 station. Fetal heart rate is 140 per minute with
satisfactory characteristics. Cervix is 6 cm dilated in vaginal examination. The
amniotic sac is presented. Diagonal conjugate has 13 cm. What is management of labor?
A. Spontaneous vaginal delivery
B. Cesarean section
C. Obstetric forceps
D. Vacuum extraction
E. Fetal destroying operation
ANSWER: B

A woman with obliquely contracted pelvis has regular birth activity. Uterine
contractions take place every 3 minutes by 30 seconds. Fetal head is presented at – 2
station. Fetal heart rate is 140 per minute with satisfactory characteristics.
Cervix is 7 cm dilated. The amniotic sac is present. Diagonal conjugate is 11 cm. What
is management of labor?
A. Spontaneous vaginal delivery
B. Cesarean section
C. Obstetric forceps
D. Vacuum extraction
E. Fetal destroying operation
ANSWER: B

25 years old woman at 38 week of gestation woman presents with complaints of regular
uterine contractions. Pelvic sizes: 23 – 25 – 29 - 17 cm. Solovjov index is 14 cm.
Uterine contractions are regular and last every 5 minutes by 20seconds. The patient is
found to have 100 % effaced cervix for 4 cm dilated. Which degree of pelvic
contraction does the patient have?
A. I
B. II
C. III
D. IV
E. V
ANSWER: B

21 years old woman at 40 week of gestation woman presents with complaints of regular
uterine contractions. Pelvic sizes: 22 – 24 – 27 - 16 cm. Solovjov index is 14 cm.
Uterine contractions are regular and last every 5 minutes by 20seconds. The patient is
found to have 100 % effaced cervix for 5 cm dilated. Which degree of pelvic
contraction does the patient have?
A. I
B. II
C. III
D. IV
E. V
ANSWER: C

21 years old woman at 40 week of gestation woman presents with complaints of regular
uterine contractions. Pelvic sizes: 22 – 24 – 27 - 16 cm. Solovjov index is 14 cm.
Uterine contractions are regular and last every 5 minutes by 20seconds. The patient is
found to have 100 % effaced cervix for 5 cm dilated. What is best management of labor?
A. Vacuum extraction
B. Forceps application
C. Cesarean section
D. Vaginal delivery
E. Fetal destroying operation
ANSWER: C

M., 28 years old, para 2. Full term of pregnancy. Initiation of labor was 8 hours ago.
The membranes ruptured 20 minutes ago. Pelvic sizes: 25, 28, 31, 20 cm. Vasten sign is
positive. Fetal heart rate is 132 per minute with satisfactory characteristics.
Uterine cervix is is 9cm dilated in vaginal examination. The amniotic sac is absent.
Fetal head is in in the pelvic inlet. The chin is palpated near sacral region. What
is the best management of labor?
A. Spontaneous vaginal delivery
B. Cesarean section
C. Obstetric forceps
D. Vacuum extraction
E. Fetal destroying operation
ANSWER: B

F., 29 years old, para 2. Full term of pregnancy. Labor has been started 8 hours ago.
The membranes ruptured 20 minutes ago. Pelvic sizes: 25, 28, 31, 20 cm. Fetal heart
rate is 132 per minute with satisfactory characteristics. Vasten sign is positive.
Uterine cervix is 9cm dilated. The amniotic sac is absent. Fetal head is in in the
pelvic inlet. The chin is palpated near sacral region. Which complication is
presented in the patient?
A. General contracted pelvis
B. Clinical contracted pelvis
C. Fetal distress
D. Primary uterine inertia
E. Secondary uterine inertia
ANSWER: B

F., 29 years old, para 2. Full term of pregnancy. Labor has been started 8 hours ago.
The membranes ruptured 20 minutes ago. Pelvic sizes: 25, 28, 31, 20 cm. Fetal heart
rate is 132 per minute with satisfactory characteristics. Vasten sign is positive.
Uterine cervix is 9cm dilated. The amniotic sac is absent. Fetal head is in in the
pelvic inlet. The chin is palpated near sacral region. What is the reason of
cephalo-pelvic disproportion?
A. General contracted pelvis
B. Uterine inertia
C. Cephalic presentation
D. Breech presentation
E. Face presentation
ANSWER: E

M., 22 years old, nullipara. Full term of pregnancy. Labor has been started 8 hours
ago. The membranes ruptured are intact. Pelvic sizes: 25,28,31,20 cm. Fetal heart rate
is 140 per minute with satisfactory characteristics. Per vaginum: the cervix is 8 cm
dilated. The amniotic sac is persent. Fetal head is in the plane of inlet. Face line
is in the right oblique size, the chin is palpated near sacral region of the symphysis.
Which complication is presented in the patient?
A. General contracted pelvis
B. Cephalopelvic disproportion
C. Fetal distress
D. Primary uterine inertia
E. Secondary uterine inertia
ANSWER: B

M., 25 years old, nullipara. Full term of pregnancy. Labor has been started 9 hours
ago. The membrane is intact. Pelvic sizes: 25,28,31,20 cm. Vasten sign is positive.
Fetal heart rate is 140 per minute with satisfactory characteristics. Cervix is 9 cm
dilated in vaginal examination. The amniotic sac is presented. Fetal head is in the
plane of inlet. Face line is in the right oblique size, the chin is palpated near
sacral region of the symphysis. What is the reason of cephalopelvic disproportion?
A. Face anterior presentation
B. General contracted pelvis
C. Face posterior presentation
D. Sinciput vertex presentation
E. Brow presentation
ANSWER: A

Primipara N., 25 years old. Delivery at term. The labor started 6 hours ago. The
membranes ruptured 1 hour ago. Pelvic sizes: 23,26,29,19 cm. Solovjov index is 15cm.
Fetal heart rate is 140 per minute with satisfactory characteristics. Uterine
contractions occur every 4-5 minutes. The probable fetal weight by Volskov is 4200 g.
Uterine cervix dilatation is 7 cm. The amniotic sac is absent. Fetal head is -1
station. Sagittal suture is in the right oblique size. Small and large fontanels are
palpated. The large fontanel is under the symphysis. Which degree of pelvic
contraction is presented in woman?
A. I
B. II
C. III
D. IV
E. V
ANSWER: A

Primipara N., 25 years old. Delivery at term. The labor started 6 hours ago. The
membranes ruptured 1 hour ago. Pelvic sizes: 23,26,29,19 cm. Solovjov index is 15cm.
Fetal heart rate 140 per minute with satisfactory characteristics. Uterine
contractions occur every 4-5 minutes. The probable fetal weight by Volskiy is 4200 g.
Uterine cervix dilatation is 8 cm. The amniotic sac is absent. Fetal head is -1
station. Sagittal suture is in the right oblique size. Small and large fontanels are
palpated. The large fontanel is under the symphysis. Which complication is presented
in labor?
A. Cephalopelvic disproportion
B. Uterine inertia primary
C. Uterine inertia secondary
D. Uterine rupture
E. Discoordinative uterine activity
ANSWER: A
The woman is admitted to the maternity home with discontinued uterine contractions and
slight bloody discharges from the vagina. The condition is severe, skin is pale, and
consciousness is confused. BP – 80/40 mm Hg. Fetal heart rate of the fetus is absent.
Lover uterine segment is painful. There was a cesarean section one year ago. What is
the clinical diagnosis:
A. Placental abruption
B. Placental presentation
C. Uterine rupture
D. Premature expulsion of the amniotic fluids
E. Couveler uterus
ANSWER: C

25-years old pregnant woman at 32 weeks of gestation is admitted to the hospital with
complaints of bloody discharge like spotting. Placenta previa is diagnosed during
ultrasonography. The uterine tone is normal, fetal heart rate is 136 beats per minute.
What is the best management of the pregnant woman?
A. Induction of labor by prostaglandins
B. Blood transfusion
C. Induction of labor by oxytocin
D. Cesarean section.
E. Tranexamic acid prescription, female monitoring for the intensity of hemorrhage and
fetal wellbeing, dexamethasone administration
ANSWER: E

Pregnant N., 25 years is delivered in the maternity department with complaints of


periodic pain in lower part of abdomen and lumbar region. Bloody discharge has
appeared one hour before. Fetal heart rate is 136 in 1min. At vaginal examination: the
uterine cervix is effaced, 6-7 cm dilated. Spongy tissue is palpated laterally.
Blood loss is 250 ml and continues. Your management.
A. Stimulation of labor
B. Amniotomy
C. Obstetric forceps
D. The fetal destroying operation
E. Cesarean section
ANSWER: E

Sharp pain in the uterine fundus with profuse bleeding has appeared in the in
primapara during the first stage of labor. Uterus is in hypertonus. Fetal heart rate
is 200 beats per minute. Uterine cervix is effaced, and 4 cm dilated on vaginal
examination. Blood was presented in amniotic fluid during amniotomy. Your management.
A. Labor induction
B. Treatment of fetal distress
C. Cesarean section immediately
D. Tocolytics prescription
E. Coagulants prescription
ANSWER: C
Sharp pain in the uterine fundus with profuse bleeding has appeared in the in
primapara during the first stage of labor. Uterus is in hypertonus. Fetal heart rate
is 200 beats per minute. Uterine cervix is effaced, and 4 cm dilated on vaginal
examination. Blood was presented in amniotic fluid during amniotomy. Your diagnosis.
A. Low lying placenta
B. Placenta previa
C. Placenta abruption
D. Hypertonic dysfunction
E. DIC-syndrome
ANSWER: C

Uterine contractions have stopped suddenly in 25 years old patient after excessive
uterine contractions. Vasten sign is positive. Bloody excretions are presented. Fetal
heart rate is absent. The condition of patient suddenly became worse, BP went down to
70 mm Hg, pulse 140 in a 1 minute, a skin is pale. What is the reason such condition?
A. Threatened rupture of uterus
B. Placenta abruption
C. Uterine rupture
D. Couveler uterus
E. Placenta previa
ANSWER: C

Postpartum patient 28 years. 3800 grams girl was born. Placenta was delivered as a
result of active management of the placenta. Bleeding continues. Cervical and vaginal
lacerations are absent. Ruptured vessels are presented during inspection of the
placenta. What is the management of this situation?
A. To perform manual exploration of uterine cavity
B. Total hysterectomy
C. Contractile drugs prescription
D. External massage of uterus
E. Antishock garment application
ANSWER: A

25 years old woman is admitted to the hospital on a 38 week of pregnancy with regular
uterine contractions and bloody discharge from vagina. The uterine cervix is effaced
and 4 cm dilated, soft spongy tissue is presented. Bleeding increased in vaginal
examination. What is the reason of bleeding increasing?
A. Amniotic fluid embolism
B. Bleeding from the rupture of uterine cervix
C. Premature removing of the normally located placenta
D. Complete placenta previa
E. DIC - syndrome
ANSWER: D

Patient at 39 week of gestation is admitted to the hospital with complaints of sudden


acute pain in upper part of abdomen, dizziness, bleeding from vagina. The uterine
contractions are regular. Skin and visible mucus membranes are pale, BP 80/50 mm Hg,
pulse 126 per minute. Painful infiltrate is palpated in the fundal area. Fetal heart
rate is 170, arrhythmic. At vaginal examination the uterine cervix is effaced and 5 cm
dilated, amniotic membranes are presented, parts of placenta are not determined. Fetal
head is at -2 station. Bloody discharges are visible. What is the most likely
diagnosis?
A. Uterine rupture
B. Placenta abruption
C. Placenta previa
D. Hypotonic uterine contraction
E. The uterine tetanus
ANSWER: B

Patient at 39 week of gestation is admitted to the hospital with complaints of acute


pain in lower abdomen, dizziness, and bloody discharge from vagina. The uterine
contractions are regular every5 minutes by 40 seconds, amniotic membranes are intact.
Objectively: skin and visible mucus membranes are pale, BP 80/50 mm Hg, pulse is 126
beats per minute. Uterus is dense. Painful infiltrate is determined on the fundus.
Fetal heart rate is 170, arrhythmic. At vaginal examination: cervix is effaced and
dilated to 4cm, amniotic membranes are presented, parts of placenta are not determined,
fetal head of is at -2 station. Bleeding discharge without clots is presented. What
is the management of this situation?
A. Therapeutic rest
B. Cesarean section
C. Oxytocin prescription
D. Fetal destroying operation
E. Uterine curettage
ANSWER: B

Multipara. Uterine contractions occur every 4-5 hours by 25 seconds. Bloody excretions
began at once after appearance of contractions. Fetal heart rate is 100-110 beats per
minute . Uterine cervix is effaced and dilated till 6 cm, placenta tissue is
presented totally. Diagnosis?
A. Threatening rupture of uterus.
B. Couveler uterus
C. Partial placenta previa
D. Complete placenta previa.
E. Placenta abruption
ANSWER: D

Multipara. Uterine contractions occur every 4-5 hours. Bloody excretions began at once
after appearance of contractions. Fetal heart rate 100-110 in min. At vaginal
examination the uterine cervix is effaced and dilated till 6 cm. In the cervix
placental tissue is determined. What will be adequate management?
A. Therapeutic rest
B. Cesarean section
C. Oxytocin prescription
D. Fetal destroying operation
E. Uterine curettage
ANSWER: B
Labor started at 39 weeks of pregnancy. There was one induced abortion, which was
complicated by endometritis in the past. At active management of labor placenta
doesn’t separated. Considerable bloody excretions appeared at an attempt to do the
manual separation of placenta. The blood loss is 600 ml. An attempt of manual
separation of placenta from the uterus was not successful. What is the doctor’
management?
A. Hysterectomy
B. Uterine curettage
C. To continue the manual separation of placenta
D. Vasoconstrictors prescription
E. Contractile drugs prescription
ANSWER: A

Massive hypotonic uterine bleeding began after labor by dead child in postpartum
period. The blood loss was reached to 1600ml. Venous bleeding was presented.
Thrombocytes level was 80 x10 9/l. There were hemorrhages from the places of
injections. Which stage of DIC-syndrome was developed at postpartum patient?
A. I
B. III
C. II
D. V
E. IV
ANSWER: B

The girl by 3100g weight was delivered in 35 years old patient. During active
management of labor placenta doesn’t separated from uterine cavity. What is the most
probable diagnosis?
A. Hypertonus of uterus
B. Placenta previa
C. Placenta accreta
D. Uterine rupture
E. Uterine atony
ANSWER: C

The boy by 3700g was delivered in 36 years old woman. During active management of
labor placenta doesn’t separated from uterine cavity. What is the most probable
diagnosis?
A. Hypertonus of uterus
B. Placenta previa
C. Abnormal placenta adherence
D. Uterine rupture
E. Uterine atony
ANSWER: C

The boy by 3300g was delivered in 36 years old woman. During active management of
labor placenta doesn’t separated from uterine cavity. What is the most probable
diagnosis?
A. Hypertonus of uterus
B. Placenta previa
C. Placenta increta
D. Uterine rupture
E. Uterine atony
ANSWER: C

The boy by 3300g was delivered in 36 years old woman. During active management of
labor placenta doesn’t separated from uterine cavity. What is the most probable
diagnosis?
A. Hypertonus of uterus
B. Placenta previa
C. Placenta percreta
D. Uterine rupture
E. Uterine atony
ANSWER: C

Widespread extravasation of blood into the uterine wall was revealed during cesarean
section which was performed as a result of placenta abruption. The uterus is soft,
hypotonic bleeding was presented. The uterus had a purplish appearance, owing to such
extravasation of blood. Diagnosis?
A. Couveler’s uterus
B. Placenta previa
C. Uterine atony
D. Placenta abruption
E. Inevitable abortion
ANSWER: A

Widespread extravasation of blood into the uterine wall was revealed during cesarean
section which was performed as a result of placenta abruption. The uterus is soft,
hypotonic bleeding was presented. The uterus had a purplish appearance, owing to such
extravasation of blood. Your initial management is:
A. Total Hysterectomy
B. Blood transfusion
C. Subtotal hysterectomy
D. Uterine curettage
E. Pabal prescription, ligation of the a. iliac internal
ANSWER: E

Widespread extravasation of blood into the uterine wall was revealed during cesarean
section. The uterus is soft, hypotonic bleeding was presented. The uterus had a
purplish appearance, owing to such extravasation of blood. What was the indication to
cesarean section in the patient?
A. Placenta abruption
B. Ectopic pregnancy
C. Low lying placenta
D. Molar pregnancy
E. Breech presentation
ANSWER: A
Widespread extravasation of blood into the uterine wall was revealed during cesarean
section which was performed as a result of placenta abruption. The uterus is soft,
hypotonic bleeding was presented. The uterus had a purplish appearance, owing to such
extravasation of blood. Which dose of pabal (Carbetocin) should be prescribed
initially for the patient?
A. 10 mkg
B. 25 mkg
C. 50 mkg
D. 100mkg
E. 1000 mkg
ANSWER: D

42 years old patient was admitted to the maternity hospital in 38 weeks of gestation
with bloody discharge like spotting. Complete placenta previa was revealed at
ultrasound. Uterine cervix is closed in vaginal examination. What is the adequate
management of labor?
A. Augmentation of labor
B. Cesarean section
C. Fetal destroying operation
D. Obstetric forceps application
E. Vacuum extraction
ANSWER: B

30-years old edematous patient is admitted to the hospital in the second stage of
labor. Fetal head is in the pelvic outlet. Profuse hemorrhage have appeared suddenly.
Fetal heart rate is 80 beats per minute. What is the adequate management of labor?
A. Cesarean section
B. Fetal heart rate monitoring
C. Obstetric forceps
D. Episiotomy
E. Induction of labor
ANSWER: C

30-years old edematous patient is admitted to the hospital in the second stage of
labor. Fetal head is in the pelvic outlet. Profuse hemorrhage have appeared suddenly.
Fetal heart rate is 80 beats per minute. What is the most probable diagnosis?
A. Placenta abruption
B. Placenta previa
C. Uterine atony
D. Couveler uterus
E. Placenta increta
ANSWER: A

The bleeding began right after childbirth. The blood loss is 300 ml. In active
management of third stage of labor placenta is nor separated. Signs of the placental
separation are negative. What is the most probable diagnosis?
A. Subtotal placenta adherens
B. Total placenta adherens
C. Couveler’s uterus
D. Placenta previa
E. Placenta abruption
ANSWER: A

The bleeding began right after childbirth. The blood loss is 550 ml. What is the most
appropriate management?
A. Manual separation of placenta and exploration of the uterine cavity
B. Prescription of contractile drugs
C. Total hysterectomy
D. Uterine curretage
E. Uterine artery ligation
ANSWER: A

On the 6 day of the postpartum period a 26 years-old woman complaints of profuse


bleeding from vagina. Pelvic examination reveals 23-24 weeks increased uterus with
clots inside. During examination bloody discharge increases. Diagnosis?
A. Total placenta adherens
B. Subtotal placenta adherens
C. Couveler’s uterus
D. Late postpartum hemorrhage
E. Placenta previa
ANSWER: D

On the 6 day of the postpartum period a 26 years-old woman complaints of profuse


bleeding from vagina. Pelvic examination reveals 22-23 weeks increased uterus with
clots inside. During examination bloody discharge increases. What is the most
appropriate management of this situation?
A. Uterine curettage
B. Uterine artery ligation
C. Total hysterectomy
D. Manual exploration of the uterine cavity
E. Prescription of contractile drugs
ANSWER: A

4500g infant was delivered in 42 years old patient. Placenta was delivered by active
management. All membranes and parts of the placenta were presented during inspection.
Blood loss was 350 ml. After 5 min bloody excretions increased. At external uterine
massage uterus did not contract well, was soft and large. Diagnosis?
A. Placental abruption
B. Atonic bleeding
C. Uterine rupture
D. Couveler’s uterus
E. Rupture of the cervix
ANSWER: B

First pregnancy 35 week of gestation. Woman complaints of spotting from the vagina.
Blood loss is 50 ml. Uterine tone is normal. Fetal heart rate is clear, rhythmic, 136
beats per minute. Head of the fetus is mobile above the pelvic inlet. Uterine cervix
is closed. What is the initial management of the patient?
A. Bed rest
B. Cesarean section immediately
C. Amniotomy
D. Bed rest, hemostatic agents
E. Bed rest, adequate tokolysis
ANSWER: D

First pregnancy 32week of gestation. Woman complaints of spotting from the vagina.
Blood loss is 50 ml. Uterine tone is normal. Fetal heart rate is clear, rhythmic, 136
beats per minute. Head of the fetus is mobile above the pelvic inlet. Uterine cervix
is closed. What is the initial management of the patient?
A. Bed rest
B. Cesarean section immediately
C. Amniotomy
D. Bed rest, hemostatic, corticosteroids prescriptions
E. Bed rest, adequate tokolysis
ANSWER: D

First pregnancy 33 week of gestation. Woman complaints of spotting from the vagina.
Blood loss is 50 ml. Uterine tone is normal. Fetal heart rate is clear, rhythmic, 136
beats per minute. Head of the fetus is mobile above the pelvic inlet. Uterine cervix
is closed. What is the aim of corticosteroids prescription?
A. Anti-inflammatory
B. Hemostatic
C. Sedative
D. Prevention of respiratory distress syndrome
E. Adequate tokolysis
ANSWER: D

Massive postpartum bleeding has developed in 34 years old patient with twins. Placenta
is intact. Vagina and perineum are without lacerations. Uterine fundus is 5cm above
the umbilicus, soft. What is the most likely reason of bleeding?
A. Couveler uterus
B. Uterine rupture
C. Uterine atony
D. Placenta abruption
E. DIC - syndrome
ANSWER: C

Patient is admitted to the hospital with regular uterine contractions. During


hospitalization the condition is satisfactory, pulse - 84 in 1 min, BP - 150/90 and
160/90 mm of Hg. Suddenly patient complains of severe pain in abdomen, general
weakness, and dizziness. Pulse - 120 in min, AT - 80/40 and 90/45 mm Hg. Uterus is
tense, very painful. Fetal heart rate is not auscultated. Uterine cervix is effaced, 5
cm dilated at vaginal examination. Amniotic membrane is whole, tense. The head is in
the pelvic inlet. Insignificant bloody discharge has appeared from vagina. What is the
most likely diagnosis?
A. Uterine rupture
B. Placental abruption
C. Amniotic fluid embolism
D. Molar pregnancy
E. Placenta previa
ANSWER: B

Considerable bloody discharge has appeared at 30week of gestation in 35 years old


patient. Total placenta previa is diagnosed during ultrasonography. General blood
loss is 500 ml and bleeding continues. What is the management?
A. Tocolitics prescription
B. Cervical cerclage
C. Cesarean section immediately
D. Hemostatics
E. Vaginal delivery
ANSWER: C

The bleeding began right after childbirth. The blood loss was 300 ml. In attempt to
remove placenta during traction it did not separate. Doctors’ tactics.
A. Cesarean section
B. Manual removal of placenta
C. Cerclage
D. Total hysterectomy
E. Ligation of uterine vessels
ANSWER: B

28 years old pregnant women complains of vaginal bleeding at 40 weeks. Fetal heart
rate is 140 BPM. The results of the obstetrics examination: cervix is incompletely
effaced, 3 cm dilated, the placental tissue is palpated. Diagnosis?
A. Abnormal placenta adherence
B. Complete placenta previa
C. Uterine atony
D. Uterine rupture
E. Placental abruption
ANSWER: B

28 years old pregnant women complaints of vaginal bleeding at 40 week of gestation.


Fetal heart rate is 140 BPM. The results of the obstetrics examination: cervix is
incompletely effaced, 4 cm dilated, the placental tissue is palpated. Management?
A. Cesarean section
B. Manual removal of placenta
C. Cerclage
D. Total hysterectomy
E. Ligation of ovarian vessels
ANSWER: A

29 years old woman at 39 weeks of pregnancy complaints of small amount of bloody


discharge till 50 ml and regular uterine contractions which have been started 5 hours
ago. Fetal head rate is 132 per minute with satisfactory characteristics. Cervix is
dilated till 5 cm in obstetric examination The amniotic sac is present. Fetal head is
in plane of pelvic inlet. The edge of the placenta lies adjacent to the internal os.
Diagnosis?
A. Abnormal placenta adherence
B. Complete placenta previa
C. Uterine atony
D. Incomplete placenta previa
E. Placental abruption
ANSWER: D

30 years old primapara at 39 week of pregnancy complaints of small amount of bloody


discharge and regular uterine contractions which have been started 5 hours ago. Fetal
head rate 132 per minute with satisfactory characteristics. At obstetric exam cervix
is dilated till 2 cm. The amniotic sac is present. Fetal head is in plane of pelvic
inlet. The edge of the placenta lies adjacent to the internal os. Blood loss is 300 ml
and continues. What is the best management of labor?
A. Cesarean section
B. Manual removal of placenta
C. Amniotomy, oxytocin prescription
D. Total hysterectomy
E. Ligation of ovarian vessels
ANSWER: A

21 years old, primapara at 38 week of pregnancy complaints of profuse bleeding till


400ml. Contractions are regular. At obstetric exam cervix is dilated till 4 cm. The
amniotic sac is present. Fetal head is in plane of pelvic inlet. The leading edge of
the placenta is 2 cm from the internal cervical os. What is the best management of
labor?
A. Cesarean section
B. Manual removal of placenta
C. Amniotomy, oxytocin prescription
D. Total hysterectomy
E. Ligation of ovarian vessels
ANSWER: A

34 years old, primapara at 37 week of pregnancy complaints of small amount of bloody


discharge. Contractions are irregular. Fetal heart rate is 132 per minute with
satisfactory characteristics. At obstetric exam cervix is closed. The leading edge of
the placenta is 1 cm from internal cervical os. What is the management of the
pregnancy?
A. Cesarean section
B. Manual removal of placenta
C. Amniotomy, oxytocin prescription
D. Total hysterectomy
E. Ligation of ovarian vessels
ANSWER: A

What is the first step in postpartum hemorrhage treatment?


A. Call for help
B. Insert intravenous access
C. Recognize circulatory problem. Administer for face oxygen mask
D. Abdominal examination to confirm uterus well contracted
E. Vaginal examination for laceration
ANSWER: A

30 years old undergoes spontaneous delivery of a 3900 g boy. 10 units of oxytocin were
administered. After successful umbilical cord traction placenta was delivered.
Placental tissue is expelled with umbilical cord, but vaginal hemorrhage ensues
immediately thereafter. The placenta is clearly not intact. What are appropriate
immediate interventions in this situation?
A. Total hysterectomy
B. Subtotal hysterectomy
C. Uterine curettage
D. Manual exploration of uterine cavity
E. External uterine massage
ANSWER: D

30 years old undergoes spontaneous delivery of a 3900 g boy. 10 units of oxytocin were
administered. After successful umbilical cord traction placenta was delivered.
Placental tissue is expelled with umbilical cord, but vaginal hemorrhage ensues
immediately thereafter. The placenta is clearly not intact. What is the most probable
diagnosis?
A. Placenta accreta
B. Placenta increta
C. Placenta percreta
D. Retained placenta tissue
E. Placenta previa
ANSWER: D

A 30-year-old pregnant was admitted to a maternity hospital with interm pregnancy. She
complains of severe pain in the uterus that started 1 hour ago, nausea, vomiting, cold
sweat. Cesarean section was performed 2 years ago. Uterine contractions have stopped.
Skin and mucous membranes are pale. Heart rate is 100/min. BP is 90/60 mm Hg. Uterus
is sharply painful. Fetal heart rate is not auscultated. Moderate bloody discharge
from the uterus is observed. Uterus cervix is 4 cm open. Presenting part is not
palpated. The most probably diagnosis is:
A. Uterine rupture
B. Placenta previa
C. Placenta abruption
D. Premature separation of normally localized placenta
E. Compression of inferior pudendal vein
ANSWER: A

Examination of a just delivered placenta reveals 2x3cm defect. Vaginal bleeding is


presented. Your initial management:
A. Manual exploration of uterine cavity
B. Abduladze method
C. Crede's method
D. Uterine curettage
E. Oxytocin intravenously
ANSWER: A

Examination of a just delivered placenta reveals 2x3cm defect. Vaginal bleeding is


absent. Your initial management:
A. Manual exploration of uterine cavity
B. Abduladze method
C. Crede's method
D. Uterine curettage
E. Oxytocin intravenously
ANSWER: A

Examination of a just delivered placenta reveals 2x3cm defect. Vaginal bleeding is


presented. Manual exploration of the uterine cavity is indicated. Which method of
anesthesia you would choose?
A. You don’t need anesthesia
B. Intrauterine
C. Paravaginal
D. Paracervical
E. Intravenous
ANSWER: E

36 years old multipara at 41 week of pregnancy complaints of vaginal bleeding like


spotting. Contractions are regular. At obstetric exam cervix is dilated till 5 cm. The
amniotic sac is present. Fetal head is in the pelvic inlet. The leading edge of the
placenta is 4 cm from the internal cervical os at ultrasonography. What is the best
management of labor?
A. Cesarean section
B. Manual removal of placenta
C. Amniotomy, oxytocin prescription
D. Total hysterectomy
E. Ligation of ovarian vessels
ANSWER: C

32 years old multipara at 40 week of pregnancy complaints of vaginal bleeding like


spotting. Contractions are every 3-4 minutes by 30 seconds. Cervix is dilated to 6 cm
during vaginal examination. Amniotic membranes are intact. Fetal head is in the pelvic
inlet. The leading edge of the placenta is 3 cm from the internal cervical os at
ultrasonography. What is the best management of labor?
A. Cesarean section
B. Manual removal of placenta
C. Amniotomy, oxytocin prescription
D. Total hysterectomy
E. Ligation of ovarian vessels
ANSWER: C

29 years old primapara at 38 week of pregnancy is admitted to the hospital with


complaints of bloody discharge, general weakness and regular uterine contractions
which have been started 5 hours ago. In examination pulse is 100 beats per minute, BP
– 100/60 and 95/55 mm of Hg. Uterus is tense, very painful in the fundal area. Fetal
heart rate is not auscultated. Uterine cervix is effaced, 5 cm dilated at vaginal
examination. Amniotic membranes are intact. The head is in the pelvic inlet. Blood
loss reaches 1000ml. Which degree of hemorrhagic shock is presented in the patient?
A. I
B. II
C. III
D. IV
E. V
ANSWER: A

29 years old primapara at 38 week of pregnancy is admitted to the hospital with


complaints of bloody discharge, general weakness and regular uterine contractions
which have been started 5 hours ago. In examination pulse is 100 beats per minute, BP
– 100/60 and 95/55 mm of Hg. Uterus is tense, very painful in the fundal area. Fetal
heart rate is not auscultated. Uterine cervix is effaced, 5 cm dilated at vaginal
examination. Amniotic membranes are intact. The head is in the pelvic inlet. Blood
loss reaches 1000ml.What is the general amount of infusion therapy should be
prescribed?
A. 500ml
B. 1000ml
C. 1500ml
D. 2000ml
E. 2500ml
ANSWER: E

34 years old primapara at 40 week of pregnancy is admitted to the hospital with


profuse vaginal bleeding, loss of consciousness. In examination pulse is 120 beats
per minute, BP – 80/60 and 75/55 mm of Hg. Uterus is tense, very painful in the fundal
area. Fetal heart rate is not auscultated. Uterine cervix is effaced, 6 cm dilated at
vaginal examination. Amniotic membranes are intact. The head is in the pelvic inlet.
Blood loss reaches 1500ml. Which degree of hemorrhagic shock is presented in the
patient?
A. I
B. II
C. III
D. IV
E. V
ANSWER: B

34 years old primapara at 40 week of pregnancy is admitted to the hospital with


profuse vaginal bleeding, loss of consciousness. In examination pulse is 120 beats
per minute, BP – 80/60 and 75/55 mm of Hg. Uterus is tense, very painful in the fundal
area. Fetal heart rate is not auscultated. Uterine cervix is effaced, 6 cm dilated at
vaginal examination. Amniotic membranes are intact. The head is in the pelvic inlet.
Blood loss reaches 1500ml. What is the general amount of infusion therapy should be
prescribed?
A. 500ml
B. 1000ml
C. 1500ml
D. 2000ml
E. 3000ml
ANSWER: E

The client who has just been diagnosed with invasive infiltrating ductal carcinoma
asks what this means. What is the physician’s best response?
A. “The cancer has spread from the breast ducts into surrounding breast tissue.”
B. “The cancer has spread from the breast into local lymph nodes and channels.”
C. “The cancer has spread from the breast into surrounding tissues and organs.”
D. “The cancer has spread from the breast into distant tissues and organs.”
E. “The cancer has spread from the breast into the ducts from surrounding breast
tissue
ANSWER: A

Which comment made by the client with breast cancer indicates correct understanding
regarding cancer causes and prevention?
A. “I will cure my cancer by eating a low-fat diet from now on.”
B. “If I had breast-fed my children, this would not have happened to me.”
C. “I hope this doesn't increase my risk for bone cancer or lung cancer.”
D. “I will have regular mammograms on my other breast to detect cancer early.”
E. “Regular physical exercise is good method of prevention of cancer in future”
ANSWER: D

The client frequently finds lumps in her breasts, especially around her period. Which
info should the physician teach the client about breast self care?
A. This is a benign process that does not need follow up.
B. Eliminate chocolate and caffeine from diet.
C. Practice breast self exam monthly.
D. This is how breast cancer starts and she needs surgery.
E. Do additional physical exercise to decrease size of lump
ANSWER: C

The physician is teaching a class on breast health to a group of ladies at the senior
center. Which is the most important risk factor to emphasize to this group?
A. Find out about family history of Breast Cancer
B. Men at this age can get breast cancer and should also be screened
C. Monthly self-exam is the key to early detection.
D. The older a woman gets, the greater the chance of developing Breast Cancer.
E. Men and women have same chance to have Breast Cancer if they are married
ANSWER: D

While the physician is obtaining a nursing history from a 52-year-old patient who has
found a small lump in her breast, which question is most pertinent?
A. "Do you currently smoke cigarettes?"
B. "Have you ever had any breast injuries?"
C. "Is there any family history of fibrocystic breast changes?"
D. "At what age did you start having menstrual periods?"
E. “Do you know somebody who have breast cancer
ANSWER: A

A 62-year-old patient complains to the physician that mammograms are painful and a
source of radiation exposure. She says she does breast self-examination (BSE) monthly
and asks whether it is necessary to have an annual mammogram. The physician's best
response to the patient is:
A. "If your mammogram was painful, it is especially important that you have it done
annually."
B. "An ultrasound examination of the breasts, which is not painful or a source of
radiation, can be substituted for a mammogram."
C. "Because of your age, it is even more important for you to have annual mammograms."
D. "Unless you find a lump while examining your breasts, a mammogram every 2 years is
recommended after age 60."
E. "If you will have two normal result of mammogram you can stop to do this procedure"
ANSWER: C

A patient with a small breast lump is advised to have a fine needle aspiration biopsy.
The physician explains that an advantage to this procedure is that
A. only a small incision is necessary, resulting in minimal breast pain and scarring.
B. if the specimen is positive for malignancy, the patient can be told at the visit.
C. if the specimen is negative for malignancy, the patient's fears of cancer can be
put to rest.
D. fine needle aspiration is guided by a mammogram, ensuring that cells are taken from
the lesion.
E. after this procedure there are no any risk for septic complication
ANSWER: B

A 33-year-old patient tells the physician that she has fibrocystic breasts but
reducing her sodium and caffeine intake and other measures have not made a difference
in the fibrocystic condition. An appropriate patient outcome for the patient is
A. calls the mammologist if any lumps are painful or tender.
B. states the reason for immediate biopsy of new lumps.
C. monitors changes in size and tenderness of all lumps in relation to her menstrual
cycle.
D. has genetic testing for BRCA-1 and BRCA-2 to determine her risk for breast cancer.
E. increases your physical activity
ANSWER: C

A 20-year-old student comes to the student health center after discovering a small
painless lump in her right breast. She is worried that she might have cancer because
her mother had cervical cancer. The physician's response to the patient is based on
the knowledge that the most likely cause of the breast lump is
A. fibrocystic complex.
B. fibroadenoma.
C. breast abscess.
D. adenocarcinoma.
E. Lactostasis
ANSWER: B
A 51-year-old woman at menopause is considering the use of hormone replacement therapy
(HRT) but is concerned about the risk of breast cancer. When discussing this issue
with the patient, the physician explains that
A. HRT does not appear to increase the risk for breast cancer unless there are other
risk factors.
B. She and her health care provider must weigh the benefits of HRT against the
possible risks of breast cancer.
C. HRT is a safe therapy for menopausal symptoms if there is no family history of BRCA
genes.
D. Alternative therapies with herbs and natural drugs are as effective as estrogen in
relieving the symptoms of menopause.
E. Additional physical exercises are as effective as estrogen in relieving the
symptoms of menopause
ANSWER: B

At a routine health examination, a woman whose mother had breast cancer asks the
physician about the genetic basis of breast cancer and the genes involved. The
physician explains that
A. her risk of inheriting BRCA gene mutations is small unless her mother had both
ovarian and breast cancer.
B. changes in BRCA genes that normally suppress cancer growth can be passed to
offspring, increasing the risk for breast cancer.
C. because her mother had breast cancer, she has inherited a 50% to 85% chance of
developing breast cancer from mutated genes.
D. genetic mutations increase cancer risk only in combination with other risk factors
such as obesity.
E. Avoiding caffeine and fatty food allows to decrease risk of breast cancer related
to inheriting BRCA gene mutations
ANSWER: B

When assessing a patient for breast cancer risk, the physician considers that the
patient has a significant family history of breast cancer if she has a
A. cousin who was diagnosed with breast cancer at age 38.
B. mother who was diagnosed with breast cancer at age 42.
C. sister who died from ovarian cancer at age 56.
D. grandmother who died from breast cancer at age 72.
E. daughter who was diagnosed lung cancer at age 16
ANSWER: B

A 32-week-pregnant client is upset and thinks she has breast disease because she has a
yellowish discharge coming from her breasts. What can the physician say or do for this
client?
A. Refer the client for a mammogram
B. This is normal toward the end of pregnancy and is called colostrum.
C. Notify the mammologist
D. Ask history questions about previous breast infections.
E. Refer the client for a sonography
ANSWER: B
The physician notes the presence of gynecomastia in a 15-year-old client. The
physician knows:
A. This is an abnormal finding for a client at this age.
B. This condition is abnormal if it is of recent onset.
C. This is a temporary condition in puberty.
D. The client may be at increased risk for the development of breast cancer.
E. This is a normal finding for a client at this age.
ANSWER: C

The physician is planning a focused breast/axillae interview and wants to include a


general health question. Which of the following questions would fit these criteria?
A. "Has your mother or sister had breast cancer?"
B. "Are you still menstruating?"
C. "Have you ever had a mammogram?"
D. "Have you had any breast trauma?
E. "When was your last visit to gynecologist?"
ANSWER: B

A 14-year-old female client is upset because her breast development is not equal. What
can the physician say to this client?
A. "It is normal for breast tissue development to be unequal during puberty. It will
even out as you get older."
B. "Don't worry; most girls have breasts that are slightly unequal in size."
C. "Did your mother experience the same thing when she was your age?"
D. "Your breasts appear equal in size."
E. "You should do sonography to rule out breast cancer"
ANSWER: A

A 65-year-old female client wants to know if she still should continue to perform self
breast assessments. The most appropriate response by the physician is:
A. "Women can stop breast self exams after menopause."
B. "Self exam is not necessary if you get yearly mammograms."
C. "After age 60, it is not really necessary."
D. "Breast cancer can still develop when you get older."
E. "In your age self breast assessments should be performed 2 times per month"
ANSWER: D

When gathering breast history information on a client, the client states that she has
noticed a few drops of clear discharge from her nipples over the past few months.
Which of the following actions is most appropriate in this situation?
A. Ask additional history questions about the discharge and medications she is
currently taking.
B. Refer the client for a mammogram.
C. Notify the mammologist.
D. Document the presence of the discharge.
E. Refer the client for a sonography.
ANSWER: A
A 20-year-old client with benign breast disease says she has increased breast pain and
tenderness with menses. What should the physician do in this situation?
A. Discuss the relationship of benign breast disease to breast cancer.
B. Discuss how reducing caffeine and salt intake and wearing a support bra might help.
C. Explain that a breast biopsy may be indicated.
D. Tell the client that this condition will soon go away
E. Explain that the additional physical exercises can reduce this pain
ANSWER: B

The physician is performing a breast examination. Which of the following reflect


abnormal findings during the inspection phase of breast examination?
A. A woman whose left breast is slightly larger than her right
B. A woman whose nipples are pointing in different directions
C. A woman whose skin is marked with linear striae
D. A pregnant woman whose breasts have a fine blue network of veins visible under the
skin
E. A woman who wear bra size 3
ANSWER: B

A client asks the physician, "Why do I need to examine my underarms when I perform my
breast exam?" Which of the following is the most appropriate response for the
physician to make to this client?
A. This is the least likely area for breast cancer to occur.
B. Breast tissue extends into the axilla.
C. This is the hardest area to feel for changes.
D. It is easier to detect abnormalities in this area than in the breast tissue.
E. There are no any another need to examine this area
ANSWER: B

Physician is teaching a client who suspects that she has a lump in her breast. The
physician instructs the client that a diagnosis of breast cancer is confirmed by:
A. breast self-examination.
B. mammography.
C. fine needle aspiration.
D. chest X-ray.
E. sonography
ANSWER: C

Physician is teaching a group of women to perform breast self-examination. The


physician should explain that the purpose of performing the examination is to discover:
A. cancerous lumps.
B. areas of thickness or fullness.
C. changes from previous self-examinations.
D. fibrocystic masses.
E. mastitis
ANSWER: C
Physician is speaking to a group of women about early detection of breast cancer. The
average age of the women in the group is 47. Following the American Cancer Society
guidelines, the physician should recommend that the women:
A. perform breast self-examination annually.
B. have a mammogram annually.
C. have a hormonal receptor assay annually.
D. have a physician conduct a clinical examination every 2 years.
E. visit to gynecologist annually
ANSWER: B

Physician is providing breast cancer education at a community facility. The American


Cancer Society recommends that women get mammograms:
A. yearly after age 40.
B. after the birth of the first child and every 2 years thereafter.
C. after the first menstrual period and annually thereafter.
D. every 3 years between ages 20 and 40 and annually thereafter.
E. in menopause
ANSWER: A

A 34-year-old female client is requesting information about mammograms and breast


cancer. She isn’t considered at high risk for breast cancer. What should the physician
tell this client?
A. She should have had a baseline mammogram before age 30.
B. She should eat a low-fat diet to further decrease her risk of breast cancer.
C. She should perform breast self-examination during the first 5 days of each
menstrual cycle.
D. When she begins having yearly mammograms, breast self-examinations will no longer
be necessary.
E. She should do additional physical exercise
ANSWER: B

Nina, an oncology educator, is speaking to a women’s group about breast cancer.


Questions and comments from the audience reveal a misunderstanding of some aspects of
the disease. Various members of the audience have made all of the following statements.
Which one is accurate?
A. Mammography is the most reliable method for detecting breast cancer.
B. Breast cancer is the leading killer of women of childbearing age.
C. Breast cancer requires a mastectomy.
D. Men can develop breast cancer.
E. Breast self examination is the first step in an early detection of Brest cancer
ANSWER: D

Physician is instructing a premenopausal woman about breast self-examination. The


physician should tell the client to do her self-examination:
A. at the end of her menstrual cycle.
B. at the same day each month.
C. at the 1st day of the menstrual cycle.
D. immediately after her menstrual period.
E. Any time when remind
ANSWER: D

Gio, a community health physician, is instructing a group of female clients about


breast self-examination. The physician instructs the client to perform the examination:
A. At the onset of menstruation
B. Every month during ovulation
C. Weekly at the same time of day
D. 1 week after menstruation begins
E. Any time during the cycle
ANSWER: D

A 70 year old woman has a palpable lesion on clinical breast examination done during a
physical examination. The hard, painless left breast mass measures approximately 5 cm.
The patient has no palpable abnormalities of the contralateral breast or either axilla,
and has no additional symptoms (such as bone pain or weight loss). It has been just
over a year since the patient had a screening mammogram. Of the following options,
which is the best first step in further evaluation of this patient’s breast lesion?
A. no further work-up is required
B. CT of the breast
C. diagnostic mammography
D. MR imaging of both breasts
E. Sonography of both breasts
ANSWER: C

GK is a 46-year-old woman who presents to your office for a well-woman examination.


She informs you that her 51-year-old friend was diagnosed with breast cancer one month
ago and that she is worried about getting breast cancer. On further inquiry, you learn
that she delivered her only child when she was 32 years of age and has no family
history for breast cancer. She does not perform breast self-examinations and has never
had a mammogram. GK asks for your advice on breast cancer screening. Which one of the
following options is best for this patient?
A. Discuss the harms and benefits of screening, and offer screening because she is
older than age 40.
B. Do not offer screening because she is younger than age 50.
C. Offer screening to obtain a baseline examination, and begin routine screening at
age 50.
D. Discuss the harms and benefits of screening, and offer screening because she is
older than age 40; continue screening every one to two years until age 70.
E. Screening is not indicated because she is at low risk for developing breast cancer.
ANSWER: A

GK is a 46-year-old woman who presents to your office for a well-woman examination.


She informs you that her 51-year-old friend was diagnosed with breast cancer one month
ago and that she is worried about getting breast cancer. On further inquiry, you learn
that she delivered her only child when she was 32 years of age and has no family
history for breast cancer. She does not perform breast self-examinations and has never
had a mammogram. GK asks for your advice on breast cancer screening. Which one of the
following strategies is best to screen for breast cancer?
A. Monthly breast self-examinations combined with an annual clinical breast
examination.
B. Monthly breast self-examinations and mammography every one to two years.
C. Clinical breast examination every one to two years.
D. Mammography with or without clinical breast examination every one to two years.
E. Routine breast self-examinations and mammography with or without clinical breast
examinations every one to two years.
ANSWER: D

24 years old woman is presented in postpartum period after physiological delivery. At


objective examination her temperature is 36, 8 0C, Ps - 72/min, BP - 120/80 mm Hg.
Mammary glands are moderately swollen, nipples are clean without fissures. Lactation
is characterized by colostrum. Abdomen is soft and painless. Uterine fundus height is
3 fingers below the umbilicus. Lochia are bloody, moderate. The most appropriate day
for postpartum period would be:
A. 3 day
B. 2 day
C. 1 day
D. 5 day
E. 4 day
ANSWER: A

28 years old woman is presented in postpartum period after physiological delivery. At


objective examination her temperature is 36, 8 0C, Ps - 84/min, BP - 120/80 mm Hg.
Mammary glands are moderately swollen, nipples are clean without fissures. Lactation
is characterized by immature milk. Abdomen is soft and painless. Uterine fundus height
is in the midway between symphysis and umbilicus. Lochia are bloody-serous, moderate.
The most appropriate day for postpartum period would be:
A. 3 day
B. 2 day
C. 1 day
D. 5 day
E. 4 day
ANSWER: D

22 years old woman is presented in postpartum period after physiological delivery. At


objective examination her temperature is 36, 7 0C, Ps - 72/min, BP - 110/70 mm Hg.
Mammary glands are moderately swollen, nipples are clean without fissures. Lactation
is characterized by colostrum. Abdomen is soft and painless. Uterine fundus - 2
fingers below the umbilicus. Lochia are bloody, moderate. The most appropriate day for
postpartum period would be:
A. 3 day
B. 2 day
C. 1 day
D. 5 day
E. 4 day
ANSWER: B
25 years old woman is presented in postpartum period after physiological delivery. At
objective examination her temperature is 36, 7 0C, Ps - 72/min, BP - 110/70 mm Hg.
Mammary glands are soft, nipples are clean without fissures. Lactation is good.
Abdomen is soft and painless. Uterine fundus is located 8cm above symphysis. The
lochia should be:
A. Bloody
B. Bloody-serous
C. Mucous
D. Purulent
E. Serous
ANSWER: B

22 years old woman is presented at postpartum period after physiological delivery. At


objective examination her temperature is 36,7 0C, Ps - 72/min, BP - 110/70 mm Hg.
Mammary glands are soft, nipples are clean without fissures. Lactation is good.
Abdomen is soft and painless. Uterine fundus is located 6cm below umbilicus. The
lochia should be:
A. Bloody
B. Bloody-serous
C. Mucous
D. Purulent
E. Serous
ANSWER: A

19 years old woman is presented in postpartum period after physiological delivery At


objective examination her temperature is 36,8 0C, Ps - 72/min, BP - 120/80 mm Hg.
Mammary glands are normal, colostrum is presented. Abdomen is soft and painless.
Uterine fundus height is 2 cm below the umbilicus. Lochia are bloody, moderate. The
most appropriate day for postpartum period would be:
A. 3 day
B. 2 day
C. 1 day
D. 5 day
E. 4 day
ANSWER: B

33 years old woman is presented in postpartum period after physiological delivery. At


objective examination her temperature is 36, 8 0C, Ps - 84/min, BP - 120/80 mm Hg.
Mammary glands are moderately swollen, nipples are clean without fissures. Lactation
characterizes by immature milk. Uterine fundus height is 4 fingers below umbilicus.
Lochia are bloody-serous, moderate. The most appropriate day for postpartum period
would be:
A. 3 day
B. 2 day
C. 1 day
D. 5 day
E. 4 day
ANSWER: E
35 years old woman is presented in postpartum period after physiological delivery. At
objective examination her temperature is 36, 8 0C, Ps - 84/min, BP - 120/80 mm Hg.
Mammary glands are normal without infiltration. Lactation characterizes by immature
milk. Uterine fundus height is 4 fingers above symphysis. Lochia are bloody-serous,
moderate. The most appropriate day for postpartum period would be:
A. 3 day
B. 2 day
C. 6day
D. 5 day
E. 4 day
ANSWER: C

29 years old woman is presented in postpartum period after physiological delivery. At


objective examination her temperature is 36, 8 0C, Ps - 84/min, BP - 120/80 mm Hg.
Mammary glands are normal. Lactation is good. Uterine fundus height is 3 fingers
above symphysis. Lochia are serous-bloody, moderate. The most appropriate day for
postpartum period would be:
A. 7 day
B. 2 day
C. 1 day
D. 5 day
E. 4 day
ANSWER: A

28 years old woman is presented in postpartum period after physiological delivery. At


objective examination her temperature is 36, 8 0C, Ps - 84/min, BP - 120/80 mm Hg.
Mammary glands are normal without infiltrations. Lactation is good. Uterine fundus
height is 4cm above symphysis. Lochia are serous-bloody, moderate. The most
appropriate day for postpartum period would be:
A. 8 day
B. 2 day
C. 1 day
D. 5 day
E. 4 day
ANSWER: A

28 years old woman is presented in postpartum period after physiological delivery. At


objective examination her temperature is 36, 8 0C, Ps - 84/min, BP - 120/80 mm Hg.
Mammary glands are normal. Lactation is good. Uterine fundus height is 2cm above
symphysis. Lochia are serous-bloody, moderate. The most appropriate day for postpartum
period would be:
A. 7 day
B. 2 day
C. 9 day
D. 5 day
E. 4 day
ANSWER: C
24 years old woman is presented in postpartum period after physiological delivery. At
objective examination her temperature is 36, 8 0C, Ps - 84/min, BP - 120/80 mm Hg.
Mammary glands are normal. Lactation is good. Lochia are serous in small amount. The
most appropriate day for postpartum period would be:
A. 7 day
B. 2 day
C. 10day
D. 5 day
E. 4 day
ANSWER: C

25 years old woman is presented in postpartum period after physiological delivery at


objective examination her temperature is 36,7 0C, Ps - 72/min, BP - 110/70 mm Hg.
Mammary glands are soft, nipples are clean without fissures. Lactation is good.
Abdomen is soft and painless. Uterine fundus is located 8cm below umbilicus. The
lochia should be:
A. Bloody
B. Bloody-serous
C. Mucous
D. Purulent
E. Serous
ANSWER: B

25 years old woman is presented in postpartum period after physiological delivery. At


objective examination her temperature is 36,7 0C, Ps - 72/min, BP - 110/70 mm Hg.
Mammary glands are soft, nipples are clean without fissures. Lactation is good.
Abdomen is soft and painless. Uterine fundus is located in the midway between
umbilicus and xiphoid process. The lochia should be:
A. Bloody
B. Bloody-serous
C. Mucous
D. Purulent
E. Serous
ANSWER: B

22 years old woman is presented at postpartum period after physiological delivery. At


objective examination her temperature is 36,7 0C, Ps - 72/min, BP - 110/70 mm Hg.
Mammary glands are soft, nipples are clean without fissures. Lactation is good.
Abdomen is soft and painless. Uterine fundus is located 2cm below umbilicus. The
lochia should be:
A. Bloody
B. Bloody-serous
C. Mucous
D. Purulent
E. Serous
ANSWER: A

22 years old woman is presented at postpartum period. At objective examination her


temperature is 36,7 0C, Ps - 72/min, BP - 110/70 mm Hg. Mammary glands are soft,
nipples are clean without fissures. Lactation is good. Abdomen is soft and painless.
Uterine fundus is located 4cm below umbilicus. The lochia should be:
A. Bloody
B. Bloody-serous
C. Mucous
D. Purulent
E. Serous
ANSWER: A

18 years old woman is presented at postpartum period. At objective examination her


temperature is 36,7 0C, Ps - 72/min, AP - 110/70 mm Hg. Mammary glands are soft,
nipples are clean without fissures. Lactation is good. Abdomen is soft and painless.
Uterine fundus is located 2cm below umbilicus. Routine postpartum care should include
searching for all of the following EXEPT:
A. Fever
B. Anemia
C. Diabetes
D. Depression
E. Thrombophlebitis
ANSWER: C

44 years old woman is presented at postpartum period after fifth physiological


delivery. At objective examination her temperature is 36,7 0C, Ps - 72/min, BP -
110/70 mm Hg. Mammary glands are soft, nipples are clean without fissures. Lactation
is good. Abdomen is soft and painless. Uterine fundus is located 6cm below umbilicus.
Condition to consider before performing a postpartum sterilization include all of the
following EXEPT:
A. A signed permit
B. Normal coagulogram
C. Negative pap smear
D. The infants’ well being
E. The patient awareness of hormonal suppression therapy
ANSWER: E

A patient is presented at 14 day of postpartum period after second physiological


delivery. She is found to have spotting and increasing of the temperature. At
objective examination her temperature is 38,7 0C, Ps - 100/min, BP - 110/70 mm Hg.
Mammary glands are soft, nipples are clean without fissures. Lactation is good.
Abdomen is soft and painful in the lover parts. In bimanual examination the uterus is
enlarged and tender. The primary diagnosis would be:
A. Pyelonephritis
B. Endometritis
C. Parametritis
D. Mastitis
E. Vaginitis
ANSWER: B

A patient is presented at 12 day of postpartum period after first physiological


delivery. She is found to have high temperature, pain in the left mammary gland. At
objective examination her temperature is 39, 7 0C, Ps - 110/min, BP - 110/70 mm Hg.
Left mammary gland is swollen, painful. Lactation is prominent. In bimanual
examination the uterus is normal. The primary diagnosis would be:
A. Pyelonephritis
B. Endometritis
C. Lactostasis
D. Peritonitis
E. Vaginitis
ANSWER: C

29 years old woman is presented in first stage of the second labor. Uterine
contractions are every 3 minutes by 25 seconds. In Leopold maneuvers you have palpated
fetal head above the pelvis inlet and fetal buttocks in the uterine fundus. Which lie
is determined by a doctor?
A. Longitudinal
B. Transverse
C. Oblique
D. Breech
E. Cephalic
ANSWER: A

34 years old woman is presented in first stage of labor. Uterine contractions are
every 4 minutes by 20 seconds. In Leopold maneuvers you have palpated fetal head above
the pelvis inlet and fetal buttocks in the uterine fundus. Which presentation is
determined by a doctor?
A. Longitudinal
B. Transverse
C. Oblique
D. Breech
E. Cephalic
ANSWER: E

32 years old woman is presented in first stage of the labor. Uterine contractions are
every 4 minutes by 20 seconds. In Leopold maneuvers you have palpated fetal buttocks
above the pelvis inlet and fetal head in the uterine fundus. Which presentation is
determined by a doctor?
A. Longitudinal
B. Transverse
C. Oblique
D. Breech
E. Cephalic
ANSWER: D

42-years old patient is hospitalized to Pathologic Pregnancy Department at 38 week of


gestation. During external obstetric examination transverse lie of the fetus was
diagnosed. Fetal head is presented to the left uterine wall. Which fetal position is
diagnosed in the patient?
A. I
B. II
C. III
D. IV
E. V
ANSWER: A

42-years old patient is hospitalized to Pathologic Pregnancy Department at 39 week of


gestation. During external obstetric examination transverse lie of the fetus is
diagnosed. Fetal head is presented to the right uterine wall. Indicate the fetal
presentation.
A. Cephalic
B. Breech
C. Shoulder
D. Knee
E. Presented part is absent
ANSWER: E

42-years old patient is hospitalized to Pathologic Pregnancy Department at 38 week of


gestation. During external obstetric examination transverse lie of the fetus is
diagnosed. Fetal head is presented to the right uterine wall. Which fetal position is
diagnosed in the patient?
A. I
B. II
C. III
D. IV
E. V
ANSWER: B

38-years old patient is hospitalized to Pathologic Pregnancy Department at 40 week of


gestation. During external obstetric examination fetal head is in the mother’s hip.
Which fetal lie is diagnosed in the patient?
A. Transverse
B. Oblique
C. Longitudinal
D. Cephalic
E. Breech
ANSWER: B

28-years old patient is hospitalized to Pathologic Pregnancy Department at 40 week of


gestation. During external obstetric examination fetal head is in the mother’s hip.
Which fetal lie is diagnosed in the patient?
A. Transverse
B. Oblique
C. Longitudinal
D. Cephalic
E. Breech
ANSWER: B

42-years old patient is hospitalized to Pathologic Pregnancy Department at 38 week of


gestation. During external obstetric examination transverse lie of the fetus is
diagnosed. Fetal head is presented to the left uterine wall. Which fetal position is
diagnosed in the patient?
A. I
B. II
C. III
D. IV
E. V
ANSWER: A

22-years old patient is hospitalized to Pathologic Pregnancy Department at 37 week of


gestation. During external obstetric examination fetal head is in the right mother’s
hip. Which fetal position is diagnosed in the patient?
A. I
B. II
C. III
D. IV
E. V
ANSWER: B

28-years old patient is hospitalized to Pathologic Pregnancy Department at 37 week of


gestation. During external obstetric examination fetal head is in the left mother’s
hip. Which fetal position is diagnosed in the patient?
A. I
B. II
C. III
D. IV
E. V
ANSWER: A

27-years old patient is hospitalized to Pathologic Pregnancy Department at 40 week of


gestation. During external obstetric examination fetal buttocks are in the mother’s
hip. Which fetal lie is diagnosed in the patient?
A. Transverse
B. Oblique
C. Longitudinal
D. Cephalic
E. Breech
ANSWER: B

22 years old woman is presented in first stage of the labor. Uterine contractions are
every 5 minutes by 20 seconds. In Leopold maneuvers fetal buttocks are palpated to the
right uterine wall and fetal head to the left. Which lie is determined by a doctor?
A. Longitudinal
B. Transverse
C. Oblique
D. Breech
E. Cephalic
ANSWER: B
Patient M. in 39 week of gestation is presented in the first stage of labor. In
vaginal examination the cervix is effaced and 4cm dilated, fetal head is in the pelvic
inlet. A sagittal suture is in the right oblique size, a small fontanel is to the
right close to the sacral bone. Determine the position and the variety of the fetus?
A. II position, the posterior variety
B. I position, the anterior variety
C. I position, the posterior variety
D. II position, the anterior variety
E. Cephalic position, anterior variety
ANSWER: A

Patient N. in 38 week of gestation is presented in the first stage of labor. In


vaginal examination the cervix is effaced and 5cm dilated, fetal head is in the pelvic
inlet. A sagittal suture is in the right oblique size, a small fontanel is to the left
anteriorly. Determine the position and the variety of the fetus?
A. II position, the posterior variety
B. I position, the anterior variety
C. I position, the posterior variety
D. II position, the anterior variety
E. Cephalic position, anterior variety
ANSWER: B

Patient F. in 38 week of gestation is presented in the first stage of labor. In


vaginal examination the cervix is effaced and 6cm dilated, fetal head is in the pelvic
inlet. A sagittal suture is in the left oblique size, a small fontanel is to the left
close to a sacral bone. Determine the position and the variety of the fetus?
A. I position, the posterior variety
B. I position, the anterior variety
C. II position, the posterior variety
D. II position, the anterior variety
E. Cephalic position, anterior variety
ANSWER: A

Patient F. in 38 week of gestation is presented in the first stage of labor. In


vaginal examination the cervix is effaced and 6cm dilated, fetal head is in the pelvic
inlet. A sagittal suture is in the left oblique size, a small fontanel is to the right
anteriorly. close to a sacral bone. Determine the position and the variety of the
fetus?
A. II position, the posterior variety
B. I position, the anterior variety
C. I position, the posterior variety
D. II position, the anterior variety
E. Cephalic position, anterior variety
ANSWER: D

Patient M. in 39 week of gestation is presented in the first stage of labor. In


vaginal examination the cervix is effaced and 4cm dilated, fetal head is in the pelvic
inlet. A sagittal suture is in the right oblique size, a small fontanel is to the
right close to the sacral bone. Choose the place for the best fetal heart rate
auscultation.
A. From the right below the umbilicus
B. From the left below the umbilicus
C. On the level of umbilicus
D. From the right above umbilicus
E. From the left above umbilicus
ANSWER: A

Patient N. in 38 week of gestation is presented in the first stage of labor. In


vaginal examination the cervix is effaced and 5cm dilated, fetal head is in the pelvic
inlet. A sagittal suture is in the right oblique size, a small fontanel is to the left
anteriorly. Determine the position and the variety of the fetus?
A. From the right below the umbilicus
B. From the left below the umbilicus
C. On the level of umbilicus
D. From the right above umbilicus
E. From the left above umbilicus
ANSWER: B

Patient F. in 38 week of gestation is presented in the first stage of labor. In


vaginal examination the cervix is effaced and 6cm dilated, fetal head is in the pelvic
inlet. A sagittal suture is in the left oblique size, a small fontanel is to the right
anteriorly. Determine the position and the variety of the fetus?
A. From the right below the umbilicus
B. From the left below the umbilicus
C. On the level of umbilicus
D. From the right above umbilicus
E. From the left above umbilicus
ANSWER: A

Patient F. in 38 week of gestation is presented in the first stage of labor. In


vaginal examination the cervix is effaced and 6cm dilated, fetal head is in the pelvic
inlet. A sagittal suture is in the left oblique size, a small fontanel is to the right
anteriorly. close to a sacral bone. Determine the position and the variety of the
fetus?
A. From the right below the umbilicus
B. From the left below the umbilicus
C. On the level of umbilicus
D. From the right above umbilicus
E. From the left above umbilicus
ANSWER: A

Patient N., II labor. The patient’ condition is satisfactory. Uterine contractions


are active. Fetal head is palpated on the left side from umbilicus, fetal buttocks
from the right side. Your initial diagnosis is:
A. Transverse lie, first position
B. Breech presentation
C. Transverse position
D. Sinciput vertex presentation
E. Transverse lie, second position
ANSWER: A

Patient S., II labor. The patient’ condition is satisfactory. Uterine contractions


are active. Fetal head is palpated on the right side from umbilicus, fetal buttocks
from the left side. Your initial diagnosis is:
A. Transverse lie, first position
B. Breech presentation
C. Transverse position
D. Sinciput vertex presentation
E. Transverse lie, second position
ANSWER: E

33 years old patient is presented in the first stage of labor. Fetal head is on -2
station. Fetal back is to the left, buttocks are presented. Fetal heart rate is
clear, rhythmic, 136 in 1min. Which lie is present in the patient?
A. Transverse lie
B. Oblique
C. Longitudinal
D. Breech
E. Shoulder
ANSWER: C

In vaginal examination of a multipara the cervix is 100% effaced and 5 cm dilated.


Fetal buttocks are palpated in the level of pelvic inlet. The intertrochanteric
diameter is in the right oblique size, the fetal sacrum is anteriorly. What is the
diagnosis?
A. I position, the posterior variety
B. I position, the anterior variety
C. II position, the posterior variety
D. II position, the anterior variety
E. Cephalic position, anterior variety
ANSWER: D

In vaginal examination of a multipara the cervix is 100% effaced and 6 cm dilated.


Fetal buttocks are palpated in -1 station. The intertrochanteric diameter is in the
left oblique size, the fetal sacrum is anteriorly. What is the diagnosis?
A. I position, the posterior variety
B. I position, the anterior variety
C. II position, the posterior variety
D. II position, the anterior variety
E. Cephalic position, anterior variety
ANSWER: B

In vaginal examination of 41 years old patient the cervix is 100% effaced and 6 cm
dilated. Fetal buttocks are palpated in -2 stations. The intertrochanteric diameter is
in the left oblique size, the fetal sacrum is posteriorly. What is the diagnosis?
A. I position, the posterior variety
B. I position, the anterior variety
C. II position, the posterior variety
D. II position, the anterior variety
E. Cephalic position, anterior variety
ANSWER: C

In vaginal examination of 33 years old patient the cervix is 100% effaced and 7 cm
dilated. Fetal buttocks are palpated in 0 station. The intertrochanteric diameter is
in the right oblique size, the fetal sacrum is posteriorly. What is the diagnosis?
A. I position, the posterior variety
B. I position, the anterior variety
C. II position, the posterior variety
D. II position, the anterior variety
E. Cephalic position, anterior variety
ANSWER: A

Patient N., II labor. The patient’ condition is satisfactory. Uterine contractions


are active. Fetal head is palpated on the right from umbilicus; fetal buttocks from
the left. Uterine cervix is dilated to 3cm, amniotic membranes are absent. Your
obstetric diagnosis:
A. Transverse lie, right position
B. Transverse lie, left position
C. Oblique lie
D. Breech presentation
E. Cephalic position, anterior variety
ANSWER: A

Patient N., II labor. The patient’ condition is satisfactory. Uterine contractions


are active. Fetal head is palpated on the left from umbilicus; fetal buttocks from the
right. Uterine cervix is dilated to 3cm, amniotic membranes are absent. Your
obstetric diagnosis is:
A. Transverse lie, right position
B. Transverse lie, left position
C. Oblique lie
D. Breech presentation
E. Cephalic position, anterior variety
ANSWER: B
The anatomically contracted pelvis is associated with:
A. true conjugate 9 cm
B. internal conjugate 11.5 cm
C. external conjugate 20.5 cm
D. diagonal conjugate 13.5 cm
E. cephalopelvic disproportion
ANSWER: A

The clinically contracted pelvis is associated with:


A. true conjugate 9 cm
B. true conjugate 11.5 cm
C. true conjugate 12.5 cm
D. true conjugate 13.5 cm
E. cephalopelvic disproportion
ANSWER: E

In a nullipara at term the diagonal conjugate is10.5 cm. What is the true conjugate?
A. 5cm
B. 6cm
C. 7cm
D. 8cm
E. 9cm
ANSWER: E

What is the main cause of cephalo-pelvic disproportion?


A. rachitis
B. fetal macrosomia
C. preterm labor
D. twins
E. fetal distress
ANSWER: B

Excessive compression of birth canal‘ soft tissues would provoke:


A. uterine rupture
B. cervical rupture
C. vesicovaginal fistulas
D. perineal rupture
E. vaginal rupture
ANSWER: C

The complication of cephalopelvic disproportion is:


A. uterine rupture
B. cervical rupture
C. vesicovaginal fistules
D. perineal rupture
E. vaginal rupture
ANSWER: A

The pathological contractile ring is a sign of:


A. anatomically contracted pelvis
B. fetal distress
C. fetal macrosomia
D. danger of uterine rupture
E. fetal distress
ANSWER: D

Management of the clinically contracted pelvis is:


A. normal vaginal delivery
B. cesarean section
C. obstetrical version
D. fetal destroying operation
E. external cephalic version
ANSWER: B

Female pelvic sizes are: 25 – 28 – 31 – 20 cm. What is the pelvic type?


A. normal pelvis
B. generally contracted pelvis
C. flat pelvis
D. flat rachitic pelvis
E. transverse contracted flat pelvis
ANSWER: A

Female pelvic sizes are: 23 – 26 – 29 – 17 cm. What is the pelvic type?


A. normal pelvis
B. transverse contracted pelvis
C. flat pelvis
D. flat rachitic pelvis
E. generally contracted pelvis
ANSWER: E

Female pelvic sizes are: 23 – 26 – 29 – 18 cm. What is the pelvic type?


A. normal pelvis
B. generally contracted pelvis
C. flat pelvis
D. flat rachitic pelvis
E. transverse contracted pelvis
ANSWER: B

Female pelvic sizes are: 25 – 28 – 31 – 18 cm. What is the pelvic type?


A. normal pelvis
B. generally contracted pelvis
C. simple flat pelvis
D. flat rachitic pelvis
E. transverse contracted pelvis
ANSWER: C

Female pelvic sizes are: 26 – 26 – 31 – 17 cm. What is the pelvic type?


A. normal pelvis
B. generally contracted pelvis
C. flat pelvis
D. flat rachitic pelvis
E. transverse contracted flat pelvis
ANSWER: D

Generally contracted pelvis is characterized by:


A. Decreasing of all pelvic diameters
B. Decreasing of all pelvic anteroposterior diameters
C. Decreasing of all pelvic transverse diameters
D. Decreasing of true conjugate and increasing of the pelvic outlet
E. Combination of generally contracted and flat pelvis
ANSWER: A

Simple flat pelvis is characterized by:


A. Decreasing of all pelvic diameters
B. Decreasing of all pelvic anteroposterior diameters
C. Decreasing of all pelvic transverse diameters
D. Decreasing of true conjugate and increasing of the pelvic outlet
E. Combination of generally contracted and flat pelvis
ANSWER: B

Flat rachitic pelvis is characterized by:


A. Decreasing of all pelvic diameters
B. Decreasing of all pelvic anteroposterior diameters
C. Decreasing of all pelvic transversal diameters
D. Decreasing of true conjugate and increasing of the pelvic outlet
E. Combination of generally contracted and flat pelvis
ANSWER: D

What is the cause of cephalo-pelvic disproportion?


A. Face presentation anterior
B. Face presentation posterior
C. Preterm labor
D. Twins
E. Fetal distress
ANSWER: A

Transverse contracted pelvis is characterized by:


A. Decreasing of all pelvic diameters
B. Decreasing of all pelvic anteroposterior diameters
C. Decreasing of all pelvic transverse diameters
D. Decreasing of true conjugate and increasing of the pelvic outlet
E. Combination of generally contracted and flat pelvis
ANSWER: C

The true conjugate of the flat pelvis is 9.5 cm. What is the degree of pelvic
contraction?
A. I degree
B. II degree
C. III degree
D. IV degree
E. V degree
ANSWER: A

The true conjugate of the flat pelvis is 8.5 cm. What is the degree of pelvic
contraction?
A. I degree
B. II degree
C. III degree
D. IV degree
E. V degree
ANSWER: B

The true conjugate of the flat pelvis is 8.0 cm. What is the degree of pelvic
contraction?
A. I degree
B. II degree
C. III degree
D. IV degree
E. V degree
ANSWER: B

The true conjugate of the generally contracted pelvis is 7.0 cm. What is the degree of
pelvic contraction?
A. I degree
B. II degree
C. III degree
D. IV degree
E. V degree
ANSWER: C

The true conjugate of the generally contracted pelvis is 6.5 cm. What is the degree of
pelvic contraction?
A. I degree
B. II degree
C. III degree
D. IV degree
E. V degree
ANSWER: C

The true conjugate of the generally contracted pelvis is 5.0 cm. What is the degree of
pelvic contraction?
A. I degree
B. II degree
C. III degree
D. IV degree
E. V degree
ANSWER: D

Indicate “rare occurred” type of contracted pelvis:


A. osteomalatic pelvis
B. generally contracted pelvis
C. flat pelvis
D. flat rachitic pelvis
E. generally contracted pelvis
ANSWER: A

Management of the patients with I degree of pelvic contraction and probable fetal
weight 2900g:
A. vaginal delivery
B. cesarean section only
C. vaginal delivery or cesarean section
D. fetal destroying operation
E. labor preparing operation
ANSWER: A

Management of the patients with II degree of pelvic contraction and probable fetal
weight 2800g:
A. vaginal delivery
B. cesarean section only
C. vaginal delivery or cesarean section
D. fetal destroying operation
E. labor preparing operation
ANSWER: A

Management of the patients with III degree of pelvic contraction:


A. vaginal delivery
B. cesarean section only
C. vaginal delivery or cesarean section
D. fetal destroying operation
E. labor preparing operation
ANSWER: B

Management of the patients with IV degree of pelvic contraction:


A. vaginal delivery
B. cesarean section only
C. vaginal delivery or cesarean section
D. fetal destroying operation
E. labor preparing operations
ANSWER: B

What is the cause of cephalo-pelvic disproportion?


A. sinciput vertex presentation and large fetus
B. face presentation posterior
C. occiput presentation
D. foot-link presentation
E. knee-link presentation
ANSWER: A

What is the cause of cephalo-pelvic disproportion?


A. frank breech presentation
B. sinciput vertex presentation and III degree of pelvic contraction
C. occiput presentation
D. foot-link presentation
E. knee-link presentation
ANSWER: B

“Three rule” catheters in hemorrhagic shock management includes:


A. Rectal enema, vein catheterization, urinary bladder catheterization
B. Intestinal intubation, vein catheterization, urinary bladder catheterization
C. Oxygen require, vein catheterization, urinary bladder catheterization
D. Vein catheterization, urinary bladder catheterization
E. Rectal enema, vein catheterization
ANSWER: C

Amniotic fluid embolism is differentiated with the all of the below pathologies EXEPT:
A. Myocardial infarction
B. Pulmonary artery embolism
C. Air embolism
D. Shyhan’syndrome
E. Mendelson’ syndrome
ANSWER: D

The bleeding began right after childbirth. The blood loss is 300 ml. Placenta doesn’t
separate in active management. What is the most probable diagnosis?
A. Abnormal placenta adherence
B. Cervical laceration
C. Uterine atony
D. Uterine rupture
E. Placental abruption
ANSWER: A

What does “А” mean in ABC steps of massive postpartum bleeding management?
A. Airways evaluation
B. Breathing evaluation
C. Circulation evaluation
D. Keep the woman warm during appropriate available measures
E. Flat position
ANSWER: A

What does “B” mean in ABC steps of massive postpartum bleeding management?
A. Airways evaluation
B. Breathing evaluation
C. Circulation evaluation
D. Keep the woman warm during appropriate available measures
E. Flat position
ANSWER: B

What does “C” mean in ABC steps of massive postpartum bleeding management?
A. Airways evaluation
B. Breathing evaluation
C. Circulation evaluation
D. Keep the woman warm during appropriate available measures
E. Flat position
ANSWER: C

Restoration of the airways patency in ABC steps of massive postpartum bleeding


includes:
A. Cleanse the mouth, opening the mouth, laryngomask, tracheal intubation
B. Intravenous access
C. Rapid warmed infusion of fluids
D. Keep the woman warm during appropriate available measures
E. By position flat
ANSWER: A

Breathing maintenance in ABC steps of massive postpartum bleeding:


A. Cleanse the mouth, opening the mouth, laryngomask, tracheal intubation
B. Intravenous access
C. Rapid warmed infusion of fluids
D. Keep the woman warm during appropriate available measures
E. Oxygen by mask, Ambu bag, pulmonary ventilation
ANSWER: E

What is the recommendation for blood transfusion in massive postpartum bleeding?


A. Blood transfusion is not recommended
B. Blood transfusion on the first day of postpartum period
C. Blood transfusion on the second day of postpartum period
D. Blood transfusion on the third day of postpartum period
E. Transfuse blood as soon as possible
ANSWER: E

Circulation assessment in ABC steps of massive postpartum bleeding include:


A. Determination of pulse
B. Determination of pulse rate, blood pressure
C. Determination of blood pressure
D. Determination of temperature
E. Determination of woman weight
ANSWER: B

First steps to maintain circulation in ABC steps of massive postpartum bleeding


include:
A. Fresh frozen plasma infusion
B. Blood transfusion
C. Leukocytes mass transfusion
D. Recombinant factor VIIa therapy
E. Infusion up to 3.5 litres of warmed crystalloid Hartmann’s solution (2 litres)
and/or colloid (1–2 litres) as rapidly as required
ANSWER: E

What is the second line contractile drug for atonic postpartum bleeding treatment?
A. Remestyp
B. Oxytocin
C. Water pepper tincture
D. Tranexamic acid
E. Carbetocin
ANSWER: E

Postpartum hemorrhage is:


A. more than 50 ml
B. more than 100 ml
C. more than 200 ml
D. more than 300 ml
E. more than 500ml
ANSWER: E

For prevention of bleeding in late postpartum period is necessary to prescribe:


A. Vicasol
B. Ultrasonic investigation of uterus
C. General blood test
D. General urine test
E. Oxytocin
ANSWER: E

How many units of oxytocin initially you should prescribe for prevention of postpartum
bleeding?
A. 5
B. 10
C. 15
D. 20
E. 25
ANSWER: B

Carbetocin is called as:


A. Pabal
B. Oxytocin
C. Enzaprost
D. Dinoprost
E. Mezaton
ANSWER: A

How many mkg of carbetocin does 1 ml of pabal contain?


A. 100
B. 200
C. 300
D. 400
E. 500
ANSWER: A

How do you called drug which contain terlipresyn?


A. Remestyp
B. Oxytocin
C. Enzaprost
D. Dinoprost
E. Mezaton
ANSWER: A

How many mkg of terlipresyn does 1 ml of remestyp contain?


A. 100
B. 200
C. 300
D. 400
E. 500
ANSWER: A

Primary postpartum bleeding:


A. Occur in early postpartum period or during 24 hours after delivery
B. Occur in early postpartum period
C. Occur in late postpartum period
D. Occur till 22 weeks of pregnancy
E. Occur in labor
ANSWER: A

Secondary postpartum bleeding:


A. Occur in early postpartum period or during 24 hours after delivery
B. Occur in early postpartum period
C. Occur after 24 hours till 6 weeks after delivery
D. Occur till 22 weeks of pregnancy
E. Occur in labor
ANSWER: C

What is the first line contractile drug for atonic postpartum bleeding treament?
A. Remestyp
B. Oxytocin
C. Enzaprost
D. Dinoprost
E. Ergometryn
ANSWER: B

Which initial dose of tranexamic acid is the maximum for postpartum bleeding treatment?
A. 1000-1500mkg
B. 200mkg
C. 300mkg
D. 100mkg
E. 500mkg
ANSWER: A

What is the second line contractile drug for atonic postpartum bleeding treatment?
A. Remestyp
B. Oxytocin
C. Water pepper tincture
D. Tranexamic acid
E. Ergometryn
ANSWER: E

What is the third line contractile drug for atonic postpartum bleeding treatment?
A. Remestyp
B. Oxytocin
C. Water pepper tincture
D. Tranexamic acid
E. Carboprost
ANSWER: E

What is the second line contractile drug for atonic postpartum bleeding treatment?
A. Remestyp
B. Oxytocin
C. Water pepper tincture
D. Tranexamic acid
E. Misoprostol
ANSWER: E

What is the next step after prescription 2-3 line contractile drugs for atonic
postpartum bleeding treatment?
A. External massage of the uterus
B. Oxytocin prescription
C. Bimanual uterine compression
D. Tranexamic acid prescription
E. Balloon tamponade of the uterus
ANSWER: C

What is the next step after bimanual uterine compression for atonic postpartum
bleeding treatment?
A. External massage of the uterus
B. Oxytocin prescription
C. Manual exploration of uterine cavity
D. Tranexamic acid prescription
E. Balloon tamponade of the uterus
ANSWER: E

What is the leading reason for early postpartum bleeding?


A. Uterine atony
B. Amniotic fluid embolism
C. DIC syndrome
D. Lacerations of the birth canal
E. Abnormal placenta adherence
ANSWER: A

Small hemorrhage in obstetrics is:


A. blood loss till 250ml which stopped
B. blood loss till 350ml which stopped
C. blood loss till 450ml which doesn’t stopped
D. blood loss till 500 ml which stopped
E. blood loss till 1000ml which doesn’t stopped
ANSWER: A

How does the anti-shock garment work:


A. Returns blood to vital organs
B. Stimulates uterine contraction
C. Stimulates prostaglandins synthesis
D. Prevents uterine atony
E. Inhibits proteolysis
ANSWER: A

How does the anti-shock garment works:


A. Decreases blood flow in the compresses organs
B. Stimulate s uterine contraction
C. Stimulates prostaglandins synthesis
D. Prevents uterine atony
E. Inhibits proteolysis
ANSWER: A

Healing of true erosion is going with:


A. connective tissue
B. single-layered cylindrical epithelium
C. multilayer pavement epithelium
D. growth of secretory glands
E. metaplasia
ANSWER: B

True erosion is diagnosed with:


A. Colposcopy
B. Bimanual assessment
C. Ultrasonography
D. Smear bacterioskopy
E. cytology
ANSWER: A

What changes of multilayer squamosus epithelium is common for true erosion?


A. Proliferation
B. desquamation
C. Ectopia
D. Metaplasia
E. Regeneration
ANSWER: D

Which process represents the IIIa type of smear at oncocytological examination?


A. the unchanged epithelium
B. mild or moderate dysplasia
C. cancer
D. inflammatory process
E. suspicion on malignisation
ANSWER: B

True erosion is diagnosed with


A. colposkopy
B. smear bacterioskopy
C. ultrasound
D. bimanual assessment
E. cytology
ANSWER: A

Etiological factors of psedoerosin is all EXCEPT:


A. Dyshormonal disturbances
B. Autoimmune process
C. Alimentary factor
D. Changes of humoral immunity
E. Inflammatory process
ANSWER: C

Cancer prevention program for women include all of the following except:
A. Smoking cessation
B. Periodic screening
C. Bone density index measurement
D. Elimination of unopposed estrogen use for menopause
E. Human papillomavirus (HPV) tests.
ANSWER: C

For polyp treatment we use all except:


A. cryodestruction
B. conization of cervix
C. endocervical curettage
D. Coagulation
E. polyp twisting
ANSWER: B

To the risk factors for dysplasia belong all EXCEPT:


A. hyperprogesteronemia
B. hyperestrogenemia
C. cervical trauma during delivery and abortion
D. A lot of sexual partner
E. Early sexual activity
ANSWER: A

What sign is not typical for proliferative myoma?


A. increased mitotic activity
B. fast growth
C. plenty of lymphoid cells
D. plenty of plasmatic cells
E. Atypical growth
ANSWER: E

In diagnostic of cervical dysplasia we use all EXCEPT:


A. colposcopy
B. metrosalpingography
C. cytogenetic examination
D. histochemical examination
E. cytological examination
ANSWER: B

In what type of ovarion tumour do we have an endometrium hyperplasia?


A. papillary cystoma
B. paraovarian cyst
C. follicle cyst
D. ovarian cancer
E. yellow body cyst
ANSWER: C

In diagnostic of cervical leucoplacia we use all EXCEPT:


A. culdoscopy
B. colposcopy
C. biopsy
D. cytology
E. Speculum assessment
ANSWER: A

Node consistency of uterine fibromyoma depends on:


A. node size
B. vessels' amount
C. fatty tissue amount
D. presence of endometrioid tissues
E. correlation of parenchyma and stroma
ANSWER: E

What is typical for proliferative myoma?


A. presence of atypia
B. atypical localization
C. multiple nodes
D. node calcification
E. increased mitotic activity
ANSWER: E

What sign is not typical for proliferative myoma?


A. increased mitotic activity
B. fast growth
C. plenty of lymphoid cells
D. plenty of plasmatic cells
E. Atypical growth
ANSWER: E

Subserouse fibromyoma node is localizes in:


A. between broad ligament layers
B. behind cervix
C. in myometrium
D. under uterine mucous layer
E. under peritoneum
ANSWER: E

Submucous myoma node is localized:


A. between broad ligament layers
B. behind cervix
C. in myometrium
D. under uterine mucous layer
E. under peritoneum
ANSWER: D

Intramural myoma node is localized:


A. between broad ligament layers
B. under uterine mucous layer
C. behind cervix
D. in myometrium
E. under peritoneum
ANSWER: D

Interstitial myoma node is localized:


A. between broad ligament layers
B. in myometrium
C. under uterine mucous layer
D. under peritoneum
E. behind cervix
ANSWER: B

Intraligamentary myoma node is localized:


A. between broad ligament layers
B. in myometrium
C. behind cervix
D. under uterine mucous layer
E. under peritoneum
ANSWER: A
Retrocervical myoma node is localized:
A. between broad ligament layers
B. under peritoneum
C. under uterine mucous layer
D. behind cervix
E. in myometrium
ANSWER: D

What is typical for hormonal status of patient with fibromyoma?


A. high level of estrogens
B. high level of progesteron
C. high level of androgens
D. high level of chorionic gonadotropin
E. high level of pituitary gland hormons
ANSWER: A

What sign is typical for submucous myoma?


A. hyperpolymenorrhea
B. amenorrhea
C. foamy vaginal discharge
D. symptomless
E. tumour destruction
ANSWER: A

What sign is typical for retrocervical myoma?


A. rectum dysfunction
B. infertility
C. amenorrhea
D. foamy vaginal discharge
E. hyperpolymenorrhea
ANSWER: A

What method should be used for diagnostic of subserous myoma?


A. curettage of uterine cavity
B. hysterography
C. uterine probing
D. hysterosalpingography
E. ultrasound
ANSWER: E

What method should be used for diagnostic submucous myoma?


A. puncture of abdominal cavity through posterior vaginal fornix
B. laparoscopy
C. biopsy
D. Doppler assessment
E. hysteroscopy
ANSWER: E
What method should be used for diagnostic interstitial myoma?
A. hysterosalpingography
B. ultrasound
C. hysteroscopy
D. curettage of uterine cavity
E. uterine probing
ANSWER: B

In what form of uterine fibromyoma we have such complication as node twisting?


A. retrocervical
B. interstitial
C. submucosal
D. subserosal
E. intraligamentous
ANSWER: D

In what form of uterine fibromyoma we have such complication as node delivering?


A. retrocervical
B. interstitial
C. submucous
D. subserous
E. intraligamentous
ANSWER: C

What endometrium conditions belong to the premalignant disease?


A. glandular-cystic hyperplasia
B. endometritis
C. polyposis of endometrium
D. endometriosis
E. glandular hyperplasia
ANSWER: C

What is typical for hormonal status of patient with hyperplasia of endometrium?


A. high level of estrogens
B. high level of pituitary gland hormons
C. high level of androgens
D. high level of progesteron
E. high level of chorionic gonadotropin
ANSWER: A

How pregnancy does influence on fibromyoma’s growth?


A. doesn’t effect at all
B. promote fibromyoma’s elimination
C. promote fibromyoma’s malignization
D. promote necrosis of node
E. promote fast growth of fibromyoma
ANSWER: E

What substance is in IUD for fibromyoma treatment?


A. levonorgestrel
B. synestrol
C. 17-oxiprogesteroni capronat
D. Zoladex
E. Etinilestradiol
ANSWER: A

At what type of ovarian tumour does endometrium hyperpasia develop?


A. papillary cystoma
B. paraovarian cyst
C. follicular cyst
D. ovarian cancer
E. yellow body cyst
ANSWER: C

What tumour of external genitalia develops from connective tissue?


A. papilloma
B. hemangioma
C. myxoma
D. lipoma
E. fibroma
ANSWER: E

What tumour of external genitalia develops from fatty and connective tissue?
A. fibroma
B. lipoma
C. myxoma
D. hemangioma
E. papilloma
ANSWER: B

What treatment should be prescribes for patient with dermoid cyst?


A. antibacterial therapy
B. Zoladex
C. cystectomy
D. substitutive hormonal therapy
E. combined oral contraceptives
ANSWER: C

Bilateral ovarian cyst on ultrasound, associated with molar pregnancy and multiple
gestation is?
A. Metastasis
B. Endometriosis
C. Theca luteum cyst
D. PID (Pelvic inflammatory disease)
E. lipoma
ANSWER: C
Bilateral solid ovarian masses on ultrasound, metastases from primary GIT cancer is
suggestive for?
A. Krukenberg tumor
B. Luteoma
C. Fibrosis
D. Endometriosis
E. Theca luteum cyst
ANSWER: A

Cervical pap smear test is recommended to this age group with the timing of:
A. Twice in a lifetime- <25 years old
B. Once a lifetime- all ages
C. Every 10 years- >65 years
D. Every 3 years- 21 to 65 years
E. Every year- 5 to 18 years
ANSWER: D

Name the ovarian cyst with thick sebaceous yellow fluid in and ectodermal (hair)
component.
A. Hydatid mole
B. Trauma-related cyst
C. Mature cystic teratoma (dermoid cyst)
D. Cancerous cyst
E. Adenocarcinoma
ANSWER: C

Call the benign uterine fibroids, causing enlarged uterus with an irregular contour:
A. Adenocarcinoma
B. Fibroma
C. Leiomyoma uteri
D. Carcinoma
E. Mole
ANSWER: C

Yellow or yellow-brown masses, solid ovarian mass on ultrasound, 50% bilateral,


regress after delivery is?
A. Bartholian
B. Cancer
C. Krukenberg tumor
D. Luteoma
E. Theca luteum cyst
ANSWER: D

Diagnostic method to diagnose uterine Leiomyomas (fibroids):


A. Normal observation
B. MRI of pelvis
C. Bimanual exam
D. Pelvic x-ray
E. Ultrasound of pelvic
ANSWER: E

Which of them is NOT an indication for endometrial biopsy?


A. Lynch syndrome in patient <45 years
B. Atypical glandular cells on pap test
C. Postmenopausal bleeding
D. Obesity/anovulation
E. Sheehan syndrome
ANSWER: E

Soft mobile mass, normally asymptomatic, commonly on the base of the labium major
(vulva) is:
A. Syphilis related mass
B. Malignancy
C. HSV related cyst
D. HPV (wart)
E. Bartholin duct cyst
ANSWER: E

Cyst resulting from incomplete regression of the wolffian duct during fetal
development?
A. Vaginal cancer
B. Gartner duct cyst
C. Fibroid
D. Fissure/ Fistula
E. Bartholin cyst
ANSWER: B

Theca lutein cyst is an ovarian hyperstimulation due to?


A. Gestational trophoblastic disease
B. X-linked related
C. Autoimmunity
D. Anatomically malformation
E. Krukenberg tumor
ANSWER: A

Leiomyomata uteri (fibroids) are a common cause of?


A. Vaginal itching
B. Heavy menstrual bleeding
C. Ectopic pregnancy
D. Severe pelvic and leg pain
E. Vaginal discharge (yellowish)
ANSWER: B

Which of the following is correct regarding polyps?


A. Polyps are the most common cysts in the fallopian tubes
B. Polyps are similar to the fistula
C. Polyps causing secondary amenorrhea but only age after 52y
D. Polyps are always associated with pain
E. Polyps are not associated with pain
ANSWER: E

Choose the most common benign neoplasm of the female genital tract:
A. Endometrial cancer
B. Cervical cancer
C. Vagina cancer
D. Uterine leiomyoma (fibroids)
E. Polyps
ANSWER: D

Which of the following describes the uterine leiomyoma?


A. Discrete/ round ,firm
B. Attached, triangle-like, dense
C. Single, huge, full of pus, dense
D. Cuboid-shaped, pinkish-color, firm
E. Cylendric-chain looking, irregular borders
ANSWER: A

Indicate the key words about uterine myomas and their effects on body:
A. On physical exam uterus is irregular and mobile
B. On physical exam uterus is innervated and shrinky
C. There is lots of yellowish-brownish discharge
D. Heavy bleeding, leading to anemia (only at age <35.y.)
E. None of them
ANSWER: A

If a uterine mass continue to grow after menopausa, suspect……?


A. PID (Pelvic inflammatory disease)
B. Chronic abscess
C. Malignancy
D. Fistula
E. Normal, will be self-limited
ANSWER: C

What for the Gardasil vaccine may be used?


A. Endometrial cancer
B. Types of breast cancer
C. For protecting the recurrent miscarriage
D. Genital warts and cervical cancer
E. Reduces the development of ovarian cancer in young women
ANSWER: D

Choose a symptom of uterine prolapsed:


A. all of them
B. vaginal pressure sensation
C. vaginal fullness
D. problems with sexual intercourse
E. the uterus or cervix protruding out of the vagina.
ANSWER: A

Indicate the explanation for the cystocele:


A. Bladder prolapse into the vagina
B. Bladder prolapse under the diaphragm
C. Bladder rupture
D. Fistula in the vaginal canal
E. Partially rupture of the bladder
ANSWER: A

Indicate the types of incontinence:


A. Stress-mechanical
B. Stress-physiological
C. Urge-mechanical
D. Urge-neurological
E. Stress-urge
ANSWER: E

Indicate testing for stress incontinence:


A. Muscle strength + sphincter test
B. Needle biopsy + nerve study
C. Pelvic exam + Q-tip test
D. Inguinal canal exam
E. Schiller's test
ANSWER: C

Choose example of anatomically caused incontinence


A. Vesicovaginal fistula
B. Vaginal canal fistula
C. Uterine tumor
D. Ovarian tumor
E. Uterine cancer
ANSWER: A

Risk factor for breast cancer include:


A. Family history
B. History of human papillomavirus (HPV) infection
C. Early menopause
D. Low-fat diet
E. multiparity
ANSWER: A

Feelings of breast swelling, tenderness, or pain can begin in some women:


A. Two weeks after conception
B. One month after conception
C. Two months after conception
D. Three months after conception
E. During ovulation
ANSWER: A
Choose the first line treatment of breast abscess:
A. Drainage
B. Warm compress
C. Cold compress
D. Needle aspiration and antibiotic therapy
E. Nonsteroidal anti-inflammatory drugs (NSAIDs)
ANSWER: D

Explanation of the mammary Paget disease:


A. Autoimmunity
B. Bacterial infection
C. Viral-related
D. Painful, itchy, eczematous and/or ulcerating rash on the nipple
E. It is a rash on the nipple and is associated with obesity
ANSWER: D

Choose the explanation to define the intraductal papilloma:


A. Unilateral, bloody nipple discharge, no associated mass or lymphadenopathy
B. Bilateral milky discharge , mobile mass
C. No discharge, no mass, well erythema
D. Unilateral mass, firm, irregular, visible by eyes
E. Multynodular lesions on both nipples, spreading to breast/neck
ANSWER: A

Cancer prevention program for women include all of the following except:
A. Smoking cessation
B. Periodic screening
C. Bone density index measurement
D. Elimination of unopposed estrogen use for menopause
E. Human papillomavirus (HPV) tests
ANSWER: C

How often breastself assessment should be performed after 20 years?


A. every day
B. every week
C. every month
D. ones a year
E. ones a two years
ANSWER: C

What belongs to the risk factors of mammary gland cancer?


A. multyparity
B. first delivery after 20 years
C. menopause after 55 years
D. menarche before 15 years
E. using combined oral contraceptives
ANSWER: C
Bilateral mammography is a gold standart of mammary gland assessment because of:
A. tissue radioperceptivance of mature mammary gland
B. tissue radioresistance of mature mammary gland
C. tissue radioresistance of mammary gland neoplasms
D. tissue radioresistance of mammary gland neoplasms
E. no correct answer
ANSWER: B

Major role in the regulation of growth and development of breast play all hormones
EXCEPT:
A. Progesterone
B. Prolactin
C. Follicle-stimulating hormone (FSH)
D. Estradiol
E. None of the above
ANSWER: C

Which method is the most effective in detecting early breast cancer?


A. Inspection
B. Palpation
C. Thermography
D. Cytological examination of discharge from the nipple
E. Mammography
ANSWER: E

What is the main complaint in patients with mastopathy?


A. Disorders of the menstrual cycle
B. Pain
C. Increased body temperature
D. Increased libido
E. Increasing of breast size
ANSWER: B

Which is the most common form of breast cancer?


A. Ductal carcinoma in situ
B. Invasive ductal carcinoma
C. Infiltrating lobular carcinoma
D. invasive lobular carcinoma
E. Medullary Carcinoma
ANSWER: B

Clinical sign of malignant mammary gland tumour?


A. symmetrical nipples, big size
B. symmetrical nipples, pigmented
C. nipples retraction or nipple's skin
D. Weight gain
E. legs edema
ANSWER: C
What is the most visible sign of puberty?
A. Social withdrawal
B. Weight gain
C. Enlargement of the breast bud
D. Anger
E. Libido
ANSWER: C

Symptoms of fibrocystic breast disease might include ALL, EXCEPT:


A. Soreness or pain
B. Redness
C. Unusual discharges
D. A bulge or fibrocystic breast lump
E. nipples retraction or nipple's skin
ANSWER: E

Which of the following is all a benign breast disease, EXCEPT?


A. Fat necrosis
B. Fibroadenoma
C. Fibrocystic changes
D. Breast cyst
E. Ductal Carcinoma In Situ
ANSWER: E

Post traumatic or postsurgery, firm, irregular mass, +/- ecchymosis, skin/nipple


retraction indicates for?
A. Fat necrosis
B. Metastasis
C. Breast cyst
D. Ductal carcinoma
E. Tubular Carcinoma
ANSWER: A

Solitary, well-circumscribed, mobile mass, plus cyclic premenstrual tenderness is


indicative for?
A. Ductal carcinoma
B. Breast cyst
C. Fibroadenoma
D. Fat necrosis
E. Tubular Carcinoma
ANSWER: C

Multiple, diffuse, nodulocystic masses, plus premenstrual tenderness are suggestive


for?
A. Breast cyst
B. Fibrocystic changes
C. Carcinoma
D. Fat necrosis
E. Tubular Carcinoma
ANSWER: B

Tamoxifen and raloxifene are prescribed for estrogen receptor-positive breast cancer,
indicative most common side effect of them?
A. Psychosis
B. Constipation
C. Headache
D. Anorexia
E. Hot flashes
ANSWER: E

Which of the following could be a risk factor for breast cancer?


A. Hormone replacement therapy
B. Working with woods
C. Genetic in second-degree relatives
D. Weight gain
E. Increased libido
ANSWER: A

Management of choice for the lactational mastitis:


A. Nonsteroidal anti-inflammatory drugs (NSAIDs)
B. Warm compress
C. Stop breast feeding and antibiotic
D. Continue breast feeding and antibiotic
E. Cabergoline orally
ANSWER: D

Choose the case of engorgement?


A. Subareolar, immobile, tender mass
B. Focal tenderness, firmness, no fever
C. Bilateral, symmetric fullness, tenderness, warmth
D. Abrasion, bruising from poor latch
E. Unilateral red nipple
ANSWER: C

Which one indicating mastitis?


A. Erythema + fever and tenderness
B. Symmetric fullness, no fever, firmness
C. Cracking, blistering from poor latch
D. Bilateral milky discharge
E. Brown discharge and nipples contractions
ANSWER: A

Indicate case of galactocele:


A. Nipples contractions + discharge (brown)
B. Focal tenderness, firmness, Erythema
C. Bilateral, symmetric fullness, warmth, tenderness
D. Mastitis symptoms + fluctuant mass
E. Subareolar, mobile, well-circumscribed, non tender mass
ANSWER: E

Indicate plugged duct related to lactation:


A. Bruising, cracking, blistering from poor latch
B. Tenderness/ Erythema + fever
C. Focal tenderness, firmness, no fever, Erythema
D. Nipples discharge (copperish)
E. Symmetric, bilateral fullness
ANSWER: C

Indicate abscess related to lactation problems?


A. Copper color nipples discharge (bilaterally)
B. Symmetric, bilateral fullness
C. Symptoms of mastitis + fluctuant mass
D. Erythema/ tenderness + fever + discharge
E. Ductal invasive cancer
ANSWER: C

Rare but aggressive breast cancer with cutaneous thickening and “peaud'orange" :
A. Ductal papillary cancer
B. Inflammatory breast carcinoma
C. Metastasis from other cancer
D. Ductal invasive cancer
E. Fibroadenoma
ANSWER: B

Etiology of engorgement:
A. Hormonal changes after delivery
B. Milk production exceeds the release
C. Trauma
D. Bacterial
E. Viral
ANSWER: B

Which drug is contraindicated for woman with chronic hypertension?


A. Peripheral vasodilatators
B. Angiotensin-converting enzyme inhibitors
C. ?-blockers
D. Ca-channel blockers
E. ?2-receptor agonists
ANSWER: B

Which drug is contraindicated for woman with chronic hypertension?


A. Peripheral vasodilatators
B. Diuretics
C. ?-blockers
D. Ca-channel blockers
E. ?2-receptor agonists
ANSWER: B
What is the main recommendation for pregnant with chronic hypertension during routine
check up?
A. Activity restriction
B. Balanced nutrition
C. Increased fluid intake to ensure adequate hydration
D. Instruction about the effect of diuretics
E. Increased nutrition
ANSWER: A

The objective of magnesium sulfate therapy for the patient with preeclampsia is to:
A. Prevent convulsions
B. Promote diaphoresis
C. Increase reflex irritability
D. Act as a saline cathartic
E. Increase peripheral circulation
ANSWER: A

Which assessment should be performed during intravenous magnesium sulfate infusion


therapy for management of severe preeclampsia?
A. Count respirations and report a rate of less than 12 breaths per minute.
B. Count respirations and report a rate of more than 20 breaths per minute.
C. Check blood pressure and report a rate of less than 100/60.
D. Monitor urinary output and report a rate of less than 100 ml per hour.
E. Monitor reflexes and report increased reflexes
ANSWER: A

Which drug should be available for immediate IV administration when magnesium sulfate
toxity is developed:
A. Ergonovine maleate
B. Oxytocin
C. Calcium gluconate
D. Hydralazine
E. ?2-receptor agonists
ANSWER: C

Several pregnant clients are waiting to be seen in the triage area of the obstetrical
unit. Which client is the highest priority?
A. A client at 13 weeks' gestation experiencing nausea and vomiting three times a day
with + 1 ketones in her urine.
B. A client at 37 weeks' gestation who is an insulin-dependent diabetic and
experiencing 3 to 4 fetal movements per day.
C. A client at 32 weeks' gestation who has preeclampsia and + 3 proteinuria who is
returning for evaluation of epigastric pain.
D. A primigravida at 17 weeks' gestation complaining of not feeling fetal movement at
this point in her pregnancy.
E. A multigravida at 38 weeks’ gestation with mild irregular abdominal pain
ANSWER: C
What is the most appropriate client centered recommendation for a primigravid client
at 30 weeks' gestation diagnosed with mild preeclampsia
A. Return visit to the prenatal clinic in approximately 4 weeks.
B. Decreased edema after 1 week of a low-protein, low-fiber diet.
C. Bed rest on the left side during the day, with bath-room privileges.
D. Immediate reporting of adverse reactions to magnesium sulfate therapy.
E. Call physician immediately after development of swelling of the feet and ankles
ANSWER: C

Which of the following types of diet should be physician discussed with a multigravid
client diagnosed with mild preeclampsia?
A. High-residue diet.
B. Low-sodium diet.
C. Regular diet.
D. High-protein diet.
E. Low-protein diet.
ANSWER: C

Which of the following assessment findings would alert the physician to suspect
magnesium sulfate toxity?
A. Decreased deep tendon reflexes.
B. Cool skin temperature.
C. Rapid pulse rate
D. Tingling in the toes.
E. Decrease of ankle edema
ANSWER: A

What is the aim of magnesium sulfate therapy prescribed for treatment of severe
preeclampsia
A. Decreased generalized edema within 8 hours.
B. Decreased urinary output during the first 24 hours
C. Sedation and decreased reflex excitability within 48 hours.
D. Absence of any seizure activity during the first 48 hours.
E. Decreased breath movement less than 12 per minute
ANSWER: D

Which group of drug does magnesium sulfate used for treatment of severe preeclampsia
belongs to?
A. Peripheral vasodilator
B. Antihypertensive.
C. Central nervous system depressant.
D. Sedative-hypnotic.
E. Diuretic
ANSWER: C

Which of the following assessment findings should be report immediately during a


continuous intravenous infusion of 4 g of magnesium sulfate in Ringer's solution?
A. Respiratory rate of 12 breaths /minute
B. Patellar reflex of+2.
C. Blood pressure of 160/88 mm Hg.
D. Urinary output exceeding intake.
E. Increase of deep tendon reflexes
ANSWER: A

What is the first action at the beginning of a seizure in client diagnosed with severe
preeclampsia?
A. Insert an airway to improve oxygenation.
B. Note the time when the seizure begins and ends.
C. Call for immediate assistance.
D. Turn the client to her left side.
E. Catheterize central vein
ANSWER: C

Which of the following assessments should the physician prescribe during intravenous
magnesium sulfate infusion for treatment of severe preeclampsia?
A. Urinary output even'8 hours.
B. Deep tendon reflexes even' 4 hours.
C. Respiratory rate every hour.
D. Blood pressure every 6 hours.
E. Uterine contraction every 4 hours
ANSWER: C
Carol LeBec, age 16, comes to the clinic for contraceptive counseling and selection.
Before counseling Ms. LeBec, what should the physician do first?
A. Obtain a complete health history.
B. Perform a thorough physical examination.
C. Evaluate personal beliefs about family planning.
D. Determine why the client seeks counseling.
E. Ask about menarche
ANSWER: C

Sintia, age 20, comes to the clinic for contraceptive counseling and selection. The
physician advises to teach Sintia about oral contraceptives because they are easy to
use. How should the physician proceed?
A. Review all contraceptive choices so that the client will understand the ease of
taking one pill every day.
B. Discuss barrier methods because the client may forget to take the pill every day.
C. Ask the client which method she would like to learn about
D. Check for contraindications and discuss methods that best fit the client's needs
and life-style.
E. Explain advantages of barrier method
ANSWER: D

Laura Hunt, age 37, comes to the family planning clinic for contraceptive advice. Her
history reveals frequent urinary tract infections. Which of the following
contraceptive methods would be most appropriate for her?
A. condom
B. cervical cap
C. contraceptive sponge
D. diaphragm
E. spermicide
ANSWER: A

Neatha, age 32, comes to the family planning clinic for contraceptive advice. Her
history reveals frequent urinary tract infections. Which of the following assessment
findings would contraindicate an oral contraceptive for Ms. Hunt?
A. history of a benign liver tumor
B. 10 pounds under her ideal weight
C. history of dysmenorrhea since age 15
D. family history of ovarian cancer
E. monogamous relationship
ANSWER: A

Terry, age 27, comes to the family planning clinic for contraceptive advice. Her
history reveals frequent urinary tract infections. Terry inquires about the
symptothermal contraceptive method. In this method, the client notes which of the
following signs to help detect the fertile period?
A. breast enlargement and tenderness
B. clear, thin, elastic cervical mucus
C. decreased libido
D. weight gain of 2 to 3 pounds
E. irregular period
ANSWER: B

Betty, age 24, has been using an oral contraceptive for the past 5 years. Now she
wants to become pregnant. What advice should the physician give Betty?
A. Attempt to conceive as soon as possible after discontinuing the oral contraceptive.
B. Discontinue the oral contraceptive and wait 1 month before attempting to conceive.
C. Discontinue the oral contraceptive and wait 3 months before attempting to conceive.
D. Attempt to conceive after a physical assessment confirms a return to normal hormone
levels.
E. You should visit to reproductologist for consultation.
ANSWER: C

Two days ago Mary had intercourse during which her partner's condom broke. She has
come to the clinic today for a morning-after contraceptive. Which of the following
statements accurately describes this form of contraception?
A. The morning-after contraceptive must be used within 24 hours of unprotected
intercourse.
B. The morning-after contraceptive may cause mild nausea for 1 to 2 days.
C. The morning-after contraceptive creates a local, sterile inflammatory reaction in
the uterus.
D. The morning-after contraceptive has a relatively low effectiveness rate.
E. The morning-after contraceptive is not used after broking of condom.
ANSWER: B

Maria, age 26, married, calls the clinic because she has forgotten to take her oral
contraceptive for the past 2 days. What advice should the physician give to Maria?
A. Discard the rest of the pack and use another contraceptive method for the rest of
the cycle.
B. Take two pills for the next 2 days and use another contraceptive method for the
rest of the cycle.
C. Continue to take the remaining pills for the rest of the cycle.
D. Take two pills immediately and then take one pill daily for the rest of the cycle.
E. IUD should be placed during next 5 days
ANSWER: B

The physician is teaching Sarah unmarried, age 24, about diaphragm use and care. Which
of the following statements should be part of this teaching session?
A. Use only petroleum jelly for vaginal lubrication, if needed.
B. Check the diaphragm for tears or holes after each use.
C. Have the diaphragm refitted after a loss or gain of 10 pounds or more.
D. Store the diaphragm in its container in a cool, dry place.
E. Continue to use diaphragm during the period
ANSWER: D

Maria, age 18, has become pregnant as a result of a rape 10 weeks ago. She asks the
physician about pregnancy interruption. The physician describes various procedures,
including vacuum curettage and dilatation and curettage (D & C). How do they compare?
A. D & C requires uterus scraping with a metal curette; vacuum curettage does not
B. D & C is performed more frequently than vacuum curettage.
C. D & C uses crushing instruments; vacuum curettage uses suction equipment
D. D & C carries a higher risk of complications than vacuum curettage.
E. D & C carries a lower risk of complications than vacuum curettage
ANSWER: D

Donna, age 23, has become pregnant as a result of a rape 7 weeks ago. She asks the
physician about pregnancy interruption. The physician describes various procedures,
including vacuum curettage and dilatation and curettage (D & C). If Donna postpones
the decision until she is 10 weeks pregnant, which method of pregnancy interruption is
most likely to be used?
A. dilatation and curettage
B. dilatation and evacuation
C. prostaglandin suppository
D. RU-4S6 administration
E. cesarean section
ANSWER: C

Liz, age 28, has become pregnant as a result of a rape 10 weeks ago. She asks the
physician about pregnancy interruption. The physician describes various procedures,
including vacuum curettage and dilatation and curettage (D & C). After Liz undergoes a
first-trimester pregnancy interruption, the physician provides postoperative care.
Which of the following statements should be part of the postoperative teaching session?
A. Wipe the perineum from front to back.
B. Douche once a week until the follow-up visit
C. Avoid sexual intercourse for 3 to 4 weeks.
D. Expect the next menstrual period in about 3 weeks.
E. Intrauterine Device (IUD) is a method of choice for you
ANSWER: A

While pregnant with her fourth child, Anita discusses sterilization with her husband
John. They ask the physician to tell them about tubal ligation and its effects. How
does this procedure terminate fertility?
A. It prevents ovulation and menstruation.
B. It removes the entire uterus.
C. It occludes the cervix.
D. It blocks ova from the fallopian tubes.
E. Suppress ovulation
ANSWER: D

While pregnant with her fourth child, Olivia discusses sterilization with her husband
Mike. They ask the physician to tell them about tubal ligation and its effects. Olivia
decides to undergo sterilization after delivery while she is still in the hospital.
Which procedure is most appropriate for her?
A. minilaparotomy
B. laparoscopy
C. hysteroscopy
D. hysterectomy
E. colposcopy
ANSWER: A

Rick is scheduled for a vasectomy on Friday. His partner Anne asks the physician when
they can begin having unprotected intercourse. What is the physician's best response?
A. when desired because sterilization is immediate
B. as soon as scrotal edema and tenderness disappear
C. when the sperm count reflects sterilization
D. after about 6 to 10 ejaculations
E. after one month
ANSWER: C

Rita is scheduled for a tubal ligation on Wednesday. Her partner John asks the
physician when they can begin having unprotected intercourse. What is the physician's
best response?
A. when desired because sterilization is immediate
B. as soon as wound edema and tenderness disappear
C. after next menstrual period
D. 6 month later
E. you should use barrier method at least 2 weeks
ANSWER: A

A 26-year-old woman is scheduled for a first trimester abortion in the morning. A


laminaria tent is inserted as part of the vacuum aspiration procedure. The physician
explains to the woman that laminaria is used to:
A. Stimulate the uterus to contract.
B. Prevent postabortion infection.
C. Reduce pain by numbing the cervix.
D. Dilate the cervix for easier insertion of the aspirator.
E. Prevent postabortion hemorrhage
ANSWER: D

A married 23 years old woman visit to family planning center for consultation. She
uses the basal body temperature method of contraception. She describes her sexual life
as mogamous with husband. She wants to use Basal Body Temperature as family planning
method. She asks physician what does the Basal Body Temperature (BBT) chart tell about.
A. Problems with ovulation
B. Time of ovulation
C. Time of next ovulation
D. None of these
E. Time of previous ovulation
ANSWER: B

A married 23 years old woman visit to family planning center for contraceptive advice.
She describes her sex life as “active” with “many” partners. She wants a
contraceptive method that is reliable and does not interfere with sex. Which method of
contraception will be most appropriate for this client:
A. Vaginal rings
B. Condoms
C. Patches
D. Progestin only pill
E. Intrauterine device
ANSWER: B

Mona, is a 23-year-old healthy, non-smoking woman without medical complications. She


has no contraindications for the use of oral contraceptives. When during her cycle
should BB begin her oral contraceptives?
A. Take the first tablet on the first day of menses; this avoids the risk of early
ovulation and the need to use alternative methods of contraception
B. Take the first tablet on the first Sunday after the beginning of menstruation and
use alternative method of contraception for the first 14 days.
C. Take the first tablet on the first day of menses; and use alternative method of
contraception for the first 14 days
D. Take the first tablet on the first Sunday after the beginning of menstruation this
avoids the risk of early ovulation and the need to use alternative methods of
contraception
E. Take the first tablet days after visit; and use alternative method of contraception
for the next cycle
ANSWER: A

Nadia is a 30-year-old mother of three children. She has been exclusively


breastfeeding her youngest child, who is 5 months old. Nadia is planning to begin
supplementing her infant’s diet and weaning her baby next month. Today, she has come
to clinic to make choice about family planning method. Which method of contraception
will be most available for Nadia?
A. Lactational amenorrhea
B. Progestin only pill
C. Sterilization
D. Calendar method
E. Basal body temperature method
ANSWER: B

Nelly is a 26-year-old mother of two children. She has been exclusively breastfeeding
her youngest child, who is 5,5 months old. Nelly is planning to begin supplementing
her infant’s diet and weaning her baby next month. Today, she has come to clinic to
make choice about family planning method. She wants to use combine oral contraceptives.
What is the mechanism of action of this method?
A. make the cervical mucus thin
B. prevent the release of eggs from the ovaries
C. stimulate peristaltic waves of fallopian tube
D. destroy ovum
E. decrease speed of sperm movement
ANSWER: B

Ashley, a 23-year-old unmarried, comes for an initial visit to request birth control.
She asks about birth control pills, but also wants information about the newer
hormonal methods on the market. Vital signs: 1,68 m tall, 95 kg., BMI 33.9, Ps 76, RR
16, T 36,6.7 C, BP 142/88 mm Hg. What information do you NOT need prior to starting
Ashley on hormonal contraception?
A. Blood pressure
B. Smoking history
C. Pap smear and pelvic exam
D. History of migraines with auras
E. Family history of thrombotic disorders
ANSWER: B

Olga, a 33-year-old unmarried, comes for an initial visit to request birth control.
She asks about birth control pills, but also wants information about the newer
hormonal methods on the market. Vital signs: 1,68 m tall, 95 kg., BMI 33.9, Ps 76, RR
16, T 36,6.7 C, BP 142/88 mm Hg. Olga should watch for all of the following side
effects with an OCP EXCEPT
A. headache
B. nausea
C. spotting
D. breast tenderness
E. amenorrhea
ANSWER: E

Irma, a 26-year-old unmarried, comes for an initial visit to request birth control.
She asks about birth control pills, but also wants information about the newer
hormonal methods on the market. After listening to your instructions on how to use an
oral contraceptive, Irma worries that she will not be reliable in taking the pill
every day, and asks about other options. Physician recommends her depot
medroxyprogesterone acetate (DMPA, Depo-Provera ® ). Her vital signs: 1,68 m tall, 95
kg., BMI 33.9, Ps 76, RR 16, T 36,6.7 C, BP 142/88 mm Hg. Which of the following side
effects is Not true about depot medroxyprogesterone acetate (DMPA, Depo-Provera ® )?
A. Irregular bleeding is uncommon during the first 6-9 months of use
B. DMPA never causes amenorrhea
C. Fertility may be delayed after discontinuation
D. Weight gain is not a reported side effect
E. DMPA affect bone mineral density
ANSWER: E

Kanisha, a 35-year-old female comes to your clinic to ask about contraception. She and
her husband have one child. She wants some type of birth control that she doesn't have
to think about for as long as possible. She is otherwise healthy, and her only
complaint is heavy periods. She is currently using the “rhythm method” and adding
condoms during high-risk times. Which of the following contraceptives would you
recommend?
A. Tubal ligation
B. Basal body temperature method
C. Spermicides
D. IUD (Intrauterine Device)
E. Progestin only pill
ANSWER: E

Screening at 24 weeks’ reveals that a pregnant woman has gestational diabetes mellitus
(GDM). In planning her care, the doctor and the woman mutually agree that an expected
outcome is to prevent injury to the fetus as a result of GDM. The doctor identifies
that the fetus is at greatest risk for:
A. Macrosomia
B. Congenital anomalies of the central nervous system
C. Preterm birth
D. Low birth weight
E. Prolong labor
ANSWER: A

In her 36th week of gestation, a client with type 1 diabetes has a 4300 g infant by
cesarean birth. When caring for an infant of a diabetic mother (IDM), the doctor
should monitor for signs of:
A. Meconium ileus
B. Physiologic jaundice
C. Increased intracranial pressure
D. Respiratory distress syndrome
E. Excessive weight loss
ANSWER: D

A client at 6 weeks' gestation who has type 1 diabetes is attending the prenatal
clinic for the first time. Why do the insulin requirements may decrease during the
first trimester?
A. Body metabolism is sluggish in the first trimester
B. Morning sickness may lead to decreased food intake
C. Fetal requirements of glucose in this period are minimal
D. Hormones of pregnancy decrease the body's need for insulin
E. Placental hormones work as insulin
ANSWER: B

A primigravida with pregestational type 1 diabetes is at her first prenatal visit.


When discussing changes in insulin needs during pregnancy and after birth, the doctor
explains that based on her blood glucose levels she should expect to increase her
insulin dosage between the:
A. 10th and 12th weeks of gestation
B. 18th and 22nd weeks of gestation
C. 24th and 28th weeks of gestation
D. 36th week of gestation and the time of birth
E. In postpartum period
ANSWER: C

A primigravid client who was successfully treated for preterm labor at 30 weeks'
gestation had a history of mild hyperthyroidism before becoming pregnant. Which
recommendation the doctor should give the client about treatment when planning her
discharge?
A. Continue taking low-dose oral propylthiouracil (PTU) as ordered.
B. Discontinue taking the methimazole (Tapazole) until after delivery.
C. Consider breast-feeding the neonate after the delivery.
D. Contact the physician if bradycardia occurs.
E. Taking medication can cause congenital fetal abnormalities
ANSWER: A

A 30-year-old multigravid client at 8 weeks' gestation has a history of insulin-


dependent diabetes since age 18. What is the insulin needs during the first trimester?
A. They will increase.
B. They will decrease.
C. They will remain constant.
D. They will be unpredictable.
E. They depend from baby’s glucose level
ANSWER: B

Primigravid client at 10 weeks' gestation who has a 5-year history of insulin-


dependent diabetes admit to the hospital for her first prenatal visit. What is NOT the
complication of pregnancy in this woman?
A. Candida albicans infection.
B. Twin-to-twin transfer.
C. Polyhydramnios.
D. Preeclampsia.
E. Diabetic fetopathy
ANSWER: B

At 38 weeks' gestation, a primigravid client with poorly controlled diabetes and


severe preeclampsia is admitted for a cesarean delivery. Which of the following
situation can be prevented with Cesarean Section?
A. Neonatal hyperbilirubinemia
B. Congenital anomalies.
C. Perinatal asphyxia.
D. Stillbirth.
E. Preterm Rupture of Membranes
ANSWER: D

A primigravid client with diabetes at 39 weeks' gestation is seen in maternity clinic.


The physician estimates that the fetus weighs at least 4,500 g. The client asks, "What
causes the baby to be so large?" The physician explains that fetal macrosomia is
usually related to?
A. Family history of large infants.
B. Fetal anomalies
C. Maternal hyperglycemia.
D. Maternal hypertension.
E. Maternal obesity
ANSWER: C

With plans to breast-feed her neonate, a pregnant client with insulin-dependent


diabetes asks the doctor about insulin needs during the postpartum period. The
physician explains that postpartal insulin needs for breastfeeding mothers
A. fall significantly in the immediate postpartum period.
B. remain the same as during the labor process.
C. usually increase in the immediate postpartum period.
D. need constant adjustment during the first 24 hours.
E. usually unpredictable
ANSWER: A

A woman seeking prenatal care relates a history of macrosomic infants, two stillbirths,
and polyhydramnios with each pregnancy. The physician recognizes that these factors
are highly suggestive of:
A. Toxoplasmosis
B. Abruptio placentae
C. Hydatidiform mole
D. Diabetes mellitus
E. Preeclampsia
ANSWER: D

A pregestational diabetic woman at 20 weeks' gestation exhibits the following: thirst,


nausea and vomiting, abdominal pain, drowsiness, and increased urination. Her skin is
flushed and dry and her breathing is rapid with a fruity odor. A priority action when
caring for this woman would be to:
A. Provide the woman with a simple carbohydrate immediately
B. Request an order for an antiemetic
C. Assist the woman into a lateral position to rest
D. Administer insulin according to the woman's blood glucose level
E. Invite endocrinologist for consultation
ANSWER: D

A woman was anemic during her pregnancy. She had been taking iron for 3 months before
the birth. She gave birth by cesarean 2 days ago and has been having problems with
constipation. After assisting her back to bed from the bathroom, the doctor notes that
the woman's stools are dark (greenish black). The doctor would:
A. Perform a guaiac test and record the results
B. Recognize the finding as abnormal and report it to head of department
C. Recognize the finding as normal as a result of iron therapy
D. Check the woman's next stool to validate the observation
E. Prescribe colonoscopy
ANSWER: C

In caring for a pregnant woman with sickle cell anemia with increased blood viscosity,
the doctor is concerned about the development of a thromboembolism. The care would
include:
A. Monitoring the client for a negative Homan sign
B. Massaging calves when the woman complains of pain
C. Applying anti-embolic stockings
D. Maintaining a restriction on fluid intake
E. Prescribe anticoagulant
ANSWER: C

When assessing a pregnant woman at 28 week’s gestation that is diagnosed with


rheumatic heart disease, it is important that the physician be alert for sign
indicating cardiac decompensation. A sign of cardiac decompensation would be:
A. Dry skin, hacking cough
B. Supine hypotension
C. Wheezing with inspiration and expiration
D. Rapid pulse that is irregular and weak
E. Bradycardia, and hypertension
ANSWER: D

A woman at 30 week’s gestation with class II cardiac disorders calls her primary
health care provider’s office and speaks to the physician. She tell the physician that
she has been experiencing a frequent, moist cough for the past few days. In addition,
she has been feeling more tired and is having difficulty completing her routine
activities of some difficulty with breathing. The physician’s best response would be:
A. Have some bring you to the office so we can assess your cardiac status
B. Try to get more rest during the day because this is a difficult time for your heart
C. Take an extra diuretic tonight before you go to bed because you may developing some
fluid in your lung
D. Ask your family to come over and do your housework fort he next few days so you can
rest
E. Monitor baby’s kick count
ANSWER: A

A pregnant woman with cardiac disorder will begin anticoagulant therapy to prevent
clot formation. In preparing this woman for this treatment measure the physician would
expect to teach the woman about self-administration of which of the following
medication
A. Furosemid
B. Propranolol
C. Heparin
D. Warfarin
E. Nidfedipine
ANSWER: C

At previous antepartal visit, the physician taught a pregnant woman diagnosed with
class II cardiac disorder about measures to use to lower her risk for cardiac
decompensation. The woman would demonstrate need for further instruction if she
A. Increased roughage in her diet
B. Remains on bed rest only getting out of bed go to the bathroom
C. Sleep 10 hours every night and rests after meals
D. States she will call the physician immediately if she experiences any pain or
swelling in her legs
E. Knows signs of cardiac decompensation
ANSWER: B

A client with a past medical history of ventricular septal defect repaired in infancy
is seen at the prenatal clinic. She is complaining of dyspnea with exertion and being
very tired. Her vital signs are t 36.7°C, Ps 80/min, Breath movement 20/min, BP 116/72
mm Hg. She has + 2 pedal edema and clear breath sounds. As the physician plans this
client's care, which class of organic heart disease she has according to the New York
Heart Association functional classification?
A. Class I
B. Class II
C. Class III
D. Class IV
E. Class V
ANSWER: B

While caring for a primigravid client with class II heart disease at 28 weeks'
gestation, the physician would instruct the client to contact her physician
immediately if the client experiences which of the following?
A. Mild ankle edema.
B. Emotional stress on the job
C. Weight gain of 500 g in 1 week.
D. Increased dyspnea at rest.
E. General weakness
ANSWER: D

A 90-year-old G7P7 woman presents with severe vaginal prolapse. The entire apex,
anterior and posterior wall are prolapsed beyond the introitus. She cannot urinate
without reduction of the prolapse. Hydronephrosis was noted on ultrasound of the
kidneys and it is thought to be related to the prolapse. She has a long-standing
history of diabetes and cardiac disease. She is not a candidate for general or
regional anesthesia. She has failed a trial of pessaries. Which of the following is
the next best step in the management of this patient?
A. Do nothing and observe.
B. Anterior and posterior repair
C. Colpocleisis
D. Sacrospinous fixation
E. Sacrocolpopexy
ANSWER: C

A 48-year-old G0 woman comes to the office for a health maintenance exam. She is
healthy and not taking any medications. She has no history of abnormal Pap smears or
sexually transmitted diseases. She is not currently sexually active. Her menstrual
cycles are normal and her last cycle was three weeks ago. She smokes one pack of
cigarettes per day. Her mother was diagnosed with endometriosis and had a hysterectomy
and removal of the ovaries at age 38. She is 5 feet 4 inches tall and weighs 130
pounds. On pelvic examination, the patient had a palpable left adnexal mass. An
ultrasound was obtained, which showed a 4 cm complex left ovarian cyst and a 2 cm
simple cyst on the right ovary. What is the most appropriate next step in the
management of this patient?
A. CAT scan of the abdomen and pelvis
B. MRI of the pelvis
C. Abdominal hysterectomy and bilateral salpingoophorectomy (TAH/BSO)
D. Repeat ultrasound in 2 months
E. Oral contraceptives
ANSWER: D

A 45 year-old G5P5 premenopausal woman was initially seen in your office for work-up
and evaluation of a FIGO grade 2 endometrial cancer that was diagnosed by her
gynecologist. Which of the following is the most appropriate treatment for this
patient?
A. Total laparoscopic hysterectomy with bilateral salpingoophorectomy
B. Vaginal hysterectomy with bilateral salpingoophorectomy
C. Total abdominal hysterectomy, bilateral salpingoophorectomy, bilateral pelvic and
paraaorticlymphadenectomy, pelvic washings
D. Supracervical abdominal hysterectomy with ovarianpreservation
E. Megace (megestrol acetate)
ANSWER: C

A 31 year-old G0 woman has been diagnosed with uterine fibroids. A fluid contrast
ultrasound confirmed the presence of two intramural fibroids measuring 5 x 6 cm and 2
x 3 cm that appear to be distorting the patient’s uterine cavity. The patient has a
two-year history of infertility. She has had a thorough infertility work up. No
etiology for her infertility has been identified. Which of the following treatments is
most appropriate for this patient?
A. Hysteroscopy
B. Uterine curettage
C. Gonadotropin-releasing hormone agonist
D. Uterine artery embolization
E. Myomectomy
ANSWER: E

A 72 year-old G3P2 postmenopausal woman is referred to your office by her internist


after a work-up for abdominal bloating revealed a large pelvic mass on transvaginal
ultrasound and an elevated CA-125. She has no significant medical history and only a
prior appendectomy. The CT scan showed a large heterogenous pelvic mass measuring 20 x
13 x 10 cm. There was a moderate amount of ascites and likely “omental caking.” There
was no significant pelvic or abdominal lymphadenopathy, and the chest x-ray showed
only a small right pleural effusion. On pelvic examination, there is minimal cul-de-
sac nodularity and the mass is readily palpable and somewhat mobile. There is an
obvious fluid wave. Which of the following is the most appropriate next step in the
management of this patient?
A. Thoracentesis
B. Exploratory laparotomy and tumor debulking (cytoreduction)
C. Paracentesis
D. Diagnostic laparoscopy and tissue biopsies
E. Neoadjuvant chemotherapy
ANSWER: A

Which of the following is the risk factor for vesicovaginal fistula?


A. Child birth trauma
B. Prolonged labour
C. Pelvic irradiation
D. Pelvic surgery
E. All of them
ANSWER: E
A previously healthy 27-year-old nulligravid woman comes to the emergency department
because of a 2-day history of moderate-to-severe pain and swelling of the left labia.
She is sexually active and uses condoms inconsistently. Her temperature is 37.2°C
(99°F), pulse is 92/min, respirations are 18/min, and blood pressure is 115/75 mm Hg.
Pelvic examination shows a 4 x 3-cm, tender, fluctuant mass medial to the left labium
majus compromising the introital opening. Which of the following is the most
appropriate next step in management?
A. Vulvectomy
B. Ultrasound-guided needle aspiration of the mass
C. Administration of intravenous penicillin G
D. Administration of intravenous metronidazole
E. Incision and drainage
ANSWER: A

26 years old patient has formed formation of purulent inflammation of the appendages
of the uterus. What would you recommend?
A. puncture through the posterior vaginal vault, draining pus cavity and the
introduction of antibiotics into it
B. surgery
C. pirogenal therapy
D. electrophoresis of zinc
E. Nothing above
ANSWER: A

Woman I., aged 38 years was admitted to gynecologic department with complaints on
cramp-like pains in the lower abdomen and moderate blood-tinged discharge from vagina.
In past-history: labor 1, medical abortions – 2. Patient notes delay of menstruation,
instantaneous loss of consciousness. Objectively: skin integuments are pale, covered
with cold perspiration. Arterial pressure – 90/50 mm Hg., pulse rate – 120 beats/min.
On vaginal examination: somewhat enlarged uterine, in the right – movable formation of
ovoid form, soft consistency is palpable. What treatment should be carried out?
A. Surgical intervention.
B. Treatment with estrogens.
C. Treatment with androgens.
D. Symptomatic treatment.
E. Treatment with gestagens.
ANSWER: A

Patient, aged 32 years was admitted to gynecologic department with complaints on sharp
pain in the lower abdomen. Menses were 2 weeks ago, timely. On bimanual vaginal
examination: vagina and uterine cervix are without peculiarities. Examination of the
uterine body and appendages is impossible to perform due to tenderness and tension of
the anterior abdominal wall. Posterior fornix overhangs, painful. Apoplexy of the
ovary is suspected. What should be done to precise diagnosis?
A. To perform bimanual examination again under narcosis.
B. Paracentesis of the abdominal cavity through posterior fornix of vagina.
C. Ultrasonic investigation.
D. Symptomatic treatment
E. Hysteroscopic examination.
ANSWER: B

Patient, aged 23 years was admitted to the gynecologic unit complaining of pain in the
lower abdomen, general weakness, collaptoid state, blood-tinged discharge from the
reproductive passages. In the past history: patient had chronic adnexitis; last 2
months – no menstruation, sometimes bloody discharge was noted. Objectively: skin and
mucous membranes are pale. Arterial pressure – 90/60, pulse rate – 100 beats/min.
Abdomen is swelled a little, painful in the lower portions. Shchyotkin’s symptom is
positive. Vaginal examination: uterine is enlarged up to 5-6 weeks, soft, painful one
on palpation. Appendages are not palpable clearly due to a sharp tension of the
anterior abdominal wall. Posterior fornix hangs over, painful one. Dark blood-tinged
discharge continues. What method of investigation is the most informative?
A. Hysteroscopy.
B. Laparoscopy.
C. Ultrasonic investigation.
D. Abdominal paracentesis.
E. Dopplerometry.
ANSWER: D

A 30-year-old woman complains of irregular copious painful menstruations, pain


irradiates to the rectum. Anamnesis states 10- year-long infertility. On bimanual
examination: uterus is of normal size; uterine appendages on the both sides are corded,
with rectricted mobility, painful; there are dense nodular painful growths detected in
the posterior fornix. A doctor suspects endometriosis. What method allows to verify
this diagnosis?
A. Laparoscopy
B. Diagnostic curettage of uterine cavity
C. Paracentesis of posterior fornix
D. Uterine probing
E. Hysteroscopy
ANSWER: A

Which treatment is followed by a 48 years patient with severe cervical dysplasia and
ovarian cyst:
A. diatermokonization of cervix
B. diathermocoagulation of cervix
C. hysterectomy with appendages
D. treatment by solkovagin
E. hysterectomy without appendages
ANSWER: C

Patient 22 years. Complains about pain in a right labia pudenda majora, rise of body
temperature to 38.0°C. At the review of genital organs the considerable increasing of
right large sexual lip definites, especially in the lower third. Erythema, edema, at
palpation acutely painful, fluctuation is determined. To conduct vaginal examination
due to acute pain is impossible. Blood test: Leucocytes — 10,0 x 109 per cu mm. What
method is main?
A. The dissection and drainage of abscess.
B. To withdraw a bartolin gland within the limits of healthy tissue.
C. To appoint physical therapy procedures.
D. To appoint compresses with liniment.
E. To expect a spontaneous regeneration of abscess.
ANSWER: A

26 years old patient has formed formation of purulent inflammation of the appendages
of the uterus. What would you recommend?
A. puncture through the posterior vaginal vault, draining pus cavity and the
introduction of
B. antibiotics into it
C. surgery
D. pirogenal therapy
E. electrophoresis of zinc
ANSWER: A

In the 30years old primapara intensive uterine contractions with an interval of 1-2
min, duration 50 sec have begun. In time of the fetal head delivery the patient
complaints on severe pain in the perineum. The perineum is 5 cm, its skin become pale.
What is it necessary to perform:
A. Perineotomy
B. Episiotomy.
C. Protection of the perineum.
D. Vacuum - extraction of the fetus.
E. Waiting tactics.
ANSWER: A

The primapara 24 is admitted in to the hospital due to high body temperature – 38,7 0,
1 stage of labor, regular uterine contractions. Sizes of pelvis: 25-28-30-20 cm.
Abdominal circumference is 100 cm, level of uterine fundus 28 cm, presenting part is
absent. Right side the fetal head is palpated, left – the breech, fetal heart sounds
are absent. Vaginal examination: the uterine cervix is fully dilated, amniotic
membrane is whole. What is the tactic of labor conducting?
A. Cesarean section after full dilatation
B. Cesarean section immediately
C. External version of the fetus on a head
D. Stimulation of uterine contractions
E. Classic obstetric version of the fetus
ANSWER: E

In patient 25 year, labor III. The pelvic sizes: 24-27-30-19 cm. After stormy uterine
contractions and pushing at a highly standing fetal head and positive Vasten’ sign
uterine contractions were stopped suddenly, bloody excretions from a vagina appeared,
fetal heart rate is not listened. The condition of patient suddenly became worse,
blood pressure went down to 70 mm Hg, pulse 140 in a 1 minute, the skin is pale.
Reason of the shock condition?
A. Uterine rupture
B. Threatened rupture of uterus
C. Abruption placentae
D. Syndrome of squeezing of lower hollow vein
E. Placenta previa
ANSWER: A

A 30 years old woman has the 2-nd labour that has been lasting for 14 hours. Hearbeat
of fetus is muffled, arrhythmic, 100/min. Vaginal examination: cervix of uterus is
completely opened, fetus head is at the pelvis outlet. Saggital suture is in the
straight diameter, small fontanel is near symphysis. What is the further tactics of
handling the delivery?
A. Applying obstetric forceps
B. Stimulation of labor activity by oxytocin
C. Cesarean section
D. Antihypoxic drugs
E. Use of mid forceps
ANSWER: A

A woman is 34- year old, it is her tenth labor at full term. It is known from the
anamnesis that the labor started 11 hours ago, labor was active, painful contractions
started after discharge of waters and became continuous. Suddenly the parturient got
knife-like pain in the lower abdomen and labor activity stopped. Examination revealed
positive symptoms of peritoneum irritation, ill-defined uterus outlines. Fetus was
easily palpable, movable. Fetal heartbeat is not auscultable. What is the most
probable diagnosis?
A. Rupture of uterus
B. Uterine inertia
C. Discoordinated labor activity
D. Risk of uterus rupture
E. II stage of labor
ANSWER: A

You might also like